Sunteți pe pagina 1din 1395

Community-Oriented Nursing

Focus: Health of the community as a whole.


Client: Community
Activity: Surveillance and evaluation of the community's
collective health, and the implementation of strategies to
promote health and prevent disease.

Community-Based Nursing
Focus: Health of individuals, families, and groups within a
community.
Client: Individuals, family, or group of individuals
Activity: Provision of direct primary care in the settings
where individuals and families live, work, and "attend"
(schools, camps, parishes)

Community Health Nursing


Systematic processes to deliver care to individuals,
families, or community groups with a focus on promoting,
preserving, protecting, and maintaining health care
provided contributes to the health of the community.

Public Health Nursing


Nursing and public health, disease and disability
prevention, promoting and protecting the health of the
entire community

Eight domains of public health


Distinguishes public health nursing from other nursing
specialties by adherence to the eight principles:
Analytical assessment skills
Policy development and program planning skills
Communication skills
Cultural competency skills
Community dimensions of practice skills
Basic public health science skills
Financial planning and management skills
Leadership and systems thinking skills

Three primary functions of public health


1. Assess health care function
2. Develop policy that provides access to services
3. Ensure services are delivered and outcomes achieved

Health People 2020: Four main goals


1. Eliminate preventable diseases, disability, injury, and
premature death

2. Achieve health equity, eliminate disparities, and improve


health of all age groups
3. Create social and physical environments that promote
good health for all
4. Promote healthy development and health behaviors
across every stage of life

Social determinants
Impact whether someone is able to attain/maintain health;
(Income, social status, education, literacy, environment,
support networks, gender, culture, available health care)

Disparities
Gaps in care experienced by one population compared to
another

Health care changes in the 21st century


Increased patient acuity and number of services in the
community.
Patient centered care
Increased patient responsibility for own health
Increased use of technology

Florence Nightingale
Studied nursing in Germany then established nursing
schools in England

District Nursing
A mode of service delivery in which a community health
nurse is responsible for addressing all the health needs of a
given population

Lemuel Shattuck
First use of demographic data to look at population health
birth and death records

Dorthea Dix
Established first hospital for the mentally ill in the US

Clara Barton
Civil war nurse: Established the American Red Cross
(providing aid for natural disasters)

Lillian Wald

Found of public health nursing in the US; Founded the


Henry Street Settlement and Visiting Nurse Service which
provided nursing and social services and organized
educational and cultural activities. She is considered the
founder of public health nursing

Mary Breckenridge
Pioneer in nurse-midwifery; established the Frontier Nursing
Service - nurses traveled on horseback to reach mothers in
the hills of Kentucky

World Health Organization


International center that collects data, advances initiatives,
and offers support related to public health

US Department of Health and Human Services (USDHHS)


US branch of government responsible for health and
welfare of citizens

Nongovernment Agencies
Agencies that acquire resources from private sources to
assist others

Philanthropic Agencies
Organization that uses endowments or private funding to
address the needs of individuals, families, and populations

Center for Disease Control


(CDC) is a United States federal agency under the
Department of Health and Human Services. It works to
protect public health and safety by providing information to
enhance health decisions, and it promotes health through
partnerships with state health departments and other
organizations.

Difference in US health care system and public health


The US has an individual focus on curative measures while
public health focuses on the population or community as a
whole and focuses on disease reduction and improved
health

Local Level of Government


Implement public health activities within the community;
Governed by the state department;
Structure varies based on community needs

State Level of Government


Focuses on regulation of the community of program
responsibility and resource allocation

Federal Level of Government


Focuses on policy development and financing

Health
A quality, an ability to adapt to change or a resource to
help cope with challenges and processes of daily living

Well Being
A subjective perception of full functional ability as a human
being

USDHAHS 10 Components of public health practice


1. Preventing Epidemics
2. Protecting the environment, workplaces, and sources of
food and water
3. Promoting health behavior
4. Monitoring the health status of the population
5. Mobilizing community resources into service
6. Responding to disasters
7. Assuring the public there are trained personnel
8. Reaching out to those at high risk
9. Researching risk, disease acquisition, and ways to
prevent injury
10. Influencing policy to acquire resources to effect change

Epidemiological Health Promotion Model


The study of relationships among an agent, a host, and an
environment. The interaction determines the development,
and cessation of communicable disease.

Learning Theory
Goal established and reinforced by nurse; Rewards given
for partial accomplishment

Health Belief Model


Cues used to remind people of health behavior and
promote action; Perceived risk influences action

Transtheoretical Health Promotion Model

5-Step approach: Pre-contemplation, Contemplation,


Preparation, Action, Maintenance

Reasoned Action Health Promotion Model


Performance of a behavior is determined by a persons
intention to perform the behavior; Intentioned determined
by attitude and behavior

Social Support Health Promotion Model


Change influenced by support of friends, families, and
communities

Primary Level of Prevention


Utilize prevention strategies, address modifiable risk
factors, and maximize health and wellness

Secondary Level of Prevention


Planned effort to minimize the impact of disease and injury
once it has occurred

Tertiary Level of Prevention


Maximizing health and wellness through strategies set in
place at the end-stage of disease and injury

Epidemiology
The study of the distribution and determinants of health
and illness

Rate
Primary measurement used to describe either the
occurrance or the existence of a specific state of health or
illness

Outbreaks
Epidemic usually limited to a localized increase in the
incidence of the illness

John Graunt
Father of demographics. recognized importance of
recording birth and death rates and age structure of human
population

William Farr

Set up a system of data collection for causes of death in


difference occupations, gender, and imprisonment.
Importance of definition of illness and population
comparison, groups, and factors such as age, health, and
environment can affect statistics

John Snow
Used mapping and rates as an objective measure to
compare populations

Epidemiological Triad
Model based on the belief that health status is determined
by the interaction of the characteristics of the host, agent,
and environment

Wheel of Causation
Epidemiologic model that deemphasizes the agent as the
sole cause of disease while emphasizing the interplay of
physical, biological, and social environments

Web of Causation
Epidemioligcal model that strongly emphasizes the concept
of multiple causation while deemphasizing the role of
agents in explaining illness

Natural History of Disease


Course of disease or condition from the onset to resolution

Rate
Primary measure used to describe either the occurrence or
existence of a specific state of health or illness within a
group during a specific time frame

Ratio
Fraction that represents the relationship between two
numbers; Divide one quantity by another quantity
Ex: # boys in class/# girls in class

Proportion
Type of ration
Ex: # boys/ total students

Crude Rate

Measures the occurrence of the condition in the whole


population; May obscure info because it does not consider
factors such as age, race, gender
Numerator: Number of events
Denominator: Total population (not just those at risk)

Adjusted Rate
Controls for differences between populations-used for
comparison

Incidence
Measures probability that people without a condition will
develop the condition over time; measures pace of new
illness

Prevalence
Existence of a condition during a period/interval at a
specific point; Studies number of people diagnosed in the
past and length of illness; Longer length=greater the
prevalence-indicates burned of community

Mortality Rates
Crude mortality: probability of dying from any cause; #
deaths in a year/total population

Proportional Mortality Ratio


Compares death from a specific illness with deaths from all
causes; # deaths from specific cause within a time
frame/total deaths

Incidence Density
Used when unequal periods of observation for study
subjects; Accounts for people who die, drop out of a study,
or acquire an illness

Attributable Risk
The risk of a condition occurring in an exposed group that
is attributable to an exposure, not other factors

Relative Risk Ratio


Measure of the risk of developing a condition; Ratio of
incidence in exposed and incidence in non-exposed

Sensitivity
Ability of the test to correctly identify people who have the
health problem

Specificity
Specific incidence and prevalence rates calculated based
on specific characteristics (demographic data), variations
based on location and variations in time (short-outbreak,
periodic- seasonal, long-years)

Crude Rate Formula


# events within time period/population at risk with same
time pt

Incidence Rate
#new cases during time period/population at risk in the
same time period

Causality
Strengths of Association
Consistency
Temporality
Plausibility
Biological Gradient

Case Studies
In-depth analysis of individual or group, often first clue to a
problem

Cohort Studies
(Longitudinal Studies) Monitor over time to find
associations between risk factors and health;
Minimize selection bias;
Relative risk is the ratio of disease incidence in an exposed
population;
Indicates strength of exposure to illness causality

Case Control
Compare group with health problems (cases) to group
without (control)

Key Informants
Persons knowledgeable about specific aspects of a problem
and the communities current and past attempts to address
it

Stakeholders
An individual, organizational, or group that has interest
(stake) in a specific community health issue or the outcome
of a community level intervention

Gantt Chart
Visual of the sequence of steps to achieve objectives;
Developed in planning stages to identify steps, a tool for
scheduling tasks, monitor progress

Community of Interest
People and groups/aggregate that will be affected by
change; Those that will help bring about change

Upstream Approach to Health Care


Changes at societal level may impact health of community
without change is individual behavior

Sustainability
How to maintain change, support system in place

Social Justice
Health care is a risk; Address root cause of illness

Health Equities
Requires elimination of health disparities in living and
working conditions

SMART objectives
S- Specific
M- Measurable
A- Achievable
R- Relevant
T- Time

Coalition Building
Community members participate in planning and
implementing changes at the community level; Need clear
mission, goals, objectives, expectations, leadership,
accountability, and should be heterogenous

Community Readiness

Assess readiness to undertake change process-issue


specific

Web of Causation
Identify multiple factors that contribute to chronic disease

Lewins Model for Change


Unfreezing:
Changing:
Freezing:

Levers of Change
Increase driving/reinforcing forces
Decrease restraining/resisting forces

Force Field Analysis


Identify factors within a community that impact change and
forces that cannot be changes

Geopolitical Population
Has identified designated boundaries with the same
governing structure

Phenomenological Population
Group with common interests or beliefs who have
interpersonal and intrapersonal connection

Community of Solution
Formed by aggregate to address a health problem
In 1988 the Institute of Medicine (IOM) published a report
on the future of public health and its mission that defined
public health:
What society does collectively to ensure the conditons in
which people can be healthy.

The scope of practice of public health nurses is:


Population focused and community oriented, with a primary
emphasis on population-level interventions that target
strategies for health promotion and disease prevention.

Barriers to implementing population focused care include:


Lack of understanding of the public health nurse role and
it's relationship to other roles in nursing.

The role and goals of the community health nursing


practice can best be described as:
Population-level strategies aimed at promoting, preserving,
and maintaing the health of populations through the
delivery of personal health services to individuals, families,
and groups in an effort to improve the health of the
community as a whole.

Community health nursing practice is:


The synthesis of nursing theory and public health theory

Both community health and public health nursing are


considered to be:
Community-oriented practices involving free-living
(noninstitutionalized) clients.

Community based nursing practice is ____ specific.


Setting specific. Care is provided to clients where they live
(home health or hospice nursing, community based clinic)
work (occupational health nursing) and/or where they
attend school (school nursing).

The emphasis of community-based nursing practice


Acute and chronic care and the provision of
comprehensive, coordinated and continuous services,
usually within a specialty area.

Assessment is a core function of public health and refers


to:
Systematically collecting data concerning the population,
monitorin the population's health status and making
information available on the health of the community.

The public health core function of assurances focuses on


the:
Responsibility of public health agencies to be sure that
activities are appropriately carried out to meet public
health goals and plans. This also involves making sure that
essential health services are available and accessible,
especially to vulnerable populations.

A nurse planning a smoking cessation clinic for


adolescents in the local middle and high schools is
providing:

Community-oriented care.

Community-oriented nurses who provide direct care


services to individuals, families or groups are engaging in
a _________ practice.
Clinically oriented practice, even when they apply concepts
of population-focused, community-oriented strategies
along with their direct care clinical strategies.

Public health specialists of define problems at the _____


level as opposed to the ____ level.
Population level as opposed to the individual level. At the
population level, public health specialists are usually
concerned with more than one subpopulation and
frequently with the health of the entire community.

Mary Breckenridge established _____ in 1925:


The Frontier Nursing Service (FNS)

Mary Breckenridge established the FNS based on a


system of care care used in ______
The highlands and islands of Scotland.

One of Breckenridge's innovative contributions to health


care in the U.S was:
Introduction of the first nurse-midwifery training.

The unique pioneering spirit of the FNS influenced the


development of public health programs to improve the
health care of:
Rural and often inaccessible populations in the Appalachian
sections of southeastern Kentucky.

The most important contribution made by Florence


Nightingale to community-oriented nursing was:
Expansion of the role of nursing to include healthpromotion practices.

Florence Nightingale introduced professional nursing


eduction in _______
1850's England.

Florence Nightingale introduced the concept that


individual health depended on _____

community health, which expanded nurses' focus from care


of the ill to include a population-based approach, health
promotion and disease prevention

Differentiate between "sick nursing" and "health nurses",


as defined by Florence Nightingale
"Health Nursing" emphasized that nurses should strive to
promote health and prevent illness.

The Social Security Act of 1935 was designed to prevent


the ______
Reoccurence of the problems of the depression.

Social Security Act of 1935, Title VI


Provided funding for expanded opportunities for health
protection and promotion through education and
employment of public health nurses. Also provided $2
million in funding for research and investigation of disease.

The most relevant strategy related to Title VI of the Social


Security Act of 1935.
Provision of funding to support employment and education.

In 1902 Lillian Wald introduced the concept of school


nursing to address ____
The problem of school absenteeism

The primary model for the school nurse program by Lillian


Wald was to work with children in the schools and make
home visits for the purpose of:
Providing and obtaining medical treatment for absent
students.

In New York City in 1897, school medical examination


focused on:
Excluding infectious children from school.

In 1902, more than ___ of children might be absent from


school on any given day
20%

The shift in the U.S Congress to categorical funding


provides federal money for _____ rather than a _____
health program.

Priority diseases or groups rather than a comprehensive


community health program.

Primary health care differs from primary care in which


way?
Primary health care encourages community participation

T/F
Primary health care is the chief means of delivery of
health care in the United States.
False

The public health system at the state level is responsible


for:
Standing ready to prevent or respond to disasters, both
human caused and natural, overseeing health care
financing and the administration of programs such as
Medicaid and CHIP, providing mental health and
professional education; establishing health codes,
licenssing facilities and personnel; regulating the insurance
industry and providing direct assistance to local health
departments.

The WHO's Declaration of Alma-Ata (1978) identified


______________ as its primary global goal.
"Health for All in the Year 2000", later amended in 1998 to
"Health for All in the 21st Century."

The agency that assumes the responsibility for improving


health by expanding access to primary care for lowincome, uninsured or rural individuals is:
Health Resources and Services Administration

The Health Resources and Services Administration of the


Department of Health and Human Services at the federal
level of the public health system is responsible for:
Directing grant programs that improve the nation's health
by expanding access to primary care for low-income and
uninsured people, focusing on mothers and their children,
people with HIV/AIDS and residents of rural areas.

The major factor that drives the current discussions about


a Medicare shortfall in the middle of the twenty-first
century is:
Percentage of elderly in the U.S population

A nurse questions whether a particular activity in her job


description is within a nurse's scope of practice. The
nurse would look to which government jurisdiction/agency
within the piblic health system to seek clarification?
State board of nurse examiners.

A factor that strongly influences the success of a primary


health care system is:
Participation of the community members in the design,
implementation and evaluation of the initiative.

The focus of public health system in the US


Primary health care (PHC)

PHC is defined as the broad range of services including


but not limited to:
basic health services, family planning, clean water supply,
sanitation, immunization and nutrition education. The
emphasis is on prevention.

The Affordable Health Care for America Act of 2010 does


what?
-Requires most Americans to have health insurance
coverage
- Expands Medicaid
- Subsidizes private coverage for low and middle-income
people
- Transforms the health care system from a sick care
system to health care system
- Lowers mortality related to preventable causes
-Institutes health insurance reform,
- Increases access to affordable health care coverage
- Increases support for nursing workforce development
programs.

Evidenced-based nursing practice (EBP) can best be


defined as:
Approach to the integration of the best research available,
nursing expertise and the preferences/values of the clients
served. (2005 Sigma Theta Tau International)

Evidence-based public health is:


A public health endeavor in which there is informed,
explicit and judicious use of evidence that has been derived

using an of a variety of scientific and social science


research and evaluation methods.

External evidence:
Includes research and other evidence

Internal evidence:
Includes the nurse's clinical experiences and client's
preferences.

Community-oriented nurses use EBP most effectively


when they:
Base care on nationally accepted clinical guidelines, involve
clients in individual care decisions and include community
input

A community-oriented nurse seeks to implement EBP in


the community clinic's programs. The best model for the
nurse to apply is:
Community development

When nurses work with communities, EBP must also be


Culturally and financially appropriate.

The use of EBP that are both culturally sensitve is a


necessity to ensure:
Acceptance by the community and appropriate allocation of
limited resources.

A systematic review of the literature was conducted on


the 3 levels of practice and 17 interventions in the ______.
Intervention wheel.

When a public health nurse uses EBP to evaluate the


effectiveness, accessibility and quality of populationbased services within the community, the nurse is
addressing the core public health function of:
Assurance.

Generally ranked as the highest level of evidence:


Randomized controlled trial (RCT)

As randomized controlled trials are often inappropriate for


evaluation public health interventions, the most common

approach to establising evidence in public health is the


use of:
Case-control studies.

Melnyk and Fineout-Overholt (2010) described 7 Steps in


the EBP process
0. Cultivating a spirit of inquiry
1. Asking clinical questions
2. Searching for the best evidence
3. Critically appraising the evidence
4. Integrating the evidence with clinical expertise and client
preferences and values.
5. Evaluating the outcomes of the practice decisions or
changes based on evidence
6. Disseminating EBP results.

Approaches to EBP include:


Systematic review, meta-analysis, integrative review and
narrative review

Three Core public health functions:


Assessment, policy development, assurance

Core public health function: Assessment


- Diagnose and investigate health problems and hazards in
the community
- Mobilize community partnerships to identify and solve
health problems.
- Link people to needed health services
- Use EBP for new insights and innovative solutions to
health problems.

Core public health function: Policy Development


-Inform, educate and empower communities about health
issues.
- Develop policies and plans using EBP that supports
individual and community health effots

Core public health function: Assurance


- Monitor health status to identify community health
problems.
- Enforce laws and regulations that protect health and
ensure safety.
- Ensure the provision of health care that is otherwise
unavailable.
- Ensure a competent public health and personal health

care workforce.
- Use EBP to evaluate effectiveness, accessibility and
quality of personal and population-based services.

"Grading the strength of evidence"


When evidence is graded, the evidence is assigned a
'grade' based on the number and type of well-designed
studies and the presence of similar findings in all of the
studies.

The 'quality' of a study refers to:


The extent to which bias is minimized.

The 'quantity' of a study refers to:


The number of studies, the magnitude of the effect, and
the sample size.

The 'consistency' of a study refers to:


Studies that have similar findings, using similar and
different study designs.

POEM
"Patient oriented evidence that matters." Proposed by
Shaughnessy, Slawson and Bennett (1994), criteria for
evaluating the usefulness of evidence. In general, the
reader should ask "What are the results? Are they
important? Are the results valid? How can the results be
applied to client care?"

Barriers to adopting EBP in the community setting can


include:
-Miscommunication among nursing leaders about the
implementation process
-Inferior quality of available research or other types of
evidence
-Inability to assess and use evidence
- Unwillingness of organizations to fund research and make
decisions based on research
- The nurse's ability to make clinical decisions
- A lack of resources for implemnation in the setting (time,
funding, computer resources)

Four approaches allow the nurse to read


research/evidence in a condensed format:

Systematic review, meta-analysis, Integrative review,


Narrative review

Systematic review:
A method of indentifying, apparaising and synthesizing
research evidence. The aim is to evaluate and interpret all
available research that is relevant to a particular research
question. Is usually done by more than one person and
describes the methods used to search for and evaluate the
evidence.

Meta-analysis:
"A specific method of statistical synthesis used in some
systematic reviews, where the results from several studies
are quantitatively combined and summarized.

Integrative review:
A form of a systematic review that does not have the
summary statistic found in the meta-analysis because of
the limitations of the studies that are reviewed (e.g small
sample size of the population)

Narrative review:
A review is done on published papers that support the
reviewer's particular point of view or opinion and is used to
provide a general discussion of the topic reviewed. This
review does not often include an explicit or systematic
review process.

The Institue of Medicine has set a goal that by 2020, the


best available evidence will be used to make ___ of all
health care decisions.
90%.

Definition of Evidenced-based Medicine


"The conscientious, explicit and judicious use of current
best evidence in making decisions about the care of
individual clients." (Sackett et al 1996)
Pg 339

Research utilization is defined as:


"The process of transforming research knowledge into
practice" and "The use of research to guide clinical
practice."

A "paradigm shift"
A change from old ways of knowing to new ways of
knowing and practicing.

4 Ways of Knowing
Empirical knowledge, personal knowledge, aesthetic
knowledge, and ethical knowledge.

Empirical knowledge
The science of nursing

Aesthetic knowledge
The art of nursing

Personal knowledge
interpersonal relationships and caring

Ethical knowledge
Moral and ethical codes of conduct usually established by
professional organizations.

The term critical appraisal


is used to describe the process of evaluating the validity
and applicability of research studies.

Colonial Americans established systems of care for the


sick, poor, aged, mentally ill and dependents based on
___________.
Elizabethan Poor Law of 1601

The first hospital in the colonies: Founded when and


where?
In 1751, Pennsylvania Hospital, founded in Philadelphia.

The Shattuck Report


Published in 1850 by the Massachusetts Sanitary
Commission, called fopr major innovations: The
establishment of state health departments and local health
boards in every town, sanitary surveys and collection of
vital statistics, environmental sanitation, food, drug and
communicable disease control; well-child care, health
education, tobacco and alcohol control, city planning, and
the teaching of preventitive medicine in medical schools.

Sister Mary Augustine


Organized the Irish Sisters of Charity in Dublin in 1815.
These sisters visited the poor at home and established
hospitals and schools.

Nightingale's vision
Trained nurses and a model of nursing education
influencing the development of professional nursing, and
indirectly, public health nursing in the United States.
Inspired by the poorly educated, untrained and often
unreliable healthcare workers of the time.

District nursing
The Kaiserwerth Lutheran deaconesses incorporated care
of the sick in the hospital with client care in their homes.

William Rathbone
Nightingale's contemporary and friend, British
philanthropist. Founded the first district nursing
association.

The first nursing schools opened in:


The 1870's

Private duty nurses:


Lived with families of clients receiving care, often 24 hours
a day.

Visiting Nurses:
Became the key to communicating the prevention
campaign, through home visits and well-baby clinics.
Visitng nurses emphasized education of family members in
the care of the sick and in personal and environmental
prevention measures such as hygiene and nutrition.

Settlement Houses
Neighborhood centers that became hubs for health care,
education and social welfare programs.

Lilian Wald and May Brewster


Trained nurses that began visiting the poor on New York's
Lower East Side. They established a nurse's settlement
called Henry Street Settlement and later the Visiting Nurse
Service of NYC.

Lillian Wald
Established leader of public health nursing during it's early
decades. Known as the First Public Health Nurse in the US.
She persuaded the American Red Cross to sponsor rural
health nursing services across the country. Beginning in
1909, Walkd worked with Dr. Lee Frankel of MetLife to
implement the first insurance payment for nursing services.
She convinced business owners that keeping working
people and their families health increased productivity.

Jessie Sleet (Scales)


A canadian graduate of Provident Hospital School of
Nursing (Chicago), became the first African-American public
health nurse.

The American Red Cross through its Rural Nursing Service


provided:
A framework to initiate home nursing care in areas outside
larger cities, provided care of the sick and instruction in
sanitation and hygiene in rural homes. Also improved living
conditions in villages and isolated farms.

Occupational health nursing:


Began as industrial nursing and was a true outgrowth of
early home visiting efforts.

Ada Mayo Stewart


Began work with employees and families of the Vermont
Marble Company in Proctor, Vermont in 1895. Provided free
service for the employees, including obstetric care,
sickness care and some post-surgical care in worker's
homes. Unlike contemporary occupational health nurses,
Stewart provided very few services for work related
injuries.

Physicians in 1897 thoughts on schools and children


Focused on excluding infectious children from school rather
than on provider or obtaining medical treatment to enable
children to return to school.

Lina Rogers
A Henry Street Settlement resident that became the first
school nurse, worked with children in New York City schools

and made home visits to instruct parents and to follow up


on children absent from school.

School nurses (including Lina Rogers) found that many


children were absent from school for these reasons:
Lack of shoes or clothing, because of malnourishment, or
because they were serving as babysitters. (Hawkins, Hayes
and Corliss, 1994 p.417)

National Organization for Public Health Nursing (NOPHN)


Established in 1912, Lillian Wald first president. NOPHN
membership included both nurses and their lay supporters.

NOPHN sought to:


"Improve the educational and services standards of the
public health nurse, and promote public understanding of
and respect for her work." The NOPHN was soon the
dominant force in public health nursing.

In 1914 the first post-training program for public health


nursing at Teachers College in NYC because
Diploma schools of nursing emphasized hospital care of
patients, and graduates were often unprepared for home
visiting and required additional training. The American Red
Cross provided scholarships to attend the piblic health
nursing course.

American Public Health Association (APHA)


Established in 1872 to facilitate interprofessional efforts
and promote the 'practical application of public hygiene.'
The APHA targeted reform efforts toward contemporary
public health issues, including sewage and garbage
disposal, occupational injuries and STD's.

Frances Root
The first trained nurse in the United States, who was
salaried as a visiting nurse.

The National Organization for Public Health Nursing


Founded in 1912, provided essential leadership and
coordination of diverse public health nursing efforts; the
organization merged into the National League of Nursing in
1952.

Sheppard-Towner Act of 1921

Expanded community health nursing roles for maternal and


child health during the 1920's.

Implementation of Medicare and Medicaid programs


established:
Established in 1966, established new possibilities for
supporting community-based nursing care but encouraged
agencies to focus on services provided after acute care
rather than on prevention.

Between 1900 and 1955, the national crude mortality


rate decreased by ___%.
47%

The American Nurses Association


Merged with the National Association of Colored Graduate
Nurses, second national nursing association.

Definition of "Community" according to the WHO


"A group of people, often living in a defined geographical
area, who may share a common culture, values and norms
and are arranged in a social structure according to
relationships which the community has developed over a
period of time. Members of a community gain their
personal and social identity by sharing common beliefs,
values and norms which have been developed by the
community in the past and may be modified in the future."
( WHO, 2004)

Typologies
List of types, which involve classifying communities by
category rather than single definitions.

"Communities of place"
In this type of community, interactions occur within a
specific geographic area. Neighborhood and face-to-face
communities are two examples of this type of community.

'Communities of special interest'


Cut across geographic areas. Common concerns and
intererst, which can be long term or short term in nature,
bring their members together (e.g a group to support a
smoke-free environment.)

"Community of problem ecology"

Created when environmental problems such as water


pollution affect a widespread area.

In most definitions, the community includes three factors:


People, place and function

Community factor: People


The community members or residents, demographic
structure of population such as age, race, socioeconomic
and racial distributions. Informal groups such as block
clubs, service clubs as well as formal groups such as
schools, churches, businesses etc.

Community factor: Place


Refers to both geographic and to time dimensions.

Community factor: Function


Refers to the aims and activities of the community.

Community partnership approach


More effective than the 'I'm the community health expert
and I know best" are more effective in identifying and
addressing health problems, and such programs are more
likely to be sustained over the long term. Healthy People
2020 cites commmunity partnership as KEY to meeting
program goals.

Aggregate:
A collection of individuals who have in common one or
more personal or environmental characteristics.

The parts of a community are _________.


Interdependent, and their function is to meet a wide variety
of collective needs.

Public Health Nurses:


"integrate community involvement and knowledge about
the entire population with personal, clinical understandings
of the health and illness experiences of individuals and
families within the population."

The community is the client only when the nursing focus


is on:

The collective or common good of the population instead of


on individual health.

Population-centered practice:
seeks healthful change for the whole community's
benefit.Although the nurse may work with individuals,
families or other interacting groups, aggregates, or
institutions or within a population, the resulting changes
are intended to affect the whole community.

Community health is characterized by which 3


dimensions:
Status, structure and process dimensions.

Community Health: Status


Otherwise known as Outcome the most well-known and
accepted approach; it involves biological, emotional and
social parts.

Indicators of health status assessment: Death Rates


Due To:
1. Race/ethnicity-specific infant mortality
2. MVCs
3. Work-related injury
4. Suicide
5. Lung/Breast cancer
6. Cardiovascular disease
7. Homicide
8. All other causes

Indicators for Assessing Community Health Status:


Reported Incidence per 100,000 of the following:
1. AIDS
2. Measles
3. Tuberculosis
4. Primary and secondary Syphilis

Indicators for Assessing Community Health Status: Risk


Factors
1. Incidence of low birth weight
2. Births to adolescents
3. Prenatal care
4. Childhood poverty
5. Proportion of persons living in counties exceeding U.S
Environmental Protection Agency standards for air quality
during previous year.

Biological attributes of community health status:


Mobidity and Mortality, information found at the Bureau of
Vital Statistics

Social attributes of health status:


Includes crime rates and functional levels.

Emotional attributes of health status:


Include consumer satisfaction and mental health indices.

Community Health: Structure


usually defined in terms of services and resources.
Measures of ocmmunity health services and resources
include service use patterns, treatment data from various
health agencies and provider to client ratios.

Measures of community structures:


Demographics, such as socioeconomic and racial
distributions, age and educational level.

Community Health: Process:


Includes the concept of community competence, or the
ability of the parts of a community-organizations, groups
and aggregates to collaborate effectively in identifying the
problems and needs of the community. It directs the study
of community health to identify community action to
address community problems.

The most important aspect of the nursing community


assessment phase can best be described as:
Formulating a community nursing diagnosis.

The 10 Essential Conditions of Community Competence:


1. Commitment
2. Awareness of self and others
3. Articulateness
4. Effective communication
5. Conflict containment and management of the
accommodation
6. Participation
7. Management of relations with larger society
8. Machinery for facilitating participant interaction and
decision making

9. Social support
10. Leadership development

The three main characteristics of a successful


partnership:
Being informed, flexibility, negotiation.

Phase One of nursing process within the community:


Define community and establishment of
contract/partnership.

4 Steps to Assessing Community Health


1. Gathering relevant existing data and generating missing
data.
2. Developing a composite database.
3. Interpreting the composite database to identify
community problems and strengths.
4. Analyzing the problem.

Nursing Process of Community: Phase Two


Assessment

Data gathering:
the process of obtaining existing, readily available data.

Community Nursing Process: Phase Three


Nursing diagnosis of the problem

Community Nursing Process: Phase Four


Planning

Three Steps of Planning


1. Prioritize problems
2. Establish goals and objectives for each problem
3. Develop intervention activities to meet established
objectives.

Community Nursing Process: Phase Five


Implement Interventions

Community Nursing Process: Phase Six


Evaluate Interventions

Data Generation

The process of developing data that do not already exist


through interaction with community members or groups.

Four methods of generating direct data:


Informant Interview, focus groups, participant observation
and windshield studies

Informant interviews
i.e focus groups, directed talks with selected members of a
community about community members or groups and
events

Participant observation
The deliberate sharing in the life of a community
Four methods of generating direct data:
Informant Interview, focus groups, participant observation
and windshield studies

Informant interviews
i.e focus groups, directed talks with selected members of a
community about community members or groups and
events

Participant observation
The deliberate sharing in the life of a community
Community
A collection of people who interact with one another and
common interest forms a unity

Geographic community
A group of people usually defined by geographic
boundaries

American Public Health Association definition of public


health
Protect. Promote. Prevent

Evidence Based Practice


using evidence (research findings), along with clinical
judgment and patient wishes, in making decisions on how
to care for patients

Roles of Community Health Nurse


Clinician, Educator, Advocate, and Researcher

community of solution
a group of people who come together to solve one problem
- shape and size may vary depending on problem

aggregate
a mass of individuals who are considered as a whole but
are loosely connected

population
all people occupying an area or may also refer to people
who share one or more characteristics

health
a holistic state of well-being including a soundness of mind,
body, spirit

U.S. leading health indicators


used to measure the health of the US - serves as basis for
Healthy People 2020

goal of health promotion


raises the level of wellness for individuals, families,
communities, and populations

health promotion goals


1) increase span of healthy life for all citizens
2) reduce health disparities among population groups
3) achieve access to preventive health services for
everyone

levels of Prevention
Primary example immunization
Secondary example breast screening with mammography
Tertiary example diabetes education after diagnosis

Florence Nightingale
Author of modern day nursing

Lillian Wald

first person to use the term, "public health nursing"; started


the Henry Street Settlement; suggested school health
nursing

Margaret Sanger
a nurse who openly opposed the Cromstock act, arrested;
her defiance lead to International Planned Parenthood
Federation

core public health functions


Assessment
Policy Development
Assurance

five steps of evidence based practice


Ask the question
Collect the data/evidence
Critically appraise the evidence
Integrate into practice
Evaluate the decision

ethical decision making


making a choice that is consistent with moral code

cultural sensitivity
recognizing that culturally based values, beliefs, and
practices influence people health's and lifestyles and
should be considered in plans for service

epidemiological triad
host
agent
environment

chain of causation
begins by identifying the reservoir, then portal of exit, then
the agent, then the portal of entry - all set by the
environment

immunity
ability to resist particular infectious disease causing agent active, passive, cross-immunity

herd immunity

describes the immunity level within a group - mandatory


school immunizations relate to herd immunity

risk
the probability that a disease or health problem will occur

relative risk ratio


incidence rate of exposed group/incidence rate of
unexposed group. if greater than 1, those with risk factor
have a greater likelihood of acquiring than those who do
not have the risk factor

census data
data taken from the population - in the US every 10 years

reportable diseases
requirements to report certain diseases to local health
authority

descriptive epidemiology
observe or describe a pattern of disease in a population

rates
statistical measures expressing proportion of people with a
given health problem among a population at risk

prevalence
at a given point of time all the people with a particular
health condition

prevalence rate
number of people with characteristic/total population

incidence
all new cases of disease or health condition within a period
of time

incidence rate
number of new cases of disease or health condition/total
number at risk for developing disease. Note: once a person
has the disease/condition and there is not the possibility of
getting the disease/condition again then the risk is not
there and that number is subjected from the total
population.

mortality rate
sum of deaths in given population at a given time

analytic epidemiology
beyond simple description, seeks to determine associations

modes of transmission communicable disease


direct
indirect
airborne
vector

DOT
direct observed therapy - client takes the medication in the
presence of health care worker

quarantine
period of forced isolation of a person exposed to
communicable disease during the incubation period to
prevent the spread of the disease

ecological perspective
total relationship or patterns of relationship among people
and environment

built environment
structures built by humans within the environment - can be
used positively or negatively - urban vs rural

lead exposure issues


neurological issues, preterm birth, learning disabilities

epidemic
disease occurring that exceeds the normal or expected
frequency in a community

pandemic
worldwide distribution of disease

endemic
naturally occurring in population

communication
transferring meaning and enhancing understanding

health literacy
the ability to read, write, understand, and act upon health
information

collaboration
purposeful exchange between nurses, clients, health
professionals with shared values, mutual participation, and
joint effort

contracting
negotiating a working agreement between two or more
parties with shared understanding and mutual consent over
a period of time.

health promotion
behavior motivated by a desire to increase well-being and
actualize human potential (Pender, Murdough, & Parsons,
2006)

health protection (disease prevention)


behavior motivated by a desire to avoid disease, detect it
early, or maintain functioning within constraints of disease

Health Belief Model


theoretical model that has been extensively tested to
understand the behaviors and actions people take to
prevent illness and injury

Pender's Health Promotion Model


revised Health Belief Model that is used to predict
behaviors that influence health promotion

Quality Improvement Elements


Structure
Process
Outcomes

social marketing
influencing behavior in targeted audiences

social justice

both a process and a goal that includes a vision of an


equitable society and all members are physically and
psychologically safe and secure

advocacy
pleading a case for another or championing a cause
Community
Group of people sharing something, interacting with one
another. May exhibit commitment with one another and
may share geographic boundary. E.g., a NORC, a school,
NYU students.

Population
Group of people having at least one thing in common and
who may or may not interact with one another. E.g., nightshift workers, children with CF.

Community-focused nursing
Delivery of nursing care to improve health of an entire
community - bigger picture, deals with prevention and
population at systems level.

Population-focused care
Interventions for a defined "at risk" population (e.g. people
with hypertension, weight problems)

Public health nursing


Promotion and protection of health of populations using
knowledge from nursing, social and public health sciences.
Population-focused; goals: promotion of health and
prevention of disability and disease by creating conditions
in which people can be healthy.

Acute Care
Provider control
Predictable routine
Hospital policy
Resources available
Collaboration and consultation
Controlled PT compliance
Standardized care

Community Health Nursing

Familiar and comfortable environment for PT


Routine more determined by PT
Diverse resources
Autonomy, choice in HC decisions for PT by PT

Health
"extent to which an individual or group is able on the one
hand, to realize aspirations and satisfy needs; and on the
other hand, to change or cope with the environment." WHO

Levels of Care
Individual - downstream
Family
Community
Population - upstream

Public health wheel


For selling health messages
R/t epidemiology

Levels of Prevention
Primary
Secondary
Tertiary

Primary Prevention
PREVENTION
Stay well and avoid problem
Reduction of risk factors before occurrence of disease
condition/injury
E.g., exercise, brushing teeth, condoms, immunizations,
nutrition.

Secondary Prevention
PREVENTION/MAINTENANCE
SCREENINGS/EARLY DX
Early detection of potential for disease/condition or
existence of such while asymptomatic.
E.g., screening programs - Pap smears, mammograms,
colonoscopy.

Tertiary Prevention
RESTORATIVE
Treatment of existing or symptomatic disease to

prevent/delay progress.
Treatment after Dx, e.g, cardiac/stroke rehab.

Healthy People 2020


New topic areas:
adolescent health, blood, dementias inc. AD, early/mid
childhood, genomics, global health, HAIs, health related
QoL and well being, LGBT health, older adults,
preparedness, sleep, social determinants.
"Prevention Agenda for the Nation"

Public Health Nursing


Promoting, protecting health of populations using
knowledge from nursing, social and public health sciences.
Population focused.

Public Health Nursing Goals


Promotion of health and prevention of disease and
disability for all through creation of conditions in which
people can be healthy.

Assurance
Ensuring healthy populations
Appropriate and accessible services for all
Healthful physical environment
Stable ecosystem
Strong, supportive, non exploitive membership
Extensive participation in decision making
Members' basic needs provided for.
Access to resources and opportunities for interaction
Sustainable vital economy
Maintain connectedness with cultural and biological
heritages
Provide governance structures that promote health.

Community health work


unstructured, "bigger picture", community is both client
and partner, work at every level but especially advocate for
primary level with population focus.

Newest Vital Sign


Health literacy test: being able to read food labels.

Nursing Process in the community

ADPIE:
Assessment
Diagnosis (written in different format for the community)
Planning
Implementation
Evaluation

Community Assessment
Population assessment
Primary informants
Gaining entry (Kauffman's 5 phases)

Collecting data
IOSFCW
7 methods
Informant interviews
Observation
Secondary analysis of existing data
Focus groups
Community forums
Windshield surveys

Community Diagnosis
Risk of X
Among X group/population

Planning and Prioritization Phase


Priorities established (set by community members)
Goals and objectives IDed
Community focused interventions

Implementation Phase
Action phase
Role of community health nurse
Social change, community action

Evaluation
formative - changes made during process
summative - at end, related to outcomes

Health education
"The pedagogy of the oppressed"
Community empowerment (WHO)
Community as partner model (Betty Neuman)

Community as Partner
Community core
8 interacting community subsystems
Community stressors/boundaries
Normal line of defense LOD = status quo
Flexible line of defense - protection vs stressors
Lines of resistance LOR - strengths, resources, coping
mechanisms

Health Education
Planned learning experiences based on sound theories to
provide individuals, groups, communities opportunity to
learn info and skills to make quality health decisions

Health Education Goals


Help individuals, groups, communities achieve, by own
actions/initiative, optimal health
Facilitate voluntary actions to promote health
Empowerment
Advocacy
Self-efficacy

Planning health education


Establish outcomes - measurable
Mutual goal setting with community as client

Elizabethan Poor Law


Assisted the poor and disabled with receiving care

Marine Hospital
Established by Congress
Facility became the Public Health Service

Florence Nightingale
Established a training school for nurses, Established
sanitary nursing care units. Founder of modern nursing.
began professional education of nursing.

Clara Barton
Established The Red Cross, Launched the American Red
Cross in 1881. An "angel" in the Civil War, she treated the
wounded in the field.

Lillian Wald

"founder of the Henry Street Settlement and Visiting Nurse


Service, which provided nursing and social services and
organized educational and cultural activities; considered
the founder of public health nursing"
- President of the National Organization for Public Health
Nursing

Mary Breckinridge
Founder of the Frontier Nursing Service, in Leslie County,
KY; 1st midwifery program in U.S.

The Shattuck Report 1850


Report by the MA Sanitary Commission - recommended
changes to the health system for control and prevention of
diseases

Sheppard-Towner Act
U.S. Act of Congress providing federal funding for maternity
and child care, a response to the lack of adequate medical
care for women and children

Nightingale's Theory of Environment


the inter-relationship nursing, health and environment in
the health/illness of an individual or community

Orem Self Care Deficit


Individuals must take action to maintain life and health

Pender Health Promotion Model


Individuals will seek health promotion experiences to
improve their well-being

Community-Oriented Nursing
Client: Community
Focus: Health of community as a "whole"

Community-Based Nursing
Client: Individual, family, or groups
Focus: Individuals, families, and groups within the
community

Epidemiological Triangle
Host - Agent - Environment and (Vector)

Intermediary (Living) Vectors


e.g. Mosquitos, fleas, rodents, birds, ticks

Vehicle (Non-living) Vectors


e.g. Clothing, food, water

Incidence (new cases)


# cases in population at a specific time / population total x
1,000

Prevalence (existing at a particular time)


# cases in population at a specific time / population total x
1,000

Crude Mortality Rate


# deaths / population total x 1,000

Infant Mortality
# infants (< 1 yr.) specific year / # live births in the same
year

Attack Rate
# people developing a certain disease / # people at risk
Better nutrition, water, antibiotics, immunizations. PUBLIC
HEALTH.
Life Expectancy - U.S.
What factors/events resulted in the 21+ additional years of
life from 1900-1950???
What factors/events resulted in the 7 additional years of
life from 1950-1990?

1. Upstream
2. Downstream
There are two views of the health care picture
We need to question the way we have thought of health
and health care...
Should we focus on treatment of illness or prevention of
disease in the first place?
How should we act?
______ or ______

1. Microscopic
2. Medical Model
3. Downstream
_______ approach to solving community health problems
(the _____ or _____):
- Individual (family) response to health and illness
- Emphasizes behavioral responses to illness or lifestyle
patterns
- Nursing interventions aimed at the individual
------Changing lifestyles
------Changing perceptions or belief system
- The individual is the locus of change
- Focuses on the "cure"

1. Macroscopic
2. Public Health
3. Upstream
______ approach to solving community health problems
(______ or _______)
- Interfamily and intercommunity themes
- Emphasizes social, economic, and environmental
precursors of illness
- Nursing interventions may include modifying social or
environmental variables
- May involve social or political action
- society (social system) is the locus of change
- focuses on "prevention"

1. Health Belief Model


2. Health Promotion Model
3. Transtheoretical Model
4. Critical Social Theory
CHN practice is based upon theory. Theories are used
regularly in CHN. Some examples:
1.
2.
3.
4.

1.
2.
3.
4.
5.

The Health Belief Model


Perceived susceptibility
Perceived severity
Perceived benefits
Perceived barriers to action

6. Cues to action
7. Self-efficacy
_________:
- focuses on a person's perceptions of a threat or a health
problems and related appraisal of a recommended
behavior to elicit change
- developed in the 1950s and was one of the first models
using concepts and assumptions from behavioral sciences
to examine health behaviors
------- Addresses attitudes and beliefs of individuals
------- Focus was on increasing the use of preventive health
services
- Primary assumptions of the model are that people fear
disease and that health actions are motivated based on the
extent of the fear and belief in benefits obtained by
preventative action
- Five (Six?) Primary Constructs:
1.
2.
3.
4.
5.
6.

- "Using the Health Belief Model to Predict Injecting Drug


Users' Intentions to Employ Harm Reduction Strategies"
(Bonar & Rosenberg, 2011)
Studied whether perceived risk of infection would
impact intention to clean site pre-injection. Also studied
perceived risk of non-fatal overdose and use of a test
shot.
Found that the perceived risk of infection and overdose
did impact behavior if the user was not in withdrawal.
Application of the HBM in CHN:

Pender's Health Promotion Model


__________:
o Developed by Nola Pender to study health promotion
behaviors; initially published in 1982
o Explores biopsychosocial processes that motivate
individuals to engage in behaviors that promote health
o Has basis in the HBM
o Depicts the complex multidimensional factors with which
people interact as they work to achieve optimum health.
o This model does not include threat as a motivator, as
threat may not be a motivating factor for client's in all age
groups.

o Used by nurses to develop and execute health-promoting


interventions
o Used to develop research studies focusing on one aspect
of health promotion
o Used frequently as a framework for research studies
o Individual Characteristics and experiences that may
affect their health actions:
- Prior related behavior
- Personal factors
- Behavior specific cognitions and affect
- Perceived benefits of action
- Perceived self-efficacy
- Activity-related affect
- Interpersonal influences
- Situational influences
- Commitment to a plan of action
- Immediate competing demands and preferences
- Health-promoting behavior

"Registered Nurses' Beliefs of the Benefits of Exercise,


Their Exercise Behavior and Their Patient Teaching
Regarding Exercise" (Esposito & Patrick, 2011).
Wanted to know if nurses who engage in healthy
behaviors were more likely to recommend them to their
patients.
Found that nurses who believe in health promotion and
embrace healthy behaviors were more likely to be
positive role models and were more likely to teach
healthy behaviors to their patients.
- More examples: Health promotion for community dwelling elders and Prevention of melanoma
Application of the HPM in CHN:

1. Transtheoretical Model (Stages of Change)


2. Precontemplation
3. Contemplation
4. Preparation
5. Action
6. Maintenance
_________:
o Based on the assumption that behavior change takes
place over time
o Change is difficult
o Change progresses through a sequence of stages
-----------One may stop at a stage, progress to the next
stage or return to the previous stage

o Stages of change: 1.______, 2._____, 3.______, 4._____,


5.______
o Decisional Balance:
---------Pros: The benefits of behavior change
---------Cons: The costs of behavior change

1. Precontemplation
2. Contemplation
3. Preparation
4. Action
5. Maintenance
Transtheoretical Model (Stages of Change):
1. _______: No intention to change behavior in the next 6
months
------May be lack of information about consequences or
previous failure.
2. _______: Individual intends to change behavior in the next
6 months.
------Weighs pros and cons
3. _______: Individual intends to act within the next month
and has taken steps toward change
------Has a plan of action
4. _______: Individual has changed behavior for less than 6
months
------Change is sufficient to reduce disease risk.
5. _______: Individual has changed behavior for more than 6
months.
------Tries to prevent relapse
------Phase may last months to years

"Adapting the Transtheoretical Model of Change to the


Bereavement Process" (Calderwood, 2011).
Applies the model to the bereavement process.
Postulates that bereaved persons never return to their
previous state but undergo change as they cope with and
adjust to life without their loved one.
Application of the TTM in CHN:

1. Smoking cessation
2. Injury prevention
3. RNs assisting families obtain health insurance
Examples for applying Transtheoretical Model in nursing:
1.
2.
3.

1. Critical Social Theory


2. Enlightenment
3. Empowerment
4. Emancipation
_____________:
- Jurgen Habermas is the best known of the theorists for
this
-------Habermas was part of a group of German scholars in
Frankfurt writing in the 1960s
-------He promoted critical social theory to describe
"distortions and constraints that impede free, equal and
uncoerced participation in society."
- began in Marx's argument that oppression requires
revolutionary action
- uses societal awareness to expose social inequalities that
keep people from reaching their full potential
-------Promote equality
-------Reduce disparities (ie. health disparities)
- aims to provide an environment in which individuals can
become empowered, enlightened and emancipated
-------- ______: raising the consciousness of the oppressed.
-------- ______: encourages people to undertake activities to
improve their situation
-------- ______: is the goal of empowerment through which
new arrangements replace oppressive ones
- This perspective is informed by the following values and
assumptions:
---------- The problem and inequalities of health and health
care are connected to the particular historically located
social arrangements and the cultural values of society
---------- Health care should be oriented toward the
prevention of disease and illness
---------- The priorities of any health care system should be
based on the needs of the clients/population and not health
care providers
---------- Ultimately, society itself must be changed for
health and medical care to improve

"Why Carers of Older People are not using Available


Respite Services: An Australian Study (Stockwell-Smith,
Kellett, & Moyle, 2010).
Examined use of respite services by caregivers to
determine why some opted not to use these services.
Found that a lack of knowledge of services and trust in
the workers were factors in using respite care.
Found that caregivers were defined by their role and
could not easily relinquish it to another.

Application of CST in CHN:

1. Health Disparities
2. Health and social justice
Examples of applying critical social theory:
1.
2.

- A state of complete well-being, physical, social, and


mental, and not merely the absence of disease or
infirmity. World Health Organization, 1958
- The extent to which an individual or group is able, on
the one hand, to realize aspirations and satisfy needs;
and, on the other hand, to change or cope with the
environment. Health is, therefore, seen as a resource for
everyday life, not the objective of living; it is a positive
concept emphasizing social and personal resources, and
physical capacities. WHO 1986
How do we define health?

1. Purposeful actions
2. Processes
3. Responses
4. Behaviors
Definitions of Health:
Health consists of:
_____, ______, _____, or _____ that leads to: "soundness,"
"wholeness," or "well-being"

1. Physical Environment
2. Social Environment
3. Individual Behavior
4. Biology & Genetics
5. Health Services
6. Policy making
Determinants of Health:
Health Outcomes due to:
1.
2.
3.
4.
5.
6.

Physical Environment
Determinants of Health - Examples of ______ factors:
- Natural environment (ie plants, weather, climate change)
- Worksites, schools and recreational settings
- Housing, homes, neighborhoods
- Exposure to toxic substances
- Physical barriers
- Aesthetic elements (ie good lighting, trees, parks)

Social Factors
Determinants of Health - Examples of ______ factors:
- Availability of resources (ie educational and job
opportunities, living wages, healthful foods)
- Social norms and attitudes
- Exposure to crime and violence
- Social interactions
- Exposure to emerging technology (ie the Internet)
- Transportation options

Individual Behaviors
Determinants of Health - Examples of ______ factors:
- Diet
- Physical activity
- Alcohol, tobacco and other drug use
- Sexual behavior
- Hand washing

Biology and Genetics


Determinants of Health - Examples of ______ factors:
- Age
- Sex
- Race/ethnicity
- Inherited conditions
- Family history of diseases

Health Service
Determinants of Health - Examples of ______ factors:
- Barriers (lack of availability, high cost, lack of insurance,
language issues)
------- Unmet health needs
------- Delays in receiving care
------- Inability to get preventive services
------- Preventable hospitalizations

Policy Making

Determinants of Health - Examples of ______ factors:


- Health curricula requirements
- Increased taxes on tobacco
- Prohibitions on smoking
- Drinking age restrictions
- Seat belt laws and child restraints

1. Life expectancy
2. Health life expectancy
3. Years of potential life lost
4. Physically and mentally unhealthy days
5. Self-assessed health status
6. Limitation of activity
7. Chronic Disease Prevalence
Indicators of General Health Status:
1.
2.
3.
4.
5.
6.
7.

Community
___________:
- It can be a physical place or a geopolitical community.
- Has boundaries
- It can be a relational, interactive group. - A community
with no physical boundaries. A phenomenological
community is abstract. Churches, universities, online
groups are examples

Community
__________:
"A collection of people who interact with one another and
whose common interests or characteristics form the basis
for a sense of unity or belonging." (Nies and McEwen,
2011)

Community
__________:
"A group of people who share something in common and
interact with one another, who may exhibit a commitment
to one another and may share a geographic boundary."

1. People
2. They must interact with each other in some way
3. Something in common: an interest, a geographical
location, a commitment
A community has to have:
1.
2.
3.

1. Primary Prevention
2. Secondary Prevention
3. Tertiary Prevention
Levels of Prevention:
1.
2.
3.

Primary Prevention
_______ level of prevention
- activities preventing a problem before it occurs
- Health promotion
- Specific protection
- Immunizations

Secondary Prevention
________ level of prevention
- Early detection and prompt intervention
- Screening
- Early referral for treatment
- Screening for STDs

Tertiary Prevention
_______ level of prevention
- Focus on limitation of disability and rehabilitation
- Prevention progression of disease
- Reduce the effects of the disease
- Teaching insulin administration

Community Health Nursing (CHN)


_________:
o Focus on individuals and families where they live, work,
and go to school; care is setting-specific and the emphasis
is on acute and chronic care
o "The synthesis of nursing practice and public health
practice applied to promoting and preserving the health of
populations.

Public Health Nursing (PHN)


_________:
o Focus is on the community as a whole and the effect of
community health status (resources) on the health of
individuals, families and groups
o Consideration of the health of individuals, families, and
groups and their effect on the health of the community as a
whole
o The synthesis of public health and nursing practice.
o "The practice of promoting and protecting the health of
populations using knowledge from nursing, social, and
public health sciences."

Public/Community Health Nursing (PHN)


__________:
- According to the ANA, PHN is the
o ...practice of promoting and protecting the health of
populations
o ...using knowledge from nursing, social and public health
sciences

Public Health Nursing


_______ Practice is:
1. Population focused
2. Goals of promoting health and preventing disease and
disability
3. Focuses on all people through the creation of conditions
in which people can be healthy

Public/Community Health Nursing


__________:
- Focuses on the entire population
- Is based on assessment of the population's health status
- Considers the broad determinants of health
- Emphasizes all levels of prevention
- Intervenes with communities, systems, individuals, and
families
- like an umbrella, Covers:
--------- Home Health Nursing
--------- Palliative and Hospice Nursing
--------- Public Health Nursing (official agencies)
--------- School Nursing
--------- Occupational Health Nursing
--------- Correctional Nursing

o ...practice is general & comprehensive; not limited to an


age or diagnostic group
o ...is continuing, not episodic
o ...is directed to communities, groups, & individuals as it
contributes to the health of the total population
o Goal is to protect and promote the health of the entire
population
o Populations can be defined by geography, demographic
characteristic or need
What is Public Health Nursing (PHN)?

Anywhere people come together


Neighborhoods
Public Health Agencies
Community Centers
Day Care Centers
Schools
Workplaces
Housing projects
Settings for Public Health Nursing:
1.
2.
3.
4.
5.
6.
7.
8.

Community-based nursing
___________:
o Nursing of individuals and families to improve their
health
o Goals: Help them manage illness while they move among
health care settings
o Promote self-care and rehabilitation; prevent disease
o Processes: NP; diagnosis and treatment

Wherever individuals and families need nursing care


outside the hospital or nursing home
Home
Community clinics
School clinics
Workplace clinics
Rehabilitation centers
Settings for Community-based nursing:
1.

2.
3.
4.
5.
6.

Clinical Nursing
_________:
o Goal: improve the health of patients
o Clients: Patients of the health care system
o Processes used: nursing process, treatment and patient
care procedures
o Settings: inpatient

The Public Health Nurse


The Standards of Care:
__________:
- Assesses the health status of populations using data,
community resource identification, input from the
population and professional judgment
- Analyzes collected assessment data and partners with the
people to determine population diagnosis and priorities
- Participates with other community partners to identify
expected outcomes in the populations and their health
status based on population diagnoses and priorities
- Develops a plan that reflects best practices by identifying
strategies, action plans and alternatives to attain expected
outcomes
- Implements the identified plan by partnering with others
- Employs multiple strategies to promote health, prevent
disease and ensure a safe environment for populations
- Evaluates the health status of the population

1. Morbidity data - illness or disability


2. Mortality data - death
Indicators of Health and Illness:
- Health Statistics:
1.
2.

1.
2.
3.
4.
5.
6.

Surveillance
Disease and other health event investigation
Outreach
Screening
Case finding
Referral and Follow-up

7. Case management
8. Delegated functions
9. Health Teaching
10. Counseling
11. Consultation
12. Collaboration
13. Coalition Building
14. Community Organizing
15. Advocacy
16. Social Marketing
17. Policy and enforcement
Public Health Interventions:
1.
2.
3.
4.
5.
6.
7.
8.
9.
10.
11.
12.
12.
13.
14.
15.
16.
17

Surveillance
Describes and monitors health events through ongoing and
systematic collection, analysis and interpretation of health
data for the purpose of planning, implementing and
evaluating public health interventions

Disease and other health event investigation


Systematically gathers and analyzes data regarding threats
to the health of populations, ascertains the source of the
threat, identifies cases and others at risk, and determines
control measures

Outreach
Locates populations of interest or populations at risk and
provides information about the nature of the concern, what
can be done about it, and how services can be obtained

Screening
Identifies individuals with unrecognized health risk factors
or asymptomatic disease conditions in populations

Case finding
Locates individuals and families with identified risk factors
and connects them with resources

Referral and Follow-up


Assists individuals, families, groups, organizations and/or
communities to identify and access necessary resources to
prevent or resolve problems or concerns

Case Management
Optimizes self-care capabilities of individuals and families
and the capacity of systems and communities to coordinate
and provide services

Delegated Functions
Direct care tasks a registered professional nurse carriers
out under the authority of a health care practitioner as
allowed by law

Health Teaching
Communicates facts, ideas, and skills that change
knowledge, attitudes, values, beliefs, behaviors, and
practices of individuals, families, systems, and/or
communities

Counseling
Establishes an interpersonal relationship intended to
increase or enhance capacity for self-care and coping with
a community, system, and family or individual

Consultation
Seeks information and generates optional solutions to
perceived problems or issues through interactive problem
solving with a community, system, and family or individual

Collaboration
Commits two or more persons or organization to achieve a
common goal through enhancing the capacity of one or
more of the members to promote and protect health

Coalition Building
Promotes and develops alliances among organizations or
constituencies for a common purpose

Community Organizing
Helps community groups to identify common problems or
goals, mobilize resources, and develop and implement
strategies for reaching the goals they collectively have set

Advocacy
Plead someone's cause or act on someone's behalf, with a
focus on developing the community, system, and individual
or family's capacity to plead their own cause or act on their
own behalf

Social Marketing
Utilizes commercial marketing principles and technologies
for programs designed to influence the knowledge,
attitudes, values, beliefs, behaviors, and practices of the
population of interest

Policy development and enforcement


Places health issues on decision-makers' agendas, acquires
a plan of resolution, and determines needed resources,
resulting in laws, rules, regulations, ordinances, and
policies. Policy enforcement compels others to comply with
laws, rules, regulations, ordinances, and policies

John Snow
_______: Father of epidemiology

1. Pasteur
2. Lister
3. Koch
____, _____, _____: On "germs" and disease causation (late
1800s)

1. Edward Jenner
2. Edwick Chadwick
History of Public Health Nursing:
Pre 1850: Home visiting to sick as an act of charity.
- ______ observed people who worked around cattle were
less likely to have smallpox.

- _______ called attention to the consequences of unsanitary


conditions that resulted in health disparities that shortened
the life span of the laboring class in particular.

1. John Snow
2. Florence Nightingale
3. Pasteur, Lister, Koch
4. Lilian Wald
5. Mary Brewster
History of Public Health Nursing:
1850-1900: Home visiting to sick
- _______ demonstrated that cholera was transmissible
through contaminated water.
-________ - credited with establishing "modern nursing."
- ________: On "germs" and disease causation (late 1800s)
- In 1893, nurses _______ ("mother of community nursing")
and _______ established a district nursing service on the
lower east side of New York City called The House on Henry
Street.

1. 1900-1960s
History of Public Health Nursing:
________:
- Nursing in community centers for the poor
- Communicable disease control
- Communicable disease control/immunization
- Rise in PHN, home visits, school and OH nursing

1. 1960-1980
History of Public Health Nursing:
________: Care provided in public health clinics

1. 1980s-present
History of Public Health Nursing:
_________:
- Health promotion and education
- Health care access improvement

1. Poverty
2. Violence
3. HIV/AIDS

4. TB
5. Bioterrorism
Public/Community Health Nursing Today in U.S.
- Concerned with contemporary problems such as _____,
______, ______, _______, and _______.
- Practice is everywhere in public health agencies, schools,
occupational settings, community-based agencies, etc.

Healthy People
__________:
- provides science-based, 10-year national objectives for
improving the health of all Americans. For 3 decades, this
has established benchmarks and monitored progress over
time in order to:
o Encourage collaborations across communities and
sectors.
o Empower individuals toward making informed health
decisions.
o Measure the impact of prevention activities.
- Vision: A society in which all people live long, healthy lives

1. Identify nationwide health improvement priorities.


2. Increase public awareness and understanding of the
determinants of health, disease, and disability and the
opportunities for progress.
3. Provide measurable objectives and goals that are
applicable at the national, State, and local levels.
4. Engage multiple sectors to take actions to strengthen
policies and improve practices that are driven by the best
available evidence and knowledge.
5. Identify critical research, evaluation, and data
collection needs.
Healthy People 2020 strives to:
1.
2.
3.
4.
5.

1. Attain high-quality, longer lives free of preventable


disease, disability, injury, and premature death.
2. Achieve health equity, eliminate disparities, and
improve the health of all groups.
3. Create social and physical environments that promote
good health for all.
4. Promote quality of life, healthy development, and
healthy behaviors across all life stages.

Community Health Nursing Theories


Nighttingale's Theory of Environment
Health Belief Model
Milio's Framework for Prevention

Nightingale's Theory of Environment


Highlights relationship between an individual's environment
and health.
Depicts health as a continuum.
Emphasize preventive care.

Health Belief Model


Purpose to predict or explain health behavior.
Assumes that preventive behaviors are taken primarily for
the purpose of avoiding disease.
Emphasizes change at the individual level.
Describes likelihood of taking an action to avoid disease
based on:
Perceived susceptibility, seriousness, and threat of a
disease
Modifying factors
Cues to action (media, disease impact on family/friends,
recommendations from health care professionals).
Perceived benefits minus perceived barriers to taking
action.

Millie's Framework for Prevention


Complements the health belief model
Emphasizes change at the community level
Identifies relationship between health deficits and
availability of health promoting resources
Theorizes that behavior changes within large number of
people can ultimately lead to social change.

Community Health Nurse


Nurses who practice in the community.
Have a facility from which they work (community health
clinic, county health department), but not limited to
institutional settings

Community
Group of people and institutions that share geographic,
civic, and/or social parameters

Community-Based Nursing
Focus of Care (FOC):
Individuals, families
Nursing Activities (NA):
Illness care: Management of acute and chronic conditions
in settings where individuals, families, and groups live,
work, and "attend" (schools, camps, prisons)

Community-Oriented Nursing
FOC:
At-risk individuals, families, and groups
Community
NA:
Health care: Determining health needs of a community,
and intervening at the individual, family, and group level to
improve the collective health of the community

Community Health Nursing Practice


FOC:
Synthesis of nursing and public health theory
NA:
Promote, preserve, and maintain the health of populations
by the delivery of health services to individuals, families,
and groups in order to impact "community health"

Public Health Nursing Practice


FOC:
Synthesis of nursing and public health theory
NA:
Promote, preserve, and maintain the health of populations
through disease and disability prevention and health
protection of the community as a whole.
Core functions:
Systematic assessment of the health of populations
Development of policies to support the health of
populations
Ensuring that essential health services are available to all
persons

Principles Of Community Health Nursing


Overview:
Ethical considerations
Advocacy
Epidemiology
Epidemiological Triangle

The Epidemiological Process


Community-Based Health Education

Ethical Considerations
The Public Health Code of Ethics identifies the ethical
practice of public health.
Preventing harm
doing no harm
promoting good,
respecting both the individual and community rights
respecting autonomy and diversity
providing confidentiality
competency
trustworthiness
advocacy

Applications of Ethical Principles To Community Health


Nursing
Respect for autonomy
individuals select those actions that fulfill their goals
Situations: respecting a client's right to self-determination
(making a decision not to pursue chemotherapy)

Applications of Ethical Principles To Community Health


Nursing
Nonmaleficence
No harm is done when applying standards of care
Situations: Developing plans of care that include a system
for monitoring and evaluating outcomes

Applications of Ethical Principles To Community Health


Nursing
Beneficence
Maximize possible benefits and minimize possible harms.
Situations: Assessing risks and benefits when planning
interventions

Applications of Ethical Principles To Community Health


Nursing
Distributive Justice
Fair distribution of the benefits and burden in society is
based on the needs and contributions of its members
Situations: Determining eligibility for health care services
based on income and fiscal resources

Advocacy

The nurse plays the role of informer, supporter, and


mediator for the client.
The following are basic to client advocacy
Clients are autonomous beings have right to make
decisions affecting their own health and welfare
Right to expect a nurse-client relationship that is based on
trust, collaboration, and shared respect, related to health,
and considerate of their thoughts and feelings.
Clients responsible for their own health.
Nurse's responsibility to advocate for resources or services
that meet the client's health care needs.
Advocating requires assertiveness, placing priority on the
client's values, and willingness to progress through the
chain of command.

Epidemiology
Study of health-related trends in populations for the
purpose of disease prevention, health maintenance, and
health protection
Relies on statistical evidence to determine the rate of
spread of disease and the proportion of people affected.
Used to evaluate the effectiveness of disease prevention
and health promotion activities and to determine the
extent to which goals have been met.
Provides broad understanding of the spread, transmission,
and incidence of disease and injury. This information is an
important component of community assessment and
program planning.
Involves the study of the relationships among an agent, a
host, and an environment (referred to as the
epidemiological traingle).

Epidemiological triangle
The Agent is the animate or inanimate object that causes
the disease.
Chemical Agents: Drugs, Toxins
Physical Agents: Noise, Temperature
Infectious Agents: Viruses, Bacteria
The host is the living being that is affected by the agent.
Susceptible Host: Age, Gender, Genetics, Ethnicity,
Immunological status, Physiological state, Occupation
The environment is the setting or surrounding that sustains

the host.
Physical Environment: Geography, Water/food supply,
Presence of reservoirs/vectors
Social Environment: Access to health care, High-risk
working conditions, Poverty

Epidemiological Calculations
Incidence: number of new cases in the population at a
specific time/population total X 1,000 = per 1,000
Prevalence: number of existing cases in the population at a
specific time/ population total X 1,000 = per 1,000
Crude Mortality Rate: Number of deaths/population total X
1,000 = per 1,000
Infant Mortality Rate: Number of infant deaths before 1
year of age in a year/numbers of live births in the same
year X 1,000 = per 1,000
Attack Rate: Number of people exposed to a specific agent
who develops the disease/total number of people exposed

Epidemic
When the rate of disease exceeds the usual level of the
condition in a defined population.

The Epidemiological Process


Determine the nature, extent, and possible significance of
the problem.
Nurse collects information from as many sources as
possible. This information is then used to determine the
scope of the problem.

The Epidemiological Process


Using gathered data, formulate a possible theory.
Nurse projects and explores the possible explanations.

The Epidemiological Process


Make the plan
Focus on breaking the cycle of disease.
All factors influencing the spread of the disease must be
considered and identified.
Priorities are established to break the chain of transmission
and to control the spread of disease.

The Epidemiological Process


Put the plan into Action
Using all available means, the nurse puts the plan for
controlling the disease into action

The Epidemiological Process


Evaluate the plan
Gather pertinent information to determine the success of
the plan.
Using this plan, evaluate the success in prevention of the
spread of the disease

The Epidemiological Process


Report and follow up
Nures will synthesize evaluation data into a format that is
understandable. Then evaluates the success and failures
and bases follow-up on the evaluation information.

Community Health Education


Learning Theories
Behavioral Theory: use of reinforcement methods to
change learners' behaviors
Cognitive Theory: Use of sensory input and reputation to
change learners patterns of thought, thereby changing
behavior
Critical Theory: Use of ongoing discussion and inquiry to
increase learners depth of knowledge, thereby changing
thinking and behaviors
Development Theory: Use of techniques specific to learners
developmental stages to determine readiness to learn, and
to impart knowledge
Humanistic Theory: Assists learners to grow by
emphasizing emotions and relationship and believing that
free choice will prompt actions that are in their own best
interest
Social Learning Theory: Links information to beliefs and
values to change or shift the learners expectations

Community Health Education


Learning Styles

Visual: learns through seeing, note taking, video viewing,


and presentations. Think in pictures.
Auditory: learns through listening, verbal lectures,
discussion, and reading aloud. Interpret meaning while
listening.
Tactile-kinesthetic: learn through doing, trial and error,
hands-on approaches, and return demonstration. Gain
meaning through exploration.

Development of a Community Health Education Plan


First, Identify population-specific learning needs.
Consider population-specific concerns and effect of health
needs on the population to determine the priority learning
need.
Select aspects of learning theories, to use in educational
program based on identified learning needs.
Identify barriers to learning, and learning styles
Design the educational program

Disease Prevention
Levels Of Prevention Focus
Primary Prevention: Prevention of the initial occurrence of
disease injury
Nutrition education
Family planning and sex education
Smoking cessation education
Communicable disease education
Education about health and hygiene issues to specific
groups (day care workers, restaurant workers)
Safety education (seat belt use, helmet use)
Prenatal classes
Providing immunizations
Advocating for access to health care, healthy environments

Disease Prevention
Levels Of Prevention Focus
Secondary Prevention: Early detection and treatment of
disease with the goal of limiting severity and adverse
effects
Community assessments
Disease surveillance (communicable diseases)
Screenings
*Cancer (breast, cervical, testicular, prostate, colorectal)
*Diabetes mellitus
*Hypertension

*Hypercholesterolemia
*Sensory impairments
*Tuberculosis
*Lead exposure
*Genetic disorders/metabolic deficiencies in newborns
Control of outbreaks of communicable diseases

Disease Prevention
Levels Of Prevention Focus
Tertiary Prevention: Maximization of recovery after an
injury or illness (rehabilitation)
Nutrition counseling
Exercise rehabilitation
Case management (chronic illness, mental illness)
Physical and occupational therapy
Support groups
Exercise for hypertensive clients (individual)
Community
Group of people sharing something, interacting with one
another. May exhibit commitment with one another and
may share geographic boundary. E.g., a NORC, a school,
NYU students.

Population
Group of people having at least one thing in common and
who may or may not interact with one another. E.g., nightshift workers, children with CF.

Community-focused nursing
Delivery of nursing care to improve health of an entire
community - bigger picture, deals with prevention and
population at systems level.

Community-based nursing
Nursing care provided outside of acute care setting. Care
for families, individuals within a community. At individual,
"hands on" level. Does not require public health
background.

Population-focused care
Interventions for health promotion, disease prevention to
shape a community's overall health status.

Public health nursing

Promotion and protection of health of populations using


knowledge from nursing, social and public health sciences.
Population-focused; goals: promotion of health and
prevention of disability and disease by creating conditions
in which people can be healthy.

Acute Care
Provider control
Predictable routine
Hospital policy
Resources available
Collaboration and consultation
Controlled PT compliance
Standardized care

Community Health Nursing


Familiar and comfortable environment for PT
Routine more determined by PT
Diverse resources
Autonomy, choice in HC decisions for PT by PT

Health
"extent to which an individual or group is able on the one
hand, to realize aspirations and satisfy needs; and on the
other hand, to change or cope with the environment." WHO

Eudaimonistic Model
HOLISTIC
UPSTREAM
Greek origins. R/t Maslow's hierarchy of needs.
MOST HOLISTIC
Health = actualization/realization of complete fulfillment
and development. Illness impedes/prevents selfactualization.
Aim to redirect thinking away from mechanistic view
toward holistic view.
HOLISM, UNITY, INDIVIDUALITY

Adaptive Model
ABLE TO ADAPT
Medicine beyond treatment of disease: from writings of
Dubos.
Health means ability to adapt to a changing environment.
Disease is failure to adapt.
How flexible is someone in a changing environment?

Role-performance Model
ABLE TO DO JOB
Medical sociology, work of Parsons.
Person is healthy if they can perform their job.
Shapes health policies of military, industry.
Limited since one person may have many roles.

Clinical Model
MOST LIMITING
DOWNSTREAM
Health extreme = absence of S/S of disease/disability as
IDed by medical science
This model = downstream pattern of thinking rather than
upstream which is holistic.

Levels of Care
Individual - downstream
Family
Community
Population - upstream

Public health wheel


For selling health messages
R/t epidemiology

Levels of Prevention
Primary
Secondary
Tertiary

Primary Prevention
PREVENTION
Stay well and avoid problem
Reduction of risk factors before occurrence of disease
condition/injury
E.g., exercise, brushing teeth, condoms, immunizations,
nutrition.

Secondary Prevention
PREVENTION/MAINTENANCE
SCREENINGS/EARLY DX
Early detection of potential for disease/condition or
existence of such while asymptomatic.
E.g., screening programs - Pap smears, mammograms,
colonoscopy.

Tertiary Prevention
RESTORATIVE
Treatment of existing or symptomatic disease to
prevent/delay progress.
Treatment after Dx, e.g, cardiac/stroke rehab.

Healthy People 2020


New topic areas:
adolescent health, blood, dementias inc. AD, early/mid
childhood, genomics, global health, HAIs, health related
QoL and well being, LGBT health, older adults,
preparedness, sleep, social determinants.
"Prevention Agenda for the Nation"

Public Health Nursing


Promoting, protecting health of populations using
knowledge from nursing, social and public health sciences.
Population focused.

Public Health Nursing Goals


Promotion of health and prevention of disease and
disability for all through creation of conditions in which
people can be healthy.

Assurance
Ensuring healthy populations
Appropriate and accessible services for all
Healthful physical environment
Stable ecosystem
Strong, supportive, non exploitive membership
Extensive participation in decision making
Members' basic needs provided for.
Access to resources and opportunities for interaction
Sustainable vital economy
Maintain connectedness with cultural and biological
heritages
Provide governance structures that promote health.

Community health work


unstructured, "bigger picture", community is both client
and partner, work at every level but especially advocate for
primary level with population focus.

Newest Vital Sign

Health literacy test: being able to read food labels.

Nursing Process in the community


ADPIE:
Assessment
Diagnosis (written in different format for the community)
Planning
Implementation
Evaluation

Community Assessment
Population assessment
Primary informants
Gaining entry (Kauffman's 5 phases)

Kauffman's Five Phases


IBSBC
Impressing
Behaving - building rapport
Swapping - sharing information
Belonging - becoming part of the group
Chilling out - near end of relationship, beginning longer
term partnership with the community.

Collecting data
IOSFCW
7 methods
Informant interviews
Observation
Secondary analysis of existing data
Focus groups
Community forums
Windshield surveys

Community Diagnosis
Risk of X
Among X group/population

Planning and Prioritization Phase


Priorities established (set by community members)
Goals and objectives IDed
Community focused interventions

Implementation Phase

Action phase
Role of community health nurse
Social change, community action

Evaluation
formative - changes made during process
summative - at end, related to outcomes

Health education
"The pedagogy of the oppressed"
Community empowerment (WHO)
Community as partner model (Betty Neuman)

Community as Partner
Community core
8 interacting community subsystems
Community stressors/boundaries
Normal line of defense LOD = status quo
Flexible line of defense - protection vs stressors
Lines of resistance LOR - strengths, resources, coping
mechanisms

Health Education
Planned learning experiences based on sound theories to
provide individuals, groups, communities opportunity to
learn info and skills to make quality health decisions

Health Education Goals


Help individuals, groups, communities achieve, by own
actions/initiative, optimal health
Facilitate voluntary actions to promote health
Empowerment
Advocacy
Self-efficacy

Steps in Teaching/Learning Process


ADDIE
Assessment
Development of outcomes
Development of teaching plan
Implementation of teaching plan
Evaluation of outcomes (did it work?)

Facilitate learning

Stimulate senses
Active learning
Comfortable
Learner's readiness?
Relevant information
Repetition
Learning should be positive
Start simple, go to complex
Generalize and pace appropriately

Transtheoretical Model
Most common learning theory
Stages of Change
Precontemplation - resistance; don't see problem
Contemplation - know there's a problem and thinking about
it
Preparation - decision making; prepared for action, e.g.,
cutting down on cigarettes.
Action - modifies behavior/environment to overcome
problem. Takes lots of time and energy. W/D S/S now.
Maintenance - constant work to avoid relapse.

Transtheoretical Model: Stages of Change


PCPAM
Precontemplation
Contemplation
Preparation
Action
Maintenance

Nature of learning
CAP = B F D
Cognitive Domain
Affective Domain
Psychomotor Domain

Cognitive Domain
BRAIN
memory, recognition, understanding, application, problem
solving.

Affective Domain
FEELINGS
attitudes, values:appreciates, chooses, accepts - role play,
talk to s.o., feelings.

Psychomotor Domain
DEMONSTRATE
Hands on manipulation, "teach back"
performance of skills requiring coordination and motor
skills.

Planning health education


Establish outcomes - measurable
Mutual goal setting with community as client

Educational objectives
Cognitive
Affective
Psychomotor
Community Health Nursing:
-Is it broad or specific?
-Wide variety of settings or limited?
-Do they promote health and welfare of pts?
-Specific age groups?
-Diverse populations or specific cultures?
-Nurses working in the community should have an
understanding of what 3 things?
Community Health Nursing:
-broad
-wide variety of settings
-allows nurses to practice in wide variety of settings
-Promote health and welfare of clients
-across the lifespan
-Nurses working in the community should understand the
foundations of CHN, the principles of CHN, and health
promo and disease prevention

What provides the basis for care of the community and


family?
Nursing theory

What are three examples of CHN theories?


Nightingale's theory of environment
Health belief model
Milio's framework for prevention

Nightingale's Theory of environment:


-What relationship does it highlight?
-What does it depict health as?
-What does it emphasis?

Nightingale's theory of environment:


-Highlights the relationship b/w an individual's environment
and health
-Depicts health as a continuum
-Emphasizes preventive care

Health belief model:


-What is the purpose?
-Purpose of preventive health behaviors?
-Emphasizes change at what level?
-Describes the likelihood of taking an action to avoid
disease based on what 4 things?
Health belief model:
-Purpose is to predict or explain health behaviors
-Preventive health behaviors- primarily taken to avoid
disease
-Emphasizes change at the individual level
-Describes the likelihood of taking an action to avoid
disease based on:
1)Perceived susceptibility, seriousness, and threat of a
disease
2)Modifying factors (e.g., demographics, knowledge level)
3)Cues to action (media campaigns, disease impact on
family/friends, recommendations from HCP)
4) Perceived benefits minus perceived barriers to taking
action

Milio's framework for prevention:


-What does it complement?
-What does it emphasize?
-What relationship does it identify?
-What does it theorize?
Milio's framework for prevention:
-Compliments the health belief model
-Emphasizes change at the community level
-Relationship b/w health deficits and availability of heath
promoting resources
-Behavior changes w/in a large number of people can
ultimately lead to social change

-What is a community?
-Do communities vary in their characteristics and health
needs?
-What is a community's health determined by?
-Community- group of people and institutions that share
geographic, civic, and/or social parameters.
-Communities vary in the characteristics and health needs.

-Community's health is determined by the degree to which


the community's collective health needs are identified and
met.

-What are health indicators?


-What are some examples?
-What do they serve as targets for?
-Used to describe the health status of a community.
-Health indicators- mortality rates, disease prevalence,
levels of physical activity, obesity, tobacco and substance
use
-Serve as targets for the improvement of a community's
health.

-Describe how community health nurses work: facilities,


limitations, etc.
-Community health nurses practice in the community. They
usually have a facility from which they work (community
health clinic, health department, etc.) but aren't limited to
institutional settings.

What does this describe: an aggregate who shares one or


more personal characteristic
A population

Who/what is the client in CHN?


Client is a community or a population w/in the community

When does community partnership occur?


It occurs when community members, agencies, and
businesses actively participate in the processes of health
promo and disease prevention

What is the development of community partnerships


critical to?
The development of community partnerships is critical to
the accomplishment of health promo and disease
prevention strategies

What are the purposes of population-focused nursing?


Purposes of population-focused nursing:
-Assessing to determine needs
-Intervening to protect and promote health
-Preventing disease w/in a specific population (people at

risk for HTN, people w/o insurance, people w/knowledge


deficits, etc.)

Community based nursing:


-Who is the focus of care?
-What are the nursing activities?
-Focus of care: individuals and families
-Nursing activities are r/t illness care: mgmt. of acute and
chronic conditions in settings where individuals, families,
and groups live, work, and "attend" (schools, camps,
prisons)

Community-oriented nursing:
-Who is the focus of care?
-What are the nursing activities?
-Focus of care: at risk individuals, family, groups as well as
community.
-Nursing activities are r/t health care: determining health
needs of a community, and intervening at the individual,
family, and group level to improve the collective health of
the community.

Community health nursing practice:


-Who is the focus of care?
-What are the nursing activities?
-Focus of care: synthesis of nursing and public health
theory
-Nursing activities are to promote, preserve, and maintain
health of populations by the delivery of health services to
individual, family, and groups in order to impact
"community health"

Public health nursing practice:


-Who is the focus of care?
-What are the nursing activities, including the core
functions?
-Focus of care: synthesis of nursing and public health
theory
-Nursing activities are to promote, preserve, and maintain
the health of population through disease and disability
prevention and health protection of community as a whole.
-Core functions: systematic assessment of health of
population, devotion of policies to support the health of
populations, and ensuring essential health services
available to everyone.

What do the principles for guiding CHN include?


Ethical considerations
Advocacy
Epidemiology
Epidemiological calculations
Epidemiological triangle
Epidemiological process
Community-based health education

What does the Public Health Code of Ethics identify?


It identifies the ethical practice of public health

What do ethical considerations include?


Ethical considerations include preventing harm, doing no
harm, promoting good, respecting both individual and
community rights, respecting autonomy and diversity, and
providing confidentiality, competency, trustworthiness, and
advocacy

What are community health nurses concerned with and


what do those concerns reflect?
-Community health nurses are concerned with protecting,
promoting, preserving, and maintaining health, as well as
preventing disease.
-These concerns reflect the ethical principle of promoting
good and preventing harm.

Is it easy or hard to balance individual and community


rights?
Balancing individual rights vs rights of community groups is
a challenge.

What challenge do community health nurses face?


Community health nurses address the challenges of
autonomy and providing ethical care

What do client rights include?


Client rights include the right to info disclosure, privacy,
informed consent, info confidentiality, and participation in
TX decisions

Autonomy:
-Definition
-Situation

-Individuals select those actions that fulfill their goals.


-Respecting a client's right to self-determination (making a
decision not to pursue chemo)

Nonmaleficence:
-Definition
-Situation
-No harm is done with applying standards of care.
-Developing plans of care that include a system for
monitoring and evaluation outcomes.

Beneficence:
-Definition
-Situation
-Maximize possible benefits and minimize possible harms.
-Assessing risks and benefits when planning interventions.

Distributive justice:
-Definition
-Situation
-Fair distribution of the benefits and burden in society is
based on the needs and contributions of its members.
-Determining eligibility for health care services based on
income and fiscal resources.

What are some basics for client advocacy?


Nurse plays the role of the informer, supporter, and
mediator for the client.

-Are clients autonomous beings?


-Do clients have the right to make decisions affecting
their own health and welfare?
-Are clients responsible for their own health?
Clients are autonomous beings, have the right to make
decisions affecting their own health and welfare, and are
responsible for their own health.

Whose responsibility is it to advocate for resources or


services that meet the client's health care needs?
It's the nurse's responsibility to advocate for resources or
services that meet the client's health care needs.

What three things are needed to advocate for the clients?

Advocating for clients requires assertiveness, placing


priority on the client's values, and willingness to progress
through the chain of command for resolution.

What is epidemiology?
The study of health-related trends in populations for the
purpose of disease prevention, health maintenance, and
health protection.

-What does epidemiology rely on to determine there rate


of spread of disease and the proportion of people
affected?
-What is it also used to evaluate?
It relies on statistical evidence to determine the rate of
spread of disease and the proportion of people affected.
-It is also used to evaluate the effectiveness of disease
prevention and health promo activities and to determine
the extent to which goals have been met.

What does epidemiology provide a broad understanding


of? What is that information an important component of?
It provides a broad understanding of the spread,
transmission, and incidence of disease and injury. This
information is an important component of community
assessment and program planning.

What type of things do community nurses do?


Identify cases, recognize patterns of disease, eliminate
barriers to disease control, and provide education and
counseling targeted at a disease condition or specific risk
factors.

What involves the study of the relationships among an


agent, host, and an environment?
Epidemiology

What does the interaction of the epidemiological triangle


determine?
It determines the development and cessation of
communicable diseases, and they form a web of causality,
which increases or decreases the risk for disease.

Epidemiological triangle:
What are the 3 components?
Agent, host, environment

What is the agent? Host? Environment?


Agent- animate or inanimate object that causes the
disease.
H0st- living being that is affected by the agent.
Environment- setting or surrounding that sustains the host.

What are some of the epidemiological calculations?


Incidence
Prevalence
Crude mortality rate
Infant mortality rate
Attack rate

What is the incidence?


Number of new cases in the population at a specific time
divided by population total x 1,000= ____ per 1,000

What is the prevalence?


Number of existing cases in the population at a specific
time divided by population total x 1,000 = ____ per 1,000

What is the crude mortality rate?


Number of deaths divided by population total x 1,000 =
_____ per 1,000

What is the infant mortality rate?


Number of infant deaths before 1 year of age in a year
divided by numbers of live births in the same year x 1,000
= _____ per 1,000

What is the attack rate?


Number of people exposed to a specific agent who develop
the disease divided by total number of people exposed

What is an epidemic?
The rate of disease exceeds the usual level of the condition
in a defined population

What are the 7 phases of the epidemiological process?


1) determine the nature, extent, and possible significance
of the problem.
2) using the gathered data, formulate a possible theory
3) Gather info from a variety of sources in order to narrow

down the possibilities


4) Make the plan
5) Put the plan into action
6) Evaluate the plan
7) Report and follow up

-What things influence the host?


-What things influence the physical and social
environment?
-What things influence chemical, physical, and infectious
agents?
-Host: age, gender, genetics, ethnicity, immunological
status, physiological state, occupation
-Physical environment: geography, water/food supply,
presence of reservoirs/vectors
-Social environment: access to health care, high-risk
working conditions, poverty
-Chemical agents: drugs and toxins
-Physical agents: noise and temperature
-Infectious agents: viruses and bacteria

Why do nurses regularly provide health education?


In order to promote, maintain, and restore the health of
populations

What barriers must nurses take into account when


designing community education programs?
Age, cultural barriers, poor reading and comprehension
skills, language barriers, barriers to access, and lack of
motivation

What does effective community health education require?


Planning

What are the 6 learning theories used in CHN?


Behavioral theory
Cognitive theory
Critical theory
Developmental theory
Humanistic theory
Social learning theory

What is the behavioral theory?


Use of reinforcement methods to change learners'
behaviors

What is the cognitive theory?


Use of sensory input and repetition to change learners'
patterns of thought, thereby changing behaviors

What is the critical theory?


Use of ongoing discussion and inquiry to increase learners'
depth of knowledge, thereby changing thinking and
behaviors

What is the developmental theory?


Use of techniques specific to learners' developmental
stages to determine readiness to learn, and impart
knowledge

What is the humanistic theory?


Assists learners to grow by emphasizing emotions and
relationships and believing that free choice will prompt
actions that are in their own best interest.

What is the social learning theory?


Links info to beliefs and values to change or shift the
learners' expectations

How do visual learners best learn?


They learn through "seeing" and methods such as note
taking, video viewing, and presentations. They "think in
pictures"

How do auditory learners best learn?


They learn through "listening" and methods such as verbal
lectures, discussion, and reading aloud. They "interpret
meaning while listening"

How do tactile-kinesthetic learners best learn?


They learn through "doing" and methods such as trial and
error, hands-on approaches, and return demonstration.
They gain "meaning through exploration"

What are the steps to develop a community health


education plan?
1) Identify population-specific learning needs.
2) Consider population-specific concerns and effect of
health needs on the population to determine the priority

learning need.
3) Select aspects of learning theories to use in the
educational program based on the identified learning
need.
4) Identify barriers to learning, and learning styles.
5) Design the education program.
6) Implement the education program. Ensure an
environment that is conducive to learning
7) Evaluate the achievement of learning objectives and the
effectiveness of instruction.

What are national health goals derived from? What are


these goals based on
Scientific data and trends collected during the prior
decade. These goals are based on those issues that are
considered major risks to the health and wellness of the US
population.

What is Healthy People?


Every 10 years it publishes the national health objectives
that serve as a guide for promoting health and preventing
disease.

What are some examples of preventive services?


Health education and counseling, immunizations, and other
actions that aim to prevent a potential disease or disability

What do successful screening programs provide?


Successful screening programs provide accurate, reliable
results, can be inexpensively and quickly administered to
large groups, and produce few if any side effects

What is primary prevention?


What are some examples?
Primary prevention is prevention of the initial occupancy of
disease or injury.
Nutrition education, family planning and sex ed., smoking
cessation education, communicable disease education,
education about health and hygiene issues to specific
groups (day care workers, restaurant workers), safety
education (seat belt use, helmet use), prenatal classes,
providing immunizations, advocating for access to health
care, healthy environments

What is secondary prevention?


What are some examples?

Secondary prevention is early detection and TX of disease


w/the goal of limiting severity and adverse effects.
Community assessments, disease surveillance
(communicable diseases), screenings, control of outbreaks
of communicable diseases

What is tertiary prevention?


What are some examples?
Tertiary prevention is maximization of recovery after an
injury or illness (rehab).
Nutrition counseling, exercise rehab, case mgmt. (chronic
illness, mental illness), PT and OT, support groups, exercise
for hypertensive clients (individual)

A nurse manager at a community agency is developing


an orientation program for newly hired nurses. When
discussing the differences between community-based and
community-oriented nursing, the nurse should include
which of the following as examples of community-based
nursing? (Select all that apply.)
A. A home health nurse performing wound care for a
client who is immobile.
B. An occupational health nurse providing classes on body
mechanics at a local industrial plant.
C. A school nurse teaching a student who has asthma
about medications.
D. A parish nurse teaching a class on low-sodium cooking
techniques.
E. A mental health nurse discussing stress mgmt.
techniques with a support group.
Answer: A and C.
A: Correct- community based nursing involves mgmt. of
acute and chronic conditions in a community setting
B: Incorrect- this is community-oriented nursing, which
involves health care of individuals, families, and groups to
improve the collective health of the community.
C: Correct- community based nursing involves mgmt. of
acute and chronic conditions in a community setting
D: Incorrect- this is community oriented nursing
E: Incorrect- this is community oriented nursing

A nurse is advocating for local leaders to place a newly


approved community health clinic in an area of the city
that has fewer resources than other areas. The nurse is
advocating for the leaders to uphold which of the
following ethical principles?
A. Distributive justice

B. Fidelity
C. Respect for autonomy
D. Veracity
Answer: A
A: Correct- distributive justice is fair distribution of benefits
and burden in society
B: Incorrect. Fidelity involves keeping commitments and
following through with promises.
C: Incorrect. Autonomy is supporting the rights of
individuals to determine and pursue personal HC goals.
D: Incorrect. Veracity is the concept of telling the truth.

A nurse is preparing an education program on disease


transmission for employees at a local day care facility.
When discussing epidemiological triangle, the nurse
should include which of the following as agents? (Select
all that apply.)
A. Resource availability
B. Ethnicity
C. Toxins
D. Bacteria
E. Altered immunity
Answer: C and D.
A: Incorrect- resource availability is r/t environment
B: Incorrect- ethnicity is r/t host
C: Correct- toxins are r/t agents
D: Correct- bacteria is r/t agents
E: Incorrect- altered immunity is r/t host

A nurse is developing a community health education


program for a group of clients who have a new diagnosis
of DM. Which of the following learning strategies should
the nurse include for clients who are auditory learners?
A. Showing informational videos
B. Providing equipment to practice hands-on skills
C. Supplying outlines for note-taking
D. Facilitating small group discussions
Answer: D
A: Incorrect- this is for visual learners
B: Incorrect- this is for tactile-kinesthic learners
C: Incorrect- this is for visual learners
D: Correct- this is for auditory learners

A community health nurse is implementing health


programs with several populations in the local area. In
which of the following situations is the nurse using
primary prevention?

A. Performing a home safety check in a client's home.


B. Teaching healthy nutrition to clients how have HTN.
C. Providing influenza immunization to employees at a
local preschool.
D. Implementing a program to notify individuals exposed
to communicable disease.
Answer: C.
A: Incorrect- performing a home safety check is secondary
prevention
B: Incorrect- teaching healthy nutrition to clients with HTN
is tertiary prevention
C: Correct- immunizations is a primary prevention task.
D: Incorrect- notifying exposed individuals is secondary
prevention
Community Health Nursing
blend of primary health care and nursing practice with
public health nursing

primary goal of community health nursing


raise level of health for citizens
promote physical/ mental health
prevent disease, injury, disability

home health care


enable individuals of all ages to remain in the comfor and
security of their homes while reiciving health care

services of home health care


skilled nursing
physical therapy
psychiatric therapy
pain education/managment
speech language therapy
occupational therapy
social services
intravenous therapy
acquistition of medical supplies/ equipment
home health aide
homemaker
petcare assistance
companion care
respiratory therapy
nutritional support

goal of home health care

allow as much independent as possible

4 perspectives of home health care


1. official
2. patient
3. family
4. provider

OFFICIAL home health care


services in home to promote, maintain, restore health
minimize effect of illness/ disability

PATIENT home health care


one on one health care provider in home
basic care/ individualized needs on personalized schedule
over a period of time = adjustment, change, learning takes
place effectively

FAMILY home health care


family together as functioning unit
goals: learning to adapt to change, prevent dysfunction,
family wellness, emotional support, community support

PROVIDER home health care


provide excellent care in less than excellent surroundings
goal: independence, creativity, communication, clinical
skills, daily practices

Earliest organized home health care


1617
st. vincent de paul
sisterhood of the dames of charite
met social welfare and visiting nursing needs

Boston Dispensary
1796
1st home care program in US

1886
1st visiting nurse service in US
Philadelphia

Lillian Wald & Mary Brewster

1893
visiting nurse service
for the poor
NYC
Nurses Settlement House on Henry Street

Social Security Act of 1935


1st provided government rather than local charitable
funding for select services such as maternal health,
treatment for communicable diseases, training of public
health professionals

medicare became effective in ?


1996
revolutionized home care

medicare revolutionized home care by


1. changing it to a medical rather than nursing model of
practice
2. defining and limiting the services it reimbursed
3. changing the payment source and even changing the
reason for providing home care

Prospective Payment System (PPS)


1983
part of the Tax Equity and Fiscal Responsibility Act for
hospitals receiving Medicare reimbursement

Diagnosis-Related Groups (DRGs)


pays a set rate accordig to diagnosis for the hospitalized
patient's care rather than the "cost" or charges and
instituion traditionally bills according to its own schedule of
fees - discharge for such patients occured earlier and
patient required more nursing care

Types of Home Care Agencies


1. licensure by state
2. certification by state
3. certificate of need
4. accreditation by an outside agency

Licensure by state
gives legal permission to operate w/i state only

Certification by state

federal gov. set the rules governing certificates

Certificate of need
some states grant according to rules and formulas that
state regulator devise

Accredidation by outside agency


evaluates and judges how well the agency meets certain
standsards that the accrediting organization sets
(NLN community health accreditation program)

Before Medicare agencies classified by


Visiting nurse associations, state and local health
departments, nursing divisions, hospitals controlled
provision of home health care

Agencies classified according to


1. tax status - profit or non
2. location - freestanding or institution
3. governance - private or public

Joint Commission (TJC)


looking for agencies to establish ethics committees to
handle ethical issues that arise in the home

Changes to Home Health Care


1. psychiatric patients are required to be under the care of
psychiatrist & have diagnosis
2. social workers taking more active role
3. nurse pain specialists
4. agencies obtaining separate medicare certification for
hospice care
5.pet-care programs to reduce patient stress
6. electronic home visits
7. telemonitoring
8. home infusion therapy
9. home IV therapy

Types of Home Care Agencies


1. Voluntary
2. official
3. comvination
4. hospital
5. proprietary

6. private not for profit


7. other

State licensing boards & Professional organizations


dictate what?
functions and scopes of practice
including:
skilled nursing, physical therapy, speech-language therapy,
occupational therapy, medical social services, homemakerhome health aide

RNs provide what?


direct skilled nursing services

LVNs provide what?


basic nursing services under the supervision of the RN
needed skills:
self-direction, motivation, creativity, clinical proficiency,
flexibility, compassion, empathy, patience

Service Goals of Skilled Nursing


1. Restorative
2. improvement
3. maintenance
4. promotion

Restorative
return to previous level of function as appropriate/ realistic

Improvement
achieving better health and highter level of function than at
admission

maintenance
preserving functional capacities and independence by
maintaining current level of health

Promotion
teaching healthy lifestyles that keep the effect of illness or
disaibility to a minimum and prevent the recurrence of
illness

Skills of home health nurse

1.
2.
3.
4.
5.

technically proficiency
self motivated
independent decision maker
respond prompltly to problems
able to adapt to family/patient/ home enviroment

LVN home health duties


catheter care
ostomy care
wound care/ sterile dressing
obtaining specimen
injections
prefilling insulin
fingersticks for blood glucose
monitor physical status
set up/ monitor meds including IV
nutrition
therapeutic diet teaching/reinforcement
respiratory care, ventalation
tracheostomy care and suctioning
enemas
pain management
emotional support
preventative health measures
vital signs
patient / family teaching

physical therapy
licensed / qualified physical therapist is required, PT
assistant
goals: restorative and maintanance
rehab plan taught to patient/ family to promote self care

Speech Language Therapist


to be reimbursed by medicare:
masters prepared physician
certified by american speech/ hearing association
range from:
language relearning
eating swallowing disorders
lipreading

Occupational therapist

bachelor's level preparation


registered
choose / teach theraputic activities to restore funtional
levels:
promote independance
analysis of activities relating to patients' skin
disease management lifestyle
design , fabricate, fit, orthotic or self help devices
improve performance of activities of daily living, sensorymotor, cognitive, and neuromuscular function

Medical social services


focus on emotional and social aspects of illness
care plan includes:
edu
counseling
payment source identification
referrals
coping w/ stress and crisis intervention

Homemaker- Home Health Aide


Provide basic support services
medicare requires that a primary skilled or therapy service
(speech or physical) be provided before HHA services are
arranged

3 categories of aide services


1. personal
2. physical assistance
3. household chores

medicare / medicaid requirments of home health aide


onsite supervision every 2 weeks by RN
services provided in blocks of time ranging from 1-2 hours

Home Health Process


1. referral
2. admission
3. care plan
4. visits
5. documentation
6. discharge planning (by case manager)

Referral

comes from patient, family, social service, hospital,


physician, other agency

Admission
RN makes the initial eval and admission vist within 24-48
hours of the referreal
1. evaluation
2. enviromental assess.
3. i.d. of impairments
4. i.d. of impact of disease/ disability
5. assess support system
6. determine knowledge/adherence of treatment/ meds
7. involvment of patient/ fam in care plan
8. determin desire for care/ services
9. notify patient of rights
10. explain rights of self determination
11. initial nursing interventions (every 60 days)

Care Plan
from physician
-describes current physical status, meds, treatments,
disciplines, duration, goals, outcomes, time frame

Visits
to serve / meet patients- centered goals

Documentation
concise and complete doc. to provide accurate info of type
and quality of care
linked to legal implications

Discharge planning
done by case manager
begins with admission and ends when
patient goals or other specific criteria are met

Quality Assurance, Assessment, & Improvement


provide doc for evaluating specific criteria dn measures in
each area & evaluate them for compliance and
effectiveness
-reflect standards, objectives, measureable outcomes,
include plans for remediation or improvement
1. structural criteria
2. process criteria
3. outcome criteria

Structural Criteria
agencies overall practices

Process criteria
eval of care delivery

Outcome criteria
Measurement of change

Medicare
agency required to be certified
meet federal conditions of :
organization
staffing
training
services covered
agency eval
65 OR OLDER, DISABLED, END STAGE RENAL DISEASE

Medicaid
pays for home care services for LOW INCOME PPL OF ALL
AGES
state administers, federal subsidized

Third Party
pay for limited home care services
payment rates vary
posthospitalization recoveries tied to reimbursment

Private Pay
directly pay for home health services
charges range

HMOs
prefered provider organization
prepaid
based on prevention

PPOs
negotiated contracts with home health agencies

Nursing Process of LVN in home helth care

Plan care for patients based on needs


Review patients care plan / recommend provisions
Review / follow defined prioritization of patient care
Use clinical pathways, care maps, care plans to guide/
review patient care

NURSING PROCESS
assessment
analysis/ nursing diagnosis
planning
implementing
evaluation

Take aways
- current trends support growth of home care as an
economic, humane, preferred health delivery system
-medical management and control rather than cure are the
standards of care for illness
-home care provides assessment and eval of chronic
illnesses and is helpful for preventation in future
-aide and homemakers can provide necessary support in
ADLs to enable patient to remain @ home
-skilled nursing and therapy offer rehab and prevention of
deterioration, and methods to cope with physical changes

Long-Term care
defined by ANA as provison of physical, psychological,
spiritual, social, economic services to help ppl attain,
maintain, regain their optimum level of functioning
-range of services:
health maintenance care, to ppl who have lost ability to
function independantly due to chronic illness or condition

need for long term care


acute stage of an illness has resolved, patient continues to
need services to support and maintain physical and
psychological status and functional abilities

Patient-centered approach
achieve/ maintain an individualized plan of carre to assist
patient in preserving meaningful quality of life

quality of life
measure of optimum energy/ force that endows a person
with the power to cope successfully with full range of
challenges he or she encounters in the real world

Hospice
services to patients/families as end of life approaches.
available to any age-group
-maintain comfort as death approaches

pallative care
broader pop. having ability to benefit from comfort care
earlier in their illness or disease process.
provides care for
-basic needs (ADLs)
-ADLs
-pain and symptoms management
-spiritual/ psychosocial support

Adult DayCare
community-based programs to meet needs of functionally
or cognitibely impaired through supervised health and
social/ recreational activities
provide:
-physical care
-mental stimulation
-socialization
-assistance w/ maintanance
-health referrals
(during any part of the day, but for less than 24 hours)

Assisted Living
residential care
rental of small apartment
serveral personal care services:
-bathing, dressing, medications
(choice, autonomy, independance w/ supervision)
-communal dining and various social activities

HOME
least restrictive

INSTITUTIONAL
most restrictive

Institutional settings
1. subacute unit
2. long-term care facility

subacute unit
not limited by reimbursment
less expensive alt. to acute care when patient has high
acuity medical and nursing intervention needs
-bridge between acute care and long-term care
-located in free standing skilled nursing facilities

Long-Term care facility


dominant form
-24-hour care to individuals who do not require expensive
inpatient hospital services but who do not have options for
care at home or by other community agencies
-long or short term basis
-resident rather than patient
-most residents have more than one health disorder

short term resident


6 months

long term resident


duration of life

long-term interdisciplinary care team


professionals work togethre as an interdisciplinary team to
meet the needs of older adults

restorative nursing care


basic concepts of physical therapy for maintenance of
functional mobility and physical activity

omnibus budget reconciliation Act (OBRA)


nursing home reform legislation
defines requirements for the quality of care given to
residents

Health Care Financing Administration (HCFA)


administers and monitors the OBRA guidelines through
institutional surveys

Medicare
federally funded national health insurance program in US
for ppl older than age 65

Applications of the Omnibus budget reconciliation act of


1987
1. resident rights - fully informed resident / participates in
own health care plan
2. physical restraints - no restraints for discipline/
convinence
3. resident assessment - assessment is foundation of
planning and delivery of care
4. licensed nursing services - 24-hours a day
5. registered nurses - 8 hrs a day 7 days a week
6. nursing assistants - trained and competency tested

Ethical Issues for Long term care


-adherence to patients bill of rights
- advance directives
-DNR orders
-Power of attorney
-guardianship
-responsible party designation

Functional Nursing
each has a function adn reports back to head nurse

Team nursing
all work together toward common goal

residential assessment instrument (RAI)


three parts
1. MDS
2. RAPs
3. Utilization guidelines

Minimum Data Set (MDS)


provides a system for assessment of each residents
functional, medical, mental, psychosocial status

Resident Assessment Protocol (RAPs)


assessment guides that adress common clinical problems
-delirium, falls, urinary incontinence

Utilization Guidelines
wealth of clinical info to assist in assessment and care
planning

OBRA guideline for functional assessment docs

diff from acute care setting


vital signs/ weights required monthly

Nursing process for LVN in long term care


1. assessment -review plan of care every 90 days
2. Nursing diagnosis- id from assessment, prioritieze risk
3. Expected Outcomes/ planning
4. evaluation
autonomy
individuals select those actions that fulfill their goals,
clients decision

nonmaleficience
no harm is done when applying standards of care

beneficience
maximize possible benefits and minimize possible harms

distributive justice
fair distribution of the benefits and burden in society based
on the needs and contributions of its members

epidemiology
investigative study of disease trends in populations for the
purposes of disease prevention and health maintenance,
relies on statistics

agent
animate or inanimate object that causes the disease (virus,
trauma, fumes)

host
the living being that will be affected by the agent

environment
setting or surrounding that sustains the host

incidence
number of new cases

prevalence
existing disease in population at a particular time

infant mortality rate


number of infant deaths before 1 year of age in a year

attack rate
number of people at risk who develop certain disease

epidemic
the rate of disease exceeds the usual level of the condition

behavioral theory
focus on changing behavior through the use of
reinforcement menthods

cognitive theory
focus on changing thought patters through sensory input
and repetition

critical theory
focus on increasing depth of knowledge through discussion
and inquiry

developmental theory
focus on human developmental stage with methods that
are age-specific and age appropriate with importance given
to "readiness to learn"

humanistic theory
focus on feelings and relationships, principle that learners
will do what is in their best interest

social learning theory


focus on changing the learners expectations and beliefs
through methods that link information to beliefs and values

primary prevention
prevent occurrence, immunizations, classes

secondary prevention
early detection and treatment, screenings, treat STD

tertiary prevention
recovery after occurrence, rehab, shelter

airborne illnesses
measles, chickenpox, strep, TB, pneumonia, flu

foodborne illnesses
salmonella, Hep A, trich, E coli

waterborne illness (fecal contamination of water)


cholera, typhoid fever, giarda lamblia

vector borne -mosquito/tick illnesses


lyme disease, rocky mountain spotted fever, malaria

direct contact illnesses


STD's, mono, impetigo, lice, scabies

change agent
advocate for needed change at the local, state, or federal
level

lobbyists
persuade or influence legislators

coalitions
facilitation of goal achievement through the collaboration
of two or more groups

public office
serving society and advocating for change by influencing
policy development through public service

community based nursing


health of individuals, and groups within a community

community oriented nursing


health of the community as a whole

population focused nursing


assessments/ interventions for at risk populations

culture
beliefs, values, and assumptions about life that are widely
held among a group of people and are transmitted across
generations

environmental health
influence of environmental conditions on the development
of disease or injury

acculturation
process of learning a new culture

cultural awareness
self awareness of one's own cultural background, biases,
and differences

cultural competence
knowing, appreciating, and considering the culture of
someone else in resolving problems

WHO
World health organization, provides daily information
regarding the occurrence of internationally important
diseases, establishes world standards for antibiotics and
vaccines, focus on health care workforce and education,
environment, sanitation, infectious diseases, maternal and
child health, and primary care

Federal Health Agencies


under direction of secretary of health, funded through
federal taxes, Medicare, Medicaid, CDC, FDA

State Health Departments


WIC, handle licensing of nurses and nursing schools,
funded through state taxes and federal funding

Local Health Department


focus on health of its citizens, funded through local taxes,
state funds, and federal funds

informant interviews
direct discussion with community members for the purpose
of obtaining ideas and opinions from key informants

community forum
open public meeting

secondary data

use of existing data to assess problem

windshield survey
descriptive approach that assesses several community
components by driving through a community

focus groups
directed talk with a representative sample

surveys
specific questions asked in a written format

participant observation
observation of formal or informal community activities
A nurse manager at a community agency is developing
an orientation program for newly hired nurses. When
discussing the differences between community-based and
community-oriented nursing, the nurse should include
which of the following as examples of community-based
nursing? (Select all that apply.)
a. A home health nurse performing wound care for a
client who is immobile
b. An occupational health nurse providing classes on body
mechanics at a local industrial plant
c. A school nurse teaching a student who has asthma
about medications
d. A parish nurse teaching a class on low-sodium cooking
techniques
e. A mental health nurse discussing stress management
techniques with a support group
a, c

A nurse is advocating for local leaders to place a newly


approved community health clinic in an area of the city
that has fewer resources than other areas. The nurse is
advocating for the leaders to uphold which of the
following ethical principles?
a. Distributive justice
b. Fidelity
c. Respect for autonomy
d. Veracity
a

A nurse is preparing an education program on disease


transmission for employees at a local day care facility.
When discussing the epidemiological triangle, the nurse
should include which of the following as agents? (Select
all that apply.)
a. Resource availability
b. Ethnicity
c. Toxins
d. Bacteria
e. Altered immunity
c, d

A nurse is developing a community health education


program for a group of clients who have a new diagnosis
of diabetes mellitus. Which of the following learning
strategies should the nurse include for clients who are
auditory learners?
a. Showing informational videos
b. Providing equipment to practice hands-on skills
c. Supplying outlines for note-taking
d. Facilitating small group discussions
d

A community health nurse is implementing health


programs with several populations in the local area. In
which of the following situations is the nurse using
primary prevention?
a. Performing a home safety check at a client's home
b. Teaching healthy nutrition to clients who have
hypertension
c. Providing influenza immunizations to employees at a
local preschool
d. Implementing a program to notify individuals exposed
to a communicable disease
c

A nurse is preparing an educational program on cultural


perspectives in nursing. The nurse should include that
which of the following are influenced by an individual's
culture? (Select all that apply.)
a. Nutritional practices
b. Family structure
c. Health care interactions

d. Biological variations
e. Views about illness
a, b, c, e

A nurse is caring for a client who is from a different


culture than himself. When beginning the cultural
assessment, which of the following actions should the
nurse take first?
a. Determine the client's perception of his current health
status.
b. Gather data about the client's cultural beliefs.
c. Determine how the client's culture may impact the
effectiveness of nursing actions.
d. Gather information about previous client interactions
with the health care system.
b

A nurse is using the I PREPARE mnemonic to assess a


client's potential environmental exposures. Which of the
following is an appropriate question for the nurse to ask
to assess for "A" in the mnemonic?
a. "What do you like to do for fun?"
b. "What year was your residence built?"
c. "What jobs have you had in the past?"
d. "What industries are near where you live?"
a

A nurse is conducting health screenings at a statewide


health fair and identifies several clients who require
referral to a provider. Which of the following statements
by a client indicates a barrier to accessing health care?
a. "I don't drive, and my son is only available to take me
places in the mornings."
b. "I can't take off during the day and the local after-hours
clinic is no longer in operation."
c. "Only one doctor in my town is a designated provider
by my health maintenance organization."
d. "I would like to schedule an appointment with the local
doctor in my town who speaks Spanish and English."
b

A nurse is interviewing for a position at the local health


department. When preparing for the interview, the nurse

should find that which of the following are responsibilities


of this agency? (Select all that apply.)
a. Managing the Women, Infants, and Children program
b. Providing education to achieve community health goals
c. Coordinating directives from state personnel
d. Reporting communicable diseases to the CDC
e. Licensing of registered nurses
b, c

A nurse is preparing to conduct a windshield survey.


Which of the following data should the nurse collect as a
component of this assessment? (Select all that apply.)
a. Ethnicity of community members
b. Individuals who hold power within the community
c. Natural community boundaries
d. Prevalence of disease
e. Presence of public protection
a, c, e

A nurse is completing a needs assessment and beginning


analysis of data. Which of the following actions should the
nurse take first?
a. Determine health patterns within collected data.
b. Compile collected data into a database.
c. Ensure data collection is complete.
d. Identify health needs of the local community.
b

A nurse is planning a community health program. Which


of the following should the nurse include as part of the
evaluation plan?
a. Determine availability of resources to initiate the plan.
b. Gain approval for the program from local leaders.
c. Establish a timeline for implementation of
interventions.
d. Compare program impact to similar programs.
d

A nurse is conducting a community assessment. Which of


the following data collection methods is the nurse using
when having direct conversations with individual
members of the community?

a. Key informant interviews


b. Participant observation
c. Focus groups
d. Health surveys
a

A nurse is collecting data to identify health needs in the


local community. Which of the following are examples of
secondary data the nurse should review? (Select all that
apply.)
a. Birth statistics
b. Previous health survey results
c. Windshield survey
d. Community forum
e. Health records
a, b, e

A nurse is talking to a client who asks for additional


information about hospice. Which of the following is an
appropriate statement by the nurse?
a. "Clients who require skilled nursing care at home
qualify for hospice care."
b. "One function of hospice is to provide teaching to
clients about life-sustaining measures."
c. "Hospice assists clients to develop the skills needed to
care for themselves independently."
d. "A component of hospice care is to control the client's
symptoms."
d

An occupational health nurse is consulting with senior


management of a local industrial facility. When discussing
work-related illness and injury, the nurse should include
which of the following as physical agents? (Select all that
apply.)
a. Noise
b. Age
c. Lighting
d. Viruses
e. Stress
a,c

A newly hired occupational health nurse at an industrial


facility is performing an initial workplace assessment.
Which of the following should the nurse determine when
conducting a work site survey?
a. Work practices of employees
b. Past exposure to specific agents
c. Past jobs of individual employees
d. Length of time working in current role
a

A school nurse is scheduling visits with a physical


therapist for a child who has cerebral palsy. In which of
the following roles is the nurse functioning?
a. Direct caregiver
b. Consultant
c. Case manager
d. Counselor
c

A school nurse is planning health promotion and disease


prevention activities for the upcoming school year. In
which of the following situations is the nurse planning a
secondary prevention strategy?
a. Placing posters with images of appropriate hand
hygiene near restrooms
b. Routinely checking students for pediculosis throughout
the school year
c. Implementing age-appropriate injury prevention
programs for each grade level
d. Working with a dietician to determine carbohydrate
counts for students who have diabetes mellitus
b

A nurse at a community clinic is conducting a well-child


visit with a preschool-age child. The nurse should identify
which of the following as a manifestation of child neglect?
(Select all that apply.)
a. Underweight
b. Healing spiral fracture of the arm
c. Genital irritation
d. Burns on the palms of the hands
e. Poor hygiene
a, e

A nurse is caring for a client who is experiencing alcohol


withdrawal. Which of the following findings is a
manifestation of withdrawal?
a. Decreased blood pressure
b. Diaphoresis
c. Pin-point pupils
d. Bradycardia
b

A community health nurse is developing an education


program on substance use disorders for a group of
adolescents. Which of the following should the nurse
include when discussing nicotine and smoking?
a. Smoking is the fifth-most preventable cause of death in
the United States.
b. Nicotine is a central nervous system depressant.
c. Withdrawal effects from smoking are minimal.
d. Tolerance to nicotine develops quickly.
d

A community health nurse is developing strategies to


prevent or improve mental health issues in the local area.
In which of the following situations is the nurse
implementing a tertiary prevention strategy?
a. Providing support programs for new parents
b. Screening a client whose spouse recently died for
suicide risk
c. Teaching a client who has schizophrenia about
medication interactions
d. Discussing stress reduction techniques with employees
at an industrial site
c

A nurse at an urban community health agency is


developing an education program for city leaders about
homelessness. Which of the following should the nurse
include as the fastest-growing segment of the homeless
population?
a. Families with children
b. Adolescent runaways

c. Intimate partner abuse victims


d. Older adults
a

A nurse is preparing a community health program on


communicable diseases. When discussing modes of
transmission, the nurse should include which of the
following as an airborne illness?
a. Cholera
b. Malaria
c. Influenza
d. Salmonellosis
c

A home health nurse is discussing portals of entry with a


group of newly hired assistive personnel. Which of the
following are portals of entry the nurse should discuss?
(Select all that apply.)
a. Respiratory secretions
b. Skin
c. Genitourinary tract
d. Saliva
e. Mucous membranes
b, c, e

A newly hired public health nurse is familiarizing himself


with the levels of disaster management. Which of the
following actions is a component of disaster prevention?
a. Outlining specific roles of community agencies
b. Identifying community vulnerabilities
c. Prioritizing care of individuals
d. Providing stress counseling
b

A community health nurse is educating the public on the


agents of bioterrorism. Which of the following are
Category A biological agents? (Select all that apply.)
a. Hantavirus
b. Typhus
c. Plague
d. Tularemia
e. Botulism

c, d, e

A community health nurse is determining available and


needed supplies in the event of a bioterrorism attack. The
nurse should be aware that community members exposed
to anthrax will need access to which of the following
medications?
a. Metronidazole (Flagyl)
b. Ciprofloxacin (Cipro)
c. Zanamivir (Relenza)
d. Fluconazole (Diflucan)
b

A nurse is creating partnerships to address health needs


within the community. The nurse should be aware that
which of the following characteristics must exist for
partnerships to be successful? (Select all that apply.)
a. A leading partner with decision-making authority
b. Flexibility among partners when considering new ideas
c. Adherence of partners to ethical principles
d. Varying goals for the different partners
e. Willingness of partners to negotiate roles
b, c, e

A nurse is reviewing the various roles of a community


health nurse. Which of the following is an example of a
nurse functioning as a consultant?
a. Advocating for federal funding of local health screening
programs
b. Updating state officials about health needs of the local
community
c. Facilitating discussion of a client's ongoing needs with
an interprofessional team
d. Performing health screenings for high blood pressure at
a local health fair
b

A case management nurse at an acute care facility is


conducting an initial visit with a client to identify needs
prior to discharge home. After developing a working
relationship with the client, the nurse is engaging in the
referral process. Which of the following should be the first
action by the nurse?

a. Monitor the client's satisfaction with the referral.


b. Provide client information to referral agencies.
c. Review available resources with the client.
d. Identify referrals that the client needs.
d

A nurse developing a community health program is


determining barriers to community resource referrals.
Which of the following is an example of a resource
barrier?
a. Costs associated with services
b. Decreased motivation
c. Inadequate knowledge of resources
d. Lack of transportation
a

A nurse is working with a client who has systemic lupus


erythematosus and recently lost her health insurance.
Which of the following is an appropriate action by the
nurse in the implementation phase of the case
management process?
a. Coordinating services to meet the client's needs
b. Comparing outcomes with original goals
c. Determining the client's financial constraints
d. Clarifying roles of interprofessional team members
a
One of the primary focuses of improving the health of the
American people in the twenty-first century is to address:
a.Bioterrorism and global health threats
b.Delivery of individual care and hygiene
c.The need for increased hospital and acute care
d.Chronic disease and disability management
a. Bioterrorism and global health threats

The public health nurse must participate in the essential


services of public health. These include:
a.Monitoring health status by completing a community
assessment
b.Diagnosing and investigating health problems in the
world
c.Informing, educating, and empowering people about
health issues

d.Working in law enforcement to regulate health and


ensure safety
c. Informing, educating, and empowering people about
health issues

Which of the following is an example of the mission of


public health according to the Institute of Medicine?
a.Tracking avian flu outbreaks and doing surveillance in
the United States
b.Providing a flu shot for an elderly person at the health
department
c.Keeping track of alternative therapies in use in the
United States
d.Keeping snake antivenom at the Centers for Disease
Control and Prevention in Atlanta
a. Tracking avian flu outbreaks and doing surveillance in
the United States

Making sure that essential community-oriented health


services are available defines which of the core public
health functions?
a.Policy development
b.Assessment
c.Assurance
d.Scientific knowledge-based care
c. Assurance

Which of the following statements about public health is


accurate?
a.Prevention of early deaths can be more effectively
accomplished by medical treatment than by public health
approaches.
b.Expenditures and resources for public health have
increased in recent years.
c.Historically, gains in the health of populations have
been related largely to changes in safety, sanitation, and
personal behavior.
d.Reform of the medical insurance system is the single
change needed to improve the health of Americans.
c. Historically, gains in the health of populations have been
related largely to changes in safety, sanitation, and
personal behavior.

Collecting data and monitoring the health status of the


population defines which of the core public health
functions?

a.Assessment
b.Prevention
c.Assurance
d.Policy development
a. Assessment

Public health nurses who develop and implement local


public health policies through partnerships with agencies,
organizations, and consumers within the community are
using which core public health function?
a.Assessment
b.Prevention
c.Assurance
d.Policy development
d. Policy development

Providing for the availability of essential personal health


services for people who would otherwise not receive
health care defines which public health core function?
a.Assessment
b.Prevention
c.Assurance
d.Policy development
c. Assurance

Which is an example of the primary goal of public health?


a.Ensuring that a newly diagnosed 40-year-old
hypertensive man takes his medication
b.Finding home care for a 70-year-old client recuperating
from a hip replacement
c.Conducting an infant car seat safety check
d.Contacting a local hospice to admit a terminally ill 60year-old woman
c. Conducting an infant car seat safety check

The public health workforce should demonstrate


competency in which of the following competency
categories?
a.Financial planning and management
b.Workforce needs assessment
c.Acute care services
d.Curriculum development
a. Financial planning and management

Public health nursing is a specialty because:


a.It has a distinct focus and scope of practice.

b.It must be done by a registered nurse with a master's


degree.
c.It is focused on disadvantaged citizens.
d.It performs interventions at the acute care level.
a. It has a distinct focus and scope of practice.

A public health nurse provides a clinic for HIV-positive


citizens in the community. This is an example of:
a.Primary prevention
b.Secondary prevention
c.Tertiary prevention
d.Policy making
c. Tertiary prevention

Which statement about community health nursing


practice is correct?
a.It focuses on the delivery of personal health services to
individuals and families.
b.It provides care to protect the health of the community
as a whole.
c.It emphasizes the setting where care is provided for
clients and families.
d.It requires a baccalaureate preparation for practice.
a. It focuses on the delivery of personal health services to
individuals and families.

The nurse who conducts vision screenings on children in


the school setting is practicing _____ nursing practice.
a.Community-oriented
b.Public health
c.Community health
d.Community-based
d. Community-based

Which is an example of an aggregate or population?


a.Students in a county school system
b.Christians around the world
c.A patient in the intensive care unit at the local hospital
d.People who play cards together once a week at private
homes
a. Students in a county school system

A population is best defined as a:


a.High-risk group
b.School or institutional setting
c.Collection of individuals who share at least one common

characteristic
d.Geographical location within a community
c. Collection of individuals who share at least one common
characteristic

Population-focused practice focuses on defining the


problems or needs of and implementing solutions for:
a.Individuals
b.Aggregates
c.Communities
d.Geographical regions
b. Aggregates

Which of the following statements is true with regard to a


population focus in public health nursing?
a.Priority is given to the highest risk population.
b.Direct caregiving is limited to preventive measures,
such as administration of immunizations.
c.Attention is given to the population or community as a
whole, regardless of whether they do or do not access the
health care system.
d.Only populations outside institutional settings are
considered.
c. Attention is given to the population or community as a
whole, regardless of whether they do or do not access the
health care system.

A key opportunity for public health nurses to improve


population-focused care is by:
a.Assuming traditional nursing roles
b.Influencing public health policy
c.Conducting community assessments
d.Specializing in community-based nursing practice
b. Influencing public health policy

Which of the following are considered barriers to public


health nursing? Select all that apply.
a.The mindset that the only role for the nurse is at the
bedside.
b.The structures within which nurses work and the
process of role socialization within those structures.
c.Few nurses receive graduate-level preparation in the
concepts and strategies of the disciplines basic to public
health.
d.The number of job opportunities in the area has been
steadily declining.

a. The mindset that the only role for the nurse is at the
bedside.
b. The structures within which nurses work and the process
of role socialization within those structures.
c. Few nurses receive graduate-level preparation in the
concepts and strategies of the disciplines basic to public
health.

The threat of bioterrorism has the potential to:


a.Dissolve community-based programs.
b.Cause the health care system to collapse.
c.Divert funds from other public safety health care
programs.
d.Increase the need for shelters.
c. Divert funds from other public safety health care
programs

Population-based preventive programs launched in the


1970s are responsible for increased:
a.Use of tobacco
b.Use of automobile safety restraints
c.Incidence of hypertension
d.Incidence of obesity
b. Use of automobile safety restraints

The number and proportion of persons aged 25 or older


with less than a high school education is an example of:
a.Sociodemographic characteristics
b.Health status
c.Health risk factors
d.Health care resource consumption
a. Sociodemographic characteristics

The purpose of public health core functions is to:


a. Clarify the role of the government in fulfilling the
mission of public health.
b. Ensure the safety of populations in receiving quality
health care.
c. Provide community-based individualized care to every
person in the United States.
d. Unite public and private providers of care in a
comprehensive approach to providing health care.
a. Clarify the role of the government in fulfilling the mission
of public health.

The nurse who compares the rate of teenage pregnancy


in various areas of the city is practicing the public health
core function of:
a.Assurance
b.Assessment
c.Prevention
d.Policy development
b. Assessment

The nurse manager who makes sure that the staff


members who work in a local clinic are competent is
demonstrating the public health core function of:
a.Assurance
b.Assessment
c.Prevention
d.Policy development
a. Assurance

The public health nurse analyzes data related to the


number and type of United States Environmental
Protection agency air quality standards that a community
failed to meet. This data is an example of using which
community health profile indicator?
a.Sociodemographic characteristics
b.Health status
c.Health risk factor
d.Functional status
c. Health risk factor

The necessary basic preparation for public health nursing


is a(n) _____ in nursing.
a.Associate's degree
b.Baccalaureate degree
c.Master's degree
d.PhD
b. Baccalaureate degree

Public health practitioners prepared at the graduate level


should be able to:
a.Teach public and community health nursing
b.Assess and intervene successfully at the aggregate
level
c.Diagnose and treat disease and have prescriptive
authority
d.Run for political office as experts in public health policy

b. Assess and intervene successfully at the aggregate level

An example of a community health nursing practice is:


a.Administrating a flu shot to a client in a physician's
office
b.Conducting a flu shot clinic at a community center
c.Performing a client assessment in a hospital
d.Providing supervision of staff in a rehabilitation center
a.Administrating a flu shot to a client in a physician's office

The term community-based nursing care is applied when


implementing which nursing intervention?
a. Assessing the health needs of a defined community
b. Providing care to families in a community
c.Promoting the health of an entire community
d.Investigating environmental health problems in a
community
b. Providing care to families in a community

The nurse investigating environmental health problems


caused by contaminated ground water is best described
as practicing:
a.Community-oriented nursing
b.Community-based nursing
c.Policy development
d.Tertiary care
a. Community-oriented nursing

A public health staff nurse who has a clear understanding


of population-focused practice:
a.Is frustrated providing care to individuals
b.Would rather perform population-focused interventions
c.Is able to improve the effectiveness of care provided
d.Is considered a public health nurse specialist
c. Is able to improve the effectiveness of care provided

Proposed changes to the health care delivery system in


the United States will:
a.Provide new opportunities for public health specialists.
b.Result in isolated care being provided to individuals.
c.Emphasize specialty care.
d.Increase the utilization of acute care services.
a. Provide new opportunities for public health specialists.

Implementing quality performance standards in public


health is important because:

a. They are used to guide improvement in the public


health system.
b. They rigidly control public health.
c. Administrators will not monitor public health at the
local level, but instead at the national level.
d. They can be used as hiring guidelines for nurses.
a. They are used to guide improvement in the public health
system.

To better address emerging public health issues, a public


health nurse enrolls in a course addressing which content
area?
a.Leadership
b.Ethics
c.Communication
d.Finance
b. Ethics
Community-Oriented Nursing
Focus: Health of the community as a whole.
Client: Community
Activity: Surveillance and evaluation of the community's
collective health, and the implementation of strategies to
promote health and prevent disease.

Community-Based Nursing
Focus: Health of individuals, families, and groups within a
community.
Client: Individuals, family, or group of individuals
Activity: Provision of direct primary care in the settings
where individuals and families live, work, and "attend"
(schools, camps, parishes)

Community Health Nursing


Systematic processes to deliver care to individuals,
families, or community groups with a focus on promoting,
preserving, protecting, and maintaining health care
provided contributes to the health of the community.

Public Health Nursing


Nursing and public health, disease and disability
prevention, promoting and protecting the health of the
entire community

Eight domains of public health

Distinguishes public health nursing from other nursing


specialties by adherence to the eight principles:
Analytical assessment skills
Policy development and program planning skills
Communication skills
Cultural competency skills
Community dimensions of practice skills
Basic public health science skills
Financial planning and management skills
Leadership and systems thinking skills

Three primary functions of public health


1. Assess health care function
2. Develop policy that provides access to services
3. Ensure services are delivered and outcomes achieved

Health People 2020: Four main goals


1. Eliminate preventable diseases, disability, injury, and
premature death
2. Achieve health equity, eliminate disparities, and improve
health of all age groups
3. Create social and physical environments that promote
good health for all
4. Promote healthy development and health behaviors
across every stage of life

Social determinants
Impact whether someone is able to attain/maintain health;
(Income, social status, education, literacy, environment,
support networks, gender, culture, available health care)

Disparities
Gaps in care experienced by one population compared to
another

Health care changes in the 21st century


Increased patient acuity and number of services in the
community.
Patient centered care
Increased patient responsibility for own health
Increased use of technology

Florence Nightingale
Studied nursing in Germany then established nursing
schools in England

District Nursing
A mode of service delivery in which a community health
nurse is responsible for addressing all the health needs of a
given population

Lemuel Shattuck
First use of demographic data to look at population health
birth and death records

Dorthea Dix
Established first hospital for the mentally ill in the US

Clara Barton
Civil war nurse: Established the American Red Cross
(providing aid for natural disasters)

Lillian Wald
Found of public health nursing in the US; Founded the
Henry Street Settlement and Visiting Nurse Service which
provided nursing and social services and organized
educational and cultural activities. She is considered the
founder of public health nursing

Mary Breckenridge
Pioneer in nurse-midwifery; established the Frontier Nursing
Service - nurses traveled on horseback to reach mothers in
the hills of Kentucky

World Health Organization


International center that collects data, advances initiatives,
and offers support related to public health

US Department of Health and Human Services (USDHHS)


US branch of government responsible for health and
welfare of citizens

Nongovernment Agencies
Agencies that acquire resources from private sources to
assist others

Philanthropic Agencies
Organization that uses endowments or private funding to
address the needs of individuals, families, and populations

Center for Disease Control


(CDC) is a United States federal agency under the
Department of Health and Human Services. It works to
protect public health and safety by providing information to
enhance health decisions, and it promotes health through
partnerships with state health departments and other
organizations.

Difference in US health care system and public health


The US has an individual focus on curative measures while
public health focuses on the population or community as a
whole and focuses on disease reduction and improved
health

Local Level of Government


Implement public health activities within the community;
Governed by the state department;
Structure varies based on community needs

State Level of Government


Focuses on regulation of the community of program
responsibility and resource allocation

Federal Level of Government


Focuses on policy development and financing

Health
A quality, an ability to adapt to change or a resource to
help cope with challenges and processes of daily living

Well Being
A subjective perception of full functional ability as a human
being

USDHAHS 10 Components of public health practice


1. Preventing Epidemics
2. Protecting the environment, workplaces, and sources of
food and water
3. Promoting health behavior
4. Monitoring the health status of the population
5. Mobilizing community resources into service
6. Responding to disasters
7. Assuring the public there are trained personnel
8. Reaching out to those at high risk

9. Researching risk, disease acquisition, and ways to


prevent injury
10. Influencing policy to acquire resources to effect change

Epidemiological Health Promotion Model


The study of relationships among an agent, a host, and an
environment. The interaction determines the development,
and cessation of communicable disease.

Learning Theory
Goal established and reinforced by nurse; Rewards given
for partial accomplishment

Health Belief Model


Cues used to remind people of health behavior and
promote action; Perceived risk influences action

Transtheoretical Health Promotion Model


5-Step approach: Pre-contemplation, Contemplation,
Preparation, Action, Maintenance

Reasoned Action Health Promotion Model


Performance of a behavior is determined by a persons
intention to perform the behavior; Intentioned determined
by attitude and behavior

Social Support Health Promotion Model


Change influenced by support of friends, families, and
communities

Primary Level of Prevention


Utilize prevention strategies, address modifiable risk
factors, and maximize health and wellness

Secondary Level of Prevention


Planned effort to minimize the impact of disease and injury
once it has occurred

Tertiary Level of Prevention


Maximizing health and wellness through strategies set in
place at the end-stage of disease and injury

Epidemiology

The study of the distribution and determinants of health


and illness

Rate
Primary measurement used to describe either the
occurrance or the existence of a specific state of health or
illness

Outbreaks
Epidemic usually limited to a localized increase in the
incidence of the illness

John Graunt
Father of demographics. recognized importance of
recording birth and death rates and age structure of human
population

William Farr
Set up a system of data collection for causes of death in
difference occupations, gender, and imprisonment.
Importance of definition of illness and population
comparison, groups, and factors such as age, health, and
environment can affect statistics

John Snow
Used mapping and rates as an objective measure to
compare populations

Epidemiological Triad
Model based on the belief that health status is determined
by the interaction of the characteristics of the host, agent,
and environment

Wheel of Causation
Epidemiologic model that deemphasizes the agent as the
sole cause of disease while emphasizing the interplay of
physical, biological, and social environments

Web of Causation
Epidemioligcal model that strongly emphasizes the concept
of multiple causation while deemphasizing the role of
agents in explaining illness

Natural History of Disease

Course of disease or condition from the onset to resolution

Rate
Primary measure used to describe either the occurrence or
existence of a specific state of health or illness within a
group during a specific time frame

Ratio
Fraction that represents the relationship between two
numbers; Divide one quantity by another quantity
Ex: # boys in class/# girls in class

Proportion
Type of ration
Ex: # boys/ total students

Crude Rate
Measures the occurrence of the condition in the whole
population; May obscure info because it does not consider
factors such as age, race, gender
Numerator: Number of events
Denominator: Total population (not just those at risk)

Adjusted Rate
Controls for differences between populations-used for
comparison

Incidence
Measures probability that people without a condition will
develop the condition over time; measures pace of new
illness

Prevalence
Existence of a condition during a period/interval at a
specific point; Studies number of people diagnosed in the
past and length of illness; Longer length=greater the
prevalence-indicates burned of community

Mortality Rates
Crude mortality: probability of dying from any cause; #
deaths in a year/total population

Proportional Mortality Ratio

Compares death from a specific illness with deaths from all


causes; # deaths from specific cause within a time
frame/total deaths

Incidence Density
Used when unequal periods of observation for study
subjects; Accounts for people who die, drop out of a study,
or acquire an illness

Attributable Risk
The risk of a condition occurring in an exposed group that
is attributable to an exposure, not other factors

Relative Risk Ratio


Measure of the risk of developing a condition; Ratio of
incidence in exposed and incidence in non-exposed

Sensitivity
Ability of the test to correctly identify people who have the
health problem

Specificity
Specific incidence and prevalence rates calculated based
on specific characteristics (demographic data), variations
based on location and variations in time (short-outbreak,
periodic- seasonal, long-years)

Crude Rate Formula


# events within time period/population at risk with same
time pt

Incidence Rate
#new cases during time period/population at risk in the
same time period

Causality
Strengths of Association
Consistency
Temporality
Plausibility
Biological Gradient

Case Studies

In-depth analysis of individual or group, often first clue to a


problem

Cohort Studies
(Longitudinal Studies) Monitor over time to find
associations between risk factors and health;
Minimize selection bias;
Relative risk is the ratio of disease incidence in an exposed
population;
Indicates strength of exposure to illness causality

Case Control
Compare group with health problems (cases) to group
without (control)

Key Informants
Persons knowledgeable about specific aspects of a problem
and the communities current and past attempts to address
it

Stakeholders
An individual, organizational, or group that has interest
(stake) in a specific community health issue or the outcome
of a community level intervention

Gantt Chart
Visual of the sequence of steps to achieve objectives;
Developed in planning stages to identify steps, a tool for
scheduling tasks, monitor progress

Community of Interest
People and groups/aggregate that will be affected by
change; Those that will help bring about change

Upstream Approach to Health Care


Changes at societal level may impact health of community
without change is individual behavior

Sustainability
How to maintain change, support system in place

Social Justice
Health care is a risk; Address root cause of illness

Health Equities
Requires elimination of health disparities in living and
working conditions

SMART objectives
S- Specific
M- Measurable
A- Achievable
R- Relevant
T- Time

Coalition Building
Community members participate in planning and
implementing changes at the community level; Need clear
mission, goals, objectives, expectations, leadership,
accountability, and should be heterogenous

Community Readiness
Assess readiness to undertake change process-issue
specific

Web of Causation
Identify multiple factors that contribute to chronic disease

Lewins Model for Change


Unfreezing:
Changing:
Freezing:

Levers of Change
Increase driving/reinforcing forces
Decrease restraining/resisting forces

Force Field Analysis


Identify factors within a community that impact change and
forces that cannot be changes

Geopolitical Population
Has identified designated boundaries with the same
governing structure

Phenomenological Population
Group with common interests or beliefs who have
interpersonal and intrapersonal connection

Community of Solution
Formed by aggregate to address a health problem
The factors, exposures, characteristics, and behaviors
that determine patterns of disease are described using:
a. Descriptive epidemiology
b. Analytic epidemiology
c. Distribution
d. Determinants
d. Determinants

To understand the causes of health and disease,


epidemiology studies:
a. Individuals
b. Families
c. Groups
d. Populations
d. Populations

When a nurse examines birth and death certificates


during an epidemiologic investigation, what data category
is being used?
a. Routinely collected data
b. Data collected for other purposes but useful for
epidemiologic research
c. Original data collected for specific epidemiologic
studies
d. Surveillance data
a. Routinely collected data

An epidemiologist wanting to know what caused severe


diarrhea and vomiting in several people at a local
banquet would be using:
a. Descriptive epidemiology
b. Analytic epidemiology
c. Distribution
d. Determinants
b. Analytic epidemiology

Which is an example of an epidemic?


a. "Bird" flu in China
b. Adult obesity in the United States
c. An isolated case of smallpox in Africa
d. The nursing shortage in the United States
b. Adult obesity in the United States

John Snow is called the "father of epidemiology" because


of his work with:
a. Cholera
b. Malaria
c. Polio
d. Germ theory
a. Cholera

The interaction between an agent, a host, and the


environment is called:
a. Natural history of disease
b. Risk
c. Web of causality
d. The epidemiologic triangle
d. The epidemiologic triangle

Public health professionals refer to three levels of


prevention as tied to specific stages in the:
a. Epidemiologic triangle
b. Web of causation
c. Natural history of disease
d. Surveillance process
c. Natural history of disease

Which is an example of an agent in the epidemiologic


triangle?
a. Human population distribution
b. Salmonella
c. Genetic susceptibility
d. Climate
b. Salmonella

When studying chronic disease, the multifactorial etiology


of illness is considered. What does this imply?
a. Genetics and molecular structure of disease is
paramount.
b. Single organisms that cause a disease, such as cholera,
must be studied in more detail.
c. Focus should be on the factors or combinations and
levels of factors contributing to disease.
d. The recent rise in infectious disease is the main focus.
c. Focus should be on the factors or combinations and
levels of factors contributing to disease.

Immunization for measles is an example of:


a. Primary prevention
b. Secondary prevention
c. Tertiary prevention
d. Health promotion
a. Primary prevention

Screening for hearing defects is an example of:


a. Primary prevention
b. Secondary prevention
c. Tertiary prevention
d. Health promotion
b. Secondary prevention

Vocational rehabilitation of a person with a neuromuscular


disease is an example of:
a. Primary prevention
b. Secondary prevention
c. Tertiary prevention
d. Health promotion
c. Tertiary prevention

An example of primary prevention is:


a. Pap smear
b. Blood pressure screening
c. Diet and exercise
d. Physical therapy
c. Diet and exercise

An example of secondary prevention is:


a. Rehabilitation
b. Avoidance of high-risk behaviors
c. Immunization
d. Mammogram
d. Mammogram

An example of tertiary prevention is:


a. Rehabilitative job training
b. Parenting education
c. Testicular self-examination
d. Family counseling
a. Rehabilitative job training

The probability an event will occur within a specified


period of time is called:
a. Rate

b. Risk
c. Epidemiology
d. Epidemic
b. Risk

Which statement is true about mortality rates? Mortality


rates:
a. Are informative only for fatal diseases
b. Provide information about existing disease in the
population
c. Are calculated using a population estimate at year-end
d. Reveal the risk of getting a particular disease
a. Are informative only for fatal diseases

A screening for diabetes revealed 20 previously


diagnosed diabetics and 10 probable new cases, which
were later confirmed, for a total of 30 cases. This is
called:
a. Prevalence
b. Incidence
c. Attack
d. Morbidity
a. Prevalence

An example of an attack rate is the:


a. Number of cases of cancer recorded at a medical
center
b. Number of people who died of cholera in India in a
given year
c. Number of beef cattle inoculated against mad-cow
disease on a farm
d. Proportion of people becoming ill after eating at a fastfood restaurant
d. Proportion of people becoming ill after eating at a fastfood restaurant

Which would be considered a serious epidemic of


influenza?
a. 50 cases
b. 100 cases
c. 500 cases
d. Unable to determine
d. Unable to determine

The most important predictor of overall mortality is:


a. Race

b. Age
c. Gender
d. Income
b. Age

An outbreak of a gastrointestinal illness from a food-borne


pathogen is an example of a(n):
a. Attack rate
b. Point epidemic
c. Secular trend
d. Event-related cluster
b. Point epidemic

A study that uses information on current health status,


personal characteristics, and potential risk factors or
exposures all at once is called:
a. Cross-sectional
b. Ecological
c. Case-control
d. Cohort
a. Cross-sectional

The type of epidemiologic study that is used to describe a


group of persons enrolled in a study who share some
characteristic of interest and who are followed over a
period of time to observe some health outcome is a(n):
a. Case control study
b. Cross-sectional study
c. Cohort study
d. Experimental study
c. Cohort study

Voters have recently decided to have fluoride added to


the city water system. Epidemiologists wanting to study
the effect of fluoride on dental caries would be
conducting a(n):
a. Ecological study
b. Double-blind study
c. Community trial
d. Screening
c. Community trial

The proportion of persons with positive test results who


actually have a disease, interpreted as the probability
that an individual with a positive test result has the
disease, is the:

a. Sensitivity
b. Specificity
c. Positive predictive value
d. Negative predictive value
c. Positive predictive value

A nurse is told that a screening test has high specificity.


This means that the test:
a. Provides precise and consistent readings
b. Accurately identifies those with the condition or trait
c. Accurately identifies those without the trait
d. Has a high level of false positives
c. Accurately identifies those without the trait

In which situation does the nurse need to be aware of the


potential for selection bias?
a. Determining the population to be studied
b. Considering how the participants will enter the study
c. Studying cause and effect relationships
d. Documenting results of the study
b. Considering how the participants will enter the study

As a result of an outbreak of influenza in a community, a


nurse encourages members of the community to receive
the influenza vaccine. Which level of prevention is being
used?
a. Primary prevention
b. Secondary prevention
c. Tertiary prevention
d. Multifactorial prevention
a. Primary prevention

Epidemiologic studies of diseases conducted by nurses


during the twentieth century were influenced by the
(select all that apply):
a. Increasing rate of poverty
b. Declining child mortality rates
c. Overcrowding in major cities
d. Development of new vaccinations
b. Declining child mortality rates
d. Development of new vaccinations

Nurses incorporate epidemiology into their practice and


function in epidemiologic roles through (select all that
apply):

a. Policy making and enforcement


b. Collection, reporting, analysis, and interpretation of
data
c. Environmental risk communication
d. Documentation on patient charts and records
b. Collection, reporting, analysis, and interpretation of data
c. Environmental risk communication
d. Documentation on patient charts and records
community
a group of people sharing common interests, needs,
resources, and environment; an interrelating and
interacting group of people with shared needs and
interests.

geopolitical community
a type of community that is formed by human or natural
boundaries.
a specific area possessing geographic boundaries and
sharing the same governing structure.
*could also include people who work within the community

phenomenological community
a type of community that is formed because its members
have a common interest, experience or a feeling of
belonging.
boundaries are less well defined, members decide what is
meaningful to them

3 critical attributes of a community


people
place
function/interaction pattern

communities of solution
this type of community is formed by an aggregate
specifically to address health concerns within a particular
area. It is composed of persons not only from the area of
need but also members of neighboring communities with a
vested interest.

3 characteristics of Community health

1. Status dimension (physical, social, emotional)


2. Structural dimension (services and resources)
3. Process dimension (effective community function)

Community health
is a synthesis of the status (what do they look like),
structure (are there services to deal with the issues),
process (how effective are they- do they require outside
help?)

4 healthy people 2020 goals


1. Attain high-quality, longer lives free of preventable
disease, disability, injury, and premature death.
2. Achieve health equity, eliminate disparities, and improve
the health of all groups.
3. Create social and physical environments that promote
good health for all.
4. Promote quality of life, healthy development, and
healthy behaviors across all life stages

epidemiological community assessment model


-this community assessment model has the focus of
Community health.
-This model examines factors that affect the health of the
community such as: Biological, environmental,
lifestyle/consumption/recreation patterns, access to the
health care system
-this model looks at morbidity and mortality data
-this model guides the development of programs targeted
at specific aggregates
(particularly important are at-risk or vulnerable
populations)

2 main reasons for community assessment


-to gain information and clarify the need for change
-to empower those responsible in the context of that
change

community as partner framework


-this framework for community assessment views people as
central members of the community
-the parts of the community that interact with members of
the community:
physical environment, health and social service, economy,
transportation and safety, politics and government,
communication, education, recreation

(wheel representing a core surrounded by the 8


subsystems listed above)

coalition
a group of consumers, health professionals, policy makers,
and others working together to improve community health
status or to solve a specific community health problem

stakeholder
an individual, organization, or group that has an interest in
a specific community health issue or the outcome of a
community-level intervention

key informant
person knowledgeable about specific aspects of a problem
and the communities current and past attempts to address
it

developmental community assessment model


-this type of community assessment gathers information
within the context of the community over history.
-this type of assessment examines issues such as:
population aging, accessibility of community resources,
changes in government structure as the community grows
or shrinks
-this type of assessment analyzes trends or historic events
over time to assess the relative health of the community

Community assessment model asset based approach


-this type of assessment model looks at the components of
a community including their interaction with each other
(within) and interaction with the environment (external)
-community is dynamic, ever adapting system
-incorporates multiple frameworks and includes
components of the other assessment models
simultaneously
-empowers the community; looks at the communities
strengths and then what is needed or not actualized

tools for collecting data


-self
-data sources (national, state, local)
-surveys (windshield, health/risk assessment)
-interviews with key informants

quantitative
numbers, statistical data

qualitative
perception, beliefs, values, attitudes, health concerns

data sources
statistics, surveys, key informants, libraries, telephone
books, drive-through, churches, school systems,
government offices, public safety depts., maps,
professional, business, voluntary and social organizations

assessment issues
gaining entrance/acceptance
trust
confidentiality

planning for community health


-problem analysis and prioritization
-establishing goals and objectives
-identifying intervention activities

Health planning
problems identified, priorities selected, objectives set for
the development of community health programs based on
findings of community health assessments and health
surveillance data

current national/international health plans


include providing health promotion, disease prevention at
population level, addressing social determinants of health
and achieving health equity

levels of health planning


impetus of health planning and program development may
come from the decision at local, state, regional or global
level

bottom-up approach to health planning


impetus for health planning comes from the local/state
level to address a health problem

top-down approach to health planning

impetus for health planning and program development


comes from a regional, national or global initiative to
improve health status

ongoing health planning


immunizations are an example of this type of health
planning

episodic health planning


outbreak of pertussis
bullying
-these are examples of this type of health planning
(fix it and move on)

social ecological model in health planning


-this model of health planning is based on general systems
theory and health promotion theory
-on the premise that multiple determinants of health
interact at different levels to affect the health status of
individual people, population aggregates or communities
-this type of health planning takes into consideration how
many factors (social, political, cultural) influence health
behaviors and health status within a community

multilevel interventions for health planning


-interventions are made at more than one system level
-upstream at the societal, environmental or policy level
-maintstream is at the population or community level
-downtstream is at the individual level

change theory for health planning


unfreezing, changing, refreezing
-sustainability must always be a concern when planning
changes in health beliefs, knowledge, behavior or social
conditions

logic model for health planning


-this model for a community health plan is a visual
representation of the logic behind the operation of the
program
-who will receive the services, what will be done, when will
it happen, where will it happen, why will it happen

SMART objectives

Specific
measurable
achievable
relevant
time bound

Health planning sequence


1. involve community members
2. select the planning group
3. formulate philosophy, goals, forecast
4. evaluate alternative solutions
5. select a solution to fit the community
6. identify resources
7. develop an action plan

steps in health planning program


Assessment
Analysis
Planning
Implementation
Evaluation

Nurse's role in program planning


-collaborator
-coordinator
-facilitator

Community organization models


-community development model
-social planning model
-social action model

community development model


this community organization model would be when you
develop a program with the community
-grassroots level

social planning model


this community organization model would be used when
there is a problem that needs to be addressed

social action model


this community organization model is not often used in
nursing. It is often in emergency situations where

authorities take over, it is highly organized and it is


required.

evaluating community health planning


These categories of questions are answered:
-outcome attainment
-appropriateness of care
-adequacy of care
-relationship of resources to results
-process

community-based nursing
-minor acute and chronic care that is comprehensive and
coordinated where people live, work or attend school
-illness care provided outside of acute care setting
-focus on "illness care"
-interacts 1 to 1
-levels of prevention : 2, 3

community-oriented nursing
-focus on "health care"
-Preserve, Promote, Protect
-interacts with groups and 1 to 1
-levels of prevention :1, 2, 3
-uses systematic processes to deliver care

social determinants of health


social conditions in which people live and work

health care disparities


gaps in health care experienced by one population
compared with another

examples of community health nursing


Public health nurse
school nurse
board of health nurse
community based clinic nurses
home care/visiting nurses
hospice nurses
occupational health/employee health
homeless shelter

public health nursing

population based practice, defined as a synthesis of


nursing and public health in the context of preventing
disease and disability and promoting and protecting the
health of the entire community

public health responsibilities


communicable disease prevention
occupational health
environmental health programs
reproductive health
chronic disease prevention
injury prevention
-newly emerging issues of preventing bioterrorism and
violence, handling disposing of hazardous wastes

principles of public health


1. focus on the aggregate
2. promote prevention
3. encourage community organization
4. ethical theory of the greater good
5. leadership in health
6. use of epidemiologic knowledge and methods

levels of prevention
Primary- prevents disease
Secondary-controls outbreak
Tertiary-limits disability

primary prevention
promotes health
prevents occurrence of disease
(education, vaccinations)

secondary prevention
early diagnosis and interventions
contains/controls outbreak
(mammograms, hearing and vision screening, flu shots)

tertiary prevention
rehabilitation and limiting disability
(cardiac rehab, management of chronic conditions)

goals of community health nursing


care of ill, disabled in a non-institutional setting
-support of well being throughout the life cycle

-promote human relatedness and mutual caring


-promote self respect regarding health and well being
-promote safety in environment

Florence Nightingale
formed a team of nurses that assisted soldiers during the
Crimean War and statistically documented her successes
saving lives through prevention of infections and improving
environmental conditions
-also she opened the first school of nursing

Lillian Wald
She was the founder of public health nursing
Founded the Henry street settlement house

Mary Breckinridge
Frontier Nursing- developed a system of rural healthcare in
remote regions.
Traveled on horseback
opened school for frontier nursing and midwifery

Clara Barton
Red Cross

#1 goal of community health nursing


Health Promotion

purpose of health promotion


-Enable/facilitate optimal health
-decrease risk
-early problem identification
-minimize disability

Definition of health
maximal development- physical, social, emotional
positive interaction with physical environment
effective role performance
absence of disease

4 factors that influence health


1. Biology
2. Physical environment
3. Sociocultural environment
4. Health care access

Primary prevention
maximizing health and wellness through strategies that are
set in place before illness or injury is present

Secondary prevention
maximizing health and wellness through strategies that are
set in place at the early and active chronic stages of
pathogenesis of illness and injury
-interventions designed to diagnose and treat illness early
and restore to health; includes screenings

Tertiary Prevention
maximizing health and wellness through strategies that are
set in place at the palliation and end-stage of disease and
injury trajectories
-rehabilitative/restorative; limites disability

behavior change models


models that assist clients, groups, and communities to
redirect activities toward health and wellness

motivational interviewing
client-centered communication style for eliciting behavior
change by helping clients and groups explore and resolve
ambivalence to change

7 behavior change models


1. learning model
2. health belief model
3. transtheoretical model
4. theory of reasoned action
5. social learning
6. the relapse prevention model
7. ecological model

primary prevention
use of seat belts, hand washing, proper preparation of food,
exercise, balanced nutrition, vaccinations

secondary prevention
women diagnosed with HIV/AIDS
student with scoliosis

learning model for behavior change


a behavior change model emphasizing reinforcement of
social competence, problem solving, autonomy, and sense
of purpose.
-incremental increases are made as a pattern of behavior is
shaped
-uses reinforcement

Health belief model for behavior change


a behavior change model that considers the severity of the
potential illness or physical challenge, the level of
conceivable susceptibility, the benefits of taking
preventative action and the challenges that may be faced
in taking action toward the goal of health promotion.
-belief that the cue messages can be accomplished

transtheoretical model for behavior change


1. precontemplation
2. contemplation
3. preparation
4. action
5. maintenance
6. relapse

theory of reasoned action for behavior change


a behavior model that emphasizes that individual
performance of a given behavior is primarily determined by
a person's intention to perform that behavior
-person's attitude toward the behavior (beliefs about the
outcomes and value of outcomes)

Social learning model for behavior change


a behavior change model that considers environmental
influences, personal factors, and behavior as key
components to change

Relapse prevention model for behavior change


behavior change model that identifies that relapse often
happens because of:
-negative emotional states
-lack of, or limited coping skills
-decreased motivation
-stress
-high risk experiences

ecological model for behavior change

belief that all processes occurring within individual people


and their environment should be viewed as interdependent
-belief that behavior is a result of the knowledge, values,
and beliefs of people as well as many social influences

ontogenic system influences


personal factors
-attitudes and beliefs that influence decisions

microsystem influences
relationship between women and the environment
-relationship stage, intimacy, communication patterns

exosystem influences
formal and informal social structures
-socioeconomic and employment status, degree of social
isolation, social networks and supports, poverty

macrocultural influences
values and beliefs of culture
-social norms, attitudes about gender roles, race

theories of learning
behavioral
cognitive
humanistic
developmental
social learning

behavioral theory of learning


operant conditioning
-target behavior change in response to reinforcement

cognitive theory of learning


information organization, recognition and retrieval
-change in response to changing patterns of thought

humanistic theory of learning


interior motivation for learning
-change response related to feelings, emotions and
relationships

developmental theory of learning

readiness to learn, congruence with developmental stage


-change in learning which occurs across the lifespan

social learning theory


outcome expectations, desired outcomes
-change comes from wanting and believing change is
possible

Domains of learning
-Affective
-Psychomotor
-Cognitive

affective domain of learning


outcome is change in attitude or value; new behaviors
reflect attitude change
- learner will value exercise because it improves sense of
well being

psychomotor domain of learning


improve hands on skills; practice dependent; skill
dependent
-learner will demonstrate how to prepare a nutritious meal

cognitive domain of learning


recall, comprehension, ability to use information learned in
a different way
-learner will describe most important risk behaviors
associated with cardiac events

follower
seeks and follows others direction/authority

gatekeeper
controls/limits outsiders access
ensures all members get an equal voice

leader
guides and directs

peacemaker/conflict manager
facilitate harmony and compromise

task specialists

focuses and directs the group toward accomplishments


(initiator, contributor, information seeker and giver,
summarizer, encourager, elaborator, coordinator)

4 purposes of health education


1. assist clients and populations to make health-related
decisions
2. increases awareness of resources and assists with
resource allocation
3. informs the public about health issues
4. improves personal health
The factors, exposures, characteristics, and behaviors
that determine patterns of disease are described using:
a. Descriptive epidemiology
b. Analytic epidemiology
c. Distribution
d. Determinants
d. Determinants

To understand the causes of health and disease,


epidemiology studies:
a. Individuals
b. Families
c. Groups
d. Populations
d. Populations

When a nurse examines birth and death certificates


during an epidemiologic investigation, what data category
is being used?
a. Routinely collected data
b. Data collected for other purposes but useful for
epidemiologic research
c. Original data collected for specific epidemiologic
studies
d. Surveillance data
a. Routinely collected data

An epidemiologist wanting to know what caused severe


diarrhea and vomiting in several people at a local
banquet would be using:
a. Descriptive epidemiology
b. Analytic epidemiology
c. Distribution
d. Determinants

b. Analytic epidemiology

Which is an example of an epidemic?


a. "Bird" flu in China
b. Adult obesity in the United States
c. An isolated case of smallpox in Africa
d. The nursing shortage in the United States
b. Adult obesity in the United States

John Snow is called the "father of epidemiology" because


of his work with:
a. Cholera
b. Malaria
c. Polio
d. Germ theory
a. Cholera

The interaction between an agent, a host, and the


environment is called:
a. Natural history of disease
b. Risk
c. Web of causality
d. The epidemiologic triangle
d. The epidemiologic triangle

Public health professionals refer to three levels of


prevention as tied to specific stages in the:
a. Epidemiologic triangle
b. Web of causation
c. Natural history of disease
d. Surveillance process
c. Natural history of disease

Which is an example of an agent in the epidemiologic


triangle?
a. Human population distribution
b. Salmonella
c. Genetic susceptibility
d. Climate
b. Salmonella

When studying chronic disease, the multifactorial etiology


of illness is considered. What does this imply?
a. Genetics and molecular structure of disease is
paramount.
b. Single organisms that cause a disease, such as cholera,

must be studied in more detail.


c. Focus should be on the factors or combinations and
levels of factors contributing to disease.
d. The recent rise in infectious disease is the main focus.
c. Focus should be on the factors or combinations and
levels of factors contributing to disease.

Immunization for measles is an example of:


a. Primary prevention
b. Secondary prevention
c. Tertiary prevention
d. Health promotion
a. Primary prevention

Screening for hearing defects is an example of:


a. Primary prevention
b. Secondary prevention
c. Tertiary prevention
d. Health promotion
b. Secondary prevention

Vocational rehabilitation of a person with a neuromuscular


disease is an example of:
a. Primary prevention
b. Secondary prevention
c. Tertiary prevention
d. Health promotion
c. Tertiary prevention

An example of primary prevention is:


a. Pap smear
b. Blood pressure screening
c. Diet and exercise
d. Physical therapy
c. Diet and exercise

An example of secondary prevention is:


a. Rehabilitation
b. Avoidance of high-risk behaviors
c. Immunization
d. Mammogram
d. Mammogram

An example of tertiary prevention is:


a. Rehabilitative job training
b. Parenting education

c. Testicular self-examination
d. Family counseling
a. Rehabilitative job training

The probability an event will occur within a specified


period of time is called:
a. Rate
b. Risk
c. Epidemiology
d. Epidemic
b. Risk

Which statement is true about mortality rates? Mortality


rates:
a. Are informative only for fatal diseases
b. Provide information about existing disease in the
population
c. Are calculated using a population estimate at year-end
d. Reveal the risk of getting a particular disease
a. Are informative only for fatal diseases

A screening for diabetes revealed 20 previously


diagnosed diabetics and 10 probable new cases, which
were later confirmed, for a total of 30 cases. This is
called:
a. Prevalence
b. Incidence
c. Attack
d. Morbidity
a. Prevalence

An example of an attack rate is the:


a. Number of cases of cancer recorded at a medical
center
b. Number of people who died of cholera in India in a
given year
c. Number of beef cattle inoculated against mad-cow
disease on a farm
d. Proportion of people becoming ill after eating at a fastfood restaurant
d. Proportion of people becoming ill after eating at a fastfood restaurant

Which would be considered a serious epidemic of


influenza?
a. 50 cases

b. 100 cases
c. 500 cases
d. Unable to determine
d. Unable to determine

The most important predictor of overall mortality is:


a. Race
b. Age
c. Gender
d. Income
b. Age

An outbreak of a gastrointestinal illness from a food-borne


pathogen is an example of a(n):
a. Attack rate
b. Point epidemic
c. Secular trend
d. Event-related cluster
b. Point epidemic

A study that uses information on current health status,


personal characteristics, and potential risk factors or
exposures all at once is called:
a. Cross-sectional
b. Ecological
c. Case-control
d. Cohort
a. Cross-sectional

The type of epidemiologic study that is used to describe a


group of persons enrolled in a study who share some
characteristic of interest and who are followed over a
period of time to observe some health outcome is a(n):
a. Case control study
b. Cross-sectional study
c. Cohort study
d. Experimental study
c. Cohort study

Voters have recently decided to have fluoride added to


the city water system. Epidemiologists wanting to study
the effect of fluoride on dental caries would be
conducting a(n):
a. Ecological study
b. Double-blind study

c. Community trial
d. Screening
c. Community trial

The proportion of persons with positive test results who


actually have a disease, interpreted as the probability
that an individual with a positive test result has the
disease, is the:
a. Sensitivity
b. Specificity
c. Positive predictive value
d. Negative predictive value
c. Positive predictive value

A nurse is told that a screening test has high specificity.


This means that the test:
a. Provides precise and consistent readings
b. Accurately identifies those with the condition or trait
c. Accurately identifies those without the trait
d. Has a high level of false positives
c. Accurately identifies those without the trait

In which situation does the nurse need to be aware of the


potential for selection bias?
a. Determining the population to be studied
b. Considering how the participants will enter the study
c. Studying cause and effect relationships
d. Documenting results of the study
b. Considering how the participants will enter the study

As a result of an outbreak of influenza in a community, a


nurse encourages members of the community to receive
the influenza vaccine. Which level of prevention is being
used?
a. Primary prevention
b. Secondary prevention
c. Tertiary prevention
d. Multifactorial prevention
a. Primary prevention

Epidemiologic studies of diseases conducted by nurses


during the twentieth century were influenced by the
(select all that apply):
a. Increasing rate of poverty
b. Declining child mortality rates

c. Overcrowding in major cities


d. Development of new vaccinations
b. Declining child mortality rates
d. Development of new vaccinations

Nurses incorporate epidemiology into their practice and


function in epidemiologic roles through (select all that
apply):
a. Policy making and enforcement
b. Collection, reporting, analysis, and interpretation of
data
c. Environmental risk communication
d. Documentation on patient charts and records
b. Collection, reporting, analysis, and interpretation of data
c. Environmental risk communication
d. Documentation on patient charts and records
Which of the following statments best decribes the tterm
epidemiology?
a. applied science that uses surveys to track
communicable diseases(spread from person to person
b. applied science tht uses research and statistics to find
out about how disease affects populations and the
reasons disease occurs
c. applied science whose sole function is to find the
etiology of all diseases
d. applied science that uses correlation studies to
determine the appropriate tertiary prevention strategies
for a community
B

Communiity health is made up of which of the following?


A. epidemiology, parasitology, and correlation studies
B. etiology, studies, prevention, and demographics
C. epidemiology, human ecology, and demography
D. infectious agents, natural hisory of a disease, and
demography
C

Which ofs the following factors can contribute to a person


being a host?
A. exposure to physical, chemical, or or nutritive agents
B. being young or old

C. poverty
D. all above
D

Which of the following statments about the natural


history of a disease is correct
A. it has nothing to do with epidemiological sudies
B. using this method works only for communicable
diseases
C. following the course the disease would take if there
were no interventions can lead to devel strategies or
therapeutic
D. none above
C

A community health RN is participating in an analytic


study of ppl who are at risk for devel diabetes. Studies of
this type are classified
A. retropective
B. concurrent
C. cross-sectional
D. prospective
D

A community health RN is particiipating in an analytic


study of ppl who devel melanoma following freq visits to
tanning salons. Studies of this type are classified
A. retropective
B. concurrent
C. cross-sectional
D. prospective
A

Which of the following characteristics apply to


intervention studies?
A. they don't require altering the behavior of study
subjects, only observing them
B. they include prevention or therapeutic trials
C. They follow the group being studied into the future to
determine risk factors
D. They are the same as correlation studies
B

Which statement most accurately reflects prevalencerate


and incidence rate?
A. there is no difference: they mean the same thing

B. prevalence rate indicates the rate of change from ppl


who do not have the disease, to their having it
C. both cover unspecified, unlimited periods of time
D. incidence rate reflects new cases of a disease during a
specified time
D

Which of the following are considered key vital statistics


A. attack rate and morbidity rate
B. incidence and prevalence rate
C. maternal mortality rate and fetal death rate
D. relative risk
C

Which of the following statements best describes a


community
A. pl living in a particular geographic location
B. organizations, family groups, or friend groups or friend
groups that interact
C. grps that have common interest or needs
D. all above
D

A community of interest could be described as:


A. ppl who live in the same geographic area and share
common interest
B. ppl who share beliefs, values or interest but not
geographic location
C. ppl with the ability to iddentify their own needs
D. ppl who have athe same life expectancy
B

A community that is described as having community


competence has which of the following characteristics
A. the ability to perform their won cross-sectional
epidemiological studies
B. the ability to delegate any community processes to an
outside expert such as a community health nurse
C. the ability to identify their needs, achieve consensus,
and plan and implement goals
D. the ability to predict morbidity and mortality rates for
the population or geographic area
C

The key role for the community health RN in dealing with


communities is to:

A. make sure ppl in the community are empowered and


able to participate
B. provide incentives for community members to follow
the protocols
C. establish project teams that will collect and analyze
data
D. closely direct community members so tht the
community assessments are
A

Which of the following describes a valid way to colllect


data for a community assesment
A. using a bibrary database to conduct a lit review
B. reading government docs to find if similar data have
already been collected]
C. using surveys or questionaires to gather info form
members of communit
D. all
D

Which of the following is NOT a part of the community


assessment
A. identifying available resources such as stime, money,
and team skills
B. collecting and analyzing information
C. withholding results from the community until they can
be statisticall confirmed, to avoid alarming ppl
D. setting action priorties based on the needs of the
community and avail resources
C

a younger community member's reluctance to speak to


an older community health RN might be due to a
difference in cultural perception about
A. personal space
B. honesty
C. authority
D. perception of time
C

Cultural differences in perceptions about time might be


shown as:
A. engaging in several minutes of casual conversation
before talking with the nurse about a healh issue
B. alway arriving at appt times min earlier thatn
scheduled

C. arriving at appt min later tharn scheduled time


D. all
D

Which of the following statements about families is true


A. they are always made up of ppl who are related by
birth or adoption
B. theycurrent definition of family is much less flexible
than it was in the past
C. alternate family configuratins are increasingly common
D. never-married couples are not included wunder the
definition of family
C

A nuclear family of husband, wife, and iffediate children


plus other persons related by blod or voluntary
relationships is called
A. blended family
B. family of procreation
C family of origin
D. extended family
D

An ectramarital family configuration is characterized by"


A. both adults in the family working outside the home
B. child abuse
C. open marriage or co-primary rrelationships
D. adults who voluntarily decide not to have children
C

The family living in Anytown USA consist of 2 adults who


are cohabiting and 4 children from previous
relationshipsThe adults adhere to strict traditional gender
roles and expect the children to be respectful and
obedient to the adults in the household
A. social enciorment
B. psychological enciorment
C. community enviorment
D. dysfunction//B

A family's social enviorment consist of


A. family members communication skills
B. housing conditions inside and outside the house
C. the extended family status
D. religion, race, and socioeconomic class
C

Which of the following would be considered appropriate


parts of a family assessment by a community health
nurse
A. relationships between family members and between
the family and community
B. ability of family members to grow, help 1 another, and
help themselves
C. ability of family membbers to be flexible about roles
and responsibilities
D all
D

Which of the following would be considered a


dysfunctional coping strategy
A. making a genogram
B. triangulation
C. role flexibility
D. addressing the problem
B

a family health tree is used to track which of the following


A. mental health disorders that occur in a family
B marriage and divorce
C. health conditions with no genetic basis
D. places of residence
A

A family thtat copes with stressful situations by saying


"our family is always upbeat, no matter what happen" is
using which of the following dysfunctional coping
strategies
A. pseudomutuality
B. scapgoating
C. triangulation
D. family myth
D

which of the following is useful for showing the difference


between a family's needs and th resources that are
available to them
A. Omaha system
B. family health tree
C. ecomap
D. genogram

nutritional factors for young to middle adulthood would


include
A. control of obesity
B. avoiding eating problems due to poor-fitting dentures
C. dealing with vision loss that could affect eating
D. avoiding all takeout or prepared food
A

Which of these conditions could cause an older adult to


have nutritional problems
A. ill-fitting dentures or other mouth problems
B. poor appetite due to chronic illness
C. low vision
d. all
D

In perfroming a nutritional assessment during a home


visit, you need to be sure
A. that ppl can cook their own food
B. to ignore complaints about food allergies or dislikes
since they interfere with attaining proper nutrition
C. you have clear and complete information about what
the client eats, how much and when
D. that all clients get enough red meat to maintain their
iron levels
C

How can chronic illness impact feeding and nutrition


A. chronicall ill people comoplain a lot abou their food
B. medications for chronic illnesses can cause side effects
that interfere with eating
C. mostchronicall ill people have increased appetites,
putting them at risk for obesiey, a major health risk factor
D none
B

Which of the following would be an appropriate strategy


for preventing nutrition problems in populations
A. sending clients to the hospital for intravenous feeding
B gathering data to screen ppl at risk for nutritional
deficits
C. avoiding use of food pantries or meal delivery because
it is best for clients to cook their won food at home
D. all

10 reportable STD's in Allegheny County with in 5 days


HIV, chancroid, chlamydia trachomaatis, gonococcal
infections, granuloma inguinale, lymphogranuloma
venereum, hep B , syphillis, hep A
Change for the community as client must often occur at
several levels because:
a. Health problems caused by lifestyle are
multidimensional.
b. Most individuals can change their habits alone.
c. Aggregates are responsible for social change.
d. Geographic areas often have health risks that the
nurse must identify.
a. Health problems caused by lifestyle are
multidimensional.

A nurse cares for the community as the client when


focusing on:
a. Providing care for aggregates living in the community
b. The collective good of the population
c. The provision of care for families in the home setting
d. Providing health education in the community
b. The collective good of the population

A collection of individuals who have in common one or


more personal or environmental characteristics is the
definition of a(n):
a. Community
b. Group
c. Family
d. Aggregate
d. Aggregate

What are the critical attributes in the definition of


community?
a. Families, groups, and health organizations
b. Health needs, geographical boundaries, and target
population
c. People, place, and functions
d. Populations and health resources
c. People, place, and functions

A group of nursing students forms a student nursing


association to provide support while they are enrolled in a

nursing program. This is known as a community of:


a. Place
b. Special interest
c. Problem ecology
d. Function
b. Special interest

A public health nurse is collecting data about the


biological aspects of community health. What data will
the nurse collect?
a. Race/ethnicity
b. Traditional morbidity and mortality rates
c. Policy making and social change
d. Homicide rates
b. Traditional morbidity and mortality rates

A community that has residents who are very involved in


its activities and encourage activities that promote the
health of the community displays which condition of
community competence?
a. Commitment
b. Participation
c. Articulateness
d. Effective communication
b. Participation

Which data source provides information about the


function of the community?
a. Maps
b. Census data
c. State departments, business and labor, local library
d. Civic groups
c. State departments, business and labor, local library

Which characteristic is an indicator of community health


process?
a. Participation and community action
b. Live birth rate
c. Racial distribution
d. Socioeconomic levels
a. Participation and community action

A community that demonstrates commitment would most


likely support the development of a:
a. Community club to facilitate community involvement
b. Children's recreation program

c. Curb-side recycling program and community-based


education about recycling
d. Cooperative agreement with a neighboring city to
share needed services
c. Curb-side recycling program and community-based
education about recycling

The main characteristics of partnership are:


a. Awareness, flexibility, and distribution of power
b. Rights, responsibilities, and consensus
c. Commitment, participation, and articulation
d. Collaboration, advocacy, and utility
a. Awareness, flexibility, and distribution of power

When completing a community assessment, the nurse


will:
a. Identify community needs and clarify problems.
b. Determine the weaknesses of a community.
c. Perform the core functions of public health nursing.
d. Assess individual needs within a community.
a. Identify community needs and clarify problems.

When a nurse focuses intervention strategies on the


structural dimension of community health, they are
directed toward:
a. Health services
b. Primary prevention
c. Health promotion
d. Secondary prevention
a. Health services

Which characteristic is an indicator of community health


structure?
a. Infant mortality rate
b. Effective communication
c. Crime rate
d. Emergency room utilization
d. Emergency room utilization

After identifying the boundaries of a community, what is


the next step the nurse should take in completing a
community assessment?
a. Gather relevant existing data and generating missing
data.
b. Plan interventions that benefit the entire community.

c. Formulate nursing diagnoses.


d. Evaluate the interventions that were used.
a. Gather relevant existing data and generating missing
data

What intervention would be the most appropriate for the


nurse to use when trying to develop community
partnerships?
a. Involve only the community residents.
b. Use nurses as the source of information and leadership.
c. Rely on the power of local officials.
d. Include a variety of disciplines.
d. Include a variety of disciplines.

When writing a community-level diagnosis, "Among"


refers to the:
a. Specific problem or health risk faced by the community
b. Characteristics of the community
c. Community client experiencing the risk or problem
d. Likelihood that the problem will be solved
c. Community client experiencing the risk or problem

Which nursing diagnosis is written at the communitylevel?


a. Risk of hypertension related to poor diet and sedentary
lifestyle
b. Risk of obesity among school-age children related to
lack of opportunities to engage in physical activity
c. Risk of ineffective health maintenance among
individuals who do not have access to a primary care
provider
d. Ineffective coping related to multiple stressors as
evidenced by client crying and stating she has no support
system
b. Risk of obesity among school-age children related to lack
of opportunities to engage in physical activity

A nurse surveys the school nurses in a community to


determine their roles in the schools because this data is
not available. The nurse is engaging in the process of
data:
a. Collection
b. Gathering
c. Generation
d. Interpretation
c. Generation

A nurse schedules an appointment with a physician who


has a practice in the community to learn more about the
community's beliefs regarding childhood immunizations.
This is known as:
a. Informant interview
b. Participant observation
c. Secondary analysis
d. Windshield survey
a. Informant interview

When conducting informant interviews in a small


community, the nurse would most likely contact:
a. The state department of health for death records
b. A local priest for congregation information
c. Surrounding communities for crime comparison
d. The Centers for Disease Control and Prevention (CDC)
for illnesses in the area
b. A local priest for congregation information

A nurse collects data about seat belt usage by


interviewing key informants and observing behaviors in
the community. What type of data is being collected?
a. Quantitative
b. Qualitative
c. Focus-groups
d. Survey
b. Qualitative

A nurse who reads the local community newspaper is


using which method to gather data about the community?
a. Informant interview
b. Survey
c. Participant observation
d. Windshield survey
c. Participant observation

Which method of data collection is being used as a nurse


gathers information about the condition of homes, size of
lots, neighborhood hangouts, road conditions, and modes
of transportation?
a. Participant observation
b. Windshield survey
c. Survey
d. Informant interviews
b. Windshield survey

A nurse who uses such data as minutes from a


community meeting is using:
a. Secondary analysis
b. Informant interviews
c. A survey
d. A windshield survey
a. Secondary analysis

A nurse plans on implementing a community-wide


influenza immunization program. Which factor should the
nurse consider when implementing this program?
a. The number of community members who have already
received the immunization
b. The existence of formal groups in the community
c. Public policy that mandates influenza immunization for
certain populations
d. The community's readiness to participate in the
program
d. The community's readiness to participate in the program

To effectively complete the evaluation phase, the nurse


must have:
a. Demonstrated the ability to improve the health of the
participants
b. Developed measureable objectives and goals before
implementation
c. Encouraged full participation by community members
d. Improved the health of the population through the
program
b. Developed measureable objectives and goals before
implementation

To analyze the problem of increased adolescent alcohol


use in the community, a nurse brings together several
groups of people to address the issue. These groups come
together at the same time to work on identifying potential
causes of the problem, but do not interact during the
process. Which model is the nurse applying?
a. Delphi technique
b. Program planning model
c. Community-as-partner model
d. Community empowerment model
b. Program planning model

A nurse is trying to increase participation in a free


colorectal screening program for middle-aged adults who
lack health insurance. Which implementation mechanism
would be most effective?
a. Small interacting groups
b. Health policy
c. Lay advisors
d. Mass media
d. Mass media

A nurse may define a community as a(n) (select all that


apply):
a. Social group determined by geographic boundaries
b. Group of people who share common values and
interests
c. Group of people defined by their interactions
d. Individual with a specific health concern
a. Social group determined by geographic boundaries
b. Group of people who share common values and interests

Which data would be useful for a nurse to collect when


assessing safety in a community? Select all that apply.
a. Number of billboards in the area
b. Interviews with health care providers who are familiar
with the community
c. Observation of community members
d. Nurse's own observations
b. Interviews with health care providers who are familiar
with the community
c. Observation of community members
d. Nurse's own observations
An enduring process in which a manager establishes
systems and monitors the health status, resources, and
outcomes for a targeted aggregate of the population is
called:
a. Case management
b. Care management
c. Advocacy
d. Continuity of care
b. Care management

When would a nurse use a critical path?


a. When implementing the five "rights" of case

management
b. When demonstrating competencies required for
practicing case management
c. When attempting to achieve a measurable outcome for
a specific client
d. When effectively managing conflict resolution
c. When attempting to achieve a measurable outcome for a
specific client

A patient with a long history of asthma with many


hospital admissions is referred to a case manager to
assist in the management of her disease. Disease
management targets:
a. Chronic and costly disease conditions that require longterm care interventions
b. Patients who cannot handle their disease
c. Those who seek to control use by providing clients with
correct information
d. Patients who will need an advanced practice nurse
instead of physician for monitoring
a. Chronic and costly disease conditions that require longterm care interventions

Which type of management would be most appropriate


for a nurse to use when helping clients improve their
health status?
a. Utilization management
b. Case management
c. Disease management
d. Demand management
d. Demand management

Why must a nurse who is using population management


be able to work with integrated care delivery systems?
a. Management has shifted from inpatient care to primary
care providers as points of entry.
b. Emphasis is on episodic illness care for individuals
rather than on population management.
c. Care management services and programs do not
provide access and accountability, as provided by case
management services.
d. Assessment of the needs of the population is no longer
necessary.
a. Management has shifted from inpatient care to primary
care providers as points of entry.

Which activity would a nurse engage in when using


population management?
a. Census taking to determine the total number of people
in the population
b. Assessing the needs of the client population through
compilation of health histories
c. Providing case management services for every citizen
in the community
d. Selecting programs for wellness that are repeated
annually
b. Assessing the needs of the client population through
compilation of health histories

What are the five "rights" of case management?


a. Care, time, provider, setting, price
b. Patient, medication, route, time, documentation
c. Place, setting, patient, plan, outcomes
d. Disease process, time, place, beneficence, advocate
a. Care, time, provider, setting, price

Which model addresses the structure and processes of


using the population-based tools of disease management
and critical pathways to offer care for client populations?
a. Client-focused
b. System-focused
c. Social service
d. Long-term care
b. System-focused

Which are examples of knowledge domains used in case


management?
a. Legal issues, malpractice recognition, and community
involvement
b. Teaching, counseling, and education skills
c. Advocacy, political campaigning and legislative change
d. Grant application, bargaining contracts, and securing
funding
b. Teaching, counseling, and education skills

Which case management activity would most likely be


implemented by a bachelor's prepared nurse rather than
an advanced practice nurse (APN)?
a. Working with community aggregates
b. Working with systems of disease
c. Working with individuals
d. Working with outcomes management processes

c. Working with individuals

The case manager who uses effective collaboration and


team strategies to make arrangements for services is
performing the role of:
a. Broker
b. Negotiator
c. Liaison
d. Facilitator
b. Negotiator

When working with a client, when would it be most


appropriate for a nurse to use life care planning?
a. When organizing a timeline of life events
b. When documenting client information and requests
c. When assessing present and future client needs
d. When estimating future costs for medical care
c. When assessing present and future client needs

When case managers provide a formal communication


link among all parties concerning the plan of care
management, they are assuming the role of:
a. Facilitator
b. Liaison
c. Coordinator
d. Negotiator
b. Liaison

Which nursing intervention does the case manager


implement when fulfilling the role of coordinator?
a. Providing information to all parties about the situations
affecting the client
b. Educating the client and providers in order to make
informed decisions
c. Supporting all parties to work toward mutual goals
d. Arranging, regulating, and balancing needed health
services for the client
d. Arranging, regulating, and balancing needed health
services for the client

Which are examples of case-managed conditions?


a. Bankruptcy, financial distress, depression
b. Flu, colds, frequent headaches
c. Malaria, bird flu, Dengue fever
d. AIDS, spinal cord injury, ventilator dependency
d. AIDS, spinal cord injury, ventilator dependency

The public health nurse working with the chronically ill


and families with young children needing age-specific
health maintenance is using which model of case
management?
a. Client-focused
b. System-focused
c. Social service
d. Long-term care
a. Client-focused

In case management, the process of performing


interdisciplinary, family, and client conferences would
occur during which phase of the nursing process?
a. Assessment
b. Diagnosis
c. Planning/outcome
d. Implementation
b. Diagnosis

When the case manager contacts providers and


negotiates services and price, which phase of the nursing
process is used?
a. Assessment
b. Diagnosis
c. Planning/outcome
d. Implementation
d. Implementation

A case manager supports a client's decision to return


home after having a total hip replacement rather than go
to a skilled nursing facility. Which phase of the nursing
process is being used?
a. Assessment
b. Planning/outcome
c. Implementation
d. Evaluation
d. Evaluation

The process of moving conflicting parties toward an


outcome is called:
a. Negotiation
b. Conflict management
c. Problem-purpose expansion method
d. Brainstorming
a. Negotiation

The first stage of collaboration is:


a. Trust building
b. Consensus
c. Collegiality
d. Awareness
d. Awareness

Mutual benefit with limited loss for everyone is a goal of:


a. Negotiating
b. Assertiveness
c. Conflict management
d. Cooperation
c. Conflict management

The nurse and client, a 20-year-old expectant mother, are


discussing the advantages and disadvantages of
breastfeeding. To enhance understanding of the
implications of breastfeeding, the nurse says, "Tell me
more about how you will work full time and breastfeed."
Asking this question is an example of which aspect of the
advocacy process?
a. Verification
b. Clarification
c. Amplification
d. Affirmation
b. Clarification

Upholding a client's right to make a choice and to act on


the choice involves:
a. Affirming
b. Supporting
c. Informing
d. Decision making
b. Supporting

An individual who pursues neither his or her concerns nor


another's concerns is using which conflict management
behavior?
a. Accommodating
b. Collaborating
c. Avoiding
d. Compromising
c. Avoiding

The nurse is involved in a conflict resolution situation with


the parents of a 2-year-old boy. The parents are deciding
if "spanking" the child is a disciplinary method that they
will employ. The mother says, "I do not believe in
spanking. I see it as abusive and demeaning." The
mother's statement is an example of:
a. Cooperation
b. Assertiveness
c. Bargaining
d. Collaboration
b. Assertiveness

Which principle of justice is influenced when excessive


attention to cost containment impairs the nurse's duty to
provide measures to improve health?
a. Autonomy
b. Beneficence
c. Confidentiality
d. Justice
b. Beneficence

Which ethical principle is influenced when care in a


managed system is provided by less-experienced
providers?
a. Autonomy
b. Beneficence
c. Confidentiality
d. Justice
d. Justice

A case manager submits documentation that a nursing


visit was completed at a client's home, but it was never
performed. According to the general areas of legal risk,
how would this action best be categorized?
a. Liability for managing care
b. Fraud/abuse
c. Negligent referral
d. Confidentiality/security
b. Fraud/abuse

What knowledge and skills are required in order to


become a competent case manager? Select all that apply.
a. Knowledge of community resources and financing
mechanisms
b. Written and oral communication skills
c. Proficient negotiation and conflict-resolving practices

d. Application of evidence-based practices and outcomes


measurements
a. Knowledge of community resources and financing
mechanisms
b. Written and oral communication skills
c. Proficient negotiation and conflict-resolving practices
d. Application of evidence-based practices and outcomes
measurements

What action should a nurse take in order to reduce the


risk of liability? Select all that apply.
a. Provide accurate documentation of client visits.
b. Inform clients of their rights of appeal.
c. Refer clients to the providers of their choice.
d. Utilize care planning whenever possible.
Humanistic Nursing Theory
studies the existence of reality of nursing

Transcultural Nursing
comparison of differing cultures to understand their
similarities and their differences across human groups

Culture
set of values, beliefs and traditions that are held by a
specific group of people and that are handed down,
generation to generation

Leininger
1. cultural preservation or maintenance
2. cultural care accommodations or negotiation
3. cultural care re-patterning or restructuring

Nightingale's 10 Canons
1. ventilation and warming
2. light and noise
3. cleanliness of the pt area
4.health of houses
5. bed and bedding
6. personal cleanliness
7. variety
8. offering hope and advice

9. food
10. observation

Primary prevention
patient assessment and intervention to identify and reduce
possible risk factors

Secondary prevention
detection of symptoms that are a reaction to stressors,
appropriately prioritizing interventions, and treatments that
reduce the toxic effects of stressors

Health
seen as a dynamic, positive state and not just absence of
disease

5 A's of Health Care Access


availability, accessibility, affordability, acceptability, and
accommodation

Market justice
people are only entitled to what they can earn; the focus is
on individual responsibility "minimal collective action and
freedom from collective obligations except with respect to
others fundamental rights

Vulnerable populations
low income, less education, homelessness, military
veterans, immigrants, prisoners, migrant and seasonal
farm workers and their families, chronic illness, mental
illness

social determinants
are the circumstances in which people are born; grow up,
live, work, and age as well as the systems put in place to
deal with illness

Health disparities
difference in health outcomes between populations, social,
demographic and geographic

Health inequities
modifiable associate with social inequalities and are
considered unfair

Leininger- Transcultural Nursing


requires an awareness of self, sensitivity to others,
respectful dialogue

Ethnocentric
tendency to believe others think the same was as "they
do," have the same world view, culture

Altruism
putting others interests before one's own, being present is
an example

Autonomy
PHN independent and responsible for actions - respects
community and individual right to self-determination;
maintain privacy, honesty, and fidelity

Beneficence
seek to do good, not harm

Common Good Perspective


good for the whole

Ethics
standards of behavior that tell us how we ought to act in
various situations we encounter in life

Fairness Perspective
focuses on how fairly or unfairly actions affect a group how benefits and burdens are distributed; requires
consistency in the way people are treated

Non-maleficence
first do no harm - primum non nocere

Advance directive
legal document in which an individual conveys their
decisions about end-of-life care

Health literacy
includes the ability to understand instructions on
prescription drug bottles, appointment slips, medical

education brochures, doctor's directions and consent forms


and the ability to negotiate complex health care systems

Pre-crisis
planning and test messages

Initial
audience wants information NOW

Maintenance
on-going assessment of the situation

Resolution
community will not be ready to conduct risk reduction
immediately after emergency

Evaluation
Crisis and emergency risk communication response

4 P's of Social Marketing


product, price, place, promotion

NICs
Nursing interventions classification

NOCs
nursing outcomes classification

Nursing minimum data set (NMDS)


standardizes the collection of nursing data; can provide
accurate description of nursing dx, care and resources used
to help define cost and quality of nursing care

Tele-Health
electronic technologies to support clinical health care,
patient and practitioner education, public health and health
administration by long distance

"Right-to-Know" Laws
give people and communities the right to know about
possible chemical exposures where they live and work

Infancy

trust vs. mistrust

Early childhood (18 mo-3 yrs)


autonomy vs. shame

play age (3-5 yrs)


initiative vs. guilt

Adolescence (12-16yrs)
Identity vs. inferiority diffusion

Middle adult (35-55/65 yrs)


generativity vs. self-absorption/stagnation

late adulthood (55/65- death)


integrity vs. despair/wisdom

Young adult (18-35 yrs)


intimacy vs. solidarity/isolation

Sensory-motor stage
birth to two

Pre-operational stage
about two to seven

concrete operational stage


ages seven to eleven

Formal operational stage


eleven years and older

Epidemiology
distribution and determinants of health related
states/events in populations and the use of this study to
control health problems

Prevalence
measures the proportion of the population affected by a
specific condition in a specific time period

Incidence

measures the risk of developing a new condition (disease,


injury, symptom, or death), over a specific period of time,
such as a year

Mortality rate
also called death rate=an estimation of the proportion of a
population that dies during a specified time period

Epidemic
rates exceed normal or expected frequency in a given
population/community/region

Pandemic
worldwide epidemic (influenza and plague)

Chain of causation
chain of events where every event caused by the previous
event; the sequence of stops that have been identified as
linked to the disease

Web of causation
describes events that have multiple causative factors

Passive Immunity
newborns via maternal antibodies

Active immunity
acquired after exposure to a disease or via immunizations

Cross-immunity
immunity to one pathogen confers immunity to another
such as cowpox providing immunity to smallpox

Herd-immunity
present in a population, the higher the immunity decreases
risk of disease (use of immunizations); increasing
immunizations reduces risk of exposure to disease of those
not immunized

Primary prevention
used before the person gets the disease or condition to
reduce the incidence and prevalence of the disease;
includes health promotion (education) and protection
(immunization)

Secondary prevention
applied after the disease has occurred but before signs are
evident; focuses on early dx (screening) and prompt tx to
limit disability

Tertiary prevention
applied when person already has the disease, focuses on
preventing damage from the condition, slowing the process
of the disease, preventing complications, and returning the
person afflicted to their optimal level of health
(rehabilitation)

Case Series and Case Reports


collection of reports; no control group, easy to understand
and can be written quickly

Case Control Studies


compare those with the disease to those without. estimates
odds of getting the disease or condition, less reliable than
randomized control trials, works well for rare conditions

Cohort Studies
may be prospective or retrospective, used to establish
causation or evaluate the impact of treatment when
randomized control trials are not possible over time-longitudinal; not good for rare conditions, requires large
study group

Randomized Controlled studies


requires a treatment group and a control group, pts are
randomly assigned to all groups, considered "gold
standard" of medical research to determine cause and
effect

double blind method


neither patient nor researcher knows if the subject is in the
control or treatment group

meta-analysis
combines data from numerous studies and arrives at
pooled estimates of treatment effectiveness and statistical
significance

Descriptive studies
are observational studies that seek to describe phenomena

Qualitative studies
describe phenomena of interest; include participant
observation, in-depth interviews and focus groups

Quantitative studies
seeks to confirm hypotheses about phenomena; uses more
rigid style eliciting and categorizing responses

sulfur dioxide
product of industry produces acid rain and contributes to
respiratory illness

nitrous oxide
combustion product contributes to illness of lungs, immune
system and asthma

carbon monoxide
car exhaust reduces oxygen delivery at high concentrations

radon
naturally occurring radioactive gas that cannot be detected
by our senses, but is the leading cause of lung cancer in
non-smokers; it is the second cause of lung-cancer behind
smoking

consultation
involves seeking advice or an opinion of an expert; the
consultant does not solve the problems, but assists and
guides the group seeking consultation to solve their own
problems

collaboration
taking an active role in implementing the plan, this requires
understanding of each party's perspectives, on-going
communication and feedback

coalition-building
alliance or group of stakeholders with a common
interest/goal

Advocacy

purpose is policy change; critical role for the public health


nurse

Cognitive
thinking (comprehension, analysis, synthesis, application,
evaluation); evaluate the level of the learner

affective
feeling (receptive, active participation, valuation,
internalization), includes changes in attitude and values

psychomotor
acting - demonstrates performance skills

6 stages of disaster life cycle


prepare, respond, mitigate, reduce the risk, and prevent

natural disasters
due to weather or other natural occurrences

man-made disasters
intentional power outages, chemical spills, acts of bio
terrorism

bacterial agents
anthrax, plague

viral agents
smallpox

Biological toxins-category A
considered high priority and pose national security threathigh morbidity and mortality
smallpox, anthrax, plague, Ebola, Lassa fever, Q fever,
botulism, tularemia, hemorrhagic fever

Biological toxins-category B
moderate to low morbidity and mortality, difficult to detect
Bucellosis, clostridia toxin, staph enterotoxin, salmonella,
shigella, e.coli, cholera

Biological toxins-category C

emerging pathogens such as yellow fever, drug resistant


TB

Family disaster plan


encourage families to prepare a family emergency/disaster
kit, they might be deployed to local disaster sites

Health outcomes
health promotion interventions there must be outcomes to
measure analysis of proper structure and constructive
process, operationalizing, outcomes (improved health =
weight loss, lower blood sugars)

Health status & quality of life indicators


individual perception based on health, cultural background,
education, moral and ethical values, difficult to asses
because it is very subjective

Disparities
exist due to poverty, lower educational levels, unequal
access to healthcare, language barriers, environmental
conditions, racism

Health Belief Model (HBM)


predicts health related behavior and compliance to prevent
diseases; health promotion empowers populations so they
can have control over personal health and access
resources; perception, attract positive value (health) and
repel negative value (illness)

Transtheoretical Model (TTM)


five stages of progression (1) pre-contemplation- no
thought of change (2) contemplation-owning the
problem/evaluate options (3) preparation- plan of action (4)
action-changes are attempted (5) maintenance-behavior
sustained usually 6 mos or longer

Health Promotion Model (Pender)


actions to improve health using the following driversindividual characteristics and experiences, behaviors,
behavioral outcomes, health promotion, activities involve
removing internal and external barriers and promoting
public policy to remove barriers
type of model

Motivation
moved from contemplation to action

Self-efficacy
belief about capabilities to produce effects

Self-management
manage independently choices and consequences of life

Compliance
degree to which a plan is followed (passive)

Adherence
agreement to follow a plan of action (active)

pedagogy
literally means the "art and science of teaching children"

andrgogy
intentional and professionally guided activity the aims at a
change in adults

Nurse-managed centers
nurse-managed centers provide a safety net for the poor
and undeserved in rural and urban areas; run by advanced
practice nurses, and may be associated with universities or
may be independent

parish nursing
licensed, registered nurses practicing holistic care in faith
communities

correctional health nursing


nurses working with populations in jails, prisons, detention
centers, holding facilities

occupational health nursing


nurses working with populations within their work
environment; provide workplace clinics, conduct safety
education and improving working conditions to prevent
injury; balance the company's need for productivity with
knowledge of occupational health and workforce/place
safety

OSHA
sets health and safety work standards, employers are
legally required to protect workers from hazards;
employees have right to know what hazards they are
exposed to

HIPPA
protects the privacy of identifiable, private health
information

EPA
reduce environmental risk and protect the public, maintain
safe air and water, establish pollution regulations, solid
waste and toxic substance disposal, regulate pesticides,
oversee radiation hazards, and noise abatement

HHS (US Dept. of Health and Human Services)


principle government agency that protects the health of
our country; under its umbrella are: CMS, CDC, NIH, FDA,
SAMHSA

CMS
Center for Medicare and Medicaid Services

CDC
monitors and tracts diseases, research injury, infectious
disease, environmental health, genomics, global health,
chronic disease prevention, birth defects and
developmental disorders; publishes the morbidity and
mortality weekly report

NIH
mission is to seek fundamental scientific truth about the
natures and behavior of the living systems and the
application of science to improve health, lengthen life and
reduce the heavy load of illness and disability

SAMHSA
Substance Abuse and Mental Health Administration;
coordinates and funds sustainable substance abuse and
mental health programs in communities or community
health agencies

stakeholders

those who stand to gain or lose from the success or failure


of an initiative

documentation standards
an agency's standards for how care is
recorded/documented on an individual or program level;
provides a legal record, and should be performed in a
timely manner to ensure accuracy

risk management protocol


identification of areas where mistakes of omission or
commission can be made

performance standards
defined standards/expectations for how business is to be
conducted and how performance will be measured

mentoring
expert who establishes a long-term relationship with a
mentee to help them meet professional goals or learn the
role

preceptor
guidance and role modeling are ways to nurture colleagues
or students

professional credibility
clearly demonstrates expertise in an area

expert
perform as an expert maintain standards of behavior and
accountability
health promotion
assessing the health of our community, planning activities
to help our participants to gain some control over their
health to improve their health; emphasis on helping people
change their lifestyles and move toward a state of optimal
health

impact of health promotion


Health promotion impacts communities; health promotion
activities can also impact an entire system; involves all
levels of prevention to promote health across the lifespan.

Goals of Healthy People 20/20


provides science-based, 10 year national objectives for
promoting health and preventing disease--1) to eliminate
preventable disease, disability, injury, and premature
death; 2) to achieve health equity, eliminate disparities,
and improve the health of all groups; 3) to create social
and physical environments that promote good health for
all; and 4) to promote healthy development and healthy
behaviors across every stage of life

determinants of health
factors which influence an individual's or population's
health; influences include the availability and access to:
high quality education, nutritious food, decent and safe
housing, affordable, reliable public transportation, culturally
sensitive health care providers, health insurance, and clean
water and non-polluted air

MAPP
Mobilizaing for Action through Planning & Partnerships; this
tool helps communities improve health and quality of life
through community-wide strategic planning; communities
seek to achieve optimal health by identifying and using
their resources wisely, taking into account their unique
circumstances and needs, and forming effective
partnerships for strategic action

Principles of MAPP
systems thinking, dialogue, shared vision, data,
partnerships and collaboration, strategic thinking, and
celebration of successes

incidence
the number of people in a population who develop the
condition during a specified period of time (number of new
cases over a period of time; rate: (# of new cases of
disease/total population at risk)x100,000

prevalence
the total number of people in the population who have the
condition at a particular time; rate: (# of existing cases of
disease/# of in total population)x100,000

infant mortality rate (IMR)


# of infant deaths (before age of 1 yr) during the year is
divided by the # of live births (infants born alive) during

that year (multiply answer by 1000 to obtain meaningful


rate)

importance of IMR (in epidemiology)


because IMR is influenced by a variety of biologic and
environmental factors affecting the infant and mother, the
IMR is both a direct measure of infant health and an
indirect measure of community health as a whole

relative risk
compares the risk of developing the health condition for
the population exposed to the factor with the risk for the
population not exposed to the factor; indicates the benefit
that might accrue to the person if the risk factor is
removed; rate: incidence among those exposed/incidence
rate among those not exposed

Census (epidemiology data)


source of epidemiological community data; most
comprehensive source of health-related data for the US;
age, race, and sex along with factors such as employment,
income, migration and education; used to calculate rates
(denominator)--collected every 10 yrs

Vital Statistics (epidemiology data)


data that describe legally registered events such as births,
deaths, marriages, and divorces; obtained from CDC's
National Center for Health Statistics and U.S. Public health
Service--collected on an ongoing basis

Notifiable Disease Reports (epidemiology data)


data collected on diseases based on the ability to cause
death, and communicability of the disease; notification of
public health officials of diseases posing a threat to large
populations provides an initial starting point for local
epidemiologic investigations

vital record linkage (epidemiology data)


connects data and information contained in two or more
medical, or mortality records, and other vital event records;
provides excellent sources of information on the courses of
diseases, demographic data, health are services utilization,
fertility, maternal health issues, child health concerns,
chronic disease tracking, and the natural history of specific
disease or morbidity related events

medical and hospital records (epidemiology data)


provide valuable information for community health
research; but do not provide a completely representative or
valid picture of community health

autopsy records (epidemiology data)


provide severely biased information; not performed for all
deaths, violent deaths are included, and sometimes cause
of death is unknown

national electronic disease surveillance system


(epidemiology data)
provides an integrated surveillance system that transmits
public health laboratory and clinical data via the Internet;
improves the nations ability to identify and track emerging
infectious diseases and potential bioterrorism attacks--aids
in surveillance and analysis of notifiable diseases (CDC)

epidemiologic triangle
agent-host-environment model that is a traditional view of
health and disease developed when epidemiology was
concerned chiefly with communicable disease; all 3 can
coexist-disease occurs with interaction of agent, host and
environment

agent
an organism capable of causing disease

host
the population at risk for developing the disease

environment
a combination of physical, biological, and social factors that
surround and influence both the agent and the host

web of causation
views a health condition as the result not of individual
factors but of complex interrelationships of numerous
factors interacting to increase or decrease the risk of
disease; a number of interrelated variables are almost
always involved in the cause of particular outcome

surveillance

systematic collection, analysis, and interpretation of data


related to the occurrence of disease and the health status
of a given population

passive surveillance
more common form of surveillance used by most local and
state health departments; health care providers report
notifiable conditions

active surveillance
purposeful, ongoing search for new cases of disease by
public health personnel

social determinants of health


factors which influence an individual's or population's
health: complex relationships between health and biology,
genetics, and individual behavior, and between health and
health services, socioeconomic status, the physical
environment, discrimination, racism, literacy levels, and
legislative policies

community analysis
multi-step process used to derive a community nursing
diagnosis; phases used are: categorization, summarization,
comparison, and inference elaboration

traditional categorization of community assessment data


(community analysis)
demographic characteristics; geographic characteristics;
socioeconomic characteristics; and health resources and
services

demographic characteristics
family size, age, sex, and ethnic and racial groupings

geographic characteristics
area boundaries, number and size of neighborhoods, public
spaces and roads

socioeconomic characteristics
occupation and income categories, educational attainment,
and rental or home ownership patterns

summarize (community analysis)

once categorization method has been selected, the next


task is to summarize the data within each category; both
summary statements and summary measures, such as
rates, charts, and graphs, are required

compare (community analysis)


identification of data gaps, incongruencies, and omissions;
comparative data are needed to determine if a pattern or
trend exits or if data do not seem correct and if revalidation
of original information is required

draw inferences (community analysis)


drawing logical conclusion form the evidence once data
collected has been categorized, summarized, and
compared; synthesize what is known about a community

community nursing diagnosis


Synthesizes assessment data; a dx both describes a
situation (state) & Implies an etiology (reason) and has
three parts: Description of problem, response, rate; ID of
factors etiologically related to problem; Signs/symptoms
characteristic of problem

Community based planning


are based on the nursing diagnoses and contain specific
goals and interventions for achieving desired outcomes; a
process and has to be completed in partnership with the
community.

Factors in planning a community program


Community partnership (do they agree); Influences of
social, economic, environmental, political issues; Etiologic
antecedents; Health needs of at-risk populations (and do
they agree with this); Awareness/application of planned
change

goals and objectives in planning community health


program
can be written in sequential steps that are required to
reach the goal; must be measurable; include a time frame
for attaining change; the direction and magnitude of the
change; and method of measuring the change

Precise terms to use in making goals and objectives


measurable (learning objectives)

to identify; to discuss; to list; to compare and contrast; to


state; to decrease by 20%

Reinkemeyer's stages of planned change


Stage 1: Identification and development of felt need and
desire for change
Stage 2: Development of a change relationship between
the community health nurse and the community
Stage 3: Clarification or diagnosis of the community's
problem, need, or objective
Stage 4: Examination of alternative routes and tentative
goals and intention of actions (planning)

primary prevention
improves the health and well being of the community,
making it less vulnerable to stressors; health promotion
programs, programs that focus on protection from specific
disease; usually nonspecific and directed toward raising the
general healthy of the total community

secondary prevention
begins after a disease or condition is present, although
there may be no symptoms; emphasis on screening, early
diagnosis, and treatment of possible stressors that may
adversely affect the community's health

tertiary prevention
focuses on restoration and rehab; act to return the
community to an optimal level of functioning.

evaluation
must be planned before implementation; measures
progress toward goals and learning objectives; revises
assessment database and community nursing diagnosis

process (formative) evaluation


initial implementation/development of program; answers:
are we doing what we said we would do?

impact (summative; short term) evaluation


concerned with the immediate impact of a program on a
target group

outcome (long-term) evaluation

has incidence/prevalence changed? determine if change


had a lasting and real effect

Phases of MAPP
Phase 1: organizing for success, partnership development
Phase 2: visioning
Phase 3: The MAPP assessments
Phase 4: Identifying strategic issues
Phase 5: developing goals and strategies
Phase 6: the action cycle

Priority Health Promotion of HP 20/20


unintentional injury; violence; suicide; tobacco use and
addiction; alcohol or other drug use; unintended
pregnancy, HIV/AIDS, and STD infection; unhealthy dietary
patterns; and inadequate physical activity.

Cognitive learning domain


memory, recognition, understanding, reasoning,
application, and problem solving and is divided into
hierachical classification of behaviors; hierarchical
classification of behaviors (knowledge, comprehension,
application, analysis, synthesis and evaluation)

affective learning domain


changes in attitudes and the development of values by
influencing what learners feel, think, and value

psychomotor learning domain


involves the performance of skills that require some degree
of neuromuscular coordination and emphasize motor skill;
in learning a skill, first show learners how to do the skill;
next allow learners to practice (teach back or show back
approach)

educational process
1. identify client needs-needs assessment
2. establish educational goals and objectives
3. select appropriate educational strategies
4. develop effective skills as an educator
5. develop effective health education programs

goals
long term group overall learning needs or expected
outcomes after instruction or program activities

objectives
specific, short term, measurable, action oriented behaviors
to be met as steps towards achieving the long term goal

Educator Skill (education process)


1. gain attention
2. inform the learner of the objectives of instruction
3. stimulate recall of prior learning
4. present the stimulus
5. provide learning guidance
6. elicit performance
7. provide feedback
8. assess performance
9. enhance retention and transfer of knowledge

global burden of disease


the disability-adjusted life year (DALY), which is a timebased measure combining years of life lost due to
premature mortality and years of life lost due to time lived
in states of less than full health

Millennium Development Goals


1. eradicate extreme poverty and hunger
2. achieve universal primary education
3. promote gender equality and empower women
4. reduce child mortality
5. improve maternal health
6. combat HIV/AIDS, malaria, and other diseases
7. ensure environmental sustainability
8. develop a global partnership for development

child mortality and millennium goal


reduce by 2/3, between 1990 and 2015, the under 5
mortality ratio

how to influence maternal health


increased access to prenatal care, to skilled birth
attendants, and the prevention of teen and unplanned
pregnancies

8 essential elements of primary health care


1. education for the identification and prevention/control of
prevailing health problems
2. proper food supplies and nutrition

3. adequate supply of safe water and basic sanitation


4. maternal and child care, including family planning
5. immunization against the major infectious diseases,
prevention and control of locally endemic diseases
6. appropriate treatment of common diseases using
appropriate technology
7. promotion of mental health
8. provision of essential drugs

genomics
the study of functions and interactions of all genetic
material in the genome, including interactions with
environmental factors

public health genomics


the study and application of knowledge about the elements
of the human genome and their functions, including
interactions with the environment, in relation to health and
disease populations

nursing informatics
specialty that integrates nursing science, computer
science, and information science to manage and
communicate data, information, and knowledge in nursing
practice

information literacy
the ability to recognize when information is needed as well
as the skills to find, evaluate, and use needed information
effectively

public policy
applies to all members of society; includes prescribed
sanctions for failure to comply; formulated by government
bodies; frequently restricts personal choice to improve
public welfare

social welfare policy


policy that promotes the welfare of the public; child labor
laws and income assistance (social security)

institutional policy
governs work sites; based on the institution's mission and
goals, and they determine how the institution will function
and relate to employees

organizational policy
formed an applies to groups who have similar interests or
special interests, such as professional associations-state
nurses association or specialty nursing organization that
determines governing rules for membership

medicare
health insurance program; 65 years or older, disabled
people younger than 65, end stage renal disease or Lou
Gehrig disease

medicaid
healthcare, insurance assistance, nursing home coverage;
low-income residents and pregnant women, a parent or
relative caretaker of a dependent child under age 19, the
blind, people with a disability or with a family member in
their household with a disability, or people aged 65 years
or older

State Children's Health Insurance Program (SCHIP)


doctor visits, immunizations, hospitalizations, emergency
room visits; children 19 years and younger from families
who earn to much to qualify for Medicaid but who meet
SCHIP eligibility

What nurses can do to influence health policy


1. Learn the legislative process
2. Stay informed on current issues
3. Identify government representatives
4. Be an active constituent
5. Speak out

health economic
concerned with how scarce resources affect the health care
industry

public health economics


focuses on producing, distributing, and consuming goods
and services as related to public health and where limited
public resources might best be spent to save lives or cause
the most increase in quality of life

macroeconomics

the study of the economy as a whole; focuses on the "big


picture" -the total, or aggregate (usually a country or
nation), of all individuals and organizations; business cycle
and economic growth; human capital approach

microeconomics
the branch of economics that studies the economy of
consumers or households or individual firms; the behavior
of individuals and organizations & how this effects prices,
costs and resources; behaviors that result from tradeoffs in
the use of service and budget limits

efficiency (economics)
refers to producing maximal output, such as a good or
service, using a given set of resources (or inputs), such as
labor, time, and available money; suggests that inputs
used in such a way that there is no better way to produce
the service, or output, and that no other improvements can
be made

effectiveness (economics)
refers to the extent to which a health care service meets a
stated goal or objective, or how well a program or service
achieves what is intended; immunization=herd immunity

environmental health
promotion of safe, healthful living conditions and protection
from environmental factors that may adversely affect
human health or the ecologic balances essential to longterm human health and environmental quality, whether in
the natural or man-made environment

radon (Rn)
a radioactive gas that is formed naturally from the
radioactive decay of uranium in rocks and soil; it can
remain in the soil, move to the soil surface and enter the
air, or enter groundwater; colorless, odorless, and
tasteless, but extremely toxic; when cooled below the
freezing point, it becomes phosphorescent, in yellow and
orange-red tones.

risk for exposure to radon


You can be exposed to higher levels of radon at work if you
are a uranium miner, a hard rock miner, or work in tunnels,
power stations, public baths, or spas.

exacerbation of asthma in kids


pollutants reduce lung function by inflaming the lining of
the lungs; exposure to pollutants in the air can cause flareups and may increase the chance of upper respiratory
infections, which can worsen asthma symptoms. If
allergens in the air are an asthma trigger, pollutants can
make the lungs even more sensitive to them.

ozone action days


declared on hot, sunny, still days when conditions are most
favorable for the accumulation of unhealthy levels of
ground-level ozone; forcasted by Texas Commission on
Environmental Quality (TCEQ) make their forecasts a day in
advance: Austin, Beaumont-Port Arthur, Corpus Christi,
Dallas-Fort Worth (where it's called an "Air Pollution
Watch"), El Paso, Houston-Galveston-Brazoria (where it's
called an "Ozone Watch"), San Antonio (where it's called an
"Air Quality Health Alert") and Tyler-Longview-Marshall.

6 ways climate change can impact health


air pollution, extreme heat, infectious disease, drought,
flooding, and extreme weather

air pollution and health


Rising heat worsens smog. Burning coal and oil emits
carbon and particle pollution; plants produce more
allergenic pollen, affecting respiratory health threats like
asthma.

extreme heat and health


Heat waves send thousands to emergency rooms and cost
health care systems millions of dollars; climate change
brings longer, more intense heat waves.

infectious diseases and health


Hotter summers can make disease-carrying insects more
active, for longer seasons; illnesses like dengue, West Nile,
and Lyme can spread into new areas

drought and health


Hotter days and nights, and changing rainfall patterns
reduce water supply quantity and quality, and diminish
food security.

flooding and health


Climate change intensifies rainfall; heavy rains increase
risk of drinking water contamination and illness; floods can
force communities to relocate.

extreme weather and health


In 2011, thousands of record-breaking extreme weather
events harmed communities and health in the US. Climate
change is contributing to more intense and frequent
extreme weather events.

Fracking
common but controversial practice among companies that
drill underground for oil and natural gas; drillers inject
millions of gallons of water, sand, salts and chemicalsall
too often toxic chemicals and human carcinogens such as
benzeneinto shale deposits or other sub-surface rock
formations at extremely high pressure, to fracture the rock
and extract the raw fuel.

dangers of fracking
leaves behind a toxic sludge that companies and
communities must find some way to manage; toxic
chemicals used in the fracking process remain stranded
underground where they can, and often do, contaminate
drinking water, soil and other parts of the environment that
support plant, animal and human life; Methane from
fracture wells can leak into groundwater, creating a serious
risk of explosion and contaminating drinking water supplies

What to look for in sunscreen


active ingredients (zinc oxide, titanium dioxide, Mexoryl SX
or avobenzone-3%)
SPF 15-50, depending on your skin tone and sun intensity
Lotions, not sprays or powder
Water-resistant for the beach or pool

What to avoid in sunscreen


vitamin A causes skin cancer in laboratory tests
Oxybenzone, a hormone disruptor and skin allergen
High SPF misleads consumers and offers little additional
benefit

public health policy


works to improve the publics health through legislative
action at the local, state and federal level; institutional and

legislative efforts to promote health and meet national


health objectives...support initiatives that address barriers
to health such as poverty, homelessness, unsafe living
conditions, abuse and violence, and lack of access to
health services

human capital
the knowledge and skills that workers acquire through
education, training, and experience

relative risk factor


factor of 1.0 means the risk of disease among the exposed
is no different from the risk of disease among the nonexposed; an rr of 2.0 implies that the risk is twice as high,
whereas a rr of 0.5 indicates that the exposure factor is
associated with 1/2 the risk of disease (protective factor)

strategies for health for 2015 (bexar county)


1. evidence-based teen pregnancy prevention-reduce teen
birth rate by 15%
2. built environment for active living-reduce physical
inactivity by 10%
3. neighbors engaged in health-community asset mapping,
action plans and active engagement in 10 neighborhoods
4. blood and sexually transmitted disease-reduce
congenital syphilis by 50% and reverse trends in syphilis
incidence

benefits of MAPP
creates a healthy community; increases visibility of public
health; manages change; stronger public health
infrastructure; builds stronger partnerships; builds public
health leadership; creates advocates

Questions to track outcome (MAPP)


How do we track?
How do we affect public policy?
How do we promote a sense of community?
How do we assure access to care?
How do we encourage healthy lifestyles?
How do we provide a safe environment?

pandemic
steady occurrence of disease over a large geographic area
or worldwide, such as malaria

epidemic
refers to the unexpected increase of an infectious disease
in a geographic are over an extended period of time

endemic
diseases that occur at a consistent, expected level in a
geographic area

outbreak
refers to the unexpected occurrence of an infectious
disease in a limited geographic area during a limited period
of time

infectious agent
an organism (virus, Rickettsia, bacteria, fungus, protozoan,
helminth, or prion) capable of producing infection or
infectious disease

reservoirs
the environment in which a pathogen lives and multiplies

portal of exit
means by which an infectious agent is transported from the
host

mode of transmission
method whereby the infectious agent is transmitted from
one host (or reservoir) to another host

portal of entry
means by which an infectious agent enters a new host

host susceptibility
the presence or lack of sufficient resistance to an infectious
agent to avoid prevent contracting an infection or acquiring
infectious disease

direct transmission
implies the immediate transfer of an infectious agent form
an infected host or reservoir to a portal of entry in the
human host through physical contact such as touching,
biting, kissing, or sexual contact

indirect transmission
the spread of infection through a vehicle of transmission
outside the host

fomites
an inanimate object, material, or substance that acts as a
transport agent for a microbe

vectors
can be animals, insects or artropods, and they can transmit
infection through biologic and mechanical routes

susceptible
at risk for contracting an infection or developing infectious
disease; affected by general health status, personal
behaviors, presence of healthy lines of defense, immune
system and immunization status

natural immunity
an innate resistance to a specific antigen or toxin

acquired immunity
derived from actual exposure to the specific infectious
agent, toxin, or appropriate vaccine

active immunity
when the body produces its own antibodies against an
antigen, either as result of infection with the pathogen or
introduction of the pathogen in a vaccine

passive immunity
the temporary resistance that has been donated to the
host through transfusions of plasma proteins,
immunoglobulins, and antitoxins, or transplacentally from
mother to neonate

herd immunity
refers to a state in which those not immune to an infectious
agent will be safe if a certain proportion of the population
has been vaccinated or is otherwise immune

descriptive epidemiology
focuses on the distribution of frequencies and patterns of
health events with groups in a population; examine disease

patterns and other health-related phenomena according to


"person" "place" and "time"

analytic epidemiology
seeks to identify associations between a particular disease
or health problem and its etiology; directed toward finding
answers to the "how" and "why" of health disease to
determine casuality

synergism
concept central to the web of causation model, wherein the
whole is more than the sum of its separate parts
Epidemiology is...
the study of the distribution and determinants of health
and disease in human populations

Person-place-time model
Person: "Who" factors, such as demographic
characteristics, health, and disease status
Place: "Where" factors, such as geographic location,
climate and environmental conditions, political and social
environment
Time: "When" factors, such as times of day, week, or
month and secular trends over months and year

Descriptive epidemiology:
-Study of the amount and distribution of disease
-Used by public health professionals
-Identified patterns frequently indicate possible causes of
disease

Analytic Epidemiology
-Examine complex relationships among the many
determinants of disease
-Investigation of the causes of disease, or etiology

Epidemiological Triangle
Host, Agent, Environment

Agent of Disease (Etiologic Factors)


-Nutritive elements (excesses, deficiencies)
-Chemical agents (Poisons, allergens)
-Physical agents (Ionizing radiation, mechanical)

-Infectious agents (metazoa, protozoa, bacteria, fungi,


rickettsia, viruses)

Host Factors of disease- Intrinsic Factors (Susceptibility, or


Response to Agent)
Genetic, age, sex, ethnic group, physiological state, prior
immunological experience (active, passive), intercurrent or
preexisting disease, human behavior

Environmental Factors of disease- Extrinsic Factors:


influence existence of the agent, exposure, or susceptibility
to agent. This includes...
-Physical environment
-Biological environment such as human populations, flora,
fauna
-Socioeconomic environment such as occupation,
urbanization and economic development, disruption

Wheel Model of Human-Environment Interaction


Page 73
-In the middle: Host (human); this is the genetic core
-Surrounding that in the wheel is Biological environment,
social environment, and physical environment

Web of Causation
illustrates the complexity of relationships among causal
variables for heart disease; page 73

Ecosocial approach
-Emphasizes the role of evolving macro-level
socioenvironmental factors along with microbiological
process in understanding health and illness
-challenges the more individually focused risk factor
approach to understanding disease origins

What are rates?


arithmetic expressions that help practitioners consider a
count of an event relative to the size of the population from
which it is extracted

The numerator of a rate is:


The number of health events in a specified period

The denominator of a rate is:

population in same area in same specified period (e.g., per


day, per week, or per year)

What is the proportion rate multiplied by?


a constant (k); For example, the rate can be the number of
cases of a disease occurring for every 1000, 10,000 or
100,000 people in the population

Morbidity rates
rates of illness (page 75)

What do incidence rates describe?


the occurrence of new cases of a disease or condition in a
community over a period of time relative to the size of the
population at risk for that disease or condition during that
same time period.

What do attack rates document?


the number of new cases of a disease in those exposed to
the disease (ex. food poisoning; denominator is the number
of people exposed to a suspect food, and the numerator is
the number of people who were exposed and became ill.

What are prevalence rates?


All cases of a specific disease or condition at a given time

Those who survive a chronic disease without cure remain


in the:
"prevalence pot"

What is the prevalence pot?


The relationship between incidence and prevalence

What are mortality rates?


routinely collected birth and death rates

Risk refers to:


the probability of an adverse event

Risk factor
-Refers to the specific exposure factor
-Often external to the individual

Attributable risk

-Estimate of the disease burden in a population

Relative risk ratio


Divide the incidence rate of disease in the exposed
population by the incidence rate of disease in the
nonexposed population

Primary prevention:
-when interventions occur before disease development
-Includes health promotion and specific prevention

Secondary prevention:
occurs after pathogenesis; screening and physical
examinations that are aimed at early diagnosis

Tertiary prevention:
focuses on limitation of disability and the rehabilitation of
those with irreversible diseases such as diabetes and spinal
cord injury

Why is it important to identify causality of disease?


to encourage the most effective prevention activities and
develop treatment modalities

What is the purpose of screening?


to identify risk factors and diseases in their earliest stages

What is surveillance?
a mechanism for the ongoing collection of community
health information

Why is surveillance important?


because monitoring for changes in disease frequency is
essential to effective and responsive public health
programs.

Should community health nurses and community


members collaborate? If so, why?
YES! This is so that they can combine epidemiological
knowledge and aggregate-level strategies to affect change
on the broadest scale

Descriptive epidemiology focuses on:

the amount and distribution of health and health problems


within a population

Why are observational studies used?


-Descriptive purposes
-Etiology of disease
-No manipulation by investigator

What are cross-sectional studies also called?


Prevalence or correlational studies

What is the purpose of a cross-sectional study?


It examines relationships between potential causal factors
and disease at a specific time

Why is it impossible for a cross-sectional study to make


causal inferences?
because the study cannot establish the temporal sequence
of events (i.e., the cause preceded the effect)

What does a retrospective study do?


-Compares individuals with a particular condition or disease
with those who do not have the disease
-Data collection extends back in time

What does a prospective study monitor?


a group of disease-free individuals to determine if and
when disease occurs

In a prospective study, cohort...


shares a common experience within a defined time period
Which role of the nurse is most important in achieving the
goal of Healthy People 2020: "To achieve health equity,
eliminate disparities, and improve the health of all
groups"?
A) Advocate
B) Researcher
C) Teacher
D) Leader
A
Rationale: Nurses have a role in seeing that deficiencies in
the health care system are addressed. Caring is the focus

of nursing and in many cases this caring is manifested as


advocacy for the client. Nurses advocate for clients to
achieve health within their own abilities, opportunities, and
social contexts.

The nurse explains to the student nurse that nursing


interventions for families in community-based settings
are similar to those for individuals and involve three
types: cognitive, affective, and behavioral. Which of the
following is considered an affective intervention?
A) The nurse demonstrates to a client with a colostomy
how to change the drainage bag
B) The nurse counsels a client to help her overcome her
grief over the death of a loved one
C) The nurse teaches a client how exercise will have a
beneficial effect on her cardiovascular system
D) The nurse teaches a teenager which foods to choose
as snacks to avoid consuming empty calories
B
Rationale: Affective interventions have to do with feelings,
attitudes, and values, such as occurs with grief counseling.
Cognitive interventions involve the act of knowing,
perceiving, or understanding, such as teaching the effects
of exercise and proper nutrition. Behavioral interventions
involve performing skills and demonstrating behaviors,
such as changing a colonoscopy bag.

A nurse coordinates all the services needed for the family


of an infant born with cerebral palsy. The nurse's efforts
focus on empowering the family to maximize self-care
through health promotion, disease prevention, and
increased continuity of care. What is the term for this
process?
A) Case Management
B) Consultation
C) Discharge Planning
D) Screening
A
Rationale: Case management, also known as care
management or care coordination, involves activities that
enhance the self-care capacities of clients and families by
coordinating care. Discharge planning is an accepted
nursing intervention aimed at the prevention of problems

after discharge. Screening identifies individuals with


unrecognized health risk factors of asymptomatic diseases.
During consultation, the nurse seeks information and
generates solutions to problems through an interactive
problem-solving process.

A community-based nurse is providing care for an elderly


male client and his family in his home. The family
includes a spouse and a daughter who is living at home
as well as a 10-year-old grandson. The nurse plans care
for the client in the context of the family. Which of the
following statements best describes this model of care?
A) This model emphasizes how family structure, function,
developmental stage, and interpersonal interactions
influence the recovery of the client
B) This model focuses on the family as a system as well
as the unit of service; the individual and family are
concentrated on simultaneously
C) This model considers the family as it relates to the
recovery of the individual client; the client is the focus
and the family is the context
D) This model allows the family to be seen as one of the
many institutions or social systems in the community that
interact with other systems for services
C
Rationale: Providing care in the context of the family occurs
when the nurse considers the family as it relates to the
recovery of the client. When the focus is on the family's
potential impact on the recovery of the client, the nurse
considers how family structure, function, developmental
stage, and interpersonal interactions influence the recovery
of the client. When the family is seen as the client, the
nurse focuses on the family as a system as well as the unit
of service. When the family is viewed as a component of
society, this model allows the family to be seen as one of
the many institutions or social systems in the community
that interact with other systems for services.

The nurse caring for families is aware that family


functions are outcomes of family structure and are the
reason families exist. Which of the following is an
example of an affective family function?
A) A father works two jobs to help provide for his family
B) A mother schedules a dental checkup for her

adolescent son
C) A new mother cuddles and sings to her newborn
D) A father arranges a play date for his 5-year old
daughter
C
Rationale: The affective family function is the family's
ability to meet the psychological needs of its members,
such as bonding with a newborn. The economic function of
the family involves the allocation of adequate resources for
the family members. The providing for health care and
physical necessities function is involved in scheduling
checkups. Arranging a play date is an example of the
socialization function.

The nurse providing care in a community-based health


care setting knows that prevention of disease and injury
is a key component of the practice. Which of the following
interventions best exemplifies disease prevention on the
secondary level?
A) A nurse screens school-aged children for scoliosis
B) A nurse teaches new mothers how to bathe their
infants
C) A nurse coordinates home care for an elderly client
with dementia
D) A nurse teaches stress management to clients
recovering from heart attacks
A
Rationale: Primary disease prevention involves prevention
of the initial occurrence of the disease or injury, such as
teaching hygiene for newborns. Secondary disease
prevention is early identification of disease or injury with
prompt intervention to limit its effects. This includes
screening for disorders such as scoliosis. Teaching stress
management following MI and coordinating home care are
examples of tertiary disease prevention to halt further
disease progress and meet help the person meet their
maximum potential.

The nurse working in community-based nursing explains


to the student nurse how certain factors affect the
delivery of nursing care in the community. Which of the
following statements regarding communities is accurate?
A) Social determinants of health are shaped by economics

and politics
B) Culture plays a minor role in the overall character of
the community
C) Community boundaries cannot limit the services
available to individuals
D) Social systems have little or no impact on a
community's health
A)
Rationale: Social determinants of health are defined as the
circumstances in which people are born, grow up, live,
work, and age, and the systems put in place to deal with
illness. These circumstances are shaped by economics,
social policies, and politics. Social systems have an impact
on a community, and consequently the health of that
community. Culture contributes to the overall character of a
community and, in turn, influences its health needs. A
community is defined by boundaries that often determine
what services are available to individuals.

A nurse caring for migrant workers of Hispanic descent in


a community clinic practices etic care that is acceptable
and successful for these particular clients. Which of the
following statements best describes etic care?
A) It is professional care based on an understanding of
the client's cultural background
B)It is informal care directed an involving the client in selfcare based on his or her own culture
C) It is allowing the local customs of the client to dictate
the type of care that is provided
D) It involves using cultural blindness to treat clients from
diverse background equally and fairly
A
Rationale: "Etic" refers to the outsider's views and values
about a phenomenon. Etic care involves using an emic
(insider's) understanding of the client's beliefs about health
issues to coordinate professional (etic) care that is
acceptable to the client.

Nurses preparing for a career in community-based


nursing should be aware of the current trends affecting
the future of nursing education and how these trends will
affect their practices. Which of the following accurately
describes one of these trends?

A) There are changing demographics and decreasing


diversity in communities
B) There is a shift to hospital-based care and increased
complexity of client cases
C) There are lower costs of health care using managed
care practices
D) There are current nursing shortages and workforce
development continues
D
Rationale: Among other factors, the future of nursing
education is affected by: (1) current nursing shortages,
opportunities for lifelong learning, and work development;
(2) changing demographics and increasing diversity; (3)
higher costs of health care and the challenges of managed
care; and (4) a shift to population-based care and the
complexity of client care.

A nurse initiates a regular physical activity program for


residents of a nursing home. Which of the following
statistics accurately portrays the incidence of existing
health problems in the United States that can be
influenced by regular physical exercise?
A) 10 million people have type 2 diabetes
B) 8.5 million people have coronary heart disease
C) 50 million people have high blood pressure
D) Over 25 million people are overweight
C
Rationale: Regular physical activity reduces the risk of
heart disease, cancer, and diabetes. In the United States,
50 million people have high blood pressure, 13.5 million
people have coronary heart disease, more than 60 million
people are overweight, and 8 million people have type 2
diabetes.

Upon graduation, a nurse chooses to practice communitybased nursing instead of working in an acute care setting.
Which of the following statements best defines
community-based nursing?
A) It is a "flowing" kind of care that does not necessarily
occur in one setting
B) It does not have a defined philosophy but rather is
defined by the setting itself
C) It is defined by the level of academic preparation

needed to preform the skills


D) It is all about where the nurse practices as opposed to
how the nurse practices
A
Rationale: The emphasis of community-based nursing is a
"flowing" kind of care that does not necessarily occur in
only one setting. Community-based nursing care is directed
toward specific individuals and families within a community
and is designed to meet the needs of people as they move
between and among health care settings. It is all about how
the nurse practices, not where the nurse practices.
Community-based nursing has a defined philosophy of
practice that requires specific knowledge and skill. It is not
defined by the setting or level of academic preparation
needed to perform the care.

The nurse is preparing to discharge a client with a spinal


cord injury from an acute care facility in preparation for a
transfer to a rehabilitation facility. Which of the following
statements accurately describes a focus of this process?
A) It is one dimensional and prescribed in facilitating
transfer from one setting to another
B) It identifies services and referrals that may be needed
and passes this information along to the case manager
C) It is unique in that it does not follow the nursing
process, but instead focuses entirely on preventive care
D) Its focus is on the client functioning as an individual
and the client's unique medical needs
A
Rationale: Discharge planning is one dimensional and is
prescribed in facilitating transfer from one setting to the
other. It follows the nursing process and includes
assessment, planning, and interventions. Discharge
planning focuses on the client and family needs and
abilities. The interventions involve making referrals and
giving the client and family important phone numbers,
names, and community services in writing, not just a
referral to a case manager.

The nurse is caring for a male client with HIV who is living
unmarried with another male and has two adopted
children. How would the nurse document this client's
family structure?

A) Nuclear Family
B) Nuclear Dyad
C) Single-Parent Family
D) Multigenerational Family
A
Rationale: A nuclear family consists of a married couple
with children, or unmarried, heterosexual, or same-sex
couples with children. A nuclear dyad is a couple, married
or unmarried; heterosexual or same sex. A single-parent
family is one adult with children, and a multigenerational
family is any combination of these family structures.

A nurse is designing a physical activity program for a 48year-old male client who has been diagnosed with mild
hypertension. Which of the following is an appropriate
long-term goal for this client?
A) Jog for 15 minutes and weight train for 15 minutes, 3
days a week
B) Walk briskly for 30 minutes most days of the week
C) Bicycle for 1 hour every day of the week
D) Walk briskly for 30 minutes every other day
B
Rationale: Moderate physical activity is recommended for
30 to 45 minutes, 3 to 5 days a week as an initial goal. An
appropriate long-term goal is 30 minutes of moderateintensity physical activity all or most days of the week.

Which of the following nursing skills is most imperative


when practicing community-based nursing?
A) A nurse attends and in-service on using new IV and
infusion equipment
B) A nurse delegates nursing care appropriate for an LPN
to perform
C) A nurse collaborates with a physical therapist treating
a client with paraplegia
D) A nurse institutes a new format for documenting client
care in the field
C
Rationale: Although all these skills are important, the most
critical skills needed by a nurse practicing communitybased nursing are highly developed assessment skills,
effective communication skills, collaboration with the

interdisciplinary team, and working with culturally diverse


clients.

A nurse demonstrates how to change the bandages on


the stump of a client with a new below-the-knee
amputation. This is an example of learning according to
which of the following learning domains?
A) Cognitive
B) Psychosocial
C) Psychomotor
D) Affective
C
Rational: Psychomotor learning consists of acquired
physical skills that can be demonstrated, such as changing
bandages on a stump. Cognitive learning involves the
storage and recall of new knowledge and affective learning
refers to the feelings, values, and attitudes that affect
learning. Psychosocial is a factor for learning readiness, not
a learning domain.

The nurse is counseling a 24-year-old working mother


who has a 2-year-old and a newborn who was diagnosed
with cerebral palsy. The mother states that she is
overwhelmed with trying to care for her family as well as
schedule work and take care of her home. The nurse
formulates a diagnosis based on which of the following?
A) Role overload
B) Role stress
C) Role conflict
D) Role flexibility
A
Rationale: Role strain or overload arises when an individual
is confronted with too many role responsibilities at one
time. Role conflict occurs when the demands of one role
conflict with or contradict another. Role flexibility occurs
when other family members take on the roles of a family
member in times of need. Role stress is a distractor.

The nursing process is an important tool to guide the


nurse in thinking through assessment, planning,
implementation and evaluation of the nursing care plan.
Which of the following describes how the nursing process
is used differently by nurses practicing community-based

nursing versus nurses practicing acute care nursing?


A) Client teaching is more important in the hospital
setting than in the community setting because of
shortened hospital stays
B) In community-based nursing, the nursing process is a
mutual endeavor used to plan care, and also to develop
therapeutic relationships with the client's family members
C) Nursing diagnoses are critical in the hospital setting,
but are not defined in community-based nursing
D) In the hospital setting, nurses follow a plan of care
from the health care provider, which does not occur in the
community setting
B
Rationale: Nurses in both settings use clinical decisionmaking skills and clinical reasoning to identify what to
assess in the client, family, and community. In contrast to
care typically provided in hospitals, in the community
setting the nursing process guides the nurse to a mutual
care plan and helps the nurse to develop therapeutic
relationships with the client, family, and caregiver.
Community-based nurses may follow a plan of care from a
health care provider, but they usually need to modify the
care plan for the individual needs of the client and family.
Client teaching is important in both settings and occurs
with the client and family and/or caregiver. Nursing
diagnoses or problem statements are formed for clients in
both settings and interventions are identified that are
reasonable and acceptable to all parties involved in the
planning process.

A nurse conducting a parenting class teaches parents the


leading causes of death in different age groups. Which of
the following age groups has the highest rate of death
related to cancer?
A) 1-4 year olds
B) Under 1 year-old
C) 15-24 year olds
D) 5-9 year olds
D
Rationale: The leading causes of death in 5- to 9-year-olds
are unintentional injuries, cancer, and congenital
anomalies. For infants less than 1 year of age, the leading
causes of death are congenital anomalies, disorders related

to premature birth, and SIDS. In the 1- to 4-year-old group,


the leading causes of death are unintentional injuries,
congenital anomalies, and homicide. In the 10- to 14-yearold group, the leading causes of death are unintentional
injuries, cancer, and homicide. In the 15- to 24-year-old
group the leading causes of death are unintentional
injuries, homicide, and suicide.

A nurse promotes public health interventions at all


possible levels of practice in the community. Which of the
following is the term for this public health practice?
A) Individual-focused
B) Systems-focused
C) Population-based
D) Community-focused
C
Rationale: Public health interventions are population-based
if they consider all levels of practice, including the
community, the systems, and the individuals or families in
that system who are known to be at risk. Communityfocused interventions change community norms, attitudes,
awareness, practices, and behaviors. Systems-focused
interventions change organizations, policies, laws, and
power structures. Individual-focused interventions change
knowledge, attitudes, beliefs, practices, and behaviors of
individuals.

The nurse practicing in a community public health clinic


sees the effects of fragmentation of care on client health
care. Which of the following is an example of a nursing
intervention to help address fragmented care?
A) Encourage health care practitioners to keep formal and
informal care separate
B) Encourage clients to use home health care for more
intense health care needs
C) Encourage clients to routinely use a medical home
D) Encourage clients to use a variety of providers for
health care needs
C
Rationale: Clients are increasingly seen by a large variety
of providers in many organizations and agencies, often
resulting in fragmentation of care. Encouraging routine use
of a medical home is one attempt to address fragmented

care as opposed to using a variety of providers for health


care needs. The use of inpatient facilities for briefer. more
intense encounters should be encouraged. Continuity of
care is often accomplished by combining formal and
informal care.

A home health care nurse is performing an assessment of


an 18-month-old child. Which of the following normal
parameters for the development of hearing and its
assessment should the nurse keep in mind during the
screening?
A) Parents should not be concerned about their child's
hearing until age 3, when hearing is fully developed
B) Parents should be advised to tell the nurse if their
infant is not reacting to loud noises by the age of 2
C) The child's speech patterns should be assessed
separately, since hearing impairment has no impact on
the development of speech
D) Children are not routinely screened for hearing until
three years-old; however, a parent's observations could
indicate a possible hearing problem
D
Rationale: Infants and toddlers are not routinely assessed
for hearing until 3 years of age. However, a parent's
observations may indicate the possible presence of hearing
or vision impairment. By the age of 1, infants should react
to loud noises and hearing their name called. By age 2,
children should repeat words and enjoy games like peek-aboo. Speech should also be assessed because hearing
impairments often become apparent when the child begins
to talk.

A nurse is counseling a 65-year-old female client who


expresses a desire to quit smoking. Which of the following
would be an appropriate response to this client?
A) "If you smoke you have a 50% greater heart disease
death rate than nonsmokers"
B) "Older smokers are more likely to stay off cigarettes
once they quit than younger smokers"
C) "In some cases, ex-smokers who have had a heart
attack can cut their risk of having another one by 25%"
"Since you have been smoking most of your life, it's not
important to stop now since the benefits would be
minimal"

B
Rationale: Older smokers are more likely to stay off
cigarettes once they quit than younger smokers since they
know more about the short-term and long-term benefits of
quitting. It is always good to quit smoking at any age, and
ex-smokers live longer and healthier lives. People who
smoke have a 70% greater heart disease death rate than
do nonsmokers. In some cases, ex-smokers who have had a
heart attack can cut their chance of having another one by
50%.

Most cultures combine orientation to the past, present,


and future, but one orientation usually dominates. For
which of the following clients would teaching strategies
for preventive care be most accepted based on his or her
orientation to the future?
A) A Native American male athlete prone to stress
fractures
B) A white American female lawyer who is experiencing
job stress
C) An African American male accountant who is
overweight
D) A Hindu business executive who is experiencing
migraines
B
Rationale: Traditionally, American culture is future oriented;
however, Native American and Hindu culture tend to focus
on the past while African American culture focuses on the
"now" and day to day activities. When discussing
preventive care, persons without a future orientation
should be approached differently than those with future
orientation.

The nurse is counseling a couple involved in the divorce


process. Which of the following emotional responses is
common during the separation stage of divorce?
A) Mourning the losses associated with a separating
family
B) Negotiating viable arrangements for all family
members
C) Revealing the fact that the marriage has major
problems

D) Working on emotional recovery by overcoming hurt,


anger, or guilt
A
Rationale: During the separation stage of divorce the family
mourns the losses associated with the separation and
works on resolving attachment to the spouse. In the stage
of planning the dissolution of the family system, the family
negotiates viable arrangements for all family members. In
the divorce stage, the family continues working on an
emotional recovery by overcoming hurt, anger, or guilt. In
the stressors leading to marital differences stage, the
family reveals the fact that the marriage has major
problems.

Nurses in community-based settings on many occasions


provide teaching plans to clients with sensory deficits.
Which of the following interventions reflects a
recommended guideline when teaching sensory-impaired
clients?
A) A nurse asks a client with a hearing aid to remove the
hearing aid when providing teaching
B) A nurse speaks to a visually impaired client when
approaching him from the front
C) A nurse directs questions to the family of a client who
has aphasia when taking a nursing history
D) A nurse asks other people in the room to remain silent
when approaching a visually impaired client
B
Rationale: For a visually impaired client, the nurse should
speak to the client when approaching him and avoid
speaking from behind the client. The nurse should also ask
other people to introduce themselves to allow the client to
hear people's voices. The nurse should always ask if the
client wears a hearing aid and if it is working properly. The
nurse should provide some means of conversation (such as
a computer or letter board) for a client who has aphasia
(language deficit).

A nurse caring for clients in a woman's health clinic


advises them on the recommended frequency of Pap
smears, mammograms, and fecal occult blood testing.
Which of the following is a guideline for these tests?
A) Women who have been sexually active should have a

Pap test every three years except if they have genital


warts, multiple partners, or abnormal Pap tests
B) Women over age 75 with a history of normal Pap
smears may stop having Pap tests after consulting with
their health care practitioner
C) Starting at age 50, fecal occult blood testing should be
done every other year in combination with other
screening tests recommended by the practitioner
D) Women over 60 years old should have a mammogram
every 2 years unless a family history suggests other wise
A
Rationale: Women who have ever been sexually active
should have a Pap test every three years except if they
have genital wars, multiple partners, or abnormal Pap tests
in the past. Women over age 65 with a history of normal
Pap smears may stop having Pap tests after consulting with
their health care practitioner. Women over 50 years old
should have a mammogram every two years unless their
family history suggests otherwise. Starting at age 50, fecal
occult blood testing should be done every year in
combination with other screening tests recommended by
the practitioner.

Which of the following is an example of an activity aimed


at providing primary disease prevention?
A) Counseling a rape victim
B) Performing mental health screening
C) Providing well-child care
D) Teaching breast self-examination techniques
C
Rationale: Providing well-child care is an example of
primary disease prevention to prevent the initial
occurrence of a disease or injury. Teaching breast selfexamination and performing mental health screening are
examples of secondary prevention activities, and
counseling a rape victim is an example of tertiary disease
prevention.

One of the components of community-based care is selfcare. Which of the following nursing interventions is the
best example of promoting self-care in the community
setting?
A) A nurse arranges for a client who smokes a pack of

cigarettes a day to attend a smoking cessation program


in his neighborhood
B) A nurse locates physical therapy facilities for a client
who is recovering from hip replacement surgery
C) A nurse considers the cultural preferences of a client
who has diabetes when helping to prepare a meal plan for
the client
D) A nurse helps a client with end-stage liver cancer who
is filling out an advanced directive to make informed
health care decisions
D
Rationale: Empowering individuals to make informed health
care decisions is an essential component of self-care. One
way of empowering individuals is through advance
directives that allow clients to participate in decisions
about their care or refuse treatment. Arranging a smoking
cessation program for a client is an example of preventive
health care. When providing care within the context of the
community, the nurse considers the culture, values, and
resources of the client, the family, and the community.
Collaborative care among health care professionals is an
essential component of community-based care rather than
self-care. Arranging for needed services is an example of
this continuity of care.

A nurse caring for clients in a community health care


clinic assesses clients on a daily basis for various health
risks. Which of the following is a recommended guideline
for these assessments?
A) The nurse should immediately refer clients suspected
of being a victim of intimate partner violence to a
professional for counseling
B) The nurse should recommend weight control if the
client has a BMI of 30 or more or a BMI of 25-29 and two
or more weight-related health problems
C) The nurse should not assess sexual health unless the
client states a concern about having a sexually
transmitted disease
D)The nurse should not assess the risk for suicide for a
client suspected of having depression to prevent planting
the idea in the client's mind
B
Rationale: The nurse should approach the topic of weight
control for clients who have a BMI of 30 or more and two or

more weight-related health problems. The nurse should not


be afraid of assessing a client for sexually transmitted
diseases or suicide risk. The nurse should also use refined
assessment skills to assess for IPV.

The nurse providing care for culturally diverse clients


acknowledges the biological variations for each racial or
ethnic group in the community. Which of the following is
an example of a biological variation as a factor in cultural
assessment
A) Body language
B) Personal space
C) Time orientation
D) Skin color
D
Rationale: Biological variations are the biological
differences among racial and ethnic groups, which include
skin color, and physiologic variations, such as specific
disease processes. Body language is an aspect of
communication. Time orientation and personal space are
separate phenomenon of cultural assessment.

A nurse is teaching parents about the recommended


hours of sleep per night for their infants. Which of the
following statements accurately describes the amount of
sleep needed by a particular age group?
A) Most toddlers sleep around 8-10 hours per day
B) Most school-aged children need 8 hours of sleep per
night
C) Newborns generally sleep 16-20 hours a day
D) Most adolescents need 7 hours of sleep a night
C
Rationale: Newborns generally sleep 16 to 20 hours per
day. Toddlers sleep around 10 to 13 hours per day. Most
school-age children need 10 hours of sleep at night and
adolescents need 8 to 9.5 hours of sleep at night.

A nurse is providing free cholesterol screening for adults


in a neighborhood clinic. Which of the following clients is
most likely to have higher cholesterol?
A) A person who has just lost weight
B) A female who is going into menopause

C) A male in his 30's


D) A female with a musculoskeletal disorder
B
Rationale: People at a higher risk for high cholesterol levels
are middle-age men, women just before menopause, and
anyone who has just gained weight.

A home health care nurse is performing a psychosocial


assessment of a new client who was discharged following
delivery of a newborn who has Down syndrome. Which of
the following questions is the best example of a
psychosocial assessment question?
A) Who makes the decisions in your home?
B) Do you have a smoke detector on every level of your
home?
C) Are there any family rituals passed down from
generation to generation?
D) Who provides you with strength and hope?
A
Rationale: Psychosocial assessments are performed to
understand the client in the context of the family. An
aspect of this type of assessment is asking about the family
structure and who makes the decisions in the family. Family
rituals are an aspect of cultural assessment. Assessing for
smoke detectors is an environmental assessment. Asking a
client who provides strength and hope is a spiritual
assessment.

A client has a network of health care providers that


deliver services at a lower fee in return for prompt
payment at a pre-negotiated price. This is an example of
which of the following types of group insurance plans?
A) Health Maintenance Organization (HMO)
B) Private Insurrance Plan
C) Third-Party Payment Plan
D) Preferred Provider Organization (PPO)
D
Rationale: Preferred provider organizations allow a network
of providers to deliver services at a lower fee in return for
prompt payment at pre-negotiated rates. Health
maintenance organizations are prepaid, structured
managed systems in which providers deliver a

comprehensive range of health care services to enrollees.


Private insurance may obtained through large nonprofit
organizations or from small, private, for-profit insurance
companies as third-party payment.

A home health care nurse visits the home of an 82-yearold female client who is receiving nursing care and
physical therapy following discharge from the hospital.
The client has right-sided paralysis following a TIA. Upon
assessment, the nurse finds that the client has lost five
pounds since the last visit a week ago. The client is living
alone with daily visits from her two children. Which of the
following would be the most important assessment
question for this client?
A) Are your children still visiting you daily?
B) Who is preparing your meals for you?
C) Are you able to walk to the kitchen to get a snack?
D)How has your physical therapy affected your appetite?
B
Rationale: Assessment is a continuous process designed to
collect information used for an immediate intervention or
as a foundation for additional assessments at a later time.
After assessing this client for weight loss, the nurse would
make further assessments to determine if the client needs
help with grocery shopping, meal planning, and cooking
meals. Tracking the client's eating pattern and intervening
where appropriate are the most important assessments in
this situation.

In which of the following facilities would admission most


likely include an assessment focusing on functional
abilities and orientation to new surroundings?
A) Nursing home
B) Acute care setting
C) Physician's office
D) Psychiatric office
A
Rationale: In the nursing home, the nursing history and
assessment focuses on functional abilities and orientation
to new surroundings. In an acute care setting, the
admission focuses on a complete nursing history, vital
signs, and other physical assessments. In the physician's
office, admission assessment focuses on the reason(s) for

seeking medical care. The psychiatric facility admission


focuses on introduction and mental health evaluation, as
well as orientation to room and unit.

A nurse recommends that a pregnant woman consume


600 ug of folic acid per day. This nurse is providing a
preventive intervention for what infant complication of
pregnancy?
A) Low Birth Weight (LBW)
B) Fetal Alcohol Syndrome
C) Failure to Thrive
D) Neural Tube Defects
D
Rationale: Approximately 50% of all neural tube defects
may be prevented with adequate consumption of folic acid
in the first trimester of pregnancy. A pregnant woman who
smokes is more likely to have an LBW infant. Failure to
thrive has various causes, and abstaining from alcohol is a
preventive measure for fetal alcohol syndrome.

The nurse who is caring for clients in a physician's office


knows that which of the following clients is engaging in
the activity that most negatively affects health?
A) A client who is 20 pounds overweight
B) A client who regularly has three alcoholic beverages
before dinner
C) A client who does not engage in physical activity
D) A client who smokes a pack of cigarettes a day
D
Rationale: Smoking remains the health indicator that most
negatively impacts health. Drinking, living a sedentary
lifestyle, and overeating are also risk factors for health
disorders, but smoking has the greatest impact on health.

A nurse is assessing a family structure using the family


systems theory. Which of the following is true regarding
the principles of this family social system framework?
A) Family systems tend to move from a state of
equilibrium to allow for developmental changes in the
family members
B) The family system has a boundary that is selectively
semi-permeable according to the family's wishes

C) The family with closed boundaries is more apt to use


community services than a family with semi-permeable
boundaries
D) Nurses may assist families to return to a state of
equilibrium, but the system does not allow for families to
move to a higher level of health
B
Rationale: The family system has a boundary that is
selectively permeable according to the family's wishes, so
items such as material goods, people, and information are
allowed in or out according to the perceived needs of the
family. The family with closed boundaries is less apt to use
community services than the family with semi-permeable
boundaries. Family systems tend to move to a state of
equilibrium, and nurses may assist families to equilibrium
and sometimes to a higher level of health.

A nurse is caring for an 87-year-old female client who is 5'


7" and weighs 140 pounds. What is the client's Body Mass
Index (BMI)?
A) 21.9
B) 26.9
C) 25.0
D) 23.5
A
Rationale: BMI = weight (pounds) x 703 divided by height
squared (inches squared). In this case the client is:
140 lbs. x 703 = 98420 lbs.
Height squared = 67 inches x 67 inches = 4489 inches
98420 lbs. 9 4489 inches = 21.9 BMI

Nurses working in community-based settings often


concentrate on preventive care for their clients. Which of
the following is an example of an intervention based on
primary prevention?
A) A nurse initiates a program to start free blood pressure
screening in the neighborhood
B) A nurse promotes a new exercise program designed for
older adults in the community
C) A nurse performs range-of-motion exercises for a client
with debilitating arthritis
D) A nurse recommends a colonoscopy for a client with a
family history of colorectal cancer

B
Rationale: Primary prevention is used to prevent the initial
occurrence of a disease, such as preventing obesity or
heart disease through exercise. Screening for hypertension
and recommending a colonoscopy are secondary
preventive measures. Performing range-of-motion exercises
to improve the mobility of a client with arthritis is an
example of tertiary prevention.

A nurse writes a proposal to fund and staff a mobile


health clinic in a community with an at-risk low-income
population. This project is an example of which of the
following public health interventions?
A) Advocacy
B) Coalition Building
C) Outreach
D) Case Management
C
Rationale: Outreach locates populations of interest or at
risk and provides information about the nature of the
concern, what can be done about it, and how services can
be obtained. Case management is working with individual
clients and families to optimize self-care capabilities.
Advocacy is pleading someone's case or acting on
someone's behalf. Coalition building is promoting and
developing alliances among organizations for a common
purpose.

The nurse working with families in the community uses


Maslow's Hierarchy of Needs theory when assessing
family function. Which of the following is an example of
the lowest level of needs that must be met before the
person can move on to higher-level needs?
A) A mother breastfeeds her infant
B) A mother goes back to school to obtain her GED
C) A father praises his son for making his school's Dean's
List
D) A father makes sure his daughter wears a helmet when
riding a bike
A
Rationale: The basic physiological needs (food, fluids,
shelter, sleep, oxygen) must be met first. Safety (bicycle

helmet), self-esteem (parental praise), and selfactualization (attaining a GED) are higher-level needs that
can be met after physiological needs are met.

Community health care nurses provide teaching to clients


based on three levels of prevention. Which of the
following is an example of client teaching related to the
secondary level?
A) A nurse teaches the parents of a newborn who
experienced RDS to place their baby on his back to sleep
to minimize the risk of SIDS
B) A nurse provides information on anorexia to parents of
a teenager diagnosed with the disorder
C) A nurse volunteers at a free mobile clinic to immunize
vulnerable populations in the community against
influenza
D) A nurse refers a client with Parkinson's disease to a
support group in the community
A
Rationale: Teaching occurs at all levels of prevention.
Teaching primary prevention involves preventing the initial
occurrence of disease or injury (e.g., giving immunizations).
Teaching secondary prevention targets early identification
of, and intervention for, a health condition (e.g., SIDS).
Tertiary prevention teaching involves attempts to restore
health and facilitate self-care management and coping with
a health condition (e.g., encouraging support groups and
providing information on eating disorders).

A nurse is caring for an 80-year-old male client who


admits drinking a six-pack of beer a night to "help with
the loneliness." The nurse should screen for what other
co-morbid condition related to this client's alcohol use?
A) Depression
B) Diabetes
C) High-risk behaviors
D) Tobacco use
A
Rationale: Because of the comorbidity of alcohol abuse and
depression, the nurse should screen for alcohol abuse in all
cases where depression is expected.

A nurse is assessing a frail elderly male client for


placement using the Blaylock Discharge Planning Risk
Assessment Screen and gives the client a score of 15.
What would be the nurse's next intervention related to
this score?
A) Discharge the client to his home
B) Refer the client to the discharge planning coordinator
C) Refer the client to a social worker
D) Transfer the client to an assisted living facility
B
Rationale: On the Blaylock Discharge Planning Risk
Assessment Screen, a score of 11 to 19 places the client at
risk for extended discharge planning. A score of 10 means
the client is at risk for home care resources. A score over
20 means the client is at risk for placement other than
home.
If the score is greater than a 10 refer the patient to the
discharge planning coordinator or discharge planning team

The nurse caring for clients in a nursing home knows that


falls are a leading cause of death among the elderly.
Which of the following statistics accurately states the
incidence and consequences of falls in the elderly?
A) In the United States, 1 of every 3 people over age 65
falls each year
B) Half of people over age 75 who fall and fracture a hip
die within 2 years
C) Exercise in the elderly is not recommended due to the
risk for falling
D) By 2020 the cost of fall injury is expected to rise to 50
billion dollars
A
Rationale: In the United States, one of every three people
over age 65 falls each year. Half of people over age 75 who
fall and fracture a hip die within a year. By 2020 the cost of
fall injury is expected to rise to 32 billion dollars. Regular
exercise in the elderly is recommended, as it can
strengthen muscles and stamina and prevent osteoporosis
and consequent falls.

A nurse is caring for a 45-year-old client from Japan who


underwent a stent placement following a myocardial

infarction. The client only speaks Japanese and the nurse


calls in an interpreter to work with them. Which of the
following is a recommended guideline for working with
interpreters?
A) Ask several questions at once and allow the interpreter
to convey them to the client
B) If possible, a relative of the client should be asked to
interpret
C) In possible, an interpreter of the same gender as the
client should be used
D) Request that the interpreter translate exactly what is
said, without interrupting
C
Rationale: In general, an interpreter of the same gender as
the client is preferred. The nurse should ask one question
at a time and allow the interpreter time to convey the
question. Qualified professional interpreters instead of
relatives should be used. The interpreter should be invited
to ask for clarification as needed, even if it means
interrupting.

The nurse working in the community-based setting


follows the nursing process when providing client
teaching. Which of the following steps of the learning
process provides for reimbursement of the teaching
provided?
A) Evaluation
B) Documentation
C) Planning
D) Assessment
B
Rationale: Documentation of teaching is important as a
legal record, as communication of teaching and learning for
other health care professionals, and for determination of
eligibility for care needed and reimbursement for care and
teaching provided.

A school nurse is teaching safety issues in a lecture for


parents of school-age children. Which of the following
topics would the nurse describe as having the biggest
impact on preventing loss of life and injury in children?
A) Using appropriate automobile restraints

B) Learning basic life-saving skills


C) Using smoke detectors in the home
D) Locking up poisonous substances
A
Rationale: Although all these topics are important to cover,
the one topic that prevents the most loss of life and injury
is using appropriate restraints when riding in an
automobile. Child safety seats reduce the risk of death in
passenger cars by 71% for infants and 54% for preschool
children.

The nurse explains to the novice nurse how communitybased nursing differs from nursing care provided in
hospital settings. Which of the following statements
accurately describes one of these differences?
A) In the community setting, the nurse is in charge of all
aspects of client care whereas in the hospital setting the
client directs the care
B) In the community setting, the nurse is the primary
facilitator of self-care as opposed to being solely a care
provider
C) In the hospital setting, a holistic assessment is
facilitated by the collaboration of many professionals,
which is not available in the community setting
D) In the community setting, the client's environment
must be changed to facilitate care, which is not an issue
in the hospital setting
B
Rationale: Because the client and family are involved in
nearly all aspects of care the majority of the time, the
community-based nurse is primarily a facilitator of self-care
as opposed to being the sole care provider. In the
community setting, the delivery of care must be considered
within the family and environment. A holistic assessment
often occurs through the collaboration of many
professionals.

A nurse asks a homebound client what type of activities


she enjoys to determine if these activities can be brought
to the client. Which of the following aspects of the client's
environment is the nurse assessing?
A) Physiologic and survival needs
B) Self-esteem and self-actualization needs

C) Love and belonging needs


D) Safety and security needs
B
Rationale: An environmental assessment assesses for selfesteem and self-actualization needs, such as activities the
client enjoys. An example of a love and belonging need is
having a friendly, competent caregiver. Physiologic and
survival needs are physical needs that need to be met,
such as food and shelter. Safety and security needs include
mobility and fall prevention.

A client who is being discharged from an acute-care


facility sits down with the nurse and agrees how, when,
and where outpatient treatments will occur. What is the
term for this type of agreement?
A) Concordance
B) Collaboration
C) Adherence
D) Self-efficacy
A
Rationale: Concordance is an agreement between the
nurse and the client about whether, when, and how
treatments will occur. Adherence refers to the client
following the prescribed treatment. Self-efficacy refers to
the ability of the client to influence events that affect his or
her life. In the collaborative process, the client chooses a
goal, and the nurse and client negotiate a specific plan to
reach that goal.

A family nurse assesses a family to determine its


developmental stage. The nurse notes that the family's
first child is moving out of their home. What
developmental stage does this represent?
A) Aging family
B) Middle-aged parents
C) Launching center
D) Teenage
C
Rationale: The launching center stage of the family life
cycle occurs from when the first to the last child leaves
home. The "teenage" stage occurs when the oldest child is
13 to 20 years old. The middle-aged parents stage occurs

from empty nest (when no children are living at home) to


retirement. The aging family stage occurs from retirement
to moving out of home.

The nurse practicing in a community setting is aware that


as new groups enter the community, acculturation may
occur. Which of the following is an example of this
concept?
A) A Portuguese family moves into a neighborhood that is
predominantly Portuguese to continue their cultural
practices
B) An Asian couple refuses to allow their children to play
with the neighborhood children
C) The children of a traditional Muslim family chooses to
practice the norms and values of the community as a
whole
D) An Ethiopian man who came to America on a work visa
learns to take the bus to work
D
Rationale: Individuals within a group may adhere to the
traditional culture, as occurs with the Portuguese and Asian
family. Or, they may choose to adapt to the dominant
culture in varying degrees through acculturation, as is seen
with the Ethiopian man. The children of the Muslim family
have assimilated into the existing culture.

A nurse is teaching elderly clients in a senior center how


to modify their homes to prevent falls and promote safety.
Which of the following is a recommended guideline when
providing community-based teaching to the older learner?
A) Face the client and speak in a low, slow voice so lipreading is possible
B) Meet in a quiet, dimly lit room where there is no
background noise
C) Encourage dependent decision making to ensure
family and/or caregiver support
D) Relate new information to the current time frame as
opposed to past experience
A
Rationale: The nurse should face the client and speak in a
low, slow voice so lip-reading is possible. The nurse should
relate new information to past experiences if possible and
encourage independent decision making to support ego

integrity. The nurse should meet the client in a quiet, welllit room with no background noise.

Nurses use the assessment step of the nursing process


when planning client care. Which of the following best
describes the focus of client assessment in the
community-based setting?
A) Providing ongoing monitoring of acute conditions and
planning appropriate nursing interventions
B) Providing a baseline to evaluate physiologic,
psychological, and functional capacity of the client
C) Ensuring continuity of discharge planning based on the
functional capacity of the client
D) Determining the pathology of the client's disease state
and initiating therapeutic interventions
B
Rationale: In the hospital and community setting,
assessment is a dynamic, ongoing method that uses
observations and interactions to collect information,
recognize changes, analyze needs, and plan care. In the
community, the nurse uses assessment to provide a
baseline to evaluate the physiologic, psychological, and
functional capacity of the client and identify environmental
factors that may affect the client's health status. In the
hospital setting, assessment is used to provide ongoing
monitoring of acute conditions and plan appropriate
interventions, as well as to ensure continuity of discharge
planning. Physicians primarily use assessment to
determine pathology and appropriate therapeutic
interventions.

The nurse researches current health trends and statistics


in the United States to identify vulnerable populations.
Which of the following is a critical indicator of the health
of a population?
A) Elderly poverty rates
B) Infant mortality rates
C) Child poverty rates
D) Percentage of individuals with chronic diseases
B
Rationale: Infant death is a critical indicator of the health of
a population because it reflects the overall state of
maternal health, as well as the quality of and access to

primary health care for pregnant women and infants. Infant


mortality in the United States ranks the highest among
industrialized nations.

A nurse working in a community health care clinic travels


around the community to identify data related to the
people, places, and social systems that define that
community. The nurse uses this information to improve
the health of the individuals using the clinic. What type of
assessment tool is this nurse using?
A) Informant interviews
B) Participant observations
C) Windshield survey
D) Existing data
C
Rationale: A windshield survey is designed to assist the
nurse traveling the neighborhood to identify data related to
the people, places, and social systems that define that
community. This information may help identify trends,
stability, and changes affecting the health of the
community members. Informant interviews involve
interviewing key informants or members of the general
public. During participant observations the nurse observes
formal and informal community activities to determine
significant events and occurrences. Existing data is used to
assess people, places, or social systems.

A female nurse is providing care for a 29-year-old male


Muslim client who fractured his pelvis in an automobile
accident. What dimension of cultural knowledge should
the nurse keep in mind when providing care for this
client?
A) In the Muslim culture, physical touching and expression
is valued and expected
B) In the Muslim culture, close face-to-face conversations
are the norm
C) In the Muslim culture, it is highly distressing for a male
client to be assessed by a male nurse
D) In the Muslim culture, contact between males and
females who are not married to each other is
inappropriate
D
Rationale: In the Muslim culture, contact between unrelated

males and females is forbidden. In Muslim culture, it would


be distressing for a male nurse to care for a female patient,
but not for a male patient. Close face-to-face conversations
are the norm In Middle Eastern cultures, and physical
touching and expression is valued in the Mexican culture.

A nurse is preparing for a next-day home visit of a client


discharged from a birthing center 12 hours after a normal
delivery of a healthy baby girl. Which of the following is a
characteristic of home care that differs from nursing care
in the hospital setting?
A) The focus is on the client individually
B) Family support is critical in achieving client outcomes
C) The resources needed for this client are
predetermined
D) The client will assume a highly dependent role
B
Rationale: In the home setting, family support is critical in
accomplishing client outcomes, whereas in the hospital
setting family support is helpful in accomplishing client
outcomes but not a necessity. The resources for hospital
stays are predetermined, but are highly variable in the
home setting. In the home setting the client assumes an
autonomous role as opposed to a dependent role. The
focus of home care is on the client in the family setting, not
just the client individually in a hospital setting.

A nurse is counseling a 35-year-old male client who has a


BMI of 35 (obese) and who has type 2 diabetes. Which of
the following is the best example of assessing this client's
readiness for a weight-control program?
A) What changes are you ready to make in your eating
habits right now?
B) Are you embarrassed by the way you look and feel
right now?
C) Are you aware that being obese is a risk factor for type
2 diabetes?
D) Do you feel you have lost control over your eating
habits?
A
Rationale: The nurse should ask questions to assess a
client's readiness to control weight, such as what changes
he is willing to make in eating habits or physical activity

level. The nurse should address the client's chief


complaints independent of weight first and avoid
judgmental comments such as "lost control" or
"embarrassed".

A nurse is preparing a pamphlet for distribution in the


community regarding sexually transmitted diseases
(STDs) in teenagers and young adults. Which of the
following is a point that should be covered in this
pamphlet?
A) Most new HIV infections occur in people between 13
-21 years of age
B) Teens and young adults are less likely than others to
have multiple partners
C) Comprehensive sex education should begin in high
school
D) Talking openly about sex with teens makes them more
likely to try it
A
Rationale: The incidence of STDs has skyrocketed and most
new HIV infections occur in people between 13 and 21
years (National Center for HIV/AIDS, 2010). Teenagers and
young adults are more likely than others to have multiple
partners. Talking openly and frankly about sex with teens
may allow them to see how it is a choice to be sexually
active that need not be made at this age. Comprehensive
sex education should begin in primary school.

Nurses accomplish the goals of health promotion by


working with individuals in all types of care settings.
Which of the following is an example of this type of health
promotion?
A) A nurse participates in a campaign to create a law that
all bikers wear safety helmets
B) A nurse volunteers at a free health clinic to provide
immunizations for low-income families
C) A school nurse provides rallies for students to show
how eating healthy and exercising can be "fun" activities
D) A nurse counsels teenagers on methods to prevent
contracting sexually transmitted infections
C
Rationale: Health promotion is the science and art of
helping individuals change their lifestyle to move toward a

state of optimal health. This is accomplished through


creating supportive environments to enhance awareness
and change behavior to support good practices, such as
eating healthy and exercising. Providing immunizations and
counseling about preventing STIs are examples of disease
prevention. Helping to create a helmet law is an example of
a health protection strategy.

A nurse is assessing a 2-year-old male child for vision


disorders. Which of the following examples of assessment
techniques would be appropriate for this client?
A) Ask the mother if the child returns her smile
B) Ask the child if he can see pictures in a book
C) Ask the mother if the child tilts his head to look at
things
D) Observe whether the child's eyes follow the mother
when she leaves the room
C
Rationale: Nurses should assess the vision of infants and
toddlers by asking the mother certain assessment
questions. For a toddler, the nurse should ask the mother if
the child tilts his head to look at objects, covers an eye
when looking at objects, squints, frowns or blinks
frequently, or holds objects very close or far away to look
at them. The other assessments are appropriate for an
infant older than 6 weeks.

The nurse caring for elderly clients in a nursing home


researches nursing interventions aimed at health
promotion and disease prevention for the elderly. Which
of the following statements accurately reflects the current
health status of older adults in the United States?
A) The elderly population in the United States is growing
B) Since 1900, the percentage of people over 65 has
doubled
C) As people age, they are more likely to live with a
partner or family member
D) People who live to age 65 can expect to live around 10
more years
A
Rationale: The elderly population in the United States is
growing. Life expectancies at ages 65 and 85 have
increased over the past 50 years. People who live to age 65

can expect to live an average of nearly 18 more years.


Since 1900, the percentage of people 65 years and older
has tripled with the aging of baby boomers. As people age,
they are more likely to live alone.

During an annual physical, the nurse is reviewing the


immunization record of a 22-year-old female client. The
client has no record of immunizations and does not
remember being immunized. Which of the following is a
recommended immunization for this client?
A) One dose of influenza vaccine annually
B) 4 doses of Measles, Mumps and Rubella (MMR)
C) 2 doses of Tdap, then boost with Td every 10 years
D) 2 doses of HPV vaccination
A
Rationale: The 19- to 26-year-old should have one dose of
influenza vaccine annually, 3 doses of HPV (if female), 1
dose of Tdap boosted with Td every 10 years, and 1 or 2
doses of MMR.

The nurse is counseling parents about common chronic


diseases of childhood and their prevention. Which of the
following would the nurse state as the single most
common chronic disease in children?
A) Dental caries
B) Juvenile diabetes
C) Ear infections
D) Asthma
A
Rationale: Dental caries represent the single most common
chronic disease of childhood, occurring five times as
frequently as the second most common disease (asthma).
Unless identified and cared for early, caries are irreversible.

An African American nurse is providing care in a health


clinic in a primarily Hispanic neighborhood. What is the
first step in intervening appropriately with clients from
another culture?
A) Practicing cultural blindness
B) Promoting ethnocentrism
C) Preventing stereotyping
D) Practicing cultural awareness

D
Rationale: The first step in providing culturally competent
care is to understand one's own cultural background,
influences, and biases. Only with cultural awareness can
nurses appreciate and be sensitive to values, beliefs, life
practices, and problem-solving methods of a client's
culture. Cultural blindness occurs when the nurse does not
recognize his or her own beliefs or practices, or those of
others. Ethnocentrism is the idea that one's own ideas or
beliefs are the best way to behave, which has a negative
effect on cultural awareness. Preventing stereotyping
(generalizing about others) is important, but is not the first
step in this process.

A nurse providing care in a community health care clinic


initiates a grant proposal to help identify pregnant women
in the community who are not receiving prenatal care and
provide this care on a sliding scale based on income. In
which of the following nursing interventions is this nurse
involved?
A) Health teaching
B) Consultation
C) Collaboration
D) Case finding
D
Rationale: Case finding is a set of activities nurses working
in community settings use to identify clients who are not
currently receiving nursing care but could benefit from it.
Health teaching is the communication of information and
skills that change the knowledge and attitudes of clients
and their families. Collaboration involves two or more
individuals or agencies to achieve a common health goal
for a client. Consultation is an interactive problem-solving
process between the nurse and client.

The nurse providing teaching in the home care setting


knows that in order to be reimbursed by Medicare or
third-party payors the teaching must meet certain
criteria. Which of the following is an example of a
questionable teaching activity that may not be paid?
A) A nurse teaches a diabetic client how to inject insulin
B) A nurse teaches a client with a broken neck how to
care for a halo device

C) A nurse teaches a client how to organize multiple


medications into a pill box
D) A nurse teaches a client with a new colostomy how to
care for the stoma
C
Rationale: Questionable teaching includes the
administration of oral medications or teaching for the client
who is capable of independent ambulation, dressing,
feeding, and hygiene. Teaching care for a new colostomy
and administration of insulin is generally covered, as well
as teaching the patient how to use and care for braces,
splints, and orthotics.

A nurse is teaching a new mother about the advantages


and disadvantages of breastfeeding a newborn. Which of
the following would the nurse report?
A) Breastfeeding should be supplemented with iron-rich
formula to prevent anemia
B) Breastfeeding decreases the rate of respiratory
infections in newborns
C) Breastfeeding must be monitored as the newborn is
more likely to be overweight
D) Breastfeeding prolongs the return to pre-pregnancy
weight in the mother
B
Rationale: Breastfeeding is the most complete form of
nutrition for infants, with various benefits including health,
growth, immunity, and development. Breastfed babies
have a lower incidence of diarrhea, respiratory infections,
and ear infections and are less likely to be overweight.
Breastfeeding promotes the return of pre-pregnancy
weight. Breastfeeding should not be supplemented with
formula feeding.

A nurse is using surveillance to improve the health of a


culturally diverse community. Which of the following
interventions might this nurse perform?
A) Collecting and analyzing health data to plan and
implement health care services in a free clinic
B) Placing ads in the local newspaper to announce a new
recreation program for the community population
C) Interacting with community members in a town
meeting to provide health screening to uninsured children

D) Identifying individuals in the community with


unrecognized health risk factors, such as homeless
people
A
Rationale: Surveillance is used to describe and monitor
health events through ongoing and systematic collection,
analysis, and interpretation of health data. This process is
used to plan, implement, and evaluate public health
interventions. Identifying individuals in the community with
unrecognized health risks factors, such as homeless people
is a function of screening. Interacting with the community
to solve problems such as uninsured children is a function
of consultation. Using public marketing is social marketing.

Some of the leading causes of death and disability in the


United States can often be prevented by making lifestyle
changes. Staying active, eating right, and not smoking
are preventive measures for which of the following
leading causes of death in the United States?
A) Heart disease
B) Unintentional injuries
C) HIV/AIDS
D) Mental disorders / Suicide
A
Rationale: About two thirds of all mortalities and
morbidities result from three behaviors: tobacco use, poor
dietary patterns, and smoking. Heart disease and cancer
are the leading causes of death in adults, and staying fit,
eating right, and not smoking are key strategies to
preventing these diseases.

The nurse uses the health-illness continuum as a model


for practice in the community-based health care setting.
Which of the following accurately represents the
philosophy of this model of health?
A) Improvement in health is seen as an outcome related
to available medical services
B) technology drives care in this model
C) It considers health rather than illness as the essence of
care
D) Care is directed at resolving immediate health
problems

C
Rationale: The health-illness continuum considers health
rather than illness as the essence of care, which requires a
shift in thinking. Improvement in health is not seen as an
outcome of the amount and type of medical services or the
size of the hospital. Treatment efficacy rather than
technology drives care in this model. Care provided in
acute care facilities is directed at resolving immediate
health problems, whereas in this model the focus is on
maximizing individual potential for self-care.

The nurse caring for clients in a nursing home is advising


a new resident couple regarding safe sex practices. Which
of the following is an appropriate teaching point for this
couple?
A) Both partners should be tested for STD's prior to
having sex
B) In the United States, the incidence of HIV is minimal in
the older population
C) Older men should not be encouraged to use
performance-enhancing drugs
D) As men get older, impotency is not a problem unless it
was in the past.
A
Rationale: Having safe sex is important for people of all
ages, and both members of new couples should be tested
for STDs prior to having sex. In some areas of the country,
HIV is on the rise in the elderly population. As men age,
impotence increases with some chronic diseases
contributing to the cause. Pharmaceutical and mechanical
options are available to enhance sexual enjoyment.

The nurse formulates learning objectives for clients being


treated in a long-term health care facility. Which of the
following is an example of an affective learning objective?
A) The client will state three healthy snacks to substitute
for sweets by 10/24/12
B) The client will demonstrate the proper method of
crutch walking by 10/24/12
C) The client will express a desire to start a smoking
cessation class by 10/24/12
D) The client will ambulate the length of the hallway by
10/24/12

C
Rationale: An affective learning objective relates to learning
activities that enhance the acceptance of and the
adjustment to illness and subsequent treatment (e.g. the
client expressing a desire to stop smoking). Stating three
healthy snacks to substitute for sweets is a cognitive
objective. Ambulating the length of a hallway and
demonstrating crutch walking are psychomotor learning
objectives.

The nurse working in a hospital knows that the client's


diagnosis is categorized according to the federal DRG
coding system instituted in the early 1980s. Which of the
following is an outcome of this prospective payment
system?
A) Improved efficiency means less revenue for the
hospital
B) The length of client hospital stays has increased
C) More and more health services are being provided
outside the hospital
D) Hospital readmission for clients has decreased
C
Rationale: Over time, more and more services are being
provided outside the hospital because it is more costeffective. Improved efficiency means more revenue for the
hospital because of shorter stays and use of fewer
resources. Since shorter stays are financially favorable for
hospitals, clients are discharged "quicker and sicker,"
resulting in an increase in readmissions.

It is important for the community-based nurse to assess


the family or kinship patterns of the groups within the
community being served. Which of the following is the
best method to assess family structure?
A) Ask the client who is the head of the family
B) Ask the client who is in his or her family
C) Describe the typical family and ask the client how his
or her family differs
D) Ask the client how many parents and how many
children comprise the family
B
Rationale: The family is the basic unit of society, but there

are many meanings of "family." In order to avoid


misunderstanding, it is appropriate for the nurse to ask the
client directly who is in his or her family. The nurse should
note if the client views "family" as only the nuclear family
or also includes the extended family.

A nurse provides a referral to an occupational therapist


for an elderly client with limited mobility related to
rheumatoid arthritis. Which of the following is an example
of a service that would be provided by this specialist?
A) Teaching a client how to use a grabber to reach higher
objects
B) Teaching the client how to use a walker to ambulate
C) Performing range-of-motion exercises to strengthen the
client's muscles
D) Assisting the client to take a shower and get dressed
A
Rationale: An occupational therapist helps the client adjust
to limitations by teaching new vocational skills or better
ways to perform activities of daily living. Performing range
of motion exercises and teaching the use of a walker are
roles of the physical therapist. Assisting the client with
hygiene is a role of the home health aide.

Disparities in American health care are well documented,


and numerous federal initiatives have been undertaken to
reduce these disparities. Which of the following is the
best example of two factors that are intrinsically related
and underlie many of these health disparities?
A) Culture and health behaviors
B) Environment and genetics
C) Gender and religion
D) Economics and education
D
Rationale: Income and education are intrinsically related
and underlie many of the heath disparities (lack of equity)
in the United States. People with the worst health status
are among those with the highest poverty rates and least
education. Environment, genetics, and health behaviors
also play a role in the major causes of death in the United
States. Gender, religion, and culture may influence health
status, but are not major causes of the disparity in health
care.

A nurse is teaching safety guidelines to a group of


parents participating in a parenting class. Which of the
following teaching points follows the recommended safety
guidelines for child safety?
A) Keep hot water temperatures below 120 F
B) Use a car safety seat until the child is at least 30
pounds
C) Keep the child safety seat in the front passengers side
seat
D) Be sure their are screens in all windows
A
Rationale: The hot water temperature should be set at 120
degrees F. A child safety seat should be used until the child
is at least 40 lbs and should be placed in the middle of the
back seat. Window guards and netting should be used to
prevent falls from windows. Screens alone do not always
prevent a fall.

A nurse is caring for a 48-year-old male client recently


diagnosed with lung cancer. The client has a wife and two
children living at home. Which of the following is
considered the fundamental component to providing
continuity of care for this family?
A) Referring the client to cancer specialists
B) Establishing a trusting nurse-client relationship
C) Assessing the client's financial resources
D) Teaching the client self-care measures
B
Rationale: The nurse-client relationship is the fundamental
component in developing continuity of care. Counseling is
established through the nurse-client relationship and
woven into all aspects of the plan as the foundation of care
coordination. The other options are aspects of the care plan
that may need to be addressed, but are not considered the
foundation of continuity of care.

A home health care nurse discusses the incidence of


adults with limitation in activities of daily living (ADLs)
and instrumental activities of daily living (IADLs) with a
physical therapist. Which of the following statistics
accurately portrays the percentage of adults in the United
States with these types of limitations?

A) Approximately 20% of adults over 75 years old have


limitations in IADLs
B) Approximately 30% of adults over 75 years old have
limitations in ADLs
C) Approximately 10% adults aged 65-74 have limitations
in ADLs
D) Approximately 15% of adults aged 65-74 have
limitations in IADLs
A
Rationale: Approximately 20% of adults over 75 years old
have limitations in IADLS. Approximately 10% of this age
group have limitations in ADLs. About 3% of adults aged 65
to 74 have limitations in ADLs, and about 7% have
limitations in IADLs.

The nurse working in a culturally diverse communitybased setting knows that a person's cultural belief system
influences his or her health attitudes, beliefs, and
practices. Which of the following clients would be more
likely to look for a supernatural counterforce to get rid of
a health problem?
A) An Asian woman who has breast cancer
B) An Eastern European client who is experiencing
seizures
C) A client of Caribbean culture who is diagnosed with
lung cancer
D) An Arab client who is scheduled for a postatectomy
C
Rationale: The magico-religious view, practiced by people
from Hispanic and Caribbean cultures, sees illness as
having a supernatural force. People with this perspective
will look for a counterforce to rid them of the problem.
People of Arab or Asian descent may ascribe to the hot and
cold theory based on the balance of the four humors.

Competencies for nurses working in the community


setting include skill and knowledge in assessment,
program planning, communications, cultural competency,
public health science, management, and leadership.
Which of the following examples of competencies best
reflects the use of public health science skills?
A) A nurse researches the availability of free health care

clinics and transportation for low-income families in a


nearby community
B) A nurse plans a series of parenting classes for an inner
city neighborhood and decides which clients should be
invited to attend
C) A nurse plans and organizes care for an elderly client
who has dementia and is living at home with her
daughter
D) A nurse interviews a client with advanced Parkinson's
disease to determine which care should be initiated,
continued, altered, or discontinued
A
Rationale: Nurses understand factors contributing to health
promotion and disease prevention (such as free health care
clinics) and factors contributing to the use of health
services (such as transportation) by using basic public
health science skills. Planning and organizing care for a
client involves the use of management skills. The nurse
uses program planning skills to plan parenting classes, and
interviewing clients to plan care is related to the use of
assessment skills.

Which of the following interventions might the nurse


perform in the home health care setting when providing
tertiary prevention?
A) Informing a client about smoking cessation classes
offered in the community
B) Referring a caregiver to a program designed to prevent
caregiver burnout
C) Teaching range-of-motion exercises to a client who is
recovering from a stroke
D) Providing teaching materials about diabetes in a
family's native language
C
Rationale: Tertiary prevention focuses on rehabilitation,
management of chronic conditions, or postsurgical care at
home. Informing a client about smoking cessation classes
is a form of secondary prevention. Referrals for classes and
providing teaching materials are examples of prevention on
the primary level.

The nurse is performing a functional assessment on Mrs.


Ruiz, an elderly female client discharged from the hospital
following hip replacement surgery. Mrs. Ruiz is

temporarily living with her daughter and her daughter's


two school-age children, but would like to return to her
own home. Which of the following is the best example of
a question to assess this client for functional status?
A) Have you been experiencing any cramping in your
legs?
B) Are you able to dress yourself independently?
C) Have you noticed any oozing or pain at the incision site
D) How would you rate your pain on a scale of 1 to 10
B
Rationale: Function assessment is performed to determine
if the client needs the assistance of another person for
daily function, such as dressing, bathing, and cooking
meals. Pain assessment, assessment for infection, and DVT
may impact functional ability but are not direct functional
assessment parameters.

A nurse is caring for clients in a mostly Puerto Rican


community. Which of the following is an example of using
a cultural care accommodation modality to treat a
teenage client who is pregnant?
A) Be familiar with religious symbols and protective care
symbols that may be used
B) Develop a nutrition outreach program for expectant
mothers in the community
C) Use the Spanish language to include the teenager's
mother in the care plan
D) Treat the mother and daughter with respect and
maintain eye contact
C
Rationale: Using the Spanish language is a cultural care
accommodation modality, which is an assistive, supportive,
or enabling technique to help people of a certain culture
adapt to others for a beneficial outcome. Treating the
clients with respect and being familiar with their religious
symbols are forms of cultural care preservation and/or
maintenance. Developing a nutrition outreach program is a
cultural care repatterning or restructuring modality.

A nurse is performing a mini nutritional assessment for


78-year-old Mr. Parker. He is in good health (no
psychological distress or recent acute illnesses) and lives
in an assisted-living facility. Upon assessment, the nurse

learns that Mr. Parker has experienced a moderate


decrease in his food intake, has lost 6 pounds in the last 3
months, is able to go out, and has no neuropsychological
problems. His body mass index is 20 and calf
circumference is 30. How would the nurse document Mr.
Parker's score on this assessment tool?
A) 10 points = at risk of malnutrition
B) 13 points = normal nutritional status
C) 7 points = malnourished
A
Rationale: When assessing nutrition using the mini
nutritional assessment tool, a score of 12 to 14 points =
normal nutritional status, 8 to 11 points = at risk of
malnutrition, and 0 to 7 points = malnourished.

The family nurse knows that providing nursing care to


families is a logical development of the holistic approach
to the care of the client. Which of the following accurately
describes a consideration when caring for clients in the
context of their families?
A) A change in one part of a family system does not
usually change other parts of the system
B) Communication patterns between family members do
not affect the natural functioning of the family
C) A family's structure and organization cannot be
understood in isolation from the rest of the family system
D) It is important to first isolate the client from the family
to fully understand the client
C
Rationale: A family's structure and organization, or a part of
that family, cannot be understood in isolation from the rest
of the family system. A change in one part of a family
system affects other parts of the family. Communication
patterns between family members are essential to the
functioning of the family.

A nurse is teaching preventive measures to adults in an


assisted living facility. Which of the following is a
recommended guideline for adult health screening?
A) Women ages 50-74 should have a mammogram every
year
B) Men ages 50-75 should have an annual PSA screening

for prostate cancer


C) People ages 35-75 should undergo a colonoscopy
every 5 years
D) All women should have an annual pap smear starting
at age 18 or when sexually active
D
Rationale: All women should have an annual Pap smear
starting at age 18 or when they become sexually active.
After three normal Pap smears they may be tested less
frequently according to their practitioner. The U.S.
Preventive Services Task Force (2010) recommends that
women ages 50 to 74 have a mammogram every 2 years.
Experts do not universally support the PSA screening for all
men regularly, since the test can be abnormal for reasons
other than prostate cancer. People ages 50 to 75 should be
screened regularly for colorectal cancer, though not
necessarily every 5 years.

A nurse is counseling elderly clients in a mental health


facility. Which of the following statistics should the nurse
keep in mind when devising a plan of care for these
clients?
A) Aging has little or no effect on the emergence of
mental health issues
B) Older Americans are disproportionately likely to die
from suicide
C) Only severe depression responds well to treatment
D) The incidence of depression is lower for clients
receiving home health care
B
Rationale: Older Americans are disproportionately likely to
die from suicide; they make up only 12% of the population,
but account for 16% of suicide deaths. Many issues related
to mental health emerge as people age. This may be
related to loss of health, family, friends, and spouse. The
incidence of depression is higher in clients receiving home
health care and even higher in hospitalized clients. All
depression should be treated, and there are many options
for treatment.
Advocate
...

Assessment

- what is the systematic collection, assembly, analysis, &


dissemination of information about the health of a
community?
- as a community health nurse you must gather & analyze
information that will affect the health of the people to be
served.
- Data may be gathered in many ways which includes:
*Interviewing people
*Conducting surveys
*Public records

Assurance
- what are activities that make certain that services are
provided?
- you must improve the level of "?" what

Case Management
- what is a systematic process by which a nurse assesses
clients needs, plans for & coordinates services, refers to
other appropriate providers, & monitors and evaluates
progress to ensure that clients multiple service needs are
met in a cost-effective manner?

Clinician
...

Collaborator
- what role of a community nurse involves working jointly
w/ others in a common endeavor, cooperating as partners?
- successful community health practice depends on this
multidisciplinary collegiality & leadership.

Policy Development
- what is enhanced by the synthesis & analysis of
information obtained during assessment?
- the nurse recommends specific training & programs to
meet identified health needs of target populations.

Researcher
- this role of the community nurse involves engaging in a
systematic investigation, collection & analysis of data for
solving problems & enhancing community health practice?

Core Public Health Functions

- the various roles & settings for practice hinge on three


primary functions of public health:
*Assessment
*Policy Development
*Assurance
- these are applied at three levels of service: Individuals,
Families, Communities.
- Study (Display 3.1) pg 44.

Roles of Community Nurse: Clinician Role


- what is the most familiar role of the community nurse that
involves being a 'care provider'?
- this role means that the nurse ensures health services are
provided not just to individuals & families, but also to
groups & populations.
-

Roles of Community Nurse: Holistic Practice


- what role of the nurse seeks to be broad & holistic?
- in community health, this approach means considering
the broad range of interacting needs that affect the
collective health of the "client" as a larger system.
- this type of nursing care encompasses the comprehensive
& total care of the client in all areas such as Physical,
Emotional, Social, Spiritual, & Economic.
- the client is a composite of ppl whose relationships &
interactions w/ each other must be considered in totality.

Roles of Community Nurse: Focus On Wellness


- what role of the community nurse is characterized by its
focus on promoting wellness?
- especially emphasizes promotion of health & prevention
of illness.
- effective services include seeking out clients who are at
risk for poor health & offering preventive & healthpromoting services, rather than waiting for them to come
for help after problems arise.
- Ex, immunization of preschoolers, cholesterol screening,
family planning programs, prevention of behavioral
problems in adolescents.

Roles of Community Nurse: Expanded Skills


- what role of the community nurse involves the many
different skills used by the nurse?
- instead of skills being placed solely on physical care skills,
it is now encouraged to expand skills in Observation,

Listening, Communication & Counseling & also to


encompass Sociocultural and Psychological.
- due to environmental & community-wide considerations,
there is a need for stronger skills in assessing the needs of
groups & populations & intervening at the community level.

Roles of Community Nurse: Educator Role


- this role of the community nurse focuses on health
teaching & is a major function of this nurse.
- w/ emphasis on populations & aggregates these teaching
efforts of the community nurse are appropriately targeted
to reach many people.
- community living clients need to know about a widevariety of issues such as Family Planning, Weight Control,
Smoking Cessation, & Stress reduction.
- aggregate level concerns include Environmental safety,
Sexual discrimination or Harassment, Violence and Drugs.

Roles of Community Nurse: Advocate Role


- what role of the community nurse involves pleading for
the clients cause or acting on their behalf?
- every pt has the right to receive just, equal, & humane
treatment.
- Ex, explain which services to expect, which services they
ought to receive, make referrals as needed, & write letters
to agencies or health care providers for them.
- the two main goals of this role are:
*To help pt's gain greater Independence/Self
Determination.
*To make the system more responsive to the needs of
the clients.
- this role is performed via 4 main actions:
*Being Assertive
*Taking Risks
*Communicating/Negotiating
*Identifying Resources
*Obtaining Results.

Roles of Community Nurse: Manager Role


- what community nurse role involves exercising
administrative direction toward the accomplishment of
specified goals by identifying client needs,
planning/organizing to meet those needs, directing &
leading to achieve results, controlling/evaluating the
progress to ensure that goals are met?

Manager Role: Nurse as a "Planner"


- this manager role involves setting the goals & direction
for the organization or project & determines the means to
achieve them?

Manager Role: Nurse as an "Organizer"


- this manger role involves designing a structure within
which people & tasks function to reach the desired
objectives?
- includes deciding on the tasks to be done, who will do
them, how to group the tasks, who reports to whom, &
where decisions will be made.

Manager Role: Nurse as a "Leader"


- this manager role is when the nurse directs, influences, or
persuades others to effect change that will positively
impact peoples health & move them towards a goal.
- includes persuading/motivating ppl, directing activities,
ensuring effective two-way communication, resolving
conflicts, & coordinating the plan.

Manager Role: Nurse as a "Controller & Evaluator"


- a 'controller' monitors the plan & ensures that it stays on
course. an 'Evaluator' compares & judges performance and
outcomes against previously set goals/standards.

Management Behaviors
- these type of behaviors are grouped into 3 sets of
behaviors:
*Decision Making Behavior
*Transferring of Information Behaviors
*Interpersonal Relationships

Management Skills
- the three basic management skills include:
"Human Skills" which refers to ability to understand,
communicate, motivate, delegate & work well with ppl.
"Conceptual Skills" the mental ability to analyze & interpret
abstract ideas for the purpose of understanding &
diagnosing situations and formulating solutions.
"Technical Skills" the ability to apply special managementrelated knowledge & expertise to a particular situation.

Leadership Role
- this leadership role is separate from the manager role &
focuses on effecting change; thus the nurse becomes an

agent of change.
- as leaders, community health nurses seek to initiate
changes that positively effect ppl's health.
Epidemiology
The study of the distribution and the determinants of states
of health and illness in human populations.

Distribution
Refers to the frequency of occurrences of health and illness

Determinants
Refers to agents or factors that contribute to the cause of
various states of health and illness.

Epidemiology ultimate goal


Use the information obtained from the study of the
distribution and determinants of health and illness to
prevent or limit the consequences of illness and disability
in humans and maximize their state of health.

John Snow
Cholera epidemic, linked to the water source in the center
of the city.

Epidemiological triad
Agent (organism like the measles virus), host (human), and
environment

Web of Causation
Based on the belief that health status is multifactorial,
determined by the interaction of many agent, host, and
environment characteristics, and not by any single factor.

Ecological model
Expands epidemiological studies upward into broader
contexts such as neighborhood characteristics and
community and social contexts, and downward to the
genetic molecular level. Model encompasses determinants
at many levels: biological, emotional, behavioral,
socioeconomic, cultural, political, and environmental.

Risk
the probability that a particular event or outcome will occur
within a specified time period.

Genomics
is the study of all the genes in a person, as well as the
interaction of those genes with each other and a person's
environment.

Pre-pathogenesis
factors within individuals and their environments that may
predispose or precipitate the disease. The initial
interactions among agent, host, and environment occur
during this period.

Pathogenesis
begins when the host begins to respond with biological,
psychological, or other changes. Manifested by S&S until
the issue is resolved.

Primary prevention
Activities that prevent a disease from becoming established
and occurs during the pre pathogenesis period.

Secondary prevention
Includes activities designed to detect disease and provide
early treatment.

Tertiary prevention
the treatment, care, and rehabilitation of people with acute
and chronic illness to achieve their maximum potential.
Tertiary prevention is initiated after irreversible changes
have resulted.

Descriptive Epidemiology
Focuses on the frequency and distribution of states of
health within a population. Describe the problem (what
occurs) in terms of person, place , and time.

Risk factors
Factors that are associated primarily with the people who
have the illness.

Ratio
is a fraction that is obtained by dividing one quantity by
another quantity; it represents the relationship between
two numbers.

Proportion
is a type of ratio that include the quantity in the numerator
also as a part of the denominator.

Rate
A measure of frequency of an event or diagnosis in a
defined population within a given time period. (rate is a
proportion that includes the factor of time). best indications
of the probability that a disease, condition, or event will
occur.

Incidence
Measures the occurrence of new illness in a previously
disease-free group of people within a specific time frame.
Measure of the probability that people without a certain
condition will develop the condition over a period of time.

Mortality
Death

Morbidity
illness

Prevalence rates
Measures the amount of morbidity that exists in a
community as a result of the health problem under
investigation.

Period prevalence
indicates the existence of a condition during an interval of
time.

Point prevalence
refers to the existence of a condition at a specific point in
time.

Crude rates
Measure the experience of health problems in populations
of designated geographic areas.

Specific rates

Calculated to describe the distribution of health problems


by age, gender, ethnicity, and other demographic
characteristics.

Adjusted rates
Have been standardized, removing differences in
composition of populations, such as age.

Vital statistics
Data collected from the continuous recording of events
such as births, deaths, marriages, divorces, and adoptions,
usually by state agencies.

Morbidity and Mortality Weekly Report (MMWR)


Published by the CDC is the vehicle for distributing current
information to healthcare professionals.

National Health Survey


Provides information about the health needs of the
population of the United States. Most of this information is
prevalence data and is the only nationwide source of data
on chronic illness, minor conditions, and functional
problems.

Behavioral Risk Factor Surveillance System (BRFSS)


Was established to collect, analyze, and interpret
behavioral risk factor data from all states.

Person
Characterizes the "Who" develops the health problem.

Place
"Where" the rates of the health problem are the highest or
the lowest can be determined by examining the
characteristics of place.

Time
"When" health problems occur can be describe the
identifying short-term fluctuations measured in hours,
days, weeks, or months; buy periodic changes that are
seasonal or cyclical; or by long-term changes over decades
that reflect gradual changes.

Epidemic curve

gives a graphical display of the numbers of incident cases


in an outbreak

Analytic Epidemiology
Focuses on the determinants of health problems, or the
"why". Uses 4 different types of studies: cross-sectional,
retrospective (case-control), prospective, and intervention
(experimental) studies.
What is implied by the web of causation model?
a. Variables interact resulting in higher probability of
illness.
b. One disease causes another, especially in vulnerable
populations.
c. The greater the poverty, the more likely people are to
have diseases.
d. Immunizations are necessary because vulnerable
populations spread disease.
a. Variables interact resulting in higher probability of
illness.

Which is a characteristic of a vulnerable population?


a. Have worse health outcomes and an increased
sensitivity to risk factors than the general population
b. Have a single risk factor but experience worse health
outcomes than the general population
c. Have multiple risk factors but equal health outcomes
than the general population
d. Have worse outcomes with better access to health care
than the general population
a. Have worse health outcomes and an increased
sensitivity to risk factors than the general population

The differential vulnerability hypothesis refers to the:


a. Resistance of certain groups to risk factors
b. Increased susceptibility to cumulative risk factors
among vulnerable groups
c. Variability in the effects of stressors according to
socioeconomic status
d. Increased sensitivity of the very young and the very old
to risk factors
b. Increased susceptibility to cumulative risk factors among
vulnerable groups

Which is an example of a priority population group?


a. People earning more than $100,000/year

b. People earning less than $100,000/year


c. Residents of remote rural areas of Canada
d. African American physicians
c. Residents of remote rural areas of Canada

An example of a vulnerable group experiencing multiple


risk factors is:
a. Smokers who use chewing tobacco as well as
cigarettes
b. Substance abusers who test positive for human
immunodeficiency virus (HIV)
c. Persons with limited access to care because they live in
a rural area
d. New mothers needing information about baby and child
care
b. Substance abusers who test positive for human
immunodeficiency virus (HIV)

A 17-year-old pregnant cocaine addict who is homeless is


considered:
a. At risk
b. A special population
c. A Healthy People 2020 target group
d. A vulnerable individual
d. A vulnerable individual

The wide variations in health services and health status


between certain population groups are called:
a. Priority population groups
b. Health disparities
c. Disadvantaged populations
d. Risk markers
b. Health disparities

What is the primary cause of vulnerability?


a. Race
b. Age
c. Poverty
d. Illness
c. Poverty

Which population is most likely to be affected by poverty?


a. Women
b. Men
c. Two-parent families
d. Adoptive families

a. Women

A nurse operates a school-based clinic in a local school,


making it easier for children to access health care. This is
an example of:
a. Case finding
b. Wrap-around services
c. Outreach
d. Comprehensive services
c. Outreach

An example of a health disparity is:


a. Childhood immunization rates
b. High dropout rates
c. Unemployment
d. Income below poverty level
a. Childhood immunization rates

A nurse is developing a one-stop service to meet the


needs of a vulnerable group. Which is an example of this
type of service?
a. Wrap-around services where mental services are linked
b. Giving all immunizations on a single clinic visit
c. Providing multiple services during a single clinic visit
d. Providing free services to the medically indigent
c. Providing multiple services during a single clinic visit

Which vulnerable group would be most sensitive to


adverse effects?
a. Pregnant teenager living with her parents for financial
support
b. Poor, older woman with no means of transportation
c. Two-year-old boy of underinsured parents
d. Recently unemployed father of five
b. Poor, older woman with no means of transportation

Which federal program created support for older and poor


Americans?
a. Social Security Act
b. Medicare Amendment
c. Medicaid Amendment
d. Hill-Burton Act
a. Social Security Act

A client is self-employed as a mechanic and has limited


health insurance coverage. This client is considered:

a. Poor
b. Near poor
c. Medically indigent
d. Uninsured
c. Medically indigent

A mental health nurse working in the 1960s received


additional education to provide care for the severely
mentally ill in the community because of which
legislation?
a. Community Mental Health Centers Act
b. Stewart B. McKinney Homeless Act
c. National Health Planning and Resource Development
Act
d. Patient Protection and Affordable Care Act
a. Community Mental Health Centers Act

A nurse considers an audience's ability to read,


comprehend, and act on medical instructions while
preparing health education materials. Which factor is the
nurse considering?
a. Health literacy
b. Ethnicity
c. Medical training
d. Disparity
a. Health literacy

Which is an example of a social determinant of health?


a. Ethnicity
b. Income
c. Gender
d. Marital status
b. Income

A nurse caring for a homeless population recognizes that


those who are homeless:
a. Need more nursing care than other vulnerable groups
b. Have no desire to seek medical care
c. Have even fewer resources than poor people who have
adequate housing
d. Are living in despair with no hope or resilience
c. Have even fewer resources than poor people who have
adequate housing

Which demonstrates the removal of a barrier to health


care?

a. Discrimination against certain groups


b. Treatment of pets at the same facility
c. Provision of free food at a food bank
d. Providing services for a rural population by using a
mobile clinic
d. Providing services for a rural population by using a
mobile clinic

A set of actions one undertakes on behalf of another is:


a. Social justice
b. Advocacy
c. Linguistically appropriate health care
d. A waiver
b. Advocacy

A nurse works with a group of abused women to enhance


their levels of self-esteem. Which level of prevention is
being performed?
a. Primary level of prevention
b. Secondary level of prevention
c. Tertiary level of prevention
d. Health promotion
c. Tertiary level of prevention

How can nurses promote social justice?


a. By contacting lawmakers about environmental health
issues
b. By assisting at homeless shelters
c. By advocating for policies to improve social conditions
d. By serving on a local coalition to prevent obesity
c. By advocating for policies to improve social conditions

Which is the most appropriate action for a nurse when


planning and implementing care for vulnerable
populations?
a. Setting up multiple clinics in a wide geographic area
b. Advising legal consultants on a variety of issues
c. Making laws to protect the homeless
d. Teaching vulnerable individuals strategies to prevent
illness and promote health
d. Teaching vulnerable individuals strategies to prevent
illness and promote health

Which trends are occurring among vulnerable


populations? Select all that apply.
a. Community-based care and interorganizational

partnerships
b. Outreach and case finding
c. Elimination of disparities
d. Culturally and linguistically appropriate care
a. Community-based care and interorganizational
partnerships
b. Outreach and case finding
d. Culturally and linguistically appropriate care
What are the 10 great public health achievements in the
us from 1900-1999?
vaccinations, motor vehicle safety, safer workplaces,
control of infectious diseases, decrease in coronary heart
disease and stroke deaths, safer and healthier foods,
healthier mothers and babies, fluoridation of drinking
water, recognition of tobacco as a health hazard.

What are social determinants of health?


social conditions in which people live & work

What are health care disparities?


gaps in healthcare experienced by one population
compared to another.

What is the role of government in health care?


Assess health care problems. Developing relevant health
care policies that provide access to services. Ensure that
services are delivered and outcomes are achieved.

Who said, "it is cheaper to promote health than to


maintain people in sickness?
Florence Nightingale

What is the goal of the patient protection and affordable


care act?
The goal of the PPACA (obama 2010) is to provide
affordable health insurance coverage and to improve
access to primary care. and lower costs.

What were 3 major changes in health care in the 21st


century?
development of patient centered care. increased use of
technology. increased personal responsibility for health.

What are the 4 models of health?


clinical model. role performance model. adaptive model.
eudaimonistic model.

clinical model def.


elimination of disease or symptoms

role performance model def.


health that involves a fit b/w ppl and social roles

adaptive model def.


health that involves adaptation to the environment

eudaimonistic model def.


health that is the actualization or realization of human
potential

behavior models
motivational interviewing. behavior change models.
learning model. health belief model.

motivational interviewing def


patient centered communication styles for eliciting
behavior change by helping patients/ groups explore and
resolve ambivalence to change

behavior change model def


models that assist patients, groups and communities to
direct activities toward health and wellness

Learning model def


a behavior change model emphasizing reinforcement of
social competence problem solving autonomy and sense of
purpose

Health belief model


a behavior change model that considers the severity of the
potential illness or physically challenge, the level of
conceivable susceptibility the benefits of taking
preventative action and the challenges that may be faced
in taking action toward the goal of health promotion

ontogenic system

personal factors

microsystem
relationship b/w women and their environment

exosystem
formal and informal social structures

macroculture
values and beliefs of culture

social determinants of health


social conditions in which people live and work

what is one of the core functions of the role of


government in health care?
assess health care problems

community based nursing


minor acute and chronic care that is comprehensive and
coordinated where people work, live or attend school;
illness care provided outside the acute care setting

community health nursing


use of systematic processes to deliver care to individuals
families and community groups with a focus on promoting
preserving protecting and maintaining health

public health nursing


population based practice defined as a sythesis of nursing
and public health within the context of preventing disease
and disability and promoting and protecting the health of
the entire community

shattuck report
provided the 1st systematic use of birth and death records
and demographic data to describe the health

lemuel shattuck
recommended establishment of state health

dorothea dix
prison and mental health

clara barton
red cross/ civil war

lillian wald
founder of public health nursing. henry st settlement.

mary breckinridge
frontier of nursing service 1925 needs of health in the
mountain region. founded midwifery program

philanthropic organization
an organizations that used endowed funds or private
fundraising to address the needs of individuals, families,
and populations

nongovernmental organizations
agency that acquires resources to help others from
private(vs. public) sources

philanthropic organization
an organization that used endowed funds or private
fundraising to address the needs of individuals, families,
and populations

TRUE or FALSE: the united states has found the most


efficient and effective ways to care for all by decreasing
health disparities and giving access to care equality
FALSE: despite the economic strength of the united states
and other industrialized nations, many countries have
found more efficient and effective ways to care for all by
decreasing health disparities and giving access to care
equality

Definition of health
a state of complete physical, mental and social well-being,
not merely the absence of disease or infirmity

what are the 8 millennium developmental goals


1. eradicate extreme poverty & hunger 2. achieve universal
primary education 3. promote gender equality & empower
women 4. reduce child mortality 5. improve maternal
health 6. HIV/AIDS and malaria 7. ensure environmental
sustainability 8. development.

TRUE or FALSE: Health worker migration increases the


burden to care for a society and results in the need to
task shift primarily to primary care providers and
community health workers.
FALSE:
health worker migration increases the burden to care for a
society and results in the need to task shift primarily to
nurses and community health workers.

Health
a quality an ability to adapt to change or a resource to help
cope with challenges and processes of daily living

well-being
a subjective perception of full functional ability as human
beings

what are modifiable risk factors


susceptibility to disease or injury that can be controlled by
individual people, families or communities.

Primary prevention
maximizing health and wellness through strategies that are
set in place before illness or injury is present

secondary prevention
maximizing health and wellness through strategies that are
set in place at the early and active chronic stages of
parthenogenesis of illness and injury (mass screening,
selective screening)

Tertiary prevention
maximizing health and wellness through strategies that are
set in place at the palliation and end stage of disease and
injury trajectories.

epidemiology
study of the distribution and determinants of states of
health and illness in human populations used both as a
research methodology used to study states of health and
illness and as a body of knowledge that results from the
study of a specific state of health for or illness

epidemic
an outbreak that occurs when there is an increased
incidence of a diseases beyond that which is normally
found in the population

rates
the primary measurement used to describe either the
occurrence or or the existence of a specific state of health
or illness

who is perhaps the best known epidemiologist of the 19th


century
john snow

epidemiological triad
host, agent, and environment

wheel of causation
emphasizing the interplay of physical biological and social
environments

The who defines health as


complete physical mental and social well being

during which time period was public health first seen as a


gov. responsibility
the roman era

what is the major force behind the movement toward


home care, hospice care, outpatient tx clinics and
outpatient surgeries
cost containment

when was the first state board of health formed in the us?
1869-north carolina

what was the leading cause of death in the US in 1900?


major cardiovascular-renal disease

what are the only two major industrialized nations without


some form of universal insurance coverage?
US and south Africa

what was established by 1960 amendments to the social


security act of 1935
medicare and medicaid

which of the following is a strategy to contain the cost of


health care
focusing on prevention of illness rather than on treatment

what was the first federal program to provide health care


for citizens other than federal employees
medicare

what decade in the twentieth century saw the most


sweeping federal legislation enacted
1960

which of the following is an example of a public health


prevention policy?
higher cigarette taxes

edward jenner
smallpox 1790

louis paster
1822-1895 pasteurization started as a wine experiment

joseph lister
antiseptic solution

robert koch
anthrax and cholera

john snow
first epidemiologist linked cholera rates with water street
pump golden square in london

isabel hampton
1890s helped found ANA

Margaret Sanger
1910s birth control

Lillian Wald

henry street settlement public health nursing

Clara Barton
civil war nurse leader, US red cross founder

population
group of people who have at least one thing in common
and who may or may not interact w/ one another

types of health care systems


private (BC/BS) public (medicare, medicaid), military
(VA/Tricare)

Role of nurse in managed care


benefits interpreter, client advocate/educator, triage nurse,
utilization/ resource reviewer, risk manager, provider
liaison, primary care provider, case/care manager.
dominant values
the beliefs and sanctions of the dominant or majority
culture

culture
the beliefs, values, and behavior that are shared by
members of a society and provide a design or "roadmap"
for a living

cultural diversity/plurality
a variety of cultural patterns coexist within a designated
geographic area

race
refers to biologically designated groups of people whose
distinguishing features, such as skin color, are inherited

ethnic group
a collection of people who have common origins and a
shared culture and identity; they may share a common
geographic origin, race, language, religion, traditions,
values, and food preferences

ethnicity
the group of qualities that mark one's association with a
particular ethnic group

subcultures
large aggregates of people w/in a society who share
separate distinguishing characteristics, such as ethnicity,
occupation, religion, geographic area, age, gender, or
sexual preferences

microcultures
systems of cultural knowledge characteristic of subgroups
within larger societies

ethnocentrism
the belief or feeling that one's own culture is best and
reflects our tendency to judge other people's beliefs and
behavior using values of our own native culture

ethnorelativism
seeing all behavior in a cultural context

Characteristics of culture
Culture is: learned, integrated, shared, mostly tacit,
dynamic

enculturation
learning about culture through socialization with the family
or significant group

biomedical view
relies on scientific principle and sees diseases/injuries as
life events controlled by physical and biochem processes
that can be manipulated through meds, surgery, and other
treatments

magicoreligious view
focus on the control of health and illness by supernatural
forces; illness - punishment of god, health - gift from god

holistic view
viewing the world in terms of harmonious balance - if
principles guiding natural laws to maintain order are
disturbed, an imbalance in the forces of nature is created,
resulting in chaos and disease

folk medicine

a body of preserved treatment practices that has been


handed down verbally from generation to generation and
exists today as the 1st line of treatment for many people

home remedies
individualized caregiving practices that are passed down
w/in families

complementary therapies
practices used to complement contemporary Western
medical and nursing care and are designed to promote
comfort, health, and wellbeing

primary prevention
a type of prevention that seeks to promote heath and
prevent disease from the beginning

secondary prevention
an intervention that seeks to detect disease early in its
progression (early pathogenesis) before clinical signs and
symptoms become apparent in order to make an early
diagnosis and begin treatment

tertiary prevention
intervention that begins once the disease is obvious; the
aim is to interrupt the course of the disease, reduce the
amount of disability that might occur, and begin
rehabilitation

medicaid
a jointly sponsored state and federal program that pays for
medical services for the aged, poor, blind, disabled, and
families with dependent children

medicare
a federally funded health insurance program for the elderly
and disabled and persons with end-stage renal disease

health care economics


a specialized field of economics that describes and
analyzes the production, distribution, and consumption of
goods and services, as well as a variety of related problems
such as finance, labor, and taxation

legal immigrant
not citizens, but they are by law allowed to both live and
work in the US, often because they have useful job skills or
family ties;
must live in US for 10 years to be eligible for all
entitlements, such as Aid to Families of Dependent
Children, food stamps, Medicaid, and unemployment
insurance

refugees
refugees are admitted outside the usual quota restrictions
based on fear of perseution due to their race, religion,
nationality, social group, or political views;
may receive Temporary Assistance for Needy Families,
Supplemental Securty Inome, and Medicaid

nonimmigrants
admitted to the US for a limited duration of time and for a
specific purpose

unauthorized immigrant
undocumented or illegal aliens;
eligible only for emergency medical services,
immunizations, treatment for the symptoms of
communicable diseases, and access to school lunches

3 public health core functions


assessment, policy development, assurance

assessment
systematic data collection on the population, monitoring
the populations health status, and making information
available about the health of the community

policy development
efforts to develop policies that support the health of the
population, including a scientific knowledge base to make
policy decisions

assurance
making sure that essential community-oriented health
services are available;
includes providing essential personal health services or
individuals, as well as a competent PH workforce

cultural shock
the feeling of helplessness, discomfort, and disorientation
experienced by an individual attempting to understand or
effectively adapt to another cultural group that differs in
practices, values, and beliefs. it results from the anxiety
caused by losing familiar sights, sounds, and behaviors

cultural repatterning
working with clients to make changes in health practices
when the clients cultural behaviors are harmful or decrease
their well-being

cultural preservation
the use by clients of those aspects of their culture that
promote healthy behaviors

cultural knowledge
the information necessary to provide nurses with an
understanding of the organizational elements of cultures
and to provide effective nursing care

cultural imposition
the process of imposing one's values on others

cultural encounter
interaction with a client related to all aspects of his or her
life

cultural blindness
when differences between cultures are ignored and persons
act as though these differences don't exist

quarantine
period of enforced isolation of persons exposed to a
communicable disease during the incubation period of the
disease to prevent its spread should infection occur

sanitation
promotion of hygiene and prevention of disease by
maintenance of health-enhancing (sanitary) conditions

Marine Hospital Service Act of 1798

first federal institution for health problems - subsidized


medical and hospital care for injured merchant seamen
w/first marine hospital in Boston

Shattuck Report
advocate the establishment of state and local boards of
health, enviro sanitation, collection and use of vital
statistics, systematic study of diseases, control of food and
drugs, urban planning, est of nurses' training schools, and
preventive medicine

official health agencies/public health agencies


publicly funded and operated by state or local gov'ts with a
goal of providing population-based health services

voluntary health agencies/private agencies/NGOs


privately funded, operated to address specific health needs

public sector health agencies


Provide important record-keeping service, including
collection/monitoring of vital statistics
Conduct research, provide consultation, and sometimes
financially support other community health efforts

antigen
A live or inactivated substance (e.g., protein,
polysaccharide) capable of producing an immune response

antibody
Protein molecules (immunoglobulin) produced by B
lymphocytes to help eliminate an antigen

active immunity
Protection produced by the person's own immune system;
usually permanent

passive immunity
Protection transferred from another person or animal;
temporary protection

live attenuated vaccines


Measles, Mumps & Rubella (MMR)
Varicella

LAIV
Rotavirus
Herpes Zoster

inactivated vaccines
Toxoids (DTaP, Td, Tdap)
Whole (Hepatitis A, IPV)
Split (Influenza - TIV)
Recombinant vaccines (Hepatitis B, HPV4, HPV2)
Polysaccharide vaccines (PPSV23, MPSV4)
Conjugated vaccines (Hib, PCV13, MCV4)

seven rights of vaccine administration


Right Patient
Right Vaccine or Diluent
Right Time
Right Dosage
Right Route, Needle Length, Technique
Right Site for route indicated
Right Documentation

retrospective payment
Fee established in advance
Reimburse after service rendered
Abused through the requesting and ordering of
unnecessary tests
Encouraged sickness rather than wellness

prospective payment
External authority sets rates
Rates derived from predictions set in advance
Fixed rates rather than cost coverage
Imposes constraints on spending
Providers at risk for losses or surpluses
Which statement about disasters is true?
a. They can be natural or human-made.
b. They can be relieved without assistance.
c. There is always injury and death when a disaster
occurs.
d. The timing of a disaster does not influence the types of
injuries that will occur.
a. They can be natural or human-made.

The use of weapons of mass destruction or natural


disasters, such as earthquakes, will often have more

casualties because:
a. They cause the most widespread destruction.
b. Victims have little time to make evacuation
preparations.
c. Those with chronic conditions cannot escape in time.
d. The early warning systems are not effective.
b. Victims have little time to make evacuation preparations.

Which is an example of a natural disaster?


a. Transportation accident
b. Pollution
c. Communicable disease epidemic
d. Fire
c. Communicable disease epidemic

Which is an example of a human-made disaster?


a. Structural collapse
b. Communicable disease epidemics
c. Mud slides
d. Floods
a. Structural collapse

What emotion would a nurse anticipate when working


with an older person following a disaster?
a. Anger
b. Fear of loss of independence
c. Violence
d. Regression
b. Fear of loss of independence

In which situation would a nurse experience vicarious


traumatization?
a. When sustaining an injury during the disaster
b. When exhausted and unable to adjust to the pace at
home
c. When frustrated about the response time during the
disaster
d. When listening to survivors' stories about traumatic
events
d. When listening to survivors' stories about traumatic
events

A nurse is working in a temporary shelter for victims


following a natural disaster. Which condition is the nurse
most likely to encounter?
a. Stress

b. Communicable disease
c. Chronic illness
d. Injuries requiring first aid
a. Stress

A riverfront community builds a retaining wall to divert


flood water away from the town. This is an example of:
a. Mitigation
b. A natural disaster
c. Disaster reduction
d. Disaster health
a. Mitigation

An example of community preparedness for a disaster is:


a. Assembling emergency supplies
b. Understanding the workplace disaster plan
c. Taking a disaster training course
d. Developing an evacuation plan to remove individuals
from danger
d. Developing an evacuation plan to remove individuals
from danger

What would most likely occur during the preparedness


stage of disaster management?
a. Heightened inspection and increased security in the
community
b. Incorporation of provision of pets into local disaster
plans
c. Purchase of personal protective equipment for all
citizens
d. Assembly of disaster kits for the home, workplace, and
car
d. Assembly of disaster kits for the home, workplace, and
car

When working in a disaster, which attribute would be


most important for the nurse to possess?
a. Flexibility
b. Training
c. Experience
d. Special interest
a. Flexibility

What is most likely to occur in the community during the


Honeymoon Phase after a disaster?
a. First responders work tirelessly to save others.

b. Survivors share their stories.


c. Medical personnel experience exhaustion.
d. Community organizations rebuild the community.
b. Survivors share their stories.

What is the first action a nurse should take when


assessing persons arriving at a shelter following a
disaster?
a. Limit the amount of equipment and medications
brought into the shelter.
b. Determine if the person has a psychological condition
requiring special attention.
c. Assess whether this type of facility is appropriate for
the person.
d. Provide medical care for persons as if they were in a
hospital.
c. Assess whether this type of facility is appropriate for the
person.

What action would the nurse take when triaging victims


following a disaster?
a. Allocation of treatment based on the victim's potential
for survival
b. Assignment of tasks to the appropriate disaster
response personnel
c. Determination of the level of disaster and agency
involvement
d. Participation in community preparedness for the
disaster response
a. Allocation of treatment based on the victim's potential
for survival

Which task would a nurse be most likely to perform while


volunteering on a disaster medical assistant team?
a. Set up immunization clinics.
b. Assist in triaging disaster victims.
c. Provide all the medical care for disaster victims.
d. Complete a needs assessment of the community.
b. Assist in triaging disaster victims.

Which is an example of a bioterrorism and emergency


readiness competency for a public health worker?
a. Recognition of unusual events that might indicate an
emergency
b. Involvement in the chain of command

c. Using a set of preplanned activities for every disaster


d. Leaving communication to the authorities
a. Recognition of unusual events that might indicate an
emergency

What is the purpose of the National Response


Framework?
a. Create a new branch of government that deals with
bioterrorism.
b. Establish a way for the Red Cross to carry out its
mission.
c. Develop a nationwide all-hazards approach to domestic
incident management.
d. Extend presidential power to act quickly upon weapons
of mass destruction.
c. Develop a nationwide all-hazards approach to domestic
incident management.

Which organization would a nurse collaborate with when


developing and coordinating an emergency response plan
for a community?
a. American Red Cross (ARC)
b. Stress Management in Disaster Program (SMID)
c. Centers for Disease Control and Prevention (CDC)
d. Emergency Management Agency (EMA)
d. Emergency Management Agency (EMA)

Following a disaster, what is the priority of the disaster


response?
a. Clean up the environment.
b. Handle the stress reaction of the victims.
c. Bring in as many aid workers and nurses as possible.
d. Reestablish sanitary barriers and focus on basic needs.
d. Reestablish sanitary barriers and focus on basic needs.

If a bioterrorism attack was to occur in the United States,


which program would be used to provide large quantities
of medication to the American public?
a. Cities Readiness Initiative
b. Strategic National Stockpile (SNS)
c. Public Health Information Network (PHIN)
d. Project BioShield
b. Strategic National Stockpile (SNS)

What is anticipated to happen in the future with disaster


management?

a. Vague and unorganized future because constant


emergence of new disasters
b. Increasing sophistication in technology and
surveillance
c. Lack of involvement by national and state officials
d. Decreasing need for public health workers to provide
care
b. Increasing sophistication in technology and surveillance

Which populations are at greatest risk for disruption after


a disaster? Select all that apply.
a. Single-parent families
b. Children
c. Substance abusers
d. Middle-class families
a. Single-parent families
b. Children
c. Substance abusers

Which organizations offer volunteer opportunities in


disaster work for nurses? Select all that apply.
a. American Red Cross
b. Citizens Corps
c. American Nurses Association
d. National Public Health Training Centers
a. American Red Cross
b. Citizens Corps
Which statement about the Intervention Wheel is true?
a. It provides a graphic illustration of population-based
public health practice.
b. It describes in detail the components of public health
nursing.
c. It demonstrates the practice of community health
nurses for policy and lawmakers.
d. It is a framework used by all health departments in the
United States.
a. It provides a graphic illustration of population-based
public health practice.

Since the Intervention Wheel was first published in 1998,


it has:
a. Guided national policy
b. Been used as a tool in deciding licensure issues for

State Boards of Nursing


c. Been incorporated into the public health curricula of
many nursing programs
d. Gained wide acceptance internationally
c. Been incorporated into the public health curricula of
many nursing programs

Public health interventions are implemented with:


a. Legislators, policy makers, and community leaders
b. Individuals and families, communities, and systems
c. Children, adolescents, and adults
d. Health departments, public health agencies, and
visiting nurses associations
b. Individuals and families, communities, and systems

Which statement is true about the origins of the


Intervention Wheel?
a. A panel of nurses from Iowa, Minnesota, North Dakota,
South Dakota, and Wisconsin developed and refined the
Intervention Wheel.
b. It was conceived by a group of international nurses
from Norway, Kazakhstan, and Japan.
c. It was a result of a qualitative analysis carried out by
the State Boards of Nursing.
d. It resulted from a grounded theory process carried out
by public health consultants at the Minnesota Department
of Health.
d. It resulted from a grounded theory process carried out by
public health consultants at the Minnesota Department of
Health.

When public health nurses conduct an assessment of a


community's health, they
a. Define one problem that will be the focus for a year.
b. Assess a social network of interacting individuals
usually in a defined territory.
c. Minimize the effects of health risks and hazards.
d. Intervene at the population-level by changing laws and
regulations.
b. Assess a social network of interacting individuals usually
in a defined territory.

A public health nurse utilizes the nursing process at all


levels of practice by:
a. Including specific goals for community health nurses
b. Developing an accurate nursing diagnosis

c. Analyzing the needs of the community, system, and


individuals and families
d. Utilizing primary, secondary, and tertiary prevention
c. Analyzing the needs of the community, system, and
individuals and families

Public health nurses use a common set of interventions


to:
a. Describe the proper order of implementation.
b. Emphasize surveillance as the main focus of public
health practice.
c. Guide practice and generate agency protocols.
d. Improve the health status of communities, systems,
individuals, and families.
d. Improve the health status of communities, systems,
individuals, and families.

A public health nurse uses Assumption 2, "Public health


nursing practice focuses on populations," to guide
practice. Which would be considered a population of
interest?
a. Healthy school children
b. Homeless individuals
c. A person recently diagnosed with diabetes
d. Teenage parents
a. Healthy school children

What is the purpose of the color-coded wedges on the


Intervention Wheel?
a. The interventions are grouped together in related
wedges.
b. The wedges consist of referral information for each
wedge.
c. The element of health teaching is the predominant
feature of each wedge.
d. Coalition building must be implemented with each
wedge.
a. The interventions are grouped together in related
wedges.

A nurse who is involved in identifying individuals with


unrecognized health risk factors or asymptomatic disease
is using which intervention?
a. Screening
b. Referral and follow-up

c. Surveillance
d. Health teaching
a. Screening

A nurse promotes alliances among organizations for a


common purpose. Which public health intervention is
being implemented?
a. Health teaching
b. Coalition building
c. Surveillance
d. Referral and follow-up
b. Coalition building

A nursing student develops a teaching plan about hand


washing to present to a group of elementary school
children at the local school. Which public health
intervention is being implemented?
a. Collaboration
b. Surveillance
c. Health teaching
d. Screening
c. Health teaching

A public health nurse uses collaboration, coalition


building, and community organizing when:
a. Providing case management, referral, and follow-up
services with individuals
b. Carrying out collective action at the systems or
community levels of practice
c. Conducting a community assessment
d. Implementing primary and secondary prevention
strategies
b. Carrying out collective action at the systems or
community levels of practice

The levels of practice encompassed by the Intervention


Wheel are:
a. Communities, individuals and families, and systems
b. Assessment, diagnosis, and evaluation
c. Primary, secondary, and tertiary
d. Communities, populations, and aggregates
a. Communities, individuals and families, and systems

Public health nurses utilize registries to identify children


with delayed or missing immunizations. They
subsequently follow up with families by phone calls or

home visits. This is an example of ____-level of practice.


a. Systems
b. Community
c. Policy
d. Individual/family
d. Individual/family

A social marketing campaign urging community members


to avoid driving motorized vehicles after consuming
alcohol is implemented in a local community. This
intervention is occurring the _____-level of practice.
a. Individual/family
b. Systems
c. Community
d. Government
c. Community

An occupational health nurse works with an employer to


develop a workplace wellness program for its employees.
This intervention is occurring at the _____-level of
practice.
a. Individual/family
b. Systems
c. Community
d. Government
b. Systems

When implementing interventions at the systems-level of


practice, the public health nurse would:
a. Involve the entire community in solving the health
problem.
b. Identify health problems in the community.
c. Change laws, policies, and practices that influence
population-based issues.
d. Provide outreach services to populations at risk.
c. Change laws, policies, and practices that influence
population-based issues.

Which is an example of a public health nurse conducting


a community assessment?
a. Visiting an elderly person at home to assess and
evaluate safety and fall risk
b. Developing diagnoses to identify nursing interventions
at a health clinic
c. Evaluating services at an immunization clinic where a
translator provides services

d. Compiling recent data from the county health


department on child abuse cases
d. Compiling recent data from the county health
department on child abuse cases

An example of a measurable outcome health status


indicator at the individual-level of practice that would be
used by a public health nurse is:
a. A 50-year-old woman receives annual mammograms
b. School absences in a community decline
c. Teachers have increased awareness of health problems
d. Those in poverty utilize the free mammogram program
b. School absences in a community decline

Which nursing diagnosis would a public health nurse use


when addressing the problem of obesity at the
community-level of practice?
a. Alteration in nutrition: More than body requirements
b. Need for increased knowledge of proper nutrition
c. Families at risk for obesity because of inactivity
d. Overweight child related to poor dietary habits
c. Families at risk for obesity because of inactivity

A public health nurse implements the public health


intervention of health teaching at the systems-level of
practice by:
a. Participating in the "Great American Smokeout"
b. Working with a local employer to provide smoking
cessation education
c. Providing one-on-one counseling to smokers
d. Advocating for increased taxes on tobacco products
b. Working with a local employer to provide smoking
cessation education

The cornerstones of public health nursing practice (select


all that apply):
a. Focus on the health of the entire population
b. Reflect community priorities and needs
c. Promote health through strategies driven by
epidemiological evidence
d. Are grounded in an ethic of collaboration
a. Focus on the health of the entire population
b. Reflect community priorities and needs

c. Promote health through strategies driven by


epidemiological evidence
Official agencies are financed primarily by:
a. Charities
b. Individual clients
c. Third-party payers
d. Tax funds
d. Tax funds

Medicare-certified home health agencies place emphasis


on _____ care.
a. Chronic
b. Distributive
c. Intermittent
d. Primary
c. Intermittent

A nurse who implements a population-focused model of


home care delivery would most likely:
a. Use an assessment protocol when conducting visits.
b. Collaborate with other disciplines.
c. Provide care for a specific population.
d. Apply high-intensity interventions.
a. Use an assessment protocol when conducting visits.

A home care nurse is employed by a proprietary agency.


What does this mean?
a. The agency is exempt from federal income taxes.
b. The agency is governed by a board of directors.
c. The agency is a profit-making agency.
d. The agency is reimbursed primarily by tax funds.
c. The agency is a profit-making agency.

Which factor is important for the nurse to consider when


coordinating care to ensure a comfortable and peaceful
death?
a. The unique stress dying patients often experience
b. The cultural values, expectations, and preferences of
the family
c. The communication style of each family member
d. The abilities of the nurse and health care team
b. The cultural values, expectations, and preferences of the
family

What is the primary objective of hospice programs?


a. To provide comfort and peace at a difficult time
b. To reduce the cost of care for the terminally ill
c. To keep clients at home until the end of life
d. To provide care for terminally ill clients expected to live
longer than 6 months
a. To provide comfort and peace at a difficult time

The nurse provides direct care services to a stroke victim


to avoid complications. This is an example of which level
of prevention?
a. Primary prevention
b. Secondary prevention
c. Tertiary prevention
d. Assessment
c. Tertiary prevention

Which nursing intervention is an example of a skilled


nursing service?
a. Teaching the client and family about medication
administration
b. Consulting with other nurses about the care of a client
in the home
c. Coordinating services for maximum benefit at minimal
cost
d. Reviewing nursing assistant charting at the end of each
day
a. Teaching the client and family about medication
administration

A nurse employed in home health nursing practice


provides:
a. Intermittent nursing visits
b. Custodial care
c. Family caregiving
d. Palliative care
a. Intermittent nursing visits

Which organization publishes the current scope and


standards of home care practice?
a. Centers for Medicare and Medicaid Services
b. American Nurses Association
c. Robert Wood Johnson Foundation
d. Visiting Nurse Association
b. American Nurses Association

During the performance of which standard of care would


the nurse discover the need for Meals on Wheels?
a. Diagnosis
b. Planning
c. Assessment
d. Implementation
c. Assessment

Which concepts are used to evaluate client outcomes


when using the Omaha System Problem Rating Scale for
Outcomes?
a. Problem, Intervention, and Outcome
b. Knowledge, Behavior, and Status
c. Knowledge, Skill, and Attitude
d. Problem, Category, and Modifiers
b. Knowledge, Behavior, and Status

After conducting a comprehensive client assessment, the


nurse would organize information about the client's
medication regimen into which domain of the Omaha
System Problem Classification Scheme?
a. Environmental
b. Psychosocial
c. Physiological
d. Health-related behaviors
d. Health-related behaviors

In which situation would a nurse use the Omaha System


Problem Classification Scheme?
a. When measuring client outcomes
b. When collaborating with other professionals
c. When providing health education
d. When organizing a comprehensive assessment
d. When organizing a comprehensive assessment

Why would a nurse use the Omaha System?


a. To define each health care provider's role
b. To use nursing diagnoses in community health nursing
practice
c. To foster collaborative practice
d. To provide practitioners a method for communication
c. To foster collaborative practice

What action should a nurse take to facilitate


interprofessional collaboration?
a. Understand who is in charge of the client's care.

b. Make appropriate referrals.


c. Recognize what other professionals do and how they
view their roles.
d. Work in isolation when performing complex skills.
c. Recognize what other professionals do and how they
view their roles.

A home care agency has applied for accreditation from


the Joint Commission. What is the next step the agency
will take?
a. Attend a conference to learn more about the
accreditation process.
b. Schedule a site visit with the Joint Commission.
c. Complete a self-study of the agency.
d. Improve methods of documentation of client visits.
c. Complete a self-study of the agency.

A nurse uses the first stage of the Outcomes and


Assessment Information Set (OASIS) outcomes analysis
to:
a. Compare an agency's performance to a national
sample.
b. Report data on patient care to a national registry.
c. Stabilize a client's condition to continue to receive
funding.
d. Document client assessment data and findings.
a. Compare an agency's performance to a national sample.

Which historical event caused dramatic changes in home


care nursing?
a. Passage of Medicare legislation
b. Establishment of Visiting Nurse Associations
c. Creation of the Montefiore Hospital Home Care Program
d. Provision of funds by the Civil Works Administration
a. Passage of Medicare legislation

A nurse using telehealth technology in the home setting:


a. Uses WebTV to teach clients about their health
b. Shares health information using electronic
communications
c. Makes regular visits to clients to check the technology
d. Risks violation of the Health Insurance Portability and
Accountability Act
b. Shares health information using electronic
communications

Which populations may require a high-intensity


transitional care program? Select all that apply.
a. Adults with cognitive impairments
b. Women with high-risk pregnancies
c. Older adults with heart failure
d. People who are moving from one state to another
a. Adults with cognitive impairments
b. Women with high-risk pregnancies
c. Older adults with heart failure

Which are categories of classifying interventions


according to the Omaha System Intervention Scheme?
Select all that apply.
a. Health education
b. Case management
c. Treatments and procedures
d. Direct care services
b. Case management
c. Treatments and procedures

Which illegal and unethical activities have occurred in


health care systems in the past? (Select all that apply.)
a. Inappropriate use of home health services
b. Inaccurate billing for services
c. Excessive administrative staff
d. "Kick-backs" for referrals
a. Inappropriate use of home health services
b. Inaccurate billing for services
c. Excessive administrative staff
d. "Kick-backs" for referrals
School nurses give comprehensive nursing care to the
children and staff at a school. Which of the following are
also activities school nurses may do?
a. Care for children who are home sick from school
b. Coordinate the education program of the school and
consult with school officials to help identify and care for
other persons in the community
c. Allow students with communicable diseases to attend
classes
d. Provide tertiary levels of care as their primary focus

b. Coordinate the education program of the school and


consult with school officials to help identify and care for
other persons in the community

The National Association of School Nurses (NASN)


developed 11 criteria for school nurses. Among them are:
a. Interact with interdisciplinary health team members.
b. Communicate with teachers to effectively manage
classrooms.
c. Teach CPR to every member of the school staff.
d. Do research on students at the school.
a. Interact with interdisciplinary health team members.

Community outreach is a role or function that many


school nurses participate in. This may involve:
a. Case management that helps coordinate health care of
children with complex health problems
b. Health education within the school, such as teaching
about proper nutrition
c. Providing direct care to ill and injured children
d. Participating in community activities, such as health
fairs and immunization clinics
d. Participating in community activities, such as health fairs
and immunization clinics

Working with the national Safe Kids Campaign, the school


nurse can give educational programs reminding children
to use their seat belts or bicycle helmets to prevent
injuries. Another area where education is needed because
of the high potential for many injuries is:
a. School sports
b. Classrooms
c. Library activities
d. School busses
a. School sports

All states have requirements that children receive


immunizations or vaccinations against communicable
diseases before they attend school. Some parents request
that their child be exempt from the required
immunizations for medical, religious, or philosophical
reasons. What action might a school nurse take related to
this request? The nurse:
a. Will deny entry into school for children without the
required immunizations
b. Must educate the parents about the need for

immunizations
c. Should be aware of the laws in the state regarding
acceptable reasons for immunization exemption
d. Should allow the student to attend school without the
immunizations
c. Should be aware of the laws in the state regarding
acceptable reasons for immunization exemption

Which of the following are general guidelines for


administration of medications at school?
a. A current medication book should be available in case
information is needed.
b. The nurse should give medications brought in from
home by the child.
c. Medications cannot be given without a physician order.
d. Narcotics and controlled substances should be locked
up.
a. A current medication book should be available in case
information is needed.

When a nurse identifies a child who may be abused, or


receives information from a teacher that a child may have
been abused, the nurse must:
a. Call the police immediately to take the child to a safe
place
b. Contact the parents and ask about it
c. Contact the legal authorities as well as the school's
principal
d. Examine the child to see if the abuse really occurred
c. Contact the legal authorities as well as the school's
principal

The future of school nursing is strong. Which of the


following may occur in the future in the practice of school
health?
a. Telehealth and telecounseling to teach health
education
b. Robotic school nurses
c. Nurses working as teachers as well as nurses
d. Setting up private practices for treatment of the entire
family
a. Telehealth and telecounseling to teach health education

The Individuals with Disabilities Education Act (IDEA)


guarantees:
a. Sex education classes

b. Educational services
c. Day care
d. Orthodontics
b. Educational services

The National Association of School Nurses recommends


that school nurses be which of the following?
a. LVNs/LPNs
b. RNs with BSN and special certification in school nursing
c. School nurse practitioner
d . MSN with specialization in school nursing
b. RNs with BSN and special certification in school nursing

When coordinating the health care for children with


complex health problems, the nurse is performing which
of the following roles?
a. Health educator
b. Case manager
c. Consultant
d. Counselor
b. Case manager

Which of the following roles is the school nurse


performing when providing health information to school
administrators, teachers, and parent-teacher groups?
a. Health educator
b. Case manager
c. Consultant
d. Counselor
c. Consultant

A site that provides primary health care services to


children and youths by nurse practitioners in the school
setting is:
a. A primary care clinic
b. A school-based health center
c. School health services
d. A child health improvement plan
b. A school-based health center

Education to prevent health problems in children is:


a. Primary prevention
b. Secondary prevention
c. Tertiary prevention
d. Assessment
a. Primary prevention

The leading cause of death in children and teenagers is:


a. Leukemia
b. Injuries
c. Suicide
d. Diabetes
b. Injuries

A very high body temperature can lead to death when


taking which of the following drugs?
a. GHB
b. Rohypnol
c. Alcohol
d Ecstasy
d Ecstasy

The nurse caring for children when they need health care
is providing which of the following levels of prevention?
a. Primary prevention
b. Secondary prevention
c. Tertiary prevention
d. Health promotion
b. Secondary prevention

Which of the following is needed for the school nurse to


give a student prescription medication?
a. Copy of the prescription label on bottle
b. The smallest container needed for the medication
c. Contact with the physician
d. Signed consent from the student
c. Contact with the physician

19. Which of the following is true of lice?


a. Most cases are found in teenagers.
b. Most cases are found in white, middle-class children.
c. Most cases found in those with dirty hair.
d. There are between 1 and 5 million cases every year in
the United States.
b. Most cases are found in white, middle-class children.

The third-leading cause of death in teenagers is:


a. Cancer
b. Suicide
c. Accidents
d. Drug overdose
b. Suicide

The leading cause of children being absent from school as


a result of a chronic illness is:
a. Diabetes
b. Cancer
c. Asthma
d. Autism
c. Asthma

Providing ongoing care to the pregnant teenager is


considered:
a. Primary prevention
b. Secondary prevention
c. Tertiary prevention
d. Health promotion
c. Tertiary prevention

The school nurse cares for children who are injured or


become ill in the schools. A plan should be in place in
case of an emergency. Elements of this plan should
include which of the following? Select all that apply.
a. How to make arrangements to transfer children to the
hospital via ambulance
b. Knowledge of the staff about standard precautions
c. Individual emergency plans for students with health
problems
d. The role the children will play in treatment and triage
of fellow classmates
a. How to make arrangements to transfer children to the
hospital via ambulance
b. Knowledge of the staff about standard precautions
c. Individual emergency plans for students with health
problems

Violence in schools has become a serious problem. The


school nurse may be able to identify students who will act
in this way. There are several characteristics that may
help point out a student who may be thinking about such
violence. They include which of the following? Select all
that apply.
a. Venting
b. Vocalizing

c. Vandalism
d. Victimizing
a. Venting
b. Vocalizing
c. Vandalism
d. Victimizing
Carrier
A person who harbors an infectious organism and transmits
the organism to others although having no symptoms of
the disease

colonization
The presence and multiplication of infectious organism
without invading damage to tissuse

Common soucre outbreak


An outbreak characterized by exposure to a common,
harmful substance

Contagious
Able to be passed easily from one person to another.

endemic
The constant or ususal prevalence of a specific disease or
infectious agent within a population or geographic area

Epidemic
A widespread outbreak of an infectious disease.

Healthcare-associated infection
An infection that was not present on admission to the
hospital and develops during the course of treatment for
other conditions (nosocomial)

Incubation period
The period between infection and the appearance of
symptoms of the disease.

Infectious disease
A disease that is caused by a pathogen and that can be
spread from one individual to another.

propagated outbreak
an outbreak that spreads from person to person rather than
from a common source

pathogenicity
The ability to cause disease

Reservoir
A place where microorganisms survive, multiply, and await
transfer to a susceptible host

Secondary infection
caused by an opportunistic pathogen after the primary
infection has weakened the body's defense

Surveillance
- collecting, analyzing, & reporting data on rates of
occurrence, mortality, morbidity, and transmission of
infections

transmission
An incident in which an infectious disease is transmitted to
another person/people.

Antigenic drift
Slow and progressive genetic changes that take place in
DNA and RNA as organisms replicate in multiple hosts

Antigentic shift
dramatic change,

Convergence model
Model illustrating the interaction of 13 factors that
contribute to the emergence of infectious disease.

Directly observed therapy


best treatment for TB, when patients take their drugs in
front of healthcare workers-- decreases odds of drugresistant strains

Ecosystem
A community of interdependent organisms and the physical
environment they inhabit.

Emerging infectious disease


Diseases that are new, increasing in incidence, or showing
a potential to increase in the near future

Herd immunity
The resistance of a group to invasion and spread of an
infectious agent, based on the resistance to infection of a
high proportion of individual members of a group; the
resistance is a product of the number susceptible and the
probability that those who are susceptible will come into
contact with an infected person

Microbial adaption
Process by which organisms adjust and change to their
enviroment

Pandemic
A worldwide epidemic.

Femicide
A term used to refer to a homicide that occurs in the
context of intimate partner violence (IPV).

Gender-based violence
any act of gender violence that results in or is likely to
result in physical, sexual or psychological harm and
suffering to women, including theats of such acts, coercion,
or arbitrary deprivations of liberty, whether occuring in
public or private life.

Human rights
The basic rights to which all people are entitled as human
beings.

Incidence
Disease is the number of new cases occurring in a specific
period

Intimate partner violence


domestic violence or battering between two people in a
close relationship

Lethality assessment

Attempt to predict likelihood of suicide

Perpetrate
To commit, as a crime or other antisocial act

Prevalence
'The number or proportion of cases of a particular disease
or condition present in a population at a given time.'"snap
shot" (AIHW, 2008)

Violence
1. Violence is a broad concept that ranges from homicide to
physical assault or intentional use of force against another
person, community, or even against oneself (suicide)

World Health Organization


An international body of health care professionals,
including clinicians and epidemiologists among many
others, that studies and responds to health needs and
trends worldwide.

Abstinence
Act of refraining from for 12 months or more

Addiction
habitual use of a drug, a disease in itself. Caused by taking
a chemically dependent drug "prescribed or illegally used
compulsively psycological and physiologicaL dependency

Craving
an intense desire for some particular thing

Intoxication
An altered state of consciousness as a result of drinking
alcohol or ingesting other substances

Substance abuse
A) Substance abuse differs from dependence in that
withdrawal or tolerance have never been present, but use
of the substance has caused significant impairment or
distress - e.g., missing work, school failure, DUI, legal
problems

Substance dependence

A diagnostic term used in clincial psychology and


psychiatry that identifies an individual displaying
significant signs of a dependent relationship with a
psychoactive drug.

Substance use
This is the use of a substance that alters physical or mental
functions: It can be legal or illegal, therapeutic or
recreational and can be done by sniffing, snorting, inhaling,
swallowing, drinking, smoking, or injecting the substance or
absorbing it throught the skin.

Substance use disorder


Dependence and abuse

Tolerance
A process in which your body needs more and more of a
drug to get the same effect

Withdrawal
Physical signs of discomfort associated with the
discontinuation of an abused substance.

Health professional shortage area


rural health care services, especially in regions with
insufficient numbers of all types of health care providers
have become a national priority since the early 1990s

Inmate
a person serving a sentence in a jail or prison

Medically underserved area


as a result nurses work in community-based settings
providing health promotion and disease prevention to the
homeless, mentally ill, and others who have limited access
to health care or who lack health care insurance

Medically underserved population


Rising rates of unemplyoment, homelessness, and rising
health care cost contribute to the increase in the medically
underserved population. Many nurses are working to serve
this popultaion (building clinics where medical help is not
available)

Trimorbidity
Three common chronic illnesses of homeless people
(mental illness, chronic physical illness, substance
addiction)

Underserved population
A subgroup of the population that has a higher risk of
developing health problems because of a greater exposure
to health risks because of marginalization in sociocultural
status, access to economic resources, age or gender

Bioavailabilty
The amount of a contaiminant that actually ends up inthe
systemic circulation

Biomonitoring
Analysis of blood, urine, tissues, and so forth to measure
chemical exposure in humans

Environmental epidemiology
field of public health science that focuses on the incidence
& prevalence of disease or illness in a population from
exposures in their enviroments

Environmental health
Environmental factors that influence human health and
ecological systems

Environmental justice
A legal strategy based on claims that racial minorities are
subjected disproportionately to environmental hazards

Exposure
A situation in which a person has had contact with blood,
body fluids, tissues, or airborne particles in a manner that
suggests disease transmission may occur.

Exposure estimate
Factors that determine a persons level of exposure to a
contaminant.

Exposure pathway
The pathway linking the environmental source of a
contaminant to the point of exposure

Exposure history
process to help determine whether an individual has been
exposed to environmental contaminants

Precautionary principle
States that when there is a threat of serious or irreversible
environmental damage, we shouldn't wait for scientific
proof before taking action.

Healthy communities
communities that optimize the physcial, social, and
economic environment of the community

Risk assessment
Assessing each risk involves determining likelihood that the
risk event will occur and the degree of impact the event
will have on the project objective.

Toxicology
study of poisonous substances and their effects upon body
parts

After-action report
Retrospective analysis used to evaluate emergency
response drills.

Decontamination
A process in which recently used and soiled medical
devices, including instruments, are rendered safe for
personnel to handle.

Evacation
Moving people from a dangerous place to safety

Incident Command system


a system by which facilities, equipment, personnel,
procedures, and communications are organized to operate
within a common organizational structure designed to aid
in the management of resources at emergency incidents.

Invacuation
Moving people from one area to another, within the same
facility.

National response framework


framework that guides how the nation conducts all-hazards
incident response

National incident management system


the management system used by federal, state, and local
governments to manage emergencies in the United States.

Personal protective equipment


Those items of protection worn to minimize exposure to
hazards; those items worn by the embalmer to avoid
contact with blood and other body fluids.

Point of distribution
centralized location where the public picks up emergency
supplies following a disaster

Real time
Means that the people who are conversing on a computer
are online at the same time.

Scenario
A sequence of events that is imagined, assumed, or
suggested

Shelter-in-place
Taking refuge inside an interior room with few or no
windows, in the event of the release of chemical biological,
or radiological contaminants:

Simulation
A representation of a situation or problem with a similar but
simpler model or a more easily manipulated model in order
to determine experimental results.

Terrorism
Is the use or threatened use of violence to gain a political
objective

Black box warning


A type of warning that appears in a drug's prescribing
information, required by the U.S. Food and Drug

Administration alerting prescribers of serious adverse


events that have occurred with the given drug

Community mental health center


A facility offering a wide range of mental health services,
such as prevention, counseling, consultation, and crisis
intervention.

Culturally competent mental health service


Health care that is sensitive to the needs and health status
of different population groups.

Deinstitutionaliztion
a focus on reducing costly and neglectful institutional care
and on providing more humane services in the community

Diagnostic and Statistical manual


A widely used system for classifying psychological
disorders

Early intervention programs


program directed at young children who are at risk for later
problems, intended to prevent problems from developing

First-generation antipsychotics
improve positive symptoms of schizophrenia more
effectively than negative symptoms or cognitive
dysfunction.

Nonadherence
an informed decision on the part of the patient not to
adhere to or follow a therapeutic plan or suggestion. Same
thing as noncompliance.

President's new freedom commission on mental health


2003, called from streamlined system with less fragmented
delivery, advocated early diagnosis and treatment, new
expectation for principles of recovery, increased assistance
in helping people find housing and work.

Prodromal stage
This is the Initial stage of disease, characterized by
common general complaints of illness malasia and fever ?

Second-generation antipsychotics
drugs that alleviate schizophrenia without serious risk of
producing movement disorders

tardive dyskinesia
A side effect of long-term use of traditional antipsychotic
drugs causing the person to have uncontrollable facial tics,
grimaces, and other involuntary movements of the lips,
jaw, and tongue.

Americans with disabilities act


Passed by Congress in 1991, this act banned discrimination
against the disabled in employment and mandated easy
access to all public and commerical buildings.

Community school model


Collaborative design that uses the resources of a
community to provide structure prevention services such
as after-school programs

Early periodic screening diagnosis and treatment


(EPSDT) program covering screening and diagnosis
services in recipients younger than 21 years of age

Individuals with disabilities education act


Persons who have a physical or mental impairment that
substantially affects one or more major life activites

School health council


coalition of a wide range of community stakeholders
including family & student representives who contribute to
the development of action plans designed to improve the
health & safety of the students

School health index


data collection tool that helps school identify the strengths
& weaknesses of an individual school's policies & programs
for promoting health & safety. Action plans can be
developed

School health nursing


specialized practice of professional nursing that advances
the well being accademic successs & lifelong achievement
of the students

Youth risk behavior surveillance survey


survey conducted biannually; measures the prevalence of
six categories of health risk behaviors among youths
through representative national, state, and local surveys
using a self-report questionnaire

congregation
A gathering of people, especially for religious services

congregration-based model
faith based nurse serving a particular faith community by
virtue of a contract or job description

Institution-based model
the parish nurse serves as a liasion and helps plan and
coordinate care

Parish nursing
a model of care for members of a faith community that can
include wellness care through home visiting

Spiritual care
Recognition of spiritual needs and the assistance given
toward meeting those needs.

Adjuvant drugs
relieve pain either alone or in combination with analgesics.
These drugs can potentiate or enhance the effectiveness of
the analgesic.

Advance directives
Documents that indicate a patient's wishes in the event
that the patient becomes incapacitated and unable to
make decisions regarding medical care

Bereavement
Feeling of grief or desolation, especially at the death or loss
of a loved one.

Breakthrough pain
Pain that occurs between doses of pain medication. (p.
157)

Comfort measures only


goal of treatment is a comfortable dignified death & that
further life-sustaining measures are no longer indicated.

Grief
Extreme sadness after someone dies or goes away.

Healthcare proxy
an individual who has been appointed to make health
decisions by the maker in the event of their incapacitation

Hospice home care


care of clients and families who have 6 months or less to
live in what they consider their home

Mourning
Outward, social expressions of grief and the behavior
associated with loss.

Pallative care
care designed not to treat an illness but to provide physical
and emotional comfort to the patient and support and
guidance to his or her family

Postmortem care
Care given immediately after death before the body is
moved to the mortuary usually by nurses

American Association of occupational health nurses


Professional assocation for nurses working in a business
setting, dedicated to the health and saftey of workers,
worker populations and community group

Biological hazards
Any living organisms or its properties that can cause an
adverse response in humans

Chemical hazards
Toxicity and the use of the chemicals. Excessive airborne
concentrations of mists, vapors, gases, or solids in the form
of dusts or fumes

Ergonomics

The study of workplace equipment design or how to


arrange and design devices, machines, or workspace so
that people and things interact safely and most efficiently.

National institute of occupational safety and health


Fedral agency established to help ensure safe and healthy
working conditions by conducting scientific research,
gathering information, and providing education and
training in occupational saftey and health

Occupational health history


An assessment of the characteristics of the workers present
job, a chronological record of all past exposures,
occupational exposure inventory, and lists of other
exposures at home or in community.

Occupational safety and health administration


U.S. law authorizing the federal government to establish
and enforce occupational safety and health standards for
all places of employment engaging in interstate commerce

Physical hazards
Hazards that result from the transfer of physical energy to
workers

Psychosocial hazards
- all organizational factors and interpersonal relationships
in the workplace that may affect the health of the workers

Root cause analysis


An analytical technique used to determine the basic
underlying reason that causes a variance or a defect or a
risk. A root cause may underlie more than one variance or
defect or risk.

Workplace walk through


A complete survey of the workplace, inside and outside,
compiling information as to the presence of hazards, the
location of entries and exists, the availability of emergency
equipment and potential trouble spots.
Which of the following are responsible for the majority of
food borne illnesses?
A. E.Coli and Salmonella

B. Salmonella dna Campylobacter


C. Botulism and Salmonella
B. Salmonella dna Campylobacter

A resturant had a food borne illness and several people


were affected. Which of the following would you consider
sending to the ER for treatment?
A. A 23-year-old with 2 day history of diarrhea
B. A 15-month-old with vomiting and diarrhea X 1 day
C. A 78-year-old woman with a temperature of 101
B. A 15-month-old with vomiting and diarrhea X 1 day

___ is the measure of the potential ability of the host to


transmit to another.
Infectiousness

___ is the time interval between invasions of the


infectious agent and the appearance of symptoms.
Incubation period

___ is the interval where the infectious agent can be


transferred to another.
Communicable period

___ is the constant presence of a disease in a geographic


area or population.
Endemic

___ is the occurrence of a disease in excess of the normal


distribution.
Epidemic

___ is a worldwide occurrence of a disease in excess of


the normal distribution.
Pandemic

___ is the irreversible extermination of an infectious


agent.
Eradication

___ is the removal of a disease from a large geographic


area.
Elimination

___ is a disease endemic in Arizona.


Coccidiodomycosis

___ is a disease that is pandemic.


HIV/AIDS

___ is an environmental cause of emerging diseases.


Global warming

Flu shots are an example of ___ ___.


Primary prevention

What are the three points on the epidemiological triangle


stemming from vector?
Host
Agent
Environment

Name the vaccine-preventable diseases.


Hepatitis A
Hepatitis B
Measles, mumps, rubella (MMR)
Diphtheria, pertussis, tetanus (DTaP)
Varicella
Polio (IVP)
Haemophilus influenza type B (HIB)
Pneumonia & influenza
Human papilloma virus (HPV)
Rotovirus

What event might potentially indicate acts of


bioterrorism, leading to the need for follow-up and
surveillance by the public health nurse?
Healthy adults getting chickenpox without exposure to
children with chickenpox

What actions might a public health nurse take to prevent


the spread of communicable disease?
Intensify surveillance
Increase healthcare surge capacity
Develop public education & communication
Stockpile medical supplies & antiviral drugs
Vaccine development & distribution
Develop plans for businesses & schools

Ensure access to clean water


Prevent food-borne illness

(True/false) Food-borne illness will affect all people at a


site, not just one person.
True

When should someone with suspected food-borne illness


seek medical treatment?
High fever (101.5 F + orally)
Blood in stool
Prolonged vomiting
Dehydration
Prolonged diarrhea 3+ days

What are the symptoms of food-borne illness?


Fever
Diarrhea
Abdominal cramps
Vomiting

What is the typical treatment for food-borne illness?


Dehydration treatment (IV/oral)
Antiemetics
Antidiarrheals

What should someone do if he is bitten by an animal and


is unsure if that animal has rabies?
Get rabies treatment (shot in abdomen) - secondary
prevention

What are the three types of agents that cause infection


and infectious disease?
Infectious agents: bacteria, viruses, fungi, parasites
Chemical agents: heavy metals, toxins, chemicals,
pesticides
Physical agents: radiation, heat, cold, machinery

What host characteristics influence the spread of disease?


Genetic susceptibility
Immutable characteristics
Acquired characteristics (immunity)
Lifestyle factors (diet & exercise)

What environmental factors facilitate transmission of


infectious agents from an infected host to other
susceptible hosts?
Climate: temp, rainfall
Plant & animal life
Human population distribution
Socioeconomic factors: education, access to care,
resources
Working conditions

(True/false) All diseases require a vector.


False

Vectors, such as anthropoids (ticks, mosquitos), transmit


infectious agents by biting or depositing infective
material near the host. They may be necessary to the life
cycle of the organism (mosquitos and malaria), or may
act as ___ ___, as flies do with food.
Mechanical transmitters

What intervention might the health department


implement for foreclosed homes with unkept swimming
pools?
Provide fish that eat mosquitos that breed in standing
water

A(n) ___ ___ is a disease transmitted through direct


contact with an infected individual or indirectly through a
vector.
Communicable (contagious) disease

A(n) ___ ___ is a disease resulting from the presence and


activity of a pathogenic microbial agent. It is potentially
transferrable.
Infectious disease

What is the number-one cause of death worldwide?


Communicable/infectious disease

What are the three leading causes of death for children


under 5?
Pneumonia
Diarrhea
Malaria

How is hepatitis B transmitted?


Blood/body fluids
Perinatal
Unsafe injections & transfusions
Sexual contact
Can survive for at least one week, dried, at room
temperature on environmental surfaces

What risk factors increase an individual's risk for


contracting an STI?
Multiple sex partners
Unprotected sex
Drugs & alcohol
Having other STIs
Failure to use safety measures
Age less than 25 years
Sharing needles
Douching

How can STI transmission be prevented?


Abstinence
Condom use (not 100% effective)
Long-term, monogamous relationships
Early detection through case-finding & screening

How can hepatitis B transmission be prevented?


Immunization
Universal precautions
Testing of pregnant women
Reducing sexual & injection drug use exposure

What are the major symptoms of most STIs?


Itching
Burning with urination
Pain
Bumps, blisters, vesicles
Drainage
Epididymitis
Enlarged lymph nodes
Eye infection
Sore throat
Sores in/on mouth
(Anything someone wasn't born with)

What are the major symptoms of acute hepatitis B, which


can last several months to a year during which time
antibodies are developed?
Flu-like symptoms
Jaundice
Extreme lethargy
Nausea
Fever
Joint pain

What are the major symptoms of chronic hepatitis B?


Anorexia
Fatigue
Hepatomegaly
Jaundice
Causes liver diseases such as cancer & cirrhosis

Which STIs are curable with antibiotics?


Chlamydia
Gonorrhea
Syphilis

What chronic STIs require a lifetime of symptom


management and infection control?
HIV
HPB
HPV
HSV

Why might younger persons be at greater risk for


contracting an STI?
Drug & alcohol use, leading to poor decisions/lack of
protection

A patient presents at a community clinic complaining of


extreme itching in her genital region. What would be an
appropriate action by the nurse?
Assess & determine if other symptoms are present
Ask, "have you had unprotected sex?"

What is expedited partner therapy (EPT)?


Individuals diagnosed with an STI are given medication for
himself/herself AND his or her partner

A ___ period is the time during which a disease is present,


but symptoms have not appeared.
Latency period

(True/false) An individual tests positive for TB, but her


chest x-ray is negative, indicating latent TB. This person
is still contagious because the disease is present in her
body.
False

(True/false) Someone with a latent STI can still transmit it


to others.
True

How are gonorrhea and chlamydia diagnosed?


Urine
Swabs - endocervical, anus, urethra, pharynx, tonsils

How are syphilis, herpes, and hepatitis B diagnosed?


Blood test

What is the CDC's recommendation for STI testing?


Annual testing of all sexually active women 25 or younger
Older women with new sex partner or multiple sex partners
All pregnant women

(True/false) Partner notification must always be done for


patients who test positive for HIV.
False - don't know where/when it was contracted

(True/false) Community health nurses may treat patients


for more than one STI without confirmation if the
individual tests positive for one STI.
True - cocktail that knocks out most STIs can be given
Cheaper, don't have to keep testing, and able to acquire
consent & treat immediately

Partners of the last ___ days of individuals diagnosed with


STIs must be treated at the same time.
60 days

(True/false) Expedited partner therapy (EPT) is used for


individuals who are diagnosed with HIV.
False - other STIs

When an individual is diagnosed with an STI, notification


of partners is (mandatory/voluntary).
Voluntary

What are some chronic complications that can result from


STIs?
Syphilis: dementia, blindness, joint disease, heart disease
G/C: PID, infertility, ectopic pregnancy, serious reproductive
problems
HPV: abnormal Paps, cervical cancer

Nearly ___% of children with congenital syphilis, which is


acquired during gestation when syphilis is not treated, will
die before or shortly after birth.
50%

Which STIs can cause eye infections in newborns if


untreated in pregnant women?
Gonorrhea/chlamydia - prophylaxis with erythromycin or
tetracycline eye drops

What education components would a nurse include when


teaching teens about safe sex using condoms?
Always use new one
Use latex
Water-based lube
Put on and take off erect penis
Use spermicidal
Use latex dams for oral sex

Who is at the highest risk for contracting hepatitis B?


IV drug users
Persons with STIs/multiple sex partners
Immigrants & refugees
Descendants from areas of high endemic rates of HBV
Health care workers
Hemodialysis clients
Inmates of long-term correctional institutions

(True/false) Symptoms of hepatitis A are present only in


the acute phase of the disease process.
True

(True/false) Someone with open sores due to a syphilis


infection puts that individual at greater risk for
contracting other STIs, such as HIV.
True

How is TB transmitted?
Exposure to bacilli in airborne droplets form infected person
during coughing, sneezing, or talking

Describe the incubation period and appearance of


symptoms in someone with TB.
Incubation period: 4 - 12 weeks; time from inhaling droplets
until S/S appear
Clinical symptoms: 6 - 12 months

(True/false) Sharing a drink with someone who tested


positive for TB can potentially result in transmission.
False - must be breathed in

What is the second leading cause of death from infectious


disease worldwide?
TB
Always worse in other countries

With regard to TB, the infection in about ___% of those


infected becomes latent and may be reactivated later in
life.
95%

Why might someone who is exposed to TB have a


nonreactive test result, despite having inhaled the
droplets?
Have not built up antibodies to TB yet - may take up to 6
weeks

Why might someone with HIV have a nonreactive TB skin


test, despite having inhaled TB droplets?
Immunosuppressed - WBC count is 0 & cannot build up
antibodies for test to be reactive

A patient arrives to the community clinic 48 hours after


receiving a TB skin test to have it evaluated and the
nurse discovers 15 mm induration. The patient is
diagnosed with latent TB. What should the nurse say/do?
"What we do know at this point is that you have been
exposed to TB because you responded to the test. This
means that you may go on to develop TB."
Patient must go to health department - will conduct
thorough history (traveling included) & CXR

A patient with diabetes has her TB skin test read and the
nurse discovers 11 mm induration. Further testing reveals
no symptoms and a negative chess x-ray. What does the
nurse suspect?
A. Active TB infection
B. No TB infection
C. Latent TB infection
D. Further testing is required
C. Latent TB infection

(True/false) An HIV-positive individual is diagnosed with


latent TB; this person can either choose to be treated
prophylactically or not receive treatment.
False - HIV patients must always be treated for latent TB

Why might patients diagnosed with latent TB choose not


to receive the prophylactic medications?
Develop antibiotic resistance
Hepatotoxic drugs - alcoholics, drug users, or liver disease
patients want to keep using & not take TB meds

What testing must be completed before administering TB


medications to a patient?
Liver status/LFT: enzymes, clotting factors, etc.

What populations are at an increased risk for contracting


TB?
Homeless
Prisons
Nursing homes
Migrants/immigrants/TB endemic in home countries
Immunocompromised (HIV, chemo, organ transplant)

What preventive action might a nurse take when


admitting older adults to a nursing home?
Test for TB before allowing in facility

Why might an individual show induration on a TB skin test


when he or she has not ever been exposed or had TB?
Had BCG vaccine

What diagnostic tools are used to determine the presence


of TB?
Skin test (48 - 72 hours)
Quantiferon test (blood draw rather tan skin test - can get
next day)
Chest x-ray
Acid-fast bacillus (AFB) smear if symptoms (cough sputum
into petri dish, stained)
H & P (traveling history)
Cultures (can take up to 6 weeks - quarantine until then)

For patients with HIV, a positive chest x-ray, or close


contact with someone who has TB, a TB skin test with ___
mm induration is considered positive.
5 mm

For patients with diabetes, ETOH abuse, drug use/IVDs,


foreign-born where TB is common, medically
underserved, low-income, long-term care, in prisons, or
children under 4, a TB skin test with ___ mm induration is
considered positive.
10 mm

For patients without definite medical conditions, a TB skin


test with ___ mm induration is considered positive.
15 mm

A homeless patient arrives at the community clinic to


have his TB skin test read and the nurse discovers 10 mm
induration. What is the best course of action?
Considered positive
Assess for symptoms
CXR to ensure not contagious

(True/false) An annual chest x-ray is recommended for


medical personnel to ensure TB is not present.

False - too much radiation

(True/false) Anergy testing, which is used to obtain


information regarding the competence of the cellular
immune system, is recommended routinely for screening
for those with HIV.
False - no longer recommended; now look at WBC count to
assess immune system

Two-step TB testing, which utilizes a booster effect, is


used for what individuals?
People over 55 years
New employees to establish accurate baseline

A 60-year-old woman receives a TB skin test and nothing


happens. What might be the basis for this? What is the
best course of action?
Immunocompromised
False negative: sluggish immune system does not respond
quickly enough
Repeat skin test in 1 week

A 71-year-old man receives a TB skin test and upon


reading it, the nurse notes that there is no response. The
man receives a second skin test (per two-step testing) a
week later. Why might the second test be positive when
the first was negative?
Booster effect - sluggish immune system did not respond
fast enough to first skin test, even though the man was
positive for TB

The Bacillus Calmette-Guerin (BCG) vaccine has what


effect on TB skin tests?
Can lead to false positive due to antibodies

Patients with active TB infections are considered


contagious for at least ___ weeks after starting medication
treatments.
2 weeks

What intervention may be necessary for a patient with an


active TB infection who refuses to wear a mask outside or
take his medications?
Quarantine in hospital - DOT!

In addition to medications, what interventions are part of


the treatment plan for TB?
Contagious for 2 weeks after beginning meds
Repeat labs - 3 negative smears (early morning sputum)
Home isolation
Masks when outside
Monroe House (for homeless with TB)

Persons who have latent TB infections have a ___%


chance of developing TB disease. This risk is greatest
during the first year after becoming infected and can be
activated by immunosuppression.
10%

On average, people have a ___% chance of becoming


infected with TB if they spend 8 hours a day for 6 months
or 24 hours a day for 2 months working or living with
someone with active TB.
50%

How is treatment for multi-drug-resistant (MDR) TB


different than that of regular TB?
Prolonged treatment (2 years)
Multiple drugs
High mortality rate

What types of interventions may be necessary for


individuals with drug-resistant TB to ensure adherence?
Directly-observed therapy (DOT)
Directly-observed prophylactic therapy (DOPT)

Why has there been a resurgence of TB in the United


States?
HIV/AIDS epidemic
Increased immigration
Increased poverty, injection drug use, homelessness
Poor compliance with treatment
Increased residents in long-term care
Deterioration of public health services
Onset of MDR TB

Giant rats may now be used to sniff out TB infections.


How might this be beneficial compared to traditional
treatment?

Cheaper
More efficient - success rate 67%
2000 samples/day vs. technicians that can only process 20
samples/day

What institutions in Arizona are aimed at controlling TB?


Maricopa County Department of Health
Diagnosis & treatment
DOT/DOPT
Monroe House

What factors should be evaluated when determining if a


patient should receive prophylactic treatment of latent
TB?
Benefits of treatment - evaluate individual's risk for
developing TB
Assess level of commitment to completion of treatment &
resources available to ensure adherence

What kinds of people should be given high priority for


latent TB infection treatment?
+IGRA/5+ mm induration:
HIV
Recent contacts of TB case
Fibrotic changes on chest radiograph consistent with old TB
Organ transplant recipients
Immunocompromised
Taking steroids, etc.
+IGRA/10+ mm induration:
Recent immigrants (<5 years) from high-prevalence
countries
Injection drug users
Residents/employees of high-risk congregate settings
(nursing homes, homeless shelters, correctional facilities)
Mycobacteriology lab
Children under 4
Children/adolescents exposed to adults in high-risk
categories

What symptoms often result from medications used to


treat TB?
Hepatitis
GI upset
Drug interactions

Bleeding problems
Flu-like symptoms
Rash
Turns urine, saliva, tears orange (discolors contact lens)
Photosensitivity
Birth control pills/implants less effective
Hepatic enzyme elevation
Peripheral neuropathy
Mild CNS effects
Joint aches
Hyperericemia (uric acid in blood)
Optic neuritis
Ototoxicity & nephrotoxicity

A school nurse notes that 60 children have missed days of


high school because of pertussis this past year and this
rate has been relatively constant for the past 5 years. The
nurse plans to work with the community to increase
awareness of the seriousness of this disease for children
younger than 6 months of age and to raise and maintain
the immunization rates, because in this community the
pertussis is:
A. Endemic.
B. Epidemic.
C. Pandemic.
D. Sporadic.
A. Endemic.

The World Health Organization (WHO) developed the Five


Keys to Safer Food campaign in 2001 to address the
problem of foodborne and waterborne diarrheal diseases
worldwide. This campaign emphasizes which of the
following practices?
A. Keep clean, separate raw and cooked, cook thoroughly.
B. Never use raw, always cook, buy better.
C. Wash, cut, cook, and throw away.
D. Wash, cover, and always refrigerate.
A. Keep clean, separate raw and cooked, cook thoroughly.

An American takes a long-awaited vacation in sunny


Mexico, spending days on the beach eating fresh
raspberries from a nearby vendor and drinking bottled
water. The tourist may be altering:
A. Agent-host-environment interaction.
B. Circadian rhythms.
C. Herd immunity.
D. Host resistance.

A. Agent-host-environment interaction.

An example of secondary prevention of infectious disease


is:
A. Malaria chemoprophylaxis.
B. Pneumocystis carinii pneumonia chemoprophylaxis for
people with AIDS.
C. Quarantine.
D. Restaurant inspections.
C. Quarantine.

In an effort to address West Nile virus, a community


increased livestock immunization, began a vector control
program, and initiated a community campaign to
eliminate standing water reservoirs. This best exemplifies
communicable disease control through:
A. Health education.
B. Multisystem approach.
C. Improved public health infrastructure.
D. Reduction of environmental hazards.
B. Multisystem approach.

Emerging infectious diseases may arise as a result of


factors operating singly or in combination, and these
factors may include which of the following (select all that
apply)?
A. Environmental changes.
B. Host behavior.
C. Improved surveillance.
D. Microbial adaptation.
E. Public health infrastructure deterioration.
A. Environmental changes.
B. Host behavior.
D. Microbial adaptation.

What are some primary prevention techniques for


preventing the occurrence of disease?
Responsible sexual behavior
Malaria chemoprophylaxis
Tetanus boosters, flu shots
Rabies pre-exposure immunization
Safe food-handling practices in the home
Repellants for preventing vector-borne disease
Not allowing unvaccinated kids to attend school
Regulated & inspected municipal water supplies

Bloodborne pathogen regulations


Restaurant inspections

What are some secondary prevention techniques for


preventing the occurrence of disease?
Immunoglobulin after hepatitis A exposure
Immunization & chemoprophylaxis for meningococcal
outbreak
Rabies post-exposure immunization
TB screening for healthcare workers
STI partner notification
HIV testing & treatment
Quarantine

What are some tertiary prevention techniques for


preventing the occurrence of disease?
Pneomocystis pneumonia (PCP) chemoprophylaxis for
people with AIDS
Regular inspection of hands & feet/protective footwear &
gloves to avoid trauma/infection for leprosy clients who
have lost sensation in those areas

A school nurse is teaching a class of sophomores about


the relationship between the risk of sexually transmitted
disease (STD) and risk-taking behaviors. A key point to
include is:
A. All STDs are easily preventable with consistent condom
use.
B. Once a young woman is pregnant, she is no longer at
risk for most STDs.
C. STDs are most likely to be transmitted during a
student's initial sexual encounter.
D. Use of alcohol and drugs makes a student more likely
to make decisions that result in exposure to and infection
with STDs.
D. Use of alcohol and drugs makes a student more likely to
make decisions that result in exposure to and infection with
STDs.

A client newly diagnosed with human papillomavirus


(HPV) infection, herpes simplex virus 2 (HSV-2) infection,
and syphilis asks, "Okay, so how do I get rid of all this
stuff?" In developing a plan of care, the nurse recognizes
that it is essential to address:
A. Correct use of condoms to prevent transmission of all
sexually transmitted diseases (STDs).

B. Cures for each of the STDs identified.


C. Risk of skin-to-skin contact in transmitting the
identified STDs.
D. Safety of sexual contact in the absence of lesions.
C. Risk of skin-to-skin contact in transmitting the identified
STDs.

A nurse is assigned to teach clients sexually transmitted


disease (STD) prevention information. The nurse updates
her teaching plan to incorporate new guidelines from the
Centers from Disease Control and Prevention (CDC). She
includes which of the following as updated information
during her next teaching session?
A. Always use spermicides with condoms to reduce the
risk of contracting chlamydia or gonorrhea.
B. Condoms can be effective in preventing infections
transmitted by fluids from mucosal surfaces, but are not
always effective in preventing infections transmitted by
skin-to-skin contact.
C. Condoms should not be used during oral sex, because
they are not effective in preventing transmission of
infection.
D. When genital ulcers are present, condoms should be
used to prevent the spread of infection.
B. Condoms can be effective in preventing infections
transmitted by fluids from mucosal surfaces, but are not
always effective in preventing infections transmitted by
skin-to-skin contact.

A woman comes to the community health center


complaining of increasing lower abdominal pain, fever,
and abnormal menses for several months. During the
assessment, the client indicates that she is aware that
her husband has had multiple sex partners in the past
two years. Appropriate intervention by the nurse would
be to:
A. Arrange to have the client referred for medical
evaluation for pelvic inflammatory disease (PID) and
appropriate intervention and treatment.
B. Contact the health department to confirm the spouse's
diagnosis of Chlamydia infection to determine the client's
exposure, give the client antibiotics, and have her return
to the clinic if symptoms worsen.
C. Provide sexually transmitted disease (STD) prevention
and treatment education and refer the client to the health
department for STD screening for gonorrhea and/or
Chlamydia infection.

D. Supply the client with nonsteroidal antiinflammatory


drugs and caution her to call the after-hours call doctor if
her symptoms worsen.
A. Arrange to have the client referred for medical
evaluation for pelvic inflammatory disease (PID) and
appropriate intervention and treatment.

The clients most at risk of reactivation of latent infections


of tuberculosis (TB) are:
A. Immunocompromised persons, substance abusers, and
those with diabetes.
B. Individuals previously treated for TB.
C. Long-term cigarette smokers.
D. Persons with new-onset asthma or emphysema.
A. Immunocompromised persons, substance abusers, and
those with diabetes.

Community health nurses conducting health education


among populations vulnerable to HIV infection should
explain the natural history of the infection, including the
fact that HIV infection may go undetected during the
primary infection stage because:
A. Antibody test results are typically negative.
B. Antibody production by the immune system increases.
C. Incubation period is prolonged.
D. Symptoms include myalgias, sore throats, and rash.
A. Antibody test results are typically negative.

A client comes to the local clinic with acute symptoms of


fever, nausea, lack of appetite, malaise, and abdominal
discomfort. During the course of the assessment, the
nurse determines that the client is a health care aide
working at a daycare center. These facts are important
because:
A. Acute hepatitis B is self-limiting.
B. Hepatitis A outbreaks commonly occur in facilities
where staff change diapers.
C. Hepatitis C is a "silent stalker."
D. Individuals with chronic liver disease are at greater risk
for hepatitis A.
B. Hepatitis A outbreaks commonly occur in facilities where
staff change diapers.

Many behaviors place any individual-regardless of age,


gender, ethnicity, or other characteristics-at greater risk
for sexually transmitted diseases (STDs). The nurse

should include primary prevention interventions in all


client encounters through the discussion of:
A. Partner notification.
B. Safer sex.
C. Standard precautions.
D. STD testing.
B. Safer sex.

A population-level tertiary prevention intervention


typically carried out by nurses caring for those with
communicable disease in the community is:
A. HIV test results counseling.
B. Needle exchange.
C. Partner notification.
D. Instruction in standard precautions.
D. Instruction in standard precautions.

Disease and event surveillance systems exist to help


improve the health of the public through the systematic
and ongoing collection, distribution, and use of healthrelated data. A public health nurse utilizes the data in
available surveillance systems to make a presentation to
a community planning committee that is interested in the
problem of childhood obesity. This use of the data for
facilitating community planning is a good example of
helping the community to:
A. Avert a health care epidemic.
B. Evaluate prevention and control measures.
C. Identify cases for fact finding.
D. Manage endemic health problems.
D. Manage endemic health problems.

Disease surveillance, a critical role function for the public


health nurse, can best be defined as the ongoing,
systematic collection, analysis, interpretation, and
dissemination of specific health data relevant to public
health. The nurse's knowledge and understanding of
surveillance systems is critical to:
A. Detecting epidemics and pandemics.
B. Facilitating planning and policy changes.
C. Improving the quality and usefulness of the data
collected.
D. Understanding the relationship between epidemiologic
principles.
C. Improving the quality and usefulness of the data
collected.

A state public health region reported 39 cases of


meningitis in children 15 years of age and younger to
date this year. Seven of those children died. The total
population of the region is 780,000, of whom 84,000 are
children age 15 years old and younger. Only four cases of
meningitis were reported in the public health region
during the previous year. No other public health region in
the state has an incidence of meningitis that is higher
than expected for that region. Based on the information
given, the relative frequency of meningitis in the region at
this time can best be described as:
A. Endemic.
B. Epidemic.
C. Pandemic.
D. Sporadic.
B. Epidemic.

A business executive develops flu-like symptoms 1 day


after returning by air from a trans-Atlantic 2-day
conference that involved lengthy meetings into the
evening. The scenario best illustrates the interaction of:
A. Host and agent.
B. Host, agent, and environment.
C. Risk and causality.
D. Morbidity and disease.
B. Host, agent, and environment.

A nurse teaches an asthmatic client to recognize and


avoid exposure to asthma triggers and assists the client's
family in implementing specific protection strategies in
the home, such as removing carpets and avoiding pets.
This nurse's activities can best be described as:
A. Comprehensive assessment.
B. Primary prevention.
C. Secondary prevention.
D. Treatment intervention.
C. Secondary prevention.

A nurse is concerned about the accuracy of the purified


protein derivative (tuberculin) test in screening
individuals with tuberculosis exposure for followup chest
radiography. The nurse's concern is related to which
aspect of the test's validity?
A. Reliability.
B. Sensitivity.

C. Specificity.
D. Variability.
B. Sensitivity.

During the last stage of AIDS, many opportunistic


infections proliferate in persons with weakened immune
systems. HIV-infected individuals who live near one
another in long-term care facilities, drug treatment
facilities, and prisons should be carefully screened before
admission because they are at greater risk for acquiring:
A. Fungal disease.
B. Invasive cervical cancer.
C. Oral candidiasis.
D. Pulmonary tuberculosis (TB).
D. Pulmonary tuberculosis (TB).

A young female client visits a local clinic, seeking


treatment for a urinary tract infection (UTI). During the
assessment, the nurse conducts an extensive sexual
history/activity screening to determine the risk for a
sexually transmitted disease and potential complication
of:
A. Genital warts.
B. Herpes simplex virus 1.
C. Pelvic inflammatory disease (PID).
D. Primary syphilis.
C. Pelvic inflammatory disease (PID).

The primary prevention of hepatitis C virus (HCV)


includes:
A. Counseling and follow-up of infected clients.
B. Routine testing for health care workers.
C. Screening of blood products.
D. Testing of high-risk individuals.
C. Screening of blood products.

A strategy that may improve compliance with tuberculosis


(TB) treatment is the:
A. Expansion of directly observed therapy.
B. Increased tuberculosis testing protocols.
C. Increased confirmatory testing protocols.
D. Treatment compliance education.
A. Expansion of directly observed therapy.

Factors that contribute to newly emerging or reemerging


infectious disease can be related to microbial adaptation

and changes made by the infectious agent. However,


most of the emergence factors are related to
environmental changes and:
A. Consequences of human activities/behaviors.
B. Increase in the number of vectors.
C. Industrialization and urbanization.
D. Unpredictable variances in the climate.
A. Consequences of human activities/behaviors.

For the nurse to fully understand the threat associated


with the release of biological agents and participate in an
appropriate response, the Centers for Disease Control and
Prevention (CDC) has stated that the biological agents
most likely to be employed are those that both have a
potential for high mortality and can be easily
disseminated to produce:
A. Immediately severe symptoms.
B. Major panic and social disruption.
C. Multisyndrome effect.
D. Person-to-person transmission.
B. Major panic and social disruption.

To reach the desired goal of maximizing the full


immunization rates for preventable communicable
disease and increasing herd immunity levels, it is crucial
for the nurse to:
A. Assume that the primary care physician has provided
all appropriate immunizations.
B. Check an individual's immunization status at each and
every visit.
C. Support parents who are reluctant to immunize their
children.
D. Understand the difficulties in obtaining and
maintaining immunization schedules.
B. Check an individual's immunization status at each and
every visit.

Protecting the nation's food supply from contamination by


all the virulent microbes is complex, costly, and time
consuming. However, much foodborne illness, regardless
of causal organisms, can be prevented through simple
changes in:
A. Food preparation, handling, and storage.
B. Importation regulations.
C. Pesticide usage.
D. Animal breeding practices.
A. Food preparation, handling, and storage.

Immunity to disease through vaccination is known as:


A. Natural immunity.
B. Resistance.
C. Acquired immunity.
D. Herd immunity.
C. Acquired immunity.

A multisystem approach to community disease control


would include such interventions as: (select all that apply)
A. Community action programs.
B. Control of vectors.
C. Improved surveillance systems.
D. Legislation.
E. Provision of chemotherapy.
A. Community action programs.
B. Control of vectors.
C. Improved surveillance systems.
D. Legislation.
E. Provision of chemotherapy.

The epidemiologic triangle consists of three components,


which are the following except:
A. Host
B. Mode of transmission
C. Agent
D. Environment
B. Mode of transmission

The organism that harbors and provide nourishment for


another organism is called a:
A. Host
B. Mode of transmission
C. Agent
D. Environment
A. Host

What is the leading communicable disease worldwide?


Influenza

The ___ facilitates the transmission of the agent from the


host to another susceptible host.
Environment

What are some environmental causes of the emergence


of new diseases?
Deforestation
Changes in ecosystem
Flood
Drought
Global warming

What human behaviors are related to the emergence of


new diseases?
Sex
Drug use
Travel
Diet
Childcare
Outdoor recreation

What type of community intervention does the


government employ to prevent food-borne illness?
Surveillance - restaurant inspections/employees washing
hands (hep A is fecal/oral)

___ is the ability of an agent to produce a specific clinical


reaction after infection occurs.
Pathogenicity

___ is an agent's ability to produce a severe pathological


reaction.
Virulence

___ is the ability of an agent to produce a poisonous


reaction.
Toxicity

___ is the ability of an agent to penetrate through and


spread throughout a tissue.
Invasiveness

___ is the ability of an agent to stimulate an


immunological response.
Antigenicity

What are some examples of environmental alterations to


prevent spread of disease?

Mosquito nets/repellants to avoid bug bites


Installing swear systems to prevent fecal contamination of
water supplies
Washing utensils after contact with raw meat to reduce
bacterial contamination
What are the symptoms of TB?
Cough
Fever
Hemoptysis
Chest pains
Fatigue
Weight loss

(True/false) Exposure to an infectious agent always leads


to an infection.
False

(True/false) Infection does not always lead to disease.


True

___ are vertebrate animals that transmit infections to


humans under natural conditions. This can occur through
bites, inhalation, ingestions, and direct contact.
Zoonoses

What are some examples of zoonoses?


Rabies
Toxoplasmosis
Cat scratch fever
Brucellosis
Listeriosis
Salmonellosis
Health care economics is ___
- it analyzes 3 things
- it also involves taxation __ and ___.
a specialized field of economics that describes and
analyzes the production,distribution, and consumption of
goods and services
-as well as a variety of related problems such as finance,
labor, and
taxation.

The goal of health care economics__

overcome scarcity by making good choices and providing


essential services.

Quarantine is
- incubation (define)
ISOLATION during a communicable disease (especially
during incubation ) to prevent spread.
incubation: The development of an infection from the time
the pathogen enters the body until signs or symptoms first
appear.

sanitation refers to the


promotion of hygiene and prevention of disease by
maintenance of health-enhancing (sanitary) conditions.

The Shattuck Report


-a landmark document, made a tremendous impact on
sanitary progress.
the first attempt to describe a model approach to
organization of public health.

official health agencies


- who operates them? (2)
- function? including 2 types of services
Agencies operated by state or local governments to provide
a wide range of "public health services" including
community and public health nursing services

voluntary health agencies


- when did it started to emerge?
sometimes called private agencies or nongovernmental
organizations [NGOs]
-1800s

3 core public health functions ( in order)


APA
- this is performed by __ or ___services
assessment, policy development,assurance
= public or private

Assessment
- define
- it is a continuous process of ____ (2) information about
__
- function?
- morbidity?
measuring and monitoring the health status and needs of a
designated community or population.
-a continuous process of collecting data and disseminating
information about health,diseases, injuries, air and water
quality, food safety, and
available resources.
This function helps to identify morbidity, mortality, and
causative factors.
- rate of incident

Policy development is the


" LEADERSHIP , ADMINISTRATION & guideline for present
and future decision affecting public's health"

Assurance is
- 2 MORE FUNCTIONS?
the process of translating established policies into
services.
1. ensures that population based services are provided,
whether by public health agencies or private sources.
2. monitors the quality of and access to those services.

medically indigent
(those who are unable to pay for and totally lack medical
services)

Public Health Service (PHS).


- its concerned with ___
It is concerned with the broad health interests of the
country and is a functional (not organizational) unit of
DHHS

DHHS
Department of Health and Human Services

Proprietary health services are


- private or public?
- profit, non profit or both?
privately owned and managed. They may be nonprofit or
for-profit.

gross domestic product (GDP)


the total amount of goods and services produced within a
year; whereas health spending.

Economics
science of making decisions regarding scarce resources.
(Study of how societies decide what to produce, how to
produce it, and how to distribute what they produce)
Economics permeates our social structureit affects and is
affected by policies.,

Microeconomic theory is
concerned with supply and demand.

Macroeconomic theory is
concerned with the broad variables that affect the status of
the economy as a whole.

Supply is
the quantity of goods or services that providers are willing
to sell at a particular price.

Demand
Consumer willingness and ability to buy products

adverse selection
-define
-ex
the tendency for people with the greatest probability of loss
to be the ones most likely to purchase insurance.
(ex: when high-risk patients are denied insurance or care)

Third-party payments are


- reimbursement made by who?

monetary reimbursements made to providers of health care


by someone other than the consumer who received the
care.

consumer-driven health plans (CDHP) and health savings


accounts (HSAs).
- deductible is high or low?
-example?
consumer-directed.
Type of medical insurance that combine a high deductible
health plan with a medical savings plan which covers some
out of pocket expenses.
- employment plan ( blue cross blue shield,

deductible
( yearly health insurance bill)
A certain amount of money that the patient must pay each
year toward his or her medical expenses before health
insurance benefits begin.

cost sharing
a situation where insured individuals pay a portion of the
healthcare costs, such as deductibles, coinsurance or copayments.
( ex: auxilio mutuo private health insurance)

Medicare
A federal program of health insurance for persons 65 years
of age and older

Medicaid
A public assistance program designed to provide healthcare
to poor, disabled and disadvatanged americans

capitation rates
(fixed amounts of money paid per person by the health
plan to the provider for covered services)

retrospective payment

which is reimbursement for a service after it has been


rendered

rendered
Provided, given

reimbursement
paying back

fee-for-service
A system under which doctors and hospitals receive a
separate payment for each service that they provide

Prospective payment
payment method based on rates derived from predictions
of
annual service costs that are set in advance of service
delivery.
-the payment amount or reimbursement with a set rate for
certain procedures is known in advance

diagnosis-related groups (DRGs)


a system of analyzing conditions and treatments for similar
groups of patients used to establish Medicare fees for
hospital inpatient services; patients are classified by their
principal diagnosis, surgical procedure, age, and other
factors.

cost shifting.
- read
- define
This practice of charging different prices to different
consumers most often affects those without health
insurance who are paying out-ofpocket for care.
- Lack of insurance, uninsured populations, and
uncompensated care are covered by charging more to
those who can pay. This practice is referred to as

managed care
It refers to systems that coordinate medical care for
specific
groups to promote provider efficiency and control costs.

Medical home
seeing the same health care provider for regular care

health maintenance organization (HMO)


- define
- certain specification
is a system in which participants prepay a fixed monthly
premium
to receive comprehensive health services delivered by a
defined network of providers.
The HMOs are the oldest model of coordinated or managed
care

A preferred provider organization (PPO)


a network of physicians, hospitals, and other health-related
services that contracts with a third-party payer
organization to provide comprehensive health services to
subscribers on a fixed FFS basis.

point-of-service (POS) plan,


which permits more freedom of choice than a standard
HMO or PPO.

rationing in health care refers to


limiting the provision of health care services to save costs.

Competition means
a contest between rival health care organizations for
resources and clients

Regulation refers to
mandated procedures and practices affecting health
services delivery that are enforced by law

Managed competition,
- what does it combine?
combine with market competition to achieve cost
savings with government regulation to achieve expanded
coverage.

single-payer system
- this lead to?
When the government collects taxes for healthcare from all
citizens and then uses the collected money to pay for the
citizens healthcare services.
- universal coverage

universal coverage
(everyone would have health insurance of some type,
assuring better access to care).

SUMMARY
SUMMARY

history
Primitive practices of early centuries were replaced with
more advanced sanitary measures by the Greeks and
Romans. The Middle Ages saw a
serious health decline in Europe, with raging epidemics
leading to extensive 19th-century reform efforts in England
and, later, in the United States.

Organized health care in the United States developed


slowly. Public health problems, such as the need for
isolation
of persons with communicable diseases and control of
environmental pollution, prompted the gradual development
of
official interventions.
For example, quarantines to control
the spread of communicable disease were imposed in the
late 1700s. Sanitary reform was pursued more vigorously
during the 1800s. Local and then state health departments
were formed starting in the late 1700s. By the early 1900s,
the federal government had assumed a more active role in
public health, with a proliferation of health, education, and
welfare services.

. Public health services include three core public health


functions:
assessment, policy development, and assurance

Private health services are the unofficial arm of the community health system. They include (2) agencies
voluntary nonprofit agencies
privately owned (proprietary) and for-profit
agencies.

The delivery and financing of community health services


has been significantly affected by various legislative acts.
These acts have prompted such innovations as (6)
HAM- steak- herring- fish
Health insurance
assistance for the poor, the elderly, and the disabled;
money to train health personnel and conduct health
research;
standards for health planning and delivery
health protection for workers on the job;
financing of health services.

Health care economics studies the(3)


- function?
production, distribution, and consumption of health care
goods and service
maximize the use of scarce resources to benefit the most
people.

This science underlies the financing of the health


care system. It is influenced by (2)
microeconomics
macroeconomics

Health care is funded through public and private


sources, which fall into three categories:
third-party payers,
direct consumer payment,
private support.

Several trends and issues have influenced community


health care financing and delivery and are important in
understanding health care economics and helping to
improve community health. They include
cost control,
financial access,
managed care,
health care rationing,

competition and regulation,


managed competition,
universal coverage, a single-payer system, and health care
reform

The changing nature of health care financing has


adversely affected community health and its practice in
three important ways:
(1) retrospective payment without
limiting costs, tax-deductible employer contributions for
health care coverage and nontaxable employee health
benefits, together with a lack of consumer involvement in cost
sharing, have created disincentives for efficient use of
resources;
(2) because the health care system traditionally
has reimbursed only for treatment of the ill or disabled,
with
no reward for health promotion and prevention efforts, it
has
promoted incentives to focus only on illness care; and
(3) the competition model, which has long driven up health
care
costs and eliminated many from being able to afford health
care services, has generated a conflict with the basic public
health values of health promotion and disease prevention
for
all persons.

Health care reforms have been proposed, but the


United States remains the only industrialized nation
without
some type of
universal health coverage.

We also rank significantly lower than most other


developed countries on health indicators, such as infant
mortality and life expectancy.
Public health nurses can lead the effort in making health
care more accessible to all citizens and encourage policies
and
practices that promote health, rather than reward illness.
active immunity
acquired after exposure to a disease or via immuneizations

OSHA
sets health and safety work standards employees are in
legally required to protect workers from hazards and
employees have a right to know what hazards they are
exposed to

passive immunity
newborns through maternal antibodies

.risk assessment
qualitative and quantitative evaluation of the risk posed to
human health for the environment by the actual or
potential presence or use of specific pollutants

needs assessment
needs assessment systemic appraisal of type depth and
scope of problems as perceived by clients health providers
or both

natural immunity
species determinate resistance to an infectious agent

WIC
woman and children Supplemental Nutrition Program
provides federal grants to states for supplemental food
health care referral and nutrition education for low-income
pregnant breastfeeding and non-breastfeeding postpartum
women infants and children up to five years of age who are
found to be at a nutritional risk

primary prevention
type of intervention that seeks to promote health and
prevent disease from the beginning involves health
promotion and education

secondary prevention
secondary prevention intervention that seeks to detect
disease by screening and providing healthcare early in the
progression before clinical signs and symptoms become a
apparent in order to make an early diagnosis and begin
treatment

tertiary prevention

continued long-term health care interventions that begins


once the disease is obvious the aim is to interrupt the
course of the disease reduce the amount of disability that
might occur and begin rehabilitation

SARS
severe acute respiratory syndrome spread via airborne
droplets indirect.contact and by close contact with an
infected person there is no cure as a person is suspected of
having sars they should be admitted to the hospital
immediately and kept under Isolation if in an outpatient
setting the person must be rapidly divertid to a separate
area to minimize transmission to others

palliative care
care to alleviate symptoms by meeting the special needs of
and providing comfort for the dying client and their families
by the nurse

long term care


care designed to help a person with basic activities of daily
living that is given to individuals over a sustained amount
of time

Native American care


verbal communication is low tone of voice and the listener
is expected it to be attentive
nonverbal communication direct eye contact is often
considered disrespectful
touch a light touch of the person's hand instead of a firm
hand is often used as a greeting
family organization usually have a close extended family
and the emphasis tends to be on the needs of the family
rather than the individual needs
time perception of health often past oriented harmony of
mind body spirit and emotions with nature
alternative healers a medicine man and shaman
self care practices Herbs cornmeal and medicine bundle
biological variations cleft uvula lactose intolerance and skin

color
food preferences include blue cornmeal fruits Game and
Fish.nutritional excess of carbohydrates and calories
risk factors are diabetes malnutrition tuberculosis infant
and maternal mortality

culture and sensitivity


appreciate and respect for individual clients beliefs and
values
appreciate and value diversity
appreciate and genuinely care about those of other
cultures
recognize how your own cultural background may influence
professional practice

Good luck
...
3 shared commonalities
geographic, common interest, community of solution

community of solution
group of people who come together to solve problems that
affects all of them, ex: guidance counselors, counties
working for water pollution

common interest
collection of people with interest or goal that bind them
together, ex: MADD, disabled community, church groups,
gay community

population
all the people occupying an area, or all those who share
one more characteristic, may not necessarily interact

aggregates
a mass or grouping of distinct individuals who are
considered as a whole and are loosely associated with each
other, broader term then population

aggregates
communities and populations are types of

goal of community health nursing


health promotion, disease prevention, and health
protection

promotion of health
health education, preventative care services, healthy
people 2010, raise levels of wellness for individuals,
families, populations and communities

prevention of health problems


anticipating and averting problems or discovering them as
early as possible to minimize potential disability and
impairment

primary prevention
to keep illness from occurring, hand rails, immunizations

secondary prevention
efforts to detect and treat existing health problems.
Screenings: htn, drug abuse, breast exam, DM test; to
intervene to control or eradicate the problem

tertiary prevention
attempts to reduce the extent and severity of health
problems so as to minimize disability and restore or reserve
function: rehab after CVA.

tertiary prevention
someone that has an eating disorder, following up with
counseling

treatment of disorders
focuses on illness by: direct service, indirect service,
development of programs to correct unhealthy conditions

indirect service
referring to someone, calling another professional to help
solve problem for client

rehab

efforts to reduce disability and restore function as much as


possible

evaluation
process by which the practice is analysis, judged, and
improved according to established goals and standards

research
investigation to discover facts affecting community health
and community health practice, solve problems, and
explore improved methods of health science

healthy people 2010


increase quality and years of life, eliminate health
disparities

healthy people 2010


emphasis on community partnerships,

determinant of health
biology

determinant of health
behavior

determinant of health
social & physical environment

determinant of health
polices and interventions

community health status


health of total population

community health status


measured by birth and death rates, morbidity statistics

healthy people 2010 indicators


physical activity, obesity, tobacco use, substance abuse,
sexual behavior, mental health, injury and violence,
environmental quality, immunization, access to healthcare

core public health functions

assessment, policy development, assurance

role of community health nurse


clinician, educator, advocate, collaborator, leader,
researcher, case manager

clinician
most familiar role, ensures health services are provided

educator / teacher
client is not acutely ill, so can absorb information better,
wide audience can be reached

advocate
2 goals: help clients gain greater independence or self
determination; make the system more responsible and
relevant to the needs of clients. Every client has the right
to receive just, equal and humane treatment

collaborator
work jointly with others, need communication skills

leader
focuses on change, nurse becomes an agent of change,
influence people to think and behave differently about their
health

researcher
evidence based practice, systematic investigation,
collection, and analysis of data for solving problems and
enhancing community heath practice

case manager
has become the standard method of managing health care
in delivery systems in US. Evaluates progress to ensure
that clients' multiple service needs are met in a costeffective manner.

settings for community health nursing practice


homes, ambulatory services, schools, occupations health
settings, hospice, parish, forensics, corrections (prison)

ambulatory service settings

community health centers, clinics, day care centers, health


departments, services for select groups such as migrant
camps, native american reservations, correctional facilities,
parishes

early home care nursing


religious and charitable groups, during industrial revolution

Florence Nightingale
early 19th century, crimean war, record keeping and
statistics, organized nursing care, kitchens, laundries,
"notes on nursing"

public health nursing


1900-1970, health and welfare of general public, not just
the poor

Lillian Wald and Mary Brewster


started Henry Street Settlement to provide nursing and
welfare services

societal influences on the the development of community


health nursing
tech, progress in thinking, education, role of women,
consumer movement, economic forces

Federal agencies
DHHS, EPA, Homeland security, D of Agriculture, Dept of
Ed, VA

State agencies
Medicare, Medicaid, SCHIPS

Local health department


carries out most state laws and policies, most
direct/immediate care

Indian health services


IHS: part of the U.S. Department of Health and Human
Services and is responsible for the medcial care provided to
Native Americans and Native Alsaskans. Established in
1954 and provides health care.

medicare

A program added to the Social Security system in 1965 that


provides hospitalization insurance for the elderly and
permits older Americans to purchase inexpensive coverage
for doctor fees and other health expenses.

medicaid
a federally aided, state operated program that provides
medical benefits for low-income persons in need of medical
care

Medicare prospective payment


"you will get 10 paid visits"

Medicare prospective payment


5 criteria will be met: OASIS form, centers for Medicare and
Medicaid Services (CMS), Medicare documentation (see
page 894)

OASIS
outcome, assessment information set: very detailed form

Medicare prospective payment


need a skilled service, not just "a bath", services must be
intermittent care

Medicare prospective payment


the payment plan usually last 60 days, then needs to be reevaluated.

durable medical equipment


bedside commodes, walkers, beds, can be used again.

supplies
thrown away after each use

home health advantages


convenience, access, relationship, cost, outcomes, teaching
opportunities

multiplicity of problems
finding more then one problem

maintaining balance

intimacy vs professional distance, risk and safety, assisting


and devaluing client, dependence and independence, cost
containment and quality

hospice
medicare is the main payment, even if you're under 65

hospice
you have 6 months or less to live

hospice
pain medication or no aggressive chemo or procedures

compounding pharmacist
combining drugs in a unique way in a hospice setting

durable power of attorney


health issues, or healthcare proxy

I-Prepare
prevent, resident, environment, past work, activities,
resources, education

web of causation
the combination of factors that lead to a problem

contaminant
organic or inorganic that enters medium that renders it
impure

toxic agent
poisonous substance in the environment

overpopulation
exceeds ability of it's ecosystem to either support needs; 4
factors (starvation, disease, wars, aide from other
countries)

air pollution
high in industrial areas

radon
odorless, colorless, causes cancer

lead poisoning
affects CNS, old paint,

contact lens solution


can cause birth defects

psychological hazards
noise, overcrowding, lack of natural beauty

phases of survey method


planning, collection, analysis & presentation

traditional nursing
work as part of team, acute setting, with other
professionals

public health nursing


in the community, often alone, no monitors, labs, use nontangible skills like listening, assessing, planning

hospice
concept of palliative care for terminally ill patients,

referral sources
hospitals (MD, MSW, Case Mgr), Clinics (RN or MD), Families
(self)

nurse role overpopulation


teaching birth control, family planning, preventing high risk
pregnancies, sex education

vectors
carry diseases such as mosquitoes, flies, ticks, roaches,
fleas, rats, mice, and squirrels

motor vehicle accidents


3 billion injuries and >42,000 deaths per year

health people 2010 environmental health


Focuses: pollutants, portable drinking water, decrease
water born diseases, eliminating toxic lead levels in

children, reduce pesticide exposure, inspecting homes for


lead
Health
which is a state of complete physical, mental and social
well-being, and not merely the absence of disease or
infirmity, is a fundamental human right and the attainment
of the highest possible level of health is a most important
world-wide social goal

Public health
a system and social enterprise; a profession; a collection of
methods, knowledge, and techniques; governmental health
services, especially medical care for the poor and
underserved; and the health status of the public

Epidemiology
the study of the occurrence and distribution of healthrelated states or events in specified populations, including
the study of the determinants influencing such states, and
the application of this knowledge to control the health
problems

Descriptive epidemiology
seeks to describe the occurrence of a disease in terms of
person, place, and time.

Determinants of health events


those factors, exposures, characteristics, behaviors, and
contexts that determine (or influence) the patterns
(answers the how and why); may be individual, relational,
social, communal, or environmental

Analytic epidemiology
focuses on investigation of causes and associations.

Distribution
determine who has the disease and where and when the
disease occurs

Epidemic occurs when


the rate of disease, injury, or other condition exceeds the
usual (endemic) level of that condition

Public health workers should be able to

Demonstrate basic knowledge of the role that genomics


play in the development of disease. Identify limits of his/her
genomic expertise. Make appropriate referrals to those with
more genomic expertise

The RN will
Demonstrate an understanding of the relationship of
genetics and genomics to health, prevention, screening,
diagnostics, prognostics, selection of treatment, and
monitoring of treatment effectiveness. Demonstrate ability
to elicit a minimum of a three-generation family health
history information. Construct a pedigree from collected
family history information using standardized symbols and
terminology. Collect personal, health, and developmental
histories that consider genetic, environmental, and
genomic influences and risk factors. Critically analyze the
history and physical assessment findings for genetic,
environmental, and genomic influences and risk factors.
Assess patients' knowledge, perceptions, and responses to
genetic and genomic information. Develop a plan of care
that incorporates genetic and genomic assessment
information.

Personal health care initiative- Goal:


to improve the safety, quality, and effectiveness of health
care for every patient in the United States

Genomics will
enable medicine to be tailored to each person's needs

Personal Health Care Initiative, Goal 1:


Link clinical and genomic information to support
personalized health care

Personal Health Care Initiative, Goal 2:


Protect individuals from discrimination based on
unauthorized use of genetic information

Personal Health Care Initiative, Goal 3:


Ensure the accuracy and clinical validity of genetic tests
performed for medical application purposes

Personal Health Care Initiative, Goal 4:


Develop common policies for access to genomic databases
for federally sponsored programs.

Genetic screening
Evaluate >1600 genetic disorders, First used in the late
1970s

Indications for genetic screening:


Predicting development of genetic disorders, Screening
populations, Confirming clinical diagnosis, Prenatal testing,
Development of individualized medical treatment

Genomics
the study of individual genes in order to understand the
structure of the genome, including the mapping of genes
and sequencing the DNA.

Genomics examines
the molecular mechanisms and the interplay of genetic and
environmental, cultural, and psychosocial factors in disease

Genomics deals with


he functions and interactions of all genes in an organism
and is the study of the total DNA structure.

DNA deoxyribonucleic acid,


is the chemical inside the nucleus of a cell that has the
genetic instructions for making living organisms.

Genes
the DNA segments that carry the genetic information

Chromosomes
long structures that organize DNA

DNA replication
chromosomes are duplicated before cells divide

Four bases of DNA


adenine (A), guanine (G), cytosine (C), and thymine (T)

Genes are comprised of


specific sequences of these bases

Mutations

alterations in the usual sequence of bases that form a gene


or changes in DNA or chromosomal structures

DNA Repair
process in which mutations are corrected; estimates
indicate repair mechanisms correct at least 99.9% of initial
errors

I
Investigate Potential Exposures Investigate potential
exposures by asking: Have you ever felt sick after coming
in contact with a chemical, such as a pesticide or other
substances? Do you have any symptoms that improve
when you are away from your home or work?

P
Present WorkAt your present work: Are you exposed to
solvents, dusts, fumes, radiation, loud noise, pesticides, or
other chemicals? Do you know where to find material
safety data sheets for chemicals with which you work? Do
you wear personal protective equipment? Are work clothes
worn home? Do co-workers have similar health problems?

R
Residence When was your residence built? What type of
heating do you have? Have you recently remodeled your
home? What chemicals are stored on your property? Where
is the source of your drinking water

E
Environmental Concerns Are there environmental concerns
in your neighborhood (e.g., air, water, soil)? What types of
industries or farms are near your home? Do you live near a
hazardous waste site or landfill?

P
Past Work What are your past work experiences? What job
did you have for the longest period of time? Have you ever
been in the military, worked on a farm, or done volunteer
or seasonal work?

A
Activities What activities and hobbies do you and your
family pursue? Do you burn, solder, or melt any products?
Do you garden, fish, or hunt? Do you eat what you catch or

grow? Do you use pesticides? Do you engage in any


alternative healing or cultural practices?

R
Referrals and Resources Use these key referrals and
resources: Environmental Protection Agency
(http://www.epa.gov) National Library of MedicineToxnet
Programs (http://www.nlm.nih.gov) Agency for Toxic
Substances and Disease Registry
(http://www.atsdr.cdc.gov) Association of Occupational and
Environmental Clinics (http://www.aoec.org) Occupational
Safety and Health Administration (http://www.osha.gov)
EnviRN website (http://www.enviRN.umaryland.edu) Local
Health Department, Environmental Agency, Poison Control
Center

E
Educate (A Checklist) Are materials available to educate
the client? Are alternatives available to minimize the risk of
exposure? Have prevention strategies been discussed?
What is the plan for follow-up?

Environmental laws and pollutants


Environmental Protection Agency, Food and Drug
Administration, Department of Agriculture, Permitting- limit
amount of pollution in air, Environmental standards,
Compliance, Monitoring

Epidemiology
The science that helps us understand the strength of the
association between exposures and health effects

Epidemiologic triangle
Agent, host, and environment, a simple model that belies
the often complex relationships between agent, which may
include chemical mixtures (i.e., more than one agent); host,
which may refer to a community with people of multiple
ages, genders, ethnicities, cultures, and disease states; and
environment, which may include dynamic factors, such as
air, water, soil, and food, as well as temperature, humidity,
and wind.

Geographic Information Systems (GIS)


is a methodology that requires the coding of data so that it
is related spatially to a place on earth.

Agent
Chemical or disease

Host
Community, patient,

Environment
water, air, cleanliness, plumbing, sewer systems, soil,
temperatures, climates, habits and customs

Key public health (PH) professionals


Food safety specialist, Sanitarians, Radiation specialists,
Industrial hygienists

Roles for nurses


Community involvement/public participation, Individual and
population risk assessment, Risk communication,
Epidemiologic investigations, Policy development

levels of public health nursing


Assessment, Diagnosis, Planning (Including selection of
interventions), Implementation, Consultation, Referral and
follow-up, Advocacy, Policy development, Surveillance,
Evaluation

Health departments
conduct assessments of community health status, a core
function of public health, on an ongoing basis

The identification of some community problems emerges


out
practice, rather than through a formal community
assessment

Applying the Nursing Process at the Individual/Family


Level
Community assessment, Assessment of a family, Diagnosis,
Planning (Including selection of interventions),
Implementation, Collaboration, Case management, Health
teaching, Delegated functions (PHN to paraprofessional),
Referral and follow-up, Evaluation

Applying the PHN Process at the Community Level of


Practice Scenario
Community assessment (PHN process: assessment),
Community diagnosis (PHN process: diagnosis), Community
action plan (PHN process: planning, including selection of
interventions), Menu changes, Classroom activities, Family
involvement, Community implementation plan (PHN
process: implementation), Community evaluation (PHN
process: evaluation).

Executive branch
Suggests, administers, and regulates policy. President,
cabinet, various administrative and regulatory
departments, and agencies. U.S. Department of Health and
Human Services.

Legislative branch
Identifies problems and proposes, debates, passes, and
modifies laws to address those problems. Identifies
problems and proposes, debates, passes, and modifies laws
to address those problems. Senate and House of
Representatives.

Judicial branch
Interprets laws and their meaning. System of federal, state,
and local courts guided by opinions of the Supreme Court.
Machine Co. v. Davis.

Supreme Court. Machine Co. v. Davis


First case to challenge federally created health programs.
Supreme Court determined that such federal governmental
action was within the powers of Congress to promote the
general welfare. Police Power: this power allows states to
act to protect the health, safety, and welfare of their
citizens. Such police power must be used fairly, and the
state must show that it has a compelling interest in taking
actions, especially actions that might infringe on individual
rights. An example is requiring immunizations for children
to enter school

Culture
A set of beliefs, values, and assumptions about life that are
widely held among a group of people and that are
transmitted intergenerationally. Develops over time and is
resistant to change. Each culture has its own unique

organizational structure. Communication. Family structure.


Child rearing practices. Religious practices. Physical space.

Race
A social classification that relies on physical markers, such
as skin color, to identify group membership. Individuals
may be the same race but of different cultures

Ethnicity
The shared feeling of peoplehood among a group of
individuals. Reflects membership in a cultural group and is
based on individuals sharing similar cultural patterns.
Represents the identifying characteristics of culture, such
as race, religion, or national origin, and affects beliefs,
behaviors, and access to resources. Members of an ethnic
group are likely to give up aspects of their identity and
society when they adopt characteristics of another group's
identity. However, when there is a strong ethnic identity,
the individual maintains the values, beliefs, behaviors,
practices, and ways of thinking of their group.

Four categories of foreign-born


Legal immigrants (lawful permanent residents), Refugees,
Nonimmigrants, Unauthorized immigrants (undocumented
or illegal aliens)

Legal immigrants
Make up 80% of immigrant population. Are not citizens but
are legally allowed to work and live in the United States.

Refugees and persons seeking asylum


are people who seek protection in the United States
because of fear of persecution (on the basis of race,
religion, nationality, political view, or membership in a
certain group) if they were to return to their homeland.

Nonimmigrants
are admitted to the United States for a limited duration and
for a specified purpose. include students, tourists,
temporary workers, business executives, career diplomats
and their spouses and children, artists, entertainers, and
reporters.

Advocacy

places the advocate's core skills (i.e., interviewing,


assertiveness and force, negotiation, self-management,
legal knowledge and research, and litigation) within the
context of six ethical principles for effective advocacy

Advocacy
Act in accordance with the patient's wishes and
instructions. Keep the patient properly informed. Carry out
instructions with diligence and competence. Act impartially
and offer frank, independent advice. Maintain patient
confidentiality.

Feminist ethics
encompasses the tenets that women's thinking and moral
experiences are important and should be taken into
account in any fully developed moral theory, and that the
oppression of women is morally wrong. Entails knowledge
about the social, cultural, political, legal, economic,
environmental, and professional contexts that insidiously
and overtly oppress women as individuals, or within a
family, group, community or society. Demand results.
Nurses were reluctant to embrace feminism and it's ethics
for many years.

The Core Functions Project (U.S. Public Health Service,


1994/2008)
developed a useful illustration, the Health Services Pyramid
(Figure 1-2), which shows that population-based public
health programs support the goals of providing a
foundation for clinical preventive services.

These services focus on


disease prevention; health promotion and protection; and
primary, secondary, and tertiary health care services.

All levels of services shown in the pyramid are


important to the health of the population and thus must be
part of a health care system with health as a goal.

Because of the importance of the basic public health


programs
members of the Core Functions Project argued that all
levels of health care, including population-based public
health care, must be funded or the goal of health of
populations may never be reached.

competencies of PH
In the spring of 2010 this Council, funded by the Centers
for Disease Control and Prevention, Health and Human
Services (HHS), adopted an updated set of Core
Competencies ("a set of skills desirable for the broad
practice of public health")
(http://www.phf.org/link/corecompetencies.htm) for all
public health professionals, including nurses.

The 72 Core Competencies are divided into eight


categories
-Analytic/assessment
-Policy development/program planning
-Communication
-Cultural competency
-Community dimensions of practice
-Basic public health sciences
-Financial planning and management
-Leadership and systems thinking

In addition, each competency is presented at three levels


(tiers) that reflect the different stages of a career. Tier 1
applies to entry level public health professionals without
management responsibilities.

In addition, each competency is presented at three levels


(tiers) that reflect the different stages of a career. Tier 2
competencies are expected in those with management
and/or supervisory responsibilities.

In addition, each competency is presented at three levels


(tiers) that reflect the different stages of a career. Tier 3
is expected of senior managers and/or leaders in public
health organizations.

In the early years of America's settlement, as in Europe,


the care of the sick was usually
informal and was provided by household members, almost
always women. The female head of the household was
responsible for caring for all household members, which
meant more than nursing them in sickness and during
childbirth. She was also responsible for growing or
gathering healing herbs for use throughout the year.

England's Elizabethan Poor Law


guaranteed medical care for poor, blind, and "lame"
individuals, even those without family. Colonial America's
ideas of social welfare and care of the sick were strongly
influenced by this law. Pennsylvania Hospital was the first
hospital in what would become the United States.

Early colonial public health efforts


included the collection of vital statistics, improvements to
sanitation systems, and control of any communicable
diseases introduced through seaports. Colonists lacked
mechanism to ensure PH efforts would be supported and
enforced. Epidemics taxed the limited organization for
health during the seventeenth, eighteenth, and nineteenth
centuries.

After the American Revolution


the threat of disease, especially yellow fever, brought
public support for establishing government-sponsored, or
official, boards of health.

Public Health Service was established in


1798 as the Marine Hospital Service; it provided health care
for merchant seamen and protected seacoast cities from
epidemics; PHS is still the most important federal public
health agency today.

Shattuck Report
was published in 1850 by the Massachusetts Sanitary
Commission with an emphasis on the public's health. The
report called for major innovations: the establishment of a
state health department and local health boards in every
town; sanitary surveys and collection of vital statistics;
environmental sanitation; food, drug, and communicable
disease control; well-child care; health education; tobacco
and alcohol control; town planning; and the teaching of
preventive medicine in medical schools. However, these
recommendations were not implemented in Massachusetts
for 19 years, and in other states much later.

Origins of organized nursing


Nightingale studied nursing "system and method" by
visiting Pastor Theodor Fliedner at his deaconess training
school in Kaiserswerth, Germany.

Nightingale implemented her ideas about nursing during

Crimean War

Nightingale progressively improved soldiers' health


outcomes using
a population-based approach that strengthened
environmental conditions and nursing care. Using simple
epidemiological measures, she documented a decreased
mortality rate from 415 per 1000 at the beginning of the
war to 11.5 per 1000 at the end (Cohen, 1984; Palmer,
1983).

Mary Breckenridge's work


Some community-oriented nursing innovations were the
result of individual commitment and private financial
support, such as the Frontier Nursing Service

Frontier Nursing Service influenced the


development of public health programs geared toward
improving the health care of the rural and often
inaccessible populations in the Appalachian region of
southeastern Kentucky (Browne, 1966; Tirpak, 1975).
Breckinridge introduced the first nurse-midwives to the
United States when she deployed Frontier Nursing Service
nurses trained in nursing, public health, and midwifery.

The Future of Public Health, Institute of Medicine (IOM,


1988):
Found the United State's PH system in disarray and
concluded that, although there was widespread agreement
about what the mission of PH should be, there was little
consensus on how to translate that mission into action
Reported that the mix and level of PH services varied
extensively across the United States

Disparities
Racial or ethnic differences in the quality of health care, not
based on access, clinical needs, preferences, or
appropriateness of an intervention
-Indicators that continue to cause disparities in the United
States' health care system:
Cost
Access
Quality

Primary care

is a component of the private health care system. The care


that is provided by a health care professional (either a
physician, physician assistant, or nurse practitioner)
trained in family practice, pediatrics, or internal medicine.
Also called individual care.

Primary health care (PHC)


the focus of the public health system in the United States,
is defined as a broad range of services, including but not
limited to basic health services, family planning, clean
water supply, sanitation, immunization, and nutrition
education; it consists of programs designed to be
affordable for the recipients of the care and the
governments that provide them. In PHC, the emphasis is on
prevention, and the means of providing the care are based
on practical, scientifically sound, culturally appropriate, and
socially acceptable methods. This care is provided at the
community level, is accessible and acceptable to the
community, and is inviting of community participation.

Public health economics


focuses on producing, distributing, and consuming goods
and services as related to public health and where limited
public resources might best be spent to save lives or cause
the most increase in quality of life.

Efficiency
Suggests that inputs are combined and used in such a way
that there is no better way to produce the service, or
output, and that no other improvements can be made
refers to producing maximal output, such as a good or
service, using a given set of resources (or inputs), such as
labor, time, and available money.

Effectiveness
For example, effectiveness of a mass immunization
program is related to the level of "herd immunity"
developed.
refers to the extent to which a health care service meets a
stated goal or objective, or how well a program or service
achieves what is intended.

supply and demand


The upward-sloping supply curve represents the seller's
side of the market, and the downward-sloping demand
curve reflects the buyer's desire for a given product. As

shown here, suppliers are willing to offer increasing


amounts of a good or service in the market for an
increasing price. The demand curve represents the amount
of a good or service the consumer is willing to purchase at
a certain price. This curve illustrates that when few
quantities of a good or service are available in the
marketplace, the price tends to be higher than when larger
quantities are available. The point on the curve where the
supply and demand curves cross is the equilibrium, or the
point where producer and consumer desires meet.

Supply and demand Shifts result of


Competition for goods or services
Increase in costs of materials used to make a product
Technological advances
Change in consumer preferences
Shortage of goods or services

macroeconomics
Focuses on the "big picture"the total, or aggregate, of all
individuals and organizations
Aggregate is usually a country or nation

Ethics
is concerned with a body of knowledge that addresses such
questions as the following: How should I behave? What
actions should I perform? What kind of person should I be?
What are my obligations to myself and to fellow humans?
There are general obligations that humans have as
members of society.

Ethical judgments
are concerned with values. The goal of an ethical judgment
is to choose that action or state of affairs that is good or
right in the circumstances.

Consequentialism
The decision is based on outcomes or consequences. That
approach to ethical decision making maintains that the
right action is the one that produces the greatest amount
of good or the least amount of harm in a given situation.

Utilitarianism
is a well-known consequentialist theory that appeals
exclusively to outcomes or consequences in determining
which choice to make.

Deontology
may conclude that the action is right or wrong in itself,
regardless of the amount of good that might come from it.

Respect for autonomy


Based on human dignity and respect for individuals,
autonomy requires that individuals be permitted to choose
those actions and goals that fulfill their life plans unless
those choices result in harm to another.

Non-maleficence
requires that we do no harm. It is impossible to avoid harm
entirely, but this principle requires that health care
professionals act according to the standards of due care,
always seeking to produce the least amount of harm
possible.

Beneficence
This principle is complementary to nonmaleficence and
requires that we do good. We are limited by time, place,
and talents in the amount of good we can do. We have
general obligations to perform those actions that maintain
or enhance the dignity of other persons whenever those
actions do not place an undue burden on health care
providers.

Distributive justice
Distributive justice requires that there be a fair distribution
of the benefits and burdens in society based on the needs
and contributions of its members. This principle requires
that, consistent with the dignity and worth of its members
and within the limits imposed by its resources, a society
must determine a minimal level of goods and services to be
available to its members.

Egalitarianism
is the view that everyone is entitled to equal rights and
equal treatment in society.

Libertarian
view of justice holds that the right to private property is the
most important right.

The dominant themes of communitarianism

are that individual rights need to be balanced with social


responsibilities; individuals do not live in isolation but are
shaped by the values and culture of their communities
(Communitarianism, 2010). Among the theories with a
communitarian focus are virtue ethics, ethic of care, and
feminist ethics.
Florence nightengale
- crimean wars- helped improve soldiers health
- used population based approach

lillian wald
- first public health nurse--started public health nursing
- known fo establishmen of henry street settlements in new
york
- national organization of public health established in 1912was the president

Lina Rogers
-first school nurse
- focused on investigating the cause of absenteeism
- did not treat illness- physicians job at the time

national organization of puplic health


- presdident- lillian wald
- established 1912
- goal- standardize public health nursing, improve
standards of services and education by public health
nurses , and promote public health nursing

segregation
- lasted until 1960s
- made it difficult for african american nurses to get
certification and graduate education

mary breckenridge
- developed health programs to improve healthcare in rural
and hard to reach populations in the applachians of
southern kentucky

leading cause of death from 1900-1955


pneumonia, tuberculosis, and diahrea enteritis

leading cause of death after 1955 . new challenge for


nurses

- heart disease, cancer, cerebrovascular disease


- new challenge for nurses was chronic illness, and disease
prevention

ruth freeman
leading healh educator, author, consultant and leader of
national health organization of the 20th century
- offeren many professional oppurtunities
- saw nursing as intellectually challenging and was about
caring for people

maritime hosptial service


currently known as the public health service
- attempt of united states federal government to protect
health of citizens

WHO conference
- @ Alma Ata in 1978
- main focus of the conference was
worldwideimplementation of primary health

population health
holistic approach that considers the total health system
and focuses on the broad range of factors and conditions
that have a strong influenceon the health of population
- determinants social factors, social support netweorks,
education,m employmentm, and healthy child development

mulitlatral organizations
organizations that recieve fundings from mulple
governmental and non-governmental sources

bio terrorism
intentiaonal use of a pathogen or biological product to
cause harm to living organisms to influence or intimidatt eh
conduct of a government and cause harm to other
people. :
- role of PHN and healh officieals is to detect pathogen ,
manage services, and communicate during threat
- anthrax, plague, and smallpox biological agens highest
concern

environmental health

- aspects of human health affecting quality of life that are


determined by biological, cheical, and social and
pssychological problems in the enviroment

chemical exposure
nurses have made discoveries based on assessing people
presenting with signs and symptoms related to known
chemical toxicity

epidemiology
studies the incidence and prevalence of disease, in
POPULATIONS
- helps us to understand the strength of the association
between exposure and health effects
- monitors health of populations , understands
determinants of health and disease in communities, and
investigates and evaluates interventions to prevent disease
and maintain health
- father of epidemiology is JOHN SNOW_ because of his
work with cholera

toxicology
study of health effects associated with chemical exposures

rule of seven
native american concept, what will be the effect of this
decision in 7 years. .
- native americans believe humans are stewards, not
propieters of the land

attack rate
-a measure of morbididity often used in infectious disease
investigations
- often specific to an exposure such as food specific attack
rates
-rate that best indicates the proportion of people exposed
to an agent who deelop the disease.

mortality rates
reflect serious health problems and changing patterns of
disease
- do not give direct info about level of existing disease or
the risk of getting a disease
-informative only for fatal diseases

descriptive epidemiology
seeks to describe a desiease entitiy according to person,
place, and time

analytical epidemiology
directed toward understanding the etiology of the disease
and attempts to dienitfy the determinants of disease in
individuals
- determinants may be indvidiual, social , communal, or
environmental
- Examples- cohort studies (prospective and retrospective);
case control studies, cross sectional studies, ecological
studies

rate
measure of frequency of a health event in a "definied
population in a specified period of time
- not a proportion because the denominator is a function of
both the population size and the dimension of time while
the numberator is the number of events
- one cannot tell the degree of seriousness or if it is an
epidemic if you do not have a deonomiator which
represents the total population

risk
the probability an even will occur within a specified time

incidence rate
the number of new cases developing in a population at risk
during a sepcified period
formula= # of new cases (divided by) total population at
risk x 1,000

incidence
quantifies the rate of development of new cases in a
population at risk

epidemic
when the rate of disease injury or other condition exceeds
the usual level of that position
- Examples- an isolatedcase of smallpox in Africa because
there is no smallpox; nursing shortage in US, adult obesity
in the US,

primary prevention

interventions aimed at preventing the occurence of


disease, injury or diability: before it happens
EXAMPLES- immunizations, diet, exercise

secondary prevention
focuses on ealy detection and prompt treatment of disease
injury or disability. things before symptoms occur . ie
screening for hearig defects, mammograms

tertiary prevention
interventions aimed at disability limitation, and
rehabilitation from disease, injury and disability
- EXAMPLES- rehbilitative job training, vocational training

specificity
a screening test has high specificity - meaning that the test
accuately identifies those without the trait
-the test accuractely identigying those without the trait

routinely collected data


-obtained through vital recors such as birth and death
certificates , - things that are uniform across the country
- example of a category of data sources commonly used in
epidemilogical investigations

most important predictor of overall mortality


age

mortality curve by age


decreases during and after first year of life, low point in
childhood, increases in adolescents and young adulthood,
then increases sharply through middle and old ages

point epidemic
- temporal and spatial pattern of disease distribution
- illustrated with freuqncy of disease plotted against time
0 shapr peak indicates concentration of cases in some
short interval of time
- example- outbreak of GI illness from a food borne
pathogen

cohort study
describes a group of persons enroled in a study that share
the same characteristic of interest for a period of time

cross sectional studyuses info on current health status, personal characteristics,


ad potential risk factors or exposures all at once

community trial
as opposed to clinical trials * treatment of existing disease)
is focused on health promotion and disease prevention
Example: Residents of a city have recently voted to add
fluoride to the H20. Epidemiologists wanting to study the
effect on dental caries would be conducting a....

provide the highest level of evidence


randomized double blind controlled trials

research utilization projects in 1970s


0 provided a guide to clinical practice and was the
forerunner of evidence based nursing practive.

systemetic review
- one of the two ways nurses can read research in
condenced format
- method of identifying, appraising and synthesizing
research evidence to evaluate all available research to a
particular question
- usually done by more than one person and describes the
methods used to search for the evidence

evidence based research


- began in canada
- nurses can participate by doing active research or
reviewing best available evidence

community development model


focuses on achieving community goals and includes t a
partnership where power and decision making are shared
between community members and academic community

information access project


serves as a reource for community healh nuses as they
indentify the findings from research that have direct links
to population focuses and communty based care

public health nurse

informs, educates, and empoowers people about health


issues.
- does not solve problems but identifies them,
- participates with regulators to protect commnties and
empower populations to address health issues globally and
locally
- BSN is the necessary basic preparation to function as a
beginning saff public health nurse

population
a collection of individuals who share at least one common
characteristic

aggregate
definied poplation made up of individuals in communities fo
a specific geographic regions

assurance
making sure that community oriented helath servies are
avaiable .
a core function

core functions
assurance, policy development, assessment, and ascietific
based care

community health nursing


practice inlclueds the delivery of personal health services
to individuals, families and groups
- practie in the community wheter or noth they have had
preparation in public health nursing (a BSN)
-ie- school nurse, a nurse in a clinic, nurses who make
home visits, to provide tertiary car

CDC
- implement quality performance standards in public
health. which are used to guid e improvents in the public
health system.
- 1998

primary goal of public health


concern for the health of many people not as much the
individual

assessment

one of the core functions includes collecting data and


monioting the health status of the population ,

policy development
one of the core functions of public health needed to provide
leadership in deveoping policies.

determinants
- factors, exposures, characteristics and behaviors that
determine patterns of disease.
- may be individual , relational or social, communal, or
environmental

prevalence
the measure of existing disease in a population at a
particular time
Ex: a screening for HTN revleaed 20 previously diagnosed
hypertensive individuals and 10 probable new cases ,
which were later confirmed for a total of 30 cases.
total # of cases divided by total population at risk times
100,000

cause specific mortality rate


estimate of the risk of death from some specific disease in
the population

crude death rate


TOTAL # OF DEATHS DURING THE YEAR (DIVIDED BY)
TOTAL POPULATION X 1,000

predictive value
Positive predictor value refers to the proportion of persons
with a positive test who actually have the disease,
interpreted as the probability that an individual with a
positive test has the disease.
nursing model
a tangible schematic and often visual representation of
relationships between and among key concepts that
explain how something works

core
people and their demographics, vital statistics, values,
beliefs, community perception, and history; center which
represents things necessary for life

8 subsystems
physical environment, education, safety and transportation,
politics and government, health and social services,
communication, economics, and recreation

normal line of defense


indicates level of health over time of a core or community;
includes characteristics such as high rate of immunity, low
infant mortality, or middle income leve; usual patterns of
coping, along with problem-solving capabilities; represents
health of the community

flexible line of defense


response to a temporary stressor; buffer zone representing
a dynamic level of health resulting from a temporary
response to stressors; may be neighborhood mobilization
against an environmental stressor such as flooding or a
social stressor such as unwanted adult bookstore

lines of resistance
internal mechanisms that act to defend against stressors;
community strength; an evening recreational program for
young people implemented to decrease vandalism

stressor
tension-producing stimuli that have the potential of causing
disequilibrium in the system; may originate outside the
community or inside the community--inadequate,
inaccessible, or unaffordable services

degree of reaction
stressors and lines of resistance become a part of
community and nursing diagnosis; amount of
disequilibrium or disruption that results

primary level intervention


education to prevent stressor from happening; general
health promotion such as nutrition, hygiene, exercise, and
environmental protection; specific health promotion, which
includes immunizations and the wearing of protection
devices to prevent injuries

secondary level intervention

early detection to reduce impact of stressor; early


detection and treatment of adverse health conditionsdectect and treat a problem at the earliest stage:
screenings!

tertiary level intervention


restore optimal function; employed after diseases or events
have already resulted in morbidity; limit disability and to
rehabilitate or restore the affected people to their
maximum possible capacities

focus of community health nursing


disease prevention; health promotion, assurance that care
is provided; focus is at the population level

health risks of poverty


malnutrition and illness; exposure to pollution; poor diet;
more likely to smoke

vulnerability
social groups who have an increased relative risk or
susceptibility to adverse health outcomes; increased
morbidity, premature mortality, and diminished quality of
life

Who is eligible for CareLink


open to San Antonio/Bexar County residents who do not
have private or public health insurance; income must not
exceed 300% of the Federal Poverty Guidelines; program
gives quality health care at University Health System

Acceptable forms of identification for Carelink


1 form of identification for each member of the household-must show full name, date of birth and must include a
photo;
driver's license, state ID card, resident alien card, U. S
immigration card, U.S naturalization, citizenship or other
approved federal documents, passport,
Birth Certificate (children only)

Acceptable proof of residency for CareLink


1 form of proof of residency dated within the past 30 days
in Bexar County: household bill (electric, telephone, or
cable), lease agreement, automobile insurance documents,

Medicaid/Medicare/CHIP letter, Attorney General letter,


food stamp letter

Acceptable proof of income for CareLink


all income that comes into the household within the last 30
days; check stubs, Wage certification form, workman's
comp, proof of retirement income, documents or checks
from Federal/State agencies, Texas Workforce Commission
notice of benefits; social security disability award letter,
complete and current income tax return

CareLink
financial assistance program for medical services received
within University Health System and our providers;
payment plan and payer of last resort; not health insurance
and not free

cultural competence
the ability to express an awareness of one's own culture, to
recognize differences between oneself and others, to adapt
behaviors to appreciate and accommodate those
differences; recognize that people with cultural
backgrounds different from our own have unique values,
life ways, health practices, and interpersonal styles.

benefits of cultural knowledge


improved relationships in one's personal and professional
life; less likelihood that conflict will develop in your
relationships with others; greater customer satisfactionimportant aspect of our professional life; develop beneficial
community partnerships

population
could include a race, ethnic group, religious group,
profession, a company, people with a certain sexual
preference, or gender; even from the area of the country
that one resides

cultural proficiency
having a vision that you can create an environment for
social change and social justice in your interactions with
colleagues, families and communities to effectively serve
the needs of all cultural groups

family

refers to two or more persons who are joined together by


bonds of sharing and emotional closeness and who identify
themselves as family

cultural encounter
process which encourages the healthcare professional to
directly engage in face-to-face cultural interactions and
other types of encounters with clients from culturally
diverse backgrounds in order to modify existing beliefs
about a cultural group and to prevent possible
stereotyping; exposure and practice

cultural knowledge
process in which the healthcare professional seeks and
obtains a sound educational base about culturally diverse
groups; healthcare professionals must focus on the
integration of three specific issues: health-related beliefs
practices and cultural values; disease incidence and
prevalence; cultural views, theoretical and conceptual
framework

cultural awareness
conducting self examination; acknowledgment and
awareness of one's own culture and willingness to explore
one's own feelings and biases; awareness of how culture
influences own ways of thinking and making decisions;
acknowledgement of how day-to-day behaviors reflect
cultural norms and values perpetuated by our families and
larger social networks

cultural assessment
use client's preferred language; be aware of client's
priorities; check client's understanding of diagnoses, and
treatment plan; ask whether client is using alternative
treatments; what or who is the support system; ask what
they think about the problem; ask what treatment they
think will help; what do they hope to receive from your
care; what do they fear most about the problem or
treatment; keep in mind background and individual and
biological differences

cultural assessment and healthcare


language-is an interpreter needed; level of language
literacy
beliefs and customs about health, illness and death

education
Religion-does the patient/family have beliefs that you need
to respect and accommodate for to enable you to care for
them
family forms and meaning of family

Vulnerability Factors of Elderly: Economic concerns


financial-paying off mortgages, healthcare expenses
health care costs-prescriptions and hospital care costs
elder abuse-domestic, institutional or self-neglect
access to health care-transportation, rural areas

vulnerability factors of elderly: social concerns


disasters-effects of hurricane, flooding
community safety and fear of violene
mental health-Alzheimer's dementia
accidents-falls or medication mismanagment

ethnocentrism
the belief that your cultural values, beliefs, and life ways
are superior to all others; the standard by which one judges
all other cultural patterns; one must be self aware of
perceptions and subsequent judgments are not
ethnocentric

breast cancer screen for elders


breast cancer: 50-74 yrs every two years

prostate cancer screen


start at 40 yrs, especially African Americans who are at
higher risk

HIV/AIDS screen for elders


15% of all cases are in adults 50yrs and older; older people
are at increased risk for HIV and STIs

preventive health screenings for elders


vaccines (flu, pneumonia-yearly-shingles after 60 yrs), BP,
cholesterol, fecal occult blood, sigmoidoscopy,
colonoscopy, vision and hearing, HIV

depression screenings for elders


individual-as needed (elder abuse may be occuring), group
screen should be part of a multicomponent multiservices
followup and treatment effort

elder abuse screening


mistreated elders are often frail, dependent, over age 70,
and women

Possible indicators of abuse


cuts, punctures, burns, bruises, welts, scratches, scalp
injury, gag marks, sprains, broken bones
sexually transmitted disease, rectal bleeding, torn or
bloody clothing
caretaker cursing, yelling and screaming, threats of harm,
intimidation

behavioral signs of abuse


fear, anxiety, agitation, anger, isolation, withdrawal,
depression, non-responsiveness, resignation, ambivalent,
contradictory statements, implausible stories, hesitation to
talk openly, confusion or disorientation

factors contributing to abuse, neglect, and exploitation of


elderly
aging population; growing number of adults with
disabilities; alcohol and drug dependency; unemployment;
lack of affordable housing and high costs of utility bills;
deinstitutionalization of mentally ill; inadequate access to
health care and cost; pathological family relationshipsviolence as coping mechanism; physical and mental stress
of caregiving

Elders and disaster


disproportionately vulnerable during disasters because
they are more likely to have chronic illnesses, functional
limitations, and sensory, physical, and cognitive
disabilities; live alone and on limited incomes making it

nearly impossible to recover from a disaster without special


assistance; often slow to request help

modifiable risk factors of vulnerable populations


poverty, low education and level of social support

non-modifiable risk factors of vulnerable populations


age, gender and ethnic status

health disparities
differences in the incidence, prevalence, mortality, and
burden of diseases and other health conditions among
specific population groups.

Health Leads
national nonprofit organization that connects low-income
patients with the basic resources they need to be healthy

social determinants of health


health is impacted by many social, economic and
environmental factors that extend far beyond individual
biology of disease and contributes to risk; aspects of
society and the social environment that impact on health,
such as poverty, early life experiences, social networks and
support, education, income, cultural traditions

upstream determinants
features of the social environment, such as socioeconomic
status and discrimination, that influence individual
behavior, disease, and health status.-preventive, health
promotion

downstream preventions/interventions
short-term, problem-specific, individual-based
interventions; after the fact--after disease process has
started.

community as partner model (CAP)


focuses on: the community as partner/the nursing process.
the community is composed of a core population and eight
subsystems, which are depicted visually as a wheel with
the population at the hub and with surrounding
subsystems. nurse works in partnership with community to
help health.

community assessment
act of becoming acquainted with a community; assessing a
community to identify factors (both positive and negative)
that impinge on the health of the people to develop
strategies for health promotion

Lillian Wald
recognized the intertwining of health status, environmental
sanitation, and social and political forces; mother of public
health nursing; contributions include establishing nursing
schools, advocating for better housing, working to change
child labor laws, teaching preventive practices, advocating
occupational health nursing, and improving the education
of public health nurses

Mary Brewster Lillian Wald


founded the Henry Street Settlement House in New York
City; taught hygiene practices, visiting the sick in homes,
and crusading for better health care in all aspects of the
community

Public Health Achievements in the 20th Century


1. Immunizations
2. Improvements in motor vehicle safety
3. Workplace safety
4. Control of infectious diseases
5. Decline in deaths from heart disease and stroke
6. Safer and healthier foods
7. Healthier mothers and babies
8. Family planning
9. Fluoridation of drinking water
10. Recognition of tobacco as a health hazard

Public health focus now


more focus on disease prevention, health promotion, and
assurance that care is provided, rather than providing oneon-one care

Neuman's system model


provides a holistic and system based perspective for
nursing; focuses on the response of the client system to
actual or potential environmental stressor; uses primary,
secondary, and tertiary prevention intervention for
retention, attainment, and maintenance of optimal system
wellness

Factors Shaping 21st Century Health


health care delivery system; demographics; globalization;
poverty and growing disparities; primary health care;
violence, injuries, and social disintegration; bioterrorism

leading causes of death now


coronary heart disease: stroke (CVA); lower respiratory
infections: chronic obstructive pulmonary disease; diarrheal
disease; HIV/AIDS; tuberculosis; trachea, bronchus, lung
cancers; road traffic accidents; prematurity and low birth
weight

windshield survey
crucial to community assessment because it serves as a
model, or map, to direct and guide that assessment
process; it guides the community assessment; learning
about the community of foot/drive by

CAP Assessment
CAP will guide assessment process; assess core and 8
subsystems (relationships); many methods used to assess
data-observation, interviews, data collection

CAP Planning
goals derived from stressors (weaknesses): aim to mitigate,
alleviate, eliminate the stressor; strengthen the
community's lines of defense; plan interventions to
strengthen lines of resistance

CAP Interventions
all are preventive in nature; Primary-aims at strengthening
the lines of defense, secondary-applied after stressor has
penetrated the community, and tertiary-applied after
stressor pentrates and the degree of reaction has taken
place resulting in system disequilibrium

CAP Evaluation
based on feedback from the community: involves
community in all phases of the nursing process to ensure
success
outcomes: may use same parameters as used in
assessment

process: working with the community as partner

Disaster Category A agents


can be easily disseminated; results in high mortality rates;
public panic and social disruption; requires public health
preparedness: anthrax, botulism, plague, smallpox,
tularemia, viral hemorrhagic fevers

Disaster Category B agents


second highest priority agents; moderately easy to
disseminate; moderate morbidity rates and low mortality
rates: brucellosis, food safety threats, glanders, melioidosis
Q fever, Ricin toxin

Disaster Category C agents


third highest priority agents; emerging pathogens that
could be engineered for mass dissemination in the future
because of availability; ease of production and
dissemination; potential for high morbidity and mortality
rates and major health impact

bioterrorism
intentional release of pathogens or biologic agents to cause
harm--biological disaster

signs of stress during disaster


minor tremors, nausea and vomiting, lack of concentration,
memory and problems, irritability, fatigue, anger, H/A

pandemic
steady occurrence of disease over a large geographic area
or worldwide, such as malaria in Africa

epidemic
unexpected increase of an infectious disease in a
geographic area over an extended period of time; defined
relative to the infectious agent and the history of the
disease in the area

endemic
occur at a consistent, expected level in a geographic areaSTI, tuberculosis

outbreak
unexpected occurrence of an infectious disease in a limited
geographic area during limited period of time

observational care site


sufficiency of care model for those who require monitoring
and skilled nursing assistance-under normal circumstances
most guests found in an assisted living center or nursing
home

low acuity patient care site


to permit "hospital decompression;" result transferring
stabilized patients from hospitals to community based
medical shelters or an ACS location to alleviate hospital
overcrowding

ambulatory care clinic


site would be utilized to provide medical care to "walking
wounded" or others whose condition is considered subacute; this type might include a point of dispensing (POD)
for the medications or vaccination

primary triage point


may be established near an impact zone or in a close
proximity to a hospital; patients would be quickly evaluated
to determine prioritization for transport to hospitals or
other designated locations

federal medical station


a cache to medial supplies and equipment that can be used
to set up a temporary acute a non-acute care facility; each
FMS has beds, supplies and medicine to treat up to 250
people for up to 3 days; sites used for this purpose must be
larger than a typical medical shelter due to bed count
requirements (40,000 square feet minimum)

alternate care site


non-hospital, community based location that my provide
additional treatment area(s) with a minimum specific level
of care for patients; may be established in a facility of
opportunity or a temporary structure such as a tent or
Zumro

Functional Needs Support Services

services that enable children and adults to maintain their


usual level of independence in general population shelters:
reasonable modifications to policies, practices and
procedures; durable medical equipment; consumable
medical supplies (pharmaceuticals); personal assistance
services; other goods and services as needed

most vulnerable in a disaster


children, seniors, poor, chronically ill, disabled,
homebound, developing countries

Stage 1 of disaster: prevention


modify homes to resist storms, quakes, etc; self-sustaining
communities; situational awareness; develop regional,
national, and international understanding and cooperation
concerning potential disasters; environmental, urban and
housing development, poverty-all have an impact

Stage 2 of disaster: Preparedness


disaster management begins with preparation; preparing in
advance will help you help others; community nurses play
key role in educating public about preparedness

Stage 3 of disaster: Response (disaster paradigm)


detect; incident command; scene security and safety;
assess hazards; support; triage and treatment; evacuation
and transport of Victims/Casualties

Stage 4 of disaster: Recovery


address immediate needs; needs several days after the
incident; address long term needs

START triage system


Simple Triage and Rapid Treatment; uses a systematic
approach based on 3 observation (respiration, perfusion,
and mental status) to triage multiple victims into 4
categories; 30-60 seconds per victim; green:
minor/delayed, yellow: delayed/urgent, red:
critical/immediate, black: expectant-dead or dying (last)

Mass Triage system


Move-request those able to move and walk to move to a
specified safe site
Assess: assess individual patients, beginning with those

unable to move
Sort: categorize victims based on urgency of care
Send: transport injured persons away from the scene

triage in disaster
emphasis may shift from meeting the most urgent
individual need to meeting the needs of the largest
numbers of victims to maximize the number of lives saved;
should be an ongoing activity. Assess, Assess, Assess-move
move move.

SARS
severe acute respiratory syndrome, outbreak in 2003

disaster
occurs as the result of an event where "normal conditions
of existence are disrupted and the level of suffering
exceeds the capacity of the hazard-affected community to
respond to it"

emergency
occurs as the result of an event where "normal procedures
are suspended and extraordinary measures are taken in
order to avert the impact of a hazard on the community"-can become a disaster

hazard
occurs when an event "has the potential to cause
disruption or damage to the community; earthquake, flood,
typhoon and cyclone--some may become emergencies but
not all become disasters

nursing roles in disaster: stage 1/prevention


organize and participate in community immunization
activities; participate in agency or community vulnerability
assessments and strategies for improvement;
develop/implement prevention strategies for worksites,
community sites, and home

nursing roles in disaster: stage 2/preparedness


volunteer for emergency response teams; locate/be able to
respond to institutional emergency plans; develop/update

institutional/community emergency plans or protocols;


develop volunteer registry to respond to emergency needs

nursing roles in disaster: stage 3/response


assist with emergency plan activation; assist with resource
mobilization; assist with public education; assist with
establishment and maintenance of shelters; provide
nursing care; assess public health needs; implement
disease surveillance, vector control, hygiene measures,
safety measures for food and water, etc
colonial period
household members (usually women) tend to the sick
(urbanization in the early 1800s causes this system to
became insufficient)

England's Elizabethan Poor LAw of 1601


1751 pennsylvania hospital founded

early colonial PH
efforts

after american revolution


first PH committee was established

Public health service was established in


1798 as the marine hospital service

early experiments in providing nurisng care at home


ladies benevolent society of Charleston 1813
philadelphia lay nurses
Roman catholic sisters of charity 1854

shattuck report 1850 by the


Massachusetts sanitary comission

nightingale and the origins of trained nursing


need for nurses
origins of organized nursing(pastor theodor fliedner,
Crimean War (improved soldiers' health using a populationbased approach)
Principles of Nursing

Principles of Nursing

health of the unity is the health of the community


differentiated"sick nursing" or "health nursing"
proper nutrition, rest, sanitation, and hygiene necessary

William Rathborne
founded first district nursing association in liverpool,
england
rathbone and nightingale recommended steps to provide
nursing in the home, and district nursing was organized
thorughout england

florence sarah lees craven


guide to district nurses

America Needs Trained Nurses


Need for PH nursing
Increase of women in workforce
more economical to have home-visiting nurses

1870s
first nightingale model nursing schools started

1877
women's board of the new york city mission hired frances
root

1878
ethical culture society of new york hired four nurses to
work in dispensaries

1885-1886
visiting nurses association were established

district nursing and settlement houses


deplorable enviornmental conditions in immigrant
tenement housing and sweatshops
District nursing and settlement houses established
Rural nursing service
occupational health

school nursing in america

grew out of school absences caused by the prevalence of


infections and communicable diseases

lina rodges
first united states school nurse
worked with children in NYC schools
She and her other school nurses found illness was often not
the reason for absence

significant impact and positive impact on school nursing


more nurses hired and school nursing was soon
implemented in los angeles, philadelphia, baltimore,
boston, chicago, and san francisco

The profession comes of age in the United States in the


Twentieth Century
National organization for PHN (NOPHN)
American Public Health Assocation (APHA)

National organization for PHN


sought to standradize PHN education
1914- first post training school course in PHN offered at
TEAchers college in NYC
1920s-1930s- many newly hired PHN had to verify
completion in a certificate program in PHN

American Public Health Association APHA


established in 1872
sought to facilitate interprofessional efforts and promote
the practical application of public hygiene

late in 1800s
local health departments formed

federal role in PH gradually expanded


1912- US public health service role defined
NOPHN loaned a nurse to the US public health service- first
federal governemtn sponsorship of nurses

WWI depleted the ranks of


PHNs

1918

world wide influenza pandemic

Paying the Bill for PHNs


Metropolitan life insurance company
sheppard-towner act
individual commitment and private financial support
frontier nursing service

African-american nurses in PHN


1919: national health care circle for colored people (Bessie
M. Hawes)
1936: PHN certificate program for African- American nurses
wage discrimination in the south
nursing education segregated until 1960s

Economic depression
agenecies and communities not prepared to address the
increased needs and numbers of impoverished (decreased
funding)
Agencies that helped to support nurse employment
(Federal Emergency relief administration, works progress
administration, relief nursing service, civil works
administratin)

1932 survery
found only 7% of nurses working in PH were adequtely
prepared

social security act of 1935


funded opportunities for education and employment of
PHNs
Funded assistance to states, counties, and medical districts
in establishing adequate health services
provided funds for research and investigation of disease

world war II
accelerated need for nurses, both for war effort and home
nursing council on national defense
many nurses joined the army, and navy nurse corps.
bolton act of 1943 established cadet nurses corps
some expansion of PHN scope of practice
Emergency maternity and infant care act of 1943
Job opportunities

Rise in chronic illness


national crude mortality rate decreased by 47%
change in leading cause of death from communicable
diseases to chornic diseases
aged population grew as did prevlaence of chronic disease
some visiting nurse associations provide home-care
programs
reimbursable by commerical health insurance and later by
medicare and medicaid
resurgance in combination agencies

declining financial support for practice and professional


organizations
hospitals preferred for illness and childbirth
funding stopped for visiting nurse services
consolidation of national nursing organizations
national leauge for nursing
american nurses association

Professsional Nursing Education for PHN


national league for nursing adopted Esther Lucile Brown's
Nursing for the Future 1948. Recomended to establish basic
nursing preparation colleges and universities. Included PHN
concepts in all baccalureate programs; however, these
were very brief components of teh cirrumula
1950s PHN practice increased focus on the psychological
elements of client, family and community care

1960s
medicare and medicad
did not include coverage for preventive services
home health care only reimbursed if ordered by a physician
increase in for-profit home health agencies
reduction in health promotion and disease prevention by
local and state departmetns

community organization and professional change


civil rights movement
funding increaes in certain areas
economic opportunity act

1970s
Nurse made significant contribution to:
the hospital movement
the development of birthing centers

day care for older adults and disabled persons


drug abuse programs
rehabilitation services in long-term care

1980s
concern about high health costs
fundign shifted away from health promotion and disease
prevention to acute care
national center for nursing research (NCNR) established in
1985 (gained offical status within NIH in 1993, beocming
NINR
IOM's report the future of public health 1988
Healthy people initaitve began

1990s
health care debate focused on cost, quality, and access to
direct care services
nursing organizations joined to support health care reform

2000s
health care reform finally passed in 2010 with the federal
patient protection and affordable care act
PHN organizations develop position papers on:
graduate education for advance practice PHN
faculty qualifactions for community/public health nursing
educators
importance of PHN within PH systems
Better nutrition, water, antibiotics, immunizations. PUBLIC
HEALTH.
Life Expectancy - U.S.
What factors/events resulted in the 21+ additional years of
life from 1900-1950???
What factors/events resulted in the 7 additional years of
life from 1950-1990?

1. Upstream
2. Downstream
There are two views of the health care picture
We need to question the way we have thought of health
and health care...
Should we focus on treatment of illness or prevention of
disease in the first place?

How should we act?


______ or ______

1. Microscopic
2. Medical Model
3. Downstream
_______ approach to solving community health problems
(the _____ or _____):
- Individual (family) response to health and illness
- Emphasizes behavioral responses to illness or lifestyle
patterns
- Nursing interventions aimed at the individual
------Changing lifestyles
------Changing perceptions or belief system
- The individual is the locus of change
- Focuses on the "cure"

1. Macroscopic
2. Public Health
3. Upstream
______ approach to solving community health problems
(______ or _______)
- Interfamily and intercommunity themes
- Emphasizes social, economic, and environmental
precursors of illness
- Nursing interventions may include modifying social or
environmental variables
- May involve social or political action
- society (social system) is the locus of change
- focuses on "prevention"

1. Health Belief Model


2. Health Promotion Model
3. Transtheoretical Model
4. Critical Social Theory
CHN practice is based upon theory. Theories are used
regularly in CHN. Some examples:
1.
2.
3.
4.

1. The Health Belief Model


2. Perceived susceptibility
3. Perceived severity
4. Perceived benefits
5. Perceived barriers to action
6. Cues to action
7. Self-efficacy
_________:
- focuses on a person's perceptions of a threat or a health
problems and related appraisal of a recommended
behavior to elicit change
- developed in the 1950s and was one of the first models
using concepts and assumptions from behavioral sciences
to examine health behaviors
------- Addresses attitudes and beliefs of individuals
------- Focus was on increasing the use of preventive health
services
- Primary assumptions of the model are that people fear
disease and that health actions are motivated based on the
extent of the fear and belief in benefits obtained by
preventative action
- Five (Six?) Primary Constructs:
1.
2.
3.
4.
5.
6.

- "Using the Health Belief Model to Predict Injecting Drug


Users' Intentions to Employ Harm Reduction Strategies"
(Bonar & Rosenberg, 2011)
Studied whether perceived risk of infection would
impact intention to clean site pre-injection. Also studied
perceived risk of non-fatal overdose and use of a test
shot.
Found that the perceived risk of infection and overdose
did impact behavior if the user was not in withdrawal.
Application of the HBM in CHN:

Pender's Health Promotion Model


__________:
o Developed by Nola Pender to study health promotion
behaviors; initially published in 1982
o Explores biopsychosocial processes that motivate
individuals to engage in behaviors that promote health
o Has basis in the HBM

o Depicts the complex multidimensional factors with which


people interact as they work to achieve optimum health.
o This model does not include threat as a motivator, as
threat may not be a motivating factor for client's in all age
groups.
o Used by nurses to develop and execute health-promoting
interventions
o Used to develop research studies focusing on one aspect
of health promotion
o Used frequently as a framework for research studies
o Individual Characteristics and experiences that may
affect their health actions:
- Prior related behavior
- Personal factors
- Behavior specific cognitions and affect
- Perceived benefits of action
- Perceived self-efficacy
- Activity-related affect
- Interpersonal influences
- Situational influences
- Commitment to a plan of action
- Immediate competing demands and preferences
- Health-promoting behavior

"Registered Nurses' Beliefs of the Benefits of Exercise,


Their Exercise Behavior and Their Patient Teaching
Regarding Exercise" (Esposito & Patrick, 2011).
Wanted to know if nurses who engage in healthy
behaviors were more likely to recommend them to their
patients.
Found that nurses who believe in health promotion and
embrace healthy behaviors were more likely to be
positive role models and were more likely to teach
healthy behaviors to their patients.
- More examples: Health promotion for community dwelling elders and Prevention of melanoma
Application of the HPM in CHN:

1. Transtheoretical Model (Stages of Change)


2. Precontemplation
3. Contemplation
4. Preparation
5. Action
6. Maintenance
_________:
o Based on the assumption that behavior change takes

place over time


o Change is difficult
o Change progresses through a sequence of stages
-----------One may stop at a stage, progress to the next
stage or return to the previous stage
o Stages of change: 1.______, 2._____, 3.______, 4._____,
5.______
o Decisional Balance:
---------Pros: The benefits of behavior change
---------Cons: The costs of behavior change

1. Precontemplation
2. Contemplation
3. Preparation
4. Action
5. Maintenance
Transtheoretical Model (Stages of Change):
1. _______: No intention to change behavior in the next 6
months
------May be lack of information about consequences or
previous failure.
2. _______: Individual intends to change behavior in the next
6 months.
------Weighs pros and cons
3. _______: Individual intends to act within the next month
and has taken steps toward change
------Has a plan of action
4. _______: Individual has changed behavior for less than 6
months
------Change is sufficient to reduce disease risk.
5. _______: Individual has changed behavior for more than 6
months.
------Tries to prevent relapse
------Phase may last months to years

"Adapting the Transtheoretical Model of Change to the


Bereavement Process" (Calderwood, 2011).
Applies the model to the bereavement process.
Postulates that bereaved persons never return to their
previous state but undergo change as they cope with and
adjust to life without their loved one.
Application of the TTM in CHN:

1. Smoking cessation
2. Injury prevention
3. RNs assisting families obtain health insurance

Examples for applying Transtheoretical Model in nursing:


1.
2.
3.

1. Critical Social Theory


2. Enlightenment
3. Empowerment
4. Emancipation
_____________:
- Jurgen Habermas is the best known of the theorists for
this
-------Habermas was part of a group of German scholars in
Frankfurt writing in the 1960s
-------He promoted critical social theory to describe
"distortions and constraints that impede free, equal and
uncoerced participation in society."
- began in Marx's argument that oppression requires
revolutionary action
- uses societal awareness to expose social inequalities that
keep people from reaching their full potential
-------Promote equality
-------Reduce disparities (ie. health disparities)
- aims to provide an environment in which individuals can
become empowered, enlightened and emancipated
-------- ______: raising the consciousness of the oppressed.
-------- ______: encourages people to undertake activities to
improve their situation
-------- ______: is the goal of empowerment through which
new arrangements replace oppressive ones
- This perspective is informed by the following values and
assumptions:
---------- The problem and inequalities of health and health
care are connected to the particular historically located
social arrangements and the cultural values of society
---------- Health care should be oriented toward the
prevention of disease and illness
---------- The priorities of any health care system should be
based on the needs of the clients/population and not health
care providers
---------- Ultimately, society itself must be changed for
health and medical care to improve

"Why Carers of Older People are not using Available


Respite Services: An Australian Study (Stockwell-Smith,
Kellett, & Moyle, 2010).
Examined use of respite services by caregivers to

determine why some opted not to use these services.


Found that a lack of knowledge of services and trust in
the workers were factors in using respite care.
Found that caregivers were defined by their role and
could not easily relinquish it to another.
Application of CST in CHN:

1. Health Disparities
2. Health and social justice
Examples of applying critical social theory:
1.
2.

- A state of complete well-being, physical, social, and


mental, and not merely the absence of disease or
infirmity. World Health Organization, 1958
- The extent to which an individual or group is able, on
the one hand, to realize aspirations and satisfy needs;
and, on the other hand, to change or cope with the
environment. Health is, therefore, seen as a resource for
everyday life, not the objective of living; it is a positive
concept emphasizing social and personal resources, and
physical capacities. WHO 1986
How do we define health?

1. Purposeful actions
2. Processes
3. Responses
4. Behaviors
Definitions of Health:
Health consists of:
_____, ______, _____, or _____ that leads to: "soundness,"
"wholeness," or "well-being"

1. Physical Environment
2. Social Environment
3. Individual Behavior
4. Biology & Genetics
5. Health Services
6. Policy making
Determinants of Health:
Health Outcomes due to:
1.

2.
3.
4.
5.
6.

Physical Environment
Determinants of Health - Examples of ______ factors:
- Natural environment (ie plants, weather, climate change)
- Worksites, schools and recreational settings
- Housing, homes, neighborhoods
- Exposure to toxic substances
- Physical barriers
- Aesthetic elements (ie good lighting, trees, parks)

Social Factors
Determinants of Health - Examples of ______ factors:
- Availability of resources (ie educational and job
opportunities, living wages, healthful foods)
- Social norms and attitudes
- Exposure to crime and violence
- Social interactions
- Exposure to emerging technology (ie the Internet)
- Transportation options

Individual Behaviors
Determinants of Health - Examples of ______ factors:
- Diet
- Physical activity
- Alcohol, tobacco and other drug use
- Sexual behavior
- Hand washing

Biology and Genetics


Determinants of Health - Examples of ______ factors:
- Age
- Sex
- Race/ethnicity
- Inherited conditions
- Family history of diseases

Health Service
Determinants of Health - Examples of ______ factors:
- Barriers (lack of availability, high cost, lack of insurance,
language issues)
------- Unmet health needs

------- Delays in receiving care


------- Inability to get preventive services
------- Preventable hospitalizations

Policy Making
Determinants of Health - Examples of ______ factors:
- Health curricula requirements
- Increased taxes on tobacco
- Prohibitions on smoking
- Drinking age restrictions
- Seat belt laws and child restraints

1. Life expectancy
2. Health life expectancy
3. Years of potential life lost
4. Physically and mentally unhealthy days
5. Self-assessed health status
6. Limitation of activity
7. Chronic Disease Prevalence
Indicators of General Health Status:
1.
2.
3.
4.
5.
6.
7.

Community
___________:
- It can be a physical place or a geopolitical community.
- Has boundaries
- It can be a relational, interactive group. - A community
with no physical boundaries. A phenomenological
community is abstract. Churches, universities, online
groups are examples

Community
__________:
"A collection of people who interact with one another and
whose common interests or characteristics form the basis
for a sense of unity or belonging." (Nies and McEwen,
2011)

Community

__________:
"A group of people who share something in common and
interact with one another, who may exhibit a commitment
to one another and may share a geographic boundary."

1. People
2. They must interact with each other in some way
3. Something in common: an interest, a geographical
location, a commitment
A community has to have:
1.
2.
3.

1. Primary Prevention
2. Secondary Prevention
3. Tertiary Prevention
Levels of Prevention:
1.
2.
3.

Primary Prevention
_______ level of prevention
- activities preventing a problem before it occurs
- Health promotion
- Specific protection
- Immunizations

Secondary Prevention
________ level of prevention
- Early detection and prompt intervention
- Screening
- Early referral for treatment
- Screening for STDs

Tertiary Prevention
_______ level of prevention
- Focus on limitation of disability and rehabilitation
- Prevention progression of disease
- Reduce the effects of the disease
- Teaching insulin administration

Community Health Nursing (CHN)


_________:
o Focus on individuals and families where they live, work,

and go to school; care is setting-specific and the emphasis


is on acute and chronic care
o "The synthesis of nursing practice and public health
practice applied to promoting and preserving the health of
populations.

Public Health Nursing (PHN)


_________:
o Focus is on the community as a whole and the effect of
community health status (resources) on the health of
individuals, families and groups
o Consideration of the health of individuals, families, and
groups and their effect on the health of the community as a
whole
o The synthesis of public health and nursing practice.
o "The practice of promoting and protecting the health of
populations using knowledge from nursing, social, and
public health sciences."

Public/Community Health Nursing (PHN)


__________:
- According to the ANA, PHN is the
o ...practice of promoting and protecting the health of
populations
o ...using knowledge from nursing, social and public health
sciences

Public Health Nursing


_______ Practice is:
1. Population focused
2. Goals of promoting health and preventing disease and
disability
3. Focuses on all people through the creation of conditions
in which people can be healthy

Public/Community Health Nursing


__________:
- Focuses on the entire population
- Is based on assessment of the population's health status
- Considers the broad determinants of health
- Emphasizes all levels of prevention
- Intervenes with communities, systems, individuals, and
families
- like an umbrella, Covers:
--------- Home Health Nursing
--------- Palliative and Hospice Nursing

---------------------------------

Public Health Nursing (official agencies)


School Nursing
Occupational Health Nursing
Correctional Nursing

o ...practice is general & comprehensive; not limited to an


age or diagnostic group
o ...is continuing, not episodic
o ...is directed to communities, groups, & individuals as it
contributes to the health of the total population
o Goal is to protect and promote the health of the entire
population
o Populations can be defined by geography, demographic
characteristic or need
What is Public Health Nursing (PHN)?

Anywhere people come together


Neighborhoods
Public Health Agencies
Community Centers
Day Care Centers
Schools
Workplaces
Housing projects
Settings for Public Health Nursing:
1.
2.
3.
4.
5.
6.
7.
8.

Community-based nursing
___________:
o Nursing of individuals and families to improve their
health
o Goals: Help them manage illness while they move among
health care settings
o Promote self-care and rehabilitation; prevent disease
o Processes: NP; diagnosis and treatment

Wherever individuals and families need nursing care


outside the hospital or nursing home
Home

Community clinics
School clinics
Workplace clinics
Rehabilitation centers
Settings for Community-based nursing:
1.
2.
3.
4.
5.
6.

Clinical Nursing
_________:
o Goal: improve the health of patients
o Clients: Patients of the health care system
o Processes used: nursing process, treatment and patient
care procedures
o Settings: inpatient

The Public Health Nurse


The Standards of Care:
__________:
- Assesses the health status of populations using data,
community resource identification, input from the
population and professional judgment
- Analyzes collected assessment data and partners with the
people to determine population diagnosis and priorities
- Participates with other community partners to identify
expected outcomes in the populations and their health
status based on population diagnoses and priorities
- Develops a plan that reflects best practices by identifying
strategies, action plans and alternatives to attain expected
outcomes
- Implements the identified plan by partnering with others
- Employs multiple strategies to promote health, prevent
disease and ensure a safe environment for populations
- Evaluates the health status of the population

1. Morbidity data - illness or disability


2. Mortality data - death
Indicators of Health and Illness:
- Health Statistics:
1.
2.

1. Surveillance
2. Disease and other health event investigation
3. Outreach
4. Screening
5. Case finding
6. Referral and Follow-up
7. Case management
8. Delegated functions
9. Health Teaching
10. Counseling
11. Consultation
12. Collaboration
13. Coalition Building
14. Community Organizing
15. Advocacy
16. Social Marketing
17. Policy and enforcement
Public Health Interventions:
1.
2.
3.
4.
5.
6.
7.
8.
9.
10.
11.
12.
12.
13.
14.
15.
16.
17

Surveillance
Describes and monitors health events through ongoing and
systematic collection, analysis and interpretation of health
data for the purpose of planning, implementing and
evaluating public health interventions

Disease and other health event investigation


Systematically gathers and analyzes data regarding threats
to the health of populations, ascertains the source of the

threat, identifies cases and others at risk, and determines


control measures

Outreach
Locates populations of interest or populations at risk and
provides information about the nature of the concern, what
can be done about it, and how services can be obtained

Screening
Identifies individuals with unrecognized health risk factors
or asymptomatic disease conditions in populations

Case finding
Locates individuals and families with identified risk factors
and connects them with resources

Referral and Follow-up


Assists individuals, families, groups, organizations and/or
communities to identify and access necessary resources to
prevent or resolve problems or concerns

Case Management
Optimizes self-care capabilities of individuals and families
and the capacity of systems and communities to coordinate
and provide services

Delegated Functions
Direct care tasks a registered professional nurse carriers
out under the authority of a health care practitioner as
allowed by law

Health Teaching
Communicates facts, ideas, and skills that change
knowledge, attitudes, values, beliefs, behaviors, and
practices of individuals, families, systems, and/or
communities

Counseling
Establishes an interpersonal relationship intended to
increase or enhance capacity for self-care and coping with
a community, system, and family or individual

Consultation

Seeks information and generates optional solutions to


perceived problems or issues through interactive problem
solving with a community, system, and family or individual

Collaboration
Commits two or more persons or organization to achieve a
common goal through enhancing the capacity of one or
more of the members to promote and protect health

Coalition Building
Promotes and develops alliances among organizations or
constituencies for a common purpose

Community Organizing
Helps community groups to identify common problems or
goals, mobilize resources, and develop and implement
strategies for reaching the goals they collectively have set

Advocacy
Plead someone's cause or act on someone's behalf, with a
focus on developing the community, system, and individual
or family's capacity to plead their own cause or act on their
own behalf

Social Marketing
Utilizes commercial marketing principles and technologies
for programs designed to influence the knowledge,
attitudes, values, beliefs, behaviors, and practices of the
population of interest

Policy development and enforcement


Places health issues on decision-makers' agendas, acquires
a plan of resolution, and determines needed resources,
resulting in laws, rules, regulations, ordinances, and
policies. Policy enforcement compels others to comply with
laws, rules, regulations, ordinances, and policies

John Snow
_______: Father of epidemiology

1. Pasteur
2. Lister
3. Koch
____, _____, _____: On "germs" and disease causation (late
1800s)

1. Edward Jenner
2. Edwick Chadwick
History of Public Health Nursing:
Pre 1850: Home visiting to sick as an act of charity.
- ______ observed people who worked around cattle were
less likely to have smallpox.
- _______ called attention to the consequences of unsanitary
conditions that resulted in health disparities that shortened
the life span of the laboring class in particular.

1. John Snow
2. Florence Nightingale
3. Pasteur, Lister, Koch
4. Lilian Wald
5. Mary Brewster
History of Public Health Nursing:
1850-1900: Home visiting to sick
- _______ demonstrated that cholera was transmissible
through contaminated water.
-________ - credited with establishing "modern nursing."
- ________: On "germs" and disease causation (late 1800s)
- In 1893, nurses _______ ("mother of community nursing")
and _______ established a district nursing service on the
lower east side of New York City called The House on Henry
Street.

1. 1900-1960s
History of Public Health Nursing:
________:
- Nursing in community centers for the poor
- Communicable disease control
- Communicable disease control/immunization
- Rise in PHN, home visits, school and OH nursing

1. 1960-1980
History of Public Health Nursing:
________: Care provided in public health clinics

1. 1980s-present
History of Public Health Nursing:

_________:
- Health promotion and education
- Health care access improvement

1. Poverty
2. Violence
3. HIV/AIDS
4. TB
5. Bioterrorism
Public/Community Health Nursing Today in U.S.
- Concerned with contemporary problems such as _____,
______, ______, _______, and _______.
- Practice is everywhere in public health agencies, schools,
occupational settings, community-based agencies, etc.

Healthy People
__________:
- provides science-based, 10-year national objectives for
improving the health of all Americans. For 3 decades, this
has established benchmarks and monitored progress over
time in order to:
o Encourage collaborations across communities and
sectors.
o Empower individuals toward making informed health
decisions.
o Measure the impact of prevention activities.
- Vision: A society in which all people live long, healthy lives

1. Identify nationwide health improvement priorities.


2. Increase public awareness and understanding of the
determinants of health, disease, and disability and the
opportunities for progress.
3. Provide measurable objectives and goals that are
applicable at the national, State, and local levels.
4. Engage multiple sectors to take actions to strengthen
policies and improve practices that are driven by the best
available evidence and knowledge.
5. Identify critical research, evaluation, and data
collection needs.
Healthy People 2020 strives to:
1.
2.
3.
4.
5.

1. Attain high-quality, longer lives free of preventable


disease, disability, injury, and premature death.
2. Achieve health equity, eliminate disparities, and
improve the health of all groups.
3. Create social and physical environments that promote
good health for all.
4. Promote quality of life, healthy development, and
healthy behaviors across all life stages.

Text Mode Text version of the exam


1. Which is the primary goal of community health nursing?
A.

To support and supplement the efforts of the medical


profession in the promotion of health and prevention of illness
B.
To enhance the capacity of individuals, families and
communities to cope with their health needs
C.
To increase the productivity of the people by providing them
with services that will increase their level of health
D.
To contribute to national development through promotion of
family welfare, focusing particularly on mothers and children.
2. CHN is a community-based practice. Which best explains this statement?
A.
B.

The service is provided in the natural environment of people.


The nurse has to conduct community diagnosis to determine
nursing needs and problems.
C.
The services are based on the available resources within the
community.
D.
Priority setting is based on the magnitude of the health
problems identified.
3. Population-focused nursing practice requires which of the following
processes?
A.
B.
C.
D.

A.
B.
C.

Community organizing
Nursing process
Community diagnosis
Epidemiologic process
4. R.A. 1054 is also known as the Occupational Health Act. Aside from
number of employees, what other factor must be considered in determining
the occupational health privileges to which the workers will be entitled?
Type of occupation: agricultural, commercial, industrial
Location of the workplace in relation to health facilities
Classification of the business enterprise based on net profit

D.

Sex and age composition of employees


5. A business firm must employ an occupational health nurse when it has at
least how many employees?

A.
B.
C.
D.

21
101
201
301
6. When the occupational health nurse employs ergonomic principles, she is
performing which of her roles?

A.
B.
C.
D.

Health care provider


Health educator
Health care coordinator
Environmental manager
7. A garment factory does not have an occupational nurse. Who shall provide
the occupational health needs of the factory workers?

A.
B.
C.
D.

Occupational health nurse at the Provincial Health Office


Physician employed by the factory
Public health nurse of the RHU of their municipality
Rural sanitary inspector of the RHU of their municipality
8. Public health services are given free of charge. Is this statement true or
false?

A.

The statement is true; it is the responsibility of government to


provide basic services.
B.
The statement is false; people pay indirectly for public health
services.
C.
The statement may be true or false, depending on the specific
service required.
D.
The statement may be true or false, depending on policies of
the government concerned.
9. According to C.E.Winslow, which of the following is the goal of Public
Health?
A.
B.
C.
D.

For
For
For
For

people to attain their birthrights of health and longevity


promotion of health and prevention of disease
people to have access to basic health services
people to be organized in their health efforts

10. We say that a Filipino has attained longevity when he is able to reach the
average lifespan of Filipinos. What other statistic may be used to determine
attainment of longevity?
A.
B.
C.
D.

Age-specific mortality rate


Proportionate mortality rate
Swaroops index
Case fatality rate
11. Which of the following is the most prominent feature of public health
nursing?

A.

It involves providing home care to sick people who are not


confined in the hospital.
B.
Services are provided free of charge to people within the
catchment area.
C.
The public health nurse functions as part of a team providing a
public health nursing services.
D.
Public health nursing focuses on preventive, not curative,
services.
12. According to Margaret Shetland, the philosophy of public health nursing is
based on which of the following?
A.
B.

Health and longevity as birthrights


The mandate of the state to protect the birthrights of its
citizens
C.
Public health nursing as a specialized field of nursing
D.
The worth and dignity of man
13. Which of the following is the mission of the Department of Health?
A.
B.
C.
D.

Health for all Filipinos


Ensure the accessibility and quality of health care
Improve the general health status of the population
Health in the hands of the Filipino people by the year 2020
14. Region IV Hospital is classified as what level of facility?

A.
B.
C.
D.

Primary
Secondary
Intermediate
Tertiary
15. Which is true of primary facilities?

A.

They are usually government-run.

B.
C.
D.

Their services are provided on an out-patient basis.


They are training facilities for health professionals.
A community hospital is an example of this level of health
facilities.
16. Which is an example of the school nurses health care provider functions?

A.

Requesting for BCG from the RHU for school entrant


immunization
B.
Conducting random classroom inspection during a measles
epidemic
C.
Taking remedial action on an accident hazard in the school
playground
D.
Observing places in the school where pupils spend their free
time
17. When the nurse determines whether resources were maximized in
implementing Ligtas Tigdas, she is evaluating
A.
B.
C.
D.

Effectiveness
Efficiency
Adequacy
Appropriateness
18. You are a new B.S.N. graduate. You want to become a Public Health
Nurse. Where will you apply?

A.
B.
C.
D.

Department of Health
Provincial Health Office
Regional Health Office
Rural Health Unit
19. R.A. 7160 mandates devolution of basic services from the national
government to local government units. Which of the following is the major goal
of devolution?

A.
B.
C.
D.

To strengthen local government units


To allow greater autonomy to local government units
To empower the people and promote their self-reliance
To make basic services more accessible to the people
20. Who is the Chairman of the Municipal Health Board?

A.
B.
C.
D.

Mayor
Municipal Health Officer
Public Health Nurse
Any qualified physician

21. Which level of health facility is the usual point of entry of a client into the
health care delivery system?
A.
B.
C.
D.

Primary
Secondary
Intermediate
Tertiary
22. The public health nurse is the supervisor of rural health midwives. Which
of the following is a supervisory function of the public health nurse?

A.
B.
C.
D.

Referring cases or patients to the midwife


Providing technical guidance to the midwife
Providing nursing care to cases referred by the midwife
Formulating and implementing training programs for midwives
23. One of the participants in a hilot training class asked you to whom she
should refer a patient in labor who develops a complication. You will answer,
to the

A.
B.
C.
D.

Public Health Nurse


Rural Health Midwife
Municipal Health Officer
Any of these health professionals
24. You are the public health nurse in a municipality with a total population of
about 20,000. There are 3 rural health midwives among the RHU personnel.
How many more midwife items will the RHU need?

A.
B.
C.
D.

1
2
3
The RHU does not need any more midwife item.
25. If the RHU needs additional midwife items, you will submit the request for
additional midwife items for approval to the

A.
B.
C.
D.

Rural Health Unit


District Health Office
Provincial Health Office
Municipal Health Board
26. As an epidemiologist, the nurse is responsible for reporting cases of
notifiable diseases. What law mandates reporting of cases of notifiable
diseases?

A.
B.

Act 3573
R.A. 3753

C.
D.

R.A. 1054
R.A. 1082
27. According to Freeman and Heinrich, community health nursing is a
developmental service. Which of the following best illustrates this statement?

A.

The community health nurse continuously develops himself


personally and professionally.
B.
Health education and community organizing are necessary in
providing community health services.
C.
Community health nursing is intended primarily for health
promotion and prevention and treatment of disease.
D.
The goal of community health nursing is to provide nursing
services to people in their own places of residence.
28. Which disease was declared through Presidential Proclamation No. 4 as a
target for eradication in the Philippines?
A.
B.
C.
D.

Poliomyelitis
Measles
Rabies
Neonatal tetanus
29. The public health nurse is responsible for presenting the municipal health
statistics using graphs and tables. To compare the frequency of the leading
causes of mortality in the municipality, which graph will you prepare?

A.
B.
C.
D.

Line
Bar
Pie
Scatter diagram
30. Which step in community organizing involves training of potential leaders
in the community?

A.
B.
C.
D.

Integration
Community organization
Community study
Core group formation

Answers and Rationales


1.

Answer: (B) To enhance the capacity of individuals,


families and communities to cope with their health needs
2.
Answer: (B) The nurse has to conduct community
diagnosis to determine nursing needs and problems.
3.
Answer: (C) Community diagnosis. Population-focused
nursing care means providing care based on the greater need of the

majority of the population. The greater need is identified through


community diagnosis.
4.
Answer: (B) Location of the workplace in relation to
health facilities. Based on R.A. 1054, an occupational nurse must
be employed when there are 30 to 100 employees and the
workplace is more than 1 km. away from the nearest health center.
5.
Answer: (B) 101. Again, this is based on R.A. 1054.
6.
Answer: (D) Environmental manager. Ergonomics is
improving efficiency of workers by improving the workers
environment through appropriately designed furniture, for example.
7.
Answer: (C) Public health nurse of the RHU of their
municipality. Youre right! This question is based on R.A.1054.
8.
Answer: (B) The statement is false; people pay
indirectly for public health services. Community health
services, including public health services, are pre-paid services,
though taxation, for example.
9.
Answer: (A) For people to attain their birthrights of
health and longevity. According to Winslow, all public health
efforts are for people to realize their birthrights of health and
longevity.
10.
Answer: (C) Swaroops index. Swaroops index is the
percentage of the deaths aged 50 years or older. Its inverse
represents the percentage of untimely deaths (those who died
younger than 50 years).
11.
Answer: (D) Public health nursing focuses on
preventive, not curative, services.The catchment area in PHN
consists of a residential community, many of whom are well
individuals who have greater need for preventive rather than
curative services.
12.
Answer: (D) The worth and dignity of man. This is a direct
quote from Dr. Margaret Shetlands statements on Public Health
Nursing.
13.
Answer: (B) Ensure the accessibility and quality of
health care
14.
Answer: (D) Tertiary. Regional hospitals are tertiary facilities
because they serve as training hospitals for the region.
15.
Answer: (B) Their services are provided on an outpatient basis. Primary facilities government and non-government
facilities that provide basic out-patient services.
16.
Answer: (B) Conducting random classroom inspection
during a measles epidemic. Random classroom inspection is

assessment of pupils/students and teachers for signs of a health


problem prevalent in the community.
17.
Answer: (B) Efficiency. Efficiency is determining whether the
goals were attained at the least possible cost.
18.
Answer: (D) Rural Health Unit. R.A. 7160 devolved basic
health services to local government units (LGUs ). The public health
nurse is an employee of the LGU.
19.
Answer: (C) To empower the people and promote their
self-reliance. People empowerment is the basic motivation behind
devolution of basic services to LGUs.
20.
Answer: (A) Mayor. The local executive serves as the
chairman of the Municipal Health Board.
21.
Answer: (A) Primary. The entry of a person into the health
care delivery system is usually through a consultation in out-patient
services.
22.
Answer: (B) Providing technical guidance to the
midwife. The nurse provides technical guidance to the midwife in
the care of clients, particularly in the implementation of
management guidelines, as in Integrated Management of Childhood
Illness.
23.
Answer: (C) Municipal Health Officer. A public health nurse
and rural health midwife can provide care during normal childbirth.
A physician should attend to a woman with a complication during
labor.
24.
Answer: (A) 1. Each rural health midwife is given a population
assignment of about 5,000.
25.
Answer: (D) Municipal Health Board. As mandated by R.A.
7160, basic health services have been devolved from the national
government to local government units.
26.
Answer: (A) Act 3573. Act 3573, the Law on Reporting of
Communicable Diseases, enacted in 1929, mandated the reporting
of diseases listed in the law to the nearest health station.
27.
Answer: (B) Health education and community organizing
are necessary in providing community health services. The
community health nurse develops the health capability of people
through health education and community organizing activities.
28.
Answer: (B) Measles. Presidential Proclamation No. 4 is on
the Ligtas Tigdas Program.
29.
Answer: (B) Bar. A bar graph is used to present comparison
of values, a line graph for trends over time or age, a pie graph for

population composition or distribution, and a scatter diagram for


correlation of two variables.
30.
Answer: (D) Core group formation. In core group
formation, the nurse is able to transfer the technology of community
organizing to the potential or informal community leaders through a
training program.
1. Which is the primary goal of community health nursing?
A. To support and supplement the efforts of the medical profession in the promotion of
health and prevention of
B. To enhance the capacity of individuals, families and communities to cope with their health
needs
C. To increase the productivity of the people by providing them with services that will
increase their level of health
D. To contribute to national development through promotion of family welfare, focusing
particularly on mothers and children
2. CHN is a community-based practice. Which best explains this statement?
A. The service is provided in the natural environment of people
B. The nurse has to conduct community diagnosis to determine nursing needs and
problems
C. The service are based on the available resources within the community
D. Priority setting is based on the magnitude of the health problems identified
3. Population- focused nursing practice requires which of the following processes?
A. Community organizing .
B. Nursing, process
C. Community diagnosis
D. Epidemiologic process
4. RA 1054 is also known as the Occupational Health Act. Aside from the number of
employees, what other factor must be considered in determining the occupational
health privileges to which the workers will be entitled?
A. Type of occupation,: agriculture, commercial, industrial
B. Location of the workplace in relation to health facilities

C. Classification of the business enterprise based on net profit


D. Sex and age composition of employees
5. A business firm must employ an occupational health nurse when it has at least
how many employees.
A. 21
B. 101
C. 201
D. 301
6. When the occupational health nurse employs ergonomic principles, she is
performing which of her roles?
A. Health care provider
B. Health educator
C. Health care coordinator
D. Environment manager
7. A garment factory does not have an occupational nurse. Who shall provide the
occupational health needs of the factory workers?
A. Occupational health nurse at the Provincial Health Office
B. Physician employed by the factory
C. Public Health nurse of the RHU of their municipality
D. Rural Sanitary inspector of the RHU in their municipality
8. Public health services are given free of charge. Is this statement true or false?
A. The statement is true; it is the responsibility of government to provide haste services
B. The statement is false; people pay indirectly for public health services
C. The statement may be true or false; depending on the Specific service required
D. The statement may be true or false; depending on policies of the government concerned.
9. According to C.E. Winslow, which of the following is the goal of Public Health?
A. For people to attain their birthrights and longevity
B. For promotion of health and prevention and diseases
C. For people to have access to basic health services
D. For people to be organized in their health efforts

10. We say that a Filipino has attained longevity when he is able to reach the average
life span of Filipinos. What other statistic may be used to determine attainment of
longevity?
A. Age-specific mortality rate
B. Proportionate mortality rate
C. Swaroops index
D. Case fatality rate
11. Which of the following is the most prominent feature of public health nursing?
A. It involves providing home care to sick people who are not confined in the hospital
B. Services are provided free of charge to people within the catchment area
C. The public health nurse functions as part of a team providing a public health nursing
service
D. Public health nursing focuses on preventive, not curative services
12. According to Margaret Shetland, the philosophy of public health nursing is based
on which of the following?
A. Health and longevity as birthrights
B. The mandate of the state to protect the birthrights of its citizens
C. Public health nursing as a specialized field of nursing
D. The worth and dignity of man
13. Which of the following is the mission of the Department of Health?
A. Health for all Filipinos
B. Ensure the accessibility and quality of health
C. Improve the general health status of the population
D. Health in the hands of the Filipino people by the year 2020
14. Region IV Hospital is classified as what level of facility?
A. Primary
B. Secondary
C. Intermediate
D. Tertiary
15. What is true of primary facilities?

A. They are usually government-run


B. Their services are provided on an out-patient basis
C. They are training facilities for health professionals
D. A community hospital is an example of this level of health facilities
16. Which is an example of the school nurses health care provider function?
A. Requesting for BCG from the RHU for school entrance immunization
B. Conducting random classroom inspection during measles epidemic
C. Taking remedial action on an accident hazard in the school playground
D. Observing places in the school where pupils spend their free times
17. When the nurse determines whether resources were maximized in implementing
Ligtas Tigdas, she is evaluating:
A. Effectiveness
B. Efficiency
C. Adequacy
D. Appropriateness
18. You are a new B.S.N. graduate. You want to become a Public Health Nurse. Where
will you apply?
A. Department of Health
B. Provincial Health Office
C. Regional Health Office
D. Rural Health Unit
19. RA 7160 mandates devolution of basic services from the national government to
local government units. Which of the following is the major goal of devolution?
A. To strengthen local government units
B. To allow greater autonomy to local government units.
C. To empower the people and promote their self-reliance
D. To make basic services more accessible to the people
20. Who is the Chairman of the Municipal Health Board?
A. Mayor
B. Municipal Health Officer

C. Public Health Nurse


D. Any qualified physician
21. Which level of health facility is the usual point of entry of a client into the health
care delivery system?
A. Primary
B. Secondary
C. Intermediate
D. Tertiary
22. The public health nurse is the supervisor of rural health midwives. Which of the
following is a supervisory function of the pubic health nurse?
A. Referring cases or patients to the midwife
B. Providing technical guidance to the midwife
C. Proving nursing care to cases referred by the midwife
D. Formulating and implementing training programs for midwives
23. One of the participants in a hilot training class asked you to whom she should
refer a patient in labor who develops a complication. You will answer, to the;
A. Public health nurse
B. Rural health midwife
C. Municipal health officer
D. Any of these health professionals
24. You are the public health nurse in a municipality with a total population of about
20,000. There are3 health midwives among the RHU personnel. How many more
midwife items will the RHU need?
A. 1
B. 2
C. 3
D. 4
25. If the RHU needs additional midwife items, you will submit the request for
additional midwife items for approval to the:

A. Rural Health Unit


B. District Health Office
C. Provincial Health Office
D. Municipal Health Board
26. As an epidemiologist, the nurse is responsible for reporting cases or notifiable
diseases. What law mandates reporting cases of notifiable diseases?
A. Act 3573
B. RA.3753
C. RA 1054
D. RA 1082
27. According to Freeman and Heinrich, community health nursing is a
developmental service. Which of the following best illustrates this statement?
A. The community health nurse continuously develops himself personally and professionally
B. Health education and community organizing are necessary in providing community
health services
C. Community health nursing in intended primarily for health promotion and prevention and
treatment of disease.
D. The goal of community health nursing is to provide nursing services to people in their
own place of .residence
28. Which disease was declared through Presidential Proclamation No. 4 as a target
for, eradication in the Philippines?
A. Pioliomyelitis
B. Measles
C. Rabies
D. Neonatal Tetanus
29. The public health nurse is responsible for presenting the municipal health
statistics using graphs and tables. To compare the frequency of the leading causes
of mortality in the municipality, which graph will you prepare?
A. Line
B. Bar
C. Pie
D. Scatter diagram

30. Which step in community organizing involves training of potential leaders in the
community?
A. Integration
B. Community organization
C. Community study
D. Core group formation
31. In which step are plans formulated for solving community problems?
A. Mobilization
B. Community organization
C. Follow-up/extension
D. Core group formation
32. The public health nurse takes an active role in community participation. What is
the primary goal of community organizing?
A. To educate the people regarding community health problems
B. To mobilize the people to resolve community health problems
C. To maximize the communitys resources in dealing with health problems
33. An indicator of success in community organizing is when people are able to:
A. Participate in community activities for the solution of a community problem
B. Implement activities for the solution of the community problem
C. Plan activities for the solution of the community problem
D. Identify the health problem as a common concern
34. Tertiary prevention is needed in which stage of the natural history of disease?
A. Pre-pathogenesis
B. Pathogenesis
C. Predromal
D. Terminal
35. Isolation of a child with measles belongs to what level of prevention?
A. Primary
B. Secondary

C. Intermediate
D. Tertiary
36. On the other hand, Operation Timbang is_____ prevention?
A. Primary
B. Secondary
C. Intermediate
D. Tertiary
37. Which type of family-nurse contact will provide you with the best opportunity to
observe family dynamics?
A. Clinic consultation
B. Group conferences
C. Home visit
D. Written communication
38. The typology of family nursing problems is used in the statement of nursing
diagnosis in the care of families. The youngest child of the delos Reyes family has
been diagnosed as mentally retarded. This is classified as:
A. Health threat
B. Health deficit
C. Foreseeable crisis
D. Stress point
39. The delos Reyes couple have 6-year old child entering school for the first time.
The delos Reyes family has a:
A. Health threat
B. Health deficit
C. Foreseeable crisis
D. Stress point
40. Which of the following is an advantage of a home visit?
A. It allows the nurse to provide nursing care to a greater number of people
B. It provides an opportunity to do first hand appraisal of the home situation

C. It allows sharing of experience among people with similar health problems


D. It develops the familys initiative in providing for health needs of its members
41. Which is CONTRARY to the principles in planning a home visit?
A. A home visit should have a purpose of objective
B. The plan should revolve around the family health .needs
C. A home visit should be conducted in the manner prescribed by RHU
D. Planning of continuing care should involve a responsible-family member
42. The PHN bag is an important tool in providing nursing care during a home visit.
The most important principle in bag technique states that it;
A. Should save time and effort
B. Should minimize if not totally prevent the spread of infection
C. Should not overshadow concern for the patient and his family
D. May be done in variety of ways depending on the home situation, etc.
43. To maintain the cleanliness of the bag and its contents, which of the following
must the nurse do?
A. Wash his/her hands before and after providing nursing care to the family members
B. In the care of family members, as much as possible, use only articles taken from the bag
C. Put on an apron to protect her uniform and fold it with the right side out before putting it
back into the bag.
D. At the end of the visit, fold the lining on which the bag was placed, ensuring that the
contaminated side is on the outside.
44. The public health conducts a study on the factors contributing to the high
morality rate due to heart diseases in the municipality where she works. Which
branch of epidemiology does the nurse practice in this situation?
A. Descriptive
B. Analytical
C. Therapeutic
D. Evaluation
45. Which of the following is a function of epidemiology?

A. Identifying the disease condition based on manifestations presented by a client


B. Determining factors that contributed to the occurrence of pneumonia in a 3 year old
C. Determining the efficacy of the antibiotic used in the treatment of the 3 year old client with
pneumonia
D. Evaluating the effectiveness of the implementation of the Integrated Management of
Childhood Illness
46. Which of the following is an epidemiologic function of the nurse during an
epidemic?
A Conducting assessment of suspected cases to detect the communicable diseases
B. Monitoring the condition of the cases affected by the communicable disease
C. Participating in the investigation to determine the source of epidemic
D. Teaching the community on preventive measures against the disease
47. The primary purpose of conducting an epidemiologic investigation is to;
A. Delineate the etiology of the epidemic
B. Encourage cooperation and support of the community
C. Identify groups who are at risk of contracting the disease
D. Identify geographical location of cases of the disease in the community
48. Which is a characteristic of person-to-person propagated epidemic?
A. There are more cases of the disease than expected
B. The disease must necessarily be transmitted through a vector
C. The spread of the disease can be attributed to a common vehicle
D. There is gradual build up of cases before we epidemic becomes easily noticeable
49. In the investigation of an epidemic, you compare the present frequency of the
disease with the usual frequency at this time of the year in this community. This is
done during which stage of the investigation?
A. Establishing the epidemic
B. Testing the hypothesis
C. Formulation of the hypothesis
D. Appraisal of facts
50. The number of cases of Dengue fever usually increases towards the end of the
rainy season. This pattern of occurrence of Dengue fever is best described as;

A. Epidemic occurrence
B. Cyclical variation
C. Sporadic occurrence
D. Secular occurrence
51. In the year 1980, the World Health Organization declared the Philippines, together
with some other countries in the Western Pacific Region, free of which disease?
A. Pneumonic plaque
B. Poliomyelitis
C. Small pox
D. Anthrax
52. In the census of the Philippines in 1995, there were about 35,299,000 males and
about 34,968,000 females. What is the sex ratio?
A. 99.06:100
B. 100.94:100
C. 50.23%
D. 49.76%
53. Primary health care is a total approach to community development. Which of the
following is an indicator of success in the use of the primary health care approach?
A. Health services are provided free of charge to individuals and families
B. Local officials are empowered as the major decision makers in matters of health
C. Health workers are able too provide care based on identified health needs of the people
D. Health programs are sustained according to the level of development of the community
54. Sputum examination is the major screening tool for pulmonary tuberculosis.
Clients would sometimes get false negative results in this exam. This means that the
test is not perfect in terms of which characteristic of a diagnostic examination?
A. Effectiveness
B. Efficacy
C. Specificity
D. Sensitivity
55. Use of appropriate technology requires knowledge of indigenous technology.
Which medical herb is given for fever, headache and cough?

A. Sambong
B. Tsaang gubat
C. Akapulko
D. Lagundi
56. What law created the Philippine institute of Traditional and Alternative Health
Care?
A. RA 8483
B. RA4823
C. RA 2483
D. RA 3482
57. In traditional Chinese medicine, the yielding, negative and feminine force is
termed:
A. Yin
B. Yang
C. Qi
D. Chai
58. What is the legal basis of Primary Health Care approach in the Philippines?
A. Alma Ata Declaration of PHC
B. Letter of Instruction No 949
C. Presidential Decree No. 147
D. Presidential Decree 996
59. Which of the following demonstrates inter-sectoral linkages?
A. Two-way referral system
B. Team approach
C. Endorsement done by a midwife to another midwife
D. Cooperation between PHN and public school teacher
60. The municipality assigned to you has a population of about 20/000. Estimate the
number of 1-4 year old children who be given Retinol capsule 200.000 every 6
months.

A. 1,500
B. 1,800
C. 2,000
D. 2,300
61. Estimate the number of pregnant women who will be given tetanus toxoid during
an immunization outreach activity in a barangay with a population of about 1,500.
A. 265
B. 300
C. 375
D. 400
62. To describe the sex composition of the population, which demographic tool may
be used?
A. Sex ratio
B. Sex proportion
C. Population pyramid
D. Any of these maybe used
63. Which of the following is a natality rate?
A. Crude birth rate
B. Neonatal mortality rate
C. Infant mortality rate
D. General fertility rate
64. You are computing the crude rate of your municipality, with a total population o
about 18,000 for last year. There were 94 deaths. Among those who died, 20 died
because of diseases of the heart and 32 were aged 50 years or older. What is the
crude death rate?
A. 4.1/1000
B. 5.2/1000
C. 6.3/1000
D. 7.3/1000

65. Knowing that malnutrition is a frequent community health problem, you decided
to conduct nutritional assessment. What population is particularly susceptible to
protein energy malnutrition (PEM)?
A. Pregnant women and the elderly
B. Under 5 year old children
C. 1-4 year old children
D. School age children
66. Which statistic can give the most accurate reflection of the health status of a
community?
A. 1-4 year old age-specific mortality rate
B. Infant mortality rate
C. Swaroops index
D. Crude death rate
67. In the past year, Barangay A had an average population of 1655. 46 babies were
born in that year, 2 of whom died less than 4 weeks after they were born. They were 4
recorded stillbirths. What is the neonatal mortality rate?
A. 27.8/1000
B. 43.5/1000
C. 86.9/1000
D. 130.4/1000
68. Which statistic best reflects the nutritional status of a population?
A. 1-4 year old age-specific mortality rate
B. Proportionate mortality rate
C. Infant mortality rate
D. Swaroops index
69. What numerator is used in computing general fertility rate?
A. Estimated midyear population
B. Number of registered live births
C. Number of pregnancies in the year
D. Number of females of reproductive age

70. You will gather data for nutritional assessment of a purok. You will gather
information only from families with members who belong to the target population for
PEM. What method of delta gathering is best for this purpose?
A. Census
B. Survey
C. Record Review
D. Review of civil registry
71. In the conduct of a census, the method of population assignment based on the
actual physical location of the people is termed;
A. De jure
B. De locus
C. De facto
D. De novo
72. The Field Health Services and information System (FHSIS) is the recording and
reporting system in public health) care in the Philippines. The monthly field health
service activity report is a form used in which of the components of the FHSIS?
A. Tally report
B. Output report
C. Target/client list
D. Individual health record
73. To monitor clients registered in long-term regimens, such as the Multi-Drug
Therapy, which component will be most useful?
A. Tally report
B. Output report
C. Target/client list
D. Individual health record
74. Civil registries are important sources of data. Which law requires registration of
births within 30 days from the occurrence of the birth?
A. PD 651
B. Act 3573

C. RA 3753
D. RA 3375
75. Which of the following professionals can sign the birth certificate?
A. Public health nurse
B. Rural health midwife
C. Municipal health officer
D. Any of these health professionals
76. Which criterion in priority setting of health problems is used only in community
health care?
A. Modifiability of the problem
B. Nature of the problem presented
C. Magnitude of the health problem
D. Preventive potential of the health problem
77. The Sentrong Sigla Movement has been launched to improve health service
delivery. Which of the following is/are true of this movement?
A. This is a project spearheaded by local government units
B. It is a basis for increasing funding from local government units
C. It encourages health centers to focus on disease prevention and control
D. Its main strategy is certification of health centers able to comply with standards
78. Which of the following women should be considered as special targets for family
planning?
A. Those who have two children or more
B. Those with medical conditions such as anemia
C. Those younger than 20 years and older than 35 years
D. Those who just had a delivery within the past 15 months
79. Freedom of choice in one of the policies of the Family Planning Program of the
Philippines. Which of the following illustrates this principle?
A. Information dissemination about the need for family planning
B. Support of research and development in family planning methods
C. Adequate information for couples regarding the different methods
D. Encouragement of couples to take family planning as a joint responsibility

80. A woman, 6 months pregnant, came to the center for consultation. Which of the
following substances is contraindicated?
A. Tetanus toxoid
B. Retinol 200,000 IU
C. Ferrous sulfate 200mg
D. Potassium iodate 200 mg, capsule
81. During prenatal consultation, a client asked you if she can have her delivery at
home. After history taking and physical examination, you advised her against a home
delivery. Which of the following findings disqualifies her for a home delivery?
A. Her OB score is G5P3
B. She has some palmar pallor
C. Her blood pressure is 130/80
D. Her baby is in cephalic presentation
82. Inadequate intake by the pregnant woman of which vitamin may cause neural
tube defects?
A. Niacin
B. Riboflavin
C. Folic Acid
D. Thiamine
83. You are in a clients home to attend to a delivery. Which of the following will you
do first?
A. Set up a sterile area
B. Put on a clean gown and apron
C. Cleanse the clients vulva with soap and water
D. Note the interval, duration and intensity of labor and contractions
84. In preparing a primigravida for breastfeeding, which of the following will you do?
A. Tell her that lactation begins within a day after delivery
B. Teach her nipple stretching exercises if her nipples are everted
C. Instruct her to wash her nipples before and after each breastfeeding
D. Explain to her that putting the baby to breast will lessen blood loss after delivery

85. A primigravida is instructed to offer her breast to the baby for the first time within
30 minutes after delivery. What is the purpose of offering the breast this early?
A. To initiate the occurrence of milk letdown
B. To stimulate milk production by the mammary acini
C. To make sure that the baby is able to get the colustrum
D. To allow the woman to practice breastfeeding in the presence of the health worker
86. In a mothers class, you discuss proper breastfeeding technique. Which of these
is a sign that the baby has lactated on the breast property?
A. The baby takes shallow, rapid sucks
B. The mother does not feel nipple pain
C. The babys mouth is only partly open
D. Only the mothers nipple is inside the babys mouth
87. You explain to a breastfeeding mother that breastmilk is sufficient for all of the
babys nutrient needs only up to:
A. 3 months
B. 6 months
C. 1 year
D. 2 years
88. What is given to a woman within a month after the delivery of a baby?
A. Malunggay capsule
B. Ferrous sutfate l00mg O.D.
C. Retinol 200.000 IU 1 capsule
D. Potassium Iodate 200 mg, 1 capsule
89. Which biological used in EPI is stored in the freezer?
A. DPT
B. Tetanus toxoid
C. Measles vaccine
D. Hepatitis B vaccine
90. Unused BCG should be discarded how many hours after reconstitution?

A. 2
B. 4
C. 6
D. At the end of the day
91. In immunity school entrants with BCG, you not obliged to secure parental
consent. This is because of which legal document?
A. PD 996
B. RA 7864
C. Presidential Proclamation No. 6
D. Presidential Proclamation No. 46
92. Which immunization produces a permanent scar?
A. DPT
B. BCG
C. Measles vaccination
D. Hepatitis B vaccination
93. A 4 week old baby was brought to the health center for his first immunization.
Which can be given to him?
A. DPT1
B. OPV1
C. Infant BCG
D. Hepatitis B Vaccin
94. You will not give DPT 2 if the mother says that the infant had?
A. Seizures a day after DPT1
B. Fever for 3 days after DPT1
C. Abscess formation after DPT1
D. Local tenderness for 3 days after DPT1
95. A 2-month old infant was brought to the health center for immunization. During
assessment, the infants temperature registered at 38.1 C. Which is the best course
of action that you will take?

A. Go on with the infants immunization


B. Give paracetamol and wait for his fever to subside
C. Refer the infant to the physician for further assessment
D. Advise the infants mother to bring him back for immunization when he is well
96. A pregnant woman had just received her 4th dose of tetanus toxoid.
Subsequently, her baby will have protection against tetanus for how long?
A. 1 year
B. 3 years
C. 10 years
D. Lifetime
97. A 4-month old infant was brought to the health center of cough. Her respiratory
rate is 42/minute. Using the IMCI guidelines of assessment, her breathing is
considered;
A. Fast
B. Slow
C. Normal
D. Insignificant
98. Which of the following signs will indicate that a young child is suffering from
severe pneumonia?
A. Dyspnea
B. Wheezing
C. Fast breathing
D. Chest indrawing
99. Using IMCI guidelines, you classify a child as having severe pneumonia. What is
the best management for the child?
A. Prescribe antibiotic
B. Refer him urgently to the hospital
C. Instruct the mother to increase fluid intake
D. Instruct the mother to continue breastfeeding
100. A 5-month old infant was brought by his mother to the health center because of
diarrhea occurring 4 to 5 times a day. His skin goes back slowly after a skin pinch

and his eyes are sunken. Using the IMCI guidelines, you will classify this infant in
which category?
A. No signs of dehydration
B. Some dehydration
C. Severe dehydration
D. The data is insufficient
101. Based on the assessment, you classified a 3-month old infant with the chief
complaint of diarrhea in the category of SOME DEHYDRATION. Based on the IMCI
management guidelines, which of the following will you do?
A. Bring the infant to the nearest facility where IV fluids can be given
B. Supervise the mother in giving 200 to 400 ml of Oresol in 4 hours
C. Give the infants mother instructions on home management
D. Keep the infant in your health center for close observation
102. A mother is using Oresol in the management of diarrhea of her 3-year old child.
She asked you what to do if her child vomits. You will tell her to:
A. Bring the child to the nearest hospital for further assessment
B. Bring the child to the health center for IV therapy
C. Bring the child to the health center for assessment by the physician
D. Let the child rest for 10 minutes then continue giving Oresol more slowly
103. A 1 1/2 year old child was classified as having 3rd degree of protein energy
malnutrition, kwashjorkor. Which of the following signs will be most apparent in this
child?
A. Voracious appetite
B. Wasting
C. Apathy
D. Edema
104. Assessment of a 2-year old child revealed baggy pants. Using the IMCI
guidelines, how will you manage this child?
A. Refer the child urgently to a hospital for confinement
B. Coordinate with the social worker to enroll the child in a feeding program
C. Make a teaching plan for the mother, focusing on the menu planning for her child
D. Assess and treat the child for health problems like infections and intestinal parasitism

105. During the physical examination of a young child, what is the earliest sign of
xerophthalmia that may observe?
A. Keratomalacia
B. Corneal opacity
C. Night blindness
D. Conjunctival xerosis
106. To prevent xerophthalmia, young children are given Retinol capsule every 6
months. What is the dose given to preschoolers?
A. 10, 000 IU
B. 20, 000 IU
C. 100, 000 IU
D. 200, 000 IU
107. The major sign of iron deficiency anemia is pallor. What part is best examined
for pallor?
A. Palms
B. Nailbeds
C. Around the lips
D. Lower conjunctival sac
108. Food fortification is one of the strategies to prevent micronutrient deficiency
conditions. RA 8976 mandates fortification of certain food items. Which of the
following is among these food items?
A. Sugar
B. Bread
C. Margarine
D. Filled milk
109. What is the best course of action when there is a measles epidemic in a nearby
municipality?
A. Give measles vaccine to babies aged 6 to 3 months
B. Give babies aged 6 to 11 months one dose of 100,000 IU of Retinol
C. Instruct mother to keep their babies at home to prevent disease transmission
D. Instruct mothers to feed their babies adequately to enhance their babies resistance

110. A mother brought her daughter, 4 years old, to the RHU because of cough and
colds. Following the IMCI assessment guide, which of the following is a danger sign
that indicates the need for urgent referral to a hospital?
A. Inability to drink
B. High grade fever
C. Signs of severe dehydration
D. Cough for more than 30 days
111. Management of a child with measles includes the administration of which of the
following?
A. Gentian violet on mouth lesions
B. Antibiotic to prevent pneumonia
C. Tetracycline eye ointment for corneal opacity
D. Retinol capsule regardless of when the last dose was given
112. A mother brought her 10 month old infant for consultation because of fever
which started 4 days prior to consultation. To determine malaria risk, what will you
do?
A. Do a tourniquet test
B. Ask where the family resides
C. Get a specimen for blood smear
D. Ask if the fever is present everyday
113. The following are strategies implemented by the DOH to prevent mosquito-borne
diseases. Which of these is most effective in the control of Dengue fever?
A. Stream seeding with larva-eating fish
B. Destroying breeding places of mosquitoes
C. Chemoprophylaxis of non-immune persons going to endemic areas
D. Teaching people in endemic areas to use chemically treated mosquito nets
114. Secondary prevention for malaria includes?
A. Planting of neem or eucalyptus trees
B. Residual spraying of insecticides at night
C. Determining whether a place is endemic or not
D. Growing larva-eating fish in mosquito breeding places

115. Scotch tape swab is done to check for which intestinal parasite?
A. Ascaris
B. Pinworm
C. Hookworm
D. Schistosoma
116. Which of the following signs indicates the need for sputum examination for
AFB?
A. Hematemesis
B. Fever for 1 week
C. Cough for 3 weeks
D. Chest pain for 1 week
117. Which clients are considered targets for DOTS category?
A. Sputum negative cavitary cases
B. Clients returning after default
C. Relapses and failures of previous PTB treatment regimens
D. Clients diagnosed for the first time through a positive sputum exam
118. To improve compliance to treatment, what innovation is being implemented in
DOTS?
A. Having the health worker follow up the client at home
B. Having the health worker or a responsible family member monitor drug intake
C. Having the patient come to the health center every month to get his medications
D. Having a target list to check on whether the patient has collected his monthly supply of
drugs
119. Diagnosis of leprosy is highly dependent on recognition of symptoms. Which of
the following is an early sign of leprosy?
A. Macular lesions
B. Inability to close eyelids
C. Thickened painful nerves
D. Sinking of the nose bridge

120. Which of the following clients should be classified as a case of mutibacillary


leprosy?
A. 3 skin lesions, negative slit skin smear
B. 3 skin lesions, positive slit skin smear
C. 5 skin lesions, negative slit skin smear
D. 5 skin lesions, positive slit skin smear
121. In the Philippines, which condition is the most frequent cause of death
associated by schistosomiasis?
A. Liver cancer
B. Liver cirrhosis
C. Bladder cancer
D. Intestinal perforation
122. What is the most effective way of controlling schistosomiasis in an endemic
area?
A. Use of molluscicides
B. Building of foot bridges
C. Proper use of sanitary toilets
D. Use of protective footwear, such as rubber boots
123. When residents obtain water from an artesian well in the neighborhood, the level
of this approved type of water facility is:
A. I
B. II
C. III
D. IV
124. For prevention of Hepatitis A, you decided to conduct health education
activities. Which of the following is Irrelevant?
A. Use of sterile syringes and needles
B. Safe food preparation and food handling by vendors
C. Proper disposal of human excreta and personal hygiene
D. Immediate reporting of water pipe leaks and illegal water connections

125. Which biological used in EPI should not be stored in the freezer?
A. DPT
B. OPV
C. Measles vaccine
D. MMR
126. You will conduct outreach immunization in a barangay with a population of
about 1500. Estimate the number of infants in the barangay.
A. 45
B. 50
C. 55
D. 60
127. In IMCI, severe conditions generally require urgent referral to a hospital. Which
of the following severe conditions Does not always require urgent referral to
hospital?
A. Mastoiditis
B. Severe dehydration
C. Severe pneumonia
D. Severe febrile disease.
128. A client was diagnosed as having Dengue Fever. You will say that there is slow
capillary refill when the color of the nailbed that you pressed does not return within
how many seconds?
A. 3
B. 5
C. 8
D. 10
129. A 3-year old child was brought by his mother to the health center because of
fever of 4-day duration. The child had a positive tourniquet test result. In the absence
of other signs, which of the most appropriate measure that the PHN may carry out to
prevent Dengue shock syndrome?
A. Insert an NGT and give fluids per NGT
B. Instruct the mother to give the child Oresol

C. Start the patient on IV Stat


D. Refer the client to the physician for appropriate management
130. The pathognomonic sign of measles is Kopliks spot. You may see Kopliks spot
by inspecting the:
A. Nasal Mucosa
B. Buccal mucosa
C. Skin on the abdomen
D. Skin on the antecubital surface
131. Among the following diseases, which is airborne?
A. Viral conjunctivitis
B. Acute poliomyelitis
C. Diphtheria
D. Measles
132. Among children aged 2 months to 3 years, the most prevalent form of meningitis
is caused by which microorganism?
A. Hemophilus Influenzae
B. Morbillivirus
C. Streptococcus Pneumoniae
D. Neisseria meningitides
133. Human beings are the major reservoir of malaria. Which of the following
strategies in malaria control is based on this fact?
A. Stream seeding
B. Stream clearing
C. Destruction of breeding places
D. Zooprophylaxis
134. The use of larvivorous fish in malaria control is the basis for which strategy of
malaria control?
A. Stream seeding
B. Stream clearing

C. Destruction of breeding places


D. Zooprophylaxis .
135. Mosquito-borne diseases are prevented mostly with the use of mosquito control
measures. Which of the following is NOT appropriate for malaria control?
A. Use of chemically treated mosquito nets
B. Seeding of breeding places with larva-eating fish
C. Destruction of breeding places of the mosquito vector
D. Use of mosquito-repelling soaps, such as those with basil or citronella
136. A 4-year old client was brought to the health center with chief complaint of
severe diarrhea and the passage of rice water. The client is most probably suffering
from which condition?
A. Giardiasis
B. Cholera
C. Amebiasis
D. Dysentery
137. In the Philippines, which specie of schistosoma is endemic in certain regions?
A. S. mansoni
B. S. japonicum
C. S. malayensis
D. S. haematobium
138. A 32 year old client came for consultation at the health center with the chief
complaint of fever for a week. Accompanying symptoms were muscle pains and body
malaise. A week after the start of fever, the client noted yellowish discoloration of his
sclera. History showed that he waded in flood waters about 2 weeks before the onset
of symptoms. Based on this history/ which disease condition will you suspect?
A. Hepatitis A
B. Hepatitis B
C. Tetanus
D. Leptospirosis
139. MWSS provides water to Manila and other cities in Metro Manila. This is an
example of which level of water facility?

A. I
B. II
C. III
D. IV
140. You are the PHN in the city health center. A client underwent screening for AIDS
using ELISA. His result was positive. What is the best course of action that you may
take?
A. Get a thorough history of the client, focusing on the practice of high risk behavior
B. Ask the client to be accompanied by a significant person before revealing the result.
C. Refer the client to the physician since he is the best person to reveal the result to the
client
D. Refer the client for a supplementary test, such as Western blot, since the ELISA result
maybe false
141. Which is the BEST control measure for AIDS?
A. Being faithful to a single sexual partner
B. Using a condom during each sexual contact
C. Avoiding sexual contact with commercial sex workers
D. Making sure that ones sexual partner does not have signs of AIDS
142. The most frequent causes of death among clients with AIDS are opportunistic
diseases. Which of the following opportunistic infections is characterized by
tonsilllopharyngitis?
A. Respiratory candidiasis
B. Infectious mononucleosis
C. Cytomegalovirus disease
D. Pneumocystis carinii pneumonia
143. To determine the possible sources of sexually transmitted infections, which is
the BEST method that may be undertaken by the public health nurse?
A. Contact tracing
B. Community survey
C. Mass screening tests
D. Interview suspects

144. Antiretroviral agents, such as AZT are used in the management of AIDS. Which
of the following is not an action expected of these drugs?
A. They prolong the life of the client with AIDS
B. They reduce the risk of opportunistic infections
C. They shorten the period of communicability of the disease
D. They are able to bring about a cure of the disease condition
145. A barangay had an outbreak of German measles. To prevent congenital rubella,
what is the BEST advice that you can give to women in the first trimester of
pregnancy in the barangay?
a. Advice them on the sign of German Measles
b. Avoid crowded places, such as markets and moviehouses
c. Consult at the health center where rubella vaccine may be given
d. Consult a physician who may give them rubella immunoglobulin

Answers and Rationale


Gauge your performance by counter-checking your answers to those below. If you have
disputes or further questions, please direct them to the comments section.
1. Answer: (B) To enhance the capacity of individuals, families and communities to cope
with their health needs.
To contribute to national development through promotion of family welfare, focusing
particularly on mothers and children.
2. Answer: (B) The nurse has to conduct community diagnosis to determine nursing needs
and problems.
Community-based practice means providing care to people in their own natural
environments: the home, school and workplace, for example.
3. Answer: (C) Community diagnosis
Population-focused nursing care means providing care based on the greater need of the
majority of the population. The greater need is identified through community diagnosis.
4. Answer: (B) Location of the workplace in relation to health facilities
Based on R.A. 1054, an occupational nurse must be employed when there are 30 to 100
employees and the workplace is more than 1 km. away from the nearest health center.
5. Answer: (B) 101

6. Answer: (D) Environmental manager


Ergonomics is improving efficiency of workers by improving the workers environment
through appropriately designed furniture, for example.
7. Answer: (C) Public health nurse of the RHU of their municipality
8. Answer: (B) The statement is false; people pay indirectly for public health services.
Community health services, including public health services, are prepaid paid services,
through taxation, for example.
9. Answer: (A) For people to attain their birthrights of health and longevity
According to Winslow, all public health efforts are for people to realize their birthrights of
health and longevity.
10. Answer: (C) Swaroops index
Swaroops index is the percentage of the deaths aged 50 years or older. Its inverse
represents the percentage of untimely deaths (those who died younger than 50 years).
11. Answer: (D) Public health nursing focuses on preventive, not curative, services.
The catchment area in PHN consists of a residential community, many of whom are well
individuals who have greater need for preventive rather than curative services.
12. Answer: (D) The worth and dignity of man
This is a direct quote from Dr. Margaret Shetlands statements on Public Health Nursing.
13. Answer: (B) Ensure the accessibility and quality of health care
14. Answer: (D) Tertiary
Regional hospitals are tertiary facilities because they serve as training hospitals for the
region.
15. Answer: (B) Their services are provided on an out-patient basis.
Primary facilities government and non-government facilities that provide basic out-patient
services.
16. Answer: (B) Conducting random classroom inspection during a measles epidemic
Random classroom inspection is assessment of pupils/students and teachers for signs of a
health problem prevalent in the community.

17. Answer: (B) Efficiency


Efficiency is determining whether the goals were attained at the least possible cost.
18. Answer: (D) Rural Health Unit
R.A. 7160 devolved basic health services to local government units (LGUs). The public
health nurse is an employee of the LGU.
19. Answer: (C) To empower the people and promote their self-reliance
People empowerment is the basic motivation behind devolution of basic services to LGUs.
20. Answer: (A) Mayor
The local executive serves as the chairman of the Municipal Health Board.
21. Answer: (A) Primary
The entry of a person into the health care delivery system is usually through a consultation
in out-patient services.
22. Answer: (B) Providing technical guidance to the midwife
The nurse provides technical guidance to the midwife in the care of clients, particularly in
the implementation of management guidelines, as in Integrated Management of Childhood
Illness.
23. Answer: (C) Municipal Health Officer
A public health nurse and rural health midwife can provide care during normal childbirth. A
physician should attend to a woman with a complication during labor.
24. Answer: (A) 1
Each rural health midwife is given a population assignment of about 5,000.
25. Answer: (D) Municipal Health Board
As mandated by R.A. 7160, basic health services have been devolved from the national
government to local government units.
26. Answer: (A) Act 3573
Act 3573, the Law on Reporting of Communicable Diseases, enacted in 1929, mandated
the reporting of diseases listed in the law to the nearest health station.
27. Answer: (B) Health education and community organizing are necessary in providing
community health services.

The community health nurse develops the health capability of people through health
education and community organizing activities.
28. Answer: (B) Measles
Presidential Proclamation No. 4 is on the Ligtas Tigdas Program.
29. Answer: (B) Bar
A bar graph is used to present comparison of values, a line graph for trends over time or
age, a pie graph for population composition or distribution, and a scatter diagram for
correlation of two variables.
30. Answer: (D) Core group formation
In core group formation, the nurse is able to transfer the technology of community
organizing to the potential or informal community leaders through a training program.
31. Answer: (B) Community organization
Community organization is the step when community assemblies take place. During the
community assembly, the people may opt to formalize the community organization and
make plans for community action to resolve a community health problem.
32. Answer: (D) To maximize the communitys resources in dealing with health problems
Community organizing is a developmental service, with the goal of developing the peoples
self-reliance in dealing with community health problems. A, B and C are objectives of
contributory objectives to this goal.
33. Answer: (A) Participate in community activities for the solution of a community problem
Participation in community activities in resolving a community problem may be in any of the
processes mentioned in the other choices.
34. Answer: (D) Terminal
Tertiary prevention involves rehabilitation, prevention of permanent disability and disability
limitation appropriate for convalescents, the disabled, complicated cases and the terminally
ill (those in the terminal stage of a disease)
35. Answer: (A) Primary
The purpose of isolating a client with a communicable disease is to protect those who are
not sick (specific disease prevention).

36. Answer: (B) Secondary


Operation Timbang is done to identify members of the susceptible population who are
malnourished. Its purpose is early diagnosis and, subsequently, prompt treatment.
37. Answer: (C) Home visit
Dynamics of family relationships can best be observed in the familys natural environment,
which is the home.
38. Answer: (B) Health deficit
Failure of a family member to develop according to what is expected, as in mental
retardation, is a health deficit.
39. Answer: (C) Foreseeable crisis
Entry of the 6-year old into school is an anticipated period of unusual demand on the family.
40. Answer: (B) It provides an opportunity to do first hand appraisal of the home situation.
Choice A is not correct since a home visit requires that the nurse spend so much time with
the family. Choice C is an advantage of a group conference, while choice D is true of a clinic
consultation.
41. Answer: (C) A home visit should be conducted in the manner prescribed by the RHU.
The home visit plan should be flexible and practical, depending on factors, such as the
familys needs and the resources available to the nurse and the family.
42. Answer: (B) Should minimize if not totally prevent the spread of infection.
Bag technique is performed before and after handling a client in the home to prevent
transmission of infection to and from the client.
43. Answer: (A) Wash his/her hands before and after providing nursing care to the family
members.
Choice B goes against the idea of utilizing the familys resources, which is encouraged in
CHN. Choices C and D goes against the principle of asepsis of confining the contaminated
surface of objects.
44. Answer: (B) Analytical
Analytical epidemiology is the study of factors or determinants affecting the patterns of
occurrence and distribution of disease in a community.
45. Answer: (D) Evaluating the effectiveness of the implementation of the Integrated
Management of Childhood Illness

Epidemiology is used in the assessment of a community or evaluation of interventions in


community health practice.
46. Answer: (C) Participating in the investigation to determine the source of the epidemic
Epidemiology is the study of patterns of occurrence and distribution of disease in the
community, as well as the factors that affect disease patterns. The purpose of an
epidemiologic investigation is to identify the source of an epidemic, i.e., what brought about
the epidemic.
47. Answer: (A) Delineate the etiology of the epidemic
Delineating the etiology of an epidemic is identifying its source.
48. Answer: (D) There is a gradual build up of cases before the epidemic becomes easily
noticeable.
A gradual or insidious onset of the epidemic is usually observable in person-to-person
propagated epidemics.
49. Answer: (A) Establishing the epidemic
Establishing the epidemic is determining whether there is an epidemic or not. This is done
by comparing the present number of cases with the usual number of cases of the disease at
the same time of the year, as well as establishing the relatedness of the cases of the
disease.
50. Answer: (B) Cyclical variation
A cyclical variation is a periodic fluctuation in the number of cases of a disease in the
community.
51. Answer: (C) Smallpox
The last documented case of Smallpox was in 1977 at Somalia.
52. Answer: (B) 100.94:100
Sex ratio is the number of males for every 100 females in the population.
53. Answer: (D) Health programs are sustained according to the level of development of the
community.
Primary health care is essential health care that can be sustained in all stages of
development of the community.
54. Answer: (D) Sensitivity
Sensitivity is the capacity of a diagnostic examination to detect cases of the disease. If a

test is 100% sensitive, all the cases tested will have a positive result, i.e., there will be no
false negative results.
55. Answer: (D) Lagundi
Sambong is used as a diuretic. Tsaang gubat is used to relieve diarrhea. Akapulko is used
for its antifungal property.
56. Answer: (A) R.A. 8423 or AN ACT CREATING THE PHILIPPINE INSTITUTE OF
TRADITIONAL AND ALTERNATIVE HEALTH CARE (PITAHC) TO ACCELERATE THE
DEVELOPMENT OF TRADITIONAL AND ALTERNATIVE HEALTH CARE IN THE
PHILIPPINES, PROVIDING FOR A TRADITIONAL AND ALTERNATIVE HEALTH CARE
DEVELOPMENT FUND AND FOR OTHER PURPOSES signed to a law on December 9,
1997.
57. Answer: (A) Yin
Yang is the male dominating, positive and masculine force.
58. Answer: (B) Letter of Instruction No. 949
Letter of Instruction 949 was issued by then President Ferdinand Marcos, directing the
formerly called Ministry of Health, now the Department of Health, to utilize Primary Health
Care approach in planning and implementing health programs.
59. Answer: (D) Cooperation between the PHN and public school teacher
Intersectoral linkages refer to working relationships between the health sector and other
sectors involved in community development.
60. Answer: (D) 2,300
Based on the Philippine population composition, to estimate the number of 1-4 year old
children, multiply total population by 11.5%.
61. Answer: (A) 265
To estimate the number of pregnant women, multiply the total population by 3.5%.
62. Answer: (D) Any of these may be used.
Sex ratio and sex proportion are used to determine the sex composition of a population. A
population pyramid is used to present the composition of a population by age and sex.
63. Answer: (A) Crude birth rate
Natality means birth. A natality rate is a birth rate.

64. Answer: (B) 5.2/1,000


To compute crude death rate divide total number of deaths (94) by total population (18,000)
and multiply by 1,000.
65. Answer: (C) 1-4 year old children
Preschoolers are the most susceptible to PEM because they have generally been weaned.
Also, this is the population who, unable to feed themselves, are often the victims of poor
intrafamilial food distribution.
66. Answer: (C) Swaroops index
Swaroops index is the proportion of deaths aged 50 years and above. The higher the
Swaroops index of a population, the greater the proportion of the deaths who were able to
reach the age of at least 50 years, i.e., more people grew old before they died.
67. Answer: (B) 43.5/1,000
To compute for neonatal mortality rate, divide the number of babies who died before
reaching the age of 28 days by the total number of live births, then multiply by 1,000.
68. Answer: (A) 1-4 year old age-specific mortality rate
Since preschoolers are the most susceptible to the effects of malnutrition, a population with
poor nutritional status will most likely have a high 1-4 year old age-specific mortality rate,
also known as child mortality rate.
69. Answer: (B) Number of registered live births
To compute for general or total fertility rate, divide the number of registered live births by the
number of females of reproductive age (15-45 years), then multiply by 1,000.
70. Answer: (B) Survey
A survey, also called sample survey, is data gathering about a sample of the population.
71. Answer: (C) De facto
The other method of population assignment, de jure, is based on the usual place of
residence of the people.
72. Answer: (A) Tally report
A tally report is prepared monthly or quarterly by the RHU personnel and transmitted to the
Provincial Health Office.

73. Answer: (C) Target/client list


The MDT Client List is a record of clients enrolled in MDT and other relevant data, such as
dates when clients collected their monthly supply of drugs.
74. Answer: (A) P.D. 651
P.D. 651 amended R.A. 3753, requiring the registry of births within 30 days from their
occurrence.
75. Answer: (D) Any of these health professionals
R.A. 3753 states that any birth attendant may sign the certificate of live birth.
76. Answer: (C) Magnitude of the health problem
Magnitude of the problem refers to the percentage of the population affected by a health
problem. The other choices are criteria considered in both family and community health
care.
77. Answer: (D) Its main strategy is certification of health centers able to comply with
standards.
Sentrong Sigla Movement is a joint project of the DOH and local government units. Its main
strategy is certification of health centers that are able to comply with standards set by the
DOH.
78. Answer: (D) Those who just had a delivery within the past 15 months
The ideal birth spacing is at least two years. 15 months plus 9 months of pregnancy = 2
years.
79. Answer: (C) Adequate information for couples regarding the different methods
To enable the couple to choose freely among different methods of family planning, they
must be given full information regarding the different methods that are available to them,
considering the availability of quality services that can support their choice.
80. Answer: (B) Retinol 200,000 IU
Retinol 200,000 IU is a form of megadose Vitamin A. This may have a teratogenic effect.
81. Answer: (A) Her OB score is G5P3.
Only women with less than 5 pregnancies are qualified for a home delivery. It is also
advisable for a primigravida to have delivery at a childbirth facility.

82. Answer: (C) Folic acid


It is estimated that the incidence of neural tube defects can be reduced drastically if
pregnant women have an adequate intake of folic acid.
83. Answer: (D) Note the interval, duration and intensity of labor contractions.
Assessment of the woman should be done first to determine whether she is having true
labor and, if so, what stage of labor she is in.
84. Answer: (D) Explain to her that putting the baby to breast will lessen blood loss after
delivery.
Suckling of the nipple stimulates the release of oxytocin by the posterior pituitary gland,
which causes uterine contraction. Lactation begins 1 to 3 days after delivery. Nipple
stretching exercises are done when the nipples are flat or inverted. Frequent washing dries
up the nipples, making them prone to the formation of fissures.
85. Answer: (B) To stimulate milk production by the mammary acini
Suckling of the nipple stimulates prolactin reflex (the release of prolactin by the anterior
pituitary gland), which initiates lactation.
86. Answer: (B) The mother does not feel nipple pain.
When the baby has properly latched on to the breast, he takes deep, slow sucks; his mouth
is wide open; and much of the areola is inside his mouth. And, youre right! The mother
does not feel nipple pain.
87. Answer: (B) 6 months
After 6 months, the babys nutrient needs, especially the babys iron requirement, can no
longer be provided by mothers milk alone.
88. Answer: (C) Retinol 200,000 I.U., 1 capsule
A capsule of Retinol 200,000 IU is given within 1 month after delivery. Potassium iodate is
given during pregnancy; malunggay capsule is not routinely administered after delivery; and
ferrous sulfate is taken for two months after delivery.
89. Answer: (C) Measles vaccine
Among the biologicals used in the Expanded Program on Immunization, measles vaccine
and OPV are highly sensitive to heat, requiring storage in the freezer.
90. Answer: (B) 4
While the unused portion of other biologicals in EPI may be given until the end of the day,

only BCG is discarded 4 hours after reconstitution. This is why BCG immunization is
scheduled only in the morning.
91. Answer: (A) P.D. 996
Presidential Decree 996, enacted in 1976, made immunization in the EPI compulsory for
children under 8 years of age. Hepatitis B vaccination was made compulsory for the same
age group by R.A. 7846.
92. Answer: (B) BCG
BCG causes the formation of a superficial abscess, which begins 2 weeks after
immunization. The abscess heals without treatment, with the formation of a permanent scar.
93. Answer: (C) Infant BCG
Infant BCG may be given at birth. All the other immunizations mentioned can be given at 6
weeks of age.
94. Answer: (A) Seizures a day after DPT 1.
Seizures within 3 days after administration of DPT is an indication of hypersensitivity to
pertussis vaccine, a component of DPT. This is considered a specific contraindication to
subsequent doses of DPT.
95. Answer: (A) Go on with the infants immunizations.
In the EPI, fever up to 38.5C is not a contraindication to immunization. Mild acute
respiratory tract infection, simple diarrhea and malnutrition are not contraindications either.
96. Answer: (A) 1 year
The baby will have passive natural immunity by placental transfer of antibodies. The mother
will have active artificial immunity lasting for about 10 years. 5 doses will give the mother
lifetime protection.
97. Answer: (C) Normal
In IMCI, a respiratory rate of 50/minute or more is fast breathing for an infant aged 2 to 12
months.
98. Answer: (D) Chest indrawing
In IMCI, chest indrawing is used as the positive sign of dyspnea, indicating severe
pneumonia.

99. Answer: (B) Refer him urgently to the hospital.


Severe pneumonia requires urgent referral to a hospital. Answers A, C and D are done for a
client classified as having pneumonia.
100. Answer: (B) Some dehydration
Using the assessment guidelines of IMCI, a child (2 months to 5 years old) with diarrhea is
classified as having SOME DEHYDRATION if he shows 2 or more of the following signs:
restless or irritable, sunken eyes, the skin goes back slow after a skin pinch.
101. Answer: (B) Supervise the mother in giving 200 to 400 ml. of Oresol in 4 hours.
In the IMCI management guidelines, SOME DEHYDRATION is treated with the
administration of Oresol within a period of 4 hours. The amount of Oresol is best computed
on the basis of the childs weight (75 ml/kg body weight). If the weight is unknown, the
amount of Oresol is based on the childs age.
102. Answer: (D) Let the child rest for 10 minutes then continue giving Oresol more slowly.
If the child vomits persistently, that is, he vomits everything that he takes in, he has to be
referred urgently to a hospital. Otherwise, vomiting is managed by letting the child rest for
10 minutes and then continuing with Oresol administration. Teach the mother to give Oresol
more slowly.
103. Answer: (D) Edema
Edema, a major sign of kwashiorkor, is caused by decreased colloidal osmotic pressure of
the blood brought about by hypoalbuminemia. Decreased blood albumin level is due a
protein-deficient diet.
104. Answer: (A) Refer the child urgently to a hospital for confinement.
Baggy pants is a sign of severe marasmus. The best management is urgent referral to a
hospital.
105. Answer: (D) Conjunctival xerosis
The earliest sign of Vitamin A deficiency (xerophthalmia) is night blindness. However, this is
a functional change, which is not observable during physical examination.The earliest
visible lesion is conjunctival xerosis or dullness of the conjunctiva due to inadequate tear
production.
106. Answer: (D) 200,000 IU
Preschoolers are given Retinol 200,000 IU every 6 months. 100,000 IU is given once to
infants aged 6 to 12 months. The dose for pregnant women is 10,000 IU.

107. Answer: (A) Palms


The anatomic characteristics of the palms allow a reliable and convenient basis for
examination for pallor.
108. Answer: (A) Sugar
R.A. 8976 mandates fortification of rice, wheat flour, sugar and cooking oil with Vitamin A,
iron and/or iodine.
109. Answer: (A) Give measles vaccine to babies aged 6 to 8 months.
Ordinarily, measles vaccine is given at 9 months of age. During an impending epidemic,
however, one dose may be given to babies aged 6 to 8 months. The mother is instructed
that the baby needs another dose when the baby is 9 months old.
110. Answer: (A) Inability to drink
A sick child aged 2 months to 5 years must be referred urgently to a hospital if he/she has
one or more of the following signs: not able to feed or drink, vomits everything, convulsions,
abnormally sleepy or difficult to awaken.
111. Answer: (D) Retinol capsule regardless of when the last dose was given
An infant 6 to 12 months classified as a case of measles is given Retinol 100,000 IU; a child
is given 200,000 IU regardless of when the last dose was given.
112. Answer: (B) Ask where the family resides.
Because malaria is endemic, the first question to determine malaria risk is where the clients
family resides. If the area of residence is not a known endemic area, ask if the child had
traveled within the past 6 months, where he/she was brought and whether he/she stayed
overnight in that area.
113. Answer: (B) Destroying breeding places of mosquitoes
Aedes aegypti, the vector of Dengue fever, breeds in stagnant, clear water. Its feeding time
is usually during the daytime. It has a cyclical pattern of occurrence, unlike malaria which is
endemic in certain parts of the country.
114. Answer: (C) Determining whether a place is endemic or not
This is diagnostic and therefore secondary level prevention. The other choices are for
primary prevention.
115. Answer: (B) Pinworm
Pinworm ova are deposited around the anal orifice.

116. Answer: (C) Cough for 3 weeks


A client is considered a PTB suspect when he has cough for 2 weeks or more, plus one or
more of the following signs: fever for 1 month or more; chest pain lasting for 2 weeks or
more not attributed to other conditions; progressive, unexplained weight loss; night sweats;
and hemoptysis.
117. Answer: (D) Clients diagnosed for the first time through a positive sputum exam
Category I is for new clients diagnosed by sputum examination and clients diagnosed to
have a serious form of extrapulmonary tuberculosis, such as TB osteomyelitis.
118. Answer: (B) Having the health worker or a responsible family member monitor drug
intake
Directly Observed Treatment Short Course is so-called because a treatment partner,
preferably a health worker accessible to the client, monitors the clients compliance to the
treatment.
119. Answer: (C) Thickened painful nerves
The lesion of leprosy is not macular. It is characterized by a change in skin color (either
reddish or whitish) and loss of sensation, sweating and hair growth over the lesion. Inability
to close the eyelids (lagophthalmos) and sinking of the nosebridge are late symptoms.
120. Answer: (D) 5 skin lesions, positive slit skin smear
A multibacillary leprosy case is one who has a positive slit skin smear and at least 5 skin
lesions.
121. Answer: (B) Liver cirrhosis
The etiologic agent of schistosomiasis in the Philippines is Schistosoma japonicum, which
affects the small intestine and the liver. Liver damage is a consequence of fibrotic reactions
to schistosoma eggs in the liver.
122. Answer: (C) Proper use of sanitary toilets
The ova of the parasite get out of the human body together with feces. Cutting the cycle at
this stage is the most effective way of preventing the spread of the disease to susceptible
hosts.
123. Answer: (B) II
A communal faucet or water standpost is classified as Level II.

124. Answer: (A) Use of sterile syringes and needles


Hepatitis A is transmitted through the fecal oral route. Hepatitis B is transmitted through
infected body secretions like blood and semen.
125. Answer: (A) DPT
DPT is sensitive to freezing. The appropriate storage temperature of DPT is 2 to 8 C only.
OPV and measles vaccine are highly sensitive to heat and require freezing. MMR is not an
immunization in the Expanded Program on Immunization.
126. Answer: (A) 45
To estimate the number of infants, multiply total population by 3%.
127. Answer: (B) Severe dehydration
The order of priority in the management of severe dehydration is as follows: intravenous
fluid therapy, referral to a facility where IV fluids can be initiated within 30 minutes,
Oresol/nasogastric tube, Oresol/orem. When the foregoing measures are not possible or
effective, tehn urgent referral to the hospital is done.
128. Answer: (A) 3
Adequate blood supply to the area allows the return of the color of the nailbed within 3
seconds.
129. Answer: (B) Instruct the mother to give the child Oresol.
Since the child does not manifest any other danger sign, maintenance of fluid balance and
replacement of fluid loss may be done by giving the client Oresol.
130. Answer: (B) Buccal mucosa
Kopliks spot may be seen on the mucosa of the mouth or the throat.
131. Answer: (D) Measles
Viral conjunctivitis is transmitted by direct or indirect contact with discharges from infected
eyes. Acute poliomyelitis is spread through the fecal-oral route and contact with throat
secretions, whereas diphtheria is through direct and indirect contact with respiratory
secretions.
132. Answer: (A) Hemophilus influenzae
Hemophilus meningitis is unusual over the age of 5 years. In developing countries, the peak
incidence is in children less than 6 months of age. Morbillivirus is the etiology of measles.
Streptococcus pneumoniae and Neisseria meningitidis may cause meningitis, but age
distribution is not specific in young children.

133. Answer: (D) Zooprophylaxis


Zooprophylaxis is done by putting animals like cattle or dogs close to windows or doorways
just before nightfall. The Anopheles mosquito takes his blood meal from the animal and
goes back to its breeding place, thereby preventing infection of humans.
134. Answer: (A) Stream seeding
Stream seeding is done by putting tilapia fry in streams or other bodies of water identified as
breeding places of the Anopheles mosquito.
135. Answer: (C) Destruction of breeding places of the mosquito vector
Anopheles mosquitoes breed in slow-moving, clear water, such as mountain streams.
136. Answer: (B) Cholera
Passage of profuse watery stools is the major symptom of cholera. Both amebic and
bacillary dysentery are characterized by the presence of blood and/or mucus in the stools.
Giardiasis is characterized by fat malabsorption and, therefore, steatorrhea.
137. Answer: (B) S. japonicum
S. mansoni is found mostly in Africa and South America; S. haematobium in Africa and the
Middle East; and S. malayensis only in peninsular Malaysia.
138. Answer: (D) Leptospirosis
Leptospirosis is transmitted through contact with the skin or mucous membrane with water
or moist soil contaminated with urine of infected animals, like rats.
139. Answer: (C) III
Waterworks systems, such as MWSS, are classified as level III.
140. Answer: (D) Refer the client for a supplementary test, such as Western blot, since the
ELISA result may be false.
A client having a reactive ELISA result must undergo a more specific test, such as Western
blot. A negative supplementary test result means that the ELISA result was false and that,
most probably, the client is not infected.
141. Answer: (A) Being faithful to a single sexual partner
Sexual fidelity rules out the possibility of getting the disease by sexual contact with another
infected person. Transmission occurs mostly through sexual intercourse and exposure to
blood or tissues.

142. Answer: (B) Infectious mononucleosis


Cytomegalovirus disease is an acute viral disease characterized by fever, sore throat and
lymphadenopathy.
143. Answer: (A) Contact tracing
Contact tracing is the most practical and reliable method of finding possible sources of
person-to-person transmitted infections, such as sexually transmitted diseases.
144. Answer: (D) They are able to bring about a cure of the disease condition.
There is no known treatment for AIDS. Antiretroviral agents reduce the risk of opportunistic
infections and prolong life, but does not cure the underlying immunodeficiency.
145. Answer: (D) Consult a physician who may give them rubella immunoglobulin.
Rubella vaccine is made up of attenuated German measles viruses. This is contraindicated
in pregnancy. Immune globulin, a specific prophylactic against German measles, may be
given to pregnant women.

1. In which step are plans formulated for solving community problems?


A.
B.
C.
D.

Mobilization
Community organization
Follow-up/extension
Core group formation
2. The public health nurse takes an active role in community participation.
What is the primary goal of community organizing?

A.
B.
C.

To educate the people regarding community health problems


To mobilize the people to resolve community health problems
To maximize the communitys resources in dealing with health
problems
D.
To maximize the communitys resources in dealing with health
problems
3. An indicator of success in community organizing is when people are able to
A.

Participate in community activities for the solution of a


community problem
B.
Implement activities for the solution of the community problem
C.
Plan activities for the solution of the community problem
D.
Identify the health problem as a common concern
4. Tertiary prevention is needed in which stage of the natural history of
disease?

A.
B.
C.
D.

Pre-pathogenesis
Pathogenesis
Prodromal
Terminal
5. Isolation of a child with measles belongs to what level of prevention?

A.
B.
C.
D.

Primary
Secondary
Intermediate
Tertiary
6. On the other hand, Operation Timbang is _____ prevention.

A.
B.
C.
D.

Primary
Secondary
Intermediate
Tertiary
7. Which type of family-nurse contact will provide you with the best opportunity
to observe family dynamics?

A.
B.
C.
D.

Clinic consultation
Group conference
Home visit
Written communication
8. The typology of family nursing problems is used in the statement of nursing
diagnosis in the care of families. The youngest child of the de los Reyes family
has been diagnosed as mentally retarded. This is classified as a:

A.
B.
C.
D.

Health threat
Health deficit
Foreseeable crisis
Stress point
9. The de los Reyes couple have a 6-year old child entering school for the first
time. The de los Reyes family has a:

A.
B.
C.
D.

Health threat
Health deficit
Foreseeable crisis
Stress point
10. Which of the following is an advantage of a home visit?

A.

It allows the nurse to provide nursing care to a greater number


of people.

B.

It provides an opportunity to do first hand appraisal of the


home situation.
C.
It allows sharing of experiences among people with similar
health problems.
D.
It develops the familys initiative in providing for health needs
of its members.
11. Which is CONTRARY to the principles in planning a home visit?
A.
B.
C.

A home visit should have a purpose or objective.


The plan should revolve around family health needs.
A home visit should be conducted in the manner prescribed by
the RHU.
D.
Planning of continuing care should involve a responsible family
member.
12. The PHN bag is an important tool in providing nursing care during a home
visit. The most important principle of bag technique states that it
A.
B.
C.
D.

Should save time and effort.


Should minimize if not totally prevent the spread of infection.
Should not overshadow concern for the patient and his family.
May be done in a variety of ways depending on the home
situation, etc.
13. To maintain the cleanliness of the bag and its contents, which of the
following must the nurse do?

A.

Wash his/her hands before and after providing nursing care to


the family members.
B.
In the care of family members, as much as possible, use only
articles taken from the bag.
C.
Put on an apron to protect her uniform and fold it with the right
side out before putting it back into the bag.
D.
At the end of the visit, fold the lining on which the bag was
placed, ensuring that the contaminated side is on the outside.
14. The public health nurse conducts a study on the factors contributing to the
high mortality rate due to heart disease in the municipality where she works.
Which branch of epidemiology does the nurse practice in this situation?
A.
B.
C.
D.

Descriptive
Analytical
Therapeutic
Evaluation
15. Which of the following is a function of epidemiology?

A.

Identifying the disease condition based on manifestations


presented by a client
B.
Determining factors that contributed to the occurrence of
pneumonia in a 3 year old
C.
Determining the efficacy of the antibiotic used in the treatment
of the 3 year old client with pneumonia
D.
Evaluating the effectiveness of the implementation of the
Integrated Management of Childhood Illness
16. Which of the following is an epidemiologic function of the nurse during an
epidemic?
A.

Conducting assessment of suspected cases to detect the


communicable disease
B.
Monitoring the condition of the cases affected by the
communicable disease
C.
Participating in the investigation to determine the source of the
epidemic
D.
Teaching the community on preventive measures against the
disease
17. The primary purpose of conducting an epidemiologic investigation is to
A.
B.
C.
D.

Delineate the etiology of the epidemic


Encourage cooperation and support of the community
Identify groups who are at risk of contracting the disease
Identify geographical location of cases of the disease in the
community
18. Which is a characteristic of person-to-person propagated epidemics?

A.
B.
C.

There are more cases of the disease than expected.


The disease must necessarily be transmitted through a vector.
The spread of the disease can be attributed to a common
vehicle.
D.
There is a gradual build up of cases before the epidemic
becomes easily noticeable.
19. In the investigation of an epidemic, you compare the present frequency of
the disease with the usual frequency at this time of the year in this community.
This is done during which stage of the investigation?
A.
B.
C.
D.

Establishing the epidemic


Testing the hypothesis
Formulation of the hypothesis
Appraisal of facts

20. The number of cases of Dengue fever usually increases towards the end
of the rainy season. This pattern of occurrence of Dengue fever is best
described as
A.
B.
C.
D.

Epidemic occurrence
Cyclical variation
Sporadic occurrence
Secular variation
21. In the year 1980, the World Health Organization declared the Philippines,
together with some other countries in the Western Pacific Region, free of
which disease?

A.
B.
C.
D.

Pneumonic plague
Poliomyelitis
Small pox
Anthrax
22. In the census of the Philippines in 1995, there were about 35,299,000
males and about 34,968,000 females. What is the sex ratio?

A.
B.
C.
D.

99.06:100
100.94:100
50.23%
49.76%
23. Primary health care is a total approach to community development. Which
of the following is an indicator of success in the use of the primary health care
approach?

A.

Health services are provided free of charge to individuals and


families.
B.
Local officials are empowered as the major decision makers in
matters of health.
C.
Health workers are able to provide care based on identified
health needs of the people.
D.
Health programs are sustained according to the level of
development of the community.
24. Sputum examination is the major screening tool for pulmonary
tuberculosis. Clients would sometimes get false negative results in this exam.
This means that the test is not perfect in terms of which characteristic of a
diagnostic examination?
A.
B.

Effectiveness
Efficacy

C.
D.

Specificity
Sensitivity
25. Use of appropriate technology requires knowledge of indigenous
technology. Which medicinal herb is given for fever, headache and cough?

A.
B.
C.
D.

Sambong
Tsaang gubat
Akapulko
Lagundi
26. What law created the Philippine Institute of Traditional and Alternative
Health Care?

A.
B.
C.
D.

R.A. 8423
R.A. 4823
R.A. 2483
R.A. 3482
27. In traditional Chinese medicine, the yielding, negative and feminine force
is termed

A.
B.
C.
D.

Yin
Yang
Qi
Chai
28. What is the legal basis for Primary Health Care approach in the
Philippines?

A.
B.
C.
D.

Alma Ata Declaration on PHC


Letter of Instruction No. 949
Presidential Decree No. 147
Presidential Decree 996
29. Which of the following demonstrates intersectoral linkages?

A.
B.
C.
D.

Two-way referral system


Team approach
Endorsement done by a midwife to another midwife
Cooperation between the PHN and public school teacher
30. The municipality assigned to you has a population of about 20,000.
Estimate the number of 1-4 year old children who will be given Retinol capsule
200,000 I.U. every 6 months.

A.
B.
C.

1,500
1,800
2,000

D.

2,300

Answers and Rationales


1.

Answer: (B) Community organization. Community


organization is the step when community assemblies take place.
During the community assembly, the people may opt to formalize
the community organization and make plans for community action
to resolve a community health problem.
2.
Answer: (D) To maximize the communitys resources in
dealing with health problems. Community organizing is a
developmental service, with the goal of developing the peoples selfreliance in dealing with community health problems. A, B and C are
objectives of contributory objectives to this goal.
3.
Answer: (A) Participate in community activities for the
solution of a community problem. Participation in community
activities in resolving a community problem may be in any of the
processes mentioned in the other choices.
4.
Answer: (D) Terminal. Tertiary prevention involves
rehabilitation, prevention of permanent disability and disability
limitation appropriate for convalescents, the disabled, complicated
cases and the terminally ill (those in the terminal stage of a disease)
5.
Answer: (A) Primary. The purpose of isolating a client with a
communicable disease is to protect those who are not sick (specific
disease prevention).
6.
Answer: (B) Secondary. Operation Timbang is done to
identify members of the susceptible population who are
malnourished. Its purpose is early diagnosis and, subsequently,
prompt treatment.
7.
Answer: (C) Home visit. Dynamics of family relationships
can best be observed in the familys natural environment, which is
the home.
8.
Answer: (B) Health deficit. Failure of a family member to
develop according to what is expected, as in mental retardation, is a
health deficit.
9.
Answer: (C) Foreseeable crisis. Entry of the 6-year old into
school is an anticipated period of unusual demand on the family.
10.
Answer: (B) It provides an opportunity to do first hand
appraisal of the home situation.. Choice A is not correct since a
home visit requires that the nurse spend so much time with the
family. Choice C is an advantage of a group conference, while choice
D is true of a clinic consultation.

11.
Answer: (C) A home visit should be conducted in the
manner prescribed by the RHU.The home visit plan should be
flexible and practical, depending on factors, such as the familys
needs and the resources available to the nurse and the family.
12.
Answer: (B) Should minimize if not totally prevent the
spread of infection. Bag technique is performed before and after
handling a client in the home to prevent transmission of infection to
and from the client.
13.
Answer: (A) Wash his/her hands before and after
providing nursing care to the family members. Choice B goes
against the idea of utilizing the familys resources, which is
encouraged in CHN. Choices C and D goes against the principle of
asepsis of confining the contaminated surface of objects.
14.
Answer: (B) Analytical. Analytical epidemiology is the study
of factors or determinants affecting the patterns of occurrence and
distribution of disease in a community.
15.
Answer: (D) Evaluating the effectiveness of the
implementation of the Integrated Management of Childhood
Illness. Epidemiology is used in the assessment of a community or
evaluation of interventions in community health practice.
16.
Answer: (C) Participating in the investigation to
determine the source of the epidemic. Epidemiology is the
study of patterns of occurrence and distribution of disease in the
community, as well as the factors that affect disease patterns. The
purpose of an epidemiologic investigation is to identify the source of
an epidemic, i.e., what brought about the epidemic.
17.
Answer: (A) Delineate the etiology of the
epidemic. Delineating the etiology of an epidemic is identifying its
source.
18.
Answer: (D) There is a gradual build up of cases before
the epidemic becomes easily noticeable. A gradual or insidious
onset of the epidemic is usually observable in person-to-person
propagated epidemics.
19.
Answer: (A) Establishing the epidemic. Establishing the
epidemic is determining whether there is an epidemic or not. This is
done by comparing the present number of cases with the usual
number of cases of the disease at the same time of the year, as well
as establishing the relatedness of the cases of the disease.
20.
Answer: (B) Cyclical variation. A cyclical variation is a
periodic fluctuation in the number of cases of a disease in the
community.

21.
Answer: (C) Small pox. The last documented case of Small
pox was in 1977 at Somalia.
22.
Answer: (B) 100.94:100. Sex ratio is the number of males
for every 100 females in the population.
23.
Answer: (D) Health programs are sustained according to
the level of development of the community. Primary health
care is essential health care that can be sustained in all stages of
development of the community.
24.
Answer: (D) Sensitivity. Sensitivity is the capacity of a
diagnostic examination to detect cases of the disease. If a test is
100% sensitive, all the cases tested will have a positive result, i.e.,
there will be no false negative results.
25.
Answer: (D) Lagundi. Sambong is used as a diuretic. Tsaang
gubat is used to relieve diarrhea. Akapulko is used for its antifungal
property.
26.
Answer: (A) R.A. 8423
27.
Answer: (A) Yin. Yang is the male dominating, positive and
masculine force.
28.
Answer: (B) Letter of Instruction No. 949. Letter of
Instruction 949 was issued by then President Ferdinand Marcos,
directing the formerly called Ministry of Health, now the Department
of Health, to utilize Primary Health Care approach in planning and
implementing health programs.
29.
Answer: (D) Cooperation between the PHN and public
school teacher. Intersectoral linkages refer to working
relationships between the health sector and other sectors involved
in community development.
30.
Answer: (D) 2,300. Based on the Philippine population
composition, to estimate the number of 1-4 year old children,
multiply total population by 11.5%.
1. Estimate the number of pregnant women who will be given tetanus toxoid
during an immunization outreach activity in a barangay with a population of
about 1,500.
A.
B.
C.
D.

265
300
375
400
2. To describe the sex composition of the population, which demographic tool
may be used?

A.
B.
C.
D.

Sex ratio
Sex proportion
Population pyramid
Any of these may be used.
3. Which of the following is a natality rate?

A.
B.
C.
D.

Crude birth rate


Neonatal mortality rate
Infant mortality rate
General fertility rate
4. You are computing the crude death rate of your municipality, with a total
population of about 18,000, for last year. There were 94 deaths. Among those
who died, 20 died because of diseases of the heart and 32 were aged 50
years or older. What is the crude death rate?

A.
B.
C.
D.

4.2/1,000
5.2/1,000
6.3/1,000
7.3/1,000
5. Knowing that malnutrition is a frequent community health problem, you
decided to conduct nutritional assessment. What population is particularly
susceptible to protein energy malnutrition (PEM)?

A.
B.
C.
D.

Pregnant women and the elderly


Under-5 year old children
1-4 year old children
School age children
6. Which statistic can give the most accurate reflection of the health status of
a community?

A.
B.
C.
D.

1-4 year old age-specific mortality rate


Infant mortality rate
Swaroops index
Crude death rate
7. In the past year, Barangay A had an average population of 1655. 46 babies
were born in that year, 2 of whom died less than 4 weeks after they were born.
There were 4 recorded stillbirths. What is the neonatal mortality rate?

A.
B.
C.
D.

27.8/1,000
43.5/1,000
86.9/1,000
130.4/1,000

8. Which statistic best reflects the nutritional status of a population?


A.
B.
C.
D.

1-4 year old age-specific mortality rate


Proportionate mortality rate
Infant mortality rate
Swaroops index
9. What numerator is used in computing general fertility rate?

A.
B.
C.
D.

Estimated midyear population


Number of registered live births
Number of pregnancies in the year
Number of females of reproductive age
10. You will gather data for nutritional assessment of a purok. You will gather
information only from families with members who belong to the target
population for PEM. What method of data gathering is best for this purpose?

A.
B.
C.
D.

Census
Survey
Record review
Review of civil registry
11. In the conduct of a census, the method of population assignment based on
the actual physical location of the people is termed

A.
B.
C.
D.

De jure
De locus
De facto
De novo
12. The Field Health Services and Information System (FHSIS) is the
recording and reporting system in public health care in the Philippines. The
Monthly Field Health Service Activity Report is a form used in which of the
components of the FHSIS?

A.
B.
C.
D.

Tally report
Output report
Target/client list
Individual health record
13. To monitor clients registered in long-term regimens, such as the Multi-Drug
Therapy, which component will be most useful?

A.
B.
C.
D.

Tally report
Output report
Target/client list
Individual health record

14. Civil registries are important sources of data. Which law requires
registration of births within 30 days from the occurrence of the birth?
A.
B.
C.
D.

P.D. 651
Act 3573
R.A. 3753
R.A. 3375
15. Which of the following professionals can sign the birth certificate?

A.
B.
C.
D.

Public health nurse


Rural health midwife
Municipal health officer
Any of these health professionals
16. Which criterion in priority setting of health problems is used only in
community health care?

A.
B.
C.
D.

Modifiability of the problem


Nature of the problem presented
Magnitude of the health problem
Preventive potential of the health problem
17. The Sentrong Sigla Movement has been launched to improve health
service delivery. Which of the following is/are true of this movement?

A.
B.
C.

This is a project spearheaded by local government units.


It is a basis for increasing funding from local government units.
It encourages health centers to focus on disease prevention
and control.
D.
Its main strategy is certification of health centers able to
comply with standards.
18. Which of the following women should be considered as special targets for
family planning?
A.
B.
C.
D.

Those who have two children or more


Those with medical conditions such as anemia
Those younger than 20 years and older than 35 years
Those who just had a delivery within the past 15 months
19. Freedom of choice is one of the policies of the Family Planning Program
of the Philippines. Which of the following illustrates this principle?

A.
B.

Information dissemination about the need for family planning


Support of research and development in family planning
methods

C.

Adequate information for couples regarding the different


methods
D.
Encouragement of couples to take family planning as a joint
responsibility
20. A woman, 6 months pregnant, came to the center for consultation. Which
of the following substances is contraindicated?
A.
B.
C.
D.

Tetanus toxoid
Retinol 200,000 IU
Ferrous sulfate 200 mg
Potassium iodate 200 mg. capsule
21. During prenatal consultation, a client asked you if she can have her
delivery at home. After history taking and physical examination, you advised
her against a home delivery. Which of the following findings disqualifies her for
a home delivery?

A.
B.
C.
D.

Her OB score is G5P3.


She has some palmar pallor.
Her blood pressure is 130/80.
Her baby is in cephalic presentation.
22. Inadequate intake by the pregnant woman of which vitamin may cause
neural tube defects?

A.
B.
C.
D.

Niacin
Riboflavin
Folic acid
Thiamine
23. You are in a clients home to attend to a delivery. Which of the following
will you do first?

A.
B.
C.
D.

Set up the sterile area.


Put on a clean gown or apron.
Cleanse the clients vulva with soap and water.
Note the interval, duration and intensity of labor contractions.
24. In preparing a primigravida for breastfeeding, which of the following will
you do?

A.
B.
C.

Tell her that lactation begins within a day after delivery.


Teach her nipple stretching exercises if her nipples are everted.
Instruct her to wash her nipples before and after each
breastfeeding.

D.

Explain to her that putting the baby to breast will lessen blood
loss after delivery.
25. A primigravida is instructed to offer her breast to the baby for the first time
within 30 minutes after delivery. What is the purpose of offering the breast this
early?

A.
B.
C.
D.

To initiate the occurrence of milk letdown


To stimulate milk production by the mammary acini
To make sure that the baby is able to get the colostrum
To allow the woman to practice breastfeeding in the presence
of the health worker
26. In a mothers class, you discuss proper breastfeeding technique. Which is
of these is a sign that the baby has latched on to the breast properly?

A.
B.
C.
D.

The baby takes shallow, rapid sucks.


The mother does not feel nipple pain.
The babys mouth is only partly open.
Only the mothers nipple is inside the babys mouth.
27. You explain to a breastfeeding mother that breast milk is sufficient for all of
the babys nutrient needs only up to ____.

A.
B.
C.
D.

3 months
6 months
1 year
2 years
28. What is given to a woman within a month after the delivery of a baby?

A.
B.
C.
D.

Malunggay capsule
Ferrous sulfate 100 mg. OD
Retinol 200,000 I.U., 1 capsule
Potassium iodate 200 mg, 1 capsule
29. Which biological used in Expanded Program on Immunization (EPI) is
stored in the freezer?

A.
B.
C.
D.

DPT
Tetanus toxoid
Measles vaccine
Hepatitis B vaccine
30. Unused BCG should be discarded how many hours after reconstitution?

A.
B.

2
4

C.
D.

6
At the end of the day

Answers and Rationales


1.

Answer: (A) 265. To estimate the number of pregnant


women, multiply the total population by 3.5%.
2.
Answer: (D) Any of these may be used. Sex ratio and sex
proportion are used to determine the sex composition of a
population. A population pyramid is used to present the composition
of a population by age and sex.
3.
Answer: (A) Crude birth rate. Natality means birth. A
natality rate is a birth rate.
4.
Answer: (B) 5.2/1,000. To compute crude death rate divide
total number of deaths (94) by total population (18,000) and
multiply by 1,000.
5.
Answer: (C) 1-4 year old children. Preschoolers are the
most susceptible to PEM because they have generally been weaned.
Also, this is the population who, unable to feed themselves, are
often the victims of poor intrafamilial food distribution.
6.
Answer: (C) Swaroops index. Swaroops index is the
proportion of deaths aged 50 years and above. The higher the
Swaroops index of a population, the greater the proportion of the
deaths who were able to reach the age of at least 50 years, i.e.,
more people grew old before they died.
7.
Answer: (B) 43.5/1,000. To compute for neonatal mortality
rate, divide the number of babies who died before reaching the age
of 28 days by the total number of live births, then multiply by 1,000.
8.
Answer: (A) 1-4 year old age-specific mortality
rate. Since preschoolers are the most susceptible to the effects of
malnutrition, a population with poor nutritional status will most likely
have a high 1-4 year old age-specific mortality rate, also known as
child mortality rate.
9.
Answer: (B) Number of registered live births. To compute
for general or total fertility rate, divide the number of registered live
births by the number of females of reproductive age (15-45 years),
then multiply by 1,000.
10.
Answer: (B) Survey. A survey, also called sample survey, is
data gathering about a sample of the population.
11.
Answer: (C) De facto. The other method of population
assignment, de jure, is based on the usual place of residence of the
people.

12.
Answer: (A) Tally report. A tally report is prepared monthly
or quarterly by the RHU personnel and transmitted to the Provincial
Health Office.
13.
Answer: (C) Target/client list. The MDT Client List is a
record of clients enrolled in MDT and other relevant data, such as
dates when clients collected their monthly supply of drugs.
14.
Answer: (A) P.D. 651. P.D. 651 amended R.A. 3753, requiring
the registry of births within 30 days from their occurrence.
15.
Answer: (D) Any of these health professionals. D. R.A.
3753 states that any birth attendant may sign the certificate of live
birth.
16.
Answer: (C) Magnitude of the health problem. Magnitude
of the problem refers to the percentage of the population affected
by a health problem. The other choices are criteria considered in
both family and community health care.
17.
Answer: (D) Its main strategy is certification of health
centers able to comply with standards. Sentrong Sigla
Movement is a joint project of the DOH and local government units.
Its main strategy is certification of health centers that are able to
comply with standards set by the DOH.
18.
Answer: (D) Those who just had a delivery within the
past 15 months. The ideal birth spacing is at least two years. 15
months plus 9 months of pregnancy = 2 years.
19.
Answer: (C) Adequate information for couples regarding
the different methods. To enable the couple to choose freely
among different methods of family planning, they must be given full
information regarding the different methods that are available to
them, considering the availability of quality services that can
support their choice.
20.
Answer: (B) Retinol 200,000 IU. Retinol 200,000 IU is a
form of megadose Vitamin A. This may have a teratogenic effect.
21.
Answer: (A) Her OB score is G5P3. Only women with less
than 5 pregnancies are qualified for a home delivery. It is also
advisable for a primigravida to have delivery at a childbirth facility.
22.
Answer: (C) Folic acid. It is estimated that the incidence of
neural tube defects can be reduced drastically if pregnant women
have an adequate intake of folic acid.
23.
Answer: (D) Note the interval, duration and intensity of
labor contractions.. Assessment of the woman should be done
first to determine whether she is having true labor and, if so, what
stage of labor she is in.

24.
Answer: (D) Explain to her that putting the baby to
breast will lessen blood loss after delivery. Suckling of the
nipple stimulates the release of oxytocin by the posterior pituitary
gland, which causes uterine contraction. Lactation begins 1 to 3
days after delivery. Nipple stretching exercises are done when the
nipples are flat or inverted. Frequent washing dries up the nipples,
making them prone to the formation of fissures.
25.
Answer: (B) To stimulate milk production by the
mammary acini. Suckling of the nipple stimulates prolactin reflex
(the release of prolactin by the anterior pituitary gland), which
initiates lactation.
26.
Answer: (B) The mother does not feel nipple pain.. When
the baby has properly latched on to the breast, he takes deep, slow
sucks; his mouth is wide open; and much of the areola is inside his
mouth. And, youre right! The mother does not feel nipple pain.
27.
Answer: (B) 6 months. After 6 months, the babys nutrient
needs, especially the babys iron requirement, can no longer be
provided by mothers milk alone.
28.
Answer: (C) Retinol 200,000 I.U., 1 capsule. A capsule of
Retinol 200,000 IU is given within 1 month after delivery. Potassium
iodate is given during pregnancy; malunggay capsule is not
routinely administered after delivery; and ferrous sulfate is taken for
two months after delivery.
29.
Answer: (C) Measles vaccine. Among the biologicals used in
the Expanded Program on Immunization, measles vaccine and OPV
are highly sensitive to heat, requiring storage in the freezer.
30.
Answer: (B) 4. While the unused portion of other biologicals
in EPI may be given until the end of the day, only BCG is discarded 4
hours after reconstitution. This is why BCG immunization is
scheduled only in the morning.
1. In immunizing school entrants with BCG, you are not obliged to secure
parental consent. This is because of which legal document?
A.
B.
C.
D.
A.

P.D. 996
R.A. 7846
Presidential Proclamation No. 6
Presidential Proclamation No. 46
2. Which immunization produces a permanent scar?
DPT

B.
C.
D.

BCG
Measles vaccination
Hepatitis B vaccination
3. A 4-week old baby was brought to the health center for his first
immunization. Which can be given to him?

A.
B.
C.
D.

DPT1
OPV1
Infant BCG
Hepatitis B vaccine 1
4. You will not give DPT 2 if the mother says that the infant had

A.
B.
C.
D.

Seizures a day after DPT 1.


Fever for 3 days after DPT 1.
Abscess formation after DPT 1.
Local tenderness for 3 days after DPT 1.
5. A 2-month old infant was brought to the health center for immunization.
During assessment, the infants temperature registered at 38.1C. Which is
the best course of action that you will take?

A.
B.
C.
D.

Go on with the infants immunizations.


Give Paracetamol and wait for his fever to subside.
Refer the infant to the physician for further assessment.
Advise the infants mother to bring him back for immunization
when he is well.
6. A pregnant woman had just received her 4th dose of tetanus toxoid.
Subsequently, her baby will have protection against tetanus for how long?

A.
B.
C.
D.

1 year
3 years
10 years
Lifetime
7. A 4-month old infant was brought to the health center because of cough.
Her respiratory rate is 42/minute. Using the Integrated Management of Child
Illness (IMCI) guidelines of assessment, her breathing is considered

A.
B.
C.
D.

Fast
Slow
Normal
Insignificant
8. Which of the following signs will indicate that a young child is suffering from
severe pneumonia?

A.
B.
C.
D.

Dyspnea
Wheezing
Fast breathing
Chest indrawing
9. Using IMCI guidelines, you classify a child as having severe pneumonia.
What is the best management for the child?

A.
B.
C.
D.

Prescribe an antibiotic.
Refer him urgently to the hospital.
Instruct the mother to increase fluid intake.
Instruct the mother to continue breastfeeding.
10. A 5-month old infant was brought by his mother to the health center
because of diarrhea occurring 4 to 5 times a day. His skin goes back slowly
after a skin pinch and his eyes are sunken. Using the IMCI guidelines, you will
classify this infant in which category?

A.
B.
C.
D.

No signs of dehydration
Some dehydration
Severe dehydration
The data is insufficient.
11. Based on assessment, you classified a 3-month old infant with the chief
complaint of diarrhea in the category of SOME DEHYDRATION. Based on
IMCI management guidelines, which of the following will you do?

A.

Bring the infant to the nearest facility where IV fluids can be


given.
B.
Supervise the mother in giving 200 to 400 ml. of Oresol in 4
hours.
C.
Give the infants mother instructions on home management.
D.
Keep the infant in your health center for close observation.
12. A mother is using Oresol in the management of diarrhea of her 3-year old
child. She asked you what to do if her child vomits. You will tell her to
A.
B.

Bring the child to the nearest hospital for further assessment.


Bring the child to the health center for intravenous fluid
therapy.
C.
Bring the child to the health center for assessment by the
physician.
D.
Let the child rest for 10 minutes then continue giving Oresol
more slowly.

13. A 1 year old child was classified as having 3rd degree protein energy
malnutrition, kwashiorkor. Which of the following signs will be most apparent
in this child?
A.
B.
C.
D.

Voracious appetite
Wasting
Apathy
Edema
14. Assessment of a 2-year old child revealed baggy pants. Using the IMCI
guidelines, how will you manage this child?

A.
B.

Refer the child urgently to a hospital for confinement.


Coordinate with the social worker to enroll the child in a
feeding program.
C.
Make a teaching plan for the mother, focusing on menu
planning for her child.
D.
Assess and treat the child for health problems like infections
and intestinal parasitism.
15. During the physical examination of a young child, what is the earliest sign
of xerophthalmia that you may observe?
A.
B.
C.
D.

Keratomalacia
Corneal opacity
Night blindness
Conjunctival xerosis
16. To prevent xerophthalmia, young children are given Retinol capsule every
6 months. What is the dose given to preschoolers?

A.
B.
C.
D.

10,000 IU
20,000 IU
100,000 IU
200,000 IU
17. The major sign of iron deficiency anemia is pallor. What part is best
examined for pallor?

A.
B.
C.
D.

Palms
Nailbeds
Around the lips
Lower conjunctival sac
18. Food fortification is one of the strategies to prevent micronutrient
deficiency conditions. R.A. 8976 mandates fortification of certain food items.
Which of the following is among these food items?

A.
B.
C.
D.

Sugar
Bread
Margarine
Filled milk
19. What is the best course of action when there is a measles epidemic in a
nearby municipality?

A.
B.

Give measles vaccine to babies aged 6 to 8 months.


Give babies aged 6 to 11 months one dose of 100,000 I.U. of
Retinol
C.
Instruct mothers to keep their babies at home to prevent
disease transmission.
D.
Instruct mothers to feed their babies adequately to enhance
their babies resistance.
20. A mother brought her daughter, 4 years old, to the RHU because of cough
and colds. Following the IMCI assessment guide, which of the following is a
danger sign that indicates the need for urgent referral to a hospital?
A.
B.
C.
D.

Inability to drink
High grade fever
Signs of severe dehydration
Cough for more than 30 days
21. Management of a child with measles includes the administration of which
of the following?

A.
B.
C.
D.

Gentian violet on mouth lesions


Antibiotics to prevent pneumonia
Tetracycline eye ointment for corneal opacity
Retinol capsule regardless of when the last dose was given
22. A mother brought her 10 month old infant for consultation because of
fever, which started 4 days prior to consultation. To determine malaria risk,
what will you do?

A.
B.
C.
D.

Do a tourniquet test.
Ask where the family resides.
Get a specimen for blood smear.
Ask if the fever is present everyday.
23. The following are strategies implemented by the Department of Health to
prevent mosquito-borne diseases. Which of these is most effective in the
control of Dengue fever?

A.

Stream seeding with larva-eating fish

B.
C.

Destroying breeding places of mosquitoes


Chemoprophylaxis of non-immune persons going to endemic
areas
D.
Teaching people in endemic areas to use chemically treated
mosquito nets
24. Secondary prevention for malaria includes
A.
B.
C.
D.

Planting of neem or eucalyptus trees


Residual spraying of insecticides at night
Determining whether a place is endemic or not
Growing larva-eating fish in mosquito breeding places
25. Scotch tape swab is done to check for which intestinal parasite?

A.
B.
C.
D.

Ascaris
Pinworm
Hookworm
Schistosoma
26. Which of the following signs indicates the need for sputum examination for
AFB?

A.
B.
C.
D.

Hematemesis
Fever for 1 week
Cough for 3 weeks
Chest pain for 1 week
27. Which clients are considered targets for DOTS Category I?

A.
B.
C.
D.

Sputum negative cavitary cases


Clients returning after a default
Relapses and failures of previous PTB treatment regimens
Clients diagnosed for the first time through a positive sputum
exam
28. To improve compliance to treatment, what innovation is being
implemented in DOTS?

A.
B.

Having the health worker follow up the client at home


Having the health worker or a responsible family member
monitor drug intake
C.
Having the patient come to the health center every month to
get his medications
D.
Having a target list to check on whether the patient has
collected his monthly supply of drugs

29. Diagnosis of leprosy is highly dependent on recognition of symptoms.


Which of the following is an early sign of leprosy?
A.
B.
C.
D.

A.
B.
C.
D.

Macular lesions
Inability to close eyelids
Thickened painful nerves
Sinking of the nosebridge
30. Which of the following clients should be classified as a case of
multibacillary leprosy?
3
3
5
5

skin
skin
skin
skin

lesions,
lesions,
lesions,
lesions,

negative slit skin smear


positive slit skin smear
negative slit skin smear
positive slit skin smear

Answers and Rationales


1.

2.
3.
4.

5.

6.

7.
8.

Answer: (A) P.D. 996. Presidential Decree 996, enacted in


1976, made immunization in the EPI compulsory for children under 8
years of age. Hepatitis B vaccination was made compulsory for the
same age group by R.A. 7846.
Answer: (B) BCG. BCG causes the formation of a superficial
abscess, which begins 2 weeks after immunization. The abscess
heals without treatment, with the formation of a permanent scar.
Answer: (C) Infant BCG. Infant BCG may be given at birth.
All the other immunizations mentioned can be given at 6 weeks of
age.
Answer: (A) Seizures a day after DPT 1. Seizures within 3
days after administration of DPT is an indication of hypersensitivity
to pertussis vaccine, a component of DPT. This is considered a
specific contraindication to subsequent doses of DPT.
Answer: (A) Go on with the infants immunizations. In
the EPI, fever up to 38.5C is not a contraindication to immunization.
Mild acute respiratory tract infection, simple diarrhea and
malnutrition are not contraindications either.
Answer: (A) 1 year. The baby will have passive natural
immunity by placental transfer of antibodies. The mother will have
active artificial immunity lasting for about 10 years. 5 doses will give
the mother lifetime protection.
Answer: (C) Normal. In IMCI, a respiratory rate of 50/minute
or more is fast breathing for an infant aged 2 to 12 months.
Answer: (D) Chest indrawing. In IMCI, chest indrawing is
used as the positive sign of dyspnea, indicating severe pneumonia.

9.

Answer: (B) Refer him urgently to the hospital. Severe


pneumonia requires urgent referral to a hospital. Answers A, C and D
are done for a client classified as having pneumonia.
10.
Answer: (B) Some dehydration. Using the assessment
guidelines of IMCI, a child (2 months to 5 years old) with diarrhea is
classified as having SOME DEHYDRATION if he shows 2 or more of
the following signs: restless or irritable, sunken eyes, the skin goes
back slow after a skin pinch.
11.
Answer: (B) Supervise the mother in giving 200 to 400
ml. of Oresol in 4 hours. In the IMCI management guidelines,
SOME DEHYDRATION is treated with the administration of Oresol
within a period of 4 hours. The amount of Oresol is best computed
on the basis of the childs weight (75 ml/kg body weight). If the
weight is unknown, the amount of Oresol is based on the childs age.
12.
Answer: (D) Let the child rest for 10 minutes then
continue giving Oresol more slowly. If the child vomits
persistently, that is, he vomits everything that he takes in, he has to
be referred urgently to a hospital. Otherwise, vomiting is managed
by letting the child rest for 10 minutes and then continuing with
Oresol administration. Teach the mother to give Oresol more slowly.
13.
Answer: (D) Edema. Edema, a major sign of kwashiorkor, is
caused by decreased colloidal osmotic pressure of the blood brought
about by hypoalbuminemia. Decreased blood albumin level is due a
protein-deficient diet.
14.
Answer: (A) Refer the child urgently to a hospital for
confinement. Baggy pants is a sign of severe marasmus. The
best management is urgent referral to a hospital.
15.
Answer: (D) Conjunctival xerosis. The earliest sign of
Vitamin A deficiency (xerophthalmia) is night blindness. However,
this is a functional change, which is not observable during physical
examination.The earliest visible lesion is conjunctival xerosis or
dullness of the conjunctiva due to inadequate tear production.
16.
Answer: (D) 200,000 IU. Preschoolers are given Retinol
200,000 IU every 6 months. 100,000 IU is given once to infants aged
6 to 12 months. The dose for pregnant women is 10,000 IU.
17.
Answer: (A) Palms. The anatomic characteristics of the
palms allow a reliable and convenient basis for examination for
pallor.
18.
Answer: (A) Sugar. R.A. 8976 mandates fortification of rice,
wheat flour, sugar and cooking oil with Vitamin A, iron and/or iodine.

19.
Answer: (A) Give measles vaccine to babies aged 6 to 8
months. Ordinarily, measles vaccine is given at 9 months of age.
During an impending epidemic, however, one dose may be given to
babies aged 6 to 8 months. The mother is instructed that the baby
needs another dose when the baby is 9 months old.
20.
Answer: (A) Inability to drink. A sick child aged 2 months to
5 years must be referred urgently to a hospital if he/she has one or
more of the following signs: not able to feed or drink, vomits
everything, convulsions, abnormally sleepy or difficult to awaken.
21.
Answer: (D) Retinol capsule regardless of when the last
dose was given. An infant 6 to 12 months classified as a case of
measles is given Retinol 100,000 IU; a child is given 200,000 IU
regardless of when the last dose was given.
22.
Answer: (B) Ask where the family resides. Because
malaria is endemic, the first question to determine malaria risk is
where the clients family resides. If the area of residence is not a
known endemic area, ask if the child had traveled within the past 6
months, where he/she was brought and whether he/she stayed
overnight in that area.
23.
Answer: (B) Destroying breeding places of
mosquitoes. Aedes aegypti, the vector of Dengue fever, breeds in
stagnant, clear water. Its feeding time is usually during the daytime.
It has a cyclical pattern of occurrence, unlike malaria which is
endemic in certain parts of the country.
24.
Answer: (C) Determining whether a place is endemic or
not. This is diagnostic and therefore secondary level prevention.
The other choices are for primary prevention.
25.
Answer: (B) Pinworm. Pinworm ova are deposited around the
anal orifice.
26.
Answer: (C) Cough for 3 weeks. A client is considered a PTB
suspect when he has cough for 2 weeks or more, plus one or more
of the following signs: fever for 1 month or more; chest pain lasting
for 2 weeks or more not attributed to other conditions; progressive,
unexplained weight loss; night sweats; and hemoptysis.
27.
Answer: (D) Clients diagnosed for the first time through
a positive sputum exam. Category I is for new clients diagnosed
by sputum examination and clients diagnosed to have a serious
form of extrapulmonary tuberculosis, such as TB osteomyelitis.
28.
Answer: (B) Having the health worker or a responsible
family member monitor drug intake. Directly Observed
Treatment Short Course is so-called because a treatment partner,

preferably a health worker accessible to the client, monitors the


clients compliance to the treatment.
29.
Answer: (C) Thickened painful nerves. The lesion of
leprosy is not macular. It is characterized by a change in skin color
(either reddish or whitish) and loss of sensation, sweating and hair
growth over the lesion. Inability to close the eyelids (lagophthalmos)
and sinking of the nosebridge are late symptoms.
30.
Answer: (D) 5 skin lesions, positive slit skin smear. A
multibacillary leprosy case is one who has a positive slit skin smear
and at least 5 skin lesions.
1. In the Philippines, which condition is the most frequent cause of death
associated with schistosomiasis?
A.
B.
C.
D.

Liver cancer
Liver cirrhosis
Bladder cancer
Intestinal perforation
2. What is the most effective way of controlling schistosomiasis in an endemic
area?

A.
B.
C.
D.

Use of molluscicides
Building of foot bridges
Proper use of sanitary toilets
Use of protective footwear, such as rubber boots
3. When residents obtain water from an artesian well in the neighborhood, the
level of this approved type of water facility is

A.
B.
C.
D.

I
II
III
IV
4. For prevention of hepatitis A, you decided to conduct health education
activities. Which of the following is IRRELEVANT?

A.
B.
C.
D.

Use of sterile syringes and needles


Safe food preparation and food handling by vendors
Proper disposal of human excreta and personal hygiene
Immediate reporting of water pipe leaks and illegal water
connections
5. Which biological used in Expanded Program on Immunization (EPI) should
NOT be stored in the freezer?

A.
B.
C.
D.

DPT
Oral polio vaccine
Measles vaccine
MMR
6. You will conduct outreach immunization in a barangay with a population of
about 1500. Estimate the number of infants in the barangay.

A.
B.
C.
D.

45
50
55
60
7. In Integrated Management of Childhood Illness, severe conditions generally
require urgent referral to a hospital. Which of the following severe conditions
DOES NOT always require urgent referral to a hospital?

A.
B.
C.
D.

Mastoiditis
Severe dehydration
Severe pneumonia
Severe febrile disease
8. A client was diagnosed as having Dengue fever. You will say that there is
slow capillary refill when the color of the nailbed that you pressed does not
return within how many seconds?

A.
B.
C.
D.

3
5
8
10
9. A 3-year old child was brought by his mother to the health center because
of fever of 4-day duration. The child had a positive tourniquet test result. In the
absence of other signs, which is the most appropriate measure that the PHN
may carry out to prevent Dengue shock syndrome?

A.
B.
C.
D.

Insert an NGT and give fluids per NGT.


Instruct the mother to give the child Oresol.
Start the patient on intravenous fluids STAT.
Refer the client to the physician for appropriate management.
10. The pathognomonic sign of measles is Kopliks spot. You may see Kopliks
spot by inspecting the _____.

A.
B.
C.

Nasal mucosa
Buccal mucosa
Skin on the abdomen

D.

Skin on the antecubital surface


11. Among the following diseases, which is airborne?

A.
B.
C.
D.

Viral conjunctivitis
Acute poliomyelitis
Diphtheria
Measles
12. Among children aged 2 months to 3 years, the most prevalent form of
meningitis is caused by which microorganism?

A.
B.
C.
D.

Hemophilus influenzae
Morbillivirus
Steptococcus pneumoniae
Neisseria meningitidis
13. Human beings are the major reservoir of malaria. Which of the following
strategies in malaria control is based on this fact?

A.
B.
C.
D.

Stream seeding
Stream clearing
Destruction of breeding places
Zooprophylaxis
14. The use of larvivorous fish in malaria control is the basis for which strategy
of malaria control?

A.
B.
C.
D.

Stream seeding
Stream clearing
Destruction of breeding places
Zooprophylaxis
15. Mosquito-borne diseases are prevented mostly with the use of mosquito
control measures. Which of the following is NOT appropriate for malaria
control?

A.
B.
C.
D.

Use of chemically treated mosquito nets


Seeding of breeding places with larva-eating fish
Destruction of breeding places of the mosquito vector
Use of mosquito-repelling soaps, such as those with basil or
citronella
16. A 4-year old client was brought to the health center with the chief
complaint of severe diarrhea and the passage of rice water stools. The client
is most probably suffering from which condition?

A.
B.

Giardiasis
Cholera

C.
D.

Amebiasis
Dysentery
17. In the Philippines, which specie of schistosoma is endemic in certain
regions?

A.
B.
C.
D.

S. mansoni
S. japonicum
S. malayensis
S. haematobium
18. A 32-year old client came for consultation at the health center with the
chief complaint of fever for a week. Accompanying symptoms were muscle
pains and body malaise. A week after the start of fever, the client noted
yellowish discoloration of his sclera. History showed that he waded in flood
waters about 2 weeks before the onset of symptoms. Based on his history,
which disease condition will you suspect?

A.
B.
C.
D.

Hepatitis A
Hepatitis B
Tetanus
Leptospirosis
19. MWSS provides water to Manila and other cities in Metro Manila. This is
an example of which level of water facility?

A.
B.
C.
D.

I
II
III
IV
20. You are the PHN in the city health center. A client underwent screening for
AIDS using ELISA. His result was positive. What is the best course of action
that you may take?

A.

Get a thorough history of the client, focusing on the practice of


high risk behaviors.
B.
Ask the client to be accompanied by a significant person before
revealing the result.
C.
Refer the client to the physician since he is the best person to
reveal the result to the client.
D.
Refer the client for a supplementary test, such as Western blot,
since the ELISA result may be false.
21. Which is the BEST control measure for AIDS?
A.

Being faithful to a single sexual partner

B.
C.
D.

Using a condom during each sexual contact


Avoiding sexual contact with commercial sex workers
Making sure that ones sexual partner does not have signs of

AIDS
22. The most frequent causes of death among clients with AIDS are
opportunistic diseases. Which of the following opportunistic infections is
characterized by tonsillopharyngitis?

A.
B.
C.
D.

Respiratory candidiasis
Infectious mononucleosis
Cytomegalovirus disease
Pneumocystis carinii pneumonia
23. To determine possible sources of sexually transmitted infections, which is
the BEST method that may be undertaken by the public health nurse?

A.
B.
C.
D.

Contact tracing
Community survey
Mass screening tests
Interview of suspects
24. Antiretroviral agents, such as AZT, are used in the management of AIDS.
Which of the following is NOT an action expected of these drugs.

A.
B.
C.
D.

They prolong the life of the client with AIDS.


They reduce the risk of opportunistic infections
They shorten the period of communicability of the disease.
They are able to bring about a cure of the disease condition.
25. A barangay had an outbreak of German measles. To prevent congenital
rubella, what is the BEST advice that you can give to women in the first
trimester of pregnancy in the barangay?

A.
B.
C.

Advice them on the signs of German measles.


Avoid crowded places, such as markets and moviehouses.
Consult at the health center where rubella vaccine may be
given.
D.
Consult a physician who may give them rubella
immunoglobulin.
26. You were invited to be the resource person in a training class for food
handlers. Which of the following would you emphasize regarding prevention of
staphylococcal food poisoning?
A.

All cooking and eating utensils must be thoroughly washed.

B.

Food must be cooked properly to destroy staphylococcal


microorganisms.
C.
Food handlers and food servers must have a negative stool
examination result.
D.
Proper handwashing during food preparation is the best way of
preventing the condition.
27. In a mothers class, you discussed childhood diseases such as chicken
pox. Which of the following statements about chicken pox is correct?
A.

The older one gets, the more susceptible he becomes to the


complications of chicken pox.
B.
A single attack of chicken pox will prevent future episodes,
including conditions such as shingles.
C.
To prevent an outbreak in the community, quarantine may be
imposed by health authorities.
D.
Chicken pox vaccine is best given when there is an impending
outbreak in the community.
28. Complications to infectious parotitis (mumps) may be serious in which
type of clients?
A.
B.
C.
D.

Pregnant women
Elderly clients
Young adult males
Young infants

Answers and Rationales


1.

2.

3.
4.

5.

Answer: (B) Liver cirrhosis. The etiologic agent of


schistosomiasis in the Philippines is Schistosoma japonicum, which
affects the small intestine and the liver. Liver damage is a
consequence of fibrotic reactions to schistosoma eggs in the liver.
Answer: (C) Proper use of sanitary toilets. The ova of the
parasite get out of the human body together with feces. Cutting the
cycle at this stage is the most effective way of preventing the
spread of the disease to susceptible hosts.
Answer: (B) II. A communal faucet or water standpost is
classified as Level II.
Answer: (A) Use of sterile syringes and
needles. Hepatitis A is transmitted through the fecal oral route.
Hepatitis B is transmitted through infected body secretions like
blood and semen.
Answer: (A) DPT. DPT is sensitive to freezing. The
appropriate storage temperature of DPT is 2 to 8 C only. OPV and

measles vaccine are highly sensitive to heat and require freezing.


MMR is not an immunization in the Expanded Program on
Immunization.
6.
Answer: (A) 45. To estimate the number of infants, multiply
total population by 3%.
7.
Answer: (B) Severe dehydration. The order of priority in
the management of severe dehydration is as follows: intravenous
fluid therapy, referral to a facility where IV fluids can be initiated
within 30 minutes, Oresol/nasogastric tube, Oresol/orem. When the
foregoing measures are not possible or effective, tehn urgent
referral to the hospital is done.
8.
Answer: (A) 3. Adequate blood supply to the area allows the
return of the color of the nailbed within 3 seconds.
9.
Answer: (B) Instruct the mother to give the child
Oresol. Since the child does not manifest any other danger sign,
maintenance of fluid balance and replacement of fluid loss may be
done by giving the client Oresol.
10.
Answer: (B) Buccal mucosa. Kopliks spot may be seen on
the mucosa of the mouth or the throat.
11.
Answer: (D) Measles. Viral conjunctivitis is transmitted by
direct or indirect contact with discharges from infected eyes. Acute
poliomyelitis is spread through the fecal-oral route and contact with
throat secretions, whereas diphtheria is through direct and indirect
contact with respiratory secretions.
12.
Answer: (A) Hemophilus influenzae. Hemophilus
meningitis is unusual over the age of 5 years. In developing
countries, the peak incidence is in children less than 6 months of
age. Morbillivirus is the etiology of measles. Streptococcus
pneumoniae and Neisseria meningitidis may cause meningitis, but
age distribution is not specific in young children.
13.
Answer: (D) Zooprophylaxis. Zooprophylaxis is done by
putting animals like cattle or dogs close to windows or doorways just
before nightfall. The Anopheles mosquito takes his blood meal from
the animal and goes back to its breeding place, thereby preventing
infection of humans.
14.
Answer: (A) Stream seeding. Stream seeding is done by
putting tilapia fry in streams or other bodies of water identified as
breeding places of the Anopheles mosquito
15.
Answer: (C) Destruction of breeding places of the
mosquito vector. Anopheles mosquitoes breed in slow-moving,
clear water, such as mountain streams.

16.
Answer: (B) Cholera. Passage of profuse watery stools is the
major symptom of cholera. Both amebic and bacillary dysentery are
characterized by the presence of blood and/or mucus in the stools.
Giardiasis is characterized by fat malabsorption and, therefore,
steatorrhea.
17.
Answer: (B) S. japonicum. S. mansoni is found mostly in
Africa and South America; S. haematobium in Africa and the Middle
East; and S. malayensis only in peninsular Malaysia.
18.
Answer: (D) Leptospirosis. Leptospirosis is transmitted
through contact with the skin or mucous membrane with water or
moist soil contaminated with urine of infected animals, like rats.
19.
Answer: (C) III. Waterworks systems, such as MWSS, are
classified as level III.
20.
Answer: (D) Refer the client for a supplementary test,
such as Western blot, since the ELISA result may be false. A
client having a reactive ELISA result must undergo a more specific
test, such as Western blot. A negative supplementary test result
means that the ELISA result was false and that, most probably, the
client is not infected.
21.
Answer: (A) Being faithful to a single sexual
partner. Sexual fidelity rules out the possibility of getting the
disease by sexual contact with another infected person.
Transmission occurs mostly through sexual intercourse and exposure
to blood or tissues.
22.
Answer: (B) Infectious mononucleosis. Cytomegalovirus
disease is an acute viral disease characterized by fever, sore throat
and lymphadenopathy.
23.
Answer: (A) Contact tracing. Contact tracing is the most
practical and reliable method of finding possible sources of personto-person transmitted infections, such as sexually transmitted
diseases.
24.
Answer: (D) They are able to bring about a cure of the
disease condition. There is no known treatment for AIDS.
Antiretroviral agents reduce the risk of opportunistic infections and
prolong life, but does not cure the underlying immunodeficiency.
25.
Answer: (D) Consult a physician who may give them
rubella immunoglobulin. Rubella vaccine is made up of
attenuated German measles viruses. This is contraindicated in
pregnancy. Immune globulin, a specific prophylactic against German
measles, may be given to pregnant women.

26.
Answer: (D) Proper handwashing during food
preparation is the best way of preventing the
condition. Symptoms of this food poisoning are due to
staphylococcal enterotoxin, not the microorganisms themselves.
Contamination is by food handling by persons with staphylococcal
skin or eye infections.
27.
Answer: (A) The older one gets, the more susceptible he
becomes to the complications of chicken pox. Chicken pox is
usually more severe in adults than in children. Complications, such
as pneumonia, are higher in incidence in adults.
28.
Answer: (C) Young adult males. Epididymitis and orchitis
are possible complications of mumps. In post-adolescent males,
bilateral inflammation of the testes and epididymis may cause
sterility.
1. May arrives at the health care clinic and tells the nurse that her last
menstrual period was 9 weeks ago. She also tells the nurse that a home
pregnancy test was positive but she began to have mild cramps and is now
having moderate vaginal bleeding. During the physical examination of the
client, the nurse notes that May has a dilated cervix. The nurse determines
that May is experiencing which type of abortion?
A.
Inevitable
B.
Incomplete
C.
Threatened
D.
Septic
2. Nurse Reese is reviewing the record of a pregnant client for her first
prenatal visit. Which of the following data, if noted on the clients record, would
alert the nurse that the client is at risk for a spontaneous abortion?
A.
Age 36 years
B.
History of syphilis
C.
History of genital herpes
D.
History of diabetes mellitus
3. Nurse Hazel is preparing to care for a client who is newly admitted to the
hospital with a possible diagnosis of ectopic pregnancy. Nurse Hazel develops
a plan of care for the client and determines that which of the following nursing
actions is the priority?
A.
Monitoring weight
B.
Assessing for edema
C.
Monitoring apical pulse
D.
Monitoring temperature

4. Nurse Oliver is teaching a diabetic pregnant client about nutrition and


insulin needs during pregnancy. The nurse determines that the client
understands dietary and insulin needs if the client states that the second half
of pregnancy require:
A.
Decreased caloric intake
B.
Increased caloric intake
C.
Decreased Insulin
D.
Increase Insulin
5. Nurse Michelle is assessing a 24 year old client with a diagnosis of
hydatidiform mole. She is aware that one of the following is unassociated with
this condition?
A.
Excessive fetal activity.
B.
Larger than normal uterus for gestational age.
C.
Vaginal bleeding
D.
Elevated levels of human chorionic gonadotropin.
6. A pregnant client is receiving magnesium sulfate for severe pregnancy
induced hypertension (PIH). The clinical findings that would warrant use of the
antidote , calcium gluconate is:
A.
Urinary output 90 cc in 2 hours.
B.
Absent patellar reflexes.
C.
Rapid respiratory rate above 40/min.
D.
Rapid rise in blood pressure.
7. During vaginal examination of Janah who is in labor, the presenting part is
at station plus two. Nurse, correctly interprets it as:
A.
Presenting part is 2 cm above the plane of the ischial spines.
B.
Biparietal diameter is at the level of the ischial spines.
C.
Presenting part in 2 cm below the plane of the ischial spines.
D.
Biparietal diameter is 2 cm above the ischial spines.
8. A pregnant client is receiving oxytocin (Pitocin) for induction of labor. A
condition that warrant the nurse in-charge to discontinue I.V. infusion of
Pitocin is:
A.
Contractions every 1 minutes lasting 70-80 seconds.
B.
Maternal temperature 101.2
C.
Early decelerations in the fetal heart rate.
D.
Fetal heart rate baseline 140-160 bpm.
9. Calcium gluconate is being administered to a client with pregnancy induced
hypertension (PIH). A nursing action that must be initiated as the plan of care
throughout injection of the drug is:
A.
Ventilator assistance
B.
CVP readings
C.
EKG tracings

D.

Continuous CPR
10. A trial for vaginal delivery after an earlier caesareans, would likely to be
given to a gravida, who had:
A.
First low transverse cesarean was for active herpes type 2
infections; vaginal culture at 39 weeks pregnancy was positive.
B.
First and second caesareans were for cephalopelvic
disproportion.
C.
First caesarean through a classic incision as a result of severe
fetal distress.
D.
First low transverse caesarean was for breech position. Fetus in
this pregnancy is in a vertex presentation.
11.Nurse Ryan is aware that the best initial approach when trying to take a
crying toddlers temperature is:
A.
Talk to the mother first and then to the toddler.
B.
Bring extra help so it can be done quickly.
C.
Encourage the mother to hold the child.
D.
Ignore the crying and screaming.
12.Baby Tina a 3 month old infant just had a cleft lip and palate repair. What
should the nurse do to prevent trauma to operative site?
A.
Avoid touching the suture line, even when cleaning.
B.
Place the baby in prone position.
C.
Give the baby a pacifier.
D.
Place the infants arms in soft elbow restraints.
13. Which action should nurse Marian include in the care plan for a 2 month
old with heart failure?
A.
Feed the infant when he cries.
B.
Allow the infant to rest before feeding.
C.
Bathe the infant and administer medications before feeding.
D.
Weigh and bathe the infant before feeding.
14.Nurse Hazel is teaching a mother who plans to discontinue breast feeding
after 5 months. The nurse should advise her to include which foods in her
infants diet?
A.
Skim milk and baby food.
B.
Whole milk and baby food.
C.
Iron-rich formula only.
D.
Iron-rich formula and baby food.
15.Mommy Linda is playing with her infant, who is sitting securely alone on
the floor of the clinic. The mother hides a toy behind her back and the infant
looks for it. The nurse is aware that estimated age of the infant would be:
A.
6 months
B.
4 months

C.
D.

8 months
10 months
16.Which of the following is the most prominent feature of public health
nursing?
A.
It involves providing home care to sick people who are not
confined in the hospital.
B.
Services are provided free of charge to people within the
catchments area.
C.
The public health nurse functions as part of a team providing a
public health nursing services.
D.
Public health nursing focuses on preventive, not curative,
services.
17.When the nurse determines whether resources were maximized in
implementing Ligtas Tigdas, she is evaluating
A.
Effectiveness
B.
Efficiency
C.
Adequacy
D.
Appropriateness
18.Vangie is a new B.S.N. graduate. She wants to become a Public Health
Nurse. Where should she apply?
A.
Department of Health
B.
Provincial Health Office
C.
Regional Health Office
D.
Rural Health Unit
19.Tony is aware the Chairman of the Municipal Health Board is:
A.
Mayor
B.
Municipal Health Officer
C.
Public Health Nurse
D.
Any qualified physician
20.Myra is the public health nurse in a municipality with a total population of
about 20,000. There are 3 rural health midwives among the RHU personnel.
How many more midwife items will the RHU need?
A.
1
B.
2
C.
3
D.
The RHU does not need any more midwife item.
21.According to Freeman and Heinrich, community health nursing is a
developmental service. Which of the following best illustrates this statement?
A.
The community health nurse continuously develops himself
personally and professionally.

B.

Health education and community organizing are necessary in


providing community health services.
C.
Community health nursing is intended primarily for health
promotion and prevention and treatment of disease.
D.
The goal of community health nursing is to provide nursing
services to people in their own places of residence.
22.Nurse Tina is aware that the disease declared through Presidential
Proclamation No. 4 as a target for eradication in the Philippines is?
A.
Poliomyelitis
B.
Measles
C.
Rabies
D.
Neonatal tetanus
23.May knows that the step in community organizing that involves training of
potential leaders in the community is:
A.
Integration
B.
Community organization
C.
Community study
D.
Core group formation
24.Beth a public health nurse takes an active role in community participation.
What is the primary goal of community organizing?
A.
To educate the people regarding community health problems
B.
To mobilize the people to resolve community health problems
C.
To maximize the communitys resources in dealing with health
problems.
D.
To maximize the communitys resources in dealing with health
problems.
25.Tertiary prevention is needed in which stage of the natural history of
disease?
A.
Pre-pathogenesis
B.
Pathogenesis
C.
Prodromal
D.
Terminal
26.The nurse is caring for a primigravid client in the labor and delivery area.
Which condition would place the client at risk for disseminated intravascular
coagulation (DIC)?
A.
Intrauterine fetal death.
B.
Placenta accreta.
C.
Dysfunctional labor.
D.
Premature rupture of the membranes.
27.A fullterm client is in labor. Nurse Betty is aware that the fetal heart rate
would be:

A.
B.
C.
D.

80 to 100 beats/minute
100 to 120 beats/minute
120 to 160 beats/minute
160 to 180 beats/minute
28.The skin in the diaper area of a 7 month old infant is excoriated and red.
Nurse Hazel should instruct the mother to:
A.
Change the diaper more often.
B.
Apply talc powder with diaper changes.
C.
Wash the area vigorously with each diaper change.
D.
Decrease the infants fluid intake to decrease saturating
diapers.
29.Nurse Carla knows that the common cardiac anomalies in children with
Down Syndrome (tri-somy 21) is:
A.
Atrial septal defect
B.
Pulmonic stenosis
C.
Ventricular septal defect
D.
Endocardial cushion defect
30.Malou was diagnosed with severe preeclampsia is now receiving I.V.
magnesium sulfate. The adverse effects associated with magnesium sulfate
is:
A.
Anemia
B.
Decreased urine output
C.
Hyperreflexia
D.
Increased respiratory rate
31.A 23 year old client is having her menstrual period every 2 weeks that last
for 1 week. This type of menstrual pattern is bets defined by:
A.
Menorrhagia
B.
Metrorrhagia
C.
Dyspareunia
D.
Amenorrhea
32. Jannah is admitted to the labor and delivery unit. The critical laboratory
result for this client would be:
A.
Oxygen saturation
B.
Iron binding capacity
C.
Blood typing
D.
Serum Calcium
33.Nurse Gina is aware that the most common condition found during the
second-trimester of pregnancy is:
A.
Metabolic alkalosis
B.
Respiratory acidosis
C.
Mastitis

D.

Physiologic anemia
34.Nurse Lynette is working in the triage area of an emergency department.
She sees that several pediatric clients arrive simultaneously. The client who
needs to be treated first is:
A.
A crying 5 year old child with a laceration on his scalp.
B.
A 4 year old child with a barking coughs and flushed
appearance.
C.
A 3 year old child with Down syndrome who is pale and asleep
in his mothers arms.
D.
A 2 year old infant with stridorous breath sounds, sitting up in
his mothers arms and drooling.
35.Maureen in her third trimester arrives at the emergency room with painless
vaginal bleeding. Which of the following conditions is suspected?
A.
Placenta previa
B.
Abruptio placentae
C.
Premature labor
D.
Sexually transmitted disease
36.A young child named Richard is suspected of having pinworms. The
community nurse collects a stool specimen to confirm the diagnosis. The
nurse should schedule the collection of this specimen for:
A.
Just before bedtime
B.
After the child has been bathe
C.
Any time during the day
D.
Early in the morning
37.In doing a childs admission assessment, Nurse Betty should be alert to
note which signs or symptoms of chronic lead poisoning?
A.
Irritability and seizures
B.
Dehydration and diarrhea
C.
Bradycardia and hypotension
D.
Petechiae and hematuria
38.To evaluate a womans understanding about the use of diaphragm for
family planning, Nurse Trish asks her to explain how she will use the
appliance. Which response indicates a need for further health teaching?
A.
I should check the diaphragm carefully for holes every time I
use it
B.
I may need a different size of diaphragm if I gain or lose
weight more than 20 pounds
C.
The diaphragm must be left in place for atleast 6 hours after
intercourse
D.
I really need to use the diaphragm and jelly most during the
middle of my menstrual cycle.

39.Hypoxia is a common complication of laryngotracheobronchitis. Nurse


Oliver should frequently assess a child with laryngotracheobronchitis for:
A.
Drooling
B.
Muffled voice
C.
Restlessness
D.
Low-grade fever
40.How should Nurse Michelle guide a child who is blind to walk to the
playroom?
A.
Without touching the child, talk continuously as the child walks
down the hall.
B.
Walk one step ahead, with the childs hand on the nurses
elbow.
C.
Walk slightly behind, gently guiding the child forward.
D.
Walk next to the child, holding the childs hand.
41.When assessing a newborn diagnosed with ductus arteriosus, Nurse Olivia
should expect that the child most likely would have an:
A.
Loud, machinery-like murmur.
B.
Bluish color to the lips.
C.
Decreased BP reading in the upper extremities
D.
Increased BP reading in the upper extremities.
42.The reason nurse May keeps the neonate in a neutral thermal environment
is that when a newborn becomes too cool, the neonate requires:
A.
Less oxygen, and the newborns metabolic rate increases.
B.
More oxygen, and the newborns metabolic rate decreases.
C.
More oxygen, and the newborns metabolic rate increases.
D.
Less oxygen, and the newborns metabolic rate decreases.
43.Before adding potassium to an infants I.V. line, Nurse Ron must be sure to
assess whether this infant has:
A.
Stable blood pressure
B.
Patant fontanelles
C.
Moros reflex
D.
Voided
44.Nurse Carla should know that the most common causative factor of
dermatitis in infants and younger children is:
A.
Baby oil
B.
Baby lotion
C.
Laundry detergent
D.
Powder with cornstarch
45.During tube feeding, how far above an infants stomach should the nurse
hold the syringe with formula?
A.
6 inches

B.
C.
D.

12 inches
18 inches
24 inches
46. In a mothers class, Nurse Lhynnete discussed childhood diseases such
as chicken pox. Which of the following statements about chicken pox is
correct?
A.
The older one gets, the more susceptible he becomes to the
complications of chicken pox.
B.
A single attack of chicken pox will prevent future episodes,
including conditions such as shingles.
C.
To prevent an outbreak in the community, quarantine may be
imposed by health authorities.
D.
Chicken pox vaccine is best given when there is an impending
outbreak in the community.
47.Barangay Pinoy had an outbreak of German measles. To prevent
congenital rubella, what is the BEST advice that you can give to women in the
first trimester of pregnancy in the barangay Pinoy?
A.
Advice them on the signs of German measles.
B.
Avoid crowded places, such as markets and movie houses.
C.
Consult at the health center where rubella vaccine may be
given.
D.
Consult a physician who may give them rubella
immunoglobulin.
48.Myrna a public health nurse knows that to determine possible sources of
sexually transmitted infections, the BEST method that may be undertaken is:
A.
Contact tracing
B.
Community survey
C.
Mass screening tests
D.
Interview of suspects
49.A 33-year old female client came for consultation at the health center with
the chief complaint of fever for a week. Accompanying symptoms were muscle
pains and body malaise. A week after the start of fever, the client noted
yellowish discoloration of his sclera. History showed that he waded in flood
waters about 2 weeks before the onset of symptoms. Based on her history,
which disease condition will you suspect?
A.
Hepatitis A
B.
Hepatitis B
C.
Tetanus
D.
Leptospirosis

50.Mickey a 3-year old client was brought to the health center with the chief
complaint of severe diarrhea and the passage of rice water stools. The client
is most probably suffering from which condition?
A.
Giardiasis
B.
Cholera
C.
Amebiasis
D.
Dysentery
51.The most prevalent form of meningitis among children aged 2 months to 3
years is caused by which microorganism?
A.
Hemophilus influenzae
B.
Morbillivirus
C.
Steptococcus pneumoniae
D.
Neisseria meningitidis
52.The student nurse is aware that the pathognomonic sign of measles is
Kopliks spot and you may see Kopliks spot by inspecting the:
A.
Nasal mucosa
B.
Buccal mucosa
C.
Skin on the abdomen
D.
Skin on neck
53.Angel was diagnosed as having Dengue fever. You will say that there is
slow capillary refill when the color of the nailbed that you pressed does not
return within how many seconds?
A.
3 seconds
B.
6 seconds
C.
9 seconds
D.
10 seconds
54.In Integrated Management of Childhood Illness, the nurse is aware that the
severe conditions generally require urgent referral to a hospital. Which of the
following severe conditions DOES NOT always require urgent referral to a
hospital?
A.
Mastoiditis
B.
Severe dehydration
C.
Severe pneumonia
D.
Severe febrile disease
55.Myrna a public health nurse will conduct outreach immunization in a
barangay Masay with a population of about 1500. The estimated number of
infants in the barangay would be:
A.
45 infants
B.
50 infants
C.
55 infants
D.
65 infants

56.The community nurse is aware that the biological used in Expanded


Program on Immunization (EPI) should NOT be stored in the freezer?
A.
DPT
B.
Oral polio vaccine
C.
Measles vaccine
D.
MMR
57.It is the most effective way of controlling schistosomiasis in an endemic
area?
A.
Use of molluscicides
B.
Building of foot bridges
C.
Proper use of sanitary toilets
D.
Use of protective footwear, such as rubber boots
58.Several clients is newly admitted and diagnosed with leprosy. Which of the
following clients should be classified as a case of multibacillary leprosy?
A.
3 skin lesions, negative slit skin smear
B.
3 skin lesions, positive slit skin smear
C.
5 skin lesions, negative slit skin smear
D.
5 skin lesions, positive slit skin smear
59.Nurses are aware that diagnosis of leprosy is highly dependent on
recognition of symptoms. Which of the following is an early sign of leprosy?
A.
Macular lesions
B.
Inability to close eyelids
C.
Thickened painful nerves
D.
Sinking of the nosebridge
60.Marie brought her 10 month old infant for consultation because of fever,
started 4 days prior to consultation. In determining malaria risk, what will you
do?
A.
Perform a tourniquet test.
B.
Ask where the family resides.
C.
Get a specimen for blood smear.
D.
Ask if the fever is present everyday.
61.Susie brought her 4 years old daughter to the RHU because of cough and
colds. Following the IMCI assessment guide, which of the following is a
danger sign that indicates the need for urgent referral to a hospital?
A.
Inability to drink
B.
High grade fever
C.
Signs of severe dehydration
D.
Cough for more than 30 days
62.Jimmy a 2-year old child revealed baggy pants. As a nurse, using the
IMCI guidelines, how will you manage Jimmy?
A.
Refer the child urgently to a hospital for confinement.

B.

Coordinate with the social worker to enroll the child in a


feeding program.
C.
Make a teaching plan for the mother, focusing on menu
planning for her child.
D.
Assess and treat the child for health problems like infections
and intestinal parasitism.
63.Gina is using Oresol in the management of diarrhea of her 3-year old child.
She asked you what to do if her child vomits. As a nurse you will tell her to:
A.
Bring the child to the nearest hospital for further assessment.
B.
Bring the child to the health center for intravenous fluid
therapy.
C.
Bring the child to the health center for assessment by the
physician.
D.
Let the child rest for 10 minutes then continue giving Oresol
more slowly.
64.Nikki a 5-month old infant was brought by his mother to the health center
because of diarrhea for 4 to 5 times a day. Her skin goes back slowly after a
skin pinch and her eyes are sunken. Using the IMCI guidelines, you will
classify this infant in which category?
A.
No signs of dehydration
B.
Some dehydration
C.
Severe dehydration
D.
The data is insufficient.
65.Chris a 4-month old infant was brought by her mother to the health center
because of cough. His respiratory rate is 42/minute. Using the Integrated
Management of Child Illness (IMCI) guidelines of assessment, his breathing is
considered as:
A.
Fast
B.
Slow
C.
Normal
D.
Insignificant
66.Maylene had just received her 4th dose of tetanus toxoid. She is aware
that her baby will have protection against tetanus for
A.
1 year
B.
3 years
C.
5 years
D.
Lifetime
67.Nurse Ron is aware that unused BCG should be discarded after how many
hours of reconstitution?
A.
2 hours
B.
4 hours

C.
D.

8 hours
At the end of the day
68.The nurse explains to a breastfeeding mother that breast milk is sufficient
for all of the babys nutrient needs only up to:
A.
5 months
B.
6 months
C.
1 year
D.
2 years
69.Nurse Ron is aware that the gestational age of a conceptus that is
considered viable (able to live outside the womb) is:
A.
8 weeks
B.
12 weeks
C.
24 weeks
D.
32 weeks
70.When teaching parents of a neonate the proper position for the neonates
sleep, the nurse Patricia stresses the importance of placing the neonate on
his back to reduce the risk of which of the following?
A.
Aspiration
B.
Sudden infant death syndrome (SIDS)
C.
Suffocation
D.
Gastroesophageal reflux (GER)
71.Which finding might be seen in baby James a neonate suspected of having
an infection?
A.
Flushed cheeks
B.
Increased temperature
C.
Decreased temperature
D.
Increased activity level
72.Baby Jenny who is small-for-gestation is at increased risk during the
transitional period for which complication?
A.
Anemia probably due to chronic fetal hyposia
B.
Hyperthermia due to decreased glycogen stores
C.
Hyperglycemia due to decreased glycogen stores
D.
Polycythemia probably due to chronic fetal hypoxia
73.Marjorie has just given birth at 42 weeks gestation. When the nurse
assessing the neonate, which physical finding is expected?
A.
A sleepy, lethargic baby
B.
Lanugo covering the body
C.
Desquamation of the epidermis
D.
Vernix caseosa covering the body

74.After reviewing the Myrnas maternal history of magnesium sulfate during


labor, which condition would nurse Richard anticipate as a potential problem
in the neonate?
A.
Hypoglycemia
B.
Jitteriness
C.
Respiratory depression
D.
Tachycardia
75.Which symptom would indicate the Baby Alexandra was adapting
appropriately to extra-uterine life without difficulty?
A.
Nasal flaring
B.
Light audible grunting
C.
Respiratory rate 40 to 60 breaths/minute
D.
Respiratory rate 60 to 80 breaths/minute
76. When teaching umbilical cord care for Jennifer a new mother, the nurse
Jenny would include which information?
A.
Apply peroxide to the cord with each diaper change
B.
Cover the cord with petroleum jelly after bathing
C.
Keep the cord dry and open to air
D.
Wash the cord with soap and water each day during a tub bath.
77.Nurse John is performing an assessment on a neonate. Which of the
following findings is considered common in the healthy neonate?
A.
Simian crease
B.
Conjunctival hemorrhage
C.
Cystic hygroma
D.
Bulging fontanelle
78.Dr. Esteves decides to artificially rupture the membranes of a mother who
is on labor. Following this procedure, the nurse Hazel checks the fetal heart
tones for which the following reasons?
A.
To determine fetal well-being.
B.
To assess for prolapsed cord
C.
To assess fetal position
D.
To prepare for an imminent delivery.
79.Which of the following would be least likely to indicate anticipated bonding
behaviors by new parents?
A.
The parents willingness to touch and hold the new born.
B.
The parents expression of interest about the size of the new
born.
C.
The parents indication that they want to see the newborn.
D.
The parents interactions with each other.
80.Following a precipitous delivery, examination of the clients vagina reveals

a fourth-degree laceration. Which of the following would be


contraindicated when caring for this client?
A.
Applying cold to limit edema during the first 12 to 24 hours.
B.
Instructing the client to use two or more peripads to cushion
the area.
C.
Instructing the client on the use of sitz baths if ordered.
D.
Instructing the client about the importance of perineal (kegel)
exercises.
81. A pregnant woman accompanied by her husband, seeks admission to the
labor and delivery area. She states that shes in labor and says she attended
the facility clinic for prenatal care. Which question should the nurse Oliver ask
her first?
A.
Do you have any chronic illnesses?
B.
Do you have any allergies?
C.
What is your expected due date?
D.
Who will be with you during labor?
82.A neonate begins to gag and turns a dusky color. What should the nurse
do first?
A.
Calm the neonate.
B.
Notify the physician.
C.
Provide oxygen via face mask as ordered
D.
Aspirate the neonates nose and mouth with a bulb syringe.
83. When a client states that her water broke, which of the following actions
would be inappropriate for the nurse to do?
A.
Observing the pooling of straw-colored fluid.
B.
Checking vaginal discharge with nitrazine paper.
C.
Conducting a bedside ultrasound for an amniotic fluid index.
D.
Observing for flakes of vernix in the vaginal discharge.
84. A baby girl is born 8 weeks premature. At birth, she has no spontaneous
respirations but is successfully resuscitated. Within several hours she
develops respiratory grunting, cyanosis, tachypnea, nasal flaring, and
retractions. Shes diagnosed with respiratory distress syndrome, intubated,
and placed on a ventilator. Which nursing action should be included in the
babys plan of care to prevent retinopathy of prematurity?
A.
Cover his eyes while receiving oxygen.
B.
Keep her body temperature low.
C.
Monitor partial pressure of oxygen (Pao2) levels.
D.
Humidify the oxygen.
85. Which of the following is normal newborn calorie intake?
A.
110 to 130 calories per kg.
B.
30 to 40 calories per lb of body weight.

C.
D.

At least 2 ml per feeding


90 to 100 calories per kg
86. Nurse John is knowledgeable that usually individual twins will grow
appropriately and at the same rate as singletons until how many weeks?
A.
16 to 18 weeks
B.
18 to 22 weeks
C.
30 to 32 weeks
D.
38 to 40 weeks
87. Which of the following classifications applies to monozygotic twins for
whom the cleavage of the fertilized ovum occurs more than 13 days after
fertilization?
A.
conjoined twins
B.
diamniotic dichorionic twins
C.
diamniotic monochorionic twin
D.
monoamniotic monochorionic twins
88. Tyra experienced painless vaginal bleeding has just been diagnosed as
having a placenta previa. Which of the following procedures is usually
performed to diagnose placenta previa?
A.
Amniocentesis
B.
Digital or speculum examination
C.
External fetal monitoring
D.
Ultrasound
89. Nurse Arnold knows that the following changes in respiratory functioning
during pregnancy is considered normal:
A.
Increased tidal volume
B.
Increased expiratory volume
C.
Decreased inspiratory capacity
D.
Decreased oxygen consumption
90. Emily has gestational diabetes and it is usually managed by which of the
following therapy?
A.
Diet
B.
Long-acting insulin
C.
Oral hypoglycemic
D.
Oral hypoglycemic drug and insulin
91. Magnesium sulfate is given to Jemma with preeclampsia to prevent which
of the following condition?
A.
Hemorrhage
B.
Hypertension
C.
Hypomagnesemia
D.
Seizure

92. Cammile with sickle cell anemia has an increased risk for having a sickle
cell crisis during pregnancy. Aggressive management of a sickle cell crisis
includes which of the following measures?
A.
Antihypertensive agents
B.
Diuretic agents
C.
I.V. fluids
D.
Acetaminophen (Tylenol) for pain
93. Which of the following drugs is the antidote for magnesium toxicity?
A.
Calcium gluconate (Kalcinate)
B.
Hydralazine (Apresoline)
C.
Naloxone (Narcan)
D.
Rho (D) immune globulin (RhoGAM)
94. Marlyn is screened for tuberculosis during her first prenatal visit. An
intradermal injection of purified protein derivative (PPD) of the tuberculin
bacilli is given. She is considered to have a positive test for which of the
following results?
A.
An indurated wheal under 10 mm in diameter appears in 6 to
12 hours.
B.
An indurated wheal over 10 mm in diameter appears in 48 to
72 hours.
C.
A flat circumcised area under 10 mm in diameter appears in 6
to 12 hours.
D.
A flat circumcised area over 10 mm in diameter appears in 48
to 72 hours.
95. Dianne, 24 year-old is 27 weeks pregnant arrives at her physicians office
with complaints of fever, nausea, vomiting, malaise, unilateral flank pain, and
costovertebral angle tenderness. Which of the following diagnoses is most
likely?
A.
Asymptomatic bacteriuria
B.
Bacterial vaginosis
C.
Pyelonephritis
D.
Urinary tract infection (UTI)
96. Rh isoimmunization in a pregnant client develops during which of the
following conditions?
A.
Rh-positive maternal blood crosses into fetal blood, stimulating
fetal antibodies.
B.
Rh-positive fetal blood crosses into maternal blood,
stimulating maternal antibodies.
C.
Rh-negative fetal blood crosses into maternal blood,
stimulating maternal antibodies.

D.

Rh-negative maternal blood crosses into fetal blood,


stimulating fetal antibodies.
97. To promote comfort during labor, the nurse John advises a client to
assume certain positions and avoid others. Which position may cause
maternal hypotension and fetal hypoxia?
A.
Lateral position
B.
Squatting position
C.
Supine position
D.
Standing position
98. Celeste who used heroin during her pregnancy delivers a neonate. When
assessing the neonate, the nurse Lhynnette expects to find:
A.
Lethargy 2 days after birth.
B.
Irritability and poor sucking.
C.
A flattened nose, small eyes, and thin lips.
D.
Congenital defects such as limb anomalies.
99. The uterus returns to the pelvic cavity in which of the following time
frames?
A.
7th to 9th day postpartum.
B.
2 weeks postpartum.
C.
End of 6th week postpartum.
D.
When the lochia changes to alba.
100. Maureen, a primigravida client, age 20, has just completed a difficult,
forceps-assisted delivery of twins. Her labor was unusually long and required
oxytocin (Pitocin) augmentation. The nurse whos caring for her should stay
alert for:
A.
Uterine inversion
B.
Uterine atony
C.
Uterine involution
D.
Uterine discomfort

Answers and Rationales


1.

Answer: (A) Inevitable. An inevitable abortion is termination


of pregnancy that cannot be prevented. Moderate to severe
bleeding with mild cramping and cervical dilation would be noted in
this type of abortion.
2.
Answer: (B) History of syphilis. Maternal infections such as
syphilis, toxoplasmosis, and rubella are causes of spontaneous
abortion.
3.
Answer: (C) Monitoring apical pulse. Nursing care for the
client with a possible ectopic pregnancy is focused on preventing or
identifying hypovolemic shock and controlling pain. An elevated
pulse rate is an indicator of shock.

4.

Answer: (B) Increased caloric intake. Glucose crosses the


placenta, but insulin does not. High fetal demands for glucose,
combined with the insulin resistance caused by hormonal changes in
the last half of pregnancy can result in elevation of maternal blood
glucose levels. This increases the mothers demand for insulin and is
referred to as the diabetogenic effect of pregnancy.
5.
Answer: (A) Excessive fetal activity. The most common
signs and symptoms of hydatidiform mole includes elevated levels
of human chorionic gonadotropin, vaginal bleeding, larger than
normal uterus for gestational age, failure to detect fetal heart
activity even with sensitive instruments, excessive nausea
and vomiting, and early development of pregnancy-induced
hypertension. Fetal activity would not be noted.
6.
Answer: (B) Absent patellar reflexes. Absence of patellar
reflexes is an indicator of hypermagnesemia, which requires
administration of calcium gluconate.
7.
Answer: (C) Presenting part in 2 cm below the plane of
the ischial spines. Fetus at station plus two indicates that the
presenting part is 2 cm below the plane of the ischial spines.
8.
Answer: (A) Contractions every 1 minutes lasting 7080 seconds. Contractions every 1 minutes lasting 70-80
seconds, is indicative of hyperstimulation of the uterus, which could
result in injury to the mother and the fetus if Pitocin is not
discontinued.
9.
Answer: (C) EKG tracings. A potential side effect of calcium
gluconate administration is cardiac arrest. Continuous monitoring of
cardiac activity (EKG) throught administration of calcium gluconate
is an essential part of care.
10.
Answer: (D) First low transverse caesarean was for
breech position. Fetus in this pregnancy is in a vertex
presentation. This type of client has no obstetrical indication for a
caesarean section as she did with her first caesarean delivery.
11.
Answer: (A) Talk to the mother first and then to the
toddler. When dealing with a crying toddler, the best approach is to
talk to the mother and ignore the toddler first. This approach helps
the toddler get used to the nurse before she attempts any
procedures. It also gives the toddler an opportunity to see that the
mother trusts the nurse.
12.
Answer: (D) Place the infants arms in soft elbow
restraints. Soft restraints from the upper arm to the wrist prevent
the infant from touching her lip but allow him to hold a favorite item

such as a blanket. Because they could damage the operative site,


such as objects as pacifiers, suction catheters, and small spoons
shouldnt be placed in a babys mouth after cleft repair. A baby in a
prone position may rub her face on the sheets and traumatize the
operative site. The suture line should be cleaned gently to prevent
infection, which could interfere with healing and damage the
cosmetic appearance of the repair.
13.
Answer: (B) Allow the infant to rest before
feeding. Because feeding requires so much energy, an infant with
heart failure should rest before feeding.
14.
Answer: (C) Iron-rich formula only. The infants at age 5
months should receive iron-rich formula and that they shouldnt
receive solid food, even baby food until age 6 months.
15.
Answer: (D) 10 months. A 10 month old infant can sit alone
and understands object permanence, so he would look for the
hidden toy. At age 4 to 6 months, infants cant sit securely alone. At
age 8 months, infants can sit securely alone but cannot understand
the permanence of objects.
16.
Answer: (D) Public health nursing focuses on
preventive, not curative, services. The catchments area in PHN
consists of a residential community, many of whom are well
individuals who have greater need for preventive rather than
curative services.
17.
Answer: (B) Efficiency. Efficiency is determining whether the
goals were attained at the least possible cost.
18.
Answer: (D) Rural Health Unit. R.A. 7160 devolved basic
health services to local government units (LGUs ). The public health
nurse is an employee of the LGU.
19.
Answer: (A) Mayor. The local executive serves as the
chairman of the Municipal Health Board.
20.
Answer: (A) 1. Each rural health midwife is given a population
assignment of about 5,000.
21.
Answer: (B) Health education and community organizing
are necessary in providing community health services. The
community health nurse develops the health capability of people
through health education and community organizing activities.
22.
Answer: (B) Measles. Presidential Proclamation No. 4 is on
the Ligtas Tigdas Program.
23.
Answer: (D) Core group formation. In core group
formation, the nurse is able to transfer the technology of community

organizing to the potential or informal community leaders through a


training program.
24.
Answer: (D) To maximize the communitys resources in
dealing with health problems. Community organizing is a
developmental service, with the goal of developing the peoples selfreliance in dealing with community health problems. A, B and C are
objectives of contributory objectives to this goal.
25.
Answer: (D) Terminal. Tertiary prevention involves
rehabilitation, prevention of permanent disability and disability
limitation appropriate for convalescents, the disabled, complicated
cases and the terminally ill (those in the terminal stage of a
disease).
26.
Answer: (A) Intrauterine fetal death. Intrauterine fetal
death, abruptio placentae, septic shock, and amniotic fluid
embolism may trigger normal clotting mechanisms; if clotting
factors are depleted, DIC may occur. Placenta accreta, dysfunctional
labor, and premature rupture of the membranes arent associated
with DIC.
27.
Answer: (C) 120 to 160 beats/minute. A rate of 120 to 160
beats/minute in the fetal heart appropriate for filling the heart with
blood and pumping it out to the system.
28.
Answer: (A) Change the diaper more often. Decreasing
the amount of time the skin comes contact with wet soiled diapers
will help heal the irritation.
29.
Answer: (D) Endocardial cushion defect. Endocardial
cushion defects are seen most in children with Down syndrome,
asplenia, or polysplenia.
30.
Answer: (B) Decreased urine output. Decreased urine
output may occur in clients receiving I.V. magnesium and should be
monitored closely to keep urine output at greater than 30 ml/hour,
because magnesium is excreted through the kidneys and can easily
accumulate to toxic levels.
31.
Answer: (A) Menorrhagia. Menorrhagia is an excessive
menstrual period.
32.
Answer: (C) Blood typing. Blood type would be a critical
value to have because the risk of blood loss is always a potential
complication during the labor and delivery process. Approximately
40% of a womans cardiac output is delivered to the uterus,
therefore, blood loss can occur quite rapidly in the event of
uncontrolled bleeding.

33.
Answer: (D) Physiologic anemia. Hemoglobin values and
hematocrit decrease during pregnancy as the increase in plasma
volume exceeds the increase in red blood cell production.
34.
Answer: (D) A 2 year old infant with stridorous breath
sounds, sitting up in his mothers arms and drooling. The
infant with the airway emergency should be treated first, because of
the risk of epiglottitis.
35.
Answer: (A) Placenta previa. Placenta previa with painless
vaginal bleeding.
36.
Answer: (D) Early in the morning. Based on the nurses
knowledge of microbiology, the specimen should be collected early
in the morning. The rationale for this timing is that, because the
female worm lays eggs at night around the perineal area, the first
bowel movement of the day will yield the best results. The specific
type of stool specimen used in the diagnosis of pinworms is called
the tape test.
37.
Answer: (A) Irritability and seizures. Lead poisoning
primarily affects the CNS, causing increased intracranial pressure.
This condition results in irritability and changes in level of
consciousness, as well as seizure disorders, hyperactivity, and
learning disabilities.
38.
Answer: (D) I really need to use the diaphragm and
jelly most during the middle of my menstrual cycle. The
woman must understand that, although the fertile period is
approximately mid-cycle, hormonal variations do occur and can
result in early or late ovulation. To be effective, the diaphragm
should be inserted before every intercourse.
39.
Answer: (C) Restlessness. In a child, restlessness is the
earliest sign of hypoxia. Late signs of hypoxia in a child are
associated with a change in color, such as pallor or cyanosis.
40.
Answer: (B) Walk one step ahead, with the childs hand
on the nurses elbow. This procedure is generally recommended
to follow in guiding a person who is blind.
41.
Answer: (A) Loud, machinery-like murmur. A loud,
machinery-like murmur is a characteristic finding associated with
patent ductus arteriosus.
42.
Answer: (C) More oxygen, and the newborns metabolic
rate increases. When cold, the infant requires more oxygen and
there is an increase in metabolic rate. Non-shievering thermogenesis
is a complex process that increases the metabolic rate and rate of

oxygen consumption, therefore, the newborn increase heat


production.
43.
Answer: (D) Voided. Before administering potassium I.V. to
any client, the nurse must first check that the clients kidneys are
functioning and that the client is voiding. If the client is not voiding,
the nurse should withhold the potassium and notify the physician.
44.
Answer: (C) Laundry detergent. Eczema or dermatitis is an
allergic skin reaction caused by an offending allergen. The topical
allergen that is the most common causative factor is laundry
detergent.
45.
Answer: (A) 6 inches. This distance allows for easy flow of
the formula by gravity, but the flow will be slow enough not to
overload the stomach too rapidly.
46.
Answer: (A) The older one gets, the more susceptible he
becomes to the complications of chicken pox. Chicken pox is
usually more severe in adults than in children. Complications, such
as pneumonia, are higher in incidence in adults.
47.
Answer: (D) Consult a physician who may give them
rubella immunoglobulin. Rubella vaccine is made up of
attenuated German measles viruses. This is contraindicated in
pregnancy. Immune globulin, a specific prophylactic against German
measles, may be given to pregnant women.
48.
Answer: (A) Contact tracing. Contact tracing is the most
practical and reliable method of finding possible sources of personto-person transmitted infections, such as sexually transmitted
diseases.
49.
Answer: (D) Leptospirosis. Leptospirosis is transmitted
through contact with the skin or mucous membrane with water or
moist soil contaminated with urine of infected animals, like rats.
50.
Answer: (B) Cholera. Passage of profuse watery stools is the
major symptom of cholera. Both amebic and bacillary dysentery are
characterized by the presence of blood and/or mucus in the stools.
Giardiasis is characterized by fat malabsorption and, therefore,
steatorrhea.
51.
Answer: (A) Hemophilus influenzae. Hemophilus
meningitis is unusual over the age of 5 years. In developing
countries, the peak incidence is in children less than 6 months of
age. Morbillivirus is the etiology of measles. Streptococcus
pneumoniae and Neisseria meningitidis may cause meningitis, but
age distribution is not specific in young children.

52.
Answer: (B) Buccal mucosa. Kopliks spot may be seen on
the mucosa of the mouth or the throat.
53.
Answer: (A) 3 seconds. Adequate blood supply to the area
allows the return of the color of the nailbed within 3 seconds.
54.
Answer: (B) Severe dehydration. The order of priority in
the management of severe dehydration is as follows: intravenous
fluid therapy, referral to a facility where IV fluids can be initiated
within 30 minutes, Oresol or nasogastric tube. When the foregoing
measures are not possible or effective, then urgent referral to the
hospital is done.
55.
Answer: (A) 45 infants. To estimate the number of infants,
multiply total population by 3%.
56.
Answer: (A) DPT. DPT is sensitive to freezing. The appropriate
storage temperature of DPT is 2 to 8 C only. OPV and measles
vaccine are highly sensitive to heat and require freezing. MMR is not
an immunization in the Expanded Program on Immunization.
57.
Answer: (C) Proper use of sanitary toilets. The ova of the
parasite get out of the human body together with feces. Cutting the
cycle at this stage is the most effective way of preventing the
spread of the disease to susceptible hosts.
58.
Answer: (D) 5 skin lesions, positive slit skin smear. A
multibacillary leprosy case is one who has a positive slit skin smear
and at least 5 skin lesions.
59.
Answer: (C) Thickened painful nerves. The lesion of
leprosy is not macular. It is characterized by a change in skin color
(either reddish or whitish) and loss of sensation, sweating and hair
growth over the lesion. Inability to close the eyelids (lagophthalmos)
and sinking of the nosebridge are late symptoms.
60.
Answer: (B) Ask where the family resides. Because
malaria is endemic, the first question to determine malaria risk is
where the clients family resides. If the area of residence is not a
known endemic area, ask if the child had traveled within the past 6
months, where she was brought and whether she stayed overnight
in that area.
61.
Answer: (A) Inability to drink. A sick child aged 2 months to
5 years must be referred urgently to a hospital if he/she has one or
more of the following signs: not able to feed or drink, vomits
everything, convulsions, abnormally sleepy or difficult to awaken.
62.
Answer: (A) Refer the child urgently to a hospital for
confinement. Baggy pants is a sign of severe marasmus. The
best management is urgent referral to a hospital.

63.
Answer: (D) Let the child rest for 10 minutes then
continue giving Oresol more slowly. If the child vomits
persistently, that is, he vomits everything that he takes in, he has to
be referred urgently to a hospital. Otherwise, vomiting is managed
by letting the child rest for 10 minutes and then continuing with
Oresol administration. Teach the mother to give Oresol more slowly.
64.
Answer: (B) Some dehydration. Using the assessment
guidelines of IMCI, a child (2 months to 5 years old) with diarrhea is
classified as having SOME DEHYDRATION if he shows 2 or more of
the following signs: restless or irritable, sunken eyes, the skin goes
back slow after a skin pinch.
65.
Answer: (C) Normal. In IMCI, a respiratory rate of 50/minute
or more is fast breathing for an infant aged 2 to 12 months.
66.
Answer: (A) 1 year. The baby will have passive natural
immunity by placental transfer of antibodies. The mother will have
active artificial immunity lasting for about 10 years. 5 doses will give
the mother lifetime protection.
67.
Answer: (B) 4 hours. While the unused portion of other
biologicals in EPI may be given until the end of the day, only BCG is
discarded 4 hours after reconstitution. This is why BCG immunization
is scheduled only in the morning.
68.
Answer: (B) 6 months. After 6 months, the babys nutrient
needs, especially the babys iron requirement, can no longer be
provided by mothers milk alone.
69.
Answer: (C) 24 weeks. At approximately 23 to 24 weeks
gestation, the lungs are developed enough to sometimes maintain
extrauterine life. The lungs are the most immature system during
the gestation period. Medical care for premature labor begins much
earlier (aggressively at 21 weeks gestation)
70.
Answer: (B) Sudden infant death syndrome
(SIDS). Supine positioning is recommended to reduce the risk of
SIDS in infancy. The risk of aspiration is slightly increased with the
supine position. Suffocation would be less likely with an infant
supine than prone and the position for GER requires the head of the
bed to be elevated.
71.
Answer: (C) Decreased temperature. Temperature
instability, especially when it results in a low temperature in the
neonate, may be a sign of infection. The neonates color often
changes with an infection process but generally becomes ashen or
mottled. The neonate with an infection will usually show a decrease
in activity level or lethargy.

72.
Answer: (D) Polycythemia probably due to chronic fetal
hypoxia. The small-for-gestation neonate is at risk for developing
polycythemia during the transitional period in an attempt to
decreasehypoxia. The neonates are also at increased risk for
developing hypoglycemia and hypothermia due to decreased
glycogen stores.
73.
Answer: (C) Desquamation of the epidermis. Postdate
fetuses lose the vernix caseosa, and the epidermis may become
desquamated. These neonates are usually very alert. Lanugo is
missing in the postdate neonate.
74.
Answer: (C) Respiratory depression. Magnesium sulfate
crosses the placenta and adverse neonatal effects are respiratory
depression, hypotonia, and bradycardia. The serum blood sugar isnt
affected by magnesium sulfate. The neonate would be floppy, not
jittery.
75.
Answer: (C) Respiratory rate 40 to 60 breaths/minute. A
respiratory rate 40 to 60 breaths/minute is normal for a neonate
during the transitional period. Nasal flaring, respiratory rate more
than 60 breaths/minute, and audible grunting are signs of
respiratory distress.
76.
Answer: (C) Keep the cord dry and open to air. Keeping
the cord dry and open to air helps reduce infection and hastens
drying. Infants arent given tub bath but are sponged off until the
cord falls off. Petroleum jelly prevents the cord from drying and
encourages infection. Peroxide could be painful and isnt
recommended.
77.
Answer: (B) Conjunctival hemorrhage. Conjunctival
hemorrhages are commonly seen in neonates secondary to the
cranial pressure applied during the birth process. Bulging fontanelles
are a sign of intracranial pressure. Simian creases are present in
40% of the neonates with trisomy 21. Cystic hygroma is a neck
mass that can affect the airway.
78.
Answer: (B) To assess for prolapsed cord. After a client
has an amniotomy, the nurse should assure that the cord isnt
prolapsed and that the baby tolerated the procedure well. The most
effective way to do this is to check the fetal heart rate. Fetal wellbeing is assessed via a nonstress test. Fetal position is determined
by vaginal examination. Artificial rupture of membranes doesnt
indicate an imminent delivery.
79.
Answer: (D) The parents interactions with each
other. Parental interaction will provide the nurse with a good

assessment of the stability of the familys home life but it has no


indication for parental bonding. Willingness to touch and hold the
newborn, expressing interest about the newborns size, and
indicating a desire to see the newborn are behaviors indicating
parental bonding.
80.
Answer: (B) Instructing the client to use two or more
peripads to cushion the area. Using two or more peripads would
do little to reduce the pain or promote perineal healing. Cold
applications, sitz baths, and Kegel exercises are important measures
when the client has a fourth-degree laceration.
81.
Answer: (C) What is your expected due date? When
obtaining the history of a client who may be in labor, the nurses
highest priority is to determine her current status, particularly her
due date, gravidity, and parity. Gravidity and parity affect the
duration of labor and the potential for labor complications. Later, the
nurse should ask about chronic illnesses, allergies, and support
persons.
82.
Answer: (D) Aspirate the neonates nose and mouth
with a bulb syringe. The nurses first action should be to clear the
neonates airway with a bulb syringe. After the airway is clear and
the neonates color improves, the nurse should comfort and calm
the neonate. If the problem recurs or the neonates color doesnt
improve readily, the nurse should notify the physician. Administering
oxygen when the airway isnt clear would be ineffective.
83.
Answer: (C) Conducting a bedside ultrasound for an
amniotic fluid index. It isnt within a nurses scope of practice to
perform and interpret a bedside ultrasound under these conditions
and without specialized training. Observing for pooling of strawcolored fluid, checking vaginal discharge with nitrazine paper, and
observing for flakes of vernix are appropriate assessments for
determining whether a client has ruptured membranes.
84.
Answer: (C) Monitor partial pressure of oxygen (Pao2)
levels. Monitoring PaO2 levels and reducing the oxygen
concentration to keep PaO2 within normal limits reduces the risk of
retinopathy of prematurity in a premature infant receiving oxygen.
Covering the infants eyes and humidifying the oxygen dont reduce
the risk of retinopathy of prematurity. Because cooling increases the
risk of acidosis, the infant should be kept warm so that his
respiratory distress isnt aggravated.
85.
Answer: (A) 110 to 130 calories per kg. Calories per kg is
the accepted way of determined appropriate nutritional intake for a

newborn. The recommended calorie requirement is 110 to 130


calories per kg of newborn body weight. This level will maintain a
consistent blood glucose level and provide enough calories for
continued growth and development.
86.
Answer: (C) 30 to 32 weeks. Individual twins usually grow at
the same rate as singletons until 30 to 32 weeks gestation, then
twins dont gain weight as rapidly as singletons of the same
gestational age. The placenta can no longer keep pace with the
nutritional requirements of both fetuses after 32 weeks, so theres
some growth retardation in twins if they remain in utero at 38 to 40
weeks.
87.
Answer: (A) conjoined twins. The type of placenta that
develops in monozygotic twins depends on the time at which
cleavage of the ovum occurs. Cleavage in conjoined twins occurs
more than 13 days after fertilization. Cleavage that occurs less than
3 day after fertilization results in diamniotic dicchorionic twins.
Cleavage that occurs between days 3 and 8 results in diamniotic
monochorionic twins. Cleavage that occurs between days 8 to 13
result in monoamniotic monochorionic twins.
88.
Answer: (D) Ultrasound. Once the mother and the fetus are
stabilized, ultrasound evaluation of the placenta should be done to
determine the cause of the bleeding. Amniocentesis is
contraindicated in placenta previa. A digital or speculum
examination shouldnt be done as this may lead to severe bleeding
or hemorrhage. External fetal monitoring wont detect a placenta
previa, although it will detect fetal distress, which may result from
blood loss or placenta separation.
89.
Answer: (A) Increased tidal volume. A pregnant client
breathes deeper, which increases the tidal volume of gas moved in
and out of the respiratory tract with each breath. The expiratory
volume and residual volume decrease as the pregnancy progresses.
The inspiratory capacity increases during pregnancy. The increased
oxygen consumption in the pregnant client is 15% to 20% greater
than in the nonpregnant state.
90.
Answer: (A) Diet. Clients with gestational diabetes are
usually managed by diet alone to control their glucose intolerance.
Oral hypoglycemic drugs are contraindicated in pregnancy. Longacting insulin usually isnt needed for blood glucose control in the
client with gestational diabetes.
91.
Answer: (D) Seizure. The anticonvulsant mechanism of
magnesium is believes to depress seizure foci in the brain and

peripheral neuromuscular blockade. Hypomagnesemia isnt a


complication of preeclampsia. Antihypertensive drug other than
magnesium are preferred for sustained hypertension. Magnesium
doesnt help prevent hemorrhage in preeclamptic clients.
92.
Answer: (C) I.V. fluids. A sickle cell crisis during pregnancy is
usually managed by exchange transfusion oxygen, and L.V. Fluids.
The client usually needs a stronger analgesic than acetaminophen
to control the pain of a crisis. Antihypertensive drugs usually arent
necessary. Diuretic wouldnt be used unless fluid overload resulted.
93.
Answer: (A) Calcium gluconate (Kalcinate). Calcium
gluconate is the antidote for magnesium toxicity. Ten milliliters of
10% calcium gluconate is given L.V. push over 3 to 5 minutes.
Hydralazine is given for sustained elevated blood pressure in
preeclamptic clients. Rho (D) immune globulin is given to women
with Rh-negative blood to prevent antibody formation from RHpositive conceptions. Naloxone is used to correct narcotic toxicity.
94.
Answer: (B) An indurated wheal over 10 mm in diameter
appears in 48 to 72 hours. A positive PPD result would be an
indurated wheal over 10 mm in diameter that appears in 48 to 72
hours. The area must be a raised wheal, not a flat circumcised area
to be considered positive.
95.
Answer: (C) Pyelonephritis. The symptoms indicate acute
pyelonephritis, a serious condition in a pregnant client. UTI
symptoms include dysuria, urgency, frequency, and suprapubic
tenderness. Asymptomatic bacteriuria doesnt cause symptoms.
Bacterial vaginosis causes milky white vaginal discharge but no
systemic symptoms.
96.
Answer: (B) Rh-positive fetal blood crosses into
maternal blood, stimulating maternal antibodies. Rh
isoimmunization occurs when Rh-positive fetal blood cells cross into
the maternal circulation and stimulate maternal
antibody production. In subsequent pregnancies with Rh-positive
fetuses, maternal antibodies may cross back into the fetal
circulation and destroy the fetal blood cells.
97.
Answer: (C) Supine position. The supine position causes
compression of the clients aorta and inferior vena cava by the fetus.
This, in turn, inhibits maternal circulation, leading to maternal
hypotension and, ultimately, fetal hypoxia. The other positions
promote comfort and aid labor progress. For instance, the lateral, or
side-lying, position improves maternal and fetal circulation,
enhances comfort, increases maternal relaxation, reduces muscle

tension, and eliminates pressure points. The squatting position


promotes comfort by taking advantage of gravity. The standing
position also takes advantage of gravity and aligns the fetus with
the pelvic angle.
98.
Answer: (B) Irritability and poor sucking. Neonates of
heroin-addicted mothers are physically dependent on the drug and
experience withdrawal when the drug is no longer supplied. Signs of
heroin withdrawal include irritability, poor sucking, and restlessness.
Lethargy isnt associated with neonatal heroin addiction. A flattened
nose, small eyes, and thin lips are seen in infants with fetal alcohol
syndrome. Heroin use during pregnancy hasnt been linked to
specific congenital anomalies.
99.
Answer: (A) 7th to 9th day postpartum. The normal
involutional process returns the uterus to the pelvic cavity in 7 to 9
days. A significant involutional complication is the failure of the
uterus to return to the pelvic cavity within the prescribed time
period. This is known as subinvolution.
100.
Answer: (B) Uterine atony. Multiple fetuses, extended labor
stimulation with oxytocin, and traumatic delivery commonly are
associated with uterine atony, which may lead to postpartum
hemorrhage. Uterine inversion may precede or follow delivery and
commonly results from apparent excessive traction on the umbilical
cord and attempts to deliver the placenta manually. Uterine
involution and some uterine discomfort are normal after delivery.
1. After business trip to an underdeveloped country 3 weeks
ago, Chris is diagnosed with hepatitis A. In completing the
assessment, the nurse might expect which of the following
responses to be most likely associated with the client's
contracting of the disease?
a) I went hunting to forests and swarmed by mosquitoes
b) three months ago, I ate oysters in Kenya
c) I drank lemonade from the roadside while on this trip
d) my business partner is a hepatitis carrier
2. A client with viral hepatitis has no appetite, and food
makes the client nauseated. Which of the following nursing
interventions are appropriate?
a) explain that high fat diets are usually better tolerated
b) encourage foods high in protein
c) explain that the majority of calories need to be consumed
in the evening hours
d) monitor of fluid and electrolyte imbalance
3. Which of the following outcomes would the nurse expect

to find in the client who has developed no complications


from viral hepatitis?
a) decreased absorption of Vitamin K in intestine
b) increasing prothrombin time values
c) presence of asterexis
d) decrease in AST
4. The client has an order to receive purified protein
derivatives (PPD) 0.1ml intradermally. The nurse
administers the medication utilizing a tuberculin syringe
with a:
a) 26G, 5/8 inch needle inserted almost parallel to the skin
with the bevel side up
b) 26G, 5/8 inch needle inserted at a 45deg. angle with
bevel side up
c) 20G, 1 inch needle inserted almost parallel to the skin
with the bevel side up
d) 20G, 1 inch needle inserted at a 45deg. angle with bevel
side up
5. The nurse reading the PPD skin test for a client with no
documented health problems. The site has no induration
and a 1mm area of ecchymosis. The nurse interprets that
the result is:
a) positive
b) negative
c) needs to be repeated
d) borderline
6. The client who is HIV positive has had a Mantoux test.
The result shows a 7mm area of induration. The nurse
evaluates that this result as:
a) negative
b) borderline
c) positive
d) needs to be repeated
7. The nurse reads the client's Mantoux skin test as
positive. The nurse notes that the previous tests were
negative. The client becomes upset and asks the nurse
what this means. The nurse response is based on the
understanding that the client has:
a) no evidence of tuberculosis
b) client has systemic tuberculosis
c) pulmonary tuberculosis
d) exposure to tuberculosis
8. The nurse is caring for the client diagnosed
with tuberculosis. Which of the following assessments, if
made by the nurse, are not consistent with the usual clinical
presentation of tuberculosis?

a) nonproductive or productive cough


b) anorexia and weight loss
c) chills and night sweats
d) high grade fever
9. The nurse is teaching the client with TB about dietary
elements that should be increased in the diet. The nurse
suggests that the client increase intake of:
a) meats and citrus fruits
b) grains and broccoli
c) eggs and spinach
d) potatoes and fish
10. The client with TB, whose status is being monitored in
an ambulatory care clinic, asks the nurse when it is
permissible to return to work. The nurse replies that the
client may resume employment when:
a) two sputum cultures are negative
b) five sputum cultures are negative
c) a sputum culture and a chest x-ray is negative
d) a sputum culture and a PPD test are negative
11. The client with TB is being started on anti-TB therapy
with Isoniazid (INH). The nurse assesses that which of the
following baseline studies has been completed before
giving the client the first dose?
a) coagulation times
b) electrolytes
c) serum creatinine
d) liver enzymes
12. The nurse has given the client
with tuberculosis instructions for proper handling and
disposal of respiratory secretions. The nurse evaluates that
the client understands the instruction if the client verbalizes
to:
a) wash hands at least four times a day
b) turn the head to the side if coughing or sneezing
c) discard the used tissues in the plastic bag
d) brush the teeth and rinse the mouth once a day
13. The client has been taking Isoniazid for month and a
half. The client complains to the nurse about numbness,
paresthesias, and tingling in the extremities. The nurse
interprets that the client is experiencing:
a) small blood vessel spasm
b) impaired peripheral circulation
c) hypercalcemia

d) peripheral neuritis
14. The client with AIDS is diagnosed with cutaneous
Kaposi's sarcoma. Based on this diagnosis, the nurse
understands that his has been determined by which of the
following?
a) appearance of reddish blue lesions noted on the skin
b) swelling in the lower extremities
c) punch biopsy of the cutaneous lesions
d) swelling in the genital area
15. Which of the following individuals is least likely at risk
for the development of Kaposi's sarcoma?
a) a man with a history of same sex partners
b) a renal transplant client
c) a client receiving antineoplastic medications
d) an individual working in an environment where exposure
to asbestos exists
16. The clinic nurse assesses the skin of the client with a
diagnosis of psoriasis. Which of the following
characteristics is not associated with this skin disorder?
a) discoloration and pitting of the nails
b) silvery white, scaly patches on the scalp, elbows, knees,
and sacral regoins
c) complaints of pruritus
d) red purplish, scaly lesions
17. The nurse is assigned to care for a client with herpes
zoster. Which of the following characteristics does the
nurse expect to note when assessing the lesions of this
infection?
a) a generalized body rash
b) small, blue-white spots with a red base
c) a fiery red, edematous rash on the cheeks
d) clustered skin vesicles
18. The nurse manager is panning in the clinical
assignments for the day. Which of the following staff
members would not be assigned to the client with herpes
zoster?
a) the nurse who had chicken fox during child hood
b) the new nurse who never had german measles
c) the nurse who never had enteric fever
d) the new nurse who had flu vaccine
19. The nurse plans to instruct a client with candidiasis
(thrush) of the oral cavity about how to care for the disorder.
Which of the following is not a component of instructions?
a) to rinse the mouth four times daily with a commercial

mouthwash
b) to avoid spicy foods
c) to avoid citrus fruits and hot liquids
d) to eat foods that are liquid
20. The clinic nurse inspects the skin of client suspect of
having scabies.Which of the following
assessment findings would the nurse note if this disorder
were present?
a. the appearance of vesicles or pustules with a thick,
honey colored crust
b) the presence of white patches scattered about the trunk
c) multiple straight or wavy, threadlike lines beneath the
skin
d) patchy hair loss and round neck macules with scales

ANSWERS
1) C ..... 2) D ..... 3) D ..... 4) A ..... 5) B
6) C ..... 7) D ..... 8) D ..... 9) A ..... 10) A
11) D ..... 12) C ..... 13) D ..... 14) A ..... 15) C
16) D ..... 17) D ..... 18) B ..... 19) A ..... 20) C
Situation: Bobby, 3 years old, from a family of four older
siblings, a mother and a father, was admitted to the hospital
with a temperature of 101 F, a sore throat of four days
duration, with difficulty of swallowing, and a moderate
cervical adenitis. A diagnosis of pharyngeal diphtheria was
made.
61. The best means for providing complete bed rest for the
child include:
a) feeding the patient
b) giving sedation
c) providing quiet diversion
d) giving complete physical care
62. A positive nose and throat culture in a member of the
family in whom no signs of the disease are present would
indicate:
a) that he is developing diphtheria
b) that he is a carrier
c) that he is susceptible to diphtheria
d) that he is immune to diphtheria
Situation: Charles, an 8 year old boy, one of several
siblings in the family, is kept home by his mother because
he has a temperature of 100.2F and a history of hacking

cough for two weeks. A definite whoop has been heard and
the doctor made a diagnosis of whooping cough. The
visiting nurse is to instruct the mother in proper care of this
patient.
64. This young boy should be:
a) kept in warm, dry room
b) placed in a sunny room
c) kept in a room with a temperature of 60F
d) allowed free run of the house
65. The organism causing whooping cough is:
a) hemolytic streptococcus
b) hemophilus influenza
c) hemophilus pertussis
d) streptococcus albus
66. Whooping cough is most contagious:
a) in the paroxysmal stage
b) in the catarrhal stage
c( in the febrile stage
d) in the incubation stage
67. During the second stage of the disease the
characteristic paroxysm is frequently accompanied by:
a) involuntary micturation
b) spontaneous epistaxis
c) inspiratory whoop
d) expulsion of mucous from the trachea
68. The diagnosis of pertussis is confirmed by which of the
following tests?
a) nose and throat culture
b) nasopharyngeal culture
c) "rapid" culture of nose and throat organism
d) sputum culture
69. Oxygen is used in whooping cough for which of the
following reasons:
a) prevents the pertussis organism from multiplying
b) seems to lessen the occurrence of paroxysms
c) reduces the danger of convulsions
d) prevents atelectasis
70. One of the first signs of mumps is:
a) pain in the back
b) diarrhea
c) pain in the region of the parotid gland
d) otitis media

71. A nurse should suspect meningitis if a patient exhibited


characteristic position which would be:
a) knees pulled up
b) joints of hands flexed
c) backward arching of the spine
d) head pulled forward onto the chest
72. The nurse should be alert for complications
of meningitis. Which of the following are the most common?
a) hydrocephalus
b) damage of the optic nerve
c) deafness
d) myocarditis
73. Which of the following is not a part of the role of the
infection control department?
a) reviewing infection control policies and procedures
b) performing venipunctures on clients to obtain blood
cultures
c) investigating outbreaks of infection
d) providing input regarding the selection of patient care
products
74. The primary human reservoir for Staphylococcus
aureus is:
a) adult genitalia
b) blood
c) skin
d) feces
75. The best example of medical asepsis is:
a) hand washing
b) use of sterile instruments
c) drawing medication into syringe
d) autoclaving
76. Which of the following is not a symptom of localized
infection?
a) redness
b) edema
c) restricted movement of the affected body part
d) lymph node enlargement
77. All of the following are part of the nurse's role in
infection control except:
a) recognizing the signs and symptoms of infection
b) collecting specimens of drainage from infected wound

sites
c) deciding upon the appropriate to be antibiotic to be
administered to the client
d) supporting the client's body defense mechanisms
78. When giving a history, the patient reports that he had
discharge from his penis and burning when he urinates. A
nurse should recognize these as indicative of:
a) herpes
b) chlamydia
c) syphilis
d) HIV infection
79. When insect serves as the host for transmission, this is
called
a) airborne
b) vectorborne
c) vehicle
d) contact
80. When a health care worker is washing her/his hands,
which of the following observations made by the nurse
would indicate that the worker understands the principle of
proper hand washing?
a) washes hands prior to removing gloves
b) wash hands for 5 seconds
c) rinses hands with fingers pointed up
d) removes rings before washing

ANSWERS
61) B ..... 62) B ..... 63) ..... 64) A .....65) C
66) B .....67) C ..... 68) A ..... 69) B ..... 70) C
71) D ..... 72) C ..... 73) D ..... 74) C ..... 75) A
76) D ..... 77) C ..... 78) A ..... 79) B ..... 80) D
Infectious Agent or its toxic products - AGENT
Directly or Indirectly - MODE OF TRANSMISSION
Person, Animal or Intermediate Vector HOST
Environment - ENVIRONMENT
ECOLOGIC TRIAD OF DISEASE
Agent element, substance, animate or inanimate
that may serve as stimulus to initiate a disease
process
Host organism that provides nourishment for

another organism
Environment physical (climate), biological (plants
& animals)
CONTAGIOUS VS. INFECTIOUS
Contagious
Diseases that are easily spread directly transmitted
from person to person (direct contact) through an
intermediary host
Infectious
Diseases that caused by a pathogen not transmitted
by ordinary contact but require a direct inoculation
through a break in the skin or mucous membrane.
NOTE: ALL CONTAGIOUS DISEASE ARE INFECTIOUS
BUT INFECTIOUS DISEASE IS NOT ALWAYS
CONTAGIOUS
What is Infection?
INFECTION - "the state or condition in which the
body or part of the body is invaded by a pathogenic
agent ( bacteria, virus, parasites etc.) which under
favorable conditions multiplies and produces effects
which are injurious"
Infectious Agent
A. RESIDENT ORGANISMS
deeply seated in the epidermis, not easily removed
by simple handwashing,
Ex: Staphylococci
B. TRANSIENT ORGANISM
represent recent contamination,
survive for a limited period of time, acquired during
contact with the infected colonized patient or
environment,
easily removed by good handwashing
Ex: ( Klebsiella & Pseudomonas)
Infectious Agent
Bacteria heama organism, systemic
Virus nuero organism, systemic
Fungi skin organism, local
Protozoa GI organism, local
Infectious Agent
FACTORS THAT AFFECTS THE AGENT TO
DEVELOP A DISEASE
Pathogenicity ability to cause a disease
Infective dose no of organism to initiate infection
Virulence ability to enter or move through tissues
Specificity ability of the organism to develop
antigens

STAGES OF INFECTIOUS PROCESS


Means of Transmission
1. CONTACT - most common means of transmitting
microorganisms from one person to another.
A. Direct Contact (person to person)
occurs when one person touches another
best vehicle is the Hands especially those of the
Health Care workers
Indirect Contact (inanimate object)
- occurs when a person touches an inanimate object
contaminated by an infected patient
2. AIRBORNE
- droplet, dust, organisms in env.
3. VECTOR - insects or animals
4. VEHICLE
- food (salmonella), water (shigellosis), blood (Hepa
B), medication ( contaminated infusion)
PREVENTION OF
COMMUNICABLE DISEASE
Prevention is worth a pound than cure
PREVENTION OF
COMMUNICABLE DISEASE
Health Education primary role of the nurse
Specific Protection- handwashing, use of protective
devices
Environmental Sanitation clean and conducive for
health
Definition of Prevention
Actions aimed at eradicating, eliminating, or
minimizing the impact of disease and disability. The
concept of prevention is best defined in the context
of levels, traditionally called primary, secondary,
and tertiary prevention
A Dictionary of Epidemiology, Fourth Edition
by John M. Last
Prevention of Needlestick Injuries
Dispose Used Needles in Puncture Proof Needle
Containers
Dont Recap Needles (Unless using the One-handed
Technique)
Use Gloves When Handling Needles (Wont Prevent
Injuries but May Lessen Chance of Transmitting
Diseases)

CONTROL OF C0MMUNICABLE DIESEASE


1. Notification
2. Epidemiological Investigation
3. Case finding; early dx and prompt treatment
4. Isolation and Quarantine
5. Disinfection; disinfestation
6. Medical Asepsis
a. Handwashing
b. Concurrent disinfection
c. Personal protective equipments (PPEs)
d. Barrier Cards/Placarding
Objectives of CCD
Restoration of health, reduce deaths and disability
Interpretation of control measures to IFC for
practice to prevent spread of CD.
Promotion of health and prevention of spread of CD
Diseases that require weekly monitoring:
1. Acute flaccid paralysis (AFP) polio
2. Measles
3. Severe acute diarrhea (SAD)
4. Neonatal tetanus
5. AIDS
Diseases that require reporting w/in 24 hrs
1. Acute flaccid paralysis (AFP) polio
2. Measles
Diseases targeted for eradication
1. Acute flaccid paralysis polio
2. Neonatal tetanus
3. Measles
4. Rabies
Epidemiology
Study of the occurrence and distribution of diseases
in the population
Patterns of occurrence of disease frequency of
disease occurrence
Sporadic
On and off occurrence of the disease
Most of the time it is not found in the community
One or two cases that occur are not related
Endemic
Persistently present in the community all year round
Ex: malaria in Palawan
Epidemic
An unexpected increase in the number of cases of
disease
Pandemic
Epidemic of a worldwide proportions
Time Related Patterns of Occurrence

cyclical variation
a periodic increase in the number of cases of a
disease
a seasonal disease, an increase is expected or there
is usual increase- dengue fever during rainy seasons
are increased but it is not considered an epidemic
because it is expected to rise at this particular time
hot spot-a rising increase that may lead to an
epidemic
Time Related Patterns of Occurrence
Short time fluctuation
A change in the frequency of occurrence of a
disease over a short period of time
Maybe (+) or (-)
Secular variation
A change in the frequency of occurrence of a diseae
taking place over a long period of time
Ex: a.) the change in the pattern of occurrence of
polio after being eradicated in 2000, then sudden
repport of cases in 2001 due to mutant restraints.
b.) small pox virus-eradicated in 1979 (last case
reported) and no another incidence as of today
Types of Epidemiology
Descriptive Epidemiology - concerned with disease
frequency & distribution
Analytic Epidemiology
Is a study of the factors affecting occurrence and
distribution of the disease.
Ex. Epidemiologic investigation
Therapeutic/Clinical
Study of the efficacy of a treatment of a particular
disease
Ex. Clinical trial of a newly proposed therapeutic
regimen
Evaluation Epidemiology
Study of the over-all effectiveness of a total/
comprehensive public health program.
Ex. Evaluation of the under five clinic
Note: We make use of the epidemiology in CHN in
order to come up a community diagnosis and also
to determine the effectiveness of a particular
treatment
Types of Epidemiologic Data
Demographic data
Demography is the study of population groups
Ex. Population size and distribution
Vital Statistics
Environmental data
Health services data
Ex. Ratio between nurse and the population being
served

Ex. Degree of utilization of health facility/ service


Epidemiologic Investigation
1st step- Statement of the problem
2nd step- Appraisal of facts describing the
epidemic in terms of time, place, person.
3rd step- formulation of hypothesis
4th step-Testing the hypothesis
5th step- Conclusion and recommendation
TERMS
Disinfection pathogens but not spores are
destroyed
Disinfectant substance use on inanimate objects
Concurrent disinfection ongoing practices in the
care of the patient to limit or control the spread of
microorganisms.
Terminal disinfection practices to remove
pathogens from the patients environment after his
illness is no longer communicable
FACTORS AFFECTING ISOLATION
Mode of Transmission
Source
Status of the clients defense mechanism
Ability of client to implement precautions
ISOLATION
EPI
Launched by DOH in cooperation with WHO and
UNICEF last July 1976
Objective reduce morbidity and mortality among
infants and children caused by the six childhood
immunizable diseases
PD No. 996 (Sept. 16, 1076) Providing for
compulsary basic immunization for infants and
children below 8 y/o
PP No. 6 (April 3, 1996) Implementing a United
Nations goal on Universal Child Immunization by
1990
RA 7846 (Dec. 30, 1994) immunization hepa B
PD No. 4 (July 29, 1998) Declaring the period of
September 16 to October 14, 1998 as Ligtas Tigdas
Month and launching the Phil Measles Elimination
Campaign
Legislation, Laws affecting EPI
Proclamation No. 46 polio eradication project
Proclamation No. 1064 AFP surveillance

Proclamation No. 1066 National Neonatal Tetanus


Elimination Campaign
EPI
BCG - TB
DPT Diptheria, Pertussis, Tetanu
OPV - Poliomyletis
Hepatitis B
Measles
Immunization
Contraindications
-conditions that require hospitalization
For DPT 2 and 3 history of seizures/ convulsions
within 3 days after the first immunization with DPT
Nursing responsibility: ask how the child reacts to
the first dose
For infant BCG clinical AIDS
The following conditions are NOT contraindications:
Fever up to 38.5 C
Simple or mild acute respiratory infection
Simple diarrhea without dehydration
Malnutrition (it is indication for immunization)
Schedule of immunization
Infant BCG
0 to 11 months or 0 to 1 year
at birth
0.05 ml (dose) ID, right arm
School entrance BCG
When the child enters Grade 1 with or without scar
on the right arm then still go on with the
vaccination except if he is repeating Grade 1
Schedule of immunization
DPT
3 doses, 4 weeks or 1 month interval
Target age: 1 to 11 months but child is eligible up
to 6 years
If 7 years old and above DT only not P
0.5 ml, IM, vastus lateralis
Schedule of immunization
OPV
3 doses, 4 weeks/1 month
Target population: same as above, eligibility until
Grade 6
2-3 drops, oral route
*Feb 8-March 8: Oplan Polio Revival Drive
No side effect, but advise the mother to avoid

feeding the child for 30 minutes after the vaccine, if


vomits within the 30 minute period, repeat the
vaccination
Schedule of immunization
Hepa B
3 doses, 4 weeks
Can be given at birth
Target age 1 to 11 months
0.5 ml, IM, vastus lateralis
Patient may experience local tenderness
Schedule of immunization
Measles
9 to 11 months
Most babies have protection because of maternal
antibodies thus this vaccine is given at 9 months
because the time where the maternal antibodies
wear off, other virus if it still active it will kill the
vaccine
0.5 ml, subcutaneous, any arm
Measles
Fever and measles rash lasting for 1 to 3 days
within 2 weeks after immunization (modified
measles)
Immunization
Fully Immunized Child
when he received all the antigens that should be
given in the first year of life (1 dose BCG, MV; 3
doses DPT, OPV, HB)
Completely Immunized Child
All vaccines given but went beyond 0ne year of age

How do children acquire HIV infections?


*contact with infected mother at birth (90% cases infant)
*Sexual contact with an infected person
*Use of contaminated needles or contact with infected
blood.

How is AIDS cause?


Retrovirus known HIV-1 attacks lymphocytes ( white blood
cells that protect against disease)

How do the doctor determined a person with HIV


treatment?
CD4 cell count

What test is use to test for kids with HIV?

Enzyme linked immunosorbent assay (ELISA) or


Western blot

What are the manifestation of HIV in infancy?


Vague, failure to thrive, lymphadenopathy, chronic
sinusitis, and non-response to the treatment of infections.

What is the opportunistic infection with infants/ HIV?


oral thrush
Pneumocystis carinii or jiroveci
Pneumonia
herpes viruses
cytomegalovirus

What are some serious bacteria infection with kids/ HIV?


meningitis, impetigo, and urinary tract

What are the goal for HIV?


slow virus down
Prevent opportunistic infections
provide adequate nutrition
supportive therapy

What are some common test for HIV


CDC 4 count
Western Blot Test
Rapid HIV test
EIA
ELISA

What are the most common STI in adolescents?


HPV ( human papillomavirus)
can cause cervical cancer

When does infant become clear of the antibodies?


about 15 months

If a pregnant have STI what happen to the baby?


blindness, birth defects or death

What do a nurse need to with a adolescence who has STI?


Asses their level of knowledge and provides information at
an understandable level.

What are the manifestation of HPV


clusters of flesh colored cauliflower shape warts in the
perineal area. Contact with warts spreading the condition.

How to prevent HPV?


2 vaccines Guardasil-6/11 16/18
bivalent Cervatrix 16/18
3 dose regime
9 and 26 years of age

What are the treatments of warts?


cryotherapy or electrocautery
topical with podofilox gel
sinecatechins oinment

What are the nursing goal for kids with STI?


-provide anticipatory guidance
-prevent infection
-identify early symptoms and provide prompt treatment if
infections occur
-To prevent sequelae

What is the nursing intervention for Sequelae?


Discuss the possible complication of specific disorders such
as birth defects and infertility

What are the nursing intervention for medication with a


child with STI?
If taking tetracycline 1 hour before or 2 hour after meals
( empty stomach) avoid dairy products, antacids, iron, and
sunlight.

Which communicable disease is highly contagious?


Smallpox (variola).
No longer recommended unless bio terrorism event

What vaccination should a child should not receive if they


have HIV?
Varicella and MMR live vaccines.

What organ does Hep B affect?


Liver
Incubation period 30-180 days
S/S- fever, anorexia, headache, abdominal pain, malaise,
jaundice, dark urine, chalklike stool

What are the nursing intervention for a child with Hep B?


Prevent contact with blood or blood products
Identify high risk mothers and newborns.
Educate concerning need for vaccination.

What to know about whooping cough?


s/s- cold and cough ( noisy gasp for air that creates a
whoop)
5- 21 days for incubation
vaccination serious of DTAP
Treatment- administer erythromycin. cool mist tent and
antibiotics

What are the nursing intervention with a child with


whooping cough?
* Droplet precautions until 5 days after antibiotics
* bed rest/ abdominal support during coughing spell
* Re-feed child if she vomits
*observe for airway obstruction and o2 stat

What medicine is giving if a child have TB?


isoniazid (INH)
rifampin
pyrazinamide all for several months

What to know about TB?


low grade fever, malaise, anorexia, weight loss, cough,
night sweats.
asymptomatic adenopathy pneumonia, and positive test
2-10 weeks airbone and droplet precaution.

What are some important factors in host resistance to


disease?
intact skin and mucous membrane
phagocytes
function of the immune system

What is use to prevent lockjaw?


tetanus serum/ passive immunity

What is example of a disease spread by a carrier?


Typhoid fever

What is the purpose of medical aseptic techniques


prevent the spread of infection from on person to another

Contact precautions
skin to skin
Example: RSV Hep A, impetigo

Airborne infection
TB, chicken pox ( Varicella) , measles ( rubeola),

What does newborn infants presence when born?


passively acuired

What immunization is giving to a newborn before


discharge?
Hep B

Droplets precaustion
when a patient cough and sneezing
Influenza, Pertussis.

When giving MMR, varicella, influenza nurse should check


for what?
Allergic to eggs.

Why might a parent does not want her child to receive


vaccines?
contains heavy metals and antibiotic or animal product

What can a nurse use to reduce pain before giving a shot?


vapocoolant spray or EMLA cream
What is the criteria for diagnosing full blown AIDS?
person is HIV positive and:
-person's T cell count is 200 or below AND/OR
-presence of an opportunistic infection

How does the virus attack the body?


attaches and takes control of the T cells in order to produce
more HIV; weakens the body's ability to fight infection

How is HIV transmitted?

-having sexual contact with an infected person


-sharing needles with an infected person
-an infected female who gives birth

Which cells does HIV attack?


T cells

What fluids transmit HIV?


-semen
-blood
-vaginal fluid
-breast milk

How do we test for HIV?


blood test for the antibodies to HIV; may take 3-6 months
for a positive result

How is HIV/AIDS treated?


AZT=slows rate at which HIV multiplies; protease inhibitors
decrease the amount of HIV and increase T cell count

How can you reduce your risk?


-abstinence
-monogamy
-don't share needles
-follow universal precautions

Can you develop an immunity to STD's?


No

How are most sexually transmitted diseases passed?


intimate sexual contact, vaginal/oral/anal sex,
outercourse(close genital contact), mucus membrane to
mucus membrane

What are infectious/communicable diseases?


transmitted from person to person by microorganisms
(pathogens) or other toxins

What are the 4 main types of pathogens?


virus
bacteria
fungus
protozoa

What are places of exit and entry for pathogens(body


orifices)?
eyes, nose, ears, mouth, penis, vagina, anus, broken skin

What are the 5 methods of transmission?


-direct contact
-through air
-contact with contaminated objects
-contact with animals, insects that are vectors or carriers of
disease
-contaminated food and water

What are the 4 body defenses?


skin
cilia
mucus
immune system

What are the 2 types of immunity?


active and passive
(short term and long term)

What are the 5 stages of infectious disease?


incubation stage
prodromal stage
acute stage
recovery stage
convalescence stage

What are the 6 ways to prevent communicable disease?


-kill the agent/pathogen
-prevent contact with reservoir
-prevent escape
-prevent transmission
-block places of entry
-resistant host
how many people in the sub-saharan desert are infected
with HIV
25 million

what is the average lifespan of a person living in the subsaharan desert


mid 40's

how many new cases a year in U.S.


40,000

how many new cases a day in Africa


7,000

what drugs can be used to extend a persons with HIV's


life by over 20 years
ARV

what percent of infected children recieve medical care


5-10 percent
communicable disease
an illness that can spread, usually from person to person

pathogen
an agent that causes a communicable disease

ear infections,whooping cough


What are some diseases caused by bacteria?

viruses
______ are smaller than bacteria and cannot reproduce on
their own.

cancer
the uncontrolled growth of abnormal cells

person's behavior
A ________ can affect the risk of developing cancer.

cured
Some noncommunicable diseases can be _____.

arthrosclerosis
a disease in which fatty substances build up on the inside
of arteries

colds, mumps, chicken pox, flu,


What are some diseases caused by viruses?

fungi

Which kind of disease causes athlete's foot?

protist
Which kind of disease causes malaria?

person to person, contaminated hands , sneezing,


coughing, contaminated water and animal bites
How can communicable diseases spread?

skin
What is a major line of defense to keep pathogens out?

vaccine
A _______ is a dose of weakened or killed pathogens of one
kind.

cardiovascular disease
a disease of the heart and blood vessels

emphysema
a disease of the lungs in which air sacs are damaged

early diagnosis
What is the key to curing, controlling, damage done by
noncommunicable diseases?

immunity
the body's resistance to a disease through the presence of
antibodies

white blood cells


What destroys the pathogens?

antibody
a substance made by certain white blood cells that
attaches to a pathogen and makes it harmless

vaccines
What is one way to develop immunity without getting sick?

smoking
What is the main cause of emphysema?

antibiotic
a medicine that kills bacteria

bacteria
Doctors give you an antibiotic when you have a disease
caused by a ________.

non-communicable disease
an illness that cannot spread and is not caused by a
pathogen

heredity
What is the main causes of noncommunicable diseases?

allergy
harmful reaction to certain substances

asthma
allergy that affects the respiratory system and may result
in breathing difficulty and wheezing

infectious dieseases
Communicable diseases are sometimes called _______,
infections, or contagious.

bacteria, viruses, protists, and fungi


Pathogens include ______, ______, _____, and ______. They
are sometimes called germs.

nose and mouth


Pathogens can enter the body through the _____and _____.

allergic reaction
All of the following are examples of _______:
itchy skin, rash, hives, stomach upset, swelling of lips or
tongue, sneezing

pollen
What triggers hay fever?
Pathogens
Disease causing microorganisms

Virus

Smallest of all pathogens that need a host cell to


reproduce. They invade cells, multiple inside the cell to the
point it explodes and releases millions more.

Bacteria
One-celled organisms that multiply rapidly and make up
sick by releasing toxins.

Fungi
Feed on both living and dead organic matter. Most are
harmless but some can cause infections such as ringworm
and athlete's foot.

Protozoa (Protists)
Single-celled organisms that can have parasitic like
characteristics and may contain flagellum.

Symptoms
A physical or mental feature- fever, swelling, rash, aches,
runny nose, cough, headache- that indicates a specific
infection

Contagious Period
Length of time that a disease can spread from person to
person

Infectious Diseases
Often called communicable diseases as they can be
passed from one person to the next.

Food, Water, Soil


When food is not prepared properly pathogens can survive
and enter the body orally, drinking or using contaminated
water without purifying it first can spread pathogens via
ingestion, pathogens in soil can get into body via open
cuts or ingestion of improperly prepared or clean food,
sharing food & drinks with infected people can also spread
pathogens

Infected animals
When pathogens spread diseases through bites

Direct Contact (Infected Individuals)

Occurs when there is physical contact with body fluidssaliva, blood, vomit, urine, feces, nasal secretionsbetween an infected person and a susceptible person via
touching, kissing & other sexual contact, direct contact
with body fluids, or contact with open wounds.

Indirect Contact (Contaminated objects)


No human-to-human contact is involved, spread when an
infected person sneezes or coughs, sending infectious
droplets into the air. A person can inhale the infectious
droplets, the infectious droplets can land directly on a
person or on fomites that people touch the with their
hands, and then touch their eyes, nose or mouth.

Agent
Germ that produced infection- Bacteria, Virus, Fungi,
Protozoa

Reservior
Place germ can live such as humans, animals, soil, objects,
air, water and food

Place of Exit
Where germ leaves reservoir such as through the mouth,
nose, rectum, genitals and wounds

Method of Transmission
How germ travels from point A to point B such as through
saliva, sneeze, wind, feces, urine, blood, water, soils,
kissing, infected sore

Place of Entry
Place where germ enters a person such as breathing in
through nose & mouth, germs in food- mouth and
digestive, cuts in skin, direct contact

Susceptible Host
Things that make a person more likely to get a disease.
These include behaviors (taking care of cuts, nutrition,
exercise, sleep, hygiene, abstaining from smoking and
drinking), conditions (receiving Chemo, AIDS/HIV+),
environment and inherited factors.

Skin

Outer layer is tough and dead which keeps germs from


entering. Cells also fall off taking germs with them

Hair
Around eyes and nose they trap germs and keep germs
from entering the body. The trachea has cilia that keep
germs from entering the lungs.

Tears
Wash away germs from eyes

Mucus
Sticky substance that exists in the nose & other places that
traps germs and contains chemicals defenses that attack
and destroy germs.

Stomach acid & Saliva


These substances have chemicals that kill most of the
germs that enter your body

Antibodies
Specialize proteins that lock onto specific antigens and
mark invaders for other cells to destroy

Leukocytes
WBC or White Blood Cells

Phagocytes
Attach/engulf invaders

Lymphocytes
Remember & help destory

T lymphocytes
Destroy invaders

B lymphocytes
Seek out targets, produce antibodies, and send defenses

Antigens
Surface structures or markings that antibodies attach onto

Innate Immune System

Comprised of your body's physical barriers plus


inflammation due to increased lymph and blood flow to
areas of infection where phagocytes (macrophages)
devour toxins or pathogens. If the infection becomes
widespread, a person may get a fever which signals the
body to produce more WBC and slows down the
pathogens.

Adaptive Immune System


This type of immunity develops throughout our lives and
involved the WBCs. This can happen through exposure to
diseases and via vaccination.

Macrophage
Engulf invaders and trigger immune response

Vaccination
Contains weakened or dead strain of the organism/virus
which is injected into the body to prepare B-Cells and TCells to remember and destroy it quickly thus giving the
person immunity towards a specific disease and prevents
them from getting sick.

Antibotics
Treatment for bacterial infections
What process enhances topical
corticosteriods efficacy?
Soaking in water for 10-15 minutes

Which corticosteroid is more potent than


the other? ointment or cream?
Ointment more potent

What increases the topical corticosteroid


potency dramatically?
covering with water impermeable barrier
for at least 4 hours.

Caution should be used placing topical


corticosteriods to areas of thin skin (face,
scrotum, vulva, skin folds). T/F
T

How much topical corticosteroid does it


take to cover an adult?
20-30g

Does systemic absorption occur with


topical corticosteroid?
Yes, but adrenal suppression, DM, HTN,
osteoporosis are very rare

Which is the best product t moisturize dry


skin?
Vanicream is allergen free and can be used
if allergic contact dermatitis is suspected.

What can be used as a drying agent for


weepy dermatoses?
water applied as repeated compresses for
15-30 minutes alone or with aluminum
salts

List some topical antipruritics.


Lotions that contain camphor and menthol
Lotions that contain pramoxine
hydrochloride
Hydrocortisone 1% or 2.5% for
inflammation
Doxepin may cause drowsiness
Capsaicin for neuropathic itch
Ice in a plastic bag covered by thin cloth

What is the med of choice for pruritus


when due to histamine such as in
urticaria?
H1 blockers-antihistamines (Hydroxyzine
25-50 mg nightly)

Antihistamines are of limited value for


treatment of pruritus associated with
_____ ____ _____.
inflammatory skin disease

List agents that may treat pruritus.


1. Antidepressants (doxepin, mirtazapine,
paroxetine)
2. Gabapentin, pregabalin, duloxetine

(centrally or peripherally act on neurons)


3. Aprepitant and opioid antagonists
(Naltrexone, butophenol) is effective on
select patients.

What strength SPF should fair


complexioned people use?
SPF 15, better 30-40 every day

Which strength SPF gives protection


against UVA as well as UVB?
SPF >30

Melanocytic Nevi (Normal Mole):


Borders.
Color.
well defined border, single shade of
pigment from beige or pink to dark brown

Define a normal mole.


well demarcated, symmetric uniform in
contour and color

Moles should be removed only if they are


suspected of being melanomas. T/F
T

Describe the criteria for atypical nevi.


1. >6mm
2. ill defined, irregular border
3. Irregularly distributed pigmentation

Melanoma is increased in what


populations?
1. patients with 50 or more nevi with one
or more atypical moles and one mole at
least 8 mm or larger
2. Patients with a few to many definitely
atypical moles
3. Familial melanoma (numerous atypical
nevi and f/h of 2 first degree relatives with
melanoma)-risk of developing melanoma is
50% by age 50.

In patients that are at increased risk for


melanoma, what is the follow up?
education and regular (6-12 month) follow
up

Describe blue nevi.


-small slightly elevated, blue-black lesions
that favor dorsal hands.
-Common in Asians.
-if present without change for many years,
may be considered benign

When do blue-black papules and nodules


have to be ruled out for nodular
melanoma?
when they are new or growing

Describe lentigines. B) How do they


appear on the upper back? C) What do
they look like if they are benign? D) How
treated?
-appear in sun exposed areas, particularly
dorsa of hands, upper back and upper
chest, starting 4th or 5th decade of life an
associated with photoaging and estrogen
and progesterone use
B) Have irregular border
C) pigmentation is homogeneous,
symmetric and flat
D) Topical 0.1% Tretinoin, 0.1% tazarotene,
2% 4-hydroxyanisole with 0.01% tretinoin,
laser or cryo

Describe Seborrheic Keratoses. B) How


treated?
Benign plaques
beige to brown or even black
3-20 mm
velvety or warty surface
appear to be stuck or pasted on skin
common esp. elderly
B) Frozen with liquid nitrogen or curetted if
they itch or inflamed, no treatment needed

A pigmented skin lesion with recent


change in appearance should be a
suspect _______ _____.
malignant melanoma

What are the essentials of diagnosis for


malignant melanoma?
1. flat or raised
2. suspect in any pigmented skin lesion
with recent change in appearance.
3. Varying colors, including red, white,
black, and bluish.
4. Borders typically irregular

What is the single most important


prognostic factor in malignant melanoma?
related to thickness, if <1 mm=95% ten
year survival

What does ABCDE stand for?


A-Asymmetry
B=Border
C=color variation
D=Diameter > 6 mm
E=evolution (history of changing mole)

What is the most important history a


patient can give regarding a mole's
appearance?
changing appearance

A good practice is to refer suspicious


______ lesions.
pigmented

Describe acral lentignious melanoma.


dark, irregularly shaped lesions on the
palms and soles.
Can be broad solitary, darkly pigmented
longitudinal streaks in the nails.

What is the treatment for melanoma?


Excision, requires referral

What medication can reduce the risk of


progression of high risk melanoma?
BB

In general, when treating with topical


corticosteriods, how will you determine
the potency you use?
Appropriate to the severity of the
dermatitis. Begin with Triamcinolone 0.1%
or a stronger corticosteroid then taper to
hydrocortisone or another slightly stronger
mild corticosteroid (alclometasone,
desonide).

What is necessary to avoid rebound flares


of dermatitis that may follow their abrupt
cessation?
tapering the corticosteriod

Which corticosteriods are safe on face


and eyelids?
Protopic (tacrolimus) and Elidel
(pimecrolimus)

Tacrolimus and pimecrolimus is


contraindicated in who?

HIV
iatrogenic immunosuppression
prior lymphoma

What is the maintenance treatment of


atopic dermatitis?
constant application of moisturizers to
prevent flares.
topical anti inflammatories only on
weekends or three times weekly can
prevent flares.

how much area does one FTU treat?


One FTU is enough to treat an area of skin
twice the size of the flat of an adult's hand
with the fingers together.

How many FUTs equal 1 g topical steroid?


Two FTUs are about the same as 1 g of
topical steroid.

What are the 5 characteristics of skin


cancer?
1. asymmetry
2. border irregularity
3. color
4. 1/4 inch diameter
5. evolution

A Vitamin D level above ______ protects us


from the sun and prevents skin, breast
and colon cancers.
50

Which type is best to treat psoriasis?


Ointments or creams?
ointments because they provide a barrier
and moisturize whereas creams can dry it
out

What body part does Clobetasol not get


used on? What is the strength of this
topical steroid?
Face
High potency

You cannot use oral steroids to treat


which skin diseases/conditions?
atopic dermatitis
Psoriasis
places where causative agents can live
reservoir

way for causative agent to escape from the reservoir


portal of exit

humans become a susceptible host when


-large number of the pathogen invade the body
-body defenses are weak

early symptoms. Indicates the start of a disease before


specific symptoms occur
prodromal phase

typical prodromal sx
fever, malaise, HA, lack of appetite, rash

when are you most contagious?


during the prodromal phase or even up to 3 days prior to
prodromal phase

things we need to teach parents about when their kids


are ill
-fever management
-fluid management
-medication
-reduction of disease transmission

hand-foot-mouth disease
-what kind of disease (viral, bac...)
-scientific name
-transmission
-clinical manifestation
-treatment
-viral
-coxsackieviruses A16
-contact with body fluids or droplet
-fever, sore throat, malaise. In 1-2 days herpangina and

blisters on palms and soles


-tx: "magic mouthwash", hydration, analgesics, mouth care

Erythema Infectiosum (Fifth disease)


-what kind of disease (viral, bac...)
-scientific name
-transmission
-clinical manifestation
-treatment
-viral
-human parvovirus B 19
-droplet or contact with blood
-persistent fever for 3-7 days in kid who is otherwise well.
"slapped cheek" appearance, mild URI
-tx: antipyretic, analgesics, anti-inflammatory

this disease begins with a low grade fever HA, mild cold
sx. these pass and the illness seems gone until a fine lacy
rash appears a few days later
fifth disease

Hep A
-what kind of disease (viral, bac...)
-scientific name
-transmission
-clinical manifestation
-treatment
-viral
-Hep A
-food born transmission
-fever, abd pain, N/V, diarrhea, jaundice.
tx: prevention, hydration, antipyretic, analgesics

which hepatitis has no chronic disease state?


hep A

average incubation for Hep A


28 days

Hep B
-transmission
-clinical manifestation
-treatment
-contact with bodily fluids
-jaundice, dark urine, fatigue, abd pain, loss of appetite,
N/V, and joint pain, skin rash

tx: prevention, hydration, antiviral meds, analgesics


acute and chronic states, may be passed mom to baby

can Hep B impact lifespan?


depends on when its contracted. If you get it as a kid, sure,
it can impact your lifespan.

Hep c
-transmission
-clinical manifestation
-treatment
-contact with bodily fluids
-asymptomatic for a very long time. Can lead to liver
cancer and death. Non-specific sx: malaise, fatigue, weight
loss, vague abd pain.
-tx: prevention, antiviral meds, long term management.
may be passed from mom--> baby

Influenza
-what kind of disease (viral, bac...)
-scientific name
-transmission
-clinical manifestation
-treatment
-viral
-influenza A (H1N1) and B
-direct contact
-abrupt fever, progressive URI-like sx, malaise, anorexia
-prevention, antiviral (tammaflu) if detected early,
supportive care

Rubella
-what kind of disease (viral, bac...)
-scientific name
-transmission
-clinical manifestation
-treatment
-viral
-rubella virus
-droplets and contact with bodily fluids
-prodromal phase is ABSENT in kids. See rash starts on
face-->neck, arms, trunk, legs
-prevention, anti-pyretics, analgesics

chicken pox
-what kind of disease (viral, bac...)
-scientific name
-transmission
-clinical manifestation
-treatment
-viral
-varicella-zoster
-direct contact and droplet
-prodromal phase: slight fever and malaise. pruritic rash
begins as macule-->vesicle. pattern is centripetal
appearing on extremities, face
tx: acyclovir 20mg/kg/dose, 4 x day, also prevention,
antihistamines and skin care

when are you no longer contagious with the chicken pox?


-no new lesion and all lesions are scabbed

shingles may develop at any age group but you are more
likely to get it if...
-you are older than 60
-you had chicken pox before the are of 1
-your immune syst is weakened by meds or disease
-you are a teen who was not immunized with two varicella
vaccines

rotovirus
-what kind of disease (viral, bac...)
-scientific name
-transmission
-clinical manifestation
-treatment
-viral
-rotovirus A-G
-contact with bodily fluids
-in kids 3-24mo, it is the most common cause of diarrhea.
also fever, vomiting, dehydration
-tx: prevention, hydration, antipyretics

Polyomyelitis (polio)
-what kind of disease (viral, bac...)
-scientific name
-transmission
-clinical manifestation
-treatment

-virus
-enterovirus
-contact with bodily fluids
-three different forms; abortive/inapparent, non-paralytic,
paralytic
-tx: prevention, bed rest, RT, PT, skin integrity, ADL's

this type of polio you see these sx: sore throat, HA,
vomiting, abdominal pain
abortive/inapparent

in this type of polio you see SEVERE sore throat, HA,


vomiting, abd pain, stiff neck, back and legs
non-paralytic

with this type of polio you see sx of sore throat, HA,


vomiting, stiff neck, back and legs and CNS paralysis with
recovery
paralytic

measles (rubeola)
-what kind of disease (viral, bac...)
-scientific name
-transmission
-clinical manifestation
-treatment
-viral
-mobillivirus
droplet and contact with bodily fluids
-prodromal state: fever and malaise, coryza (stuffy nose),
cough, conjunctivitis. Then Koplick Spots on mucosa, rash
appears on day 3-4 of illness
tx: prevention, vit A, bed rest, antipyretics

when is measles communicable?


4-5 days before rash appears

vit A helps with what communicable disease?


measles

Roseola
-what kind of disease (viral, bac...)
-scientific name
-transmission

-clinical manifestation
-treatment
-viral
-contact with body fluids
-herpesvirus ty 6
-fever >103 for 3-7 days. a macular rash that is blanchable
-tx: antipyretics

mumps (parotitis)
-what kind of disease (viral, bac...)
-scientific name
-transmission
-clinical manifestation
-treatment
-viral
-droplet or direct contact
-paramyxovirus
-fever, HA, swelling of parotid gland on 3d day
tx: prevention, analgesics, antipyretics, hydration, AIRWAY
precautions.

when is mumps contagious?


7 days before parotid swelling until 9 days after swelling
subsides

condyloma/genital warts/HPV
-what kind of disease (viral, bac...)
-scientific name
-transmission
-clinical manifestation
-treatment
viral
-human papillomavirus
-contact with body fluids
-lesions found on butt, vulva, vagina, anus, cervix
-tx: prevention, removal of warts, antimitotic drugs. not
sure if treatment actually reduces transmission...

Haemophilus Influenza Ty B (HIB)


-what kind of disease (viral, bac...)
-scientific name
-transmission
-clinical manifestation
-treatment
-bacterial
-haemophilus influenza ty B

-contact with bodily fluids and droplet


-many ways of expression (epiglottis, septic arthritis,
sepsis, pneumonia [fever, SOB, HA, chills] and meningitis
[fever, HA, NV, stiff neck, sensitive to light])
-tx: prevention, antibiotics, antipyretics, anti-inflammatory

Diptheria
-what kind of disease (viral, bac...)
-scientific name
-transmission
-clinical manifestation
-treatment
-bacterial
-corynebacterium dptheriae
-direct contact with bodily fluids
-URI which progresses to "bull's neck", white/gray mucus
membranes, fever cough
-antibiotics, bed rest, AIRWAY precautions, support

meningococcal meningitis
-what kind of disease (viral, bac...)
-scientific name
-transmission
-clinical manifestation
-treatment
-bacterial
-neisseria meningitidis
-contact with bodily fluids
-fever, HA, stiff neck, NV, photophobia, altered mental
status
-tx: prevention, antipyretics, anti inflamm, anti B

pertussis (whooping cough)


-what kind of disease (viral, bac...)
-scientific name
-transmission
-clinical manifestation
-treatment
-bacterial
-droplet or direct contact with bodily fluids
-catarrhal stage: URI 1-2 wks--> progresses to paroxysmal
stage-short, rapid cough followed by a whoop or gasp for 46 wks. cyanosis may occur
-tx: prevention, suctioning, O2, and hydration

pertussis paroxysmal stage may last 4-6 wks and persists


of 3 stages
1) cold sx 7-10 days
2) violent dry hacking 30 days
3) harsh cough continues

pertussis is most contagious during what stage?


catarrhal

pneumococcal disease
-what kind of disease (viral, bac...)
-scientific name
-transmission
-clinical manifestation
-bacterial
-streptococcus pneumonia
-direct contact with body fluids and droplet
-many manifestations (strep throat, scarletina, scarlet
fever, pneumonia, otis media, sinustitis, localized infec

who is most susceptible to strep bacteria?


6-24 mo

when are we most likely to catch a strep infection?


winter and spring months, crowded physical settings

scarlet fever
-what kind of disease (viral, bac...)
-scientific name
-transmission
-clinical manifestation
-treatment
-bacteria
-group A beta-hemolytic streptococci
-droplets and direct contact with body fluids
-prodromal: abrupt high fever, halitosis, HA enanthema:
tonsils large, edematous, "strawberry tongue, both white
then red...
tx: penicillin, bed rest, analgesics, antipruritis, hydration

tetanus
-what kind of disease (viral, bac...)
-scientific name
-transmission
-clinical manifestation
-treatment

-bacterial
-clostridium tetani
-spores in soil, dust, excretions
-muscle contractions of masseter, neck, and trunk
-tx: immunoglobulin, tetanus toxoid, analgesics

how often should we get a tetanus vaccine?


q 10 yrs

meningitis
onset
clinical manifestations
-tx
rapid onset; bac, viral, fungal, aseptic
-nucchal rigidity, change in mental status, fever, rash
-tx: antipyretics, analgesics, antifungal/antibiotics,
hydration

which type of meningitis has the most rapid onset?


bacterial

who is most susceptible to strep bacteria?


kids 6-12 mo

most frequent infections worldwide?


intestinal parasites

who are the most susceptible to intestinal parasites?


young kids

most common parasites in the US?


pinworms and giardiasis

giardiasis
-what kind of disease (viral, bac...)
-scientific name
-transmission
-clinical manifestation
-treatment
-protozoan
-giardia intestinalis
-direct contact with contaminated water or food
-diarrhea, NV, anorexia, failure to thrive, abd cramps,
constipation, bloating, flatulence
-tx: flagyl, tindamax, and prevention of reoccurance

enterboiasis (pinworms)
-what kind of disease (viral, bac...)
-scientific name
-transmission
-clinical manifestation
-diagnosis
-treatment
protozoan
-nematode enerobius vermicularis
-inhalation or ingestion of eggs from contaminated hands
-itchy butt, restless, bed wetting, short attention span, poor
sleep, possible urethral infection
-dx: tape test
tx: pyrantel pamoate or albendazole x1 then again in 2
wks. treat whole family. prevent occurrence

disorders transmitted to humans via anthropods


rickettsial infection

rickettsial infections are most common in what region?


temperate and tropical climates

lyme disease
-what kind of disease (viral, bac...)
-scientific name
-transmission
-clinical manifestation
-vector borne
-borrelia burhdorferi
-transmitted by deer tick bite
{stage 1: }"bull's eye", fever, HA, malaise
{stage 2:} (1-4mo) rash on hands/feet, fever, HA, malaise,
fatigue, lymphadenopathy, cough, stiff neck
{stage 3:} systemic involvement (2-12mo)
arthritis, CNS changes, cardio complications,
encephalopathy, cognitive and behavioral changes, chronic
arthritis

tx for lyme
>8 yo, doxycycline
<8 yo, amoxicillin

rocky mountain spotted fever


-what kind of disease (viral, bac...)
-scientific name

-transmission
-clinical manifestation
-treatment
-vector borne
-ricketsia ricketsi
-gradual or abrupt onset of fever, malaise, HA, rash on
palms/feet
-tx: antipyretic, analgesic, antibiotics

when taking a hx on a kid suspected of rocky mountian


spotted fever, inquire about
-recent tick bites
-travel to endemic area
-presentation in the spring/fall

Cat scratch disease


-what kind of disease (viral, bac...)
-scientific name
-clinical manifestation
-treatment
-vector born from a cat scratch
-bartonella henselae
-painless, nonpruritic papule, regional lymphadenitis
tx: antibiotics

west nile virus


-what kind of disease (viral, bac...)
-scientific name
-transmission
-clinical manifestation
-treatment
-preventable measures
- vector (mosquito)
-flavivirus
-fever, HA, NV, body aches, seizures, memory loss, brain
damage-esp in kids. most people recover.
-tx: hydration, antipyretic, analgesic,
preventable measures: clothing at dusk, repellents,
eliminate standing h2o

how do lice spread?


from personal contact with infected person. they don't
jump.

the condition of lice is called?


-pediculosis capitis

lice
-scientific name
-clinical manifestation
-treatment
-pediculus humanus capitis
-intense pruritus of scalp, behind ears, nape of neck, nits
(eggs) attached to hair shaft
-pediculicide and removal of nits, education and support.
kid infected, family and home must all be treated.

scabies
-scientific name
-transmission
-clinical manifestation
-treatment
-sarcoptes scabiei
-prolonged close contact. mites burrow into skin and lay
eggs in CLUSTERS
-intense itching, excoriation and burrows, inflammation
btwn fingers, neck folds and groin
tx: scabicide, >2 yr permethrin, wash linen and clothes and
dry high heat, supportive care for itching 2-3 weeks

bedbugs
-scientific name
-transmission
-clinical manifestation
-treatment
-cimex lectularis
-contact with infested mattress. mites burrow and feed on
blood.
-intense itching, all over bites, may progress to
follicularitis/cellulitis. may trigger asthma/anaphylaxis
-identification and eradication of bugs, topical application
of steroids, hygiene of linens and clothing, supportive care
pruritis 2-3 wks

which communicable disease can have conjunctivitis as a


sx?
measles

what causes conjunctivitis?


can be bacterial or viral

must admin antib for how long before a return to school


with conjunctivitis?
24 hour

general term that means swelling/lesions of the mouth


stomatitis

two types of stomatitis


ty1: aphthus ulcers-benign
ty 2: herpetic gingivostomatitis hsv

goals of stomatitis?
NSAIDs, anesthetics, prevent spread by oral secretions and
poor handwashing

4 types of bacterial skin infections


-impetigo
-folliculitis
-cellulitis
-scalded skin syndrome

impetigo rarely results in


scars

what is reye's syndrome?


exact cause in unknown, it is associated with asprin
(ibuprofen) consumption in kids with a viral illness.

early diagnosis of reye's syndrome is vital bc


it can lead to severe injury and death

classic sx or reye's syndrome


rash, vomiting, and liver damage

sx of encephalitis
personality changes, seizures, weakness

what causes encephalitis?


really any of the disease in the lecture

who is most susceptible to encephalitis?


kids

common fungal skin infections


-tinea capitis (scalp)
-tinea corporis (body/nails)
-tinea cruris (groin)
-tinea pedis (feet)
-thrush, candidiasis

tonsillar pharyngeal areas are covered with a white/gray


membrane. possible pronounced "bull's neck".
complications include myocarditis
diptheria

rash appears in phases. phase one is erythema on the


face, chiefly on the cheeks giving a "slapped cheek" look
fifth disease

rash composed of rose-pink macules, first on neck, then


on trunk. rash is non-puritic, and blanchable.
roseola

In this illness, the prodromal phase is a earache that is


aggrivated by chewing. by third day, parotid glands
enlarge.
mumps

Conjunctivitis and Koplik spots are present before the


rash. Rash appears 3-4 days after onset. Erupts first on
the face and then spreads downwards.
measles (rubeola)

discrete pinkish red maculopapular rash appears on face,


then spreads down to neck, arms, trunk and legs. Its very
benign to kid but its greatest danger is teratogenic effect
on fetus
rubella

abrupt high fever, halitosis. then the tonsils enlarge and


become red with patches of exudate. tongue becomes
red and swollen. itchy, peely rash (desquimation) occurs.
scarlet fever

fever, sore throat, malaise. In 1-2 days herpangina and


blisters on palms and soles
hand, foot and mouth disease

communicable disease
...a disease that can be given from one person to another

microbe (microorganism)
...a living thing that cannot be seen with the naked eye;
examples include bacteria and viruses

normal (resident) flora


...the harmless microbes that live in and on the body and
help it to function properly

pathogens
...a microbe that can cause illness

opportunistic microbes
...microbes that are considered normal (resident) flora
when they are in or on one part of the body, but can cause
infection if they move out of that area and into or onto
another part of the body

colonies
...groups of bacteria

aerobic
...bacteria that need oxygen in order to live

anaerobic
...bacteria that can survive withhout oxygen

antibodies
...specialized proteins produced by the immune system
that help our bodies to fight specific microbes, preventing
infection.

multidrug-resistant organisms (MDROs)


...bacteria that are resistant to one or more classes of antimicrobical agents

methicillin-resistant Staphylococcus aureus (MRSA)


...a type of bacteria that has become resistant to
methicillin, a powerful antibiotic.

vancomycin-resistant enterococcus (VRE)

...a type of bacteria that has become resistant to


vancomycin, a powerful antibiotic

infection
...disease caused by pathogenic microbes

chain of infection
...the six key conditions that must be met for a person to
get a communicable infection: pathogen, reservoir, portal
of exit, method of transmission, portal of entry, and a
susceptible host.

contaminated
...describes an object that is soiled by pathogens

fomite
...a non-living object that has been contaminated (soiled)
by pathogens

vector
...a living creature, such as an insect, that can transmit
disease

virulence
...the strength or disease-producing potential of a
pathogen

health-care associated infections (HAIs)


...infections that a patient or resident gets while receiving
treatment in a hospital or other health care facility, or that
health care workers get while performing their duties
within a health care setting.

nosocomial infections
...Infections that a patient or resident get while receiving
treatment in a hospital or other health care facility; a type
of health-care associated infection (HAI)

infection control
...basic practices designed to decrease the chance that an
infection will spread from one person to another in a health
care facility

medical asepsis

...techniques that used to physically remove or kill


pathogens

sanitization
...practices associated with basic cleanliness,such as hand
washing, cleansing of eating utensils and other surfaces
with soap and water, and providing clean linens and
clothing; one of the techniques of medical asepsis

antisepsis
...practices that kill microbes or stop them from growing;
one of the techniques associated with medical asepsis
(compare with sanitization, disinfection, and sterilization)

disinfection
...the use of strong chemicals to kill pathogens on nonliving objects that come in contact with body fluids or
substances, such as bed pans, urinals, and over the bed
tables; one of the techniques of medical asepsis (compare
with sanitization, disinfection, and sterilization)

sterilization
...the process of completely eliminating microbes from the
surface of an object using an autoclave (sterilizer) or
chemicals; (compare with sanitization, disinfection, and
sterilization)

transient flora
...microbes that are picked up by touching contaminated
objects or people who have an infectious disease

personal protective equipment (PPE)


...barriers that are worn to physically prevent microbes
from reaching a health care provider's skin or mucous
membranes, such as gloves, gowns, masks, and protective
eye wear

isolation precautions
...guidelines, based on a pathogen's method of
transmission, that health care workers follow to contain the
pathogen and limit others' exposure to it as much as
possible

standard precautions

...precautions that a health care worker takes with each


patient or resident to prevent contact with bloodborne
pathogens; include the use of barrier methods (such as
gloves) as well as certain environmental control methods

transmission-based precautions
...precautions that a health care worker takes when a
person is known to have a disease that is transmitted in a
certain way; include airborne precautions, droplet
precautions, and contact precautions

airborne precautions
...used when caring for patients or residents infected with
pathogens that can be transmitted through the air; include
placing the patient or resident in a private room with the
door closed, wearing a mask when caring for the patient or
resident, and minimizing the amount of time the person
spends out of his or her private room

airborne infection isolation room (AIIR)


...single-occupancy patient-care room with special
ventilation and filtration systems used to isolate persons
with an airborne infectious disease

droplet precautions
...used when caring for patients or residents infected with
pathogens that can be transmitted by direct exposure to
droplets released from the mouth or nose (for example
when the person coughs, sneezes or talks)

contact precautions
...used when caring for patients or residents infected with
pathogens that can be transmitted directly (by touching
the person), or indirectly (by touching fomites); include
using barrier methods whenever contact with the infected
person or items contaminated with wound drainage or
body substances is necessary

Lippincott's Textbook for Nursing Assistants


ISBN 978-1-60547-635-3
Chapter 7 Communicable Disease & Infection Control
chapter 10 ISBN 978-4511-9466-1
These terms and definitions were taken word-for word from
the Glossary of the textbook.
Pathogens

microorganisms that cause disease. There is a wide range


of them and they are transmitted in a variety of ways. The
means of transmission can be categorised as direct or
indirect.

Callose
is a large polysacchride that blocks old sieve tubes

Tylose
a balloon-like swelling, also blocks old xylem vessels

primary defences
which prevent entry of pathogens into the body

Secondary defences
help to remove a pathogen after it has entered the body.

immune response
the body's response to invasion by pathogens.

specific immune response


means that the response deals only with one pathogen
that possesses one particular antigen.

Blood clots
reduce the loss of blood and make a temporary seal,
preventing access by pathogens

Inflammation
the swelling and redness seen in infected tissues. Infected
tissue often feels hot and tender. This is caused by a cellsignalling substance called histamine.

Antibodies
proteins that are secreted in response to stimulation by the
appropriate antigen. They have specific binding sites and
are capable of acting against the pathogen.

Antigens
molecules on the surface of cells that the immune system
can use to recognise pathogens.

Antigen presentation

involves placing an antigen on the cell-surface membrane


of a phagocytic cell.

primary response
the immune system's response to a first infection

secondary response
the immune system's response to a second or subsequent
infection by the same pathogen.

Active immunity
immunity acquired by activation of the immune system

Passive immunity
when someone is given antibodies produced by someone
else

autoimmune disease
one in which the immune system attacks the body's own
healthy cells and tissues

Vaccination
the deliberate introduction of antigenic material in order to
stimulate the production of antibodies

vector
an organism that carries the pathogen from one host to
another.
disease
an illness

communicable disease
caused by pathogens from one living thing to another

pathogen
a disease causing organism

non-communicable disease
a disease that can not be spread other than through
heredity (no pathogens-heart disease)

virus
smallest pathogens (ex: common cold, herpes)

bacteria
singled-celled microorganisms (ex: strep throat,
tuberculosis, syphilis, and lyme disease)

rickettsia
pathogens that grow in living cells and resemble bacteria
(ex: rocky mountain spotted fever and typhus)

fungi
single-celled or multi-cellular plantlike organisms, yeast
and mold (ex: athlete's foot, jock itch, nail infection,
ringworm)

protozoa
tiny, single-celled organisms that produce toxins that
cause disease (ex: malaria and dysentery)

direct contact
sexual intercourse, intimate kissing, blood transfusion,
touching ulcers or sores, handling body fluids (blood or
urine)

indirect contact
contact with an object that has been used by an infected
person

contact with pathogens in the air


an infected person coughs or sneezes, pathogens are
released inot the air

contact with animals or insects


a person becomes infected by being bitten by an animal or
insect (ex: 1) Rabies virus-dog, cat, raccoon, bat
2)Mosquitos-malaria 3)Insects land on sewage and then
deposit pathogen in food

contact with Contaminated Food and Water


water must be purified to drink and cook with

skin
unbroken skin prevents pathogens from entering the bodybathing rids the body of the pathogens

tears, saliva, and perspiration


contain chemicals that kill pathogens

mucous membranes
tissues that line the body opening and secrete mucus

mucus
is thick secretions that coats the mucous membranes

stomach acid
acid in stomach that kills most pathogens

Immune System Response


contains cells and organs that fight disease

T cells
white blood cells that regulate the action of the immune
system

phagocytes
white blood cells that surround and kill pathogens by
ingesting them

B cells
...
What is the primary prevention of disease?
Immunization

How do you control the spread of disease to others?


Reduce risk of cross-transmission of organisms
Infection Control Policies
Handwashing

Children With Immunodeficiency


Receiving Steroid Therapy
Other immunosuppressive therapies
Generalized malignancies
Immunologic disorder

What is Varicella?
Chicken Pox

How is Varicella Transmitted?

Droplet (Airborne) and Contact

Incubation of Varicella
14-16 days but can be as long as 18-21 days

Prodromal Period of Varicella


Mild fever
Malaise x 24 hours prior onset of rash
MOST CONTAGIOUS AT THIS TIME

Varicella-Manifestations
Rash erupts 1 day after prodromal period
Very itchy
Begins as a macule and progresses to a papule
Then becomes a fluid filled vesicle resembling a dew drop,
which breaks and forms a crust
starts in trunk and progress to proximal extremities and
face

Varicella-Treatment
Acyclovir and Varicella Zoster Immune Globin in High Risk
Children After Transmission
Strict Isolation
Skin care-Comfort, Cool Bath, Calamine
Avoid use of aspirin/salicylates
Manage itching w/distraction, benadryl in moderation

Varicella-Complications
Secondary bacterial infections
Encephalitis
Pneumonia
Hemorrhagic Varicella
Chronic or transient thrombocytopenia

What is Erythema Infectiosum?


Fifth Disease
Caused by Parvovirus B19

How is Erythema Infectiosum Transmitted?


Unknown-? respiratory secretions and blood

Erythema Infectiosum-Manifestations
Mild Fever
Joint Pain

Lethargy
Rash-3 stages

What are the 3 stages of a fifths disease rash?


1) Slapped face appearance on cheeks
2) Maculopapular rash on extremities; resembles lace
3) Skin Appears Irritated

Erythema Infectiosum-Management
Self Limiting, can only manage symptoms
*anti-pyretics
*analgesics

Erythema Infectiosum-Risk during Pregnancy


Risk of fetal hydrops and death
May need serologic testing if exposed to monitor immune
status

Erythema Infectiosum-Complications
chronic joint pain
aplastic crisis
myocarditis

What is Exanthem Subitum?


Roseola
Human herpes virus type 6

How is Exanthem Subitum transmitted?


Unknown transmission and source

Exanthem Subitum-Characteristics
High fever with unknown etiology for several days
Followed by rash once fever is resolved
may have lymphadenopathy, inflamed pharynx, cough,
coryza

Exanthem Subitum-Treatment
Self limiting. Treat the symptoms!

What is Pertussis?
Whooping Cough
Caused by bordetella pertussis

How is Pertussis Transmitted?

Droplet or Direct Contact


Incubation: 6-20 days

Pertussis-Characteristics
Short, rapid coughs followed up a crowing or "whoop"
sound

What is the complication of Pertussis?


Pneumonia

How is Pertussis treated?


Antibiotics

What is Rubeola?
Measles
Viral Infection

How is Rubeola transmitted?


Droplet from respiratory tract, blood, urine

Rubeola-Characteristics
Fever and Malaise
Followed by coryza, cough, conjunctivitis, Koplik spots,
anorexia, lymphadenopathy then rash

Rubeola-What does the rash look like?


Initially erythematous maculopapular progressing to
brownish with desquemation

Rubeola-Treatment
Treatment includes supplementation w/vitamin A;
supportive care
Maintain isolation until 5th day of rash, bedrest

Rubeola-Complications
Complications include otitis media, pneumonia,
bronchiolitis, obstructive laryngitis/laryngotracheitis,
encephalitis

What is Mumps? How is it Transmitted?


Paramyxovirus
Transmitted by contact or droplet spread of infected saliva

Mumps-Characterized

Fever, headache, malaise, anorexia, earache w/chewing;


parotitis w/enlargement, pain and tenderness; may have
submaxillary/sublingual infection, orchitis,
meningoencephalitis

Mumps-Treatment
Supportive: analgesics, antipyretics; IV fluids if unable to
drink/vomiting due to meningoencephalitis
Maintain isolation

Mumps-Complications
sensorineural deafness, postinfectious encephalitis,
myocarditis, arthritis, hepatitis, epididymo-orchitis, sterility
in adult males (rare)

What is German Measles?


Rubella Virus

How is German Measles Transmitted?


Transmitted by nasopharyngeal secretions, blood, urine,
stool

German Measles-Characteristics
No prodromal symptoms in children; in adults, adolescents:
low grade fever, headache, malaise, anorexia,
conjunctivitis, coryza, sore throat, cough,
lymphadenopathy; followed by rash, starting on face and
progressing downward, by end of 1st day body is covered

German Measles-What does the rash look like?


Rash is maculopapular, pinkish red; usually disappears in
same order that it appeared

German Measles-Treatment
Self limiting, treatment includes antipyretics, analgesics

German Measles-Complications
Rare-but include arthritis, encephalitis, purpura

German Measles-Contracted during Pregnancy


Benign course for affected child, but more serious when
acquired in pregnancy; teratogenic effects noted on fetus
(congenital heart disease, hearing loss, growth delay,
learning disablities, mental retardation,

eye/neurologic/endocrine abnormalities); also can cause


stillbirths, miscarriages
Immune status always checked in pregnancy

What is Poliomyelitis?
3 types of enteroviruses
No wild type virus since 1979; few vaccine-induced cases
until 1998, development of IPV

How is Poliomyelitis transmitted?


Transmitted by direct contact w/infected feces and
oropharyngeal secretions

Poliopmyelitis-Chracteristics
May be inapparent (fever, sore throat, headache, vomiting,
abdominal pain), nonparalytic (sl. more severe than
inapparent)

What is the most severe form of Poliomyelitis?


Most severe form is paralytic, which is similar in course to
nonparalytic type w/recovery, and then CNS paralysis

Poliomyelitis-Treatment
No treatment other than supportive

Poliomyelitis-Complications
Complications include complete paralysis, respiratory
arrest, hypertension, and those things caused by
immobility

What is Scarlet Fever?


Group A beta-hemolytic strep

How is Scarlet Fever transmitted?


Transmitted by direct contact/droplet spread from infected
nasopharyngeal secretions

Scarlet Fever-Characteristics
Fever, malaise, headache, lymphadenopathy, vomiting,
chills, abdominal pain, enlarged tonsils w/exudate,
erythematous pharynx, strawberry tongue, and RASH

Scarlet Fever-What does the rash look like?


red, pinpoint w/sandpaper texture

Scarlet Fever-Treatment
antibiotics (penicillin, erythromycin) x 10 days, analgesics,
antipyretics.

Scarlet Fever-Complications
otitis media, peritonsillar/retropharyngeal abcess, sinusitis,
glomerulonephritis, carditis/polyarthritis

Conjunctivitis-Causes in Newborns
chlamydia, gonorrhea, or herpes simplex virus

Conjunctivitis-Causes in Infants
may be sign of tear duct obstruction

Conjunctivitis-Causes in Children
bacterial (most common), viral, allergic, or foreign body

Lead Poisoning-What does a low dose exposure cause?


Distractibility
Hyperactivity
Impulsivity
Hearing impairment
Mild intellectual deficits

Lead Poisoning-What does a high dose exposure cause?


Encephalopathy
Mental retardation
Paralysis
Blindness
Seizures
Coma
Death

Lead Poisoning treatment


Chelation
calcium disodium edetate (EDTA) and succimer (DMSA)

Which Vaccines have two series?


MMR
Varicella
Hep A

Which Vaccines have three series?

Hep B
RV

Which Vaccines have four series?


Hib
PCV
IPV

Which Vaccines have five series?


DTaP

Which Vaccine is needed yearly?


Influenza
Which 2 vaccines are given subcutaneous?
Varicella and MMR

Definition of Immunization?
Process when an animal or person becomes protected
against a disease

Definition of Vaccination?
Injection of a killed or weakened infectious organism to
prevent a disease

Definition of Vaccine?
Product that produces immunity to protect the body from a
disease.

Safety of Vaccines?
Studied before giving to public
Scientist monitor safety of vaccines
Serious adverse effects are rare
Multiple vaccines at one visit is considered safe and
quickest protection against diseases

What is infection?
Invasion of bacteria or viruses

What alerts the immune system to the invasion of


bacteria or viruses?
WBC's

What types of WBC's are the main part of the immune


system?
macrophages, B-lymphocytes (antibodies) and Tlymphocytes

What is active immunity?


when the immune system produces antibodies because of
exposure to disease organism

What are the types of active immunity?


natural and vaccine-induced immunity

What is natural immunity?


antibodies formed in the presence of active infection in the
body

What is vaccine-induced immunity?


antigens (vaccines or toxoids) are given to stimulate
antibody production

How long does naturally acquired active immunity last?


lifelong

How long does it take for vaccine acquired active


immunity to take effect?
can take several weeks to develop

How long does vaccine acquired immunity last?


many years, but must be reinforced with a booster

How long does active immunity last?


long period

How long does passive (acquired) immunity last?


short period

How long does natural passive immunity last?


6 months - 1 year

How long does artificial passive immunity last?


2-3 weeks

What is passive (acquired) immunity?

Antibodies are produced by another source, animal or


human

What is natural passive (acquired) immunity?


antibodies are transferred naturally from an immune
mother to her baby through the placenta or colostrum

What is artificially passive (acquired) immunity?


immune serum (antibody) from an animal or another
human is injected

What is heard immunity?


when a enough of a population is immune to an infectious
disease so the non-immunized people are offered some
protection and this keeps the disease from spreading

What is a live, attenuated vaccine?


A vaccine that contains a living, weakened virus

Is a live or a killed vaccine better for immunity purposes?


live, as it is the closest to the natural infection

What are examples of a live, attenuated virus?


MMR and Varicella vaccines

What is an inactivated vaccine?


A vaccine that contains a killed virus

What is the disadvantage of an inactivated vaccine?


several doses are necessary for immunity

What is an example of an inactivated vaccine?


polio vaccine

What is a subunit vaccine?


A vaccine that contains parts of a virus or bacteria
(subunits)

What are the advantages of a subunit vaccine?


side effects are less common

What is an example of a subunit vaccine?


Pertussis vaccine

What is a conjugate vaccine?


A vaccine that fights bacteria because the antigens stick to
an outer coating of sugar like substance called
polysaccharides

What is an example of a conjugate vaccine?


Haemophilus influenze type B (Hib) vaccine

What is a toxoid vaccine?


...
disease
any condition that interferes with the proper functioning of
the body or mind

communicable disease
a disease that can be passed from one person to another

pathogen
a disease causing organism when germs enter your body,
you can develop an infection

infection
a condition that occurs when pathogens enter your body,
multiply and cause harm

virus
the smallest pathogen that can be heeled with antibiotics

bacteria
tiny one celled organism that can be cured with antibiotics

fungi
more complex organism than bacteria but cannot make
their own food. Thrive in warm, moist areas ex. ringworm,
athletes foot

protozoa
one celled organism that are more complex then bacteria

direct contact with others


shaking hands or kissing one with an infected person

indirect contact with others


sharing drinks or eating utensils, contaminated needles,
tattoos, piercing or drug injections

contact with contaminated foods or water


improperly stored or uncooked

contact with animal or insects


ticks with Lyme disease, mosquito's, with West nile

protect yourself from pathogens


practicing good hygiene (cleanness)

protect yourself from pathogens


avoid contact with infected people

protect yourself from pathogens


never share eating or drinking utensils

protect yourself from pathogens


wash your hands frequently

protect yourself from pathogens


handle and prepare food frequently

protect yourself from pathogens


avoid touching eyes,mouth or nose

skin
first line of defense,
covers and protects body

mucous membranes
lines the mouth, nose , throat and eyes,
their job is to trap jerms

saliva
destroys harmful organisms

tears
wash away germs

stomach acid

acid produced in the stomach kills germs

vaccine
a preparation of dead or weakened pathogens that causes
the immune system to produce antibiotics

immunization
helps the immune system make antibiotics for certain
diseases

immunity
your body's ability to resist the germs that causes a
particular disease

2 main responses to your immune system


specific and nonspecific

non specific response


when pathogens enter your body, your immune system
reacts with a non specific reaction first. this response
began with inflammation. It reacts to injuries,
diseasesresulting in swelling, pain, heat and redness

specific reactions
some pathogens survive the nonspecific reaction. when this
happens your body is self off a specific immune system.
Our immune system recognizes a pathogen it has already
battled so it is ready to re attack

antigen
substance realised by invading pathogens

types of immunization
Hepatitis B, Polio, measles, chicken pox, tetanus shot
not only do they help you but they also surround you

colds
are caused by hundreds of different viruses and can be
spread by direct and indirect contact

treatments of cold
getting rest and drinking fluids, OTC medicines can help,
stay home for at least 24 hours

flu (influenza)
symptoms are fever, chills/ fatigue, headaches/muscle
aches and respiratory pain

mononucleosis
''mono'' or the ''kissing disease'' is a viral diseases
characterized by a severe sore throat and swelling of the
lymph glands in the neck and throat area

hepatitis
a viral disease if the liver characterized by yellowing of the
skin and whites of the eye

3 types of hepatitis
hep A,B and C

hepatitis A
common in areas of poor sanitation , like food or water
contaminated with human waste or enters the body
through an open wound

hepatitis B,C
spread through contact with contaminated blood or other
body fluids. There are vaccines to protect people with Hep
A and B. There are medicines to treat people with Hep.C

tuberculosis (TB)
bacterial disease that affects the lungs. TB can be spread
easily through air ,sometimes no symptoms but can still be
spread.

pneumonia
serious inflammation to the lungs
symptoms are fever, chills, and difficultly breathing

strep throat
sore throat causes by streptococcal bacteria
symptoms are red and painful throat, fever,and swollen
lymph nodes in neck

H.i.v
human, immunodeficiency, virus

human

this disease attacks humans

immunodeficiency
this disease attacks the immune system

virus
the smallest and simplest form of life

A.I.D.S
acquired, immune, deficiency, syndrome

Acquired
this disease comes from outside the body. A person must
do something to take it in

Immune
this disease attacks the immune system

deficiency
lack of something. Aids destroys the T-cells

syndrome
a medical condition that has no cure

opportunistic disease
a disease that takes advantage of a weakness in the body.
People die from it because aids kill the immune system.

Hiv
2-10 years looks and feels good

Arc,Aid
body shuts down

death
male 3 years
females 1-5 years

A.R.C
aids realistic complex

what is arc

a series of illness when hiv turn into full blown aids. A


person begins to looks sick and feels sick

symptoms of arc
lost of appetite

symptoms of arc
fatigue

symptoms of arc
weight loss

symptoms of arc
fever

symptoms of arc
night sweats

symptoms of arc
diarrhea

what is believed to be the organ to the virus


It is said to organite from a green monkey in Africa

ways hiv can be transmitted(spread)


sexual contact (75%)

ways hiv can be transmitted(spread)


IV drug use (22%)

ways hiv can be transmitted(spread)


mother to child (2%)

ways hiv can be transmitted(spread)


blood transfusion (1%)

Way hiv is not transmitted(spread)


breathing in ar

Way hiv is not transmitted(spread)


swimming pools

Way hiv is not transmitted(spread)


donating blood

Way hiv is not transmitted(spread)


casual contact

Way hiv is not transmitted(spread)


sharing items

medicine used to fight aids


azt

what it does
slows down the disease

what it does
decreases the number of immune cells

these drugs are very


expensive

is there a cure
no

way to prevent the disease


abstinence

way to prevent the disease


personal behaviors
The infectious disease stage in which the antigen is
present but no antibody is detectable is the ______________
stage.
A) A. Communicable
B) B. Incubation
C) C. Latent
D) D. Window
D

The hepatitis ____ virus is most commonly transmitted by


contaminated needles.
A) A. A/HAV
B) B. B/HBV

C) C. C/HCV
D) D. D/delta
C

The most severe and potentially life-threatening form of


meningitis is ______________ sometimes caused by
____________.
A) A. Bacterial/Neisseria meningitides
B) B. Bacterial/tuberculosis
C) C. Viral/streptococcus pneumoniae
D) D. Viral/viral pneumonia
A

An acute disease caused by the Epstein-Barr virus, a


member of the herpes family is:
A) A. Influenza
B) B. Mononucleosis
C) C. Pertussis
D) D. Rabies
B

Inflammation and swelling of the cornea is known as:


A) A. Keratitis
B) B. Malaise
C) C. Prodrome
D) D. Risus sardonicus
A

Who is responsible for protecting the public from disease?


A) A. FEMA
B) B. OSHA
C) C. Local hospital
D) D. Your employer
D

Incubation, window, communicable, latent, and disease


period are stages of:
A) A. Allergic reactions
B) B. Infectious disease
C) C. Infection plans
D) D. Paramedic stress factors
B

As a paramedic, you should take standard precautions


with any patient with unknown respiratory pathogens,
and you should also:

A) A. Contact the CDC.


B) B. Keep patient as far away from you.
C) C. There is need to worry because the patient will be in
your care for a short period of time.
D) D. Place surgical mask on patient.
D

The interval between exposure to an agent and the first


appearance of symptoms is the
A) A. Disease period
B) B. Incubation period
C) C. Latent phase
D) D. Window phase
B

EMS agency employers are required by law to provide


employees with appropriate medical care and treatment
in the event of an exposure and designate a person or
officer to whom the employee should report the exposure.
This is under:
A) A. FEMA
B) B. Medical control
C) C. Roe v. Wade
D) D. The Ryan White Act
D

Identify a transmission route for hepatitis C.


A) a. Contaminated feces
B) c. Semen and vaginal fluids
C) b. Infected blood
D) d. Sharing food and water
C

A viral infection resulting in swelling of the salivary and


parotid glands is:
A) b. Chickenpox
B) a. An allergic reaction
C) c. Mumps
D) d. Rubella
C

The time period from exposure until when disease


symptoms first appear is the:
A) b. Incubation period
B) d. Window phase

C) a. Communicable period
D) c. Latent period
A

You have the greatest risk of contracting which


bloodborne pathogen from a needle stick?
A) c. Human immunodeficiency virus (HIV)
B) b. Hepatitis B
C) d. Tuberculosis
D) a. Hepatitis A
B

The vulnerability or weakness to a specific pathogen is


known as:
A) c. Reactability
B) a. Immunity rate
C) d. Susceptibility
D) b. Infection rate
C

You have been splashed in the face with the blood of a


patient with hepatitis C. After washing your face and eyes
with water for at least 10 minutes, you are required to:
A) d. Request prophylaxis medications from the receiving
emergency department
B) c. Report the incident to a supervisor or the infection
control officer
C) b. Mandate that the patient pay for your medical
expenses if you get hepatitis C
D) a. Fill out a Ryan White reporting form
B

A woman has brought her daughter to the emergency


department. The girl is 4-years-old and has been suffering
from a worsening, 'hacking' cough for several days. As
you evaluate the child, her temperature is 104 F, her
eyelids appear swollen, her eyes are red with a crusty
discharge, and she has reddish spots on her forehead,
lips, and in her mouth. You quickly identify this patient as
having:
A) a. Chickenpox
B) c. Measles (rubeola)
C) d. Mumps
D) b. German measles (rubella)
B

You are presented with a 24-year-old female patient in the


back bedroom of a low-income apartment. Her husband
tells you she has had a worsening headache over the past
few days. She is laying on the covers in the dark. When
you turn on the light, she winces, and complains the light
makes her headache worse and causes dizziness. You
notice that her skin is flushed and she is sweating despite
a cold apartment, and you also notice that her neck
appears stiff as she moves. Your next action is to:
A) c. Perform a physical exam of the head and neck
B) a. Administer an antipyretic to reduce any fever and
the pain
C) b. Apply oxygen and listen to lung sounds
D) d. Remove yourself from the room and put on an N-95
mask or other protective mask
D

A paramedic can become infected with tuberculosis by:


A) d. Sharing food or water with an infected individual
B) c. Being exposed to membrane droplet nuclei expelled
during a cough, sneeze, or prolonged talking
C) b. Being stuck with a needle that had been used on a
patient with tuberculosis
D) a. Allowing feces contaminated with tubercle bacilli to
touch the skin
B

Outbreaks associated with the HSV-1 infection generally


occur:
A) d. In the oropharynx
B) b. Evenly across the body
C) c. In the genital region
D) a. Around the anus
A

Rubella affects the:


A) a. Circulatory and musculoskeletal systems
B) c. Respiratory and nervous systems, and oropharynx
C) d. Respiratory, circulatory, and nervous systems
D) b. Integumentary and musculoskeletal systems
D

The human immunodeficiency virus can be spread


through:
A) a. Coughing
B) c. Heavy sweating

C) b. Hand-to-hand contact
D) d. Sexual intercourse
D

Which form of hepatitis is not considered a complete


virus?
A) b. Hepatitis B
B) c. Hepatitis C
C) d. Hepatitis D
D) a. Hepatitis A
C

A diagnosis of bacterial meningitis is typically associated


with:
A) c. Septic shock
B) b. Internal bleeding in the spinal column
C) d. The absence of other infections
D) a. Another infection such as pneumonia or endocarditis
D

You are caring for a 73-year-old female in a nursing home


who is experiencing respiratory distress that worsened
throughout the night. Currently you find her seated
upright in bed with accessory muscle use. Audible rales
can be heard; she is diaphoretic and warm to the touch,
she has a temperature of 102 F, and her respiratory rate
is 32 breaths/min. The patient tells you that her breathing
has been getting worse all night. You auscultate the chest
and hear rales and rhonchi in all fields of her left lung and
rhonchi in her right base, but her right middle and upper
lobes sound clear. You determine that this patient is
experiencing respiratory distress caused by:
A) d. Tuberculosis
B) b. Chronic obstructive pulmonary disease
C) a. Congestive heart failure
D) c. Pneumonia
D

Males who have contracted chlamydia may experience:


A) d. Open sores on the penis and scrotum
B) b. Development of lesions on the penis
C) a. A urinary tract infection
D) c. Discharge from the urethra
D

The time period when an infected disease can be


transmitted to another host is known as the:
A) d. Window phase
B) b. Incubation period
C) c. Latent period
D) a. Communicable period
D

Gonorrhea can be cured with the use of:


A) b. Antibiotics
B) c. Antifungal drugs
C) d. There is no definitive cure
D) a. Antiparasitic drugs
C

The responsibility of appropriately disposing of equipment


and supplies potentially infected with a disease lies with
the:
A) c. Patient
B) a. Hospital staff
C) b. Paramedic
D) d. Ambulance/transporting agency
C

During the care of a 44-year-old female patient who was


ejected from her SUV during a highway-speed rollover,
her arterial bleeding results in blood splashing onto your
face and eyes. Besides stopping care to immediately flush
your face, it is your right to:
A) c. Request that you and the patient both be tested for
communicable diseases
B) b. Receive workers' compensation for the remainder of
your working years
C) d. Sue the patient
D) a. Receive postexposure prophylaxis for hepatitis and
HIV
A

The time period when an infected person cannot transmit


a disease to another individual is known as the:
A) b. Incubation period
B) c. Latent period
C) d. Window phase
D) a. Communicable period
B

The MMR vaccine protects recipients from:


A) a. Malaria, mumps, and Reye's syndrome
B) c. Measles, malaria, and Reye's syndrome
C) d. Measles, mumps, and rubella
D) b. Malaria, mumps, and rubella
C

A viral disease once commonly seen in children but that


now appears in fewer than 500 cases per year is:
A) c. Mumps
B) a. Fifth disease
C) d. RSV
D) b. Chickenpox
A

Outbreaks associated with the HSV-2 infection generally


occur:
A) c. In the genital region
B) b. Evenly across the body
C) a. Around the anus
D) d. In the oropharynx
A

The rubella virus can be spread through contact with


infected:
A) a. Blood
B) b. Feces
C) c. Saliva
D) d. Skin/open lesions and blisters
C

The herpes simplex virus type 1 (HSV-1):


A) a. Affects 80% of the human population
B) b. Has been eliminated from the human species and no
cases are currently known to exist
C) d. Affects approximately half of the human population
D) c. Is relatively rare, affecting 2% to 3% of the
population
A

The administration of aspirin to a child with chickenpox


can cause:
A) c. Raynaud's phenomenon
B) d. The lesions to worsen
C) a. Mild internal bleeding
D) b. Reye's syndrome

Treatment for influenza is often limited to:


A) b. Antibiotics
B) c. Antiviral drugs
C) a. A specific antiflu drug
D) d. Supportive care only
D

Personal protective equipment designed to prevent a


tuberculosis exposure include:
A) c. A surgical gown
B) b. A full face mask
C) a. A disposable particulate respirator
D) d. Placing the patient on a nonrebreather mask
C

Gonorrhea is an STD caused by a:


A) b. Fungus
B) a. Bacteria
C) c. Parasite
D) d. Virus
B

The condition that results in bacteria passing the bloodbrain barrier and increasing cerebral vascular
permeability, thus increasing intracranial pressure, is
known as:
A) d. Tuberculosis
B) c. Meningitis
C) a. Hepatitis
D) b. HIV
B

Hepatitis E spreads most similar to:


A) d. Tuberculosis
B) c. HIV
C) a. Hepatitis A
D) b. Hepatitis B
C

Rabies primarily affects which body system?


A) d. Respiratory system
B) c. Central nervous system
C) b. Musculoskeletal system
D) a. Circulatory system

The body's second line of defense against infections is


the:
A) a. Antibodies
B) c. Skin
C) d. White blood cells
D) b. Inflammatory response
D

Hepatitis D becomes pathogenic when a patient has:


A) d. HIV
B) a. Hepatitis A
C) b. Hepatitis B
D) c. Hepatitis C
B

Which body system attempts to fight off infectious


diseases?
A) d. Respiratory system
B) a. Circulatory system
C) b. Immune system
D) c. Nervous system
C

Which of the following patients is at greatest risk for


developing chronic hepatitis after being exposed to
hepatitis B?
A) c. A 15-year-old male
B) d. A 30-year-old male
C) a. A newborn baby boy
D) b. A 5-year-old male
C

Which of the following is considered a first-line defense


against infection for the human body?
A) c. Skin
B) a. Antibodies
C) b. Inflammatory response
D) d. White blood cells
A

Vectors for the rubella virus are


A) c. Humans only
B) b. Humans and mosquitoes

C) d. Mosquitoes and some spiders


D) a. Humans and domestic dogs
A

Infectious agents that invade a host's cell to reproduce


are:
A) a. Bacteria
B) d. Viruses
C) b. Fungi
D) c. Protozoa
B

You are managing a 32-year-old male at the state prison


who is complaining of shortness of breath and weakness.
He has a fever and a cough, and he occasionally spits up
bloody phlegm. Appropriate Standard Precautions for this
patient are:
A) c. Gloves only
B) d. A face mask and a nonrebreather mask on the
patient
C) a. Gloves and a face mask
D) b. Gloves, a gown, a face mask, and eye protection
D

The rash from a scabies infection is described as:


A) b. Generalized redness and burning across the body
B) d. Patchy red lesions that are itchy
C) a. Acne anywhere on the body
D) c. Open painful ulcers around skin folds
B

Lice are treated with:


A) a. Antibiotics
B) c. Shaving the hair in the infected region
C) b. Intense heat
D) d. Topical medications and washes
D

While working in a college campus clinic, you have


observed that several students over the past 3 days
complaining of increased coughing with nausea and
vomiting. Your latest patient, a 19-year-old male, has
periods of uncontrollable spastic coughing, a mild fever,
and intermittent vomiting. What you notice most about
his cough is that he will be fine for several minutes and
then suddenly begin to cough forcefully for up to 1

minute. He is unable to control when and for how long he


coughs, and he seems to always cough in batches. You
suspect that there may be an outbreak of what disease
on campus?
A) c. Pertussis
B) d. Pneumonia
C) a. Asthma attacks
D) b. Influenza
B

Which of the following groups is at risk for exposure to


hepatitis A?
A) d. Nurses working in a prison first-aid station
B) a. A basketball team showering in the same showers
C) c. A patient living in a home for patients with
developmental disabilities
D) b. A group of friends traveling to Africa
C

Your patient has a known history of hepatitis B and HIV


and is complaining about abdominal pain. Appropriate
Standard Precautions include:
A) a. Gloves and a face mask
B) b. Gloves, a gown, a face mask, and eye protection
C) c. Gloves only
D) d. A face mask and a nonrebreather mask on the
patient
B

If, after being tested following an exposure to an


infectious disease at work, medical screening finds you to
be positive for the disease, the testing healthcare
provider is required to report the findings to:
A) b. You and OSHA
B) a. You alone
C) c. You and the Department of Public Health
D) d. You and your employer
B

A scabies infection develops when what type of parasite


burrows under the skin?
A) b. Mites
B) d. Worms
C) c. Ticks
D) a. Lice
A

Prehospital management of gastroenteritis may include:


A) a. Administering activated charcoal to kill the source of
the virus
B) d. Initiating IV fluids
C) c. Initiating IV antibiotics
D) b. Administering syrup of ipecac
B

Proper tick removal is accomplished by:


A) d. Squeezing the body and pulling it out
B) c. Placing a heated match stick against the tick
C) b. Grabbing the tick as close to the surface of the skin
as possible with tweezers and pulling it directly out of the
skin
D) a. Applying a nonbreathable lubricant onto the tick so
it self-withdraws
Which communicable diseases cause most communicable
disease-related deaths?
pneumonia
diarrheal diseases
tuberculosis
malaria
measles
HIV/AIDS.

Which populations are at risk for communicable disease?


Young children
Older adults
Immunosuppressed clients
Intravenous drug users
Health care workers

Which diseases have airborne transmission?


Measles
Chickenpox
Streptococcal infection
Tuberculosis
Pneumonia
Influenza

Which diseases have foodborne transmission?


Salmonellosis
Hepatitis A

Trichinosis
Escherichia coli (E. coli)

Which diseases have waterborne transmission?


Cholera
Typhoid fever
Giardia lamblia

Which diseases have vector-borne transmission?


Lyme disease
Rocky Mountain spotted fever
Malaria

Which diseases have direct contact transmission?


Sexually transmitted diseases (HIV, gonorrhea,
syphilis, genital herpes, hepatitis
B, C, D)
Infectious mononucleosis
Impetigo, lice, scabies

What are some examples of portals of entry?


Respiratory system
Gastrointestinal tract
Skin
Mucous membranes

What are some examples of portals of exit?


Respiratory system
Feces
Blood
Semen/vaginal secretions
Saliva
Skin

What are defense mechanisms against infections?


Natural immunity (from the body's antigen antibody
response)
Artificial immunity (through vaccination)
Active (vaccination with live, killed, toxoid)
Passive (from antitoxin or antibodies)

What diseases are reportable to the CDC?


AIDS
Anthrax
Botulism

Cholera
Diphtheria
Encephalitis
Giardiasis
Gonorrhea
Hepatitis A-D
Influenza activity
Legionellosis/Legionnaires' disease
Leprosy
Lyme disease
Malaria
Meningococcal infections
Mumps
Pertussis
Poliomyelitis
Rabies
Rocky Mountain spotted fever
Rubella
Rubeola (measles)
Salmonellosis
Shigellosis
Severe acute respiratory syndrome-associated
Coronavirus disease (SARSCoV)
Syphilis
Smallpox
Tetanus
Toxic shock syndrome
Trichinosis
Tuberculosis
Typhoid fever
Vancomycin-resistant Staphylococcus aureus (VRSA)
Varicella (chickenpox)

What are some primary prevention strategies related to


infectious diseases?
Prevent the occurrence of
infectious disease.
Educate the public
regarding the need for
immunizations, federal and
state vaccination programs,
and immunization laws such
as the "no-shots, no school"
legislation.
Counsel clients traveling
to other countries about
protection from infectious

diseases. Refer clients to


the health department
for information about
mandatory immunizations.
Educate the public
regarding prevention
of disease and ways to
eliminate risk factors
for exposure, such as
hand hygiene, universal
precautions, proper food
handling and storage, and
use of condoms.

What are some secondary prevention strategies related to


infectious diseases?
Increase early detection
through screening and case
finding.
Refer suspected cases of
communicable disease for
diagnostic confirmation and
epidemiologic reporting.
Treat postexposure
infections (hepatitis A,
rabies).
Quarantine clients when
necessary.

What are some tertiary prevention strategies related to


infectious diseases?
Decrease complications and
disabilities due to infectious
diseases through treatment
and rehabilitation.
Monitor treatment
compliance, including
directly observed therapy.
Prevent reinfection. Identify
community resources.
Disasters

What are the three levels of disaster management?


preparedness
response
recovery

What are the s/sx of inhalation anthrax?


Sore throat
Fever
Muscle aches
Severe dyspnea
Meningitis
Shock

What are the s/sx of botulism?


Difficulty swallowing
Progressive weakness
Nausea, vomiting, abdominal cramps
Difficulty breathing

What are the s/sx of smallpox?


High fever
Fatigue
Severe headache
Rash (begins on face and tongue, quickly
spreading to the arms and legs, then hands and
feet, within 24 hr) that turns to pus-filled lesions
Vomiting
Delirium
Excessive bleeding

How long does the small pox vaccine last?


10 years

What are the s/sx of ebola?


Sore throat
Headache
High temperature
Nausea, vomiting, diarrhea
Internal and external bleeding
Shock

What is the role of the community nurse in the event of a


bioterrorism attack?
Participate in planning and preparation for immediate
response to a bioterrorist event.
Identify potential biological agents for bioterrorism.
Survey for and report bioterrorism activity (usually to
the local health department).
Promptly participate in measures to contain and

control the spread of infections


resulting from bioterrorist activity.

What is involved in assessment of a bioterrorism threat?


Is the population at risk for sudden high disease rates?
Is the vector that normally carries a specific disease
available in the geographical area
affected?
Is there a potential delivery system within the
community?

How does a community health nurse recognize a


bioterrorism event?
Is there a rapidly increasing disease incidence in a
normally healthy population?
Is a disease occurring that is unusual for the area?
Is an endemic occurring at an unusual time? For
example, is there an outbreak of
influenza in the summer?
Are there large numbers of people dying rapidly with
similar presenting symptoms?
Are there any individuals presenting with unusual
symptoms?
Are there unusual numbers of dead or dying animals,
unusual liquids/vapors/odors?

What are some primary prevention strategies for


bioterrorism?
Preparation with
bioterrorism drills, vaccines,
and antibiotics for exposure
prophylaxis
Bioterrorism planning
Design a bioterrorist
response plan using the
most probable biological
agent in the local area.
Assess and locate the
local facilities that have
Level I, Level II, Level III,
and Level IV biosafety
gear.
Identify the chain of
command for reporting
bioterrorist attacks.
Define the nursing
roles in the event of a

bioterrorist attack.
Set up protocols for
different biosafety levels
of infection control and
containment.

What are some secondary prevention strategies for


bioterrorism?
Early recognition
Activation of bioterrorism
response plan in response to
a bioterrorist event
Immediate implementation
of infection control and
containment measures,
including decontamination,
environmental disinfection,
protective equipment,
community education/
notification, and quarantines
Screening the population for
exposure, assessing rates of
infection, and administering
vaccines as available
Assisting with and educating
the population regarding
symptom identification
and management
(immunoglobulin, antiviral,
antitoxins, and antibiotic
therapy, depending on the
agent)
Monitoring mortality and
morbidity

What are some tertiary prevention strategies for


bioterrorism?
Rehabilitation of survivors
Monitoring medication
regimens and referrals
Evaluating the effectiveness
and timeliness of the
bioterrorism plan

What are some example of Class A biological agents?


highest priority agents, posing a risk to national
security because they are easily transmitted and have high

mortality rates.
Examples include smallpox (variola), botulism toxin,
anthrax, tularemia,
hemorrhagic viral fevers, and plague.

What are some example of Class B biological agents?


second highest priority because they are moderately
easy to disseminate and have moderate morbidity rates
and low mortality rates.
Examples include typhus and cholera.

What are some example of Class C biological agents?


the third highest priority, comprising emerging
pathogens that can be engineered for mass dissemination
because they are easy to
produce, and/or have a potential for high morbidity and
mortality rates.
Examples include Hantavirus.
Immunity
The state of being protected against a certain disease

Antigen
Substance capable of triggering an immune response

Antibodies
Protein that fights against a specific antigen

Lymphocytes
Specialized white blood cells (t & b)

Active immunity
Developed from natural or artificial processes (flu vaccine)

Passive immunity
Your body receives antibodies from another person or
injection

HIV
Damages T cells and leads to aids

Physical barriers
Skin, nose, hair

Chemical barriers
Tears, stomach acid, mucus

Immune system
A network of cells, tissues, organs, and chemicals that fight
off pathogens

Inflammatory response
A reaction to tissue damage caused by injury or infection

Phagocytes
White blood cells that attack invading pathogen

Helper T cells
Trigger the production of B cells and killer T cells

Killer T cells
Attack and destroy infected body cells

Suppressor T cells
Turn off T cells when infection is cleared

B cells
Produce antibodies

CDC
Centers for disease control

WHO
World health organization

Live-virus vaccines
Vaccines made from pathogens in labs

Killed-virus vaccines
Vaccines using dead pathogens

Toxoids
Vaccines with inactivated toxins from pathogens &
simulates the growth of antibodies

New and second generation viruses


It's a cutting edge of disease fighting technology

Pathogens
Microorganisms that cause disease

Rickettsias
Resembles a bacteria and enters the body through insect
bites

Protazoan
Single celled microorganisms that are larger and more
complex than bacteria

Vectors
And organism that carries and transmits pathogens to
humans or animals

Communicable disease
Disease that is spread from one living organism to another
or through the environment
This policy serves as the Sheriff's Office
"Infectious Disease Exposure Control Plan" and the
"Biomedical Waste Operating Plan."

Any microorganism that is spread by droplet nuclei


through the air by coughing, sneezing, or talking.
Airborne Pathogens

Pathogenic micro-organisms that may be present in blood


or other body fluids from infected individuals and can
spread to others through direct or indirect contact with
those fluids or infected materials
Bloodborne Pathogens

An infectious disease capable of being passed to another


by direct or indirect contact with an infected person, their
body fluids, or infected materials.
Communicable Disease

The presence, or the reasonably anticipated presence, of


blood or other potentially infectious materials on an item
or surface.
Contaminated

The use of physical or chemical means to remove,


inactivate, or destroy blood borne pathogens on a surface
or item to the point where they are no longer capable of
transmitting infectious materials. The surface or item is
then rendered safe for handling, use, or disposal.
Decontamination

Specific eye, mouth, other mucous membrane, or skin, or


parenteral (piercing) contact with blood or other
potentially infectious materials including sprayed and/or
splattered infectious materials.
Exposure Incident

Human body fluids including: blood, urine, vomitus,


semen, vaginal secretions, tears, saliva, feces, amniotic
fluid, and/or any other body fluid that is visibly
contaminated with blood. Also all body fluids in situations
where it is difficult or impossible to differentiate between
body fluids.
Infectious Materials

Specialized clothing or equipment worn by personnel for


protection against a potential hazard. General work
clothes (e.g., uniforms, pants, shirts, or blouses) are not
considered personal protective equipment.
Personal Protective Equipment (PPE)

Items that could release potentially infectious or


contaminated materials (liquid or semi-liquid) when
handled; such as sharps or blood soaked clothing.
Regulated Waste

Any individual, living or dead, whose blood, or any other


potentially infectious body fluids or materials may be a
source of occupational exposure to another individual.
Examples include, but are not limited to, inmates, visitors,
hospital and clinic patients, or human remains.
Source Individual

A set of precautionary practices to support infection


control. According to the concept of universal
precautions, all human blood and body fluids are treated
as if known to be infectious for HIV, hepatitis, and other
blood borne pathogens
Universal Precautions

These are examples of


1. Viral;
2. Bacterial; and
3. Fungal.
Airborne Pathogens

These are examples of


1. Hepatitis B Virus (HBV);
2. Hepatitis C (HBC);
3. Human Immunodeficiency Virus (HIV);
4. Syphilis.
Bloodborne Pathogens

Bloodborne Pathogen exposure primarily occurs with


contact with body fluids or droplet spray from talking,
coughing, vomiting, or contaminated objects. Body fluids
include:
1. Blood;
2. Urine;
3. Vomitus;
4. Semen;
5. Vaginal Secretions;
6. Tears;
7. Saliva;
8. Feces;
9. Amniotic fluid; or
10. Any other body fluid where blood is visible

Exposure to airborne pathogens occurs when a


contaminant is inhaled. Types of diseases transmitted
through the air are:
1. Meningitis;
2. Influenza;
3. Pneumonia;
4. Tuberculosis
5. Measles;
6. Mumps;
7. Pertussis; and
8. Varicella (chicken pox).

Exposure does not always result in an infection. The


likelihood of transmission depends on the following:
1. How contagious the infectious person is;
2. Where the exposure occurs;

3. How long the exposure lasts;


4. How healthy you are at the time of exposure.

Modes of Infectious Disease Transmission


1. Cardiopulmonary Resuscitation (CPR)
2. Fights and Assaults
3. Human Bites
4. Searches and Evidence Handling
5. Transportation
6. Breath Alcohol Testing and Drug Testing
7. Deceased Persons and Body Removal
8. Autopsies
9. Crashes With Injury

When blood is present and a suspect displays combative


or threatening behavior, an officer must attempt
to put on gloves as soon as conditions permit

Personnel should be concerned with the possible HIV and


HAV / HBV exposure through CPR administration.
Therefore, gloves and a protective resuscitation mask
("pocket mask") with _________________ must always be
used when administering CPR.
a one-way valve

Should an employee be bitten by anyone, the employee


shall clean the wound with __________________ as soon as
possible.
soap and water

The following precautions can help reduce the risk of


infection during search situations:
(1) Caution must be used when searching
prisoner/suspects' clothing. Discretion should be used to
determine if an employee should search a
prisoner/suspect's clothing or if the prisoner/suspect should
empty his own pockets;
(2) An employee should wear protective gloves if exposure
to body fluids is likely to be encountered;
(3) If cotton gloves are to be worn for evidence handling,
they should be worn over protective, disposable gloves
when exposure to body fluids may occur;
(4) A flashlight should be used when searching areas in
which the available light does not permit adequate vision;
(5) Containers such as purses and bags should be searched
by turning the container up-side-down to empty the

contents. Small compartments must not be searched by


the finger or hand-probe method;
(6) Puncture-proof containers should be used to store any
sharp objects that may be contaminated with body fluids;
(7) Caution must be used when staples are used to seal
evidence so as not to tear gloves or puncture skin;
(8) Wet or damp items of evidence should be air-dried prior
to placement in any air-tight containers;
(9) When practical and appropriate, puncture resistant
gloves should be worn anytime a search of a person,
vehicle or place is conducted. All officers, and particularly
those involved with identification and crime scene
processing, must be alert for the presence of sharp objects
such as hypodermic needles, knives, razors, broken glass,
nails, or other sharp objects. Sharp objects should not be
handled with unprotected, bare hands but rather by
mechanical means, such as forceps or tongs. Contaminated
evidence shall not be transported, handled, nor stored with
or near any food items. An employee who has had contact
with or has been near contaminated items must not smoke
or consume food until after a thorough hand washing.

All procedures involving any potentially infectious


material will be performed in such a manner as to
minimize __________, _________ and __________ of these
materials
splashing, spraying and splattering

All personnel who exhibit open cuts or sores and will be


performing tasks involving anticipated exposure to
potentially infectious materials are required to put on
___________________ or protective equipment as
appropriate BEFORE getting in contact with potentially
contaminated material.
waterproof bandages

Specially marked ___________________________ will be used


to dispose of contaminated sharps.
puncture proof containers

When moving contaminated puncture proof containers


from the area of use, they will be
closed immediately prior to moving and placed in a red
plastic biohazard bag and tape-sealed if leakage is possible

What are Personal Protective Equipment:

1.
2.
3.
4.
5.
6.

Gloves;
Masks, Eyewear, and Gowns;
CPR mask;
Boot or shoe covers;
Disposable Personal respirator
Water-soluble plastic bags (clear w/ red stripe).

If an exposure occurs, the following procedures will be


initiated:
1. Clean the exposed area with soap and warm water or
another approved substitute (gel disinfectant, alcohol
towelettes, 10% bleach/water solution);
2. Remove contaminated clothing as soon as possible;
saturated clothing will be removed immediately, and be
placed in a red biohazard bag. The "Biohazard" bag of
clothing will be disposed of at the designated collection site
at the Property Room, Pretrial Detention Facility, or nearest
Jacksonville Fire Station.
3. Decontamination of surfaces and equipment:
4. Complete Exposure Incident Form as described in Article
V below;
5. Test the source:

Vehicles which have become contaminated by blood,


vomitus or other body fluids should be cleaned
immediately following use and prior to further utilization
of the vehicle. It is the employee's responsibility to ensure
the vehicle is decontaminated in the proper manner:
1) Disinfect with the bleach and water solution and clean
with hot soapy water wiping with a towel or other
absorbent cloth;
(2) Spray with an effective germicidal disinfectant; and
(3) The towel or other material used to wipe up the body
fluid shall be disposed of by placing it in a sealable,
leakproof, red or biohazard-labeled bag or container.

In reference to testing, If the source is an incarcerated


inmate
(1) The exposed employee shall inform the ITR
officer/watch commander at the PDF that he has been
exposed to the suspect's blood;
(2) The watch commander must notify the charge nurse at
the M2 clinic;
(3) The charge nurse will bring the inmate to the clinic and
will take blood samples:

In reference to testing, If the source is a suspect absentee


booked,
JSO will request the Hospital to test the suspect

In reference to testing, If the source is a suspect but dies,


JSO will request
the Medical Examiner's Office to test the suspect;

In reference to testing, If the source is a victim (i.e.,


officer exposed assisting a victim of an accident as first
responder), who will request the hospital treating the
victim to test;
the Infection Control Nurse

The Infection Control Nurse must ensure that lab results


are obtained in a timely manner and reported back to the
officer through direct phone call. An e-mail informing the
officer that the results are available must be sent to
ensure that the officer will be notified. E-mails will be
copied to ______________________________.
the JSO Occupational Manager

The officer exposed must contact ____________________ if


the lab results have not been reported within 24 hours.
the Clinic at the PDF

If the HBV test is positive, the officer should contact ___


_________________ to get instructions on where to go to
start prophylactic treatment.
the claim adjuster

If the HCV test is positive, there's no treatment available


at this time to prevent the infection. The officer should
contact _________________________________ to start medical
follow up.
the workers compensation adjuster

The exposed employee or other available member must


notify his supervisor immediately and complete an
Electronic Exposure Incident Report Form. The supervisor
of the exposed employee, the Infection Control Nurse,
and ____________________________ shall receive email
notification of the incident.
the Occupational Manager

The following job classifications within the Sheriff's Office


have been identified as "at risk" to occupational exposure
to blood borne pathogens or other communicable
diseases:
1. All sworn officers;
2. All corrections officers;
3. All reserve officers; and
4. Civilian employees assigned to the following units:
a. Latent Print Unit;
b. Chaplaincy personnel;
c. Central Records Unit personnel assigned to handle duties
at the Public counter involving fingerprinting of persons;
d. Property and Evidence Unit Personnel;
e. Security Guards;
f. Bailiffs; or
g. Department of Corrections (DOC) personnel

All Sheriff's Office employees who have been identified as


having exposure risk to blood or other potentially
infectious materials will be offered ________________
_________, after receiving training and within ten days of
initial assignment.
the Hepatitis B vaccine

The vaccine will be offered as soon as possible, but in no


event later than ________, to all unvaccinated employees
who have rendered assistance in a first aid incident
involving the presence of blood or other potentially
infectious material regardless of whether the employee
has incurred an exposure incident.
24 hours

Employees who initially decline the vaccine but later wish


to have it may receive it at what cost.
No cost

Training for all Sheriff's Office employees in at risk


positions will be conducted at entry-level positions and
prior to being assigned duties where occupational
exposure could occur. Training of employees will include
an explanation of the following:
1. State and/or Occupational Safety and Health
Administration (OSHA) standards for bloodborne
pathogens;
2. Epidemiology and symptomatology of bloodborne
diseases;

3. Modes of transmission of bloodborne pathogens;


4. The Sheriff's Office General Order on communicable
diseases, including exposure control procedures;
5. Procedures which might cause exposure to blood or
other potentially infectious materials;
6. Protective measures taken to prevent exposure to blood
or other potentially infectious materials;
7. Personal protective equipment provided by the Sheriff's
Office;
8. Post exposure evaluation and follow-up procedures;
9. Signs, labels, and color coding used at the Sheriff's
Office regarding biohazards; and
10. The Hepatitis B vaccination program of the Sheriff's
Office.

Questions in reference to training materials should be


directed to
Infection Control Disease Nurse

The Sheriff's Office will at least _________ review the


exposure control plan with consideration for updating
procedures designed to eliminate or minimize
occupational exposure.
annually

What is the incubation time for Human Immunodeficiency


Virus/Acquired Immune Deficiency Syndrome (HIV/AIDS)
May be up to ten years
What is the infectious agent of Influenza?
Influenza Type A,B,C

Which Influenza type is associated with widespread


epidemics and pandemics?
Influenza Type A

What is Influenza type A isolated from?


humans, horses, swine, and birds

Which Influenza are isolated from humans only?


* Influenza Type B
* Influenza Type C

Which Influenza is infrequently associated with regional


and widespread epidemics?

Influenza Type B

Which Influenza is associated with sporadic cases and


minor localized outbreaks?
Influenza Type C

Which antigenic variation consists of a minor or gradual


changes in HA and NA and is only associated with
Influenza Types A and B?
Antigenic drift

Which antigenic variation consists of a major change in H


and A typically associated with pandemics and only
happens with Influenza Type A?
Antigenic shift

What is the incubation period for Influenza?


1 to 3 days

What is the period of communicability for Influenza?


3 to 5 days from clinical onset

What is the preventive measure for Influenza?


Vaccination

What are the identifying signs and symptoms of


Menigococcal Meningitis?
* Intense headache
* Stiff Neck
* Petechial rash

What is the case fatality rate for Menigococcal Meningitis?


5% to 15%

What is the infectious agent for Menigococcal Meningitis?


Neisseria meningitides

What is the treatment for Menigococcal Meningitis?


*Penicillin initially IMMEDIATELY
*DO NOT wait for lab results to return
*Ampicillin

What is the occurance of Menigococcal Meningitis?


Groups B, C, and Y are common to the U.S.

What is the incubation period for Menigococcal


Meningitis?
3 to 4 days

Meningococci usually disappear from the nasopharynx


how long after treatment?
Within 24 hours

What variety of diseases are caused by Group A


streptococci?
* Streptococcal sore throat
* Scarlet fever
* Rheumatic fever

What are the signs and symptoms of streptococcal sore


throat?
* Sore throat
* Exudative tonsillitis
* Pharyngitis
* Tender, enlarged cervical lymph nodes

Scarlet fever occurs when the infecting strain of


streptococcus produces what toxin?
Pyrogenic exotoxin

What are the signs and symptoms of scarlet fever?


* Same SNS as sore throat
* Accompanying rash that feels like sandpaper
* Strawberry (swollen) tongue

What is the infectious agent of streptococcal diseases?


Streptococcus pyogenes

What streptococci causes the majority of infections?


Group A

What is the treatment for streptococcal diseases?


Benzathine Penicillin G-IM

What is the incubation period for streptococcal diseases?


1 to 3 days

What is the leading cause of death in adults from an


infectious disease around the world?
Tuberculosis

What is the most common type of tuberculosis around the


world?
Pulmonary tuberculosis

What are the early symptoms of tuberculosis?


* Fatigue
* Fever
* Night Sweats
* Weight Loss

What are the advanced symptoms of tuberculosis?


* Cough
* Hemoptysis
* Hoarseness
* Chest Pain

What is the infectious agent for tuberculosis?


Mycobacterium tuberculosis

What are the clinical signs and symptoms of tuberculosis?


* Reactor to PPD-S
* Chest X-ray

What is the confirmatory diagnosis for tuberculosis?


Isolation of Mycobacterium tuberculosis on culture

What is the treatment of choice for tuberculosis?


Isoniazid- INH

What is the incubation period for tuberculosis?


2 to 10 weeks

What increases the risk for tuberculosis and shortens the


incubation period?
HIV

What are the signs and symptoms of diphtheria?


* Characteristic lesion
* Enlarged, tender cervical lymph nodes

Describe the characteristic lesions of diphtheria.


A patch or patches of an adherent grayish membrane with
a surrounding inflammation

What is the infectious agent of diphtheria?


Corynebacterium diptheriae

How is diphtheria diagnosed?


Confirmed by bacteriologic examination of lesions

What should be given for diphtheria, immediately after


bacteriologic specimens are taken, without waiting for
results?
Antitoxin (only antitoxin of equine origin is available)

What is the incubation period for diphtheria?


2 to 5 days

What is the preventive measure for diphtheria?


Active immunization

When controlling a patient with diphtheria, what must be


considered?
Isolation of one person versus Quarantine of a groups of
individuals

What is the most characteristic clinical feature of Rubella


that precedes the rash by 5 to 10 days?
Postauricular, occipital and posterior cervical
lymphadenopathy

What is a mild febrile viral disease characterized by


diffuse punctate and maculopapular rash resembling that
of measles or scarlet fever?
Rubella

What is Rubella also known as?


German Measles

What are the characteristic signs and symptoms of


Rubella?
* Conjunctivitis
* Postauricular, occipital and posterior cervical
lymphadenopathy

What is the infectious agent for Rubella?


Rubella virus

What laboratory test is used to confirm Rubella?


Fourfold rise via ELISA

What is the incubation period for Rubella?


* Range: 14 to 21 days
* Average: 14 to 17 days

When are patients highly communicable when infected


with Rubella?
1 week before and at least 4 days after onset of rash

What is an identifying sign and symptom of Rubeola


(Measles)?
Koplik spots on the buccal mucosa

What is another name for Rubeola?


Measles

What are the complications that can come with Rubeola?


* Otitis media
* Pneumonia

A herd immunity of > how much may be needed to


interrupt transmission?
94%

What is the infectious virus of Rubeola?


Measles virus

Pneumococcal Pneumonia is an acute bacterial infection


characterized by what "tall-tale" sign and symptom?
Productive cough of "rusty" sputum

What is the infectious agent of Pneumococcal


Pneumonia?
Streptococcus pneumoniae

What is the treatment for multiresistant strains of


Pneumococcal Pneumonia infections?

* Vancomycin
or
*Chloramphenicol

What is a method of control for Pneumococcal


Pneumonia?
Administration of polyvalent vaccine to high-risk persons

Legionellosis is an acute bacterial disease with what two


currently recognized, distinct clinical and epidemic
manifestations?
* Legionnaire's disease
* Pontiac fever

What is the infectious agent of Legionellosis?


Legionellae pneumophila

What is the treatment for Legionellosis?


E-mycin

What is the occurance of Legionellosis?


Sporadic cases and outbreaks are recognized more
commonly in summer and autumn

What is the reservoir for Legionellosis?


Primary aqueous:
-Hot water systems (showers)
-Air-conditioning cooling towers
-Evaporative condensers
-Humidifiers
-Whirlpool spas
-Respiratory therapy devices
-Decorative fountains
-Hot and cold water taps and showers
-Hot tubs
-Creeks and ponds and the soil from their banks

What is the mode of transmission for Legionellosis?


Epidemiologic eveidence supports airborne transmission

What is the incubation period for Legionellosis?


* Range from 2 to 10 days
* Average from 5 to 6 days

What is the common cold?


An acute catarrhal infection of the upper respiratory tract
lasting 2-7 days

What are the major etiologic agents of the common cold


in adults?
Rhinoviruses

Where is the highest incidence of the common cold?


Children under 5 years and gradually declines with
increasing age

What are the signs and symptoms of the common cold?


* Coryza
* Sneezing
* Lacrimation
* Irritated nasopharynx
* Chilliness
* Malaise

What is the approximate number of organisms required to


cause disease?
Dose

What is caused by ingestion of a pathogenic organism


which multiplies within the gastrointestinal tract?
Infection

Infections usually have longer incubation periods than


intoxications and are accompanied by what?
fever

Typically how long are infections?


6 hours to 45 days

What is caused by ingestion of toxins which are formed in


tissues of certain plants and animals?
Intoxication

Is fever often present or absent with intoxications?


absent

What is the incubation period for food intoxication?

* short
* 1 min to 6 hours

What types of poisons in food are there?


* Mushroom poisoning
* Poisonous plants
* Paralytic shellfish poisoning (PSP)
* Ciguatera fish poisoning
* Scrombroid dish poisoning
* Organophosphorous insecticides
* Cooking materials contaminated with toxic metals

What are the predominant signs and symptoms for


mushroom poisoning?
* Abdominal pain
* Muscle cramps

What food are involved with mushroom poisoning?


* Amanita (think super mario)
* Galerina (Brown)
* Gyromitra (Brain shaped, no stalk)

What are the etiologic agents of mushroom poisoning?


* Toxins
* Alkaloids

How many species of poisonous plants are known to


cause illness or death?
more than 700

Approximately how long does it take for paralytic shellfish


poisoning to resolve itself?
2 to 3 days

What is the source of poisoning for Paralytic shellfish


poisoning (PSP) ?
* dinoflagellates (algae) in shellfish with concentrated
neurotoxins
* Associated with massive algae blooms known as "red
tide"

What is the occurence of Paralytic shellfish poisoning


(PSP)?
* West Coast: May to Oct
* East Coast: Aug to Oct

What are the tell-tail sign and symptoms of Ciguatera fish


poisoning?
* Vomiting
* Hypotensive
* Pain and weakness in lower extremities
* Temperature reversal

What is the source of poisoning for ciguatera fish


poisoning?
* Dinoflagellate Gambierdiscus toxicus
* Released from algae
* Small fish eat algae
* Larger fish eat smaller fish
* Toxin Magnified through food chain

What is the preventive measure for Ciguatera fish


poisoning?
Avoid eating large predatory fish

What are the signs and symptoms of scombroid fish


poisoning?
* Tingling and burning sensation around the mouth
* Facial Flushing
* Rash

What is scombroid fish poisoning also known as?


Histamine poison

What is the preventive measure for scombroid fish


poisoning?
Adequate refrigeration and irradiation

What are the signs and symptoms of organophosphorous


insecticides?
* Blurred vision
* Chest pain cyanosis
* Twitching, convulsions

To prevent poisoning from cooking materials


contaminated with toxic metals, be sure to not use acidic
foods in what types of material/cookware?
* cadmium materials
* galvanized cookware

What disease is caused by water containing nitrates


greater than 45 ppm and is especially dangerous to
children?
Methmo-globinemia

What is the most common form of botulism in the USA?


Intestinal (infant)

What is a severe intoxication resulting from ingestion of


preformed toxins in contaminated foods?
Foodborne botulism (classical)

What are the signs and symptoms of foodborne botulism?


* Acute cranial nerve impairment
* Blurred or double vision
* Dry mouth
* Vomiting

What are the signs and symptoms of intestinal (infant)


botulism?
* Loss of head control
* Hypotonia (floppy baby)
* May develop respiratory arrest

What is the infectious agent of botulism?


Clostridium botulism

What type of botulism is related to fish, seafood, and


meat from marine mammals?
Type E

What is the treatment for botulism?


1 vial of polyvalent (AB or ABE) botulinum antitoxin (IT stat)

Who is the only carrier of the treatment for botulism (1


vial of polyvalent (AB or ABE) botulinum antitoxin) ?
CDC Atlanta

Where is the occurance of botulism?


More common where home canning is popular

What is the reservoir for botulism?

* Soil
* Marine sediment
* Honey

What is the period of communicability for intestinal


(infant) botulism?
Toxins are found for weeks to months in feces

What is the infectious agent of Campylobacter Enteritis?


Campylobacter jejuni

What is the treatment for Campylobacter Enteritis?


* None
* Electrolyte replacement

What are the reservoirs for Campylobacter Enteritis?


* Most raw poultry
* Cattle
* Puppies or kittens

What is the mode of transmission for Campylobacter


Enteritis?
* Undercooked chicken
* Contaminated food or water

What is the susceptibility and resistance of


campylobacter enteritis?
Immunity to serologically related strains follows infection

What is a sign and symptoms of Clostridium Perfringens?


Colic followed by diarrhea

What strain of Clostridium Perfringens causes necrotizing


enteritis?
Type C

What is the reservoir for Clostridium Perfringens?


* Soil
* GI tract of healthy persons and animals

Almost all outbreaks of Clostridium Perfringens are


associated with inadequately heated or reheated what?
* Meats
* Stews

* Meat Pies
* Gravies

What is a tell-tale sign and symptom of Staphylococcus


aureus?
Projectile vomiting

What is the infectious agent of staphylococcus


intoxication?
Staphylococcus aureus

What percentage of people are reservoirs/ carriers of


staphylococcus aureus?
25%

What has associated conditions of infected cuts, boils,


acne, and URI?
Staphylococcus aureus

What is a preventive measure for staphylococcus aureus?


Keep hot foods hot (above 140F) and cold foods cold
(below 41F)

Salmonellosis almost always has what present?


Fever

Incidence rates of salmonellosis are highest in what


group?
Small children

What is the mode of transmission for salmonellosis?


Fecal oral

For salmonellosis, what is the mode of transmision


associated with?
* Raw or undercooked eggs or egg products
* Raw milk
* Meat
* Poultry

What is the incubation period range for Salmonellosis?


6 to 72 hours

What agents are contraindicated for Shigellosis and may


prolong the illness?
Antimotility

What is the occurance of Shigellosis?


Peaks in summer

What is the mode of transmission of Shigellosis?


Direct or indirect fecal-oral from a patient or a carrier

What is the average incubation period for Shigellosis?


1 to 3 days

What vector is commonly associated with Shigellosis?


Flys

What is a method of control for Shigellosis?


Control Flies

How many successive ( 24 hours apart) negative fecal


samples greater than 48 hours after antibiotic therapy
must Food and Childcare workers provide for Shigellosis
and Escherichia Coli?
2

What are the signs and symptoms of Typhoid fever?


* Sustained fever
* Severe headache
* Rose spots on the trunk in 25% of white patients
* Typically constipation more than diarrhea

What is the infectious agent for Typhoid fever?


Salmonella typhi

What is the treatment for Typhoid fever?


Chloramphenicol

What is the average incubation period for Typhoid fever?


8 to 14 days

What is the method of control for Typhoid Fever?


* Control flies
* Immunization with booster doses every 2 years

How many successive ( 24 hours apart) negative fecal


samples greater than 48 hours after antibiotic therapy
must Food and Childcare workers provide for Typhoid
Fever?
3

What are the signs and symptoms for Cholera?


* Rice watery stools
* Rapid dehydration
* Acidosis
* Circulatory collapse

Classic and ET Tor are both biotypes associated with what


disease?
Cholera

What is the infectious agent for Cholera?


Vibrio cholerae

What is the treatment for Cholera?


* Prompt fluid replacement with oral rehydration (1 liter
solution)
* Treat for shock
* Antimicrobials

What is the average incubation period for Cholera?


2 to 3 days

What are the signs and symptoms of Vibrio


Parahemolyticus?
* dysentery-like illness
* bloody or mucoid stools
* high fever
* high WBC

What is the infectious agent for Vibrio Parahemolyticus?


Vibrio parahaemolyticus

What does Vibrio parahaemolyticus require to produce


optimal growth?
High levels of salt

What is the reservoirs for vibrio parahemolyticus?

*
*
*
*

Marine silt (cold season)


Coastal waters (warm season)
Fish (warm season)
Shellfish (warm season)

What is the mode of transmission for Vibrio


parahemolyticus?
* Ingestion of raw or inadequately cooked seafood
* Cross-contamination by handling raw seafood or rinsing
with contaminated seawater
What is special about the period of communicability for
vibrio parahemolyticus?
Non-communicable from person to person

When preventing vibrio parahemolyticus, how must you


make sure to heat your seafood?
15 minutes at 160F

Escherichia coli is a self-limiting illness that last how long


in average?
8 days

What is the infectious agent for Escherichia coli?


Escherichia coli

Escherichia coli is cultured using what media?


McConkey-sorbitol

What is the treatment for Escherichia coli?


* Fluid and electrolyte replacement
* Antibiotic treatment is uncertain

What is the reservoir for Escherichia coli?


* Cattle
* Humans

What is the period of communicability for adults with


Escherichia coli?
1 week

What is the period of communicability for children with


Escherichia coli?
3 weeks

How must beef be cooked in order to prevent Escherichia


coli?
155F internal temperature
Power to create disease
Virulence

Sites of nosocomial infections in order from most to least


common
Respiratory tract, Surgical site wounds, GI tract, Urinary
tract, Blood (bacteremia) and skin.

Occurs when pathogens change in ways where they can


survive and reproduce in the presence of antibiotics that
once stalled or killed them
Antimicrobial Resistance

Microorganism, only replicates in a host, infects other


cells
Virus

Singled celled, small, capable of growing outside of


human cells
Bacteria

Eukaryotic organisms, acquired from contact,


molds/yeasts, resist destruction by the body
Fungi

Lives in or on another organism, feeding off that organism


and using that organisms nutrition
Parasites

What are opportunistic organisms?


Microorganisms that are not usually considered pathogens.
However, they may cause infection if the resistance of the
host is decreased from events such as immunosuppression,
trauma, illness

Modes of transmission
Droplet, Vector (vehicle transmission... aka mosquitoes),
Airborne (droplet nuclei, remain suspended in air, varicella)

Period between exposure to an infection and the


appearance of the first symptoms, varies with disease
Incubation

Period between the appearance of initial symptoms and


the full development of a rash or fever
Prodromal phase

Full manifestation of symptoms


Chronic illness

Period of time after acute symptoms to full recovery of


health
Convalescence

Most common pathogens for nosocomial infections


Staphylococci, Pseudomonas, Enterobacter aerogenese,
and Escherichia coli

immunity produced by individuals own response


Active

Immunity developed in another individual and transferred


Passive

Immunity obtained through normal everyday living


Natural

Immunity obtained through a vaccine


Artificial

Contact with antigen through infection, chickenpox,


measles, mumps is which kind of immunity
active natural

Injection of live or killed antigen


Active artificial

Immunity that occurs from mother to child in utero or


through breastfeeding
passive natural

Immunization with serum from immune human

Passive artificial

What are the types of communicable diseases?


Measles (Rubeola), Mumps, Rubella (german measles),
Mononucleosis, Influenza

How do you know to differentiate between Measles


(Rubeola) and Rubella (german measles) aka.. completely
different virsuses....
Koplik spots on buccal mucosa

Viral disease, transmitted by direct contact with droplets


of an infected person..
Measles (Rubeola)

Present in nasal secretions, blood, stool, and urine of


infected persons
Rubella

Measles (Rubeola) more dangerous for


babies

Rubella more dangerous for


pregnant mother

Purple under skin


Purpura

purple dots under skin


Petechiae spots
Is Chicken Pox Bacterial or Viral?
Viral

What systems are affected with Chicken Pox?


Respiratory and Integumentary

What is the incubation period of Chicken Pox?


2-3 weeks

What is the communicability span of Chicken Pox?


1 day before the lesions appear and until all have crusted
over

What are some signs and symptoms for the Chicken Pox?
Fever, Fatigue, Upper Respiratory Infection
Maculare Rash that is more concentrated toward the center
at the body and thins out towards the distal part of the
body.

What are Diagnostic Studies that could be run for Chicken


Pox?
Look at the patient history, Determine if the rash is what
you would expect to see, Viral Culture and Antibody Screen

What are some items you can do for Therapeutic


Management when treating the Chicken Pox?
Acyclovir (medication), Varicella Zoster Immune Globulin,
Antipruritics, Analgesia, Oatmeal Baths, Avoiding Products
Containing Aspirin

What immunizations can be given to help prevent


Chicken Pox ?
Varicella Vaccine given at 12-15 months of age, and again
at 4-6 years

What are some complications that Chicken Pox can give?


SEcondary Bacterial Infection, Cellulitis, Pneumonia,
Encephalitis

What kind of Nursing Care should you implement for a


client with Chicken Pox?
Droplet and Contact Precautions
Skin care (Calamine and Oatmeal Baths)
Diversional Activities
Keep environment Cool
Have them apply pressure instead of scratching

As a nurse what can we diagnose with someone who has


the Chicken Pox?
Impaired skin integrity, Impaired comfort, Isolation, Risk for
infection

Is Mono Viral or Bacterial?


Viral

What are some signs and symptoms for Mono?

Fever, Fatigue, HA, Sore Throat, Painful Lymphadenopathy,


Epigastric Pain, Spleenomegaly

What are some Diagnostic Studies that can be run for


Mono?
Heterophil Antibody Screen, Monospot,
CBC with Differential Liver Enzymes

What is some Therapeutic Management techniques that


could be implemented for Mono?
Encourage Support from the family, Antipyretics,
Encourage Hydration, Corticosteroids, Rest, Give Throat
lozenges or gargles

What immunizations can be given to prevent Mono?


There are currently no Immunizations

What are complications that can occur due to Mono?


Though couching is the front runner other complications
follow shortly behind such as:
Tonsillitis
Ruptured Spleen
Hepatitis
Hemolytic Anemia

What Nursing care should be implemented for those


suffering from Mono?
Droplet and Contact Precautions
Frequent PO fluids and Soft Food
Restful Environment
Education on Avoiding Contact Sports till the Spleen returns
to normal
Analgesias
Throat Gargles and Lozenges

What are some items we can Diagnose for those with


Mono?
Isolation
Risk for Injury (Spleenomegaly)
Activity Intolerance
Risk for infecting others
Imbalance in Nutrition

What is the incubation period for Mono?


10 days to 6 weeks

What is the transmission for Chicken Pox?


Droplet and Contact

What are the effected Systems when dealing with Mono?


Respiratory, GI, Lymph System

How long is Mono Communicable?


Prodromal phase through the acute phase.

Is Mumps Viral or Bacterial?


Viral

What systems are affected by Mumps?


Respiratory, CNS, Exocrine Glands

How is Mumps translated?


Droplet and Contact

What is the incubation period for Mumps?


12-21 days

What is the communication period for Mumps?


Immediately before to 1-3 days after after swelling begins.

What are signs and symptoms for Mumps?


Fever, Fatigue, HA, Earache, Painful swelling of Parotid and
Submandibular Swelling, Swelling Testicles

What are some Diagnostic Studies that can be run for


Mumps?
Look at the Pt. history and symptoms, Radiography studies
( CT, US), Sputum and Nasopharyngeal cultures

What are some Therapeutic Management Techniques that


can be implemented for someone dealing with Mumps?
Antipyretics, Analgesics Adequate Hydration, Warm or Cool
Compresses, Monitor for Complication

What is an immunization that cane be given to prevent


Mumps?
MMR at 12-15 months with a second dose at 4-6 years.

What are some complications that can occur from


Mumps?
Deafness, Encephalitis and Meningitis, Myocarditis,
Hepatitis Pancreatitis, Epididymoorchitis & Oopheritis, or
Sterility

What Nursing care can you implement with someone


suffering from Mumps?
Place the Client on Contact or Droplet Precautions,
Frequent PO fluids, Avoid chewing, Scrotal Support (ICE),
use of Analgesia

What are some Nursing Diagnosis for Mumps?


Impaired Comfort, Isolation
Risk for Injury (r/t complications)
Activity Intolerance
Risk for Infection to others

Is Rubella a Virus or Bacteria?


Virus

What Systems are affected due to Rubella?


Respiratory and Integumentary

How is Rubella transmitted?


Droplet and Contact

What is the incubation period for Rubella?


14-21 days

What is the communicable period for Rubella?


7 days before until 5 days after the rash appears (total of
15 days)

What are some signs and symptoms of Rubella?


At first may be asymptomatic; have a low grade fever,
cough upper respiratory tract infection, lymphadenopathy,
Maculopapular Rash

What are some Diagnostic tests that can be run for


Rubella?
A blood test for IGM or IGA antibodies

What are therapeutic management techniques that can


be used while treating someone with Rubella?
Antipyretics
Antipuritics
MUST stay away from pregnant women

What are immunizations that could be given to prevent


Rubella?
MMR given 12-15 months of age and again at 4-6 years of
age

What are some complications from Rubella?


Very dangerous to the unborn fetus, transplacental
transmission

What are some interventions you can implement with


someone who is dealing with Rubella?
Put droplet and contact precautions into place
Avoid contact with pregnant women
Apply cool compress to the rash
Administer antipyretics and antipuritics for comfort as
ordered

What are some Nursing Diagnoses that can be


implemented to some one dealing with Rubella?
Impaired comfort
Ineffective Health Maintenance
Risk for injury
Risk for Infection

Is Pertussis a Bacteria or a Virus?


Bacteria

What systems are effect with Pertussis?


Respiratory, GI, CNS

How is Pertussis transmitted?


Droplet and Contact

What is the Incubation period of Pertussis?


6-20 days

What is the communicable period of Pertussis?


5-8 weeks

What are some signs and symptoms of Pertussis?


1-2 Weeks: Fever, Fatigue URI
4-6 Weeks: Harsh Productive cough with copious amounts
of fluid followed by high pitched inspiration (Jeff Gordon
Commercial)

What are diagnostic studies that can be run for Pertussis?


Patient History, Characteristics of Cough, Sputum and Naso
Pharyngeal Cultures

What are therapeutic management techniques that can


be used while treating someone with Pertussis?
Antibiotics, Bronchodilators, Corticosteroids, Humidified
Oxygen, Respiratory Support

What are immunizations that could be given to prevent


Pertussis?
Dtap:2,4,6,18 months and again at 4-6 years
All adults should receive a Tdap every 10 years

What are some complications of Pertussis?


Pneumonia, Atelectasis, Seizures, Hemorrhage, Hernias

What are some Nursing interventions you can implement


for someone suffering from Pertussis?
Droplet and Contact Precautions
Frequent intake of fluids
Humidified 02
Irritant free Environment

What are some Nursing Diagnosis's for Pertussis?


Ineffective Breathing Pattern
Ineffective Airway Clearance
Impaired Comfort
Activity Intolerance

Is Rubeola a Virus or Bacteria?


Virus

What systems are effected due to Rubeola?


Respiratory, Integumentary, CNS, Eyes

How is Rubeola transmitted?

Droplet and Contact?

What is the Incubation period of Rubeola?


10-20 days

What is the communicable period of Rubeola?


4 days before till 5 days after the rash appears

What are some signs and symptoms of Rubeola?


3-5 Days: Fever Fatigue, Inflamed Nasal Passages, Cough,
Conjunctivitis, KOPLIK SPOTS
Acute Phase: Maculopapular Rash, Lymphadenopathy,
Photophobia

What are diagnostic studies that could be done for


Rubeola?
Positive measles IGM anti-bdy test of respiratory Mucosa

What are therapeutic management techniques that can


be used while treating someone with Rubeola?
Vitamin A supplement
Antipyretics for fever
Relaxing Environment
Analgesia
Antibiotics to prevent secondary infection

What are immunizations that could be given to prevent


Rubeola?
MMR at 12-15 Months and again at 4-6 years
Do not give before 12 months of age or the mothers
antibodies that are still active in the child will destroy the
immune factors in the Vaccination.

What are some complications of Rubeola?


Pneumonia, Encephalitis, Otitis Media

What are some Nursing interventions you can implement


for someone suffering from Rubeola?
Put on Droplet and Contact Precautions
Promote restful environment
Use a cool compress for rash and to decrease fever
Administer Antipyretics and Antipuritics for comfort

What are some Nursing Diagnosis's you can implement


for care of Rubeola?
Impaired comfort
Ineffective Thermoregulation
Ineffective Health Maintenance
Risk for infection
Risk for injury

Is Scarletina a Virus of Bacteria?


Bacteria; Group A beta-hemolytic Strep

What systems are affected by Scarletina?


Respiratory, Renal, Cardiovascular

How is Scarletina transmitted?


Contact and Droplet

What is the incubation period for Scarletina?


2-4 days

What is the Communicable period for Scarletina?


10 day during incubation and acute phase

What are some signs and symptoms of Scarletina?


Abrupt High Fever, HA, Chills, Fatigue, halitosis, Sever sore
throat, tonsillitis, WHITE STRAWBERRY TONGUE

What are some Diagnostic tests you can do for


Scarletina?
Throat culture, CBC with differential, Blood Cultures

What are therapeutic management techniques that can


be used while treating someone with Scarletina?
Full course of Antibiotics, Antipyretics, Analgesia

What immunizations can you have for Scarletina?


There are currently no immunizations to treat this at this
time.

What are some complications with Scarletina?


Rheumatic Fever, Glomerulonephritis, Peritonsilar abscess

What are some nursing intervention that can be put into


place for someone suffering from Scarletina?

Teach client importance of medication completion


Comfort Measures
Prevent Spread and Recurrence by teaching about mode of
transmission
Institute droplet precautions

What are some Nursing diagnoses that can be given for


Scarletina?
Ineffective thermoregulation
Risk for Injury
Risk for ineffective Health Maintenance
Risk for infection

Is Polio a Virus or Bacteria?


Virus that attacks the grey matter in the spinal cord

What systems are affected by Polio?


CNS, Respiratory, Musculoskeletal, GI

How is Polio transmitted?


Fecal-Oral, Droplet, Contact

What is the incubation period for Polio?


7-38 days

What is the communicable period of Polio?


up to 4 weeks

What are the Signs and Symptoms of Polio?


Inapparent:Flu Symptoms
Non paralytic: Flu symptoms with neck pain,back
and legs
Paralytic: Starts with fly symptoms,the signs of
paralysis

What are diagnostic tests that can be done for Polio?


Throat Culture, Stool Culture, CSF culture, Antibody screen
for Polio Virus

What are therapeutic management techniques that can


be used while treating someone with Polio?
Treat Urinary Retention, Prophylactic Antibiotics, Heat to
reduce muscle spasm, Analgesics, Respiratory Support, IV
hydration and Nutrition, Orthotic Support

What is an immunization that can be given to prevent


Polio?
IPV at 2,4,6 months and again at 4-6 years of age for a
total of 4 doses.
However if there 3rd does was administer at age 4 then
there is only a need for 3 doses.

What are some complications of Polio?


Permanent Muscle Paralysis, Kidney stones from
demineralization of bones, Post Polio Sequelae

What are some Nursing intervention that can be


implemented for those suffering from Polio?
Assess for Urinary Retention
Assess Respiratory Effort and Effectiveness of Breathing
Moist heat for Muscle spasms
Insure Adequate Nutrition (High Protein)
Assistance with ADL's
Frequent Repositioning (Pneumonia and risk for
PU)
Physical Therapy and Strengthening Exercises

What are some Nursing Diagnoses that for Polio?


Risk for Ineffective breathing
Risk for nutritional deficit
Risk for Impaired Skin Integrity
Anxiety
Activity intolerance
Impaired comfort

Is Tetanus a Virus or a Bacteria?


Bacterial which causes powerful doxin which attacks the
CNs

How is Tetanus transmitted?


Bacteria enters an open wound

What is the Incubation period of Tetanus?


3 days to Several weeks

What are some signs and symptoms of Tetanus?


Muscle spasms with stiffness of the muscles in the neck
and jaw (Lock Jaw)

What diagnostic studies are available to determine if


tetanus is present?
According to the handout none

What are therapeutic management techniques that can


be used while treating someone with Tetanus?
Flagyl, Tetanus Immunoglobulin, Antitoxin, Muscle
Relaxants, Muscle relaxants, IV Hydration and Nutrition

What are some immunizations that can be given to


prevent tetanus?
Dtap at age 2,4,6,18 months and again at 4-6 years of age
After age 7 Tdap is given every 10 years as a booster

What is the survival rate of Tetanus?


90% with appropriate treatment.

What are some complications associate with Tetanus?


Respiratory Arrest, Cardiac Arrest, Decreased CNS function,
Bone fractures

What are some Nursing Interventions that can be


implemented when caring for someone with Tetanus?
Airway Support, Comfort Measures, Monitoring Hydration
and Nutrition status,Administer Antibiotics, Antitoxins,
Muscle Relaxants, and Anti-anxiety Medication as
prescribed

What are some Nursing Diagnoses for Tetanus?


Impaired Physical Mobility
Impaired Comfort
Inadequate Nutrition
Risk for impaired gas exchange
Risk for Injury
Anxiety

What systems are affected by Tetanus?


Musculoskelatal, Respiratory

Is Diptheria a Virus or Bacteria?


Bacteria

What systems are affected by Diptheria?

Respiratory and Integumentary

How is Diptheria transmitted?


Contact

What is the incubation period of Diptheria?


2-5 days

What is the Communicable period Diptheria?


2-4 weeks

What are some signs and symptoms of Diptheria?


Prodromal: URI
Acute Stage: White or Grey Membranous coating,
Hoarseness, Skin irritation from Nasal discharge, Severe
Lymphadenopathy

What are some diagnostic studies that can be run to test


for Diptheria?
Throat Culture, Gram Stain, CBC with differential
Skin testing for allergy to horse serum

What are therapeutic management techniques that can


be used while treating someone with Diptheria?
Early Administration of Antibiotics, Diptheria Antitoxin, Bed
rest, Possible Tracheostomy

What immunizations can be given to prevent Diptheria?


Dtap 2.4.6 and 18 months
Repeat Dtap at 4-6 years of age.
After 7 years of age Tdap given every 10 years

What are some complications associated with Diptheria?


Septic Shock, Cardiomyopathy, Neuropathy

What are some Nursing Interventions that can be


implemented when caring for someone with Diptheria?
~Follow contact precautions until 2 negative cultures are
achieved
~Administer Antibiotics ASAP
~Administer Antitoxin after assess for horse Serum Allergy
~Strict bedrest to prevent cardiomyopathy
~Administer humindified oxygen for decreased 02 sats

~Assess respiratory effort


~Skin care for Nasal discharge and skin breakdown

What are some Nursing Diagnoses for Diptheria?


Ineffective airway clearance
Ineffective Thermo Regulation
Risk for Injury
Ineffective Health Maintenance
Risk for Infection
Communicable Disease
Illness that is directly or indirectly transmitted from one
person or animal to another by contact with body fluids,
with contaminated objects, or by vectors (e.g., ticks,
mosquitoes)

Risk Factors for Infectious and Communicable Diseases in


Newborns and Infants
Immature immune system
Passively acquired maternal antibodies provide limited
protection
Disease protection through immunization is incomplete

Risk Factors for Infectious and Communicable Diseases in


Children
Limited prior exposure to communicable diseases
Poor health
Immunodeficiency

Common Routes of Infection Transmission in Children


Behaviors that lead to the spread of infection
Traits of children- diapers, sharing toys, poor hand hygiene

Nurse's Role
in Infection Control
Promote infection control strategies
Report infectious and communicable diseases to state
health officials

Diseases of childhood and adolescence

Measles, mumps, rubella, polio, pertussis, diphtheria,


tetanus, Haemophilus influenza type B, hepatitis B,
hepatitis A, pneumococcus, varicella, rotovirus,
meningococcal diseases, human papillomavirus

Active immunity
An antigen is introduced into the body in a vaccine, which
stimulates antibody production without causing clinical
disease

Passive immunity
Antibodies produced in another human or animal host is
given.
Provides immediate protection
Does not provide lasting immunity
Example: Tetanus immune globulin

Immunization Type
Inactivated
DTaP, IPV, Hep B, Hib, PCV, Hep A, Influenza

Live attenuated (weakened)


MMR, Varicella, Influenza (nasal spray)
*Contraindicated in immunocompromised individuals

Vaccine Side Effects


Local reactions
Erythema, swelling, pain, and induration

Vaccine Side Effects


Systemic reactions
Fever, irritability, malaise, anorexia (e.g., DTaP)

Vaccine Side Effects


Allergic reactions
(local and systemic - 0.012%))
Wheal, urticaria
Severe local
Anaphylaxis

Vaccine Side Effects


Other serious reactions
not the vaccine, but products in the vaccine

Vaccine Side Effects


Unique reactions
MMR- rash and fever (normal)
Varicella- pox that break out for 30 days

Misinformation about vaccines


Vaccine-preventable diseases have been eliminated.
Immunization weakens the immune system. Multiple
vaccines overload the immune system and cause harmful
effects.doesn't weaken immune system- vaccines not as strong as
used to be
It would be better to let the child get the disease than get
immunized.
Vaccines do not work. Children still get the disease.
Belief that they can control their child's susceptibility to
disease and the outcomes

Steps in Administering Immunization


Identify needed immunizations
Review the vaccine screening questions
Review the Vaccine Information Statement (VIS)

Fever Benefits
Rid of organisms that thrive in lower body temperatures
Mobilizes immune response
plasma iron concentration - limits microorganism growth

Fever Risk
When body temp reaches 41 C (105.9)
Febrile seizure

Sepsis
Definition: Overwhelming infection in the bloodstream
Causes from infectious diseases:
Infected pox
Can lead to septic shock

septic shock

systematic response to pathogenic organisms in the blood


resulting in circulatory dysfunction

Signs of Sepsis
Hypothermia or hyperthermia
Abdominal distention
Anorexia
Respiratory distress
Vomiting
Cool extremities
Mottling
Lethargy

Varicella (Chicken Pox)


Transmission: Airborne and lesion contact
Direct contact with mucous membranes or conjunctiva
Incubation: 14 - 21 days
Communicable -5 days before rash and until it crusts
Clinical manifestations:
mild fever, malaise, headache, itching with rash
Potential complications:
Secondary infection, cellulitis, encephalitis
Prevention:
Vaccine after age 12m

Nursing Management of Chicken Pox


*Airborne and contact precautions
Hx of immunization and exposure
Change linen frequently
Observe for signs of complications - drowsiness, meningeal
sign
Monitor for acyclovir side effects - nausea, vomiting,
diarrhea, h/a, allergic reactions

Management of Lesions
Use cool wet compresses or giving baths in cool or
lukewarm water (Oatmeal bath, e.g., Aveeno)
Keep fingernails short and clean.
Put calamine lotion on itchy areas

Give child foods that are cold, soft, and bland.


Acetaminophen regularly to help relieve pain if there are
mouth blisters.
Medication for itching.

Hand, Foot and Mouth Disease (Coxsackievirus)


Transmission: fecal-oral and respiratory routes
Incubation : 3-6 days
Clinical manifestations:
Lesions cheeks, gums, and tongue.
Papulovesicular lesions on hand and feet
Irritable, fever, anorexia, dysphagia
Treatment: supportive care
Prevention: avoid contact with infected person early in the
disease

Hand, Foot and Mouth Disease (Coxsackievirus)


Nursing Management
Standard and contact precautions
Topical lotions and systemic meds for pain
Cool drinks; soft, bland food
Warm saline mouth rinses
Observe for dehydration
Reassurance and support to parents
Nonaspirin antipyretics

Erythema Infectiosum (Fifth's Disease)


Transmission: Respiratory and blood
Incubation: 6-14 days
Clinical manifestations:
Flu-like illness for 2-3 days
1 week later - fiery-red rash on cheeks (*slapped face),
lacelike symmetric, erythematous, maculopapular rash on
trunk and limbs
Treatment: Supportive
Prevention: avoid contact with infected persons

Meningococcus
Transmission: respiratory droplets
Highest rates < 5 years of age.
Incubation: 1-10 days
Communicable until 24 hours after start of antibiotics
Clinical manifestations -onset is rapid and life-threatening
Onset - flu-like symptoms with prostration

Neuro signs
Meningococcemia - urticarial, maculopapular
Shock, hypotension, DIC, coma

Meningococcus (cont)
Complications
Loss of limbs d/t necrosis; hearing loss, arthritis, seizures,
pericarditis, hemiparesis
Up to 15% with invasive type die
Prevention
Vaccine for children > 2 years for persons at high-risk
Close contact - prophylaxis treatment
Treatment
Penicillin G (cefotaxime, ceftriaxone - alternatives)
In ICU
DIC treated with plasma, blood, or platelets

Meningococcus
Nursing management
Standard and droplet precautions
Monitor for shock and respiratory compromise
Monitor for fluid overload and ICP -watch fluids
Help identify close contacts that need prophylaxis
Help family identify and connect with supports
Surviving child will likely need rehab

Roseola
Transmission: likely to be respiratory
Incubation : 5-15 days
Clinical manifestations:
Sudden, high fever for 3-8 days; normal behavior and
appetite
Fever followed by pale, pink, discrete, maculopapular rash
Treatment: Supportive
Herpes Zoster (shingles)

-most painful of all


caused by the chicken pox virus
highly communicable
pallative treatment only
travels via nerve route

Hepatitus B
below the belt,
comes from infected or contaminated blood
vaginal fluids, saliva, urine, semen etc

Hepatitus C
affects the liver
a symptomatic
and causes cirrohsis or hardening of the liver

auto infection
the infection of a primary host with a parasite. Particularly
a helminth

fecal oral route diseases


Poliomyelitus . ecoli, and hepatitus A

Legionairres disease
produces high fever and pnumonia
out breaks occur in warm moist environments during
summer and winter
uncommon to those 20 years or younger
pennicillin ineffective

Hepatitus A
Treansmited through fecal oral route
ingestation of contaminated food
shellfish from contaminated waterrs
no treatment

Staff
natural inhabitant found in the human body
responsible for boils and other infections
inflammation of the stomach and intestines
source of food intoxication

Poliomyelitus
feared more than aids
fecal oral route

jonas slak created the vaccine in 1952


paralytic virus
spread through the nerve pathways
no cure , pallative treatment

Botulism
Paralytic type illness
recovery takes months or years
paralysis of the breathing muscles
casues respirtory failure

Mumps
contracting it equals life long immunity
inflammed glands

small pox
occurs only in humans
only infectious for a short time
1st communicable disease officially eradicated
not strong enough to survive.

e-Coli
can cause distruction of the kidneys
if excreted con become communicable
found in the lower intestines of warm blooded animals.
fecal oral transmission

Rubella
normally seen in children
communicable for one week
immunity accquired through vaccime or infection

salmonella
2000 different species
communicable for the duration of the infection
ingested of food derived from infected animal
transferred between human and non human
diarhea, feve and abdominal cramps

measles
disease is communicable
spread through respiratory contact
vaccination during second year of life

Histoplasmosis

affects the lungs


associated with dry cough, fever and chest pains
known as the cave disease
found in bird caves containing bat, or bird poop droppings
and chicken coops.
can survive in soil for years

Plague
spread to humans via fleas
depending on sanitation, van be spread through the air.
can come from contaminated or undercooked foods.

anthrax
used as a biological weapon
spread by spores

anthropod Vector
any tiny organism that can transport infectious pathogens
via bacteria or prion.
mosquitos
flies
mites
ticks

encephalitis
caused by bacterial infections by
rabies
west nile
measles
polio
and herpes simplex

toxic shock
caused by staff and strep
causing problems with several body ogans
can be treated with antibodies

conjunctivitus
eyes become red and inflammed
treated with drops
casued by bacteria rubbed into the eye
caused due to poor handwashing
affects the mucous membrane that lines the eyelid

rhino viruses

able to survive for a long period of time on fomites

commom cold
most common of all communicable diseases
created through rhnoviruses
bacterial exudate is clear
can be transmitted directly or indirectly

Reyes syndrome
causing detrimental harm to many organs such as the brain
and liver
associated with giving children asprin
damages cellular mitochondria

inluenza
communicable 3 to 5 days after inicial symptoms appear
Vaccine is 80 percent effective

Mono
kissing disease
part of the herpes group
smptoms are non specific
carrier has EBV on the throat
affected individuals restricted form contact sport.

meningitis
caused by a bacteria
can be treatd with antibiotics
starts with an upper respiratory infection

herpes Virus consists of


shingles
chicken pox

Herpes simplex 1
herpes type 1 causes sores around the mouth and lips
(sometimes called fever blisters or cold sores).

Alkalosis
The main cause of respiratory alkalosis is hyperventilation,
resulting in a loss of carbon dioxide having to use a paper
bag

conditions caused by strep and staff

empatigo,bacterimia, cellulitis, absesses

plantar warts
The HPV type 1 causes most plantar warts.
The virus gains access to the skin through direct contact.

endocarditus
inflammation on the inside lining o the heart

skin conditions are caused by


staph or strep

hansens disease
effects the skin and perrifrial nerves; leoprosy

necrotising facilitus
bacteria capable of deteriorating the skin and the tissues
covering the muscles due to bacteria entering into the skin.
What is an emergency?
An emergency is a serious event that
requires an immediate response. The
affected population is usually small in
scale. Examples include a house or
neighborhood fire, a car accident, or an
avalanche that traps several people.

What is a disaster?
A disaster affects many people in various
ways. Some examples of disasters include
an earthquake or a hurricane destroying a
city or several cities across a wide area, a
tornado that causes damage and death
over many miles and several cities, or a
man-made disaster such as a nuclear
bomb.

What are the elements of a disaster?


temporal
spatial
role
effects

What is the non-disaster stage of the


temporal element?

This stage includes the period of time


before or between disasters. Health
providers must conduct assessments and
identify disaster risks.

What is the pre-disaster stage of the


temporal element?
This stage occurs when disaster is
imminent but has not yet occurred. When a
warning or threat of a disaster is issued,
health officials must apprise the public
about imminent threats such as onset
hurricanes or tsunamis.

What is the impact stage of the temporal


element?
When disaster occurs, nurses must assess
the impact of the event on the community.
Nurses need to collect inventory on the
community's immediate needs, the amount
of damage to buildings and the
environment, and the type and extent of
bodily injuries.

What is the emergency stage of the


temporal element?
After a disaster, nurses and aid providers
must administer first aid and treatment to
victims. These individuals must also
perform search and rescue efforts to save
and protect as many lives as possible.
During the emergency stage, nurses must
conduct a triage of responsibilities.

What is the recovery stage of the


temporal element?
This stage focuses on returning the
community to equilibrium. Nurses work to
restore the community to a functional state
by removing debris and reestablishing
community services. The goal of the
recovery stage is to restore major
resources of the community within the first
six months after the disaster and to return
community members to their homes or
alternative shelters. During the recovery

stage, providers will clean up the area and


help businesses to re-open and schools to
resume session.

What is the evaluation stage of the


temporal element?
After the disaster, nurses conduct an
assessment of the response efforts. These
professionals will plan for preventing future
disasters and seek new ways to minimize
the impact of the event based on the
knowledge gained. During this stage, the
professionals will cycle back to the
nondisaster stage of planning and employ
the new information gained.

What is the total impact area of the


spatial element?
Total impact includes the zone where the
most severe effects of the disaster
occurredthe area of greatest damage
and destruction.

What is the partial (fringed) impact area


of the spatial element?
Partial, or fringed, impact are areas that
exhibit some visible damage but that did
not take the brunt of the damage from the
disaster.

What is the outside area of the spatial


element?
The outside area is an area that is not
affected by the disaster, but instead may
serve to aid the affected areas as a
resource for personnel, shelter, food,
intelligence, and assistance as needed.

What is a primary victim?


A person who has experienced the worst
exposure to the disaster

What is a second-level victim?


Someone affected indirectly, such as a
friend or relative of primary victims

What is a third-level victim?


A person who comes to the scene of a
disaster as a helper but is traumatized by
what he or she witnesses there

What is a fourth-level victim?


A community member who offers to help
primary victims and becomes affected by
the stories he or she hears and the injuries
he or she sees

What is a fifth-level victim?


A person not directly affected but who
suffers psychologically from the disaster

What is a sixth-level victim?


A person vicariously affected, but who has
no direct connection to the disaster

What are the primary effects of a


disaster?
The immediate effects of the disaster, such
as injury, death, and destruction of
property

What are the secondary effects of a


disaster?
Indirect effects, such as malnutrition, the
spread of disease, and homelessness

What are the physical effects of a


disaster?
Injuries, disabilities, death, and illness

What are the mental health effects of a


disaster?
Post-traumatic stress disorder, depression,
and anxiety

What are the economic effects of a


disaster?
Poverty, loss of property, and the cost of
rebuilding

What are the structural effects of a


disaster?
Loss of health care and other facilities for
support and resources

What are the social effects of a disaster?


Families displaced, loss of important
members of the community, and loss of
leadership

What is the APHA?


The American Public Health Association
(APHA) was initially formed by public
health professionals in 1872. With over 50
public health disciplines participating, the
APHA is an organized agency which
advocates for the health of entire
populations and is the largest association
of public health professionals in the world.
The APHA focuses on the development of
standards and policies to promote
population health.

What are some examples of natural


disasters?
Hurricanes
Tornadoes
Hailstorms
Cyclones
Blizzards
Droughts
Floods
Mudslides
Avalanches
Earthquakes
Volcanic eruptions
Communicable disease epidemics
Lightning-induced forest fires
Tsunamis
Thunderstorms and lightning
Extreme heat and cold

What are some examples of human-made


disasters?

Conventional warfare
Nonconventional warfare (e.g., nuclear,
chemical)
Transportation accidents
Structural collapse
Explosions and bombings
Fires
Hazardous materials incident
Pollution
Civil unrest (e.g., riots)
Terrorism (e.g., chemical, biological,
radiological, nuclear, explosives)
Cyber attacks
Airplane crashes
Radiological incidents
Nuclear power plant incidents
Critical infrastructure failures
Water supply contamination

What are some examples of Healthy


People 2020 objectives related to disaster
mitigation?
EH-21: Improve the utility, awareness,
and use of existing information systems for
environmental health.
FS-1: Reduce infections caused by key
pathogens transmitted commonly through
food.
HC/HIT-12: Increase the proportion of
crisis and emergency risk messages,
intended to protect the public's health, that
demonstrate the use of best practices.
IID-12: Increase the percentage of
children and adults who are vaccinated
annually against seasonal influenza.
IID-13: Increase the percentage of adults
who are vaccinated against pneumococcal
disease.

What is the disaster management cycle?

What are some common adult reactions


to disasters?
Extreme sense of urgency
Panic and fear
Disbelief
Disorientation and numbness
Reluctance to abandon property
Difficulty in making decisions
Need to help others
Anger and blaming
Blaming and scapegoating
Delayed reactions
Insomnia
Headaches
Apathy and depression
Sense of powerlessness
Guilt
Moodiness and irritability
Jealousy and resentment
Domestic violence

What are some common children


reactions to disasters?
Regressive behaviors (e.g., bedwetting,
thumb sucking, crying, clinging to parents)
Fantasies that disaster never occurred
Nightmares
School-related problems, including an
inability to concentrate and refusal to go
back to school

What populations are at greatest risk for


disruption after disaster?
Seniors
Vision or hearing impaired
Women
Children
Individuals with chronic disease
Individuals with chronic mental illness
Non-English-speaking
Low income
Homeless
Tourists, persons new to an area
Persons with disabilities
Single-parent families
Substance abusers
Undocumented residents

what is disease surveillance?


the ongoing systematic collection, analysis,
interpretation and dissemination of specific
health data for use in public health

What is biological terrorism?


the deliberate release of viruses, bacteria,
or other germs (agents) used to cause
illness or death in people, animals, or
plants

What is chemical terrorism?


the intentional release of hazardous
chemicals into the environment for the
purpose of harming or killing

What can public health surveillance be


used to facilitate?
Estimate the magnitude of a problem
(disease or event)
Determine the geographic distribution of
an illness or symptoms
Portray the natural history of a disease
Detect epidemics and define a problem
Generate hypotheses and stimulate
research
Evaluate control measures
Monitor changes in infectious agents

Detect changes in health practices


Facilitate planning

What are the features of surveillance?


Is organized and planned
Is the principal means by which a
population's health status is assessed
Involves ongoing collection of specific
data
Involves analyzing data on a regular
basis
Requires sharing the results with others
Requires broad and repeated contact
with the public about personal health
issues
Motivates public health action as a result
of data analyses to:
Reduce morbidity
Reduce mortality
Improve health

What are the purposes of surveillance?


Assess public health status
Define public health priorities
Plan public health programs
Evaluate interventions and programs
Stimulate research

What are some surveillance objective for


healthy people 2020?
EH-5 Reduce waterborne disease
outbreaks arising from water intended for
drinking among persons served by
community water systems.
FS-2 Reduce infections associated with
foodborne outbreaks from pathogens
commonly transmitted through food.
GH-1 Reduce the number of cases of
malaria reported in the United States.
IID-16 (Developmental) Increase the
scientific knowledge on vaccine safety and
adverse events.
PHI-2 Increase the proportion of tribal,
state, and local public health agencies that
incorporate core competencies for public
health professionals into the job.
PHI-7 Increase the proportion of

population-based Healthy People 2020


objectives for which national data are
available for all population groups
identified for the objective.

What is the sentinel surveillance system


provides for the monitoring of key health
events when information is not otherwise
available or for calculating or estimating
disease morbidity in vulnerable
populations

What types of surveillance systems are


there?
passive
active
sentinel
special

What is the passive system?


In the passive system, case reports are
sent to local health departments by health
care providers (i.e., physicians, nurses) or
laboratory reports of disease occurrence
are sent to the local health department.
The case reports are summarized and
forwarded to the state health department,
national government, or organizations
responsible for monitoring the problem,
such as the CDC or an international
organization such as the World Health
Organization.

What is an active surveillance system/


In the active system, the nurse, as an
employee of the health department, may
begin a search for cases through contacts
with local health care providers and
agencies. In this system, the nurse names
the disease or the event and gathers data
about existing cases to try to determine
the magnitude of the problem (how
widespread it is).

What is a sentinel surveillance system?

In the sentinel system, trends in commonly


occurring diseases or key health indicators
are monitored (Healthy People 2020). A
disease or an event may be the sentinel, or
a population may be the sentinel. In this
system a sample of health providers or
agencies is asked to report the problem.
The system is useful because it helps
monitor trends in commonly occurring
diseases and events.

What is the NNDSS?


National Notifiable Disease Surveillance
System (NNDSS).

What does infectivity refer to?


the capacity of an agent to enter a
susceptible host and produce infection or
disease

What does pathogenicity refer to?


Measures the proportion of infected people
who develop the disease

What does virulence refer to?


Refers to the proportion of people with
clinical disease who become severely ill or
die

What are biological agent factors?


Bacteria (e.g., tuberculosis, salmonellosis,
streptococcal infections)
Viruses (e.g., hepatitis A, herpes)
Fungi (e.g., tinea capitis, blastomycosis)
Parasites (e.g., protozoa-causing malaria,
giardiasis; helminths [roundworms,
pinworms]; arthropods [mosquitoes, ticks,
flies, mites])

What are types of physical agent factors?


Heat
Trauma

What are types of chemical agent factors?

Pollutants
Medications/drugs

What are types of nutrient agent factors?


Absence
Excess

What are types of psychological agent


factors?
Stress
Isolation
Social support

What is a common source outbreak?


refers to a group exposed to a common
noxious influence such as the release of
noxious gases (e.g., ricin in the Japanese
subway system several years ago and
more recently in a water system in the
United States)

What is a point source outbreak?


all persons exposed become ill at the same
time, during one incubation period.

What is a mixed outbreak?


a common source followed by secondary
exposures related to person-to-person
contact, as in the spreading of influenza

What are intermittent or continuous


source outbreaks?
cases may be exposed over a period of
days or weeks, as in the recent food
poisonings at a restaurant chain
throughout the United States as a result of
the restaurant's purchase of contaminated
green onions.

What is a propagated outbreak?


does not have a common source and
spreads gradually from person to person
over more than one incubation period,
such as the spread of tuberculosis from
one person to another.

What factors must be considered as


causes of an outbreak?
agents, hosts, and environmental factors

What are epidemiological clues that may


signal a covert bioterrorism attack?
Large numbers of ill persons with a
similar disease or syndrome
Large numbers of unexplained disease,
syndrome, or deaths
Unusual illness in a population
Higher morbidity and mortality than
expected with a common disease or
syndrome
Failure of a common disease to respond
to usual therapy
Single case of the disease caused by an
uncommon agent
Multiple unusual or unexplained disease
entities coexisting in the same person
without any other explanation
Disease with an unusual geographic or
seasonal distribution
Multiple atypical presentations of disease
agents
Similar genetic type among agents
isolated from temporally or spatially
distinct sources
Unusual, atypical, genetically
engineered, or antiquated strain of agent
Endemic disease with an unexplained
increase in incidence
Simultaneous clusters of similar illness in
noncontiguous areas, domestic or foreign
Atypical aerosol, food, or water
transmission
Ill people presenting at about the same
time
Deaths or illness among animals that
precedes or accompanies illness or death
in humans
No illness in people not exposed to
common ventilation systems, but illness
among those people in proximity to the
systems

How do you conduct an outbreak


investigation?
Confirm the existence of an outbreak.
Verify the diagnosis and/or define a case.
Estimate the number of cases.
Orient the data collected to person,
place, and time.
Develop and evaluate a hypothesis.
Institute control measures and
communicate findings.

What are some Healthy People 2020 goals


related to immunization and infectious
diseases?
IID-1: Reduce, eliminate, or maintain
elimination of cases of vaccine-preventable
diseases.
IID-4: Reduce invasive pneumococcal
infections.
IID-24: Reduce chronic hepatitis B virus
infections in infants and young children
(perinatal infections).
IID-27: Increase the percentage of
persons aware they have a chronic
hepatitis C infection.

What is toxicity?
The ability to produce a poisonous reaction

what is invasiveness?
the ability to penetrate and spread
throughout a tissue?

What is antigenicity?
the ability to stimulate an immunological
response

What characteristics of a host the may


influence the spread of disease?
host resistance
immunity
herd immunity
infectiousness of the host

What is resistance?

The ability of the host to withstand


infection

What is natural immunity?


refers to species-determined, innate
resistance to an infectious agent. For
example, opossums rarely contract rabies.

What is acquired immunity?


the resistance acquired by a host as a
result of previous natural exposure to an
infectious agent. Having measles once
protects against future infection

What is active immunization?


the resistance acquired by a host as a
result of previous natural exposure to an
infectious agent. Having measles once
protects against future infection

What is passive immunization?


immunization through the transfer of a
specific antibody from an immunized
individual to a nonimmunized individual,
such as the transfer of antibody from
mother to infant or by administration of an
antibody-containing preparation (i.e.,
immunoglobulin or antiserum)

What is herd immunity?


refers to the immunity of a group or
community. It is the resistance of a group
of people to invasion and spread of an
infectious agent. Herd immunity is based
on the resistance of a high proportion of
individual members of a group to infection.

What is infectiousness?
a measure of the potential ability of an
infected host to transmit the infection to
other hosts.

What is vertical transmission?

when the infection is passed from parent to


offspring via sperm, placenta, milk, or
contact in the vaginal canal at birth

What is horizontal transmission?


the person-to-person spread of infection
through one or more of the following four
routes: direct or indirect contact, common
vehicle, airborne, or vector-borne.

What is incubation period?


the time interval between invasion by an
infectious agent and the first appearance
of signs and symptoms of the disease. The
incubation periods of infectious diseases
vary from between 2 and 4 hours for
staphylococcal food poisoning to between
10 and 15 years for AIDS.

What is communicable period?


the interval during which an infectious
agent may be transferred directly or
indirectly from an infected person to
another person. The period of
communicability for influenza is 3 to 5 days
after the clinical onset of symptoms.
Hepatitis B-infected persons are infectious
many weeks before the onset of the first
symptoms and remain infective during the
acute phase and chronic carrier state,
which may persist for life.

What does endemic mean?


refers to the constant presence of a
disease within a geographic area or a
population. Pertussis is endemic in the
United States.

What is an epidemic?
refers to the occurrence of a disease in a
community or region in excess of normal
expectancy.

What is a pandemic?
an epidemic that occurs worldwide and
affects large populations. HIV/AIDS is both

epidemic and pandemic, because the


number of cases is growing rapidly across
various regions of the world, as well as in
the United States. SARS and novel
influenza A H1N1 are both emerging
infectious diseases and are responsible for
recent pandemics.

What are emerging infectious diseases?


those in which the incidence has actually
increased in the past two decades or has
the potential to increase in the near future.
These emerging diseases may include new
or known infectious diseases.

How does secondary prevention apply to


communicable diseases?
The goal of secondary prevention is to
prevent the spread of disease once it
occurs. Activities center on rapid
identification of potential contacts to a
reported case.

How does tertiary prevention apply to


communicable diseases?
To reduce complications and disabilities
through treatment and rehabilitation

What is a zoonosis?
an infection transmitted from a vertebrate
animal to a human under natural
conditions. The agents that cause
zoonoses do not need humans to maintain
their life cycles; infected humans have
simply somehow managed to get in their
way

What are the Healthy People 2020 goals


related to communicable diseases?
HIV-3: Reduce the rate of HIV
transmission among adults and
adolescents.
HIV-5: Reduce the number of new AIDS
cases among adolescents and adult
heterosexuals.
STD-8: Reduce congenital syphilis.

STD-6: Reduce gonorrhea rates.


IID-25: Reduce hepatitis B.
IID-26: Reduce new hepatitis C infections.
STD-1: Reduce the proportion of
adolescents and young adults with
Chlamydia trachomatis infections.

What are the stages of HIV infection?


1.The primary infection (within about 1
month of contracting the virus)
2.Clinical latency, a period with no obvious
symptoms
3.A final stage of symptomatic disease
communicable disease
Any disease that spreads from one host to another, either
directly or indirectly.

infection
A condition that occurs when pathogens enter the body,
multiply, and damage body cells

virus
(virology) ultramicroscopic infectious agent that replicates
itself only within cells of living hosts

toxins
Any of various poisonous substances produced by some
microorganisms

vectors
An organism that transmits disease by conveying
pathogens from one host to another

airborne transmission
Spread of disease-causing microorganisms over long
distances through the air.

respiratory tract
Nasal Cavity, Pharynx, Glottis, Larynx, Trachea, Bronchus,
Bronchioles, Lung, Diaphragm.

mucous membrane
Membrane that lines passages that communicate with air

influenza
Acute, contagious respiratory infection characterized by
sudden onset, fever, chills, headache.

pneumonia
An inflammation of lung tissue, wherer the alveoli in the
affected areas fill w/fluid

strep throat
Bacterial infection of the throat; if left untreated may lead
to high fever and other serious complications

tuberculosis
An infectious disease that may affect almost all tissues of
the body, especially the lungs

jaundice
yellowing of the skin and the whites of the eyes caused by
an accumulation of bile pigment (bilirubin) in the blood

hepatitis A
A virus that is spread by the fecal-oral route through
contaminated food and water or by close and intimate
contact and results in liver inflammation, flu-like
symptoms, nausea, poor appetite, abdominal pain, fatigue,
yellow eyes and skin, and dark urine that can last weeks to
months.
A microorganism that causes disease
pathogen

A condition that occurs when pathogens in the body


multiply and damage body cells
Infection

White blood cell that surrounds and destroys pathogens


phagocyte

White blood cell that coordinates and performs many of


the functions of specific immunity
lymphocyte

A global outbreak of an infectious disease

pandemic

Which disease is caused by bacteria?


A. athlete's foot
B. common cold
C. influenza
D. strep throat
D

Which disease can result in cirrhosis of the liver?


A. tuberculosis
B. Hepatitis C
C. Mononucleosis
D. Pneumonia
B

Which is not a chemical barrier against pathogens?


A. skin
B. gastric juice
C. tears
D. saliva
A

IN the immune response, what kind of cells trigger the


production of B cells and killer T cells?
A. antigens
B. macrophages
C. Helper T cells
D. phagocytes
C

Which is not an emerging disease?


A. avian influenza
B. Hepatitis A
C. Lyme disease
D. mad cow disease
D

T or F
A disease that is spread from one living organism to
another is called a(n) COMMUNICABLE DISEASE.
True

T or F
Common PATHOGENS include flies, mosquitoes, and ticks.

False vectors

T or F
The common cold and strep throat are both diseases of
the digestive tract.
False respiratory tract

T or F
PNEUMONIA can be caused by either a virus or a bacteria.
True

T or F
The RESPIRATORY SYSTEM is a network of cells, tissues,
organs, and chemicals that fight of pathogens.
False immune system

T or F
A(n) VACCINE is a protein that acts against a specific
antigen.
False

T or F
The body develops ACTIVE IMMUNITY when it is exposed
to antigens from invading pathogens.
True

T or F
LIVE-VIRUS VACCINES use dead pathogens.
False dead-virus vaccines

T or F
T or F
EMERGING INFECTIONS are communicable diseases
whose occurrence in humans has increased within the
past two decades or threatens to increase in the near
future.
True

T or F
Salmonella and E. Coli are VIRUSES that live in animal's
intestinal tracts.
False bacterias
carcinogen
chemicals known to cause cancer

malignant
cancerous

benign
noncancerous. does not spread

Cancer
abnormal, uncontrolled cell growth.
Takes over and destroys normal cells and tissue

Tumor
abnormal growth of tissue

Metastasis
the spread of cancer

Carcinomas
cancers that develop in the skin and linings of organs

Sarcomas
cancers that develop in bones, muscles and tendons

Lymphomas
cancers that develop in the lymphatic system (immune
system)

Diabetes
a chronic disease that affects the way body cells convert
food into energy

Glucose
sugar

Insulin
hormone that helps turn glucose into energy for the cells

Type 1 diabetes
Occurs most often before age 15
The pancreas produces NO insulin
Frequent urination, abnormal thirst and hunger are
symptoms
Must have daily injections

Type 2 diabetes
Usually in adults over 40
The body's cells do no use insulin correctly
Excess weight, drowsiness and blurred vision are symptoms
Must maintain diet and exercise
Usually caused from being overweight

Atherosclerosis
When fat deposits (plaque) build up in the blood vessels it
is called

Stroke
Blood vessels in the brain are blocked or broken

Biopsy
a procedure that removes and examines a group of cells
that may be cancerous.

Virus
The smallest pathogen that multiplies after entering a cell

Bacteria
Single-celled microorganism

Protozoa
Larger, more complex single-celled organism

Fungi
A plant-like organism that causes diseases of the skin

Heart Attack
The death of the cardiac muscle caused by a lack of blood
flow to the heart

Arteriosclerosis
A term used to describe several conditions that cause
hardening or thickening of the arteries

Angina Pectoris
Chest pain from narrowed coronary arteries

Vector

Commonly responsible for spreading disease (mosquitos,


flies, ticks)
Community-Oriented Nursing
Focus: Health of the community as a whole.
Client: Community
Activity: Surveillance and evaluation of the community's
collective health, and the implementation of strategies to
promote health and prevent disease.

Community-Based Nursing
Focus: Health of individuals, families, and groups within a
community.
Client: Individuals, family, or group of individuals
Activity: Provision of direct primary care in the settings
where individuals and families live, work, and "attend"
(schools, camps, parishes)

Community Health Nursing


Systematic processes to deliver care to individuals,
families, or community groups with a focus on promoting,
preserving, protecting, and maintaining health care
provided contributes to the health of the community.

Public Health Nursing


Nursing and public health, disease and disability
prevention, promoting and protecting the health of the
entire community

Eight domains of public health


Distinguishes public health nursing from other nursing
specialties by adherence to the eight principles:
Analytical assessment skills
Policy development and program planning skills
Communication skills
Cultural competency skills
Community dimensions of practice skills
Basic public health science skills
Financial planning and management skills
Leadership and systems thinking skills

Three primary functions of public health


1. Assess health care function
2. Develop policy that provides access to services
3. Ensure services are delivered and outcomes achieved

Health People 2020: Four main goals


1. Eliminate preventable diseases, disability, injury, and
premature death
2. Achieve health equity, eliminate disparities, and improve
health of all age groups
3. Create social and physical environments that promote
good health for all
4. Promote healthy development and health behaviors
across every stage of life

Social determinants
Impact whether someone is able to attain/maintain health;
(Income, social status, education, literacy, environment,
support networks, gender, culture, available health care)

Disparities
Gaps in care experienced by one population compared to
another

Health care changes in the 21st century


Increased patient acuity and number of services in the
community.
Patient centered care
Increased patient responsibility for own health
Increased use of technology

Florence Nightingale
Studied nursing in Germany then established nursing
schools in England

District Nursing
A mode of service delivery in which a community health
nurse is responsible for addressing all the health needs of a
given population

Lemuel Shattuck
First use of demographic data to look at population health
birth and death records

Dorthea Dix
Established first hospital for the mentally ill in the US

Clara Barton
Civil war nurse: Established the American Red Cross
(providing aid for natural disasters)

Lillian Wald
Found of public health nursing in the US; Founded the
Henry Street Settlement and Visiting Nurse Service which
provided nursing and social services and organized
educational and cultural activities. She is considered the
founder of public health nursing

Mary Breckenridge
Pioneer in nurse-midwifery; established the Frontier Nursing
Service - nurses traveled on horseback to reach mothers in
the hills of Kentucky

World Health Organization


International center that collects data, advances initiatives,
and offers support related to public health

US Department of Health and Human Services (USDHHS)


US branch of government responsible for health and
welfare of citizens

Nongovernment Agencies
Agencies that acquire resources from private sources to
assist others

Philanthropic Agencies
Organization that uses endowments or private funding to
address the needs of individuals, families, and populations

Center for Disease Control


(CDC) is a United States federal agency under the
Department of Health and Human Services. It works to
protect public health and safety by providing information to
enhance health decisions, and it promotes health through
partnerships with state health departments and other
organizations.

Difference in US health care system and public health


The US has an individual focus on curative measures while
public health focuses on the population or community as a
whole and focuses on disease reduction and improved
health

Local Level of Government

Implement public health activities within the community;


Governed by the state department;
Structure varies based on community needs

State Level of Government


Focuses on regulation of the community of program
responsibility and resource allocation

Federal Level of Government


Focuses on policy development and financing

Health
A quality, an ability to adapt to change or a resource to
help cope with challenges and processes of daily living

Well Being
A subjective perception of full functional ability as a human
being

USDHAHS 10 Components of public health practice


1. Preventing Epidemics
2. Protecting the environment, workplaces, and sources of
food and water
3. Promoting health behavior
4. Monitoring the health status of the population
5. Mobilizing community resources into service
6. Responding to disasters
7. Assuring the public there are trained personnel
8. Reaching out to those at high risk
9. Researching risk, disease acquisition, and ways to
prevent injury
10. Influencing policy to acquire resources to effect change

Epidemiological Health Promotion Model


The study of relationships among an agent, a host, and an
environment. The interaction determines the development,
and cessation of communicable disease.

Learning Theory
Goal established and reinforced by nurse; Rewards given
for partial accomplishment

Health Belief Model


Cues used to remind people of health behavior and
promote action; Perceived risk influences action

Transtheoretical Health Promotion Model


5-Step approach: Pre-contemplation, Contemplation,
Preparation, Action, Maintenance

Reasoned Action Health Promotion Model


Performance of a behavior is determined by a persons
intention to perform the behavior; Intentioned determined
by attitude and behavior

Social Support Health Promotion Model


Change influenced by support of friends, families, and
communities

Primary Level of Prevention


Utilize prevention strategies, address modifiable risk
factors, and maximize health and wellness

Secondary Level of Prevention


Planned effort to minimize the impact of disease and injury
once it has occurred

Tertiary Level of Prevention


Maximizing health and wellness through strategies set in
place at the end-stage of disease and injury

Epidemiology
The study of the distribution and determinants of health
and illness

Rate
Primary measurement used to describe either the
occurrance or the existence of a specific state of health or
illness

Outbreaks
Epidemic usually limited to a localized increase in the
incidence of the illness

John Graunt
Father of demographics. recognized importance of
recording birth and death rates and age structure of human
population

William Farr
Set up a system of data collection for causes of death in
difference occupations, gender, and imprisonment.
Importance of definition of illness and population
comparison, groups, and factors such as age, health, and
environment can affect statistics

John Snow
Used mapping and rates as an objective measure to
compare populations

Epidemiological Triad
Model based on the belief that health status is determined
by the interaction of the characteristics of the host, agent,
and environment

Wheel of Causation
Epidemiologic model that deemphasizes the agent as the
sole cause of disease while emphasizing the interplay of
physical, biological, and social environments

Web of Causation
Epidemioligcal model that strongly emphasizes the concept
of multiple causation while deemphasizing the role of
agents in explaining illness

Natural History of Disease


Course of disease or condition from the onset to resolution

Rate
Primary measure used to describe either the occurrence or
existence of a specific state of health or illness within a
group during a specific time frame

Ratio
Fraction that represents the relationship between two
numbers; Divide one quantity by another quantity
Ex: # boys in class/# girls in class

Proportion
Type of ration
Ex: # boys/ total students

Crude Rate

Measures the occurrence of the condition in the whole


population; May obscure info because it does not consider
factors such as age, race, gender
Numerator: Number of events
Denominator: Total population (not just those at risk)

Adjusted Rate
Controls for differences between populations-used for
comparison

Incidence
Measures probability that people without a condition will
develop the condition over time; measures pace of new
illness

Prevalence
Existence of a condition during a period/interval at a
specific point; Studies number of people diagnosed in the
past and length of illness; Longer length=greater the
prevalence-indicates burned of community

Mortality Rates
Crude mortality: probability of dying from any cause; #
deaths in a year/total population

Proportional Mortality Ratio


Compares death from a specific illness with deaths from all
causes; # deaths from specific cause within a time
frame/total deaths

Incidence Density
Used when unequal periods of observation for study
subjects; Accounts for people who die, drop out of a study,
or acquire an illness

Attributable Risk
The risk of a condition occurring in an exposed group that
is attributable to an exposure, not other factors

Relative Risk Ratio


Measure of the risk of developing a condition; Ratio of
incidence in exposed and incidence in non-exposed

Sensitivity
Ability of the test to correctly identify people who have the
health problem

Specificity
Specific incidence and prevalence rates calculated based
on specific characteristics (demographic data), variations
based on location and variations in time (short-outbreak,
periodic- seasonal, long-years)

Crude Rate Formula


# events within time period/population at risk with same
time pt

Incidence Rate
#new cases during time period/population at risk in the
same time period

Causality
Strengths of Association
Consistency
Temporality
Plausibility
Biological Gradient

Case Studies
In-depth analysis of individual or group, often first clue to a
problem

Cohort Studies
(Longitudinal Studies) Monitor over time to find
associations between risk factors and health;
Minimize selection bias;
Relative risk is the ratio of disease incidence in an exposed
population;
Indicates strength of exposure to illness causality

Case Control
Compare group with health problems (cases) to group
without (control)

Key Informants
Persons knowledgeable about specific aspects of a problem
and the communities current and past attempts to address
it

Stakeholders
An individual, organizational, or group that has interest
(stake) in a specific community health issue or the outcome
of a community level intervention

Gantt Chart
Visual of the sequence of steps to achieve objectives;
Developed in planning stages to identify steps, a tool for
scheduling tasks, monitor progress

Community of Interest
People and groups/aggregate that will be affected by
change; Those that will help bring about change

Upstream Approach to Health Care


Changes at societal level may impact health of community
without change is individual behavior

Sustainability
How to maintain change, support system in place

Social Justice
Health care is a risk; Address root cause of illness

Health Equities
Requires elimination of health disparities in living and
working conditions

SMART objectives
S- Specific
M- Measurable
A- Achievable
R- Relevant
T- Time

Coalition Building
Community members participate in planning and
implementing changes at the community level; Need clear
mission, goals, objectives, expectations, leadership,
accountability, and should be heterogenous

Community Readiness

Assess readiness to undertake change process-issue


specific

Web of Causation
Identify multiple factors that contribute to chronic disease

Lewins Model for Change


Unfreezing:
Changing:
Freezing:

Levers of Change
Increase driving/reinforcing forces
Decrease restraining/resisting forces

Force Field Analysis


Identify factors within a community that impact change and
forces that cannot be changes

Geopolitical Population
Has identified designated boundaries with the same
governing structure

Phenomenological Population
Group with common interests or beliefs who have
interpersonal and intrapersonal connection

Community of Solution
Formed by aggregate to address a health problem
In 1988 the Institute of Medicine (IOM) published a report
on the future of public health and its mission that defined
public health:
What society does collectively to ensure the conditons in
which people can be healthy.

The scope of practice of public health nurses is:


Population focused and community oriented, with a primary
emphasis on population-level interventions that target
strategies for health promotion and disease prevention.

Barriers to implementing population focused care include:


Lack of understanding of the public health nurse role and
it's relationship to other roles in nursing.

The role and goals of the community health nursing


practice can best be described as:
Population-level strategies aimed at promoting, preserving,
and maintaing the health of populations through the
delivery of personal health services to individuals, families,
and groups in an effort to improve the health of the
community as a whole.

Community health nursing practice is:


The synthesis of nursing theory and public health theory

Both community health and public health nursing are


considered to be:
Community-oriented practices involving free-living
(noninstitutionalized) clients.

Community based nursing practice is ____ specific.


Setting specific. Care is provided to clients where they live
(home health or hospice nursing, community based clinic)
work (occupational health nursing) and/or where they
attend school (school nursing).

The emphasis of community-based nursing practice


Acute and chronic care and the provision of
comprehensive, coordinated and continuous services,
usually within a specialty area.

Assessment is a core function of public health and refers


to:
Systematically collecting data concerning the population,
monitorin the population's health status and making
information available on the health of the community.

The public health core function of assurances focuses on


the:
Responsibility of public health agencies to be sure that
activities are appropriately carried out to meet public
health goals and plans. This also involves making sure that
essential health services are available and accessible,
especially to vulnerable populations.

A nurse planning a smoking cessation clinic for


adolescents in the local middle and high schools is
providing:

Community-oriented care.

Community-oriented nurses who provide direct care


services to individuals, families or groups are engaging in
a _________ practice.
Clinically oriented practice, even when they apply concepts
of population-focused, community-oriented strategies
along with their direct care clinical strategies.

Public health specialists of define problems at the _____


level as opposed to the ____ level.
Population level as opposed to the individual level. At the
population level, public health specialists are usually
concerned with more than one subpopulation and
frequently with the health of the entire community.

Mary Breckenridge established _____ in 1925:


The Frontier Nursing Service (FNS)

Mary Breckenridge established the FNS based on a


system of care care used in ______
The highlands and islands of Scotland.

One of Breckenridge's innovative contributions to health


care in the U.S was:
Introduction of the first nurse-midwifery training.

The unique pioneering spirit of the FNS influenced the


development of public health programs to improve the
health care of:
Rural and often inaccessible populations in the Appalachian
sections of southeastern Kentucky.

The most important contribution made by Florence


Nightingale to community-oriented nursing was:
Expansion of the role of nursing to include healthpromotion practices.

Florence Nightingale introduced professional nursing


eduction in _______
1850's England.

Florence Nightingale introduced the concept that


individual health depended on _____

community health, which expanded nurses' focus from care


of the ill to include a population-based approach, health
promotion and disease prevention

Differentiate between "sick nursing" and "health nurses",


as defined by Florence Nightingale
"Health Nursing" emphasized that nurses should strive to
promote health and prevent illness.

The Social Security Act of 1935 was designed to prevent


the ______
Reoccurence of the problems of the depression.

Social Security Act of 1935, Title VI


Provided funding for expanded opportunities for health
protection and promotion through education and
employment of public health nurses. Also provided $2
million in funding for research and investigation of disease.

The most relevant strategy related to Title VI of the Social


Security Act of 1935.
Provision of funding to support employment and education.

In 1902 Lillian Wald introduced the concept of school


nursing to address ____
The problem of school absenteeism

The primary model for the school nurse program by Lillian


Wald was to work with children in the schools and make
home visits for the purpose of:
Providing and obtaining medical treatment for absent
students.

In New York City in 1897, school medical examination


focused on:
Excluding infectious children from school.

In 1902, more than ___ of children might be absent from


school on any given day
20%

The shift in the U.S Congress to categorical funding


provides federal money for _____ rather than a _____
health program.

Priority diseases or groups rather than a comprehensive


community health program.

Primary health care differs from primary care in which


way?
Primary health care encourages community participation

T/F
Primary health care is the chief means of delivery of
health care in the United States.
False

The public health system at the state level is responsible


for:
Standing ready to prevent or respond to disasters, both
human caused and natural, overseeing health care
financing and the administration of programs such as
Medicaid and CHIP, providing mental health and
professional education; establishing health codes,
licenssing facilities and personnel; regulating the insurance
industry and providing direct assistance to local health
departments.

The WHO's Declaration of Alma-Ata (1978) identified


______________ as its primary global goal.
"Health for All in the Year 2000", later amended in 1998 to
"Health for All in the 21st Century."

The agency that assumes the responsibility for improving


health by expanding access to primary care for lowincome, uninsured or rural individuals is:
Health Resources and Services Administration

The Health Resources and Services Administration of the


Department of Health and Human Services at the federal
level of the public health system is responsible for:
Directing grant programs that improve the nation's health
by expanding access to primary care for low-income and
uninsured people, focusing on mothers and their children,
people with HIV/AIDS and residents of rural areas.

The major factor that drives the current discussions about


a Medicare shortfall in the middle of the twenty-first
century is:
Percentage of elderly in the U.S population

A nurse questions whether a particular activity in her job


description is within a nurse's scope of practice. The
nurse would look to which government jurisdiction/agency
within the piblic health system to seek clarification?
State board of nurse examiners.

A factor that strongly influences the success of a primary


health care system is:
Participation of the community members in the design,
implementation and evaluation of the initiative.

The focus of public health system in the US


Primary health care (PHC)

PHC is defined as the broad range of services including


but not limited to:
basic health services, family planning, clean water supply,
sanitation, immunization and nutrition education. The
emphasis is on prevention.

The Affordable Health Care for America Act of 2010 does


what?
-Requires most Americans to have health insurance
coverage
- Expands Medicaid
- Subsidizes private coverage for low and middle-income
people
- Transforms the health care system from a sick care
system to health care system
- Lowers mortality related to preventable causes
-Institutes health insurance reform,
- Increases access to affordable health care coverage
- Increases support for nursing workforce development
programs.

Evidenced-based nursing practice (EBP) can best be


defined as:
Approach to the integration of the best research available,
nursing expertise and the preferences/values of the clients
served. (2005 Sigma Theta Tau International)

Evidence-based public health is:


A public health endeavor in which there is informed,
explicit and judicious use of evidence that has been derived

using an of a variety of scientific and social science


research and evaluation methods.

External evidence:
Includes research and other evidence

Internal evidence:
Includes the nurse's clinical experiences and client's
preferences.

Community-oriented nurses use EBP most effectively


when they:
Base care on nationally accepted clinical guidelines, involve
clients in individual care decisions and include community
input

A community-oriented nurse seeks to implement EBP in


the community clinic's programs. The best model for the
nurse to apply is:
Community development

When nurses work with communities, EBP must also be


Culturally and financially appropriate.

The use of EBP that are both culturally sensitve is a


necessity to ensure:
Acceptance by the community and appropriate allocation of
limited resources.

A systematic review of the literature was conducted on


the 3 levels of practice and 17 interventions in the ______.
Intervention wheel.

When a public health nurse uses EBP to evaluate the


effectiveness, accessibility and quality of populationbased services within the community, the nurse is
addressing the core public health function of:
Assurance.

Generally ranked as the highest level of evidence:


Randomized controlled trial (RCT)

As randomized controlled trials are often inappropriate for


evaluation public health interventions, the most common

approach to establising evidence in public health is the


use of:
Case-control studies.

Melnyk and Fineout-Overholt (2010) described 7 Steps in


the EBP process
0. Cultivating a spirit of inquiry
1. Asking clinical questions
2. Searching for the best evidence
3. Critically appraising the evidence
4. Integrating the evidence with clinical expertise and client
preferences and values.
5. Evaluating the outcomes of the practice decisions or
changes based on evidence
6. Disseminating EBP results.

Approaches to EBP include:


Systematic review, meta-analysis, integrative review and
narrative review

Three Core public health functions:


Assessment, policy development, assurance

Core public health function: Assessment


- Diagnose and investigate health problems and hazards in
the community
- Mobilize community partnerships to identify and solve
health problems.
- Link people to needed health services
- Use EBP for new insights and innovative solutions to
health problems.

Core public health function: Policy Development


-Inform, educate and empower communities about health
issues.
- Develop policies and plans using EBP that supports
individual and community health effots

Core public health function: Assurance


- Monitor health status to identify community health
problems.
- Enforce laws and regulations that protect health and
ensure safety.
- Ensure the provision of health care that is otherwise
unavailable.
- Ensure a competent public health and personal health

care workforce.
- Use EBP to evaluate effectiveness, accessibility and
quality of personal and population-based services.

"Grading the strength of evidence"


When evidence is graded, the evidence is assigned a
'grade' based on the number and type of well-designed
studies and the presence of similar findings in all of the
studies.

The 'quality' of a study refers to:


The extent to which bias is minimized.

The 'quantity' of a study refers to:


The number of studies, the magnitude of the effect, and
the sample size.

The 'consistency' of a study refers to:


Studies that have similar findings, using similar and
different study designs.

POEM
"Patient oriented evidence that matters." Proposed by
Shaughnessy, Slawson and Bennett (1994), criteria for
evaluating the usefulness of evidence. In general, the
reader should ask "What are the results? Are they
important? Are the results valid? How can the results be
applied to client care?"

Barriers to adopting EBP in the community setting can


include:
-Miscommunication among nursing leaders about the
implementation process
-Inferior quality of available research or other types of
evidence
-Inability to assess and use evidence
- Unwillingness of organizations to fund research and make
decisions based on research
- The nurse's ability to make clinical decisions
- A lack of resources for implemnation in the setting (time,
funding, computer resources)

Four approaches allow the nurse to read


research/evidence in a condensed format:

Systematic review, meta-analysis, Integrative review,


Narrative review

Systematic review:
A method of indentifying, apparaising and synthesizing
research evidence. The aim is to evaluate and interpret all
available research that is relevant to a particular research
question. Is usually done by more than one person and
describes the methods used to search for and evaluate the
evidence.

Meta-analysis:
"A specific method of statistical synthesis used in some
systematic reviews, where the results from several studies
are quantitatively combined and summarized.

Integrative review:
A form of a systematic review that does not have the
summary statistic found in the meta-analysis because of
the limitations of the studies that are reviewed (e.g small
sample size of the population)

Narrative review:
A review is done on published papers that support the
reviewer's particular point of view or opinion and is used to
provide a general discussion of the topic reviewed. This
review does not often include an explicit or systematic
review process.

The Institue of Medicine has set a goal that by 2020, the


best available evidence will be used to make ___ of all
health care decisions.
90%.

Definition of Evidenced-based Medicine


"The conscientious, explicit and judicious use of current
best evidence in making decisions about the care of
individual clients." (Sackett et al 1996)
Pg 339

Research utilization is defined as:


"The process of transforming research knowledge into
practice" and "The use of research to guide clinical
practice."

A "paradigm shift"
A change from old ways of knowing to new ways of
knowing and practicing.

4 Ways of Knowing
Empirical knowledge, personal knowledge, aesthetic
knowledge, and ethical knowledge.

Empirical knowledge
The science of nursing

Aesthetic knowledge
The art of nursing

Personal knowledge
interpersonal relationships and caring

Ethical knowledge
Moral and ethical codes of conduct usually established by
professional organizations.

The term critical appraisal


is used to describe the process of evaluating the validity
and applicability of research studies.

Colonial Americans established systems of care for the


sick, poor, aged, mentally ill and dependents based on
___________.
Elizabethan Poor Law of 1601

The first hospital in the colonies: Founded when and


where?
In 1751, Pennsylvania Hospital, founded in Philadelphia.

The Shattuck Report


Published in 1850 by the Massachusetts Sanitary
Commission, called fopr major innovations: The
establishment of state health departments and local health
boards in every town, sanitary surveys and collection of
vital statistics, environmental sanitation, food, drug and
communicable disease control; well-child care, health
education, tobacco and alcohol control, city planning, and
the teaching of preventitive medicine in medical schools.

Sister Mary Augustine


Organized the Irish Sisters of Charity in Dublin in 1815.
These sisters visited the poor at home and established
hospitals and schools.

Nightingale's vision
Trained nurses and a model of nursing education
influencing the development of professional nursing, and
indirectly, public health nursing in the United States.
Inspired by the poorly educated, untrained and often
unreliable healthcare workers of the time.

District nursing
The Kaiserwerth Lutheran deaconesses incorporated care
of the sick in the hospital with client care in their homes.

William Rathbone
Nightingale's contemporary and friend, British
philanthropist. Founded the first district nursing
association.

The first nursing schools opened in:


The 1870's

Private duty nurses:


Lived with families of clients receiving care, often 24 hours
a day.

Visiting Nurses:
Became the key to communicating the prevention
campaign, through home visits and well-baby clinics.
Visitng nurses emphasized education of family members in
the care of the sick and in personal and environmental
prevention measures such as hygiene and nutrition.

Settlement Houses
Neighborhood centers that became hubs for health care,
education and social welfare programs.

Lilian Wald and May Brewster


Trained nurses that began visiting the poor on New York's
Lower East Side. They established a nurse's settlement
called Henry Street Settlement and later the Visiting Nurse
Service of NYC.

Lillian Wald
Established leader of public health nursing during it's early
decades. Known as the First Public Health Nurse in the US.
She persuaded the American Red Cross to sponsor rural
health nursing services across the country. Beginning in
1909, Walkd worked with Dr. Lee Frankel of MetLife to
implement the first insurance payment for nursing services.
She convinced business owners that keeping working
people and their families health increased productivity.

Jessie Sleet (Scales)


A canadian graduate of Provident Hospital School of
Nursing (Chicago), became the first African-American public
health nurse.

The American Red Cross through its Rural Nursing Service


provided:
A framework to initiate home nursing care in areas outside
larger cities, provided care of the sick and instruction in
sanitation and hygiene in rural homes. Also improved living
conditions in villages and isolated farms.

Occupational health nursing:


Began as industrial nursing and was a true outgrowth of
early home visiting efforts.

Ada Mayo Stewart


Began work with employees and families of the Vermont
Marble Company in Proctor, Vermont in 1895. Provided free
service for the employees, including obstetric care,
sickness care and some post-surgical care in worker's
homes. Unlike contemporary occupational health nurses,
Stewart provided very few services for work related
injuries.

Physicians in 1897 thoughts on schools and children


Focused on excluding infectious children from school rather
than on provider or obtaining medical treatment to enable
children to return to school.

Lina Rogers
A Henry Street Settlement resident that became the first
school nurse, worked with children in New York City schools

and made home visits to instruct parents and to follow up


on children absent from school.

School nurses (including Lina Rogers) found that many


children were absent from school for these reasons:
Lack of shoes or clothing, because of malnourishment, or
because they were serving as babysitters. (Hawkins, Hayes
and Corliss, 1994 p.417)

National Organization for Public Health Nursing (NOPHN)


Established in 1912, Lillian Wald first president. NOPHN
membership included both nurses and their lay supporters.

NOPHN sought to:


"Improve the educational and services standards of the
public health nurse, and promote public understanding of
and respect for her work." The NOPHN was soon the
dominant force in public health nursing.

In 1914 the first post-training program for public health


nursing at Teachers College in NYC because
Diploma schools of nursing emphasized hospital care of
patients, and graduates were often unprepared for home
visiting and required additional training. The American Red
Cross provided scholarships to attend the piblic health
nursing course.

American Public Health Association (APHA)


Established in 1872 to facilitate interprofessional efforts
and promote the 'practical application of public hygiene.'
The APHA targeted reform efforts toward contemporary
public health issues, including sewage and garbage
disposal, occupational injuries and STD's.

Frances Root
The first trained nurse in the United States, who was
salaried as a visiting nurse.

The National Organization for Public Health Nursing


Founded in 1912, provided essential leadership and
coordination of diverse public health nursing efforts; the
organization merged into the National League of Nursing in
1952.

Sheppard-Towner Act of 1921

Expanded community health nursing roles for maternal and


child health during the 1920's.

Implementation of Medicare and Medicaid programs


established:
Established in 1966, established new possibilities for
supporting community-based nursing care but encouraged
agencies to focus on services provided after acute care
rather than on prevention.

Between 1900 and 1955, the national crude mortality


rate decreased by ___%.
47%

The American Nurses Association


Merged with the National Association of Colored Graduate
Nurses, second national nursing association.

Definition of "Community" according to the WHO


"A group of people, often living in a defined geographical
area, who may share a common culture, values and norms
and are arranged in a social structure according to
relationships which the community has developed over a
period of time. Members of a community gain their
personal and social identity by sharing common beliefs,
values and norms which have been developed by the
community in the past and may be modified in the future."
( WHO, 2004)

Typologies
List of types, which involve classifying communities by
category rather than single definitions.

"Communities of place"
In this type of community, interactions occur within a
specific geographic area. Neighborhood and face-to-face
communities are two examples of this type of community.

'Communities of special interest'


Cut across geographic areas. Common concerns and
intererst, which can be long term or short term in nature,
bring their members together (e.g a group to support a
smoke-free environment.)

"Community of problem ecology"

Created when environmental problems such as water


pollution affect a widespread area.

In most definitions, the community includes three factors:


People, place and function

Community factor: People


The community members or residents, demographic
structure of population such as age, race, socioeconomic
and racial distributions. Informal groups such as block
clubs, service clubs as well as formal groups such as
schools, churches, businesses etc.

Community factor: Place


Refers to both geographic and to time dimensions.

Community factor: Function


Refers to the aims and activities of the community.

Community partnership approach


More effective than the 'I'm the community health expert
and I know best" are more effective in identifying and
addressing health problems, and such programs are more
likely to be sustained over the long term. Healthy People
2020 cites commmunity partnership as KEY to meeting
program goals.

Aggregate:
A collection of individuals who have in common one or
more personal or environmental characteristics.

The parts of a community are _________.


Interdependent, and their function is to meet a wide variety
of collective needs.

Public Health Nurses:


"integrate community involvement and knowledge about
the entire population with personal, clinical understandings
of the health and illness experiences of individuals and
families within the population."

The community is the client only when the nursing focus


is on:

The collective or common good of the population instead of


on individual health.

Population-centered practice:
seeks healthful change for the whole community's
benefit.Although the nurse may work with individuals,
families or other interacting groups, aggregates, or
institutions or within a population, the resulting changes
are intended to affect the whole community.

Community health is characterized by which 3


dimensions:
Status, structure and process dimensions.

Community Health: Status


Otherwise known as Outcome the most well-known and
accepted approach; it involves biological, emotional and
social parts.

Indicators of health status assessment: Death Rates


Due To:
1. Race/ethnicity-specific infant mortality
2. MVCs
3. Work-related injury
4. Suicide
5. Lung/Breast cancer
6. Cardiovascular disease
7. Homicide
8. All other causes

Indicators for Assessing Community Health Status:


Reported Incidence per 100,000 of the following:
1. AIDS
2. Measles
3. Tuberculosis
4. Primary and secondary Syphilis

Indicators for Assessing Community Health Status: Risk


Factors
1. Incidence of low birth weight
2. Births to adolescents
3. Prenatal care
4. Childhood poverty
5. Proportion of persons living in counties exceeding U.S
Environmental Protection Agency standards for air quality
during previous year.

Biological attributes of community health status:


Mobidity and Mortality, information found at the Bureau of
Vital Statistics

Social attributes of health status:


Includes crime rates and functional levels.

Emotional attributes of health status:


Include consumer satisfaction and mental health indices.

Community Health: Structure


usually defined in terms of services and resources.
Measures of ocmmunity health services and resources
include service use patterns, treatment data from various
health agencies and provider to client ratios.

Measures of community structures:


Demographics, such as socioeconomic and racial
distributions, age and educational level.

Community Health: Process:


Includes the concept of community competence, or the
ability of the parts of a community-organizations, groups
and aggregates to collaborate effectively in identifying the
problems and needs of the community. It directs the study
of community health to identify community action to
address community problems.

The most important aspect of the nursing community


assessment phase can best be described as:
Formulating a community nursing diagnosis.

The 10 Essential Conditions of Community Competence:


1. Commitment
2. Awareness of self and others
3. Articulateness
4. Effective communication
5. Conflict containment and management of the
accommodation
6. Participation
7. Management of relations with larger society
8. Machinery for facilitating participant interaction and
decision making

9. Social support
10. Leadership development

The three main characteristics of a successful


partnership:
Being informed, flexibility, negotiation.

Phase One of nursing process within the community:


Define community and establishment of
contract/partnership.

4 Steps to Assessing Community Health


1. Gathering relevant existing data and generating missing
data.
2. Developing a composite database.
3. Interpreting the composite database to identify
community problems and strengths.
4. Analyzing the problem.

Nursing Process of Community: Phase Two


Assessment

Data gathering:
the process of obtaining existing, readily available data.

Community Nursing Process: Phase Three


Nursing diagnosis of the problem

Community Nursing Process: Phase Four


Planning

Three Steps of Planning


1. Prioritize problems
2. Establish goals and objectives for each problem
3. Develop intervention activities to meet established
objectives.

Community Nursing Process: Phase Five


Implement Interventions

Community Nursing Process: Phase Six


Evaluate Interventions

Data Generation

The process of developing data that do not already exist


through interaction with community members or groups.

Four methods of generating direct data:


Informant Interview, focus groups, participant observation
and windshield studies

Informant interviews
i.e focus groups, directed talks with selected members of a
community about community members or groups and
events

Participant observation
The deliberate sharing in the life of a community
Four methods of generating direct data:
Informant Interview, focus groups, participant observation
and windshield studies

Informant interviews
i.e focus groups, directed talks with selected members of a
community about community members or groups and
events

Participant observation
The deliberate sharing in the life of a community
Community
A collection of people who interact with one another and
common interest forms a unity

Geographic community
A group of people usually defined by geographic
boundaries

American Public Health Association definition of public


health
Protect. Promote. Prevent

Evidence Based Practice


using evidence (research findings), along with clinical
judgment and patient wishes, in making decisions on how
to care for patients

Roles of Community Health Nurse


Clinician, Educator, Advocate, and Researcher

community of solution
a group of people who come together to solve one problem
- shape and size may vary depending on problem

aggregate
a mass of individuals who are considered as a whole but
are loosely connected

population
all people occupying an area or may also refer to people
who share one or more characteristics

health
a holistic state of well-being including a soundness of mind,
body, spirit

U.S. leading health indicators


used to measure the health of the US - serves as basis for
Healthy People 2020

goal of health promotion


raises the level of wellness for individuals, families,
communities, and populations

health promotion goals


1) increase span of healthy life for all citizens
2) reduce health disparities among population groups
3) achieve access to preventive health services for
everyone

levels of Prevention
Primary example immunization
Secondary example breast screening with mammography
Tertiary example diabetes education after diagnosis

Florence Nightingale
Author of modern day nursing

Lillian Wald

first person to use the term, "public health nursing"; started


the Henry Street Settlement; suggested school health
nursing

Margaret Sanger
a nurse who openly opposed the Cromstock act, arrested;
her defiance lead to International Planned Parenthood
Federation

core public health functions


Assessment
Policy Development
Assurance

five steps of evidence based practice


Ask the question
Collect the data/evidence
Critically appraise the evidence
Integrate into practice
Evaluate the decision

ethical decision making


making a choice that is consistent with moral code

cultural sensitivity
recognizing that culturally based values, beliefs, and
practices influence people health's and lifestyles and
should be considered in plans for service

epidemiological triad
host
agent
environment

chain of causation
begins by identifying the reservoir, then portal of exit, then
the agent, then the portal of entry - all set by the
environment

immunity
ability to resist particular infectious disease causing agent active, passive, cross-immunity

herd immunity

describes the immunity level within a group - mandatory


school immunizations relate to herd immunity

risk
the probability that a disease or health problem will occur

relative risk ratio


incidence rate of exposed group/incidence rate of
unexposed group. if greater than 1, those with risk factor
have a greater likelihood of acquiring than those who do
not have the risk factor

census data
data taken from the population - in the US every 10 years

reportable diseases
requirements to report certain diseases to local health
authority

descriptive epidemiology
observe or describe a pattern of disease in a population

rates
statistical measures expressing proportion of people with a
given health problem among a population at risk

prevalence
at a given point of time all the people with a particular
health condition

prevalence rate
number of people with characteristic/total population

incidence
all new cases of disease or health condition within a period
of time

incidence rate
number of new cases of disease or health condition/total
number at risk for developing disease. Note: once a person
has the disease/condition and there is not the possibility of
getting the disease/condition again then the risk is not
there and that number is subjected from the total
population.

mortality rate
sum of deaths in given population at a given time

analytic epidemiology
beyond simple description, seeks to determine associations

modes of transmission communicable disease


direct
indirect
airborne
vector

DOT
direct observed therapy - client takes the medication in the
presence of health care worker

quarantine
period of forced isolation of a person exposed to
communicable disease during the incubation period to
prevent the spread of the disease

ecological perspective
total relationship or patterns of relationship among people
and environment

built environment
structures built by humans within the environment - can be
used positively or negatively - urban vs rural

lead exposure issues


neurological issues, preterm birth, learning disabilities

epidemic
disease occurring that exceeds the normal or expected
frequency in a community

pandemic
worldwide distribution of disease

endemic
naturally occurring in population

communication
transferring meaning and enhancing understanding

health literacy
the ability to read, write, understand, and act upon health
information

collaboration
purposeful exchange between nurses, clients, health
professionals with shared values, mutual participation, and
joint effort

contracting
negotiating a working agreement between two or more
parties with shared understanding and mutual consent over
a period of time.

health promotion
behavior motivated by a desire to increase well-being and
actualize human potential (Pender, Murdough, & Parsons,
2006)

health protection (disease prevention)


behavior motivated by a desire to avoid disease, detect it
early, or maintain functioning within constraints of disease

Health Belief Model


theoretical model that has been extensively tested to
understand the behaviors and actions people take to
prevent illness and injury

Pender's Health Promotion Model


revised Health Belief Model that is used to predict
behaviors that influence health promotion

Quality Improvement Elements


Structure
Process
Outcomes

social marketing
influencing behavior in targeted audiences

social justice

both a process and a goal that includes a vision of an


equitable society and all members are physically and
psychologically safe and secure

advocacy
pleading a case for another or championing a cause
Community
Group of people sharing something, interacting with one
another. May exhibit commitment with one another and
may share geographic boundary. E.g., a NORC, a school,
NYU students.

Population
Group of people having at least one thing in common and
who may or may not interact with one another. E.g., nightshift workers, children with CF.

Community-focused nursing
Delivery of nursing care to improve health of an entire
community - bigger picture, deals with prevention and
population at systems level.

Population-focused care
Interventions for a defined "at risk" population (e.g. people
with hypertension, weight problems)

Public health nursing


Promotion and protection of health of populations using
knowledge from nursing, social and public health sciences.
Population-focused; goals: promotion of health and
prevention of disability and disease by creating conditions
in which people can be healthy.

Acute Care
Provider control
Predictable routine
Hospital policy
Resources available
Collaboration and consultation
Controlled PT compliance
Standardized care

Community Health Nursing

Familiar and comfortable environment for PT


Routine more determined by PT
Diverse resources
Autonomy, choice in HC decisions for PT by PT

Health
"extent to which an individual or group is able on the one
hand, to realize aspirations and satisfy needs; and on the
other hand, to change or cope with the environment." WHO

Levels of Care
Individual - downstream
Family
Community
Population - upstream

Public health wheel


For selling health messages
R/t epidemiology

Levels of Prevention
Primary
Secondary
Tertiary

Primary Prevention
PREVENTION
Stay well and avoid problem
Reduction of risk factors before occurrence of disease
condition/injury
E.g., exercise, brushing teeth, condoms, immunizations,
nutrition.

Secondary Prevention
PREVENTION/MAINTENANCE
SCREENINGS/EARLY DX
Early detection of potential for disease/condition or
existence of such while asymptomatic.
E.g., screening programs - Pap smears, mammograms,
colonoscopy.

Tertiary Prevention
RESTORATIVE
Treatment of existing or symptomatic disease to

prevent/delay progress.
Treatment after Dx, e.g, cardiac/stroke rehab.

Healthy People 2020


New topic areas:
adolescent health, blood, dementias inc. AD, early/mid
childhood, genomics, global health, HAIs, health related
QoL and well being, LGBT health, older adults,
preparedness, sleep, social determinants.
"Prevention Agenda for the Nation"

Public Health Nursing


Promoting, protecting health of populations using
knowledge from nursing, social and public health sciences.
Population focused.

Public Health Nursing Goals


Promotion of health and prevention of disease and
disability for all through creation of conditions in which
people can be healthy.

Assurance
Ensuring healthy populations
Appropriate and accessible services for all
Healthful physical environment
Stable ecosystem
Strong, supportive, non exploitive membership
Extensive participation in decision making
Members' basic needs provided for.
Access to resources and opportunities for interaction
Sustainable vital economy
Maintain connectedness with cultural and biological
heritages
Provide governance structures that promote health.

Community health work


unstructured, "bigger picture", community is both client
and partner, work at every level but especially advocate for
primary level with population focus.

Newest Vital Sign


Health literacy test: being able to read food labels.

Nursing Process in the community

ADPIE:
Assessment
Diagnosis (written in different format for the community)
Planning
Implementation
Evaluation

Community Assessment
Population assessment
Primary informants
Gaining entry (Kauffman's 5 phases)

Collecting data
IOSFCW
7 methods
Informant interviews
Observation
Secondary analysis of existing data
Focus groups
Community forums
Windshield surveys

Community Diagnosis
Risk of X
Among X group/population

Planning and Prioritization Phase


Priorities established (set by community members)
Goals and objectives IDed
Community focused interventions

Implementation Phase
Action phase
Role of community health nurse
Social change, community action

Evaluation
formative - changes made during process
summative - at end, related to outcomes

Health education
"The pedagogy of the oppressed"
Community empowerment (WHO)
Community as partner model (Betty Neuman)

Community as Partner
Community core
8 interacting community subsystems
Community stressors/boundaries
Normal line of defense LOD = status quo
Flexible line of defense - protection vs stressors
Lines of resistance LOR - strengths, resources, coping
mechanisms

Health Education
Planned learning experiences based on sound theories to
provide individuals, groups, communities opportunity to
learn info and skills to make quality health decisions

Health Education Goals


Help individuals, groups, communities achieve, by own
actions/initiative, optimal health
Facilitate voluntary actions to promote health
Empowerment
Advocacy
Self-efficacy

Planning health education


Establish outcomes - measurable
Mutual goal setting with community as client

Elizabethan Poor Law


Assisted the poor and disabled with receiving care

Marine Hospital
Established by Congress
Facility became the Public Health Service

Florence Nightingale
Established a training school for nurses, Established
sanitary nursing care units. Founder of modern nursing.
began professional education of nursing.

Clara Barton
Established The Red Cross, Launched the American Red
Cross in 1881. An "angel" in the Civil War, she treated the
wounded in the field.

Lillian Wald

"founder of the Henry Street Settlement and Visiting Nurse


Service, which provided nursing and social services and
organized educational and cultural activities; considered
the founder of public health nursing"
- President of the National Organization for Public Health
Nursing

Mary Breckinridge
Founder of the Frontier Nursing Service, in Leslie County,
KY; 1st midwifery program in U.S.

The Shattuck Report 1850


Report by the MA Sanitary Commission - recommended
changes to the health system for control and prevention of
diseases

Sheppard-Towner Act
U.S. Act of Congress providing federal funding for maternity
and child care, a response to the lack of adequate medical
care for women and children

Nightingale's Theory of Environment


the inter-relationship nursing, health and environment in
the health/illness of an individual or community

Orem Self Care Deficit


Individuals must take action to maintain life and health

Pender Health Promotion Model


Individuals will seek health promotion experiences to
improve their well-being

Community-Oriented Nursing
Client: Community
Focus: Health of community as a "whole"

Community-Based Nursing
Client: Individual, family, or groups
Focus: Individuals, families, and groups within the
community

Epidemiological Triangle
Host - Agent - Environment and (Vector)

Intermediary (Living) Vectors


e.g. Mosquitos, fleas, rodents, birds, ticks

Vehicle (Non-living) Vectors


e.g. Clothing, food, water

Incidence (new cases)


# cases in population at a specific time / population total x
1,000

Prevalence (existing at a particular time)


# cases in population at a specific time / population total x
1,000

Crude Mortality Rate


# deaths / population total x 1,000

Infant Mortality
# infants (< 1 yr.) specific year / # live births in the same
year

Attack Rate
# people developing a certain disease / # people at risk
Better nutrition, water, antibiotics, immunizations. PUBLIC
HEALTH.
Life Expectancy - U.S.
What factors/events resulted in the 21+ additional years of
life from 1900-1950???
What factors/events resulted in the 7 additional years of
life from 1950-1990?

1. Upstream
2. Downstream
There are two views of the health care picture
We need to question the way we have thought of health
and health care...
Should we focus on treatment of illness or prevention of
disease in the first place?
How should we act?
______ or ______

1. Microscopic
2. Medical Model
3. Downstream
_______ approach to solving community health problems
(the _____ or _____):
- Individual (family) response to health and illness
- Emphasizes behavioral responses to illness or lifestyle
patterns
- Nursing interventions aimed at the individual
------Changing lifestyles
------Changing perceptions or belief system
- The individual is the locus of change
- Focuses on the "cure"

1. Macroscopic
2. Public Health
3. Upstream
______ approach to solving community health problems
(______ or _______)
- Interfamily and intercommunity themes
- Emphasizes social, economic, and environmental
precursors of illness
- Nursing interventions may include modifying social or
environmental variables
- May involve social or political action
- society (social system) is the locus of change
- focuses on "prevention"

1. Health Belief Model


2. Health Promotion Model
3. Transtheoretical Model
4. Critical Social Theory
CHN practice is based upon theory. Theories are used
regularly in CHN. Some examples:
1.
2.
3.
4.

1.
2.
3.
4.
5.

The Health Belief Model


Perceived susceptibility
Perceived severity
Perceived benefits
Perceived barriers to action

6. Cues to action
7. Self-efficacy
_________:
- focuses on a person's perceptions of a threat or a health
problems and related appraisal of a recommended
behavior to elicit change
- developed in the 1950s and was one of the first models
using concepts and assumptions from behavioral sciences
to examine health behaviors
------- Addresses attitudes and beliefs of individuals
------- Focus was on increasing the use of preventive health
services
- Primary assumptions of the model are that people fear
disease and that health actions are motivated based on the
extent of the fear and belief in benefits obtained by
preventative action
- Five (Six?) Primary Constructs:
1.
2.
3.
4.
5.
6.

- "Using the Health Belief Model to Predict Injecting Drug


Users' Intentions to Employ Harm Reduction Strategies"
(Bonar & Rosenberg, 2011)
Studied whether perceived risk of infection would
impact intention to clean site pre-injection. Also studied
perceived risk of non-fatal overdose and use of a test
shot.
Found that the perceived risk of infection and overdose
did impact behavior if the user was not in withdrawal.
Application of the HBM in CHN:

Pender's Health Promotion Model


__________:
o Developed by Nola Pender to study health promotion
behaviors; initially published in 1982
o Explores biopsychosocial processes that motivate
individuals to engage in behaviors that promote health
o Has basis in the HBM
o Depicts the complex multidimensional factors with which
people interact as they work to achieve optimum health.
o This model does not include threat as a motivator, as
threat may not be a motivating factor for client's in all age
groups.

o Used by nurses to develop and execute health-promoting


interventions
o Used to develop research studies focusing on one aspect
of health promotion
o Used frequently as a framework for research studies
o Individual Characteristics and experiences that may
affect their health actions:
- Prior related behavior
- Personal factors
- Behavior specific cognitions and affect
- Perceived benefits of action
- Perceived self-efficacy
- Activity-related affect
- Interpersonal influences
- Situational influences
- Commitment to a plan of action
- Immediate competing demands and preferences
- Health-promoting behavior

"Registered Nurses' Beliefs of the Benefits of Exercise,


Their Exercise Behavior and Their Patient Teaching
Regarding Exercise" (Esposito & Patrick, 2011).
Wanted to know if nurses who engage in healthy
behaviors were more likely to recommend them to their
patients.
Found that nurses who believe in health promotion and
embrace healthy behaviors were more likely to be
positive role models and were more likely to teach
healthy behaviors to their patients.
- More examples: Health promotion for community dwelling elders and Prevention of melanoma
Application of the HPM in CHN:

1. Transtheoretical Model (Stages of Change)


2. Precontemplation
3. Contemplation
4. Preparation
5. Action
6. Maintenance
_________:
o Based on the assumption that behavior change takes
place over time
o Change is difficult
o Change progresses through a sequence of stages
-----------One may stop at a stage, progress to the next
stage or return to the previous stage

o Stages of change: 1.______, 2._____, 3.______, 4._____,


5.______
o Decisional Balance:
---------Pros: The benefits of behavior change
---------Cons: The costs of behavior change

1. Precontemplation
2. Contemplation
3. Preparation
4. Action
5. Maintenance
Transtheoretical Model (Stages of Change):
1. _______: No intention to change behavior in the next 6
months
------May be lack of information about consequences or
previous failure.
2. _______: Individual intends to change behavior in the next
6 months.
------Weighs pros and cons
3. _______: Individual intends to act within the next month
and has taken steps toward change
------Has a plan of action
4. _______: Individual has changed behavior for less than 6
months
------Change is sufficient to reduce disease risk.
5. _______: Individual has changed behavior for more than 6
months.
------Tries to prevent relapse
------Phase may last months to years

"Adapting the Transtheoretical Model of Change to the


Bereavement Process" (Calderwood, 2011).
Applies the model to the bereavement process.
Postulates that bereaved persons never return to their
previous state but undergo change as they cope with and
adjust to life without their loved one.
Application of the TTM in CHN:

1. Smoking cessation
2. Injury prevention
3. RNs assisting families obtain health insurance
Examples for applying Transtheoretical Model in nursing:
1.
2.
3.

1. Critical Social Theory


2. Enlightenment
3. Empowerment
4. Emancipation
_____________:
- Jurgen Habermas is the best known of the theorists for
this
-------Habermas was part of a group of German scholars in
Frankfurt writing in the 1960s
-------He promoted critical social theory to describe
"distortions and constraints that impede free, equal and
uncoerced participation in society."
- began in Marx's argument that oppression requires
revolutionary action
- uses societal awareness to expose social inequalities that
keep people from reaching their full potential
-------Promote equality
-------Reduce disparities (ie. health disparities)
- aims to provide an environment in which individuals can
become empowered, enlightened and emancipated
-------- ______: raising the consciousness of the oppressed.
-------- ______: encourages people to undertake activities to
improve their situation
-------- ______: is the goal of empowerment through which
new arrangements replace oppressive ones
- This perspective is informed by the following values and
assumptions:
---------- The problem and inequalities of health and health
care are connected to the particular historically located
social arrangements and the cultural values of society
---------- Health care should be oriented toward the
prevention of disease and illness
---------- The priorities of any health care system should be
based on the needs of the clients/population and not health
care providers
---------- Ultimately, society itself must be changed for
health and medical care to improve

"Why Carers of Older People are not using Available


Respite Services: An Australian Study (Stockwell-Smith,
Kellett, & Moyle, 2010).
Examined use of respite services by caregivers to
determine why some opted not to use these services.
Found that a lack of knowledge of services and trust in
the workers were factors in using respite care.
Found that caregivers were defined by their role and
could not easily relinquish it to another.

Application of CST in CHN:

1. Health Disparities
2. Health and social justice
Examples of applying critical social theory:
1.
2.

- A state of complete well-being, physical, social, and


mental, and not merely the absence of disease or
infirmity. World Health Organization, 1958
- The extent to which an individual or group is able, on
the one hand, to realize aspirations and satisfy needs;
and, on the other hand, to change or cope with the
environment. Health is, therefore, seen as a resource for
everyday life, not the objective of living; it is a positive
concept emphasizing social and personal resources, and
physical capacities. WHO 1986
How do we define health?

1. Purposeful actions
2. Processes
3. Responses
4. Behaviors
Definitions of Health:
Health consists of:
_____, ______, _____, or _____ that leads to: "soundness,"
"wholeness," or "well-being"

1. Physical Environment
2. Social Environment
3. Individual Behavior
4. Biology & Genetics
5. Health Services
6. Policy making
Determinants of Health:
Health Outcomes due to:
1.
2.
3.
4.
5.
6.

Physical Environment
Determinants of Health - Examples of ______ factors:
- Natural environment (ie plants, weather, climate change)
- Worksites, schools and recreational settings
- Housing, homes, neighborhoods
- Exposure to toxic substances
- Physical barriers
- Aesthetic elements (ie good lighting, trees, parks)

Social Factors
Determinants of Health - Examples of ______ factors:
- Availability of resources (ie educational and job
opportunities, living wages, healthful foods)
- Social norms and attitudes
- Exposure to crime and violence
- Social interactions
- Exposure to emerging technology (ie the Internet)
- Transportation options

Individual Behaviors
Determinants of Health - Examples of ______ factors:
- Diet
- Physical activity
- Alcohol, tobacco and other drug use
- Sexual behavior
- Hand washing

Biology and Genetics


Determinants of Health - Examples of ______ factors:
- Age
- Sex
- Race/ethnicity
- Inherited conditions
- Family history of diseases

Health Service
Determinants of Health - Examples of ______ factors:
- Barriers (lack of availability, high cost, lack of insurance,
language issues)
------- Unmet health needs
------- Delays in receiving care
------- Inability to get preventive services
------- Preventable hospitalizations

Policy Making

Determinants of Health - Examples of ______ factors:


- Health curricula requirements
- Increased taxes on tobacco
- Prohibitions on smoking
- Drinking age restrictions
- Seat belt laws and child restraints

1. Life expectancy
2. Health life expectancy
3. Years of potential life lost
4. Physically and mentally unhealthy days
5. Self-assessed health status
6. Limitation of activity
7. Chronic Disease Prevalence
Indicators of General Health Status:
1.
2.
3.
4.
5.
6.
7.

Community
___________:
- It can be a physical place or a geopolitical community.
- Has boundaries
- It can be a relational, interactive group. - A community
with no physical boundaries. A phenomenological
community is abstract. Churches, universities, online
groups are examples

Community
__________:
"A collection of people who interact with one another and
whose common interests or characteristics form the basis
for a sense of unity or belonging." (Nies and McEwen,
2011)

Community
__________:
"A group of people who share something in common and
interact with one another, who may exhibit a commitment
to one another and may share a geographic boundary."

1. People
2. They must interact with each other in some way
3. Something in common: an interest, a geographical
location, a commitment
A community has to have:
1.
2.
3.

1. Primary Prevention
2. Secondary Prevention
3. Tertiary Prevention
Levels of Prevention:
1.
2.
3.

Primary Prevention
_______ level of prevention
- activities preventing a problem before it occurs
- Health promotion
- Specific protection
- Immunizations

Secondary Prevention
________ level of prevention
- Early detection and prompt intervention
- Screening
- Early referral for treatment
- Screening for STDs

Tertiary Prevention
_______ level of prevention
- Focus on limitation of disability and rehabilitation
- Prevention progression of disease
- Reduce the effects of the disease
- Teaching insulin administration

Community Health Nursing (CHN)


_________:
o Focus on individuals and families where they live, work,
and go to school; care is setting-specific and the emphasis
is on acute and chronic care
o "The synthesis of nursing practice and public health
practice applied to promoting and preserving the health of
populations.

Public Health Nursing (PHN)


_________:
o Focus is on the community as a whole and the effect of
community health status (resources) on the health of
individuals, families and groups
o Consideration of the health of individuals, families, and
groups and their effect on the health of the community as a
whole
o The synthesis of public health and nursing practice.
o "The practice of promoting and protecting the health of
populations using knowledge from nursing, social, and
public health sciences."

Public/Community Health Nursing (PHN)


__________:
- According to the ANA, PHN is the
o ...practice of promoting and protecting the health of
populations
o ...using knowledge from nursing, social and public health
sciences

Public Health Nursing


_______ Practice is:
1. Population focused
2. Goals of promoting health and preventing disease and
disability
3. Focuses on all people through the creation of conditions
in which people can be healthy

Public/Community Health Nursing


__________:
- Focuses on the entire population
- Is based on assessment of the population's health status
- Considers the broad determinants of health
- Emphasizes all levels of prevention
- Intervenes with communities, systems, individuals, and
families
- like an umbrella, Covers:
--------- Home Health Nursing
--------- Palliative and Hospice Nursing
--------- Public Health Nursing (official agencies)
--------- School Nursing
--------- Occupational Health Nursing
--------- Correctional Nursing

o ...practice is general & comprehensive; not limited to an


age or diagnostic group
o ...is continuing, not episodic
o ...is directed to communities, groups, & individuals as it
contributes to the health of the total population
o Goal is to protect and promote the health of the entire
population
o Populations can be defined by geography, demographic
characteristic or need
What is Public Health Nursing (PHN)?

Anywhere people come together


Neighborhoods
Public Health Agencies
Community Centers
Day Care Centers
Schools
Workplaces
Housing projects
Settings for Public Health Nursing:
1.
2.
3.
4.
5.
6.
7.
8.

Community-based nursing
___________:
o Nursing of individuals and families to improve their
health
o Goals: Help them manage illness while they move among
health care settings
o Promote self-care and rehabilitation; prevent disease
o Processes: NP; diagnosis and treatment

Wherever individuals and families need nursing care


outside the hospital or nursing home
Home
Community clinics
School clinics
Workplace clinics
Rehabilitation centers
Settings for Community-based nursing:
1.

2.
3.
4.
5.
6.

Clinical Nursing
_________:
o Goal: improve the health of patients
o Clients: Patients of the health care system
o Processes used: nursing process, treatment and patient
care procedures
o Settings: inpatient

The Public Health Nurse


The Standards of Care:
__________:
- Assesses the health status of populations using data,
community resource identification, input from the
population and professional judgment
- Analyzes collected assessment data and partners with the
people to determine population diagnosis and priorities
- Participates with other community partners to identify
expected outcomes in the populations and their health
status based on population diagnoses and priorities
- Develops a plan that reflects best practices by identifying
strategies, action plans and alternatives to attain expected
outcomes
- Implements the identified plan by partnering with others
- Employs multiple strategies to promote health, prevent
disease and ensure a safe environment for populations
- Evaluates the health status of the population

1. Morbidity data - illness or disability


2. Mortality data - death
Indicators of Health and Illness:
- Health Statistics:
1.
2.

1.
2.
3.
4.
5.
6.

Surveillance
Disease and other health event investigation
Outreach
Screening
Case finding
Referral and Follow-up

7. Case management
8. Delegated functions
9. Health Teaching
10. Counseling
11. Consultation
12. Collaboration
13. Coalition Building
14. Community Organizing
15. Advocacy
16. Social Marketing
17. Policy and enforcement
Public Health Interventions:
1.
2.
3.
4.
5.
6.
7.
8.
9.
10.
11.
12.
12.
13.
14.
15.
16.
17

Surveillance
Describes and monitors health events through ongoing and
systematic collection, analysis and interpretation of health
data for the purpose of planning, implementing and
evaluating public health interventions

Disease and other health event investigation


Systematically gathers and analyzes data regarding threats
to the health of populations, ascertains the source of the
threat, identifies cases and others at risk, and determines
control measures

Outreach
Locates populations of interest or populations at risk and
provides information about the nature of the concern, what
can be done about it, and how services can be obtained

Screening
Identifies individuals with unrecognized health risk factors
or asymptomatic disease conditions in populations

Case finding
Locates individuals and families with identified risk factors
and connects them with resources

Referral and Follow-up


Assists individuals, families, groups, organizations and/or
communities to identify and access necessary resources to
prevent or resolve problems or concerns

Case Management
Optimizes self-care capabilities of individuals and families
and the capacity of systems and communities to coordinate
and provide services

Delegated Functions
Direct care tasks a registered professional nurse carriers
out under the authority of a health care practitioner as
allowed by law

Health Teaching
Communicates facts, ideas, and skills that change
knowledge, attitudes, values, beliefs, behaviors, and
practices of individuals, families, systems, and/or
communities

Counseling
Establishes an interpersonal relationship intended to
increase or enhance capacity for self-care and coping with
a community, system, and family or individual

Consultation
Seeks information and generates optional solutions to
perceived problems or issues through interactive problem
solving with a community, system, and family or individual

Collaboration
Commits two or more persons or organization to achieve a
common goal through enhancing the capacity of one or
more of the members to promote and protect health

Coalition Building
Promotes and develops alliances among organizations or
constituencies for a common purpose

Community Organizing
Helps community groups to identify common problems or
goals, mobilize resources, and develop and implement
strategies for reaching the goals they collectively have set

Advocacy
Plead someone's cause or act on someone's behalf, with a
focus on developing the community, system, and individual
or family's capacity to plead their own cause or act on their
own behalf

Social Marketing
Utilizes commercial marketing principles and technologies
for programs designed to influence the knowledge,
attitudes, values, beliefs, behaviors, and practices of the
population of interest

Policy development and enforcement


Places health issues on decision-makers' agendas, acquires
a plan of resolution, and determines needed resources,
resulting in laws, rules, regulations, ordinances, and
policies. Policy enforcement compels others to comply with
laws, rules, regulations, ordinances, and policies

John Snow
_______: Father of epidemiology

1. Pasteur
2. Lister
3. Koch
____, _____, _____: On "germs" and disease causation (late
1800s)

1. Edward Jenner
2. Edwick Chadwick
History of Public Health Nursing:
Pre 1850: Home visiting to sick as an act of charity.
- ______ observed people who worked around cattle were
less likely to have smallpox.

- _______ called attention to the consequences of unsanitary


conditions that resulted in health disparities that shortened
the life span of the laboring class in particular.

1. John Snow
2. Florence Nightingale
3. Pasteur, Lister, Koch
4. Lilian Wald
5. Mary Brewster
History of Public Health Nursing:
1850-1900: Home visiting to sick
- _______ demonstrated that cholera was transmissible
through contaminated water.
-________ - credited with establishing "modern nursing."
- ________: On "germs" and disease causation (late 1800s)
- In 1893, nurses _______ ("mother of community nursing")
and _______ established a district nursing service on the
lower east side of New York City called The House on Henry
Street.

1. 1900-1960s
History of Public Health Nursing:
________:
- Nursing in community centers for the poor
- Communicable disease control
- Communicable disease control/immunization
- Rise in PHN, home visits, school and OH nursing

1. 1960-1980
History of Public Health Nursing:
________: Care provided in public health clinics

1. 1980s-present
History of Public Health Nursing:
_________:
- Health promotion and education
- Health care access improvement

1. Poverty
2. Violence
3. HIV/AIDS

4. TB
5. Bioterrorism
Public/Community Health Nursing Today in U.S.
- Concerned with contemporary problems such as _____,
______, ______, _______, and _______.
- Practice is everywhere in public health agencies, schools,
occupational settings, community-based agencies, etc.

Healthy People
__________:
- provides science-based, 10-year national objectives for
improving the health of all Americans. For 3 decades, this
has established benchmarks and monitored progress over
time in order to:
o Encourage collaborations across communities and
sectors.
o Empower individuals toward making informed health
decisions.
o Measure the impact of prevention activities.
- Vision: A society in which all people live long, healthy lives

1. Identify nationwide health improvement priorities.


2. Increase public awareness and understanding of the
determinants of health, disease, and disability and the
opportunities for progress.
3. Provide measurable objectives and goals that are
applicable at the national, State, and local levels.
4. Engage multiple sectors to take actions to strengthen
policies and improve practices that are driven by the best
available evidence and knowledge.
5. Identify critical research, evaluation, and data
collection needs.
Healthy People 2020 strives to:
1.
2.
3.
4.
5.

1. Attain high-quality, longer lives free of preventable


disease, disability, injury, and premature death.
2. Achieve health equity, eliminate disparities, and
improve the health of all groups.
3. Create social and physical environments that promote
good health for all.
4. Promote quality of life, healthy development, and
healthy behaviors across all life stages.

Community Health Nursing Theories


Nighttingale's Theory of Environment
Health Belief Model
Milio's Framework for Prevention

Nightingale's Theory of Environment


Highlights relationship between an individual's environment
and health.
Depicts health as a continuum.
Emphasize preventive care.

Health Belief Model


Purpose to predict or explain health behavior.
Assumes that preventive behaviors are taken primarily for
the purpose of avoiding disease.
Emphasizes change at the individual level.
Describes likelihood of taking an action to avoid disease
based on:
Perceived susceptibility, seriousness, and threat of a
disease
Modifying factors
Cues to action (media, disease impact on family/friends,
recommendations from health care professionals).
Perceived benefits minus perceived barriers to taking
action.

Millie's Framework for Prevention


Complements the health belief model
Emphasizes change at the community level
Identifies relationship between health deficits and
availability of health promoting resources
Theorizes that behavior changes within large number of
people can ultimately lead to social change.

Community Health Nurse


Nurses who practice in the community.
Have a facility from which they work (community health
clinic, county health department), but not limited to
institutional settings

Community
Group of people and institutions that share geographic,
civic, and/or social parameters

Community-Based Nursing
Focus of Care (FOC):
Individuals, families
Nursing Activities (NA):
Illness care: Management of acute and chronic conditions
in settings where individuals, families, and groups live,
work, and "attend" (schools, camps, prisons)

Community-Oriented Nursing
FOC:
At-risk individuals, families, and groups
Community
NA:
Health care: Determining health needs of a community,
and intervening at the individual, family, and group level to
improve the collective health of the community

Community Health Nursing Practice


FOC:
Synthesis of nursing and public health theory
NA:
Promote, preserve, and maintain the health of populations
by the delivery of health services to individuals, families,
and groups in order to impact "community health"

Public Health Nursing Practice


FOC:
Synthesis of nursing and public health theory
NA:
Promote, preserve, and maintain the health of populations
through disease and disability prevention and health
protection of the community as a whole.
Core functions:
Systematic assessment of the health of populations
Development of policies to support the health of
populations
Ensuring that essential health services are available to all
persons

Principles Of Community Health Nursing


Overview:
Ethical considerations
Advocacy
Epidemiology
Epidemiological Triangle

The Epidemiological Process


Community-Based Health Education

Ethical Considerations
The Public Health Code of Ethics identifies the ethical
practice of public health.
Preventing harm
doing no harm
promoting good,
respecting both the individual and community rights
respecting autonomy and diversity
providing confidentiality
competency
trustworthiness
advocacy

Applications of Ethical Principles To Community Health


Nursing
Respect for autonomy
individuals select those actions that fulfill their goals
Situations: respecting a client's right to self-determination
(making a decision not to pursue chemotherapy)

Applications of Ethical Principles To Community Health


Nursing
Nonmaleficence
No harm is done when applying standards of care
Situations: Developing plans of care that include a system
for monitoring and evaluating outcomes

Applications of Ethical Principles To Community Health


Nursing
Beneficence
Maximize possible benefits and minimize possible harms.
Situations: Assessing risks and benefits when planning
interventions

Applications of Ethical Principles To Community Health


Nursing
Distributive Justice
Fair distribution of the benefits and burden in society is
based on the needs and contributions of its members
Situations: Determining eligibility for health care services
based on income and fiscal resources

Advocacy

The nurse plays the role of informer, supporter, and


mediator for the client.
The following are basic to client advocacy
Clients are autonomous beings have right to make
decisions affecting their own health and welfare
Right to expect a nurse-client relationship that is based on
trust, collaboration, and shared respect, related to health,
and considerate of their thoughts and feelings.
Clients responsible for their own health.
Nurse's responsibility to advocate for resources or services
that meet the client's health care needs.
Advocating requires assertiveness, placing priority on the
client's values, and willingness to progress through the
chain of command.

Epidemiology
Study of health-related trends in populations for the
purpose of disease prevention, health maintenance, and
health protection
Relies on statistical evidence to determine the rate of
spread of disease and the proportion of people affected.
Used to evaluate the effectiveness of disease prevention
and health promotion activities and to determine the
extent to which goals have been met.
Provides broad understanding of the spread, transmission,
and incidence of disease and injury. This information is an
important component of community assessment and
program planning.
Involves the study of the relationships among an agent, a
host, and an environment (referred to as the
epidemiological traingle).

Epidemiological triangle
The Agent is the animate or inanimate object that causes
the disease.
Chemical Agents: Drugs, Toxins
Physical Agents: Noise, Temperature
Infectious Agents: Viruses, Bacteria
The host is the living being that is affected by the agent.
Susceptible Host: Age, Gender, Genetics, Ethnicity,
Immunological status, Physiological state, Occupation
The environment is the setting or surrounding that sustains

the host.
Physical Environment: Geography, Water/food supply,
Presence of reservoirs/vectors
Social Environment: Access to health care, High-risk
working conditions, Poverty

Epidemiological Calculations
Incidence: number of new cases in the population at a
specific time/population total X 1,000 = per 1,000
Prevalence: number of existing cases in the population at a
specific time/ population total X 1,000 = per 1,000
Crude Mortality Rate: Number of deaths/population total X
1,000 = per 1,000
Infant Mortality Rate: Number of infant deaths before 1
year of age in a year/numbers of live births in the same
year X 1,000 = per 1,000
Attack Rate: Number of people exposed to a specific agent
who develops the disease/total number of people exposed

Epidemic
When the rate of disease exceeds the usual level of the
condition in a defined population.

The Epidemiological Process


Determine the nature, extent, and possible significance of
the problem.
Nurse collects information from as many sources as
possible. This information is then used to determine the
scope of the problem.

The Epidemiological Process


Using gathered data, formulate a possible theory.
Nurse projects and explores the possible explanations.

The Epidemiological Process


Make the plan
Focus on breaking the cycle of disease.
All factors influencing the spread of the disease must be
considered and identified.
Priorities are established to break the chain of transmission
and to control the spread of disease.

The Epidemiological Process


Put the plan into Action
Using all available means, the nurse puts the plan for
controlling the disease into action

The Epidemiological Process


Evaluate the plan
Gather pertinent information to determine the success of
the plan.
Using this plan, evaluate the success in prevention of the
spread of the disease

The Epidemiological Process


Report and follow up
Nures will synthesize evaluation data into a format that is
understandable. Then evaluates the success and failures
and bases follow-up on the evaluation information.

Community Health Education


Learning Theories
Behavioral Theory: use of reinforcement methods to
change learners' behaviors
Cognitive Theory: Use of sensory input and reputation to
change learners patterns of thought, thereby changing
behavior
Critical Theory: Use of ongoing discussion and inquiry to
increase learners depth of knowledge, thereby changing
thinking and behaviors
Development Theory: Use of techniques specific to learners
developmental stages to determine readiness to learn, and
to impart knowledge
Humanistic Theory: Assists learners to grow by
emphasizing emotions and relationship and believing that
free choice will prompt actions that are in their own best
interest
Social Learning Theory: Links information to beliefs and
values to change or shift the learners expectations

Community Health Education


Learning Styles

Visual: learns through seeing, note taking, video viewing,


and presentations. Think in pictures.
Auditory: learns through listening, verbal lectures,
discussion, and reading aloud. Interpret meaning while
listening.
Tactile-kinesthetic: learn through doing, trial and error,
hands-on approaches, and return demonstration. Gain
meaning through exploration.

Development of a Community Health Education Plan


First, Identify population-specific learning needs.
Consider population-specific concerns and effect of health
needs on the population to determine the priority learning
need.
Select aspects of learning theories, to use in educational
program based on identified learning needs.
Identify barriers to learning, and learning styles
Design the educational program

Disease Prevention
Levels Of Prevention Focus
Primary Prevention: Prevention of the initial occurrence of
disease injury
Nutrition education
Family planning and sex education
Smoking cessation education
Communicable disease education
Education about health and hygiene issues to specific
groups (day care workers, restaurant workers)
Safety education (seat belt use, helmet use)
Prenatal classes
Providing immunizations
Advocating for access to health care, healthy environments

Disease Prevention
Levels Of Prevention Focus
Secondary Prevention: Early detection and treatment of
disease with the goal of limiting severity and adverse
effects
Community assessments
Disease surveillance (communicable diseases)
Screenings
*Cancer (breast, cervical, testicular, prostate, colorectal)
*Diabetes mellitus
*Hypertension

*Hypercholesterolemia
*Sensory impairments
*Tuberculosis
*Lead exposure
*Genetic disorders/metabolic deficiencies in newborns
Control of outbreaks of communicable diseases

Disease Prevention
Levels Of Prevention Focus
Tertiary Prevention: Maximization of recovery after an
injury or illness (rehabilitation)
Nutrition counseling
Exercise rehabilitation
Case management (chronic illness, mental illness)
Physical and occupational therapy
Support groups
Exercise for hypertensive clients (individual)
Community
Group of people sharing something, interacting with one
another. May exhibit commitment with one another and
may share geographic boundary. E.g., a NORC, a school,
NYU students.

Population
Group of people having at least one thing in common and
who may or may not interact with one another. E.g., nightshift workers, children with CF.

Community-focused nursing
Delivery of nursing care to improve health of an entire
community - bigger picture, deals with prevention and
population at systems level.

Community-based nursing
Nursing care provided outside of acute care setting. Care
for families, individuals within a community. At individual,
"hands on" level. Does not require public health
background.

Population-focused care
Interventions for health promotion, disease prevention to
shape a community's overall health status.

Public health nursing

Promotion and protection of health of populations using


knowledge from nursing, social and public health sciences.
Population-focused; goals: promotion of health and
prevention of disability and disease by creating conditions
in which people can be healthy.

Acute Care
Provider control
Predictable routine
Hospital policy
Resources available
Collaboration and consultation
Controlled PT compliance
Standardized care

Community Health Nursing


Familiar and comfortable environment for PT
Routine more determined by PT
Diverse resources
Autonomy, choice in HC decisions for PT by PT

Health
"extent to which an individual or group is able on the one
hand, to realize aspirations and satisfy needs; and on the
other hand, to change or cope with the environment." WHO

Eudaimonistic Model
HOLISTIC
UPSTREAM
Greek origins. R/t Maslow's hierarchy of needs.
MOST HOLISTIC
Health = actualization/realization of complete fulfillment
and development. Illness impedes/prevents selfactualization.
Aim to redirect thinking away from mechanistic view
toward holistic view.
HOLISM, UNITY, INDIVIDUALITY

Adaptive Model
ABLE TO ADAPT
Medicine beyond treatment of disease: from writings of
Dubos.
Health means ability to adapt to a changing environment.
Disease is failure to adapt.
How flexible is someone in a changing environment?

Role-performance Model
ABLE TO DO JOB
Medical sociology, work of Parsons.
Person is healthy if they can perform their job.
Shapes health policies of military, industry.
Limited since one person may have many roles.

Clinical Model
MOST LIMITING
DOWNSTREAM
Health extreme = absence of S/S of disease/disability as
IDed by medical science
This model = downstream pattern of thinking rather than
upstream which is holistic.

Levels of Care
Individual - downstream
Family
Community
Population - upstream

Public health wheel


For selling health messages
R/t epidemiology

Levels of Prevention
Primary
Secondary
Tertiary

Primary Prevention
PREVENTION
Stay well and avoid problem
Reduction of risk factors before occurrence of disease
condition/injury
E.g., exercise, brushing teeth, condoms, immunizations,
nutrition.

Secondary Prevention
PREVENTION/MAINTENANCE
SCREENINGS/EARLY DX
Early detection of potential for disease/condition or
existence of such while asymptomatic.
E.g., screening programs - Pap smears, mammograms,
colonoscopy.

Tertiary Prevention
RESTORATIVE
Treatment of existing or symptomatic disease to
prevent/delay progress.
Treatment after Dx, e.g, cardiac/stroke rehab.

Healthy People 2020


New topic areas:
adolescent health, blood, dementias inc. AD, early/mid
childhood, genomics, global health, HAIs, health related
QoL and well being, LGBT health, older adults,
preparedness, sleep, social determinants.
"Prevention Agenda for the Nation"

Public Health Nursing


Promoting, protecting health of populations using
knowledge from nursing, social and public health sciences.
Population focused.

Public Health Nursing Goals


Promotion of health and prevention of disease and
disability for all through creation of conditions in which
people can be healthy.

Assurance
Ensuring healthy populations
Appropriate and accessible services for all
Healthful physical environment
Stable ecosystem
Strong, supportive, non exploitive membership
Extensive participation in decision making
Members' basic needs provided for.
Access to resources and opportunities for interaction
Sustainable vital economy
Maintain connectedness with cultural and biological
heritages
Provide governance structures that promote health.

Community health work


unstructured, "bigger picture", community is both client
and partner, work at every level but especially advocate for
primary level with population focus.

Newest Vital Sign

Health literacy test: being able to read food labels.

Nursing Process in the community


ADPIE:
Assessment
Diagnosis (written in different format for the community)
Planning
Implementation
Evaluation

Community Assessment
Population assessment
Primary informants
Gaining entry (Kauffman's 5 phases)

Kauffman's Five Phases


IBSBC
Impressing
Behaving - building rapport
Swapping - sharing information
Belonging - becoming part of the group
Chilling out - near end of relationship, beginning longer
term partnership with the community.

Collecting data
IOSFCW
7 methods
Informant interviews
Observation
Secondary analysis of existing data
Focus groups
Community forums
Windshield surveys

Community Diagnosis
Risk of X
Among X group/population

Planning and Prioritization Phase


Priorities established (set by community members)
Goals and objectives IDed
Community focused interventions

Implementation Phase

Action phase
Role of community health nurse
Social change, community action

Evaluation
formative - changes made during process
summative - at end, related to outcomes

Health education
"The pedagogy of the oppressed"
Community empowerment (WHO)
Community as partner model (Betty Neuman)

Community as Partner
Community core
8 interacting community subsystems
Community stressors/boundaries
Normal line of defense LOD = status quo
Flexible line of defense - protection vs stressors
Lines of resistance LOR - strengths, resources, coping
mechanisms

Health Education
Planned learning experiences based on sound theories to
provide individuals, groups, communities opportunity to
learn info and skills to make quality health decisions

Health Education Goals


Help individuals, groups, communities achieve, by own
actions/initiative, optimal health
Facilitate voluntary actions to promote health
Empowerment
Advocacy
Self-efficacy

Steps in Teaching/Learning Process


ADDIE
Assessment
Development of outcomes
Development of teaching plan
Implementation of teaching plan
Evaluation of outcomes (did it work?)

Facilitate learning

Stimulate senses
Active learning
Comfortable
Learner's readiness?
Relevant information
Repetition
Learning should be positive
Start simple, go to complex
Generalize and pace appropriately

Transtheoretical Model
Most common learning theory
Stages of Change
Precontemplation - resistance; don't see problem
Contemplation - know there's a problem and thinking about
it
Preparation - decision making; prepared for action, e.g.,
cutting down on cigarettes.
Action - modifies behavior/environment to overcome
problem. Takes lots of time and energy. W/D S/S now.
Maintenance - constant work to avoid relapse.

Transtheoretical Model: Stages of Change


PCPAM
Precontemplation
Contemplation
Preparation
Action
Maintenance

Nature of learning
CAP = B F D
Cognitive Domain
Affective Domain
Psychomotor Domain

Cognitive Domain
BRAIN
memory, recognition, understanding, application, problem
solving.

Affective Domain
FEELINGS
attitudes, values:appreciates, chooses, accepts - role play,
talk to s.o., feelings.

Psychomotor Domain
DEMONSTRATE
Hands on manipulation, "teach back"
performance of skills requiring coordination and motor
skills.

Planning health education


Establish outcomes - measurable
Mutual goal setting with community as client

Educational objectives
Cognitive
Affective
Psychomotor
Community Health Nursing:
-Is it broad or specific?
-Wide variety of settings or limited?
-Do they promote health and welfare of pts?
-Specific age groups?
-Diverse populations or specific cultures?
-Nurses working in the community should have an
understanding of what 3 things?
Community Health Nursing:
-broad
-wide variety of settings
-allows nurses to practice in wide variety of settings
-Promote health and welfare of clients
-across the lifespan
-Nurses working in the community should understand the
foundations of CHN, the principles of CHN, and health
promo and disease prevention

What provides the basis for care of the community and


family?
Nursing theory

What are three examples of CHN theories?


Nightingale's theory of environment
Health belief model
Milio's framework for prevention

Nightingale's Theory of environment:


-What relationship does it highlight?
-What does it depict health as?
-What does it emphasis?

Nightingale's theory of environment:


-Highlights the relationship b/w an individual's environment
and health
-Depicts health as a continuum
-Emphasizes preventive care

Health belief model:


-What is the purpose?
-Purpose of preventive health behaviors?
-Emphasizes change at what level?
-Describes the likelihood of taking an action to avoid
disease based on what 4 things?
Health belief model:
-Purpose is to predict or explain health behaviors
-Preventive health behaviors- primarily taken to avoid
disease
-Emphasizes change at the individual level
-Describes the likelihood of taking an action to avoid
disease based on:
1)Perceived susceptibility, seriousness, and threat of a
disease
2)Modifying factors (e.g., demographics, knowledge level)
3)Cues to action (media campaigns, disease impact on
family/friends, recommendations from HCP)
4) Perceived benefits minus perceived barriers to taking
action

Milio's framework for prevention:


-What does it complement?
-What does it emphasize?
-What relationship does it identify?
-What does it theorize?
Milio's framework for prevention:
-Compliments the health belief model
-Emphasizes change at the community level
-Relationship b/w health deficits and availability of heath
promoting resources
-Behavior changes w/in a large number of people can
ultimately lead to social change

-What is a community?
-Do communities vary in their characteristics and health
needs?
-What is a community's health determined by?
-Community- group of people and institutions that share
geographic, civic, and/or social parameters.
-Communities vary in the characteristics and health needs.

-Community's health is determined by the degree to which


the community's collective health needs are identified and
met.

-What are health indicators?


-What are some examples?
-What do they serve as targets for?
-Used to describe the health status of a community.
-Health indicators- mortality rates, disease prevalence,
levels of physical activity, obesity, tobacco and substance
use
-Serve as targets for the improvement of a community's
health.

-Describe how community health nurses work: facilities,


limitations, etc.
-Community health nurses practice in the community. They
usually have a facility from which they work (community
health clinic, health department, etc.) but aren't limited to
institutional settings.

What does this describe: an aggregate who shares one or


more personal characteristic
A population

Who/what is the client in CHN?


Client is a community or a population w/in the community

When does community partnership occur?


It occurs when community members, agencies, and
businesses actively participate in the processes of health
promo and disease prevention

What is the development of community partnerships


critical to?
The development of community partnerships is critical to
the accomplishment of health promo and disease
prevention strategies

What are the purposes of population-focused nursing?


Purposes of population-focused nursing:
-Assessing to determine needs
-Intervening to protect and promote health
-Preventing disease w/in a specific population (people at

risk for HTN, people w/o insurance, people w/knowledge


deficits, etc.)

Community based nursing:


-Who is the focus of care?
-What are the nursing activities?
-Focus of care: individuals and families
-Nursing activities are r/t illness care: mgmt. of acute and
chronic conditions in settings where individuals, families,
and groups live, work, and "attend" (schools, camps,
prisons)

Community-oriented nursing:
-Who is the focus of care?
-What are the nursing activities?
-Focus of care: at risk individuals, family, groups as well as
community.
-Nursing activities are r/t health care: determining health
needs of a community, and intervening at the individual,
family, and group level to improve the collective health of
the community.

Community health nursing practice:


-Who is the focus of care?
-What are the nursing activities?
-Focus of care: synthesis of nursing and public health
theory
-Nursing activities are to promote, preserve, and maintain
health of populations by the delivery of health services to
individual, family, and groups in order to impact
"community health"

Public health nursing practice:


-Who is the focus of care?
-What are the nursing activities, including the core
functions?
-Focus of care: synthesis of nursing and public health
theory
-Nursing activities are to promote, preserve, and maintain
the health of population through disease and disability
prevention and health protection of community as a whole.
-Core functions: systematic assessment of health of
population, devotion of policies to support the health of
populations, and ensuring essential health services
available to everyone.

What do the principles for guiding CHN include?


Ethical considerations
Advocacy
Epidemiology
Epidemiological calculations
Epidemiological triangle
Epidemiological process
Community-based health education

What does the Public Health Code of Ethics identify?


It identifies the ethical practice of public health

What do ethical considerations include?


Ethical considerations include preventing harm, doing no
harm, promoting good, respecting both individual and
community rights, respecting autonomy and diversity, and
providing confidentiality, competency, trustworthiness, and
advocacy

What are community health nurses concerned with and


what do those concerns reflect?
-Community health nurses are concerned with protecting,
promoting, preserving, and maintaining health, as well as
preventing disease.
-These concerns reflect the ethical principle of promoting
good and preventing harm.

Is it easy or hard to balance individual and community


rights?
Balancing individual rights vs rights of community groups is
a challenge.

What challenge do community health nurses face?


Community health nurses address the challenges of
autonomy and providing ethical care

What do client rights include?


Client rights include the right to info disclosure, privacy,
informed consent, info confidentiality, and participation in
TX decisions

Autonomy:
-Definition
-Situation

-Individuals select those actions that fulfill their goals.


-Respecting a client's right to self-determination (making a
decision not to pursue chemo)

Nonmaleficence:
-Definition
-Situation
-No harm is done with applying standards of care.
-Developing plans of care that include a system for
monitoring and evaluation outcomes.

Beneficence:
-Definition
-Situation
-Maximize possible benefits and minimize possible harms.
-Assessing risks and benefits when planning interventions.

Distributive justice:
-Definition
-Situation
-Fair distribution of the benefits and burden in society is
based on the needs and contributions of its members.
-Determining eligibility for health care services based on
income and fiscal resources.

What are some basics for client advocacy?


Nurse plays the role of the informer, supporter, and
mediator for the client.

-Are clients autonomous beings?


-Do clients have the right to make decisions affecting
their own health and welfare?
-Are clients responsible for their own health?
Clients are autonomous beings, have the right to make
decisions affecting their own health and welfare, and are
responsible for their own health.

Whose responsibility is it to advocate for resources or


services that meet the client's health care needs?
It's the nurse's responsibility to advocate for resources or
services that meet the client's health care needs.

What three things are needed to advocate for the clients?

Advocating for clients requires assertiveness, placing


priority on the client's values, and willingness to progress
through the chain of command for resolution.

What is epidemiology?
The study of health-related trends in populations for the
purpose of disease prevention, health maintenance, and
health protection.

-What does epidemiology rely on to determine there rate


of spread of disease and the proportion of people
affected?
-What is it also used to evaluate?
It relies on statistical evidence to determine the rate of
spread of disease and the proportion of people affected.
-It is also used to evaluate the effectiveness of disease
prevention and health promo activities and to determine
the extent to which goals have been met.

What does epidemiology provide a broad understanding


of? What is that information an important component of?
It provides a broad understanding of the spread,
transmission, and incidence of disease and injury. This
information is an important component of community
assessment and program planning.

What type of things do community nurses do?


Identify cases, recognize patterns of disease, eliminate
barriers to disease control, and provide education and
counseling targeted at a disease condition or specific risk
factors.

What involves the study of the relationships among an


agent, host, and an environment?
Epidemiology

What does the interaction of the epidemiological triangle


determine?
It determines the development and cessation of
communicable diseases, and they form a web of causality,
which increases or decreases the risk for disease.

Epidemiological triangle:
What are the 3 components?
Agent, host, environment

What is the agent? Host? Environment?


Agent- animate or inanimate object that causes the
disease.
H0st- living being that is affected by the agent.
Environment- setting or surrounding that sustains the host.

What are some of the epidemiological calculations?


Incidence
Prevalence
Crude mortality rate
Infant mortality rate
Attack rate

What is the incidence?


Number of new cases in the population at a specific time
divided by population total x 1,000= ____ per 1,000

What is the prevalence?


Number of existing cases in the population at a specific
time divided by population total x 1,000 = ____ per 1,000

What is the crude mortality rate?


Number of deaths divided by population total x 1,000 =
_____ per 1,000

What is the infant mortality rate?


Number of infant deaths before 1 year of age in a year
divided by numbers of live births in the same year x 1,000
= _____ per 1,000

What is the attack rate?


Number of people exposed to a specific agent who develop
the disease divided by total number of people exposed

What is an epidemic?
The rate of disease exceeds the usual level of the condition
in a defined population

What are the 7 phases of the epidemiological process?


1) determine the nature, extent, and possible significance
of the problem.
2) using the gathered data, formulate a possible theory
3) Gather info from a variety of sources in order to narrow

down the possibilities


4) Make the plan
5) Put the plan into action
6) Evaluate the plan
7) Report and follow up

-What things influence the host?


-What things influence the physical and social
environment?
-What things influence chemical, physical, and infectious
agents?
-Host: age, gender, genetics, ethnicity, immunological
status, physiological state, occupation
-Physical environment: geography, water/food supply,
presence of reservoirs/vectors
-Social environment: access to health care, high-risk
working conditions, poverty
-Chemical agents: drugs and toxins
-Physical agents: noise and temperature
-Infectious agents: viruses and bacteria

Why do nurses regularly provide health education?


In order to promote, maintain, and restore the health of
populations

What barriers must nurses take into account when


designing community education programs?
Age, cultural barriers, poor reading and comprehension
skills, language barriers, barriers to access, and lack of
motivation

What does effective community health education require?


Planning

What are the 6 learning theories used in CHN?


Behavioral theory
Cognitive theory
Critical theory
Developmental theory
Humanistic theory
Social learning theory

What is the behavioral theory?


Use of reinforcement methods to change learners'
behaviors

What is the cognitive theory?


Use of sensory input and repetition to change learners'
patterns of thought, thereby changing behaviors

What is the critical theory?


Use of ongoing discussion and inquiry to increase learners'
depth of knowledge, thereby changing thinking and
behaviors

What is the developmental theory?


Use of techniques specific to learners' developmental
stages to determine readiness to learn, and impart
knowledge

What is the humanistic theory?


Assists learners to grow by emphasizing emotions and
relationships and believing that free choice will prompt
actions that are in their own best interest.

What is the social learning theory?


Links info to beliefs and values to change or shift the
learners' expectations

How do visual learners best learn?


They learn through "seeing" and methods such as note
taking, video viewing, and presentations. They "think in
pictures"

How do auditory learners best learn?


They learn through "listening" and methods such as verbal
lectures, discussion, and reading aloud. They "interpret
meaning while listening"

How do tactile-kinesthetic learners best learn?


They learn through "doing" and methods such as trial and
error, hands-on approaches, and return demonstration.
They gain "meaning through exploration"

What are the steps to develop a community health


education plan?
1) Identify population-specific learning needs.
2) Consider population-specific concerns and effect of
health needs on the population to determine the priority

learning need.
3) Select aspects of learning theories to use in the
educational program based on the identified learning
need.
4) Identify barriers to learning, and learning styles.
5) Design the education program.
6) Implement the education program. Ensure an
environment that is conducive to learning
7) Evaluate the achievement of learning objectives and the
effectiveness of instruction.

What are national health goals derived from? What are


these goals based on
Scientific data and trends collected during the prior
decade. These goals are based on those issues that are
considered major risks to the health and wellness of the US
population.

What is Healthy People?


Every 10 years it publishes the national health objectives
that serve as a guide for promoting health and preventing
disease.

What are some examples of preventive services?


Health education and counseling, immunizations, and other
actions that aim to prevent a potential disease or disability

What do successful screening programs provide?


Successful screening programs provide accurate, reliable
results, can be inexpensively and quickly administered to
large groups, and produce few if any side effects

What is primary prevention?


What are some examples?
Primary prevention is prevention of the initial occupancy of
disease or injury.
Nutrition education, family planning and sex ed., smoking
cessation education, communicable disease education,
education about health and hygiene issues to specific
groups (day care workers, restaurant workers), safety
education (seat belt use, helmet use), prenatal classes,
providing immunizations, advocating for access to health
care, healthy environments

What is secondary prevention?


What are some examples?

Secondary prevention is early detection and TX of disease


w/the goal of limiting severity and adverse effects.
Community assessments, disease surveillance
(communicable diseases), screenings, control of outbreaks
of communicable diseases

What is tertiary prevention?


What are some examples?
Tertiary prevention is maximization of recovery after an
injury or illness (rehab).
Nutrition counseling, exercise rehab, case mgmt. (chronic
illness, mental illness), PT and OT, support groups, exercise
for hypertensive clients (individual)

A nurse manager at a community agency is developing


an orientation program for newly hired nurses. When
discussing the differences between community-based and
community-oriented nursing, the nurse should include
which of the following as examples of community-based
nursing? (Select all that apply.)
A. A home health nurse performing wound care for a
client who is immobile.
B. An occupational health nurse providing classes on body
mechanics at a local industrial plant.
C. A school nurse teaching a student who has asthma
about medications.
D. A parish nurse teaching a class on low-sodium cooking
techniques.
E. A mental health nurse discussing stress mgmt.
techniques with a support group.
Answer: A and C.
A: Correct- community based nursing involves mgmt. of
acute and chronic conditions in a community setting
B: Incorrect- this is community-oriented nursing, which
involves health care of individuals, families, and groups to
improve the collective health of the community.
C: Correct- community based nursing involves mgmt. of
acute and chronic conditions in a community setting
D: Incorrect- this is community oriented nursing
E: Incorrect- this is community oriented nursing

A nurse is advocating for local leaders to place a newly


approved community health clinic in an area of the city
that has fewer resources than other areas. The nurse is
advocating for the leaders to uphold which of the
following ethical principles?
A. Distributive justice

B. Fidelity
C. Respect for autonomy
D. Veracity
Answer: A
A: Correct- distributive justice is fair distribution of benefits
and burden in society
B: Incorrect. Fidelity involves keeping commitments and
following through with promises.
C: Incorrect. Autonomy is supporting the rights of
individuals to determine and pursue personal HC goals.
D: Incorrect. Veracity is the concept of telling the truth.

A nurse is preparing an education program on disease


transmission for employees at a local day care facility.
When discussing epidemiological triangle, the nurse
should include which of the following as agents? (Select
all that apply.)
A. Resource availability
B. Ethnicity
C. Toxins
D. Bacteria
E. Altered immunity
Answer: C and D.
A: Incorrect- resource availability is r/t environment
B: Incorrect- ethnicity is r/t host
C: Correct- toxins are r/t agents
D: Correct- bacteria is r/t agents
E: Incorrect- altered immunity is r/t host

A nurse is developing a community health education


program for a group of clients who have a new diagnosis
of DM. Which of the following learning strategies should
the nurse include for clients who are auditory learners?
A. Showing informational videos
B. Providing equipment to practice hands-on skills
C. Supplying outlines for note-taking
D. Facilitating small group discussions
Answer: D
A: Incorrect- this is for visual learners
B: Incorrect- this is for tactile-kinesthic learners
C: Incorrect- this is for visual learners
D: Correct- this is for auditory learners

A community health nurse is implementing health


programs with several populations in the local area. In
which of the following situations is the nurse using
primary prevention?

A. Performing a home safety check in a client's home.


B. Teaching healthy nutrition to clients how have HTN.
C. Providing influenza immunization to employees at a
local preschool.
D. Implementing a program to notify individuals exposed
to communicable disease.
Answer: C.
A: Incorrect- performing a home safety check is secondary
prevention
B: Incorrect- teaching healthy nutrition to clients with HTN
is tertiary prevention
C: Correct- immunizations is a primary prevention task.
D: Incorrect- notifying exposed individuals is secondary
prevention
Community Health Nursing
blend of primary health care and nursing practice with
public health nursing

primary goal of community health nursing


raise level of health for citizens
promote physical/ mental health
prevent disease, injury, disability

home health care


enable individuals of all ages to remain in the comfor and
security of their homes while reiciving health care

services of home health care


skilled nursing
physical therapy
psychiatric therapy
pain education/managment
speech language therapy
occupational therapy
social services
intravenous therapy
acquistition of medical supplies/ equipment
home health aide
homemaker
petcare assistance
companion care
respiratory therapy
nutritional support

goal of home health care

allow as much independent as possible

4 perspectives of home health care


1. official
2. patient
3. family
4. provider

OFFICIAL home health care


services in home to promote, maintain, restore health
minimize effect of illness/ disability

PATIENT home health care


one on one health care provider in home
basic care/ individualized needs on personalized schedule
over a period of time = adjustment, change, learning takes
place effectively

FAMILY home health care


family together as functioning unit
goals: learning to adapt to change, prevent dysfunction,
family wellness, emotional support, community support

PROVIDER home health care


provide excellent care in less than excellent surroundings
goal: independence, creativity, communication, clinical
skills, daily practices

Earliest organized home health care


1617
st. vincent de paul
sisterhood of the dames of charite
met social welfare and visiting nursing needs

Boston Dispensary
1796
1st home care program in US

1886
1st visiting nurse service in US
Philadelphia

Lillian Wald & Mary Brewster

1893
visiting nurse service
for the poor
NYC
Nurses Settlement House on Henry Street

Social Security Act of 1935


1st provided government rather than local charitable
funding for select services such as maternal health,
treatment for communicable diseases, training of public
health professionals

medicare became effective in ?


1996
revolutionized home care

medicare revolutionized home care by


1. changing it to a medical rather than nursing model of
practice
2. defining and limiting the services it reimbursed
3. changing the payment source and even changing the
reason for providing home care

Prospective Payment System (PPS)


1983
part of the Tax Equity and Fiscal Responsibility Act for
hospitals receiving Medicare reimbursement

Diagnosis-Related Groups (DRGs)


pays a set rate accordig to diagnosis for the hospitalized
patient's care rather than the "cost" or charges and
instituion traditionally bills according to its own schedule of
fees - discharge for such patients occured earlier and
patient required more nursing care

Types of Home Care Agencies


1. licensure by state
2. certification by state
3. certificate of need
4. accreditation by an outside agency

Licensure by state
gives legal permission to operate w/i state only

Certification by state

federal gov. set the rules governing certificates

Certificate of need
some states grant according to rules and formulas that
state regulator devise

Accredidation by outside agency


evaluates and judges how well the agency meets certain
standsards that the accrediting organization sets
(NLN community health accreditation program)

Before Medicare agencies classified by


Visiting nurse associations, state and local health
departments, nursing divisions, hospitals controlled
provision of home health care

Agencies classified according to


1. tax status - profit or non
2. location - freestanding or institution
3. governance - private or public

Joint Commission (TJC)


looking for agencies to establish ethics committees to
handle ethical issues that arise in the home

Changes to Home Health Care


1. psychiatric patients are required to be under the care of
psychiatrist & have diagnosis
2. social workers taking more active role
3. nurse pain specialists
4. agencies obtaining separate medicare certification for
hospice care
5.pet-care programs to reduce patient stress
6. electronic home visits
7. telemonitoring
8. home infusion therapy
9. home IV therapy

Types of Home Care Agencies


1. Voluntary
2. official
3. comvination
4. hospital
5. proprietary

6. private not for profit


7. other

State licensing boards & Professional organizations


dictate what?
functions and scopes of practice
including:
skilled nursing, physical therapy, speech-language therapy,
occupational therapy, medical social services, homemakerhome health aide

RNs provide what?


direct skilled nursing services

LVNs provide what?


basic nursing services under the supervision of the RN
needed skills:
self-direction, motivation, creativity, clinical proficiency,
flexibility, compassion, empathy, patience

Service Goals of Skilled Nursing


1. Restorative
2. improvement
3. maintenance
4. promotion

Restorative
return to previous level of function as appropriate/ realistic

Improvement
achieving better health and highter level of function than at
admission

maintenance
preserving functional capacities and independence by
maintaining current level of health

Promotion
teaching healthy lifestyles that keep the effect of illness or
disaibility to a minimum and prevent the recurrence of
illness

Skills of home health nurse

1.
2.
3.
4.
5.

technically proficiency
self motivated
independent decision maker
respond prompltly to problems
able to adapt to family/patient/ home enviroment

LVN home health duties


catheter care
ostomy care
wound care/ sterile dressing
obtaining specimen
injections
prefilling insulin
fingersticks for blood glucose
monitor physical status
set up/ monitor meds including IV
nutrition
therapeutic diet teaching/reinforcement
respiratory care, ventalation
tracheostomy care and suctioning
enemas
pain management
emotional support
preventative health measures
vital signs
patient / family teaching

physical therapy
licensed / qualified physical therapist is required, PT
assistant
goals: restorative and maintanance
rehab plan taught to patient/ family to promote self care

Speech Language Therapist


to be reimbursed by medicare:
masters prepared physician
certified by american speech/ hearing association
range from:
language relearning
eating swallowing disorders
lipreading

Occupational therapist

bachelor's level preparation


registered
choose / teach theraputic activities to restore funtional
levels:
promote independance
analysis of activities relating to patients' skin
disease management lifestyle
design , fabricate, fit, orthotic or self help devices
improve performance of activities of daily living, sensorymotor, cognitive, and neuromuscular function

Medical social services


focus on emotional and social aspects of illness
care plan includes:
edu
counseling
payment source identification
referrals
coping w/ stress and crisis intervention

Homemaker- Home Health Aide


Provide basic support services
medicare requires that a primary skilled or therapy service
(speech or physical) be provided before HHA services are
arranged

3 categories of aide services


1. personal
2. physical assistance
3. household chores

medicare / medicaid requirments of home health aide


onsite supervision every 2 weeks by RN
services provided in blocks of time ranging from 1-2 hours

Home Health Process


1. referral
2. admission
3. care plan
4. visits
5. documentation
6. discharge planning (by case manager)

Referral

comes from patient, family, social service, hospital,


physician, other agency

Admission
RN makes the initial eval and admission vist within 24-48
hours of the referreal
1. evaluation
2. enviromental assess.
3. i.d. of impairments
4. i.d. of impact of disease/ disability
5. assess support system
6. determine knowledge/adherence of treatment/ meds
7. involvment of patient/ fam in care plan
8. determin desire for care/ services
9. notify patient of rights
10. explain rights of self determination
11. initial nursing interventions (every 60 days)

Care Plan
from physician
-describes current physical status, meds, treatments,
disciplines, duration, goals, outcomes, time frame

Visits
to serve / meet patients- centered goals

Documentation
concise and complete doc. to provide accurate info of type
and quality of care
linked to legal implications

Discharge planning
done by case manager
begins with admission and ends when
patient goals or other specific criteria are met

Quality Assurance, Assessment, & Improvement


provide doc for evaluating specific criteria dn measures in
each area & evaluate them for compliance and
effectiveness
-reflect standards, objectives, measureable outcomes,
include plans for remediation or improvement
1. structural criteria
2. process criteria
3. outcome criteria

Structural Criteria
agencies overall practices

Process criteria
eval of care delivery

Outcome criteria
Measurement of change

Medicare
agency required to be certified
meet federal conditions of :
organization
staffing
training
services covered
agency eval
65 OR OLDER, DISABLED, END STAGE RENAL DISEASE

Medicaid
pays for home care services for LOW INCOME PPL OF ALL
AGES
state administers, federal subsidized

Third Party
pay for limited home care services
payment rates vary
posthospitalization recoveries tied to reimbursment

Private Pay
directly pay for home health services
charges range

HMOs
prefered provider organization
prepaid
based on prevention

PPOs
negotiated contracts with home health agencies

Nursing Process of LVN in home helth care

Plan care for patients based on needs


Review patients care plan / recommend provisions
Review / follow defined prioritization of patient care
Use clinical pathways, care maps, care plans to guide/
review patient care

NURSING PROCESS
assessment
analysis/ nursing diagnosis
planning
implementing
evaluation

Take aways
- current trends support growth of home care as an
economic, humane, preferred health delivery system
-medical management and control rather than cure are the
standards of care for illness
-home care provides assessment and eval of chronic
illnesses and is helpful for preventation in future
-aide and homemakers can provide necessary support in
ADLs to enable patient to remain @ home
-skilled nursing and therapy offer rehab and prevention of
deterioration, and methods to cope with physical changes

Long-Term care
defined by ANA as provison of physical, psychological,
spiritual, social, economic services to help ppl attain,
maintain, regain their optimum level of functioning
-range of services:
health maintenance care, to ppl who have lost ability to
function independantly due to chronic illness or condition

need for long term care


acute stage of an illness has resolved, patient continues to
need services to support and maintain physical and
psychological status and functional abilities

Patient-centered approach
achieve/ maintain an individualized plan of carre to assist
patient in preserving meaningful quality of life

quality of life
measure of optimum energy/ force that endows a person
with the power to cope successfully with full range of
challenges he or she encounters in the real world

Hospice
services to patients/families as end of life approaches.
available to any age-group
-maintain comfort as death approaches

pallative care
broader pop. having ability to benefit from comfort care
earlier in their illness or disease process.
provides care for
-basic needs (ADLs)
-ADLs
-pain and symptoms management
-spiritual/ psychosocial support

Adult DayCare
community-based programs to meet needs of functionally
or cognitibely impaired through supervised health and
social/ recreational activities
provide:
-physical care
-mental stimulation
-socialization
-assistance w/ maintanance
-health referrals
(during any part of the day, but for less than 24 hours)

Assisted Living
residential care
rental of small apartment
serveral personal care services:
-bathing, dressing, medications
(choice, autonomy, independance w/ supervision)
-communal dining and various social activities

HOME
least restrictive

INSTITUTIONAL
most restrictive

Institutional settings
1. subacute unit
2. long-term care facility

subacute unit
not limited by reimbursment
less expensive alt. to acute care when patient has high
acuity medical and nursing intervention needs
-bridge between acute care and long-term care
-located in free standing skilled nursing facilities

Long-Term care facility


dominant form
-24-hour care to individuals who do not require expensive
inpatient hospital services but who do not have options for
care at home or by other community agencies
-long or short term basis
-resident rather than patient
-most residents have more than one health disorder

short term resident


6 months

long term resident


duration of life

long-term interdisciplinary care team


professionals work togethre as an interdisciplinary team to
meet the needs of older adults

restorative nursing care


basic concepts of physical therapy for maintenance of
functional mobility and physical activity

omnibus budget reconciliation Act (OBRA)


nursing home reform legislation
defines requirements for the quality of care given to
residents

Health Care Financing Administration (HCFA)


administers and monitors the OBRA guidelines through
institutional surveys

Medicare
federally funded national health insurance program in US
for ppl older than age 65

Applications of the Omnibus budget reconciliation act of


1987
1. resident rights - fully informed resident / participates in
own health care plan
2. physical restraints - no restraints for discipline/
convinence
3. resident assessment - assessment is foundation of
planning and delivery of care
4. licensed nursing services - 24-hours a day
5. registered nurses - 8 hrs a day 7 days a week
6. nursing assistants - trained and competency tested

Ethical Issues for Long term care


-adherence to patients bill of rights
- advance directives
-DNR orders
-Power of attorney
-guardianship
-responsible party designation

Functional Nursing
each has a function adn reports back to head nurse

Team nursing
all work together toward common goal

residential assessment instrument (RAI)


three parts
1. MDS
2. RAPs
3. Utilization guidelines

Minimum Data Set (MDS)


provides a system for assessment of each residents
functional, medical, mental, psychosocial status

Resident Assessment Protocol (RAPs)


assessment guides that adress common clinical problems
-delirium, falls, urinary incontinence

Utilization Guidelines
wealth of clinical info to assist in assessment and care
planning

OBRA guideline for functional assessment docs

diff from acute care setting


vital signs/ weights required monthly

Nursing process for LVN in long term care


1. assessment -review plan of care every 90 days
2. Nursing diagnosis- id from assessment, prioritieze risk
3. Expected Outcomes/ planning
4. evaluation
autonomy
individuals select those actions that fulfill their goals,
clients decision

nonmaleficience
no harm is done when applying standards of care

beneficience
maximize possible benefits and minimize possible harms

distributive justice
fair distribution of the benefits and burden in society based
on the needs and contributions of its members

epidemiology
investigative study of disease trends in populations for the
purposes of disease prevention and health maintenance,
relies on statistics

agent
animate or inanimate object that causes the disease (virus,
trauma, fumes)

host
the living being that will be affected by the agent

environment
setting or surrounding that sustains the host

incidence
number of new cases

prevalence
existing disease in population at a particular time

infant mortality rate


number of infant deaths before 1 year of age in a year

attack rate
number of people at risk who develop certain disease

epidemic
the rate of disease exceeds the usual level of the condition

behavioral theory
focus on changing behavior through the use of
reinforcement menthods

cognitive theory
focus on changing thought patters through sensory input
and repetition

critical theory
focus on increasing depth of knowledge through discussion
and inquiry

developmental theory
focus on human developmental stage with methods that
are age-specific and age appropriate with importance given
to "readiness to learn"

humanistic theory
focus on feelings and relationships, principle that learners
will do what is in their best interest

social learning theory


focus on changing the learners expectations and beliefs
through methods that link information to beliefs and values

primary prevention
prevent occurrence, immunizations, classes

secondary prevention
early detection and treatment, screenings, treat STD

tertiary prevention
recovery after occurrence, rehab, shelter

airborne illnesses
measles, chickenpox, strep, TB, pneumonia, flu

foodborne illnesses
salmonella, Hep A, trich, E coli

waterborne illness (fecal contamination of water)


cholera, typhoid fever, giarda lamblia

vector borne -mosquito/tick illnesses


lyme disease, rocky mountain spotted fever, malaria

direct contact illnesses


STD's, mono, impetigo, lice, scabies

change agent
advocate for needed change at the local, state, or federal
level

lobbyists
persuade or influence legislators

coalitions
facilitation of goal achievement through the collaboration
of two or more groups

public office
serving society and advocating for change by influencing
policy development through public service

community based nursing


health of individuals, and groups within a community

community oriented nursing


health of the community as a whole

population focused nursing


assessments/ interventions for at risk populations

culture
beliefs, values, and assumptions about life that are widely
held among a group of people and are transmitted across
generations

environmental health
influence of environmental conditions on the development
of disease or injury

acculturation
process of learning a new culture

cultural awareness
self awareness of one's own cultural background, biases,
and differences

cultural competence
knowing, appreciating, and considering the culture of
someone else in resolving problems

WHO
World health organization, provides daily information
regarding the occurrence of internationally important
diseases, establishes world standards for antibiotics and
vaccines, focus on health care workforce and education,
environment, sanitation, infectious diseases, maternal and
child health, and primary care

Federal Health Agencies


under direction of secretary of health, funded through
federal taxes, Medicare, Medicaid, CDC, FDA

State Health Departments


WIC, handle licensing of nurses and nursing schools,
funded through state taxes and federal funding

Local Health Department


focus on health of its citizens, funded through local taxes,
state funds, and federal funds

informant interviews
direct discussion with community members for the purpose
of obtaining ideas and opinions from key informants

community forum
open public meeting

secondary data

use of existing data to assess problem

windshield survey
descriptive approach that assesses several community
components by driving through a community

focus groups
directed talk with a representative sample

surveys
specific questions asked in a written format

participant observation
observation of formal or informal community activities
A nurse manager at a community agency is developing
an orientation program for newly hired nurses. When
discussing the differences between community-based and
community-oriented nursing, the nurse should include
which of the following as examples of community-based
nursing? (Select all that apply.)
a. A home health nurse performing wound care for a
client who is immobile
b. An occupational health nurse providing classes on body
mechanics at a local industrial plant
c. A school nurse teaching a student who has asthma
about medications
d. A parish nurse teaching a class on low-sodium cooking
techniques
e. A mental health nurse discussing stress management
techniques with a support group
a, c

A nurse is advocating for local leaders to place a newly


approved community health clinic in an area of the city
that has fewer resources than other areas. The nurse is
advocating for the leaders to uphold which of the
following ethical principles?
a. Distributive justice
b. Fidelity
c. Respect for autonomy
d. Veracity
a

A nurse is preparing an education program on disease


transmission for employees at a local day care facility.
When discussing the epidemiological triangle, the nurse
should include which of the following as agents? (Select
all that apply.)
a. Resource availability
b. Ethnicity
c. Toxins
d. Bacteria
e. Altered immunity
c, d

A nurse is developing a community health education


program for a group of clients who have a new diagnosis
of diabetes mellitus. Which of the following learning
strategies should the nurse include for clients who are
auditory learners?
a. Showing informational videos
b. Providing equipment to practice hands-on skills
c. Supplying outlines for note-taking
d. Facilitating small group discussions
d

A community health nurse is implementing health


programs with several populations in the local area. In
which of the following situations is the nurse using
primary prevention?
a. Performing a home safety check at a client's home
b. Teaching healthy nutrition to clients who have
hypertension
c. Providing influenza immunizations to employees at a
local preschool
d. Implementing a program to notify individuals exposed
to a communicable disease
c

A nurse is preparing an educational program on cultural


perspectives in nursing. The nurse should include that
which of the following are influenced by an individual's
culture? (Select all that apply.)
a. Nutritional practices
b. Family structure
c. Health care interactions

d. Biological variations
e. Views about illness
a, b, c, e

A nurse is caring for a client who is from a different


culture than himself. When beginning the cultural
assessment, which of the following actions should the
nurse take first?
a. Determine the client's perception of his current health
status.
b. Gather data about the client's cultural beliefs.
c. Determine how the client's culture may impact the
effectiveness of nursing actions.
d. Gather information about previous client interactions
with the health care system.
b

A nurse is using the I PREPARE mnemonic to assess a


client's potential environmental exposures. Which of the
following is an appropriate question for the nurse to ask
to assess for "A" in the mnemonic?
a. "What do you like to do for fun?"
b. "What year was your residence built?"
c. "What jobs have you had in the past?"
d. "What industries are near where you live?"
a

A nurse is conducting health screenings at a statewide


health fair and identifies several clients who require
referral to a provider. Which of the following statements
by a client indicates a barrier to accessing health care?
a. "I don't drive, and my son is only available to take me
places in the mornings."
b. "I can't take off during the day and the local after-hours
clinic is no longer in operation."
c. "Only one doctor in my town is a designated provider
by my health maintenance organization."
d. "I would like to schedule an appointment with the local
doctor in my town who speaks Spanish and English."
b

A nurse is interviewing for a position at the local health


department. When preparing for the interview, the nurse

should find that which of the following are responsibilities


of this agency? (Select all that apply.)
a. Managing the Women, Infants, and Children program
b. Providing education to achieve community health goals
c. Coordinating directives from state personnel
d. Reporting communicable diseases to the CDC
e. Licensing of registered nurses
b, c

A nurse is preparing to conduct a windshield survey.


Which of the following data should the nurse collect as a
component of this assessment? (Select all that apply.)
a. Ethnicity of community members
b. Individuals who hold power within the community
c. Natural community boundaries
d. Prevalence of disease
e. Presence of public protection
a, c, e

A nurse is completing a needs assessment and beginning


analysis of data. Which of the following actions should the
nurse take first?
a. Determine health patterns within collected data.
b. Compile collected data into a database.
c. Ensure data collection is complete.
d. Identify health needs of the local community.
b

A nurse is planning a community health program. Which


of the following should the nurse include as part of the
evaluation plan?
a. Determine availability of resources to initiate the plan.
b. Gain approval for the program from local leaders.
c. Establish a timeline for implementation of
interventions.
d. Compare program impact to similar programs.
d

A nurse is conducting a community assessment. Which of


the following data collection methods is the nurse using
when having direct conversations with individual
members of the community?

a. Key informant interviews


b. Participant observation
c. Focus groups
d. Health surveys
a

A nurse is collecting data to identify health needs in the


local community. Which of the following are examples of
secondary data the nurse should review? (Select all that
apply.)
a. Birth statistics
b. Previous health survey results
c. Windshield survey
d. Community forum
e. Health records
a, b, e

A nurse is talking to a client who asks for additional


information about hospice. Which of the following is an
appropriate statement by the nurse?
a. "Clients who require skilled nursing care at home
qualify for hospice care."
b. "One function of hospice is to provide teaching to
clients about life-sustaining measures."
c. "Hospice assists clients to develop the skills needed to
care for themselves independently."
d. "A component of hospice care is to control the client's
symptoms."
d

An occupational health nurse is consulting with senior


management of a local industrial facility. When discussing
work-related illness and injury, the nurse should include
which of the following as physical agents? (Select all that
apply.)
a. Noise
b. Age
c. Lighting
d. Viruses
e. Stress
a,c

A newly hired occupational health nurse at an industrial


facility is performing an initial workplace assessment.
Which of the following should the nurse determine when
conducting a work site survey?
a. Work practices of employees
b. Past exposure to specific agents
c. Past jobs of individual employees
d. Length of time working in current role
a

A school nurse is scheduling visits with a physical


therapist for a child who has cerebral palsy. In which of
the following roles is the nurse functioning?
a. Direct caregiver
b. Consultant
c. Case manager
d. Counselor
c

A school nurse is planning health promotion and disease


prevention activities for the upcoming school year. In
which of the following situations is the nurse planning a
secondary prevention strategy?
a. Placing posters with images of appropriate hand
hygiene near restrooms
b. Routinely checking students for pediculosis throughout
the school year
c. Implementing age-appropriate injury prevention
programs for each grade level
d. Working with a dietician to determine carbohydrate
counts for students who have diabetes mellitus
b

A nurse at a community clinic is conducting a well-child


visit with a preschool-age child. The nurse should identify
which of the following as a manifestation of child neglect?
(Select all that apply.)
a. Underweight
b. Healing spiral fracture of the arm
c. Genital irritation
d. Burns on the palms of the hands
e. Poor hygiene
a, e

A nurse is caring for a client who is experiencing alcohol


withdrawal. Which of the following findings is a
manifestation of withdrawal?
a. Decreased blood pressure
b. Diaphoresis
c. Pin-point pupils
d. Bradycardia
b

A community health nurse is developing an education


program on substance use disorders for a group of
adolescents. Which of the following should the nurse
include when discussing nicotine and smoking?
a. Smoking is the fifth-most preventable cause of death in
the United States.
b. Nicotine is a central nervous system depressant.
c. Withdrawal effects from smoking are minimal.
d. Tolerance to nicotine develops quickly.
d

A community health nurse is developing strategies to


prevent or improve mental health issues in the local area.
In which of the following situations is the nurse
implementing a tertiary prevention strategy?
a. Providing support programs for new parents
b. Screening a client whose spouse recently died for
suicide risk
c. Teaching a client who has schizophrenia about
medication interactions
d. Discussing stress reduction techniques with employees
at an industrial site
c

A nurse at an urban community health agency is


developing an education program for city leaders about
homelessness. Which of the following should the nurse
include as the fastest-growing segment of the homeless
population?
a. Families with children
b. Adolescent runaways

c. Intimate partner abuse victims


d. Older adults
a

A nurse is preparing a community health program on


communicable diseases. When discussing modes of
transmission, the nurse should include which of the
following as an airborne illness?
a. Cholera
b. Malaria
c. Influenza
d. Salmonellosis
c

A home health nurse is discussing portals of entry with a


group of newly hired assistive personnel. Which of the
following are portals of entry the nurse should discuss?
(Select all that apply.)
a. Respiratory secretions
b. Skin
c. Genitourinary tract
d. Saliva
e. Mucous membranes
b, c, e

A newly hired public health nurse is familiarizing himself


with the levels of disaster management. Which of the
following actions is a component of disaster prevention?
a. Outlining specific roles of community agencies
b. Identifying community vulnerabilities
c. Prioritizing care of individuals
d. Providing stress counseling
b

A community health nurse is educating the public on the


agents of bioterrorism. Which of the following are
Category A biological agents? (Select all that apply.)
a. Hantavirus
b. Typhus
c. Plague
d. Tularemia
e. Botulism

c, d, e

A community health nurse is determining available and


needed supplies in the event of a bioterrorism attack. The
nurse should be aware that community members exposed
to anthrax will need access to which of the following
medications?
a. Metronidazole (Flagyl)
b. Ciprofloxacin (Cipro)
c. Zanamivir (Relenza)
d. Fluconazole (Diflucan)
b

A nurse is creating partnerships to address health needs


within the community. The nurse should be aware that
which of the following characteristics must exist for
partnerships to be successful? (Select all that apply.)
a. A leading partner with decision-making authority
b. Flexibility among partners when considering new ideas
c. Adherence of partners to ethical principles
d. Varying goals for the different partners
e. Willingness of partners to negotiate roles
b, c, e

A nurse is reviewing the various roles of a community


health nurse. Which of the following is an example of a
nurse functioning as a consultant?
a. Advocating for federal funding of local health screening
programs
b. Updating state officials about health needs of the local
community
c. Facilitating discussion of a client's ongoing needs with
an interprofessional team
d. Performing health screenings for high blood pressure at
a local health fair
b

A case management nurse at an acute care facility is


conducting an initial visit with a client to identify needs
prior to discharge home. After developing a working
relationship with the client, the nurse is engaging in the
referral process. Which of the following should be the first
action by the nurse?

a. Monitor the client's satisfaction with the referral.


b. Provide client information to referral agencies.
c. Review available resources with the client.
d. Identify referrals that the client needs.
d

A nurse developing a community health program is


determining barriers to community resource referrals.
Which of the following is an example of a resource
barrier?
a. Costs associated with services
b. Decreased motivation
c. Inadequate knowledge of resources
d. Lack of transportation
a

A nurse is working with a client who has systemic lupus


erythematosus and recently lost her health insurance.
Which of the following is an appropriate action by the
nurse in the implementation phase of the case
management process?
a. Coordinating services to meet the client's needs
b. Comparing outcomes with original goals
c. Determining the client's financial constraints
d. Clarifying roles of interprofessional team members
a
One of the primary focuses of improving the health of the
American people in the twenty-first century is to address:
a.Bioterrorism and global health threats
b.Delivery of individual care and hygiene
c.The need for increased hospital and acute care
d.Chronic disease and disability management
a. Bioterrorism and global health threats

The public health nurse must participate in the essential


services of public health. These include:
a.Monitoring health status by completing a community
assessment
b.Diagnosing and investigating health problems in the
world
c.Informing, educating, and empowering people about
health issues

d.Working in law enforcement to regulate health and


ensure safety
c. Informing, educating, and empowering people about
health issues

Which of the following is an example of the mission of


public health according to the Institute of Medicine?
a.Tracking avian flu outbreaks and doing surveillance in
the United States
b.Providing a flu shot for an elderly person at the health
department
c.Keeping track of alternative therapies in use in the
United States
d.Keeping snake antivenom at the Centers for Disease
Control and Prevention in Atlanta
a. Tracking avian flu outbreaks and doing surveillance in
the United States

Making sure that essential community-oriented health


services are available defines which of the core public
health functions?
a.Policy development
b.Assessment
c.Assurance
d.Scientific knowledge-based care
c. Assurance

Which of the following statements about public health is


accurate?
a.Prevention of early deaths can be more effectively
accomplished by medical treatment than by public health
approaches.
b.Expenditures and resources for public health have
increased in recent years.
c.Historically, gains in the health of populations have
been related largely to changes in safety, sanitation, and
personal behavior.
d.Reform of the medical insurance system is the single
change needed to improve the health of Americans.
c. Historically, gains in the health of populations have been
related largely to changes in safety, sanitation, and
personal behavior.

Collecting data and monitoring the health status of the


population defines which of the core public health
functions?

a.Assessment
b.Prevention
c.Assurance
d.Policy development
a. Assessment

Public health nurses who develop and implement local


public health policies through partnerships with agencies,
organizations, and consumers within the community are
using which core public health function?
a.Assessment
b.Prevention
c.Assurance
d.Policy development
d. Policy development

Providing for the availability of essential personal health


services for people who would otherwise not receive
health care defines which public health core function?
a.Assessment
b.Prevention
c.Assurance
d.Policy development
c. Assurance

Which is an example of the primary goal of public health?


a.Ensuring that a newly diagnosed 40-year-old
hypertensive man takes his medication
b.Finding home care for a 70-year-old client recuperating
from a hip replacement
c.Conducting an infant car seat safety check
d.Contacting a local hospice to admit a terminally ill 60year-old woman
c. Conducting an infant car seat safety check

The public health workforce should demonstrate


competency in which of the following competency
categories?
a.Financial planning and management
b.Workforce needs assessment
c.Acute care services
d.Curriculum development
a. Financial planning and management

Public health nursing is a specialty because:


a.It has a distinct focus and scope of practice.

b.It must be done by a registered nurse with a master's


degree.
c.It is focused on disadvantaged citizens.
d.It performs interventions at the acute care level.
a. It has a distinct focus and scope of practice.

A public health nurse provides a clinic for HIV-positive


citizens in the community. This is an example of:
a.Primary prevention
b.Secondary prevention
c.Tertiary prevention
d.Policy making
c. Tertiary prevention

Which statement about community health nursing


practice is correct?
a.It focuses on the delivery of personal health services to
individuals and families.
b.It provides care to protect the health of the community
as a whole.
c.It emphasizes the setting where care is provided for
clients and families.
d.It requires a baccalaureate preparation for practice.
a. It focuses on the delivery of personal health services to
individuals and families.

The nurse who conducts vision screenings on children in


the school setting is practicing _____ nursing practice.
a.Community-oriented
b.Public health
c.Community health
d.Community-based
d. Community-based

Which is an example of an aggregate or population?


a.Students in a county school system
b.Christians around the world
c.A patient in the intensive care unit at the local hospital
d.People who play cards together once a week at private
homes
a. Students in a county school system

A population is best defined as a:


a.High-risk group
b.School or institutional setting
c.Collection of individuals who share at least one common

characteristic
d.Geographical location within a community
c. Collection of individuals who share at least one common
characteristic

Population-focused practice focuses on defining the


problems or needs of and implementing solutions for:
a.Individuals
b.Aggregates
c.Communities
d.Geographical regions
b. Aggregates

Which of the following statements is true with regard to a


population focus in public health nursing?
a.Priority is given to the highest risk population.
b.Direct caregiving is limited to preventive measures,
such as administration of immunizations.
c.Attention is given to the population or community as a
whole, regardless of whether they do or do not access the
health care system.
d.Only populations outside institutional settings are
considered.
c. Attention is given to the population or community as a
whole, regardless of whether they do or do not access the
health care system.

A key opportunity for public health nurses to improve


population-focused care is by:
a.Assuming traditional nursing roles
b.Influencing public health policy
c.Conducting community assessments
d.Specializing in community-based nursing practice
b. Influencing public health policy

Which of the following are considered barriers to public


health nursing? Select all that apply.
a.The mindset that the only role for the nurse is at the
bedside.
b.The structures within which nurses work and the
process of role socialization within those structures.
c.Few nurses receive graduate-level preparation in the
concepts and strategies of the disciplines basic to public
health.
d.The number of job opportunities in the area has been
steadily declining.

a. The mindset that the only role for the nurse is at the
bedside.
b. The structures within which nurses work and the process
of role socialization within those structures.
c. Few nurses receive graduate-level preparation in the
concepts and strategies of the disciplines basic to public
health.

The threat of bioterrorism has the potential to:


a.Dissolve community-based programs.
b.Cause the health care system to collapse.
c.Divert funds from other public safety health care
programs.
d.Increase the need for shelters.
c. Divert funds from other public safety health care
programs

Population-based preventive programs launched in the


1970s are responsible for increased:
a.Use of tobacco
b.Use of automobile safety restraints
c.Incidence of hypertension
d.Incidence of obesity
b. Use of automobile safety restraints

The number and proportion of persons aged 25 or older


with less than a high school education is an example of:
a.Sociodemographic characteristics
b.Health status
c.Health risk factors
d.Health care resource consumption
a. Sociodemographic characteristics

The purpose of public health core functions is to:


a. Clarify the role of the government in fulfilling the
mission of public health.
b. Ensure the safety of populations in receiving quality
health care.
c. Provide community-based individualized care to every
person in the United States.
d. Unite public and private providers of care in a
comprehensive approach to providing health care.
a. Clarify the role of the government in fulfilling the mission
of public health.

The nurse who compares the rate of teenage pregnancy


in various areas of the city is practicing the public health
core function of:
a.Assurance
b.Assessment
c.Prevention
d.Policy development
b. Assessment

The nurse manager who makes sure that the staff


members who work in a local clinic are competent is
demonstrating the public health core function of:
a.Assurance
b.Assessment
c.Prevention
d.Policy development
a. Assurance

The public health nurse analyzes data related to the


number and type of United States Environmental
Protection agency air quality standards that a community
failed to meet. This data is an example of using which
community health profile indicator?
a.Sociodemographic characteristics
b.Health status
c.Health risk factor
d.Functional status
c. Health risk factor

The necessary basic preparation for public health nursing


is a(n) _____ in nursing.
a.Associate's degree
b.Baccalaureate degree
c.Master's degree
d.PhD
b. Baccalaureate degree

Public health practitioners prepared at the graduate level


should be able to:
a.Teach public and community health nursing
b.Assess and intervene successfully at the aggregate
level
c.Diagnose and treat disease and have prescriptive
authority
d.Run for political office as experts in public health policy

b. Assess and intervene successfully at the aggregate level

An example of a community health nursing practice is:


a.Administrating a flu shot to a client in a physician's
office
b.Conducting a flu shot clinic at a community center
c.Performing a client assessment in a hospital
d.Providing supervision of staff in a rehabilitation center
a.Administrating a flu shot to a client in a physician's office

The term community-based nursing care is applied when


implementing which nursing intervention?
a. Assessing the health needs of a defined community
b. Providing care to families in a community
c.Promoting the health of an entire community
d.Investigating environmental health problems in a
community
b. Providing care to families in a community

The nurse investigating environmental health problems


caused by contaminated ground water is best described
as practicing:
a.Community-oriented nursing
b.Community-based nursing
c.Policy development
d.Tertiary care
a. Community-oriented nursing

A public health staff nurse who has a clear understanding


of population-focused practice:
a.Is frustrated providing care to individuals
b.Would rather perform population-focused interventions
c.Is able to improve the effectiveness of care provided
d.Is considered a public health nurse specialist
c. Is able to improve the effectiveness of care provided

Proposed changes to the health care delivery system in


the United States will:
a.Provide new opportunities for public health specialists.
b.Result in isolated care being provided to individuals.
c.Emphasize specialty care.
d.Increase the utilization of acute care services.
a. Provide new opportunities for public health specialists.

Implementing quality performance standards in public


health is important because:

a. They are used to guide improvement in the public


health system.
b. They rigidly control public health.
c. Administrators will not monitor public health at the
local level, but instead at the national level.
d. They can be used as hiring guidelines for nurses.
a. They are used to guide improvement in the public health
system.

To better address emerging public health issues, a public


health nurse enrolls in a course addressing which content
area?
a.Leadership
b.Ethics
c.Communication
d.Finance
b. Ethics
Community-Oriented Nursing
Focus: Health of the community as a whole.
Client: Community
Activity: Surveillance and evaluation of the community's
collective health, and the implementation of strategies to
promote health and prevent disease.

Community-Based Nursing
Focus: Health of individuals, families, and groups within a
community.
Client: Individuals, family, or group of individuals
Activity: Provision of direct primary care in the settings
where individuals and families live, work, and "attend"
(schools, camps, parishes)

Community Health Nursing


Systematic processes to deliver care to individuals,
families, or community groups with a focus on promoting,
preserving, protecting, and maintaining health care
provided contributes to the health of the community.

Public Health Nursing


Nursing and public health, disease and disability
prevention, promoting and protecting the health of the
entire community

Eight domains of public health

Distinguishes public health nursing from other nursing


specialties by adherence to the eight principles:
Analytical assessment skills
Policy development and program planning skills
Communication skills
Cultural competency skills
Community dimensions of practice skills
Basic public health science skills
Financial planning and management skills
Leadership and systems thinking skills

Three primary functions of public health


1. Assess health care function
2. Develop policy that provides access to services
3. Ensure services are delivered and outcomes achieved

Health People 2020: Four main goals


1. Eliminate preventable diseases, disability, injury, and
premature death
2. Achieve health equity, eliminate disparities, and improve
health of all age groups
3. Create social and physical environments that promote
good health for all
4. Promote healthy development and health behaviors
across every stage of life

Social determinants
Impact whether someone is able to attain/maintain health;
(Income, social status, education, literacy, environment,
support networks, gender, culture, available health care)

Disparities
Gaps in care experienced by one population compared to
another

Health care changes in the 21st century


Increased patient acuity and number of services in the
community.
Patient centered care
Increased patient responsibility for own health
Increased use of technology

Florence Nightingale
Studied nursing in Germany then established nursing
schools in England

District Nursing
A mode of service delivery in which a community health
nurse is responsible for addressing all the health needs of a
given population

Lemuel Shattuck
First use of demographic data to look at population health
birth and death records

Dorthea Dix
Established first hospital for the mentally ill in the US

Clara Barton
Civil war nurse: Established the American Red Cross
(providing aid for natural disasters)

Lillian Wald
Found of public health nursing in the US; Founded the
Henry Street Settlement and Visiting Nurse Service which
provided nursing and social services and organized
educational and cultural activities. She is considered the
founder of public health nursing

Mary Breckenridge
Pioneer in nurse-midwifery; established the Frontier Nursing
Service - nurses traveled on horseback to reach mothers in
the hills of Kentucky

World Health Organization


International center that collects data, advances initiatives,
and offers support related to public health

US Department of Health and Human Services (USDHHS)


US branch of government responsible for health and
welfare of citizens

Nongovernment Agencies
Agencies that acquire resources from private sources to
assist others

Philanthropic Agencies
Organization that uses endowments or private funding to
address the needs of individuals, families, and populations

Center for Disease Control


(CDC) is a United States federal agency under the
Department of Health and Human Services. It works to
protect public health and safety by providing information to
enhance health decisions, and it promotes health through
partnerships with state health departments and other
organizations.

Difference in US health care system and public health


The US has an individual focus on curative measures while
public health focuses on the population or community as a
whole and focuses on disease reduction and improved
health

Local Level of Government


Implement public health activities within the community;
Governed by the state department;
Structure varies based on community needs

State Level of Government


Focuses on regulation of the community of program
responsibility and resource allocation

Federal Level of Government


Focuses on policy development and financing

Health
A quality, an ability to adapt to change or a resource to
help cope with challenges and processes of daily living

Well Being
A subjective perception of full functional ability as a human
being

USDHAHS 10 Components of public health practice


1. Preventing Epidemics
2. Protecting the environment, workplaces, and sources of
food and water
3. Promoting health behavior
4. Monitoring the health status of the population
5. Mobilizing community resources into service
6. Responding to disasters
7. Assuring the public there are trained personnel
8. Reaching out to those at high risk

9. Researching risk, disease acquisition, and ways to


prevent injury
10. Influencing policy to acquire resources to effect change

Epidemiological Health Promotion Model


The study of relationships among an agent, a host, and an
environment. The interaction determines the development,
and cessation of communicable disease.

Learning Theory
Goal established and reinforced by nurse; Rewards given
for partial accomplishment

Health Belief Model


Cues used to remind people of health behavior and
promote action; Perceived risk influences action

Transtheoretical Health Promotion Model


5-Step approach: Pre-contemplation, Contemplation,
Preparation, Action, Maintenance

Reasoned Action Health Promotion Model


Performance of a behavior is determined by a persons
intention to perform the behavior; Intentioned determined
by attitude and behavior

Social Support Health Promotion Model


Change influenced by support of friends, families, and
communities

Primary Level of Prevention


Utilize prevention strategies, address modifiable risk
factors, and maximize health and wellness

Secondary Level of Prevention


Planned effort to minimize the impact of disease and injury
once it has occurred

Tertiary Level of Prevention


Maximizing health and wellness through strategies set in
place at the end-stage of disease and injury

Epidemiology

The study of the distribution and determinants of health


and illness

Rate
Primary measurement used to describe either the
occurrance or the existence of a specific state of health or
illness

Outbreaks
Epidemic usually limited to a localized increase in the
incidence of the illness

John Graunt
Father of demographics. recognized importance of
recording birth and death rates and age structure of human
population

William Farr
Set up a system of data collection for causes of death in
difference occupations, gender, and imprisonment.
Importance of definition of illness and population
comparison, groups, and factors such as age, health, and
environment can affect statistics

John Snow
Used mapping and rates as an objective measure to
compare populations

Epidemiological Triad
Model based on the belief that health status is determined
by the interaction of the characteristics of the host, agent,
and environment

Wheel of Causation
Epidemiologic model that deemphasizes the agent as the
sole cause of disease while emphasizing the interplay of
physical, biological, and social environments

Web of Causation
Epidemioligcal model that strongly emphasizes the concept
of multiple causation while deemphasizing the role of
agents in explaining illness

Natural History of Disease

Course of disease or condition from the onset to resolution

Rate
Primary measure used to describe either the occurrence or
existence of a specific state of health or illness within a
group during a specific time frame

Ratio
Fraction that represents the relationship between two
numbers; Divide one quantity by another quantity
Ex: # boys in class/# girls in class

Proportion
Type of ration
Ex: # boys/ total students

Crude Rate
Measures the occurrence of the condition in the whole
population; May obscure info because it does not consider
factors such as age, race, gender
Numerator: Number of events
Denominator: Total population (not just those at risk)

Adjusted Rate
Controls for differences between populations-used for
comparison

Incidence
Measures probability that people without a condition will
develop the condition over time; measures pace of new
illness

Prevalence
Existence of a condition during a period/interval at a
specific point; Studies number of people diagnosed in the
past and length of illness; Longer length=greater the
prevalence-indicates burned of community

Mortality Rates
Crude mortality: probability of dying from any cause; #
deaths in a year/total population

Proportional Mortality Ratio

Compares death from a specific illness with deaths from all


causes; # deaths from specific cause within a time
frame/total deaths

Incidence Density
Used when unequal periods of observation for study
subjects; Accounts for people who die, drop out of a study,
or acquire an illness

Attributable Risk
The risk of a condition occurring in an exposed group that
is attributable to an exposure, not other factors

Relative Risk Ratio


Measure of the risk of developing a condition; Ratio of
incidence in exposed and incidence in non-exposed

Sensitivity
Ability of the test to correctly identify people who have the
health problem

Specificity
Specific incidence and prevalence rates calculated based
on specific characteristics (demographic data), variations
based on location and variations in time (short-outbreak,
periodic- seasonal, long-years)

Crude Rate Formula


# events within time period/population at risk with same
time pt

Incidence Rate
#new cases during time period/population at risk in the
same time period

Causality
Strengths of Association
Consistency
Temporality
Plausibility
Biological Gradient

Case Studies

In-depth analysis of individual or group, often first clue to a


problem

Cohort Studies
(Longitudinal Studies) Monitor over time to find
associations between risk factors and health;
Minimize selection bias;
Relative risk is the ratio of disease incidence in an exposed
population;
Indicates strength of exposure to illness causality

Case Control
Compare group with health problems (cases) to group
without (control)

Key Informants
Persons knowledgeable about specific aspects of a problem
and the communities current and past attempts to address
it

Stakeholders
An individual, organizational, or group that has interest
(stake) in a specific community health issue or the outcome
of a community level intervention

Gantt Chart
Visual of the sequence of steps to achieve objectives;
Developed in planning stages to identify steps, a tool for
scheduling tasks, monitor progress

Community of Interest
People and groups/aggregate that will be affected by
change; Those that will help bring about change

Upstream Approach to Health Care


Changes at societal level may impact health of community
without change is individual behavior

Sustainability
How to maintain change, support system in place

Social Justice
Health care is a risk; Address root cause of illness

Health Equities
Requires elimination of health disparities in living and
working conditions

SMART objectives
S- Specific
M- Measurable
A- Achievable
R- Relevant
T- Time

Coalition Building
Community members participate in planning and
implementing changes at the community level; Need clear
mission, goals, objectives, expectations, leadership,
accountability, and should be heterogenous

Community Readiness
Assess readiness to undertake change process-issue
specific

Web of Causation
Identify multiple factors that contribute to chronic disease

Lewins Model for Change


Unfreezing:
Changing:
Freezing:

Levers of Change
Increase driving/reinforcing forces
Decrease restraining/resisting forces

Force Field Analysis


Identify factors within a community that impact change and
forces that cannot be changes

Geopolitical Population
Has identified designated boundaries with the same
governing structure

Phenomenological Population
Group with common interests or beliefs who have
interpersonal and intrapersonal connection

Community of Solution
Formed by aggregate to address a health problem
The factors, exposures, characteristics, and behaviors
that determine patterns of disease are described using:
a. Descriptive epidemiology
b. Analytic epidemiology
c. Distribution
d. Determinants
d. Determinants

To understand the causes of health and disease,


epidemiology studies:
a. Individuals
b. Families
c. Groups
d. Populations
d. Populations

When a nurse examines birth and death certificates


during an epidemiologic investigation, what data category
is being used?
a. Routinely collected data
b. Data collected for other purposes but useful for
epidemiologic research
c. Original data collected for specific epidemiologic
studies
d. Surveillance data
a. Routinely collected data

An epidemiologist wanting to know what caused severe


diarrhea and vomiting in several people at a local
banquet would be using:
a. Descriptive epidemiology
b. Analytic epidemiology
c. Distribution
d. Determinants
b. Analytic epidemiology

Which is an example of an epidemic?


a. "Bird" flu in China
b. Adult obesity in the United States
c. An isolated case of smallpox in Africa
d. The nursing shortage in the United States
b. Adult obesity in the United States

John Snow is called the "father of epidemiology" because


of his work with:
a. Cholera
b. Malaria
c. Polio
d. Germ theory
a. Cholera

The interaction between an agent, a host, and the


environment is called:
a. Natural history of disease
b. Risk
c. Web of causality
d. The epidemiologic triangle
d. The epidemiologic triangle

Public health professionals refer to three levels of


prevention as tied to specific stages in the:
a. Epidemiologic triangle
b. Web of causation
c. Natural history of disease
d. Surveillance process
c. Natural history of disease

Which is an example of an agent in the epidemiologic


triangle?
a. Human population distribution
b. Salmonella
c. Genetic susceptibility
d. Climate
b. Salmonella

When studying chronic disease, the multifactorial etiology


of illness is considered. What does this imply?
a. Genetics and molecular structure of disease is
paramount.
b. Single organisms that cause a disease, such as cholera,
must be studied in more detail.
c. Focus should be on the factors or combinations and
levels of factors contributing to disease.
d. The recent rise in infectious disease is the main focus.
c. Focus should be on the factors or combinations and
levels of factors contributing to disease.

Immunization for measles is an example of:


a. Primary prevention
b. Secondary prevention
c. Tertiary prevention
d. Health promotion
a. Primary prevention

Screening for hearing defects is an example of:


a. Primary prevention
b. Secondary prevention
c. Tertiary prevention
d. Health promotion
b. Secondary prevention

Vocational rehabilitation of a person with a neuromuscular


disease is an example of:
a. Primary prevention
b. Secondary prevention
c. Tertiary prevention
d. Health promotion
c. Tertiary prevention

An example of primary prevention is:


a. Pap smear
b. Blood pressure screening
c. Diet and exercise
d. Physical therapy
c. Diet and exercise

An example of secondary prevention is:


a. Rehabilitation
b. Avoidance of high-risk behaviors
c. Immunization
d. Mammogram
d. Mammogram

An example of tertiary prevention is:


a. Rehabilitative job training
b. Parenting education
c. Testicular self-examination
d. Family counseling
a. Rehabilitative job training

The probability an event will occur within a specified


period of time is called:
a. Rate

b. Risk
c. Epidemiology
d. Epidemic
b. Risk

Which statement is true about mortality rates? Mortality


rates:
a. Are informative only for fatal diseases
b. Provide information about existing disease in the
population
c. Are calculated using a population estimate at year-end
d. Reveal the risk of getting a particular disease
a. Are informative only for fatal diseases

A screening for diabetes revealed 20 previously


diagnosed diabetics and 10 probable new cases, which
were later confirmed, for a total of 30 cases. This is
called:
a. Prevalence
b. Incidence
c. Attack
d. Morbidity
a. Prevalence

An example of an attack rate is the:


a. Number of cases of cancer recorded at a medical
center
b. Number of people who died of cholera in India in a
given year
c. Number of beef cattle inoculated against mad-cow
disease on a farm
d. Proportion of people becoming ill after eating at a fastfood restaurant
d. Proportion of people becoming ill after eating at a fastfood restaurant

Which would be considered a serious epidemic of


influenza?
a. 50 cases
b. 100 cases
c. 500 cases
d. Unable to determine
d. Unable to determine

The most important predictor of overall mortality is:


a. Race

b. Age
c. Gender
d. Income
b. Age

An outbreak of a gastrointestinal illness from a food-borne


pathogen is an example of a(n):
a. Attack rate
b. Point epidemic
c. Secular trend
d. Event-related cluster
b. Point epidemic

A study that uses information on current health status,


personal characteristics, and potential risk factors or
exposures all at once is called:
a. Cross-sectional
b. Ecological
c. Case-control
d. Cohort
a. Cross-sectional

The type of epidemiologic study that is used to describe a


group of persons enrolled in a study who share some
characteristic of interest and who are followed over a
period of time to observe some health outcome is a(n):
a. Case control study
b. Cross-sectional study
c. Cohort study
d. Experimental study
c. Cohort study

Voters have recently decided to have fluoride added to


the city water system. Epidemiologists wanting to study
the effect of fluoride on dental caries would be
conducting a(n):
a. Ecological study
b. Double-blind study
c. Community trial
d. Screening
c. Community trial

The proportion of persons with positive test results who


actually have a disease, interpreted as the probability
that an individual with a positive test result has the
disease, is the:

a. Sensitivity
b. Specificity
c. Positive predictive value
d. Negative predictive value
c. Positive predictive value

A nurse is told that a screening test has high specificity.


This means that the test:
a. Provides precise and consistent readings
b. Accurately identifies those with the condition or trait
c. Accurately identifies those without the trait
d. Has a high level of false positives
c. Accurately identifies those without the trait

In which situation does the nurse need to be aware of the


potential for selection bias?
a. Determining the population to be studied
b. Considering how the participants will enter the study
c. Studying cause and effect relationships
d. Documenting results of the study
b. Considering how the participants will enter the study

As a result of an outbreak of influenza in a community, a


nurse encourages members of the community to receive
the influenza vaccine. Which level of prevention is being
used?
a. Primary prevention
b. Secondary prevention
c. Tertiary prevention
d. Multifactorial prevention
a. Primary prevention

Epidemiologic studies of diseases conducted by nurses


during the twentieth century were influenced by the
(select all that apply):
a. Increasing rate of poverty
b. Declining child mortality rates
c. Overcrowding in major cities
d. Development of new vaccinations
b. Declining child mortality rates
d. Development of new vaccinations

Nurses incorporate epidemiology into their practice and


function in epidemiologic roles through (select all that
apply):

a. Policy making and enforcement


b. Collection, reporting, analysis, and interpretation of
data
c. Environmental risk communication
d. Documentation on patient charts and records
b. Collection, reporting, analysis, and interpretation of data
c. Environmental risk communication
d. Documentation on patient charts and records
community
a group of people sharing common interests, needs,
resources, and environment; an interrelating and
interacting group of people with shared needs and
interests.

geopolitical community
a type of community that is formed by human or natural
boundaries.
a specific area possessing geographic boundaries and
sharing the same governing structure.
*could also include people who work within the community

phenomenological community
a type of community that is formed because its members
have a common interest, experience or a feeling of
belonging.
boundaries are less well defined, members decide what is
meaningful to them

3 critical attributes of a community


people
place
function/interaction pattern

communities of solution
this type of community is formed by an aggregate
specifically to address health concerns within a particular
area. It is composed of persons not only from the area of
need but also members of neighboring communities with a
vested interest.

3 characteristics of Community health

1. Status dimension (physical, social, emotional)


2. Structural dimension (services and resources)
3. Process dimension (effective community function)

Community health
is a synthesis of the status (what do they look like),
structure (are there services to deal with the issues),
process (how effective are they- do they require outside
help?)

4 healthy people 2020 goals


1. Attain high-quality, longer lives free of preventable
disease, disability, injury, and premature death.
2. Achieve health equity, eliminate disparities, and improve
the health of all groups.
3. Create social and physical environments that promote
good health for all.
4. Promote quality of life, healthy development, and
healthy behaviors across all life stages

epidemiological community assessment model


-this community assessment model has the focus of
Community health.
-This model examines factors that affect the health of the
community such as: Biological, environmental,
lifestyle/consumption/recreation patterns, access to the
health care system
-this model looks at morbidity and mortality data
-this model guides the development of programs targeted
at specific aggregates
(particularly important are at-risk or vulnerable
populations)

2 main reasons for community assessment


-to gain information and clarify the need for change
-to empower those responsible in the context of that
change

community as partner framework


-this framework for community assessment views people as
central members of the community
-the parts of the community that interact with members of
the community:
physical environment, health and social service, economy,
transportation and safety, politics and government,
communication, education, recreation

(wheel representing a core surrounded by the 8


subsystems listed above)

coalition
a group of consumers, health professionals, policy makers,
and others working together to improve community health
status or to solve a specific community health problem

stakeholder
an individual, organization, or group that has an interest in
a specific community health issue or the outcome of a
community-level intervention

key informant
person knowledgeable about specific aspects of a problem
and the communities current and past attempts to address
it

developmental community assessment model


-this type of community assessment gathers information
within the context of the community over history.
-this type of assessment examines issues such as:
population aging, accessibility of community resources,
changes in government structure as the community grows
or shrinks
-this type of assessment analyzes trends or historic events
over time to assess the relative health of the community

Community assessment model asset based approach


-this type of assessment model looks at the components of
a community including their interaction with each other
(within) and interaction with the environment (external)
-community is dynamic, ever adapting system
-incorporates multiple frameworks and includes
components of the other assessment models
simultaneously
-empowers the community; looks at the communities
strengths and then what is needed or not actualized

tools for collecting data


-self
-data sources (national, state, local)
-surveys (windshield, health/risk assessment)
-interviews with key informants

quantitative
numbers, statistical data

qualitative
perception, beliefs, values, attitudes, health concerns

data sources
statistics, surveys, key informants, libraries, telephone
books, drive-through, churches, school systems,
government offices, public safety depts., maps,
professional, business, voluntary and social organizations

assessment issues
gaining entrance/acceptance
trust
confidentiality

planning for community health


-problem analysis and prioritization
-establishing goals and objectives
-identifying intervention activities

Health planning
problems identified, priorities selected, objectives set for
the development of community health programs based on
findings of community health assessments and health
surveillance data

current national/international health plans


include providing health promotion, disease prevention at
population level, addressing social determinants of health
and achieving health equity

levels of health planning


impetus of health planning and program development may
come from the decision at local, state, regional or global
level

bottom-up approach to health planning


impetus for health planning comes from the local/state
level to address a health problem

top-down approach to health planning

impetus for health planning and program development


comes from a regional, national or global initiative to
improve health status

ongoing health planning


immunizations are an example of this type of health
planning

episodic health planning


outbreak of pertussis
bullying
-these are examples of this type of health planning
(fix it and move on)

social ecological model in health planning


-this model of health planning is based on general systems
theory and health promotion theory
-on the premise that multiple determinants of health
interact at different levels to affect the health status of
individual people, population aggregates or communities
-this type of health planning takes into consideration how
many factors (social, political, cultural) influence health
behaviors and health status within a community

multilevel interventions for health planning


-interventions are made at more than one system level
-upstream at the societal, environmental or policy level
-maintstream is at the population or community level
-downtstream is at the individual level

change theory for health planning


unfreezing, changing, refreezing
-sustainability must always be a concern when planning
changes in health beliefs, knowledge, behavior or social
conditions

logic model for health planning


-this model for a community health plan is a visual
representation of the logic behind the operation of the
program
-who will receive the services, what will be done, when will
it happen, where will it happen, why will it happen

SMART objectives

Specific
measurable
achievable
relevant
time bound

Health planning sequence


1. involve community members
2. select the planning group
3. formulate philosophy, goals, forecast
4. evaluate alternative solutions
5. select a solution to fit the community
6. identify resources
7. develop an action plan

steps in health planning program


Assessment
Analysis
Planning
Implementation
Evaluation

Nurse's role in program planning


-collaborator
-coordinator
-facilitator

Community organization models


-community development model
-social planning model
-social action model

community development model


this community organization model would be when you
develop a program with the community
-grassroots level

social planning model


this community organization model would be used when
there is a problem that needs to be addressed

social action model


this community organization model is not often used in
nursing. It is often in emergency situations where

authorities take over, it is highly organized and it is


required.

evaluating community health planning


These categories of questions are answered:
-outcome attainment
-appropriateness of care
-adequacy of care
-relationship of resources to results
-process

community-based nursing
-minor acute and chronic care that is comprehensive and
coordinated where people live, work or attend school
-illness care provided outside of acute care setting
-focus on "illness care"
-interacts 1 to 1
-levels of prevention : 2, 3

community-oriented nursing
-focus on "health care"
-Preserve, Promote, Protect
-interacts with groups and 1 to 1
-levels of prevention :1, 2, 3
-uses systematic processes to deliver care

social determinants of health


social conditions in which people live and work

health care disparities


gaps in health care experienced by one population
compared with another

examples of community health nursing


Public health nurse
school nurse
board of health nurse
community based clinic nurses
home care/visiting nurses
hospice nurses
occupational health/employee health
homeless shelter

public health nursing

population based practice, defined as a synthesis of


nursing and public health in the context of preventing
disease and disability and promoting and protecting the
health of the entire community

public health responsibilities


communicable disease prevention
occupational health
environmental health programs
reproductive health
chronic disease prevention
injury prevention
-newly emerging issues of preventing bioterrorism and
violence, handling disposing of hazardous wastes

principles of public health


1. focus on the aggregate
2. promote prevention
3. encourage community organization
4. ethical theory of the greater good
5. leadership in health
6. use of epidemiologic knowledge and methods

levels of prevention
Primary- prevents disease
Secondary-controls outbreak
Tertiary-limits disability

primary prevention
promotes health
prevents occurrence of disease
(education, vaccinations)

secondary prevention
early diagnosis and interventions
contains/controls outbreak
(mammograms, hearing and vision screening, flu shots)

tertiary prevention
rehabilitation and limiting disability
(cardiac rehab, management of chronic conditions)

goals of community health nursing


care of ill, disabled in a non-institutional setting
-support of well being throughout the life cycle

-promote human relatedness and mutual caring


-promote self respect regarding health and well being
-promote safety in environment

Florence Nightingale
formed a team of nurses that assisted soldiers during the
Crimean War and statistically documented her successes
saving lives through prevention of infections and improving
environmental conditions
-also she opened the first school of nursing

Lillian Wald
She was the founder of public health nursing
Founded the Henry street settlement house

Mary Breckinridge
Frontier Nursing- developed a system of rural healthcare in
remote regions.
Traveled on horseback
opened school for frontier nursing and midwifery

Clara Barton
Red Cross

#1 goal of community health nursing


Health Promotion

purpose of health promotion


-Enable/facilitate optimal health
-decrease risk
-early problem identification
-minimize disability

Definition of health
maximal development- physical, social, emotional
positive interaction with physical environment
effective role performance
absence of disease

4 factors that influence health


1. Biology
2. Physical environment
3. Sociocultural environment
4. Health care access

Primary prevention
maximizing health and wellness through strategies that are
set in place before illness or injury is present

Secondary prevention
maximizing health and wellness through strategies that are
set in place at the early and active chronic stages of
pathogenesis of illness and injury
-interventions designed to diagnose and treat illness early
and restore to health; includes screenings

Tertiary Prevention
maximizing health and wellness through strategies that are
set in place at the palliation and end-stage of disease and
injury trajectories
-rehabilitative/restorative; limites disability

behavior change models


models that assist clients, groups, and communities to
redirect activities toward health and wellness

motivational interviewing
client-centered communication style for eliciting behavior
change by helping clients and groups explore and resolve
ambivalence to change

7 behavior change models


1. learning model
2. health belief model
3. transtheoretical model
4. theory of reasoned action
5. social learning
6. the relapse prevention model
7. ecological model

primary prevention
use of seat belts, hand washing, proper preparation of food,
exercise, balanced nutrition, vaccinations

secondary prevention
women diagnosed with HIV/AIDS
student with scoliosis

learning model for behavior change


a behavior change model emphasizing reinforcement of
social competence, problem solving, autonomy, and sense
of purpose.
-incremental increases are made as a pattern of behavior is
shaped
-uses reinforcement

Health belief model for behavior change


a behavior change model that considers the severity of the
potential illness or physical challenge, the level of
conceivable susceptibility, the benefits of taking
preventative action and the challenges that may be faced
in taking action toward the goal of health promotion.
-belief that the cue messages can be accomplished

transtheoretical model for behavior change


1. precontemplation
2. contemplation
3. preparation
4. action
5. maintenance
6. relapse

theory of reasoned action for behavior change


a behavior model that emphasizes that individual
performance of a given behavior is primarily determined by
a person's intention to perform that behavior
-person's attitude toward the behavior (beliefs about the
outcomes and value of outcomes)

Social learning model for behavior change


a behavior change model that considers environmental
influences, personal factors, and behavior as key
components to change

Relapse prevention model for behavior change


behavior change model that identifies that relapse often
happens because of:
-negative emotional states
-lack of, or limited coping skills
-decreased motivation
-stress
-high risk experiences

ecological model for behavior change

belief that all processes occurring within individual people


and their environment should be viewed as interdependent
-belief that behavior is a result of the knowledge, values,
and beliefs of people as well as many social influences

ontogenic system influences


personal factors
-attitudes and beliefs that influence decisions

microsystem influences
relationship between women and the environment
-relationship stage, intimacy, communication patterns

exosystem influences
formal and informal social structures
-socioeconomic and employment status, degree of social
isolation, social networks and supports, poverty

macrocultural influences
values and beliefs of culture
-social norms, attitudes about gender roles, race

theories of learning
behavioral
cognitive
humanistic
developmental
social learning

behavioral theory of learning


operant conditioning
-target behavior change in response to reinforcement

cognitive theory of learning


information organization, recognition and retrieval
-change in response to changing patterns of thought

humanistic theory of learning


interior motivation for learning
-change response related to feelings, emotions and
relationships

developmental theory of learning

readiness to learn, congruence with developmental stage


-change in learning which occurs across the lifespan

social learning theory


outcome expectations, desired outcomes
-change comes from wanting and believing change is
possible

Domains of learning
-Affective
-Psychomotor
-Cognitive

affective domain of learning


outcome is change in attitude or value; new behaviors
reflect attitude change
- learner will value exercise because it improves sense of
well being

psychomotor domain of learning


improve hands on skills; practice dependent; skill
dependent
-learner will demonstrate how to prepare a nutritious meal

cognitive domain of learning


recall, comprehension, ability to use information learned in
a different way
-learner will describe most important risk behaviors
associated with cardiac events

follower
seeks and follows others direction/authority

gatekeeper
controls/limits outsiders access
ensures all members get an equal voice

leader
guides and directs

peacemaker/conflict manager
facilitate harmony and compromise

task specialists

focuses and directs the group toward accomplishments


(initiator, contributor, information seeker and giver,
summarizer, encourager, elaborator, coordinator)

4 purposes of health education


1. assist clients and populations to make health-related
decisions
2. increases awareness of resources and assists with
resource allocation
3. informs the public about health issues
4. improves personal health
The factors, exposures, characteristics, and behaviors
that determine patterns of disease are described using:
a. Descriptive epidemiology
b. Analytic epidemiology
c. Distribution
d. Determinants
d. Determinants

To understand the causes of health and disease,


epidemiology studies:
a. Individuals
b. Families
c. Groups
d. Populations
d. Populations

When a nurse examines birth and death certificates


during an epidemiologic investigation, what data category
is being used?
a. Routinely collected data
b. Data collected for other purposes but useful for
epidemiologic research
c. Original data collected for specific epidemiologic
studies
d. Surveillance data
a. Routinely collected data

An epidemiologist wanting to know what caused severe


diarrhea and vomiting in several people at a local
banquet would be using:
a. Descriptive epidemiology
b. Analytic epidemiology
c. Distribution
d. Determinants

b. Analytic epidemiology

Which is an example of an epidemic?


a. "Bird" flu in China
b. Adult obesity in the United States
c. An isolated case of smallpox in Africa
d. The nursing shortage in the United States
b. Adult obesity in the United States

John Snow is called the "father of epidemiology" because


of his work with:
a. Cholera
b. Malaria
c. Polio
d. Germ theory
a. Cholera

The interaction between an agent, a host, and the


environment is called:
a. Natural history of disease
b. Risk
c. Web of causality
d. The epidemiologic triangle
d. The epidemiologic triangle

Public health professionals refer to three levels of


prevention as tied to specific stages in the:
a. Epidemiologic triangle
b. Web of causation
c. Natural history of disease
d. Surveillance process
c. Natural history of disease

Which is an example of an agent in the epidemiologic


triangle?
a. Human population distribution
b. Salmonella
c. Genetic susceptibility
d. Climate
b. Salmonella

When studying chronic disease, the multifactorial etiology


of illness is considered. What does this imply?
a. Genetics and molecular structure of disease is
paramount.
b. Single organisms that cause a disease, such as cholera,

must be studied in more detail.


c. Focus should be on the factors or combinations and
levels of factors contributing to disease.
d. The recent rise in infectious disease is the main focus.
c. Focus should be on the factors or combinations and
levels of factors contributing to disease.

Immunization for measles is an example of:


a. Primary prevention
b. Secondary prevention
c. Tertiary prevention
d. Health promotion
a. Primary prevention

Screening for hearing defects is an example of:


a. Primary prevention
b. Secondary prevention
c. Tertiary prevention
d. Health promotion
b. Secondary prevention

Vocational rehabilitation of a person with a neuromuscular


disease is an example of:
a. Primary prevention
b. Secondary prevention
c. Tertiary prevention
d. Health promotion
c. Tertiary prevention

An example of primary prevention is:


a. Pap smear
b. Blood pressure screening
c. Diet and exercise
d. Physical therapy
c. Diet and exercise

An example of secondary prevention is:


a. Rehabilitation
b. Avoidance of high-risk behaviors
c. Immunization
d. Mammogram
d. Mammogram

An example of tertiary prevention is:


a. Rehabilitative job training
b. Parenting education

c. Testicular self-examination
d. Family counseling
a. Rehabilitative job training

The probability an event will occur within a specified


period of time is called:
a. Rate
b. Risk
c. Epidemiology
d. Epidemic
b. Risk

Which statement is true about mortality rates? Mortality


rates:
a. Are informative only for fatal diseases
b. Provide information about existing disease in the
population
c. Are calculated using a population estimate at year-end
d. Reveal the risk of getting a particular disease
a. Are informative only for fatal diseases

A screening for diabetes revealed 20 previously


diagnosed diabetics and 10 probable new cases, which
were later confirmed, for a total of 30 cases. This is
called:
a. Prevalence
b. Incidence
c. Attack
d. Morbidity
a. Prevalence

An example of an attack rate is the:


a. Number of cases of cancer recorded at a medical
center
b. Number of people who died of cholera in India in a
given year
c. Number of beef cattle inoculated against mad-cow
disease on a farm
d. Proportion of people becoming ill after eating at a fastfood restaurant
d. Proportion of people becoming ill after eating at a fastfood restaurant

Which would be considered a serious epidemic of


influenza?
a. 50 cases

b. 100 cases
c. 500 cases
d. Unable to determine
d. Unable to determine

The most important predictor of overall mortality is:


a. Race
b. Age
c. Gender
d. Income
b. Age

An outbreak of a gastrointestinal illness from a food-borne


pathogen is an example of a(n):
a. Attack rate
b. Point epidemic
c. Secular trend
d. Event-related cluster
b. Point epidemic

A study that uses information on current health status,


personal characteristics, and potential risk factors or
exposures all at once is called:
a. Cross-sectional
b. Ecological
c. Case-control
d. Cohort
a. Cross-sectional

The type of epidemiologic study that is used to describe a


group of persons enrolled in a study who share some
characteristic of interest and who are followed over a
period of time to observe some health outcome is a(n):
a. Case control study
b. Cross-sectional study
c. Cohort study
d. Experimental study
c. Cohort study

Voters have recently decided to have fluoride added to


the city water system. Epidemiologists wanting to study
the effect of fluoride on dental caries would be
conducting a(n):
a. Ecological study
b. Double-blind study

c. Community trial
d. Screening
c. Community trial

The proportion of persons with positive test results who


actually have a disease, interpreted as the probability
that an individual with a positive test result has the
disease, is the:
a. Sensitivity
b. Specificity
c. Positive predictive value
d. Negative predictive value
c. Positive predictive value

A nurse is told that a screening test has high specificity.


This means that the test:
a. Provides precise and consistent readings
b. Accurately identifies those with the condition or trait
c. Accurately identifies those without the trait
d. Has a high level of false positives
c. Accurately identifies those without the trait

In which situation does the nurse need to be aware of the


potential for selection bias?
a. Determining the population to be studied
b. Considering how the participants will enter the study
c. Studying cause and effect relationships
d. Documenting results of the study
b. Considering how the participants will enter the study

As a result of an outbreak of influenza in a community, a


nurse encourages members of the community to receive
the influenza vaccine. Which level of prevention is being
used?
a. Primary prevention
b. Secondary prevention
c. Tertiary prevention
d. Multifactorial prevention
a. Primary prevention

Epidemiologic studies of diseases conducted by nurses


during the twentieth century were influenced by the
(select all that apply):
a. Increasing rate of poverty
b. Declining child mortality rates

c. Overcrowding in major cities


d. Development of new vaccinations
b. Declining child mortality rates
d. Development of new vaccinations

Nurses incorporate epidemiology into their practice and


function in epidemiologic roles through (select all that
apply):
a. Policy making and enforcement
b. Collection, reporting, analysis, and interpretation of
data
c. Environmental risk communication
d. Documentation on patient charts and records
b. Collection, reporting, analysis, and interpretation of data
c. Environmental risk communication
d. Documentation on patient charts and records
Which of the following statments best decribes the tterm
epidemiology?
a. applied science that uses surveys to track
communicable diseases(spread from person to person
b. applied science tht uses research and statistics to find
out about how disease affects populations and the
reasons disease occurs
c. applied science whose sole function is to find the
etiology of all diseases
d. applied science that uses correlation studies to
determine the appropriate tertiary prevention strategies
for a community
B

Communiity health is made up of which of the following?


A. epidemiology, parasitology, and correlation studies
B. etiology, studies, prevention, and demographics
C. epidemiology, human ecology, and demography
D. infectious agents, natural hisory of a disease, and
demography
C

Which ofs the following factors can contribute to a person


being a host?
A. exposure to physical, chemical, or or nutritive agents
B. being young or old

C. poverty
D. all above
D

Which of the following statments about the natural


history of a disease is correct
A. it has nothing to do with epidemiological sudies
B. using this method works only for communicable
diseases
C. following the course the disease would take if there
were no interventions can lead to devel strategies or
therapeutic
D. none above
C

A community health RN is participating in an analytic


study of ppl who are at risk for devel diabetes. Studies of
this type are classified
A. retropective
B. concurrent
C. cross-sectional
D. prospective
D

A community health RN is particiipating in an analytic


study of ppl who devel melanoma following freq visits to
tanning salons. Studies of this type are classified
A. retropective
B. concurrent
C. cross-sectional
D. prospective
A

Which of the following characteristics apply to


intervention studies?
A. they don't require altering the behavior of study
subjects, only observing them
B. they include prevention or therapeutic trials
C. They follow the group being studied into the future to
determine risk factors
D. They are the same as correlation studies
B

Which statement most accurately reflects prevalencerate


and incidence rate?
A. there is no difference: they mean the same thing

B. prevalence rate indicates the rate of change from ppl


who do not have the disease, to their having it
C. both cover unspecified, unlimited periods of time
D. incidence rate reflects new cases of a disease during a
specified time
D

Which of the following are considered key vital statistics


A. attack rate and morbidity rate
B. incidence and prevalence rate
C. maternal mortality rate and fetal death rate
D. relative risk
C

Which of the following statements best describes a


community
A. pl living in a particular geographic location
B. organizations, family groups, or friend groups or friend
groups that interact
C. grps that have common interest or needs
D. all above
D

A community of interest could be described as:


A. ppl who live in the same geographic area and share
common interest
B. ppl who share beliefs, values or interest but not
geographic location
C. ppl with the ability to iddentify their own needs
D. ppl who have athe same life expectancy
B

A community that is described as having community


competence has which of the following characteristics
A. the ability to perform their won cross-sectional
epidemiological studies
B. the ability to delegate any community processes to an
outside expert such as a community health nurse
C. the ability to identify their needs, achieve consensus,
and plan and implement goals
D. the ability to predict morbidity and mortality rates for
the population or geographic area
C

The key role for the community health RN in dealing with


communities is to:

A. make sure ppl in the community are empowered and


able to participate
B. provide incentives for community members to follow
the protocols
C. establish project teams that will collect and analyze
data
D. closely direct community members so tht the
community assessments are
A

Which of the following describes a valid way to colllect


data for a community assesment
A. using a bibrary database to conduct a lit review
B. reading government docs to find if similar data have
already been collected]
C. using surveys or questionaires to gather info form
members of communit
D. all
D

Which of the following is NOT a part of the community


assessment
A. identifying available resources such as stime, money,
and team skills
B. collecting and analyzing information
C. withholding results from the community until they can
be statisticall confirmed, to avoid alarming ppl
D. setting action priorties based on the needs of the
community and avail resources
C

a younger community member's reluctance to speak to


an older community health RN might be due to a
difference in cultural perception about
A. personal space
B. honesty
C. authority
D. perception of time
C

Cultural differences in perceptions about time might be


shown as:
A. engaging in several minutes of casual conversation
before talking with the nurse about a healh issue
B. alway arriving at appt times min earlier thatn
scheduled

C. arriving at appt min later tharn scheduled time


D. all
D

Which of the following statements about families is true


A. they are always made up of ppl who are related by
birth or adoption
B. theycurrent definition of family is much less flexible
than it was in the past
C. alternate family configuratins are increasingly common
D. never-married couples are not included wunder the
definition of family
C

A nuclear family of husband, wife, and iffediate children


plus other persons related by blod or voluntary
relationships is called
A. blended family
B. family of procreation
C family of origin
D. extended family
D

An ectramarital family configuration is characterized by"


A. both adults in the family working outside the home
B. child abuse
C. open marriage or co-primary rrelationships
D. adults who voluntarily decide not to have children
C

The family living in Anytown USA consist of 2 adults who


are cohabiting and 4 children from previous
relationshipsThe adults adhere to strict traditional gender
roles and expect the children to be respectful and
obedient to the adults in the household
A. social enciorment
B. psychological enciorment
C. community enviorment
D. dysfunction//B

A family's social enviorment consist of


A. family members communication skills
B. housing conditions inside and outside the house
C. the extended family status
D. religion, race, and socioeconomic class
C

Which of the following would be considered appropriate


parts of a family assessment by a community health
nurse
A. relationships between family members and between
the family and community
B. ability of family members to grow, help 1 another, and
help themselves
C. ability of family membbers to be flexible about roles
and responsibilities
D all
D

Which of the following would be considered a


dysfunctional coping strategy
A. making a genogram
B. triangulation
C. role flexibility
D. addressing the problem
B

a family health tree is used to track which of the following


A. mental health disorders that occur in a family
B marriage and divorce
C. health conditions with no genetic basis
D. places of residence
A

A family thtat copes with stressful situations by saying


"our family is always upbeat, no matter what happen" is
using which of the following dysfunctional coping
strategies
A. pseudomutuality
B. scapgoating
C. triangulation
D. family myth
D

which of the following is useful for showing the difference


between a family's needs and th resources that are
available to them
A. Omaha system
B. family health tree
C. ecomap
D. genogram

nutritional factors for young to middle adulthood would


include
A. control of obesity
B. avoiding eating problems due to poor-fitting dentures
C. dealing with vision loss that could affect eating
D. avoiding all takeout or prepared food
A

Which of these conditions could cause an older adult to


have nutritional problems
A. ill-fitting dentures or other mouth problems
B. poor appetite due to chronic illness
C. low vision
d. all
D

In perfroming a nutritional assessment during a home


visit, you need to be sure
A. that ppl can cook their own food
B. to ignore complaints about food allergies or dislikes
since they interfere with attaining proper nutrition
C. you have clear and complete information about what
the client eats, how much and when
D. that all clients get enough red meat to maintain their
iron levels
C

How can chronic illness impact feeding and nutrition


A. chronicall ill people comoplain a lot abou their food
B. medications for chronic illnesses can cause side effects
that interfere with eating
C. mostchronicall ill people have increased appetites,
putting them at risk for obesiey, a major health risk factor
D none
B

Which of the following would be an appropriate strategy


for preventing nutrition problems in populations
A. sending clients to the hospital for intravenous feeding
B gathering data to screen ppl at risk for nutritional
deficits
C. avoiding use of food pantries or meal delivery because
it is best for clients to cook their won food at home
D. all

10 reportable STD's in Allegheny County with in 5 days


HIV, chancroid, chlamydia trachomaatis, gonococcal
infections, granuloma inguinale, lymphogranuloma
venereum, hep B , syphillis, hep A
Change for the community as client must often occur at
several levels because:
a. Health problems caused by lifestyle are
multidimensional.
b. Most individuals can change their habits alone.
c. Aggregates are responsible for social change.
d. Geographic areas often have health risks that the
nurse must identify.
a. Health problems caused by lifestyle are
multidimensional.

A nurse cares for the community as the client when


focusing on:
a. Providing care for aggregates living in the community
b. The collective good of the population
c. The provision of care for families in the home setting
d. Providing health education in the community
b. The collective good of the population

A collection of individuals who have in common one or


more personal or environmental characteristics is the
definition of a(n):
a. Community
b. Group
c. Family
d. Aggregate
d. Aggregate

What are the critical attributes in the definition of


community?
a. Families, groups, and health organizations
b. Health needs, geographical boundaries, and target
population
c. People, place, and functions
d. Populations and health resources
c. People, place, and functions

A group of nursing students forms a student nursing


association to provide support while they are enrolled in a

nursing program. This is known as a community of:


a. Place
b. Special interest
c. Problem ecology
d. Function
b. Special interest

A public health nurse is collecting data about the


biological aspects of community health. What data will
the nurse collect?
a. Race/ethnicity
b. Traditional morbidity and mortality rates
c. Policy making and social change
d. Homicide rates
b. Traditional morbidity and mortality rates

A community that has residents who are very involved in


its activities and encourage activities that promote the
health of the community displays which condition of
community competence?
a. Commitment
b. Participation
c. Articulateness
d. Effective communication
b. Participation

Which data source provides information about the


function of the community?
a. Maps
b. Census data
c. State departments, business and labor, local library
d. Civic groups
c. State departments, business and labor, local library

Which characteristic is an indicator of community health


process?
a. Participation and community action
b. Live birth rate
c. Racial distribution
d. Socioeconomic levels
a. Participation and community action

A community that demonstrates commitment would most


likely support the development of a:
a. Community club to facilitate community involvement
b. Children's recreation program

c. Curb-side recycling program and community-based


education about recycling
d. Cooperative agreement with a neighboring city to
share needed services
c. Curb-side recycling program and community-based
education about recycling

The main characteristics of partnership are:


a. Awareness, flexibility, and distribution of power
b. Rights, responsibilities, and consensus
c. Commitment, participation, and articulation
d. Collaboration, advocacy, and utility
a. Awareness, flexibility, and distribution of power

When completing a community assessment, the nurse


will:
a. Identify community needs and clarify problems.
b. Determine the weaknesses of a community.
c. Perform the core functions of public health nursing.
d. Assess individual needs within a community.
a. Identify community needs and clarify problems.

When a nurse focuses intervention strategies on the


structural dimension of community health, they are
directed toward:
a. Health services
b. Primary prevention
c. Health promotion
d. Secondary prevention
a. Health services

Which characteristic is an indicator of community health


structure?
a. Infant mortality rate
b. Effective communication
c. Crime rate
d. Emergency room utilization
d. Emergency room utilization

After identifying the boundaries of a community, what is


the next step the nurse should take in completing a
community assessment?
a. Gather relevant existing data and generating missing
data.
b. Plan interventions that benefit the entire community.

c. Formulate nursing diagnoses.


d. Evaluate the interventions that were used.
a. Gather relevant existing data and generating missing
data

What intervention would be the most appropriate for the


nurse to use when trying to develop community
partnerships?
a. Involve only the community residents.
b. Use nurses as the source of information and leadership.
c. Rely on the power of local officials.
d. Include a variety of disciplines.
d. Include a variety of disciplines.

When writing a community-level diagnosis, "Among"


refers to the:
a. Specific problem or health risk faced by the community
b. Characteristics of the community
c. Community client experiencing the risk or problem
d. Likelihood that the problem will be solved
c. Community client experiencing the risk or problem

Which nursing diagnosis is written at the communitylevel?


a. Risk of hypertension related to poor diet and sedentary
lifestyle
b. Risk of obesity among school-age children related to
lack of opportunities to engage in physical activity
c. Risk of ineffective health maintenance among
individuals who do not have access to a primary care
provider
d. Ineffective coping related to multiple stressors as
evidenced by client crying and stating she has no support
system
b. Risk of obesity among school-age children related to lack
of opportunities to engage in physical activity

A nurse surveys the school nurses in a community to


determine their roles in the schools because this data is
not available. The nurse is engaging in the process of
data:
a. Collection
b. Gathering
c. Generation
d. Interpretation
c. Generation

A nurse schedules an appointment with a physician who


has a practice in the community to learn more about the
community's beliefs regarding childhood immunizations.
This is known as:
a. Informant interview
b. Participant observation
c. Secondary analysis
d. Windshield survey
a. Informant interview

When conducting informant interviews in a small


community, the nurse would most likely contact:
a. The state department of health for death records
b. A local priest for congregation information
c. Surrounding communities for crime comparison
d. The Centers for Disease Control and Prevention (CDC)
for illnesses in the area
b. A local priest for congregation information

A nurse collects data about seat belt usage by


interviewing key informants and observing behaviors in
the community. What type of data is being collected?
a. Quantitative
b. Qualitative
c. Focus-groups
d. Survey
b. Qualitative

A nurse who reads the local community newspaper is


using which method to gather data about the community?
a. Informant interview
b. Survey
c. Participant observation
d. Windshield survey
c. Participant observation

Which method of data collection is being used as a nurse


gathers information about the condition of homes, size of
lots, neighborhood hangouts, road conditions, and modes
of transportation?
a. Participant observation
b. Windshield survey
c. Survey
d. Informant interviews
b. Windshield survey

A nurse who uses such data as minutes from a


community meeting is using:
a. Secondary analysis
b. Informant interviews
c. A survey
d. A windshield survey
a. Secondary analysis

A nurse plans on implementing a community-wide


influenza immunization program. Which factor should the
nurse consider when implementing this program?
a. The number of community members who have already
received the immunization
b. The existence of formal groups in the community
c. Public policy that mandates influenza immunization for
certain populations
d. The community's readiness to participate in the
program
d. The community's readiness to participate in the program

To effectively complete the evaluation phase, the nurse


must have:
a. Demonstrated the ability to improve the health of the
participants
b. Developed measureable objectives and goals before
implementation
c. Encouraged full participation by community members
d. Improved the health of the population through the
program
b. Developed measureable objectives and goals before
implementation

To analyze the problem of increased adolescent alcohol


use in the community, a nurse brings together several
groups of people to address the issue. These groups come
together at the same time to work on identifying potential
causes of the problem, but do not interact during the
process. Which model is the nurse applying?
a. Delphi technique
b. Program planning model
c. Community-as-partner model
d. Community empowerment model
b. Program planning model

A nurse is trying to increase participation in a free


colorectal screening program for middle-aged adults who
lack health insurance. Which implementation mechanism
would be most effective?
a. Small interacting groups
b. Health policy
c. Lay advisors
d. Mass media
d. Mass media

A nurse may define a community as a(n) (select all that


apply):
a. Social group determined by geographic boundaries
b. Group of people who share common values and
interests
c. Group of people defined by their interactions
d. Individual with a specific health concern
a. Social group determined by geographic boundaries
b. Group of people who share common values and interests

Which data would be useful for a nurse to collect when


assessing safety in a community? Select all that apply.
a. Number of billboards in the area
b. Interviews with health care providers who are familiar
with the community
c. Observation of community members
d. Nurse's own observations
b. Interviews with health care providers who are familiar
with the community
c. Observation of community members
d. Nurse's own observations
An enduring process in which a manager establishes
systems and monitors the health status, resources, and
outcomes for a targeted aggregate of the population is
called:
a. Case management
b. Care management
c. Advocacy
d. Continuity of care
b. Care management

When would a nurse use a critical path?


a. When implementing the five "rights" of case

management
b. When demonstrating competencies required for
practicing case management
c. When attempting to achieve a measurable outcome for
a specific client
d. When effectively managing conflict resolution
c. When attempting to achieve a measurable outcome for a
specific client

A patient with a long history of asthma with many


hospital admissions is referred to a case manager to
assist in the management of her disease. Disease
management targets:
a. Chronic and costly disease conditions that require longterm care interventions
b. Patients who cannot handle their disease
c. Those who seek to control use by providing clients with
correct information
d. Patients who will need an advanced practice nurse
instead of physician for monitoring
a. Chronic and costly disease conditions that require longterm care interventions

Which type of management would be most appropriate


for a nurse to use when helping clients improve their
health status?
a. Utilization management
b. Case management
c. Disease management
d. Demand management
d. Demand management

Why must a nurse who is using population management


be able to work with integrated care delivery systems?
a. Management has shifted from inpatient care to primary
care providers as points of entry.
b. Emphasis is on episodic illness care for individuals
rather than on population management.
c. Care management services and programs do not
provide access and accountability, as provided by case
management services.
d. Assessment of the needs of the population is no longer
necessary.
a. Management has shifted from inpatient care to primary
care providers as points of entry.

Which activity would a nurse engage in when using


population management?
a. Census taking to determine the total number of people
in the population
b. Assessing the needs of the client population through
compilation of health histories
c. Providing case management services for every citizen
in the community
d. Selecting programs for wellness that are repeated
annually
b. Assessing the needs of the client population through
compilation of health histories

What are the five "rights" of case management?


a. Care, time, provider, setting, price
b. Patient, medication, route, time, documentation
c. Place, setting, patient, plan, outcomes
d. Disease process, time, place, beneficence, advocate
a. Care, time, provider, setting, price

Which model addresses the structure and processes of


using the population-based tools of disease management
and critical pathways to offer care for client populations?
a. Client-focused
b. System-focused
c. Social service
d. Long-term care
b. System-focused

Which are examples of knowledge domains used in case


management?
a. Legal issues, malpractice recognition, and community
involvement
b. Teaching, counseling, and education skills
c. Advocacy, political campaigning and legislative change
d. Grant application, bargaining contracts, and securing
funding
b. Teaching, counseling, and education skills

Which case management activity would most likely be


implemented by a bachelor's prepared nurse rather than
an advanced practice nurse (APN)?
a. Working with community aggregates
b. Working with systems of disease
c. Working with individuals
d. Working with outcomes management processes

c. Working with individuals

The case manager who uses effective collaboration and


team strategies to make arrangements for services is
performing the role of:
a. Broker
b. Negotiator
c. Liaison
d. Facilitator
b. Negotiator

When working with a client, when would it be most


appropriate for a nurse to use life care planning?
a. When organizing a timeline of life events
b. When documenting client information and requests
c. When assessing present and future client needs
d. When estimating future costs for medical care
c. When assessing present and future client needs

When case managers provide a formal communication


link among all parties concerning the plan of care
management, they are assuming the role of:
a. Facilitator
b. Liaison
c. Coordinator
d. Negotiator
b. Liaison

Which nursing intervention does the case manager


implement when fulfilling the role of coordinator?
a. Providing information to all parties about the situations
affecting the client
b. Educating the client and providers in order to make
informed decisions
c. Supporting all parties to work toward mutual goals
d. Arranging, regulating, and balancing needed health
services for the client
d. Arranging, regulating, and balancing needed health
services for the client

Which are examples of case-managed conditions?


a. Bankruptcy, financial distress, depression
b. Flu, colds, frequent headaches
c. Malaria, bird flu, Dengue fever
d. AIDS, spinal cord injury, ventilator dependency
d. AIDS, spinal cord injury, ventilator dependency

The public health nurse working with the chronically ill


and families with young children needing age-specific
health maintenance is using which model of case
management?
a. Client-focused
b. System-focused
c. Social service
d. Long-term care
a. Client-focused

In case management, the process of performing


interdisciplinary, family, and client conferences would
occur during which phase of the nursing process?
a. Assessment
b. Diagnosis
c. Planning/outcome
d. Implementation
b. Diagnosis

When the case manager contacts providers and


negotiates services and price, which phase of the nursing
process is used?
a. Assessment
b. Diagnosis
c. Planning/outcome
d. Implementation
d. Implementation

A case manager supports a client's decision to return


home after having a total hip replacement rather than go
to a skilled nursing facility. Which phase of the nursing
process is being used?
a. Assessment
b. Planning/outcome
c. Implementation
d. Evaluation
d. Evaluation

The process of moving conflicting parties toward an


outcome is called:
a. Negotiation
b. Conflict management
c. Problem-purpose expansion method
d. Brainstorming
a. Negotiation

The first stage of collaboration is:


a. Trust building
b. Consensus
c. Collegiality
d. Awareness
d. Awareness

Mutual benefit with limited loss for everyone is a goal of:


a. Negotiating
b. Assertiveness
c. Conflict management
d. Cooperation
c. Conflict management

The nurse and client, a 20-year-old expectant mother, are


discussing the advantages and disadvantages of
breastfeeding. To enhance understanding of the
implications of breastfeeding, the nurse says, "Tell me
more about how you will work full time and breastfeed."
Asking this question is an example of which aspect of the
advocacy process?
a. Verification
b. Clarification
c. Amplification
d. Affirmation
b. Clarification

Upholding a client's right to make a choice and to act on


the choice involves:
a. Affirming
b. Supporting
c. Informing
d. Decision making
b. Supporting

An individual who pursues neither his or her concerns nor


another's concerns is using which conflict management
behavior?
a. Accommodating
b. Collaborating
c. Avoiding
d. Compromising
c. Avoiding

The nurse is involved in a conflict resolution situation with


the parents of a 2-year-old boy. The parents are deciding
if "spanking" the child is a disciplinary method that they
will employ. The mother says, "I do not believe in
spanking. I see it as abusive and demeaning." The
mother's statement is an example of:
a. Cooperation
b. Assertiveness
c. Bargaining
d. Collaboration
b. Assertiveness

Which principle of justice is influenced when excessive


attention to cost containment impairs the nurse's duty to
provide measures to improve health?
a. Autonomy
b. Beneficence
c. Confidentiality
d. Justice
b. Beneficence

Which ethical principle is influenced when care in a


managed system is provided by less-experienced
providers?
a. Autonomy
b. Beneficence
c. Confidentiality
d. Justice
d. Justice

A case manager submits documentation that a nursing


visit was completed at a client's home, but it was never
performed. According to the general areas of legal risk,
how would this action best be categorized?
a. Liability for managing care
b. Fraud/abuse
c. Negligent referral
d. Confidentiality/security
b. Fraud/abuse

What knowledge and skills are required in order to


become a competent case manager? Select all that apply.
a. Knowledge of community resources and financing
mechanisms
b. Written and oral communication skills
c. Proficient negotiation and conflict-resolving practices

d. Application of evidence-based practices and outcomes


measurements
a. Knowledge of community resources and financing
mechanisms
b. Written and oral communication skills
c. Proficient negotiation and conflict-resolving practices
d. Application of evidence-based practices and outcomes
measurements

What action should a nurse take in order to reduce the


risk of liability? Select all that apply.
a. Provide accurate documentation of client visits.
b. Inform clients of their rights of appeal.
c. Refer clients to the providers of their choice.
d. Utilize care planning whenever possible.
Humanistic Nursing Theory
studies the existence of reality of nursing

Transcultural Nursing
comparison of differing cultures to understand their
similarities and their differences across human groups

Culture
set of values, beliefs and traditions that are held by a
specific group of people and that are handed down,
generation to generation

Leininger
1. cultural preservation or maintenance
2. cultural care accommodations or negotiation
3. cultural care re-patterning or restructuring

Nightingale's 10 Canons
1. ventilation and warming
2. light and noise
3. cleanliness of the pt area
4.health of houses
5. bed and bedding
6. personal cleanliness
7. variety
8. offering hope and advice

9. food
10. observation

Primary prevention
patient assessment and intervention to identify and reduce
possible risk factors

Secondary prevention
detection of symptoms that are a reaction to stressors,
appropriately prioritizing interventions, and treatments that
reduce the toxic effects of stressors

Health
seen as a dynamic, positive state and not just absence of
disease

5 A's of Health Care Access


availability, accessibility, affordability, acceptability, and
accommodation

Market justice
people are only entitled to what they can earn; the focus is
on individual responsibility "minimal collective action and
freedom from collective obligations except with respect to
others fundamental rights

Vulnerable populations
low income, less education, homelessness, military
veterans, immigrants, prisoners, migrant and seasonal
farm workers and their families, chronic illness, mental
illness

social determinants
are the circumstances in which people are born; grow up,
live, work, and age as well as the systems put in place to
deal with illness

Health disparities
difference in health outcomes between populations, social,
demographic and geographic

Health inequities
modifiable associate with social inequalities and are
considered unfair

Leininger- Transcultural Nursing


requires an awareness of self, sensitivity to others,
respectful dialogue

Ethnocentric
tendency to believe others think the same was as "they
do," have the same world view, culture

Altruism
putting others interests before one's own, being present is
an example

Autonomy
PHN independent and responsible for actions - respects
community and individual right to self-determination;
maintain privacy, honesty, and fidelity

Beneficence
seek to do good, not harm

Common Good Perspective


good for the whole

Ethics
standards of behavior that tell us how we ought to act in
various situations we encounter in life

Fairness Perspective
focuses on how fairly or unfairly actions affect a group how benefits and burdens are distributed; requires
consistency in the way people are treated

Non-maleficence
first do no harm - primum non nocere

Advance directive
legal document in which an individual conveys their
decisions about end-of-life care

Health literacy
includes the ability to understand instructions on
prescription drug bottles, appointment slips, medical

education brochures, doctor's directions and consent forms


and the ability to negotiate complex health care systems

Pre-crisis
planning and test messages

Initial
audience wants information NOW

Maintenance
on-going assessment of the situation

Resolution
community will not be ready to conduct risk reduction
immediately after emergency

Evaluation
Crisis and emergency risk communication response

4 P's of Social Marketing


product, price, place, promotion

NICs
Nursing interventions classification

NOCs
nursing outcomes classification

Nursing minimum data set (NMDS)


standardizes the collection of nursing data; can provide
accurate description of nursing dx, care and resources used
to help define cost and quality of nursing care

Tele-Health
electronic technologies to support clinical health care,
patient and practitioner education, public health and health
administration by long distance

"Right-to-Know" Laws
give people and communities the right to know about
possible chemical exposures where they live and work

Infancy

trust vs. mistrust

Early childhood (18 mo-3 yrs)


autonomy vs. shame

play age (3-5 yrs)


initiative vs. guilt

Adolescence (12-16yrs)
Identity vs. inferiority diffusion

Middle adult (35-55/65 yrs)


generativity vs. self-absorption/stagnation

late adulthood (55/65- death)


integrity vs. despair/wisdom

Young adult (18-35 yrs)


intimacy vs. solidarity/isolation

Sensory-motor stage
birth to two

Pre-operational stage
about two to seven

concrete operational stage


ages seven to eleven

Formal operational stage


eleven years and older

Epidemiology
distribution and determinants of health related
states/events in populations and the use of this study to
control health problems

Prevalence
measures the proportion of the population affected by a
specific condition in a specific time period

Incidence

measures the risk of developing a new condition (disease,


injury, symptom, or death), over a specific period of time,
such as a year

Mortality rate
also called death rate=an estimation of the proportion of a
population that dies during a specified time period

Epidemic
rates exceed normal or expected frequency in a given
population/community/region

Pandemic
worldwide epidemic (influenza and plague)

Chain of causation
chain of events where every event caused by the previous
event; the sequence of stops that have been identified as
linked to the disease

Web of causation
describes events that have multiple causative factors

Passive Immunity
newborns via maternal antibodies

Active immunity
acquired after exposure to a disease or via immunizations

Cross-immunity
immunity to one pathogen confers immunity to another
such as cowpox providing immunity to smallpox

Herd-immunity
present in a population, the higher the immunity decreases
risk of disease (use of immunizations); increasing
immunizations reduces risk of exposure to disease of those
not immunized

Primary prevention
used before the person gets the disease or condition to
reduce the incidence and prevalence of the disease;
includes health promotion (education) and protection
(immunization)

Secondary prevention
applied after the disease has occurred but before signs are
evident; focuses on early dx (screening) and prompt tx to
limit disability

Tertiary prevention
applied when person already has the disease, focuses on
preventing damage from the condition, slowing the process
of the disease, preventing complications, and returning the
person afflicted to their optimal level of health
(rehabilitation)

Case Series and Case Reports


collection of reports; no control group, easy to understand
and can be written quickly

Case Control Studies


compare those with the disease to those without. estimates
odds of getting the disease or condition, less reliable than
randomized control trials, works well for rare conditions

Cohort Studies
may be prospective or retrospective, used to establish
causation or evaluate the impact of treatment when
randomized control trials are not possible over time-longitudinal; not good for rare conditions, requires large
study group

Randomized Controlled studies


requires a treatment group and a control group, pts are
randomly assigned to all groups, considered "gold
standard" of medical research to determine cause and
effect

double blind method


neither patient nor researcher knows if the subject is in the
control or treatment group

meta-analysis
combines data from numerous studies and arrives at
pooled estimates of treatment effectiveness and statistical
significance

Descriptive studies
are observational studies that seek to describe phenomena

Qualitative studies
describe phenomena of interest; include participant
observation, in-depth interviews and focus groups

Quantitative studies
seeks to confirm hypotheses about phenomena; uses more
rigid style eliciting and categorizing responses

sulfur dioxide
product of industry produces acid rain and contributes to
respiratory illness

nitrous oxide
combustion product contributes to illness of lungs, immune
system and asthma

carbon monoxide
car exhaust reduces oxygen delivery at high concentrations

radon
naturally occurring radioactive gas that cannot be detected
by our senses, but is the leading cause of lung cancer in
non-smokers; it is the second cause of lung-cancer behind
smoking

consultation
involves seeking advice or an opinion of an expert; the
consultant does not solve the problems, but assists and
guides the group seeking consultation to solve their own
problems

collaboration
taking an active role in implementing the plan, this requires
understanding of each party's perspectives, on-going
communication and feedback

coalition-building
alliance or group of stakeholders with a common
interest/goal

Advocacy

purpose is policy change; critical role for the public health


nurse

Cognitive
thinking (comprehension, analysis, synthesis, application,
evaluation); evaluate the level of the learner

affective
feeling (receptive, active participation, valuation,
internalization), includes changes in attitude and values

psychomotor
acting - demonstrates performance skills

6 stages of disaster life cycle


prepare, respond, mitigate, reduce the risk, and prevent

natural disasters
due to weather or other natural occurrences

man-made disasters
intentional power outages, chemical spills, acts of bio
terrorism

bacterial agents
anthrax, plague

viral agents
smallpox

Biological toxins-category A
considered high priority and pose national security threathigh morbidity and mortality
smallpox, anthrax, plague, Ebola, Lassa fever, Q fever,
botulism, tularemia, hemorrhagic fever

Biological toxins-category B
moderate to low morbidity and mortality, difficult to detect
Bucellosis, clostridia toxin, staph enterotoxin, salmonella,
shigella, e.coli, cholera

Biological toxins-category C

emerging pathogens such as yellow fever, drug resistant


TB

Family disaster plan


encourage families to prepare a family emergency/disaster
kit, they might be deployed to local disaster sites

Health outcomes
health promotion interventions there must be outcomes to
measure analysis of proper structure and constructive
process, operationalizing, outcomes (improved health =
weight loss, lower blood sugars)

Health status & quality of life indicators


individual perception based on health, cultural background,
education, moral and ethical values, difficult to asses
because it is very subjective

Disparities
exist due to poverty, lower educational levels, unequal
access to healthcare, language barriers, environmental
conditions, racism

Health Belief Model (HBM)


predicts health related behavior and compliance to prevent
diseases; health promotion empowers populations so they
can have control over personal health and access
resources; perception, attract positive value (health) and
repel negative value (illness)

Transtheoretical Model (TTM)


five stages of progression (1) pre-contemplation- no
thought of change (2) contemplation-owning the
problem/evaluate options (3) preparation- plan of action (4)
action-changes are attempted (5) maintenance-behavior
sustained usually 6 mos or longer

Health Promotion Model (Pender)


actions to improve health using the following driversindividual characteristics and experiences, behaviors,
behavioral outcomes, health promotion, activities involve
removing internal and external barriers and promoting
public policy to remove barriers
type of model

Motivation
moved from contemplation to action

Self-efficacy
belief about capabilities to produce effects

Self-management
manage independently choices and consequences of life

Compliance
degree to which a plan is followed (passive)

Adherence
agreement to follow a plan of action (active)

pedagogy
literally means the "art and science of teaching children"

andrgogy
intentional and professionally guided activity the aims at a
change in adults

Nurse-managed centers
nurse-managed centers provide a safety net for the poor
and undeserved in rural and urban areas; run by advanced
practice nurses, and may be associated with universities or
may be independent

parish nursing
licensed, registered nurses practicing holistic care in faith
communities

correctional health nursing


nurses working with populations in jails, prisons, detention
centers, holding facilities

occupational health nursing


nurses working with populations within their work
environment; provide workplace clinics, conduct safety
education and improving working conditions to prevent
injury; balance the company's need for productivity with
knowledge of occupational health and workforce/place
safety

OSHA
sets health and safety work standards, employers are
legally required to protect workers from hazards;
employees have right to know what hazards they are
exposed to

HIPPA
protects the privacy of identifiable, private health
information

EPA
reduce environmental risk and protect the public, maintain
safe air and water, establish pollution regulations, solid
waste and toxic substance disposal, regulate pesticides,
oversee radiation hazards, and noise abatement

HHS (US Dept. of Health and Human Services)


principle government agency that protects the health of
our country; under its umbrella are: CMS, CDC, NIH, FDA,
SAMHSA

CMS
Center for Medicare and Medicaid Services

CDC
monitors and tracts diseases, research injury, infectious
disease, environmental health, genomics, global health,
chronic disease prevention, birth defects and
developmental disorders; publishes the morbidity and
mortality weekly report

NIH
mission is to seek fundamental scientific truth about the
natures and behavior of the living systems and the
application of science to improve health, lengthen life and
reduce the heavy load of illness and disability

SAMHSA
Substance Abuse and Mental Health Administration;
coordinates and funds sustainable substance abuse and
mental health programs in communities or community
health agencies

stakeholders

those who stand to gain or lose from the success or failure


of an initiative

documentation standards
an agency's standards for how care is
recorded/documented on an individual or program level;
provides a legal record, and should be performed in a
timely manner to ensure accuracy

risk management protocol


identification of areas where mistakes of omission or
commission can be made

performance standards
defined standards/expectations for how business is to be
conducted and how performance will be measured

mentoring
expert who establishes a long-term relationship with a
mentee to help them meet professional goals or learn the
role

preceptor
guidance and role modeling are ways to nurture colleagues
or students

professional credibility
clearly demonstrates expertise in an area

expert
perform as an expert maintain standards of behavior and
accountability
health promotion
assessing the health of our community, planning activities
to help our participants to gain some control over their
health to improve their health; emphasis on helping people
change their lifestyles and move toward a state of optimal
health

impact of health promotion


Health promotion impacts communities; health promotion
activities can also impact an entire system; involves all
levels of prevention to promote health across the lifespan.

Goals of Healthy People 20/20


provides science-based, 10 year national objectives for
promoting health and preventing disease--1) to eliminate
preventable disease, disability, injury, and premature
death; 2) to achieve health equity, eliminate disparities,
and improve the health of all groups; 3) to create social
and physical environments that promote good health for
all; and 4) to promote healthy development and healthy
behaviors across every stage of life

determinants of health
factors which influence an individual's or population's
health; influences include the availability and access to:
high quality education, nutritious food, decent and safe
housing, affordable, reliable public transportation, culturally
sensitive health care providers, health insurance, and clean
water and non-polluted air

MAPP
Mobilizaing for Action through Planning & Partnerships; this
tool helps communities improve health and quality of life
through community-wide strategic planning; communities
seek to achieve optimal health by identifying and using
their resources wisely, taking into account their unique
circumstances and needs, and forming effective
partnerships for strategic action

Principles of MAPP
systems thinking, dialogue, shared vision, data,
partnerships and collaboration, strategic thinking, and
celebration of successes

incidence
the number of people in a population who develop the
condition during a specified period of time (number of new
cases over a period of time; rate: (# of new cases of
disease/total population at risk)x100,000

prevalence
the total number of people in the population who have the
condition at a particular time; rate: (# of existing cases of
disease/# of in total population)x100,000

infant mortality rate (IMR)


# of infant deaths (before age of 1 yr) during the year is
divided by the # of live births (infants born alive) during

that year (multiply answer by 1000 to obtain meaningful


rate)

importance of IMR (in epidemiology)


because IMR is influenced by a variety of biologic and
environmental factors affecting the infant and mother, the
IMR is both a direct measure of infant health and an
indirect measure of community health as a whole

relative risk
compares the risk of developing the health condition for
the population exposed to the factor with the risk for the
population not exposed to the factor; indicates the benefit
that might accrue to the person if the risk factor is
removed; rate: incidence among those exposed/incidence
rate among those not exposed

Census (epidemiology data)


source of epidemiological community data; most
comprehensive source of health-related data for the US;
age, race, and sex along with factors such as employment,
income, migration and education; used to calculate rates
(denominator)--collected every 10 yrs

Vital Statistics (epidemiology data)


data that describe legally registered events such as births,
deaths, marriages, and divorces; obtained from CDC's
National Center for Health Statistics and U.S. Public health
Service--collected on an ongoing basis

Notifiable Disease Reports (epidemiology data)


data collected on diseases based on the ability to cause
death, and communicability of the disease; notification of
public health officials of diseases posing a threat to large
populations provides an initial starting point for local
epidemiologic investigations

vital record linkage (epidemiology data)


connects data and information contained in two or more
medical, or mortality records, and other vital event records;
provides excellent sources of information on the courses of
diseases, demographic data, health are services utilization,
fertility, maternal health issues, child health concerns,
chronic disease tracking, and the natural history of specific
disease or morbidity related events

medical and hospital records (epidemiology data)


provide valuable information for community health
research; but do not provide a completely representative or
valid picture of community health

autopsy records (epidemiology data)


provide severely biased information; not performed for all
deaths, violent deaths are included, and sometimes cause
of death is unknown

national electronic disease surveillance system


(epidemiology data)
provides an integrated surveillance system that transmits
public health laboratory and clinical data via the Internet;
improves the nations ability to identify and track emerging
infectious diseases and potential bioterrorism attacks--aids
in surveillance and analysis of notifiable diseases (CDC)

epidemiologic triangle
agent-host-environment model that is a traditional view of
health and disease developed when epidemiology was
concerned chiefly with communicable disease; all 3 can
coexist-disease occurs with interaction of agent, host and
environment

agent
an organism capable of causing disease

host
the population at risk for developing the disease

environment
a combination of physical, biological, and social factors that
surround and influence both the agent and the host

web of causation
views a health condition as the result not of individual
factors but of complex interrelationships of numerous
factors interacting to increase or decrease the risk of
disease; a number of interrelated variables are almost
always involved in the cause of particular outcome

surveillance

systematic collection, analysis, and interpretation of data


related to the occurrence of disease and the health status
of a given population

passive surveillance
more common form of surveillance used by most local and
state health departments; health care providers report
notifiable conditions

active surveillance
purposeful, ongoing search for new cases of disease by
public health personnel

social determinants of health


factors which influence an individual's or population's
health: complex relationships between health and biology,
genetics, and individual behavior, and between health and
health services, socioeconomic status, the physical
environment, discrimination, racism, literacy levels, and
legislative policies

community analysis
multi-step process used to derive a community nursing
diagnosis; phases used are: categorization, summarization,
comparison, and inference elaboration

traditional categorization of community assessment data


(community analysis)
demographic characteristics; geographic characteristics;
socioeconomic characteristics; and health resources and
services

demographic characteristics
family size, age, sex, and ethnic and racial groupings

geographic characteristics
area boundaries, number and size of neighborhoods, public
spaces and roads

socioeconomic characteristics
occupation and income categories, educational attainment,
and rental or home ownership patterns

summarize (community analysis)

once categorization method has been selected, the next


task is to summarize the data within each category; both
summary statements and summary measures, such as
rates, charts, and graphs, are required

compare (community analysis)


identification of data gaps, incongruencies, and omissions;
comparative data are needed to determine if a pattern or
trend exits or if data do not seem correct and if revalidation
of original information is required

draw inferences (community analysis)


drawing logical conclusion form the evidence once data
collected has been categorized, summarized, and
compared; synthesize what is known about a community

community nursing diagnosis


Synthesizes assessment data; a dx both describes a
situation (state) & Implies an etiology (reason) and has
three parts: Description of problem, response, rate; ID of
factors etiologically related to problem; Signs/symptoms
characteristic of problem

Community based planning


are based on the nursing diagnoses and contain specific
goals and interventions for achieving desired outcomes; a
process and has to be completed in partnership with the
community.

Factors in planning a community program


Community partnership (do they agree); Influences of
social, economic, environmental, political issues; Etiologic
antecedents; Health needs of at-risk populations (and do
they agree with this); Awareness/application of planned
change

goals and objectives in planning community health


program
can be written in sequential steps that are required to
reach the goal; must be measurable; include a time frame
for attaining change; the direction and magnitude of the
change; and method of measuring the change

Precise terms to use in making goals and objectives


measurable (learning objectives)

to identify; to discuss; to list; to compare and contrast; to


state; to decrease by 20%

Reinkemeyer's stages of planned change


Stage 1: Identification and development of felt need and
desire for change
Stage 2: Development of a change relationship between
the community health nurse and the community
Stage 3: Clarification or diagnosis of the community's
problem, need, or objective
Stage 4: Examination of alternative routes and tentative
goals and intention of actions (planning)

primary prevention
improves the health and well being of the community,
making it less vulnerable to stressors; health promotion
programs, programs that focus on protection from specific
disease; usually nonspecific and directed toward raising the
general healthy of the total community

secondary prevention
begins after a disease or condition is present, although
there may be no symptoms; emphasis on screening, early
diagnosis, and treatment of possible stressors that may
adversely affect the community's health

tertiary prevention
focuses on restoration and rehab; act to return the
community to an optimal level of functioning.

evaluation
must be planned before implementation; measures
progress toward goals and learning objectives; revises
assessment database and community nursing diagnosis

process (formative) evaluation


initial implementation/development of program; answers:
are we doing what we said we would do?

impact (summative; short term) evaluation


concerned with the immediate impact of a program on a
target group

outcome (long-term) evaluation

has incidence/prevalence changed? determine if change


had a lasting and real effect

Phases of MAPP
Phase 1: organizing for success, partnership development
Phase 2: visioning
Phase 3: The MAPP assessments
Phase 4: Identifying strategic issues
Phase 5: developing goals and strategies
Phase 6: the action cycle

Priority Health Promotion of HP 20/20


unintentional injury; violence; suicide; tobacco use and
addiction; alcohol or other drug use; unintended
pregnancy, HIV/AIDS, and STD infection; unhealthy dietary
patterns; and inadequate physical activity.

Cognitive learning domain


memory, recognition, understanding, reasoning,
application, and problem solving and is divided into
hierachical classification of behaviors; hierarchical
classification of behaviors (knowledge, comprehension,
application, analysis, synthesis and evaluation)

affective learning domain


changes in attitudes and the development of values by
influencing what learners feel, think, and value

psychomotor learning domain


involves the performance of skills that require some degree
of neuromuscular coordination and emphasize motor skill;
in learning a skill, first show learners how to do the skill;
next allow learners to practice (teach back or show back
approach)

educational process
1. identify client needs-needs assessment
2. establish educational goals and objectives
3. select appropriate educational strategies
4. develop effective skills as an educator
5. develop effective health education programs

goals
long term group overall learning needs or expected
outcomes after instruction or program activities

objectives
specific, short term, measurable, action oriented behaviors
to be met as steps towards achieving the long term goal

Educator Skill (education process)


1. gain attention
2. inform the learner of the objectives of instruction
3. stimulate recall of prior learning
4. present the stimulus
5. provide learning guidance
6. elicit performance
7. provide feedback
8. assess performance
9. enhance retention and transfer of knowledge

global burden of disease


the disability-adjusted life year (DALY), which is a timebased measure combining years of life lost due to
premature mortality and years of life lost due to time lived
in states of less than full health

Millennium Development Goals


1. eradicate extreme poverty and hunger
2. achieve universal primary education
3. promote gender equality and empower women
4. reduce child mortality
5. improve maternal health
6. combat HIV/AIDS, malaria, and other diseases
7. ensure environmental sustainability
8. develop a global partnership for development

child mortality and millennium goal


reduce by 2/3, between 1990 and 2015, the under 5
mortality ratio

how to influence maternal health


increased access to prenatal care, to skilled birth
attendants, and the prevention of teen and unplanned
pregnancies

8 essential elements of primary health care


1. education for the identification and prevention/control of
prevailing health problems
2. proper food supplies and nutrition

3. adequate supply of safe water and basic sanitation


4. maternal and child care, including family planning
5. immunization against the major infectious diseases,
prevention and control of locally endemic diseases
6. appropriate treatment of common diseases using
appropriate technology
7. promotion of mental health
8. provision of essential drugs

genomics
the study of functions and interactions of all genetic
material in the genome, including interactions with
environmental factors

public health genomics


the study and application of knowledge about the elements
of the human genome and their functions, including
interactions with the environment, in relation to health and
disease populations

nursing informatics
specialty that integrates nursing science, computer
science, and information science to manage and
communicate data, information, and knowledge in nursing
practice

information literacy
the ability to recognize when information is needed as well
as the skills to find, evaluate, and use needed information
effectively

public policy
applies to all members of society; includes prescribed
sanctions for failure to comply; formulated by government
bodies; frequently restricts personal choice to improve
public welfare

social welfare policy


policy that promotes the welfare of the public; child labor
laws and income assistance (social security)

institutional policy
governs work sites; based on the institution's mission and
goals, and they determine how the institution will function
and relate to employees

organizational policy
formed an applies to groups who have similar interests or
special interests, such as professional associations-state
nurses association or specialty nursing organization that
determines governing rules for membership

medicare
health insurance program; 65 years or older, disabled
people younger than 65, end stage renal disease or Lou
Gehrig disease

medicaid
healthcare, insurance assistance, nursing home coverage;
low-income residents and pregnant women, a parent or
relative caretaker of a dependent child under age 19, the
blind, people with a disability or with a family member in
their household with a disability, or people aged 65 years
or older

State Children's Health Insurance Program (SCHIP)


doctor visits, immunizations, hospitalizations, emergency
room visits; children 19 years and younger from families
who earn to much to qualify for Medicaid but who meet
SCHIP eligibility

What nurses can do to influence health policy


1. Learn the legislative process
2. Stay informed on current issues
3. Identify government representatives
4. Be an active constituent
5. Speak out

health economic
concerned with how scarce resources affect the health care
industry

public health economics


focuses on producing, distributing, and consuming goods
and services as related to public health and where limited
public resources might best be spent to save lives or cause
the most increase in quality of life

macroeconomics

the study of the economy as a whole; focuses on the "big


picture" -the total, or aggregate (usually a country or
nation), of all individuals and organizations; business cycle
and economic growth; human capital approach

microeconomics
the branch of economics that studies the economy of
consumers or households or individual firms; the behavior
of individuals and organizations & how this effects prices,
costs and resources; behaviors that result from tradeoffs in
the use of service and budget limits

efficiency (economics)
refers to producing maximal output, such as a good or
service, using a given set of resources (or inputs), such as
labor, time, and available money; suggests that inputs
used in such a way that there is no better way to produce
the service, or output, and that no other improvements can
be made

effectiveness (economics)
refers to the extent to which a health care service meets a
stated goal or objective, or how well a program or service
achieves what is intended; immunization=herd immunity

environmental health
promotion of safe, healthful living conditions and protection
from environmental factors that may adversely affect
human health or the ecologic balances essential to longterm human health and environmental quality, whether in
the natural or man-made environment

radon (Rn)
a radioactive gas that is formed naturally from the
radioactive decay of uranium in rocks and soil; it can
remain in the soil, move to the soil surface and enter the
air, or enter groundwater; colorless, odorless, and
tasteless, but extremely toxic; when cooled below the
freezing point, it becomes phosphorescent, in yellow and
orange-red tones.

risk for exposure to radon


You can be exposed to higher levels of radon at work if you
are a uranium miner, a hard rock miner, or work in tunnels,
power stations, public baths, or spas.

exacerbation of asthma in kids


pollutants reduce lung function by inflaming the lining of
the lungs; exposure to pollutants in the air can cause flareups and may increase the chance of upper respiratory
infections, which can worsen asthma symptoms. If
allergens in the air are an asthma trigger, pollutants can
make the lungs even more sensitive to them.

ozone action days


declared on hot, sunny, still days when conditions are most
favorable for the accumulation of unhealthy levels of
ground-level ozone; forcasted by Texas Commission on
Environmental Quality (TCEQ) make their forecasts a day in
advance: Austin, Beaumont-Port Arthur, Corpus Christi,
Dallas-Fort Worth (where it's called an "Air Pollution
Watch"), El Paso, Houston-Galveston-Brazoria (where it's
called an "Ozone Watch"), San Antonio (where it's called an
"Air Quality Health Alert") and Tyler-Longview-Marshall.

6 ways climate change can impact health


air pollution, extreme heat, infectious disease, drought,
flooding, and extreme weather

air pollution and health


Rising heat worsens smog. Burning coal and oil emits
carbon and particle pollution; plants produce more
allergenic pollen, affecting respiratory health threats like
asthma.

extreme heat and health


Heat waves send thousands to emergency rooms and cost
health care systems millions of dollars; climate change
brings longer, more intense heat waves.

infectious diseases and health


Hotter summers can make disease-carrying insects more
active, for longer seasons; illnesses like dengue, West Nile,
and Lyme can spread into new areas

drought and health


Hotter days and nights, and changing rainfall patterns
reduce water supply quantity and quality, and diminish
food security.

flooding and health


Climate change intensifies rainfall; heavy rains increase
risk of drinking water contamination and illness; floods can
force communities to relocate.

extreme weather and health


In 2011, thousands of record-breaking extreme weather
events harmed communities and health in the US. Climate
change is contributing to more intense and frequent
extreme weather events.

Fracking
common but controversial practice among companies that
drill underground for oil and natural gas; drillers inject
millions of gallons of water, sand, salts and chemicalsall
too often toxic chemicals and human carcinogens such as
benzeneinto shale deposits or other sub-surface rock
formations at extremely high pressure, to fracture the rock
and extract the raw fuel.

dangers of fracking
leaves behind a toxic sludge that companies and
communities must find some way to manage; toxic
chemicals used in the fracking process remain stranded
underground where they can, and often do, contaminate
drinking water, soil and other parts of the environment that
support plant, animal and human life; Methane from
fracture wells can leak into groundwater, creating a serious
risk of explosion and contaminating drinking water supplies

What to look for in sunscreen


active ingredients (zinc oxide, titanium dioxide, Mexoryl SX
or avobenzone-3%)
SPF 15-50, depending on your skin tone and sun intensity
Lotions, not sprays or powder
Water-resistant for the beach or pool

What to avoid in sunscreen


vitamin A causes skin cancer in laboratory tests
Oxybenzone, a hormone disruptor and skin allergen
High SPF misleads consumers and offers little additional
benefit

public health policy


works to improve the publics health through legislative
action at the local, state and federal level; institutional and

legislative efforts to promote health and meet national


health objectives...support initiatives that address barriers
to health such as poverty, homelessness, unsafe living
conditions, abuse and violence, and lack of access to
health services

human capital
the knowledge and skills that workers acquire through
education, training, and experience

relative risk factor


factor of 1.0 means the risk of disease among the exposed
is no different from the risk of disease among the nonexposed; an rr of 2.0 implies that the risk is twice as high,
whereas a rr of 0.5 indicates that the exposure factor is
associated with 1/2 the risk of disease (protective factor)

strategies for health for 2015 (bexar county)


1. evidence-based teen pregnancy prevention-reduce teen
birth rate by 15%
2. built environment for active living-reduce physical
inactivity by 10%
3. neighbors engaged in health-community asset mapping,
action plans and active engagement in 10 neighborhoods
4. blood and sexually transmitted disease-reduce
congenital syphilis by 50% and reverse trends in syphilis
incidence

benefits of MAPP
creates a healthy community; increases visibility of public
health; manages change; stronger public health
infrastructure; builds stronger partnerships; builds public
health leadership; creates advocates

Questions to track outcome (MAPP)


How do we track?
How do we affect public policy?
How do we promote a sense of community?
How do we assure access to care?
How do we encourage healthy lifestyles?
How do we provide a safe environment?

pandemic
steady occurrence of disease over a large geographic area
or worldwide, such as malaria

epidemic
refers to the unexpected increase of an infectious disease
in a geographic are over an extended period of time

endemic
diseases that occur at a consistent, expected level in a
geographic area

outbreak
refers to the unexpected occurrence of an infectious
disease in a limited geographic area during a limited period
of time

infectious agent
an organism (virus, Rickettsia, bacteria, fungus, protozoan,
helminth, or prion) capable of producing infection or
infectious disease

reservoirs
the environment in which a pathogen lives and multiplies

portal of exit
means by which an infectious agent is transported from the
host

mode of transmission
method whereby the infectious agent is transmitted from
one host (or reservoir) to another host

portal of entry
means by which an infectious agent enters a new host

host susceptibility
the presence or lack of sufficient resistance to an infectious
agent to avoid prevent contracting an infection or acquiring
infectious disease

direct transmission
implies the immediate transfer of an infectious agent form
an infected host or reservoir to a portal of entry in the
human host through physical contact such as touching,
biting, kissing, or sexual contact

indirect transmission
the spread of infection through a vehicle of transmission
outside the host

fomites
an inanimate object, material, or substance that acts as a
transport agent for a microbe

vectors
can be animals, insects or artropods, and they can transmit
infection through biologic and mechanical routes

susceptible
at risk for contracting an infection or developing infectious
disease; affected by general health status, personal
behaviors, presence of healthy lines of defense, immune
system and immunization status

natural immunity
an innate resistance to a specific antigen or toxin

acquired immunity
derived from actual exposure to the specific infectious
agent, toxin, or appropriate vaccine

active immunity
when the body produces its own antibodies against an
antigen, either as result of infection with the pathogen or
introduction of the pathogen in a vaccine

passive immunity
the temporary resistance that has been donated to the
host through transfusions of plasma proteins,
immunoglobulins, and antitoxins, or transplacentally from
mother to neonate

herd immunity
refers to a state in which those not immune to an infectious
agent will be safe if a certain proportion of the population
has been vaccinated or is otherwise immune

descriptive epidemiology
focuses on the distribution of frequencies and patterns of
health events with groups in a population; examine disease

patterns and other health-related phenomena according to


"person" "place" and "time"

analytic epidemiology
seeks to identify associations between a particular disease
or health problem and its etiology; directed toward finding
answers to the "how" and "why" of health disease to
determine casuality

synergism
concept central to the web of causation model, wherein the
whole is more than the sum of its separate parts
Epidemiology is...
the study of the distribution and determinants of health
and disease in human populations

Person-place-time model
Person: "Who" factors, such as demographic
characteristics, health, and disease status
Place: "Where" factors, such as geographic location,
climate and environmental conditions, political and social
environment
Time: "When" factors, such as times of day, week, or
month and secular trends over months and year

Descriptive epidemiology:
-Study of the amount and distribution of disease
-Used by public health professionals
-Identified patterns frequently indicate possible causes of
disease

Analytic Epidemiology
-Examine complex relationships among the many
determinants of disease
-Investigation of the causes of disease, or etiology

Epidemiological Triangle
Host, Agent, Environment

Agent of Disease (Etiologic Factors)


-Nutritive elements (excesses, deficiencies)
-Chemical agents (Poisons, allergens)
-Physical agents (Ionizing radiation, mechanical)

-Infectious agents (metazoa, protozoa, bacteria, fungi,


rickettsia, viruses)

Host Factors of disease- Intrinsic Factors (Susceptibility, or


Response to Agent)
Genetic, age, sex, ethnic group, physiological state, prior
immunological experience (active, passive), intercurrent or
preexisting disease, human behavior

Environmental Factors of disease- Extrinsic Factors:


influence existence of the agent, exposure, or susceptibility
to agent. This includes...
-Physical environment
-Biological environment such as human populations, flora,
fauna
-Socioeconomic environment such as occupation,
urbanization and economic development, disruption

Wheel Model of Human-Environment Interaction


Page 73
-In the middle: Host (human); this is the genetic core
-Surrounding that in the wheel is Biological environment,
social environment, and physical environment

Web of Causation
illustrates the complexity of relationships among causal
variables for heart disease; page 73

Ecosocial approach
-Emphasizes the role of evolving macro-level
socioenvironmental factors along with microbiological
process in understanding health and illness
-challenges the more individually focused risk factor
approach to understanding disease origins

What are rates?


arithmetic expressions that help practitioners consider a
count of an event relative to the size of the population from
which it is extracted

The numerator of a rate is:


The number of health events in a specified period

The denominator of a rate is:

population in same area in same specified period (e.g., per


day, per week, or per year)

What is the proportion rate multiplied by?


a constant (k); For example, the rate can be the number of
cases of a disease occurring for every 1000, 10,000 or
100,000 people in the population

Morbidity rates
rates of illness (page 75)

What do incidence rates describe?


the occurrence of new cases of a disease or condition in a
community over a period of time relative to the size of the
population at risk for that disease or condition during that
same time period.

What do attack rates document?


the number of new cases of a disease in those exposed to
the disease (ex. food poisoning; denominator is the number
of people exposed to a suspect food, and the numerator is
the number of people who were exposed and became ill.

What are prevalence rates?


All cases of a specific disease or condition at a given time

Those who survive a chronic disease without cure remain


in the:
"prevalence pot"

What is the prevalence pot?


The relationship between incidence and prevalence

What are mortality rates?


routinely collected birth and death rates

Risk refers to:


the probability of an adverse event

Risk factor
-Refers to the specific exposure factor
-Often external to the individual

Attributable risk

-Estimate of the disease burden in a population

Relative risk ratio


Divide the incidence rate of disease in the exposed
population by the incidence rate of disease in the
nonexposed population

Primary prevention:
-when interventions occur before disease development
-Includes health promotion and specific prevention

Secondary prevention:
occurs after pathogenesis; screening and physical
examinations that are aimed at early diagnosis

Tertiary prevention:
focuses on limitation of disability and the rehabilitation of
those with irreversible diseases such as diabetes and spinal
cord injury

Why is it important to identify causality of disease?


to encourage the most effective prevention activities and
develop treatment modalities

What is the purpose of screening?


to identify risk factors and diseases in their earliest stages

What is surveillance?
a mechanism for the ongoing collection of community
health information

Why is surveillance important?


because monitoring for changes in disease frequency is
essential to effective and responsive public health
programs.

Should community health nurses and community


members collaborate? If so, why?
YES! This is so that they can combine epidemiological
knowledge and aggregate-level strategies to affect change
on the broadest scale

Descriptive epidemiology focuses on:

the amount and distribution of health and health problems


within a population

Why are observational studies used?


-Descriptive purposes
-Etiology of disease
-No manipulation by investigator

What are cross-sectional studies also called?


Prevalence or correlational studies

What is the purpose of a cross-sectional study?


It examines relationships between potential causal factors
and disease at a specific time

Why is it impossible for a cross-sectional study to make


causal inferences?
because the study cannot establish the temporal sequence
of events (i.e., the cause preceded the effect)

What does a retrospective study do?


-Compares individuals with a particular condition or disease
with those who do not have the disease
-Data collection extends back in time

What does a prospective study monitor?


a group of disease-free individuals to determine if and
when disease occurs

In a prospective study, cohort...


shares a common experience within a defined time period
Which role of the nurse is most important in achieving the
goal of Healthy People 2020: "To achieve health equity,
eliminate disparities, and improve the health of all
groups"?
A) Advocate
B) Researcher
C) Teacher
D) Leader
A
Rationale: Nurses have a role in seeing that deficiencies in
the health care system are addressed. Caring is the focus

of nursing and in many cases this caring is manifested as


advocacy for the client. Nurses advocate for clients to
achieve health within their own abilities, opportunities, and
social contexts.

The nurse explains to the student nurse that nursing


interventions for families in community-based settings
are similar to those for individuals and involve three
types: cognitive, affective, and behavioral. Which of the
following is considered an affective intervention?
A) The nurse demonstrates to a client with a colostomy
how to change the drainage bag
B) The nurse counsels a client to help her overcome her
grief over the death of a loved one
C) The nurse teaches a client how exercise will have a
beneficial effect on her cardiovascular system
D) The nurse teaches a teenager which foods to choose
as snacks to avoid consuming empty calories
B
Rationale: Affective interventions have to do with feelings,
attitudes, and values, such as occurs with grief counseling.
Cognitive interventions involve the act of knowing,
perceiving, or understanding, such as teaching the effects
of exercise and proper nutrition. Behavioral interventions
involve performing skills and demonstrating behaviors,
such as changing a colonoscopy bag.

A nurse coordinates all the services needed for the family


of an infant born with cerebral palsy. The nurse's efforts
focus on empowering the family to maximize self-care
through health promotion, disease prevention, and
increased continuity of care. What is the term for this
process?
A) Case Management
B) Consultation
C) Discharge Planning
D) Screening
A
Rationale: Case management, also known as care
management or care coordination, involves activities that
enhance the self-care capacities of clients and families by
coordinating care. Discharge planning is an accepted
nursing intervention aimed at the prevention of problems

after discharge. Screening identifies individuals with


unrecognized health risk factors of asymptomatic diseases.
During consultation, the nurse seeks information and
generates solutions to problems through an interactive
problem-solving process.

A community-based nurse is providing care for an elderly


male client and his family in his home. The family
includes a spouse and a daughter who is living at home
as well as a 10-year-old grandson. The nurse plans care
for the client in the context of the family. Which of the
following statements best describes this model of care?
A) This model emphasizes how family structure, function,
developmental stage, and interpersonal interactions
influence the recovery of the client
B) This model focuses on the family as a system as well
as the unit of service; the individual and family are
concentrated on simultaneously
C) This model considers the family as it relates to the
recovery of the individual client; the client is the focus
and the family is the context
D) This model allows the family to be seen as one of the
many institutions or social systems in the community that
interact with other systems for services
C
Rationale: Providing care in the context of the family occurs
when the nurse considers the family as it relates to the
recovery of the client. When the focus is on the family's
potential impact on the recovery of the client, the nurse
considers how family structure, function, developmental
stage, and interpersonal interactions influence the recovery
of the client. When the family is seen as the client, the
nurse focuses on the family as a system as well as the unit
of service. When the family is viewed as a component of
society, this model allows the family to be seen as one of
the many institutions or social systems in the community
that interact with other systems for services.

The nurse caring for families is aware that family


functions are outcomes of family structure and are the
reason families exist. Which of the following is an
example of an affective family function?
A) A father works two jobs to help provide for his family
B) A mother schedules a dental checkup for her

adolescent son
C) A new mother cuddles and sings to her newborn
D) A father arranges a play date for his 5-year old
daughter
C
Rationale: The affective family function is the family's
ability to meet the psychological needs of its members,
such as bonding with a newborn. The economic function of
the family involves the allocation of adequate resources for
the family members. The providing for health care and
physical necessities function is involved in scheduling
checkups. Arranging a play date is an example of the
socialization function.

The nurse providing care in a community-based health


care setting knows that prevention of disease and injury
is a key component of the practice. Which of the following
interventions best exemplifies disease prevention on the
secondary level?
A) A nurse screens school-aged children for scoliosis
B) A nurse teaches new mothers how to bathe their
infants
C) A nurse coordinates home care for an elderly client
with dementia
D) A nurse teaches stress management to clients
recovering from heart attacks
A
Rationale: Primary disease prevention involves prevention
of the initial occurrence of the disease or injury, such as
teaching hygiene for newborns. Secondary disease
prevention is early identification of disease or injury with
prompt intervention to limit its effects. This includes
screening for disorders such as scoliosis. Teaching stress
management following MI and coordinating home care are
examples of tertiary disease prevention to halt further
disease progress and meet help the person meet their
maximum potential.

The nurse working in community-based nursing explains


to the student nurse how certain factors affect the
delivery of nursing care in the community. Which of the
following statements regarding communities is accurate?
A) Social determinants of health are shaped by economics

and politics
B) Culture plays a minor role in the overall character of
the community
C) Community boundaries cannot limit the services
available to individuals
D) Social systems have little or no impact on a
community's health
A)
Rationale: Social determinants of health are defined as the
circumstances in which people are born, grow up, live,
work, and age, and the systems put in place to deal with
illness. These circumstances are shaped by economics,
social policies, and politics. Social systems have an impact
on a community, and consequently the health of that
community. Culture contributes to the overall character of a
community and, in turn, influences its health needs. A
community is defined by boundaries that often determine
what services are available to individuals.

A nurse caring for migrant workers of Hispanic descent in


a community clinic practices etic care that is acceptable
and successful for these particular clients. Which of the
following statements best describes etic care?
A) It is professional care based on an understanding of
the client's cultural background
B)It is informal care directed an involving the client in selfcare based on his or her own culture
C) It is allowing the local customs of the client to dictate
the type of care that is provided
D) It involves using cultural blindness to treat clients from
diverse background equally and fairly
A
Rationale: "Etic" refers to the outsider's views and values
about a phenomenon. Etic care involves using an emic
(insider's) understanding of the client's beliefs about health
issues to coordinate professional (etic) care that is
acceptable to the client.

Nurses preparing for a career in community-based


nursing should be aware of the current trends affecting
the future of nursing education and how these trends will
affect their practices. Which of the following accurately
describes one of these trends?

A) There are changing demographics and decreasing


diversity in communities
B) There is a shift to hospital-based care and increased
complexity of client cases
C) There are lower costs of health care using managed
care practices
D) There are current nursing shortages and workforce
development continues
D
Rationale: Among other factors, the future of nursing
education is affected by: (1) current nursing shortages,
opportunities for lifelong learning, and work development;
(2) changing demographics and increasing diversity; (3)
higher costs of health care and the challenges of managed
care; and (4) a shift to population-based care and the
complexity of client care.

A nurse initiates a regular physical activity program for


residents of a nursing home. Which of the following
statistics accurately portrays the incidence of existing
health problems in the United States that can be
influenced by regular physical exercise?
A) 10 million people have type 2 diabetes
B) 8.5 million people have coronary heart disease
C) 50 million people have high blood pressure
D) Over 25 million people are overweight
C
Rationale: Regular physical activity reduces the risk of
heart disease, cancer, and diabetes. In the United States,
50 million people have high blood pressure, 13.5 million
people have coronary heart disease, more than 60 million
people are overweight, and 8 million people have type 2
diabetes.

Upon graduation, a nurse chooses to practice communitybased nursing instead of working in an acute care setting.
Which of the following statements best defines
community-based nursing?
A) It is a "flowing" kind of care that does not necessarily
occur in one setting
B) It does not have a defined philosophy but rather is
defined by the setting itself
C) It is defined by the level of academic preparation

needed to preform the skills


D) It is all about where the nurse practices as opposed to
how the nurse practices
A
Rationale: The emphasis of community-based nursing is a
"flowing" kind of care that does not necessarily occur in
only one setting. Community-based nursing care is directed
toward specific individuals and families within a community
and is designed to meet the needs of people as they move
between and among health care settings. It is all about how
the nurse practices, not where the nurse practices.
Community-based nursing has a defined philosophy of
practice that requires specific knowledge and skill. It is not
defined by the setting or level of academic preparation
needed to perform the care.

The nurse is preparing to discharge a client with a spinal


cord injury from an acute care facility in preparation for a
transfer to a rehabilitation facility. Which of the following
statements accurately describes a focus of this process?
A) It is one dimensional and prescribed in facilitating
transfer from one setting to another
B) It identifies services and referrals that may be needed
and passes this information along to the case manager
C) It is unique in that it does not follow the nursing
process, but instead focuses entirely on preventive care
D) Its focus is on the client functioning as an individual
and the client's unique medical needs
A
Rationale: Discharge planning is one dimensional and is
prescribed in facilitating transfer from one setting to the
other. It follows the nursing process and includes
assessment, planning, and interventions. Discharge
planning focuses on the client and family needs and
abilities. The interventions involve making referrals and
giving the client and family important phone numbers,
names, and community services in writing, not just a
referral to a case manager.

The nurse is caring for a male client with HIV who is living
unmarried with another male and has two adopted
children. How would the nurse document this client's
family structure?

A) Nuclear Family
B) Nuclear Dyad
C) Single-Parent Family
D) Multigenerational Family
A
Rationale: A nuclear family consists of a married couple
with children, or unmarried, heterosexual, or same-sex
couples with children. A nuclear dyad is a couple, married
or unmarried; heterosexual or same sex. A single-parent
family is one adult with children, and a multigenerational
family is any combination of these family structures.

A nurse is designing a physical activity program for a 48year-old male client who has been diagnosed with mild
hypertension. Which of the following is an appropriate
long-term goal for this client?
A) Jog for 15 minutes and weight train for 15 minutes, 3
days a week
B) Walk briskly for 30 minutes most days of the week
C) Bicycle for 1 hour every day of the week
D) Walk briskly for 30 minutes every other day
B
Rationale: Moderate physical activity is recommended for
30 to 45 minutes, 3 to 5 days a week as an initial goal. An
appropriate long-term goal is 30 minutes of moderateintensity physical activity all or most days of the week.

Which of the following nursing skills is most imperative


when practicing community-based nursing?
A) A nurse attends and in-service on using new IV and
infusion equipment
B) A nurse delegates nursing care appropriate for an LPN
to perform
C) A nurse collaborates with a physical therapist treating
a client with paraplegia
D) A nurse institutes a new format for documenting client
care in the field
C
Rationale: Although all these skills are important, the most
critical skills needed by a nurse practicing communitybased nursing are highly developed assessment skills,
effective communication skills, collaboration with the

interdisciplinary team, and working with culturally diverse


clients.

A nurse demonstrates how to change the bandages on


the stump of a client with a new below-the-knee
amputation. This is an example of learning according to
which of the following learning domains?
A) Cognitive
B) Psychosocial
C) Psychomotor
D) Affective
C
Rational: Psychomotor learning consists of acquired
physical skills that can be demonstrated, such as changing
bandages on a stump. Cognitive learning involves the
storage and recall of new knowledge and affective learning
refers to the feelings, values, and attitudes that affect
learning. Psychosocial is a factor for learning readiness, not
a learning domain.

The nurse is counseling a 24-year-old working mother


who has a 2-year-old and a newborn who was diagnosed
with cerebral palsy. The mother states that she is
overwhelmed with trying to care for her family as well as
schedule work and take care of her home. The nurse
formulates a diagnosis based on which of the following?
A) Role overload
B) Role stress
C) Role conflict
D) Role flexibility
A
Rationale: Role strain or overload arises when an individual
is confronted with too many role responsibilities at one
time. Role conflict occurs when the demands of one role
conflict with or contradict another. Role flexibility occurs
when other family members take on the roles of a family
member in times of need. Role stress is a distractor.

The nursing process is an important tool to guide the


nurse in thinking through assessment, planning,
implementation and evaluation of the nursing care plan.
Which of the following describes how the nursing process
is used differently by nurses practicing community-based

nursing versus nurses practicing acute care nursing?


A) Client teaching is more important in the hospital
setting than in the community setting because of
shortened hospital stays
B) In community-based nursing, the nursing process is a
mutual endeavor used to plan care, and also to develop
therapeutic relationships with the client's family members
C) Nursing diagnoses are critical in the hospital setting,
but are not defined in community-based nursing
D) In the hospital setting, nurses follow a plan of care
from the health care provider, which does not occur in the
community setting
B
Rationale: Nurses in both settings use clinical decisionmaking skills and clinical reasoning to identify what to
assess in the client, family, and community. In contrast to
care typically provided in hospitals, in the community
setting the nursing process guides the nurse to a mutual
care plan and helps the nurse to develop therapeutic
relationships with the client, family, and caregiver.
Community-based nurses may follow a plan of care from a
health care provider, but they usually need to modify the
care plan for the individual needs of the client and family.
Client teaching is important in both settings and occurs
with the client and family and/or caregiver. Nursing
diagnoses or problem statements are formed for clients in
both settings and interventions are identified that are
reasonable and acceptable to all parties involved in the
planning process.

A nurse conducting a parenting class teaches parents the


leading causes of death in different age groups. Which of
the following age groups has the highest rate of death
related to cancer?
A) 1-4 year olds
B) Under 1 year-old
C) 15-24 year olds
D) 5-9 year olds
D
Rationale: The leading causes of death in 5- to 9-year-olds
are unintentional injuries, cancer, and congenital
anomalies. For infants less than 1 year of age, the leading
causes of death are congenital anomalies, disorders related

to premature birth, and SIDS. In the 1- to 4-year-old group,


the leading causes of death are unintentional injuries,
congenital anomalies, and homicide. In the 10- to 14-yearold group, the leading causes of death are unintentional
injuries, cancer, and homicide. In the 15- to 24-year-old
group the leading causes of death are unintentional
injuries, homicide, and suicide.

A nurse promotes public health interventions at all


possible levels of practice in the community. Which of the
following is the term for this public health practice?
A) Individual-focused
B) Systems-focused
C) Population-based
D) Community-focused
C
Rationale: Public health interventions are population-based
if they consider all levels of practice, including the
community, the systems, and the individuals or families in
that system who are known to be at risk. Communityfocused interventions change community norms, attitudes,
awareness, practices, and behaviors. Systems-focused
interventions change organizations, policies, laws, and
power structures. Individual-focused interventions change
knowledge, attitudes, beliefs, practices, and behaviors of
individuals.

The nurse practicing in a community public health clinic


sees the effects of fragmentation of care on client health
care. Which of the following is an example of a nursing
intervention to help address fragmented care?
A) Encourage health care practitioners to keep formal and
informal care separate
B) Encourage clients to use home health care for more
intense health care needs
C) Encourage clients to routinely use a medical home
D) Encourage clients to use a variety of providers for
health care needs
C
Rationale: Clients are increasingly seen by a large variety
of providers in many organizations and agencies, often
resulting in fragmentation of care. Encouraging routine use
of a medical home is one attempt to address fragmented

care as opposed to using a variety of providers for health


care needs. The use of inpatient facilities for briefer. more
intense encounters should be encouraged. Continuity of
care is often accomplished by combining formal and
informal care.

A home health care nurse is performing an assessment of


an 18-month-old child. Which of the following normal
parameters for the development of hearing and its
assessment should the nurse keep in mind during the
screening?
A) Parents should not be concerned about their child's
hearing until age 3, when hearing is fully developed
B) Parents should be advised to tell the nurse if their
infant is not reacting to loud noises by the age of 2
C) The child's speech patterns should be assessed
separately, since hearing impairment has no impact on
the development of speech
D) Children are not routinely screened for hearing until
three years-old; however, a parent's observations could
indicate a possible hearing problem
D
Rationale: Infants and toddlers are not routinely assessed
for hearing until 3 years of age. However, a parent's
observations may indicate the possible presence of hearing
or vision impairment. By the age of 1, infants should react
to loud noises and hearing their name called. By age 2,
children should repeat words and enjoy games like peek-aboo. Speech should also be assessed because hearing
impairments often become apparent when the child begins
to talk.

A nurse is counseling a 65-year-old female client who


expresses a desire to quit smoking. Which of the following
would be an appropriate response to this client?
A) "If you smoke you have a 50% greater heart disease
death rate than nonsmokers"
B) "Older smokers are more likely to stay off cigarettes
once they quit than younger smokers"
C) "In some cases, ex-smokers who have had a heart
attack can cut their risk of having another one by 25%"
"Since you have been smoking most of your life, it's not
important to stop now since the benefits would be
minimal"

B
Rationale: Older smokers are more likely to stay off
cigarettes once they quit than younger smokers since they
know more about the short-term and long-term benefits of
quitting. It is always good to quit smoking at any age, and
ex-smokers live longer and healthier lives. People who
smoke have a 70% greater heart disease death rate than
do nonsmokers. In some cases, ex-smokers who have had a
heart attack can cut their chance of having another one by
50%.

Most cultures combine orientation to the past, present,


and future, but one orientation usually dominates. For
which of the following clients would teaching strategies
for preventive care be most accepted based on his or her
orientation to the future?
A) A Native American male athlete prone to stress
fractures
B) A white American female lawyer who is experiencing
job stress
C) An African American male accountant who is
overweight
D) A Hindu business executive who is experiencing
migraines
B
Rationale: Traditionally, American culture is future oriented;
however, Native American and Hindu culture tend to focus
on the past while African American culture focuses on the
"now" and day to day activities. When discussing
preventive care, persons without a future orientation
should be approached differently than those with future
orientation.

The nurse is counseling a couple involved in the divorce


process. Which of the following emotional responses is
common during the separation stage of divorce?
A) Mourning the losses associated with a separating
family
B) Negotiating viable arrangements for all family
members
C) Revealing the fact that the marriage has major
problems

D) Working on emotional recovery by overcoming hurt,


anger, or guilt
A
Rationale: During the separation stage of divorce the family
mourns the losses associated with the separation and
works on resolving attachment to the spouse. In the stage
of planning the dissolution of the family system, the family
negotiates viable arrangements for all family members. In
the divorce stage, the family continues working on an
emotional recovery by overcoming hurt, anger, or guilt. In
the stressors leading to marital differences stage, the
family reveals the fact that the marriage has major
problems.

Nurses in community-based settings on many occasions


provide teaching plans to clients with sensory deficits.
Which of the following interventions reflects a
recommended guideline when teaching sensory-impaired
clients?
A) A nurse asks a client with a hearing aid to remove the
hearing aid when providing teaching
B) A nurse speaks to a visually impaired client when
approaching him from the front
C) A nurse directs questions to the family of a client who
has aphasia when taking a nursing history
D) A nurse asks other people in the room to remain silent
when approaching a visually impaired client
B
Rationale: For a visually impaired client, the nurse should
speak to the client when approaching him and avoid
speaking from behind the client. The nurse should also ask
other people to introduce themselves to allow the client to
hear people's voices. The nurse should always ask if the
client wears a hearing aid and if it is working properly. The
nurse should provide some means of conversation (such as
a computer or letter board) for a client who has aphasia
(language deficit).

A nurse caring for clients in a woman's health clinic


advises them on the recommended frequency of Pap
smears, mammograms, and fecal occult blood testing.
Which of the following is a guideline for these tests?
A) Women who have been sexually active should have a

Pap test every three years except if they have genital


warts, multiple partners, or abnormal Pap tests
B) Women over age 75 with a history of normal Pap
smears may stop having Pap tests after consulting with
their health care practitioner
C) Starting at age 50, fecal occult blood testing should be
done every other year in combination with other
screening tests recommended by the practitioner
D) Women over 60 years old should have a mammogram
every 2 years unless a family history suggests other wise
A
Rationale: Women who have ever been sexually active
should have a Pap test every three years except if they
have genital wars, multiple partners, or abnormal Pap tests
in the past. Women over age 65 with a history of normal
Pap smears may stop having Pap tests after consulting with
their health care practitioner. Women over 50 years old
should have a mammogram every two years unless their
family history suggests otherwise. Starting at age 50, fecal
occult blood testing should be done every year in
combination with other screening tests recommended by
the practitioner.

Which of the following is an example of an activity aimed


at providing primary disease prevention?
A) Counseling a rape victim
B) Performing mental health screening
C) Providing well-child care
D) Teaching breast self-examination techniques
C
Rationale: Providing well-child care is an example of
primary disease prevention to prevent the initial
occurrence of a disease or injury. Teaching breast selfexamination and performing mental health screening are
examples of secondary prevention activities, and
counseling a rape victim is an example of tertiary disease
prevention.

One of the components of community-based care is selfcare. Which of the following nursing interventions is the
best example of promoting self-care in the community
setting?
A) A nurse arranges for a client who smokes a pack of

cigarettes a day to attend a smoking cessation program


in his neighborhood
B) A nurse locates physical therapy facilities for a client
who is recovering from hip replacement surgery
C) A nurse considers the cultural preferences of a client
who has diabetes when helping to prepare a meal plan for
the client
D) A nurse helps a client with end-stage liver cancer who
is filling out an advanced directive to make informed
health care decisions
D
Rationale: Empowering individuals to make informed health
care decisions is an essential component of self-care. One
way of empowering individuals is through advance
directives that allow clients to participate in decisions
about their care or refuse treatment. Arranging a smoking
cessation program for a client is an example of preventive
health care. When providing care within the context of the
community, the nurse considers the culture, values, and
resources of the client, the family, and the community.
Collaborative care among health care professionals is an
essential component of community-based care rather than
self-care. Arranging for needed services is an example of
this continuity of care.

A nurse caring for clients in a community health care


clinic assesses clients on a daily basis for various health
risks. Which of the following is a recommended guideline
for these assessments?
A) The nurse should immediately refer clients suspected
of being a victim of intimate partner violence to a
professional for counseling
B) The nurse should recommend weight control if the
client has a BMI of 30 or more or a BMI of 25-29 and two
or more weight-related health problems
C) The nurse should not assess sexual health unless the
client states a concern about having a sexually
transmitted disease
D)The nurse should not assess the risk for suicide for a
client suspected of having depression to prevent planting
the idea in the client's mind
B
Rationale: The nurse should approach the topic of weight
control for clients who have a BMI of 30 or more and two or

more weight-related health problems. The nurse should not


be afraid of assessing a client for sexually transmitted
diseases or suicide risk. The nurse should also use refined
assessment skills to assess for IPV.

The nurse providing care for culturally diverse clients


acknowledges the biological variations for each racial or
ethnic group in the community. Which of the following is
an example of a biological variation as a factor in cultural
assessment
A) Body language
B) Personal space
C) Time orientation
D) Skin color
D
Rationale: Biological variations are the biological
differences among racial and ethnic groups, which include
skin color, and physiologic variations, such as specific
disease processes. Body language is an aspect of
communication. Time orientation and personal space are
separate phenomenon of cultural assessment.

A nurse is teaching parents about the recommended


hours of sleep per night for their infants. Which of the
following statements accurately describes the amount of
sleep needed by a particular age group?
A) Most toddlers sleep around 8-10 hours per day
B) Most school-aged children need 8 hours of sleep per
night
C) Newborns generally sleep 16-20 hours a day
D) Most adolescents need 7 hours of sleep a night
C
Rationale: Newborns generally sleep 16 to 20 hours per
day. Toddlers sleep around 10 to 13 hours per day. Most
school-age children need 10 hours of sleep at night and
adolescents need 8 to 9.5 hours of sleep at night.

A nurse is providing free cholesterol screening for adults


in a neighborhood clinic. Which of the following clients is
most likely to have higher cholesterol?
A) A person who has just lost weight
B) A female who is going into menopause

C) A male in his 30's


D) A female with a musculoskeletal disorder
B
Rationale: People at a higher risk for high cholesterol levels
are middle-age men, women just before menopause, and
anyone who has just gained weight.

A home health care nurse is performing a psychosocial


assessment of a new client who was discharged following
delivery of a newborn who has Down syndrome. Which of
the following questions is the best example of a
psychosocial assessment question?
A) Who makes the decisions in your home?
B) Do you have a smoke detector on every level of your
home?
C) Are there any family rituals passed down from
generation to generation?
D) Who provides you with strength and hope?
A
Rationale: Psychosocial assessments are performed to
understand the client in the context of the family. An
aspect of this type of assessment is asking about the family
structure and who makes the decisions in the family. Family
rituals are an aspect of cultural assessment. Assessing for
smoke detectors is an environmental assessment. Asking a
client who provides strength and hope is a spiritual
assessment.

A client has a network of health care providers that


deliver services at a lower fee in return for prompt
payment at a pre-negotiated price. This is an example of
which of the following types of group insurance plans?
A) Health Maintenance Organization (HMO)
B) Private Insurrance Plan
C) Third-Party Payment Plan
D) Preferred Provider Organization (PPO)
D
Rationale: Preferred provider organizations allow a network
of providers to deliver services at a lower fee in return for
prompt payment at pre-negotiated rates. Health
maintenance organizations are prepaid, structured
managed systems in which providers deliver a

comprehensive range of health care services to enrollees.


Private insurance may obtained through large nonprofit
organizations or from small, private, for-profit insurance
companies as third-party payment.

A home health care nurse visits the home of an 82-yearold female client who is receiving nursing care and
physical therapy following discharge from the hospital.
The client has right-sided paralysis following a TIA. Upon
assessment, the nurse finds that the client has lost five
pounds since the last visit a week ago. The client is living
alone with daily visits from her two children. Which of the
following would be the most important assessment
question for this client?
A) Are your children still visiting you daily?
B) Who is preparing your meals for you?
C) Are you able to walk to the kitchen to get a snack?
D)How has your physical therapy affected your appetite?
B
Rationale: Assessment is a continuous process designed to
collect information used for an immediate intervention or
as a foundation for additional assessments at a later time.
After assessing this client for weight loss, the nurse would
make further assessments to determine if the client needs
help with grocery shopping, meal planning, and cooking
meals. Tracking the client's eating pattern and intervening
where appropriate are the most important assessments in
this situation.

In which of the following facilities would admission most


likely include an assessment focusing on functional
abilities and orientation to new surroundings?
A) Nursing home
B) Acute care setting
C) Physician's office
D) Psychiatric office
A
Rationale: In the nursing home, the nursing history and
assessment focuses on functional abilities and orientation
to new surroundings. In an acute care setting, the
admission focuses on a complete nursing history, vital
signs, and other physical assessments. In the physician's
office, admission assessment focuses on the reason(s) for

seeking medical care. The psychiatric facility admission


focuses on introduction and mental health evaluation, as
well as orientation to room and unit.

A nurse recommends that a pregnant woman consume


600 ug of folic acid per day. This nurse is providing a
preventive intervention for what infant complication of
pregnancy?
A) Low Birth Weight (LBW)
B) Fetal Alcohol Syndrome
C) Failure to Thrive
D) Neural Tube Defects
D
Rationale: Approximately 50% of all neural tube defects
may be prevented with adequate consumption of folic acid
in the first trimester of pregnancy. A pregnant woman who
smokes is more likely to have an LBW infant. Failure to
thrive has various causes, and abstaining from alcohol is a
preventive measure for fetal alcohol syndrome.

The nurse who is caring for clients in a physician's office


knows that which of the following clients is engaging in
the activity that most negatively affects health?
A) A client who is 20 pounds overweight
B) A client who regularly has three alcoholic beverages
before dinner
C) A client who does not engage in physical activity
D) A client who smokes a pack of cigarettes a day
D
Rationale: Smoking remains the health indicator that most
negatively impacts health. Drinking, living a sedentary
lifestyle, and overeating are also risk factors for health
disorders, but smoking has the greatest impact on health.

A nurse is assessing a family structure using the family


systems theory. Which of the following is true regarding
the principles of this family social system framework?
A) Family systems tend to move from a state of
equilibrium to allow for developmental changes in the
family members
B) The family system has a boundary that is selectively
semi-permeable according to the family's wishes

C) The family with closed boundaries is more apt to use


community services than a family with semi-permeable
boundaries
D) Nurses may assist families to return to a state of
equilibrium, but the system does not allow for families to
move to a higher level of health
B
Rationale: The family system has a boundary that is
selectively permeable according to the family's wishes, so
items such as material goods, people, and information are
allowed in or out according to the perceived needs of the
family. The family with closed boundaries is less apt to use
community services than the family with semi-permeable
boundaries. Family systems tend to move to a state of
equilibrium, and nurses may assist families to equilibrium
and sometimes to a higher level of health.

A nurse is caring for an 87-year-old female client who is 5'


7" and weighs 140 pounds. What is the client's Body Mass
Index (BMI)?
A) 21.9
B) 26.9
C) 25.0
D) 23.5
A
Rationale: BMI = weight (pounds) x 703 divided by height
squared (inches squared). In this case the client is:
140 lbs. x 703 = 98420 lbs.
Height squared = 67 inches x 67 inches = 4489 inches
98420 lbs. 9 4489 inches = 21.9 BMI

Nurses working in community-based settings often


concentrate on preventive care for their clients. Which of
the following is an example of an intervention based on
primary prevention?
A) A nurse initiates a program to start free blood pressure
screening in the neighborhood
B) A nurse promotes a new exercise program designed for
older adults in the community
C) A nurse performs range-of-motion exercises for a client
with debilitating arthritis
D) A nurse recommends a colonoscopy for a client with a
family history of colorectal cancer

B
Rationale: Primary prevention is used to prevent the initial
occurrence of a disease, such as preventing obesity or
heart disease through exercise. Screening for hypertension
and recommending a colonoscopy are secondary
preventive measures. Performing range-of-motion exercises
to improve the mobility of a client with arthritis is an
example of tertiary prevention.

A nurse writes a proposal to fund and staff a mobile


health clinic in a community with an at-risk low-income
population. This project is an example of which of the
following public health interventions?
A) Advocacy
B) Coalition Building
C) Outreach
D) Case Management
C
Rationale: Outreach locates populations of interest or at
risk and provides information about the nature of the
concern, what can be done about it, and how services can
be obtained. Case management is working with individual
clients and families to optimize self-care capabilities.
Advocacy is pleading someone's case or acting on
someone's behalf. Coalition building is promoting and
developing alliances among organizations for a common
purpose.

The nurse working with families in the community uses


Maslow's Hierarchy of Needs theory when assessing
family function. Which of the following is an example of
the lowest level of needs that must be met before the
person can move on to higher-level needs?
A) A mother breastfeeds her infant
B) A mother goes back to school to obtain her GED
C) A father praises his son for making his school's Dean's
List
D) A father makes sure his daughter wears a helmet when
riding a bike
A
Rationale: The basic physiological needs (food, fluids,
shelter, sleep, oxygen) must be met first. Safety (bicycle

helmet), self-esteem (parental praise), and selfactualization (attaining a GED) are higher-level needs that
can be met after physiological needs are met.

Community health care nurses provide teaching to clients


based on three levels of prevention. Which of the
following is an example of client teaching related to the
secondary level?
A) A nurse teaches the parents of a newborn who
experienced RDS to place their baby on his back to sleep
to minimize the risk of SIDS
B) A nurse provides information on anorexia to parents of
a teenager diagnosed with the disorder
C) A nurse volunteers at a free mobile clinic to immunize
vulnerable populations in the community against
influenza
D) A nurse refers a client with Parkinson's disease to a
support group in the community
A
Rationale: Teaching occurs at all levels of prevention.
Teaching primary prevention involves preventing the initial
occurrence of disease or injury (e.g., giving immunizations).
Teaching secondary prevention targets early identification
of, and intervention for, a health condition (e.g., SIDS).
Tertiary prevention teaching involves attempts to restore
health and facilitate self-care management and coping with
a health condition (e.g., encouraging support groups and
providing information on eating disorders).

A nurse is caring for an 80-year-old male client who


admits drinking a six-pack of beer a night to "help with
the loneliness." The nurse should screen for what other
co-morbid condition related to this client's alcohol use?
A) Depression
B) Diabetes
C) High-risk behaviors
D) Tobacco use
A
Rationale: Because of the comorbidity of alcohol abuse and
depression, the nurse should screen for alcohol abuse in all
cases where depression is expected.

A nurse is assessing a frail elderly male client for


placement using the Blaylock Discharge Planning Risk
Assessment Screen and gives the client a score of 15.
What would be the nurse's next intervention related to
this score?
A) Discharge the client to his home
B) Refer the client to the discharge planning coordinator
C) Refer the client to a social worker
D) Transfer the client to an assisted living facility
B
Rationale: On the Blaylock Discharge Planning Risk
Assessment Screen, a score of 11 to 19 places the client at
risk for extended discharge planning. A score of 10 means
the client is at risk for home care resources. A score over
20 means the client is at risk for placement other than
home.
If the score is greater than a 10 refer the patient to the
discharge planning coordinator or discharge planning team

The nurse caring for clients in a nursing home knows that


falls are a leading cause of death among the elderly.
Which of the following statistics accurately states the
incidence and consequences of falls in the elderly?
A) In the United States, 1 of every 3 people over age 65
falls each year
B) Half of people over age 75 who fall and fracture a hip
die within 2 years
C) Exercise in the elderly is not recommended due to the
risk for falling
D) By 2020 the cost of fall injury is expected to rise to 50
billion dollars
A
Rationale: In the United States, one of every three people
over age 65 falls each year. Half of people over age 75 who
fall and fracture a hip die within a year. By 2020 the cost of
fall injury is expected to rise to 32 billion dollars. Regular
exercise in the elderly is recommended, as it can
strengthen muscles and stamina and prevent osteoporosis
and consequent falls.

A nurse is caring for a 45-year-old client from Japan who


underwent a stent placement following a myocardial

infarction. The client only speaks Japanese and the nurse


calls in an interpreter to work with them. Which of the
following is a recommended guideline for working with
interpreters?
A) Ask several questions at once and allow the interpreter
to convey them to the client
B) If possible, a relative of the client should be asked to
interpret
C) In possible, an interpreter of the same gender as the
client should be used
D) Request that the interpreter translate exactly what is
said, without interrupting
C
Rationale: In general, an interpreter of the same gender as
the client is preferred. The nurse should ask one question
at a time and allow the interpreter time to convey the
question. Qualified professional interpreters instead of
relatives should be used. The interpreter should be invited
to ask for clarification as needed, even if it means
interrupting.

The nurse working in the community-based setting


follows the nursing process when providing client
teaching. Which of the following steps of the learning
process provides for reimbursement of the teaching
provided?
A) Evaluation
B) Documentation
C) Planning
D) Assessment
B
Rationale: Documentation of teaching is important as a
legal record, as communication of teaching and learning for
other health care professionals, and for determination of
eligibility for care needed and reimbursement for care and
teaching provided.

A school nurse is teaching safety issues in a lecture for


parents of school-age children. Which of the following
topics would the nurse describe as having the biggest
impact on preventing loss of life and injury in children?
A) Using appropriate automobile restraints

B) Learning basic life-saving skills


C) Using smoke detectors in the home
D) Locking up poisonous substances
A
Rationale: Although all these topics are important to cover,
the one topic that prevents the most loss of life and injury
is using appropriate restraints when riding in an
automobile. Child safety seats reduce the risk of death in
passenger cars by 71% for infants and 54% for preschool
children.

The nurse explains to the novice nurse how communitybased nursing differs from nursing care provided in
hospital settings. Which of the following statements
accurately describes one of these differences?
A) In the community setting, the nurse is in charge of all
aspects of client care whereas in the hospital setting the
client directs the care
B) In the community setting, the nurse is the primary
facilitator of self-care as opposed to being solely a care
provider
C) In the hospital setting, a holistic assessment is
facilitated by the collaboration of many professionals,
which is not available in the community setting
D) In the community setting, the client's environment
must be changed to facilitate care, which is not an issue
in the hospital setting
B
Rationale: Because the client and family are involved in
nearly all aspects of care the majority of the time, the
community-based nurse is primarily a facilitator of self-care
as opposed to being the sole care provider. In the
community setting, the delivery of care must be considered
within the family and environment. A holistic assessment
often occurs through the collaboration of many
professionals.

A nurse asks a homebound client what type of activities


she enjoys to determine if these activities can be brought
to the client. Which of the following aspects of the client's
environment is the nurse assessing?
A) Physiologic and survival needs
B) Self-esteem and self-actualization needs

C) Love and belonging needs


D) Safety and security needs
B
Rationale: An environmental assessment assesses for selfesteem and self-actualization needs, such as activities the
client enjoys. An example of a love and belonging need is
having a friendly, competent caregiver. Physiologic and
survival needs are physical needs that need to be met,
such as food and shelter. Safety and security needs include
mobility and fall prevention.

A client who is being discharged from an acute-care


facility sits down with the nurse and agrees how, when,
and where outpatient treatments will occur. What is the
term for this type of agreement?
A) Concordance
B) Collaboration
C) Adherence
D) Self-efficacy
A
Rationale: Concordance is an agreement between the
nurse and the client about whether, when, and how
treatments will occur. Adherence refers to the client
following the prescribed treatment. Self-efficacy refers to
the ability of the client to influence events that affect his or
her life. In the collaborative process, the client chooses a
goal, and the nurse and client negotiate a specific plan to
reach that goal.

A family nurse assesses a family to determine its


developmental stage. The nurse notes that the family's
first child is moving out of their home. What
developmental stage does this represent?
A) Aging family
B) Middle-aged parents
C) Launching center
D) Teenage
C
Rationale: The launching center stage of the family life
cycle occurs from when the first to the last child leaves
home. The "teenage" stage occurs when the oldest child is
13 to 20 years old. The middle-aged parents stage occurs

from empty nest (when no children are living at home) to


retirement. The aging family stage occurs from retirement
to moving out of home.

The nurse practicing in a community setting is aware that


as new groups enter the community, acculturation may
occur. Which of the following is an example of this
concept?
A) A Portuguese family moves into a neighborhood that is
predominantly Portuguese to continue their cultural
practices
B) An Asian couple refuses to allow their children to play
with the neighborhood children
C) The children of a traditional Muslim family chooses to
practice the norms and values of the community as a
whole
D) An Ethiopian man who came to America on a work visa
learns to take the bus to work
D
Rationale: Individuals within a group may adhere to the
traditional culture, as occurs with the Portuguese and Asian
family. Or, they may choose to adapt to the dominant
culture in varying degrees through acculturation, as is seen
with the Ethiopian man. The children of the Muslim family
have assimilated into the existing culture.

A nurse is teaching elderly clients in a senior center how


to modify their homes to prevent falls and promote safety.
Which of the following is a recommended guideline when
providing community-based teaching to the older learner?
A) Face the client and speak in a low, slow voice so lipreading is possible
B) Meet in a quiet, dimly lit room where there is no
background noise
C) Encourage dependent decision making to ensure
family and/or caregiver support
D) Relate new information to the current time frame as
opposed to past experience
A
Rationale: The nurse should face the client and speak in a
low, slow voice so lip-reading is possible. The nurse should
relate new information to past experiences if possible and
encourage independent decision making to support ego

integrity. The nurse should meet the client in a quiet, welllit room with no background noise.

Nurses use the assessment step of the nursing process


when planning client care. Which of the following best
describes the focus of client assessment in the
community-based setting?
A) Providing ongoing monitoring of acute conditions and
planning appropriate nursing interventions
B) Providing a baseline to evaluate physiologic,
psychological, and functional capacity of the client
C) Ensuring continuity of discharge planning based on the
functional capacity of the client
D) Determining the pathology of the client's disease state
and initiating therapeutic interventions
B
Rationale: In the hospital and community setting,
assessment is a dynamic, ongoing method that uses
observations and interactions to collect information,
recognize changes, analyze needs, and plan care. In the
community, the nurse uses assessment to provide a
baseline to evaluate the physiologic, psychological, and
functional capacity of the client and identify environmental
factors that may affect the client's health status. In the
hospital setting, assessment is used to provide ongoing
monitoring of acute conditions and plan appropriate
interventions, as well as to ensure continuity of discharge
planning. Physicians primarily use assessment to
determine pathology and appropriate therapeutic
interventions.

The nurse researches current health trends and statistics


in the United States to identify vulnerable populations.
Which of the following is a critical indicator of the health
of a population?
A) Elderly poverty rates
B) Infant mortality rates
C) Child poverty rates
D) Percentage of individuals with chronic diseases
B
Rationale: Infant death is a critical indicator of the health of
a population because it reflects the overall state of
maternal health, as well as the quality of and access to

primary health care for pregnant women and infants. Infant


mortality in the United States ranks the highest among
industrialized nations.

A nurse working in a community health care clinic travels


around the community to identify data related to the
people, places, and social systems that define that
community. The nurse uses this information to improve
the health of the individuals using the clinic. What type of
assessment tool is this nurse using?
A) Informant interviews
B) Participant observations
C) Windshield survey
D) Existing data
C
Rationale: A windshield survey is designed to assist the
nurse traveling the neighborhood to identify data related to
the people, places, and social systems that define that
community. This information may help identify trends,
stability, and changes affecting the health of the
community members. Informant interviews involve
interviewing key informants or members of the general
public. During participant observations the nurse observes
formal and informal community activities to determine
significant events and occurrences. Existing data is used to
assess people, places, or social systems.

A female nurse is providing care for a 29-year-old male


Muslim client who fractured his pelvis in an automobile
accident. What dimension of cultural knowledge should
the nurse keep in mind when providing care for this
client?
A) In the Muslim culture, physical touching and expression
is valued and expected
B) In the Muslim culture, close face-to-face conversations
are the norm
C) In the Muslim culture, it is highly distressing for a male
client to be assessed by a male nurse
D) In the Muslim culture, contact between males and
females who are not married to each other is
inappropriate
D
Rationale: In the Muslim culture, contact between unrelated

males and females is forbidden. In Muslim culture, it would


be distressing for a male nurse to care for a female patient,
but not for a male patient. Close face-to-face conversations
are the norm In Middle Eastern cultures, and physical
touching and expression is valued in the Mexican culture.

A nurse is preparing for a next-day home visit of a client


discharged from a birthing center 12 hours after a normal
delivery of a healthy baby girl. Which of the following is a
characteristic of home care that differs from nursing care
in the hospital setting?
A) The focus is on the client individually
B) Family support is critical in achieving client outcomes
C) The resources needed for this client are
predetermined
D) The client will assume a highly dependent role
B
Rationale: In the home setting, family support is critical in
accomplishing client outcomes, whereas in the hospital
setting family support is helpful in accomplishing client
outcomes but not a necessity. The resources for hospital
stays are predetermined, but are highly variable in the
home setting. In the home setting the client assumes an
autonomous role as opposed to a dependent role. The
focus of home care is on the client in the family setting, not
just the client individually in a hospital setting.

A nurse is counseling a 35-year-old male client who has a


BMI of 35 (obese) and who has type 2 diabetes. Which of
the following is the best example of assessing this client's
readiness for a weight-control program?
A) What changes are you ready to make in your eating
habits right now?
B) Are you embarrassed by the way you look and feel
right now?
C) Are you aware that being obese is a risk factor for type
2 diabetes?
D) Do you feel you have lost control over your eating
habits?
A
Rationale: The nurse should ask questions to assess a
client's readiness to control weight, such as what changes
he is willing to make in eating habits or physical activity

level. The nurse should address the client's chief


complaints independent of weight first and avoid
judgmental comments such as "lost control" or
"embarrassed".

A nurse is preparing a pamphlet for distribution in the


community regarding sexually transmitted diseases
(STDs) in teenagers and young adults. Which of the
following is a point that should be covered in this
pamphlet?
A) Most new HIV infections occur in people between 13
-21 years of age
B) Teens and young adults are less likely than others to
have multiple partners
C) Comprehensive sex education should begin in high
school
D) Talking openly about sex with teens makes them more
likely to try it
A
Rationale: The incidence of STDs has skyrocketed and most
new HIV infections occur in people between 13 and 21
years (National Center for HIV/AIDS, 2010). Teenagers and
young adults are more likely than others to have multiple
partners. Talking openly and frankly about sex with teens
may allow them to see how it is a choice to be sexually
active that need not be made at this age. Comprehensive
sex education should begin in primary school.

Nurses accomplish the goals of health promotion by


working with individuals in all types of care settings.
Which of the following is an example of this type of health
promotion?
A) A nurse participates in a campaign to create a law that
all bikers wear safety helmets
B) A nurse volunteers at a free health clinic to provide
immunizations for low-income families
C) A school nurse provides rallies for students to show
how eating healthy and exercising can be "fun" activities
D) A nurse counsels teenagers on methods to prevent
contracting sexually transmitted infections
C
Rationale: Health promotion is the science and art of
helping individuals change their lifestyle to move toward a

state of optimal health. This is accomplished through


creating supportive environments to enhance awareness
and change behavior to support good practices, such as
eating healthy and exercising. Providing immunizations and
counseling about preventing STIs are examples of disease
prevention. Helping to create a helmet law is an example of
a health protection strategy.

A nurse is assessing a 2-year-old male child for vision


disorders. Which of the following examples of assessment
techniques would be appropriate for this client?
A) Ask the mother if the child returns her smile
B) Ask the child if he can see pictures in a book
C) Ask the mother if the child tilts his head to look at
things
D) Observe whether the child's eyes follow the mother
when she leaves the room
C
Rationale: Nurses should assess the vision of infants and
toddlers by asking the mother certain assessment
questions. For a toddler, the nurse should ask the mother if
the child tilts his head to look at objects, covers an eye
when looking at objects, squints, frowns or blinks
frequently, or holds objects very close or far away to look
at them. The other assessments are appropriate for an
infant older than 6 weeks.

The nurse caring for elderly clients in a nursing home


researches nursing interventions aimed at health
promotion and disease prevention for the elderly. Which
of the following statements accurately reflects the current
health status of older adults in the United States?
A) The elderly population in the United States is growing
B) Since 1900, the percentage of people over 65 has
doubled
C) As people age, they are more likely to live with a
partner or family member
D) People who live to age 65 can expect to live around 10
more years
A
Rationale: The elderly population in the United States is
growing. Life expectancies at ages 65 and 85 have
increased over the past 50 years. People who live to age 65

can expect to live an average of nearly 18 more years.


Since 1900, the percentage of people 65 years and older
has tripled with the aging of baby boomers. As people age,
they are more likely to live alone.

During an annual physical, the nurse is reviewing the


immunization record of a 22-year-old female client. The
client has no record of immunizations and does not
remember being immunized. Which of the following is a
recommended immunization for this client?
A) One dose of influenza vaccine annually
B) 4 doses of Measles, Mumps and Rubella (MMR)
C) 2 doses of Tdap, then boost with Td every 10 years
D) 2 doses of HPV vaccination
A
Rationale: The 19- to 26-year-old should have one dose of
influenza vaccine annually, 3 doses of HPV (if female), 1
dose of Tdap boosted with Td every 10 years, and 1 or 2
doses of MMR.

The nurse is counseling parents about common chronic


diseases of childhood and their prevention. Which of the
following would the nurse state as the single most
common chronic disease in children?
A) Dental caries
B) Juvenile diabetes
C) Ear infections
D) Asthma
A
Rationale: Dental caries represent the single most common
chronic disease of childhood, occurring five times as
frequently as the second most common disease (asthma).
Unless identified and cared for early, caries are irreversible.

An African American nurse is providing care in a health


clinic in a primarily Hispanic neighborhood. What is the
first step in intervening appropriately with clients from
another culture?
A) Practicing cultural blindness
B) Promoting ethnocentrism
C) Preventing stereotyping
D) Practicing cultural awareness

D
Rationale: The first step in providing culturally competent
care is to understand one's own cultural background,
influences, and biases. Only with cultural awareness can
nurses appreciate and be sensitive to values, beliefs, life
practices, and problem-solving methods of a client's
culture. Cultural blindness occurs when the nurse does not
recognize his or her own beliefs or practices, or those of
others. Ethnocentrism is the idea that one's own ideas or
beliefs are the best way to behave, which has a negative
effect on cultural awareness. Preventing stereotyping
(generalizing about others) is important, but is not the first
step in this process.

A nurse providing care in a community health care clinic


initiates a grant proposal to help identify pregnant women
in the community who are not receiving prenatal care and
provide this care on a sliding scale based on income. In
which of the following nursing interventions is this nurse
involved?
A) Health teaching
B) Consultation
C) Collaboration
D) Case finding
D
Rationale: Case finding is a set of activities nurses working
in community settings use to identify clients who are not
currently receiving nursing care but could benefit from it.
Health teaching is the communication of information and
skills that change the knowledge and attitudes of clients
and their families. Collaboration involves two or more
individuals or agencies to achieve a common health goal
for a client. Consultation is an interactive problem-solving
process between the nurse and client.

The nurse providing teaching in the home care setting


knows that in order to be reimbursed by Medicare or
third-party payors the teaching must meet certain
criteria. Which of the following is an example of a
questionable teaching activity that may not be paid?
A) A nurse teaches a diabetic client how to inject insulin
B) A nurse teaches a client with a broken neck how to
care for a halo device

C) A nurse teaches a client how to organize multiple


medications into a pill box
D) A nurse teaches a client with a new colostomy how to
care for the stoma
C
Rationale: Questionable teaching includes the
administration of oral medications or teaching for the client
who is capable of independent ambulation, dressing,
feeding, and hygiene. Teaching care for a new colostomy
and administration of insulin is generally covered, as well
as teaching the patient how to use and care for braces,
splints, and orthotics.

A nurse is teaching a new mother about the advantages


and disadvantages of breastfeeding a newborn. Which of
the following would the nurse report?
A) Breastfeeding should be supplemented with iron-rich
formula to prevent anemia
B) Breastfeeding decreases the rate of respiratory
infections in newborns
C) Breastfeeding must be monitored as the newborn is
more likely to be overweight
D) Breastfeeding prolongs the return to pre-pregnancy
weight in the mother
B
Rationale: Breastfeeding is the most complete form of
nutrition for infants, with various benefits including health,
growth, immunity, and development. Breastfed babies
have a lower incidence of diarrhea, respiratory infections,
and ear infections and are less likely to be overweight.
Breastfeeding promotes the return of pre-pregnancy
weight. Breastfeeding should not be supplemented with
formula feeding.

A nurse is using surveillance to improve the health of a


culturally diverse community. Which of the following
interventions might this nurse perform?
A) Collecting and analyzing health data to plan and
implement health care services in a free clinic
B) Placing ads in the local newspaper to announce a new
recreation program for the community population
C) Interacting with community members in a town
meeting to provide health screening to uninsured children

D) Identifying individuals in the community with


unrecognized health risk factors, such as homeless
people
A
Rationale: Surveillance is used to describe and monitor
health events through ongoing and systematic collection,
analysis, and interpretation of health data. This process is
used to plan, implement, and evaluate public health
interventions. Identifying individuals in the community with
unrecognized health risks factors, such as homeless people
is a function of screening. Interacting with the community
to solve problems such as uninsured children is a function
of consultation. Using public marketing is social marketing.

Some of the leading causes of death and disability in the


United States can often be prevented by making lifestyle
changes. Staying active, eating right, and not smoking
are preventive measures for which of the following
leading causes of death in the United States?
A) Heart disease
B) Unintentional injuries
C) HIV/AIDS
D) Mental disorders / Suicide
A
Rationale: About two thirds of all mortalities and
morbidities result from three behaviors: tobacco use, poor
dietary patterns, and smoking. Heart disease and cancer
are the leading causes of death in adults, and staying fit,
eating right, and not smoking are key strategies to
preventing these diseases.

The nurse uses the health-illness continuum as a model


for practice in the community-based health care setting.
Which of the following accurately represents the
philosophy of this model of health?
A) Improvement in health is seen as an outcome related
to available medical services
B) technology drives care in this model
C) It considers health rather than illness as the essence of
care
D) Care is directed at resolving immediate health
problems

C
Rationale: The health-illness continuum considers health
rather than illness as the essence of care, which requires a
shift in thinking. Improvement in health is not seen as an
outcome of the amount and type of medical services or the
size of the hospital. Treatment efficacy rather than
technology drives care in this model. Care provided in
acute care facilities is directed at resolving immediate
health problems, whereas in this model the focus is on
maximizing individual potential for self-care.

The nurse caring for clients in a nursing home is advising


a new resident couple regarding safe sex practices. Which
of the following is an appropriate teaching point for this
couple?
A) Both partners should be tested for STD's prior to
having sex
B) In the United States, the incidence of HIV is minimal in
the older population
C) Older men should not be encouraged to use
performance-enhancing drugs
D) As men get older, impotency is not a problem unless it
was in the past.
A
Rationale: Having safe sex is important for people of all
ages, and both members of new couples should be tested
for STDs prior to having sex. In some areas of the country,
HIV is on the rise in the elderly population. As men age,
impotence increases with some chronic diseases
contributing to the cause. Pharmaceutical and mechanical
options are available to enhance sexual enjoyment.

The nurse formulates learning objectives for clients being


treated in a long-term health care facility. Which of the
following is an example of an affective learning objective?
A) The client will state three healthy snacks to substitute
for sweets by 10/24/12
B) The client will demonstrate the proper method of
crutch walking by 10/24/12
C) The client will express a desire to start a smoking
cessation class by 10/24/12
D) The client will ambulate the length of the hallway by
10/24/12

C
Rationale: An affective learning objective relates to learning
activities that enhance the acceptance of and the
adjustment to illness and subsequent treatment (e.g. the
client expressing a desire to stop smoking). Stating three
healthy snacks to substitute for sweets is a cognitive
objective. Ambulating the length of a hallway and
demonstrating crutch walking are psychomotor learning
objectives.

The nurse working in a hospital knows that the client's


diagnosis is categorized according to the federal DRG
coding system instituted in the early 1980s. Which of the
following is an outcome of this prospective payment
system?
A) Improved efficiency means less revenue for the
hospital
B) The length of client hospital stays has increased
C) More and more health services are being provided
outside the hospital
D) Hospital readmission for clients has decreased
C
Rationale: Over time, more and more services are being
provided outside the hospital because it is more costeffective. Improved efficiency means more revenue for the
hospital because of shorter stays and use of fewer
resources. Since shorter stays are financially favorable for
hospitals, clients are discharged "quicker and sicker,"
resulting in an increase in readmissions.

It is important for the community-based nurse to assess


the family or kinship patterns of the groups within the
community being served. Which of the following is the
best method to assess family structure?
A) Ask the client who is the head of the family
B) Ask the client who is in his or her family
C) Describe the typical family and ask the client how his
or her family differs
D) Ask the client how many parents and how many
children comprise the family
B
Rationale: The family is the basic unit of society, but there

are many meanings of "family." In order to avoid


misunderstanding, it is appropriate for the nurse to ask the
client directly who is in his or her family. The nurse should
note if the client views "family" as only the nuclear family
or also includes the extended family.

A nurse provides a referral to an occupational therapist


for an elderly client with limited mobility related to
rheumatoid arthritis. Which of the following is an example
of a service that would be provided by this specialist?
A) Teaching a client how to use a grabber to reach higher
objects
B) Teaching the client how to use a walker to ambulate
C) Performing range-of-motion exercises to strengthen the
client's muscles
D) Assisting the client to take a shower and get dressed
A
Rationale: An occupational therapist helps the client adjust
to limitations by teaching new vocational skills or better
ways to perform activities of daily living. Performing range
of motion exercises and teaching the use of a walker are
roles of the physical therapist. Assisting the client with
hygiene is a role of the home health aide.

Disparities in American health care are well documented,


and numerous federal initiatives have been undertaken to
reduce these disparities. Which of the following is the
best example of two factors that are intrinsically related
and underlie many of these health disparities?
A) Culture and health behaviors
B) Environment and genetics
C) Gender and religion
D) Economics and education
D
Rationale: Income and education are intrinsically related
and underlie many of the heath disparities (lack of equity)
in the United States. People with the worst health status
are among those with the highest poverty rates and least
education. Environment, genetics, and health behaviors
also play a role in the major causes of death in the United
States. Gender, religion, and culture may influence health
status, but are not major causes of the disparity in health
care.

A nurse is teaching safety guidelines to a group of


parents participating in a parenting class. Which of the
following teaching points follows the recommended safety
guidelines for child safety?
A) Keep hot water temperatures below 120 F
B) Use a car safety seat until the child is at least 30
pounds
C) Keep the child safety seat in the front passengers side
seat
D) Be sure their are screens in all windows
A
Rationale: The hot water temperature should be set at 120
degrees F. A child safety seat should be used until the child
is at least 40 lbs and should be placed in the middle of the
back seat. Window guards and netting should be used to
prevent falls from windows. Screens alone do not always
prevent a fall.

A nurse is caring for a 48-year-old male client recently


diagnosed with lung cancer. The client has a wife and two
children living at home. Which of the following is
considered the fundamental component to providing
continuity of care for this family?
A) Referring the client to cancer specialists
B) Establishing a trusting nurse-client relationship
C) Assessing the client's financial resources
D) Teaching the client self-care measures
B
Rationale: The nurse-client relationship is the fundamental
component in developing continuity of care. Counseling is
established through the nurse-client relationship and
woven into all aspects of the plan as the foundation of care
coordination. The other options are aspects of the care plan
that may need to be addressed, but are not considered the
foundation of continuity of care.

A home health care nurse discusses the incidence of


adults with limitation in activities of daily living (ADLs)
and instrumental activities of daily living (IADLs) with a
physical therapist. Which of the following statistics
accurately portrays the percentage of adults in the United
States with these types of limitations?

A) Approximately 20% of adults over 75 years old have


limitations in IADLs
B) Approximately 30% of adults over 75 years old have
limitations in ADLs
C) Approximately 10% adults aged 65-74 have limitations
in ADLs
D) Approximately 15% of adults aged 65-74 have
limitations in IADLs
A
Rationale: Approximately 20% of adults over 75 years old
have limitations in IADLS. Approximately 10% of this age
group have limitations in ADLs. About 3% of adults aged 65
to 74 have limitations in ADLs, and about 7% have
limitations in IADLs.

The nurse working in a culturally diverse communitybased setting knows that a person's cultural belief system
influences his or her health attitudes, beliefs, and
practices. Which of the following clients would be more
likely to look for a supernatural counterforce to get rid of
a health problem?
A) An Asian woman who has breast cancer
B) An Eastern European client who is experiencing
seizures
C) A client of Caribbean culture who is diagnosed with
lung cancer
D) An Arab client who is scheduled for a postatectomy
C
Rationale: The magico-religious view, practiced by people
from Hispanic and Caribbean cultures, sees illness as
having a supernatural force. People with this perspective
will look for a counterforce to rid them of the problem.
People of Arab or Asian descent may ascribe to the hot and
cold theory based on the balance of the four humors.

Competencies for nurses working in the community


setting include skill and knowledge in assessment,
program planning, communications, cultural competency,
public health science, management, and leadership.
Which of the following examples of competencies best
reflects the use of public health science skills?
A) A nurse researches the availability of free health care

clinics and transportation for low-income families in a


nearby community
B) A nurse plans a series of parenting classes for an inner
city neighborhood and decides which clients should be
invited to attend
C) A nurse plans and organizes care for an elderly client
who has dementia and is living at home with her
daughter
D) A nurse interviews a client with advanced Parkinson's
disease to determine which care should be initiated,
continued, altered, or discontinued
A
Rationale: Nurses understand factors contributing to health
promotion and disease prevention (such as free health care
clinics) and factors contributing to the use of health
services (such as transportation) by using basic public
health science skills. Planning and organizing care for a
client involves the use of management skills. The nurse
uses program planning skills to plan parenting classes, and
interviewing clients to plan care is related to the use of
assessment skills.

Which of the following interventions might the nurse


perform in the home health care setting when providing
tertiary prevention?
A) Informing a client about smoking cessation classes
offered in the community
B) Referring a caregiver to a program designed to prevent
caregiver burnout
C) Teaching range-of-motion exercises to a client who is
recovering from a stroke
D) Providing teaching materials about diabetes in a
family's native language
C
Rationale: Tertiary prevention focuses on rehabilitation,
management of chronic conditions, or postsurgical care at
home. Informing a client about smoking cessation classes
is a form of secondary prevention. Referrals for classes and
providing teaching materials are examples of prevention on
the primary level.

The nurse is performing a functional assessment on Mrs.


Ruiz, an elderly female client discharged from the hospital
following hip replacement surgery. Mrs. Ruiz is

temporarily living with her daughter and her daughter's


two school-age children, but would like to return to her
own home. Which of the following is the best example of
a question to assess this client for functional status?
A) Have you been experiencing any cramping in your
legs?
B) Are you able to dress yourself independently?
C) Have you noticed any oozing or pain at the incision site
D) How would you rate your pain on a scale of 1 to 10
B
Rationale: Function assessment is performed to determine
if the client needs the assistance of another person for
daily function, such as dressing, bathing, and cooking
meals. Pain assessment, assessment for infection, and DVT
may impact functional ability but are not direct functional
assessment parameters.

A nurse is caring for clients in a mostly Puerto Rican


community. Which of the following is an example of using
a cultural care accommodation modality to treat a
teenage client who is pregnant?
A) Be familiar with religious symbols and protective care
symbols that may be used
B) Develop a nutrition outreach program for expectant
mothers in the community
C) Use the Spanish language to include the teenager's
mother in the care plan
D) Treat the mother and daughter with respect and
maintain eye contact
C
Rationale: Using the Spanish language is a cultural care
accommodation modality, which is an assistive, supportive,
or enabling technique to help people of a certain culture
adapt to others for a beneficial outcome. Treating the
clients with respect and being familiar with their religious
symbols are forms of cultural care preservation and/or
maintenance. Developing a nutrition outreach program is a
cultural care repatterning or restructuring modality.

A nurse is performing a mini nutritional assessment for


78-year-old Mr. Parker. He is in good health (no
psychological distress or recent acute illnesses) and lives
in an assisted-living facility. Upon assessment, the nurse

learns that Mr. Parker has experienced a moderate


decrease in his food intake, has lost 6 pounds in the last 3
months, is able to go out, and has no neuropsychological
problems. His body mass index is 20 and calf
circumference is 30. How would the nurse document Mr.
Parker's score on this assessment tool?
A) 10 points = at risk of malnutrition
B) 13 points = normal nutritional status
C) 7 points = malnourished
A
Rationale: When assessing nutrition using the mini
nutritional assessment tool, a score of 12 to 14 points =
normal nutritional status, 8 to 11 points = at risk of
malnutrition, and 0 to 7 points = malnourished.

The family nurse knows that providing nursing care to


families is a logical development of the holistic approach
to the care of the client. Which of the following accurately
describes a consideration when caring for clients in the
context of their families?
A) A change in one part of a family system does not
usually change other parts of the system
B) Communication patterns between family members do
not affect the natural functioning of the family
C) A family's structure and organization cannot be
understood in isolation from the rest of the family system
D) It is important to first isolate the client from the family
to fully understand the client
C
Rationale: A family's structure and organization, or a part of
that family, cannot be understood in isolation from the rest
of the family system. A change in one part of a family
system affects other parts of the family. Communication
patterns between family members are essential to the
functioning of the family.

A nurse is teaching preventive measures to adults in an


assisted living facility. Which of the following is a
recommended guideline for adult health screening?
A) Women ages 50-74 should have a mammogram every
year
B) Men ages 50-75 should have an annual PSA screening

for prostate cancer


C) People ages 35-75 should undergo a colonoscopy
every 5 years
D) All women should have an annual pap smear starting
at age 18 or when sexually active
D
Rationale: All women should have an annual Pap smear
starting at age 18 or when they become sexually active.
After three normal Pap smears they may be tested less
frequently according to their practitioner. The U.S.
Preventive Services Task Force (2010) recommends that
women ages 50 to 74 have a mammogram every 2 years.
Experts do not universally support the PSA screening for all
men regularly, since the test can be abnormal for reasons
other than prostate cancer. People ages 50 to 75 should be
screened regularly for colorectal cancer, though not
necessarily every 5 years.

A nurse is counseling elderly clients in a mental health


facility. Which of the following statistics should the nurse
keep in mind when devising a plan of care for these
clients?
A) Aging has little or no effect on the emergence of
mental health issues
B) Older Americans are disproportionately likely to die
from suicide
C) Only severe depression responds well to treatment
D) The incidence of depression is lower for clients
receiving home health care
B
Rationale: Older Americans are disproportionately likely to
die from suicide; they make up only 12% of the population,
but account for 16% of suicide deaths. Many issues related
to mental health emerge as people age. This may be
related to loss of health, family, friends, and spouse. The
incidence of depression is higher in clients receiving home
health care and even higher in hospitalized clients. All
depression should be treated, and there are many options
for treatment.
Advocate
...

Assessment

- what is the systematic collection, assembly, analysis, &


dissemination of information about the health of a
community?
- as a community health nurse you must gather & analyze
information that will affect the health of the people to be
served.
- Data may be gathered in many ways which includes:
*Interviewing people
*Conducting surveys
*Public records

Assurance
- what are activities that make certain that services are
provided?
- you must improve the level of "?" what

Case Management
- what is a systematic process by which a nurse assesses
clients needs, plans for & coordinates services, refers to
other appropriate providers, & monitors and evaluates
progress to ensure that clients multiple service needs are
met in a cost-effective manner?

Clinician
...

Collaborator
- what role of a community nurse involves working jointly
w/ others in a common endeavor, cooperating as partners?
- successful community health practice depends on this
multidisciplinary collegiality & leadership.

Policy Development
- what is enhanced by the synthesis & analysis of
information obtained during assessment?
- the nurse recommends specific training & programs to
meet identified health needs of target populations.

Researcher
- this role of the community nurse involves engaging in a
systematic investigation, collection & analysis of data for
solving problems & enhancing community health practice?

Core Public Health Functions

- the various roles & settings for practice hinge on three


primary functions of public health:
*Assessment
*Policy Development
*Assurance
- these are applied at three levels of service: Individuals,
Families, Communities.
- Study (Display 3.1) pg 44.

Roles of Community Nurse: Clinician Role


- what is the most familiar role of the community nurse that
involves being a 'care provider'?
- this role means that the nurse ensures health services are
provided not just to individuals & families, but also to
groups & populations.
-

Roles of Community Nurse: Holistic Practice


- what role of the nurse seeks to be broad & holistic?
- in community health, this approach means considering
the broad range of interacting needs that affect the
collective health of the "client" as a larger system.
- this type of nursing care encompasses the comprehensive
& total care of the client in all areas such as Physical,
Emotional, Social, Spiritual, & Economic.
- the client is a composite of ppl whose relationships &
interactions w/ each other must be considered in totality.

Roles of Community Nurse: Focus On Wellness


- what role of the community nurse is characterized by its
focus on promoting wellness?
- especially emphasizes promotion of health & prevention
of illness.
- effective services include seeking out clients who are at
risk for poor health & offering preventive & healthpromoting services, rather than waiting for them to come
for help after problems arise.
- Ex, immunization of preschoolers, cholesterol screening,
family planning programs, prevention of behavioral
problems in adolescents.

Roles of Community Nurse: Expanded Skills


- what role of the community nurse involves the many
different skills used by the nurse?
- instead of skills being placed solely on physical care skills,
it is now encouraged to expand skills in Observation,

Listening, Communication & Counseling & also to


encompass Sociocultural and Psychological.
- due to environmental & community-wide considerations,
there is a need for stronger skills in assessing the needs of
groups & populations & intervening at the community level.

Roles of Community Nurse: Educator Role


- this role of the community nurse focuses on health
teaching & is a major function of this nurse.
- w/ emphasis on populations & aggregates these teaching
efforts of the community nurse are appropriately targeted
to reach many people.
- community living clients need to know about a widevariety of issues such as Family Planning, Weight Control,
Smoking Cessation, & Stress reduction.
- aggregate level concerns include Environmental safety,
Sexual discrimination or Harassment, Violence and Drugs.

Roles of Community Nurse: Advocate Role


- what role of the community nurse involves pleading for
the clients cause or acting on their behalf?
- every pt has the right to receive just, equal, & humane
treatment.
- Ex, explain which services to expect, which services they
ought to receive, make referrals as needed, & write letters
to agencies or health care providers for them.
- the two main goals of this role are:
*To help pt's gain greater Independence/Self
Determination.
*To make the system more responsive to the needs of
the clients.
- this role is performed via 4 main actions:
*Being Assertive
*Taking Risks
*Communicating/Negotiating
*Identifying Resources
*Obtaining Results.

Roles of Community Nurse: Manager Role


- what community nurse role involves exercising
administrative direction toward the accomplishment of
specified goals by identifying client needs,
planning/organizing to meet those needs, directing &
leading to achieve results, controlling/evaluating the
progress to ensure that goals are met?

Manager Role: Nurse as a "Planner"


- this manager role involves setting the goals & direction
for the organization or project & determines the means to
achieve them?

Manager Role: Nurse as an "Organizer"


- this manger role involves designing a structure within
which people & tasks function to reach the desired
objectives?
- includes deciding on the tasks to be done, who will do
them, how to group the tasks, who reports to whom, &
where decisions will be made.

Manager Role: Nurse as a "Leader"


- this manager role is when the nurse directs, influences, or
persuades others to effect change that will positively
impact peoples health & move them towards a goal.
- includes persuading/motivating ppl, directing activities,
ensuring effective two-way communication, resolving
conflicts, & coordinating the plan.

Manager Role: Nurse as a "Controller & Evaluator"


- a 'controller' monitors the plan & ensures that it stays on
course. an 'Evaluator' compares & judges performance and
outcomes against previously set goals/standards.

Management Behaviors
- these type of behaviors are grouped into 3 sets of
behaviors:
*Decision Making Behavior
*Transferring of Information Behaviors
*Interpersonal Relationships

Management Skills
- the three basic management skills include:
"Human Skills" which refers to ability to understand,
communicate, motivate, delegate & work well with ppl.
"Conceptual Skills" the mental ability to analyze & interpret
abstract ideas for the purpose of understanding &
diagnosing situations and formulating solutions.
"Technical Skills" the ability to apply special managementrelated knowledge & expertise to a particular situation.

Leadership Role
- this leadership role is separate from the manager role &
focuses on effecting change; thus the nurse becomes an

agent of change.
- as leaders, community health nurses seek to initiate
changes that positively effect ppl's health.
Epidemiology
The study of the distribution and the determinants of states
of health and illness in human populations.

Distribution
Refers to the frequency of occurrences of health and illness

Determinants
Refers to agents or factors that contribute to the cause of
various states of health and illness.

Epidemiology ultimate goal


Use the information obtained from the study of the
distribution and determinants of health and illness to
prevent or limit the consequences of illness and disability
in humans and maximize their state of health.

John Snow
Cholera epidemic, linked to the water source in the center
of the city.

Epidemiological triad
Agent (organism like the measles virus), host (human), and
environment

Web of Causation
Based on the belief that health status is multifactorial,
determined by the interaction of many agent, host, and
environment characteristics, and not by any single factor.

Ecological model
Expands epidemiological studies upward into broader
contexts such as neighborhood characteristics and
community and social contexts, and downward to the
genetic molecular level. Model encompasses determinants
at many levels: biological, emotional, behavioral,
socioeconomic, cultural, political, and environmental.

Risk
the probability that a particular event or outcome will occur
within a specified time period.

Genomics
is the study of all the genes in a person, as well as the
interaction of those genes with each other and a person's
environment.

Pre-pathogenesis
factors within individuals and their environments that may
predispose or precipitate the disease. The initial
interactions among agent, host, and environment occur
during this period.

Pathogenesis
begins when the host begins to respond with biological,
psychological, or other changes. Manifested by S&S until
the issue is resolved.

Primary prevention
Activities that prevent a disease from becoming established
and occurs during the pre pathogenesis period.

Secondary prevention
Includes activities designed to detect disease and provide
early treatment.

Tertiary prevention
the treatment, care, and rehabilitation of people with acute
and chronic illness to achieve their maximum potential.
Tertiary prevention is initiated after irreversible changes
have resulted.

Descriptive Epidemiology
Focuses on the frequency and distribution of states of
health within a population. Describe the problem (what
occurs) in terms of person, place , and time.

Risk factors
Factors that are associated primarily with the people who
have the illness.

Ratio
is a fraction that is obtained by dividing one quantity by
another quantity; it represents the relationship between
two numbers.

Proportion
is a type of ratio that include the quantity in the numerator
also as a part of the denominator.

Rate
A measure of frequency of an event or diagnosis in a
defined population within a given time period. (rate is a
proportion that includes the factor of time). best indications
of the probability that a disease, condition, or event will
occur.

Incidence
Measures the occurrence of new illness in a previously
disease-free group of people within a specific time frame.
Measure of the probability that people without a certain
condition will develop the condition over a period of time.

Mortality
Death

Morbidity
illness

Prevalence rates
Measures the amount of morbidity that exists in a
community as a result of the health problem under
investigation.

Period prevalence
indicates the existence of a condition during an interval of
time.

Point prevalence
refers to the existence of a condition at a specific point in
time.

Crude rates
Measure the experience of health problems in populations
of designated geographic areas.

Specific rates

Calculated to describe the distribution of health problems


by age, gender, ethnicity, and other demographic
characteristics.

Adjusted rates
Have been standardized, removing differences in
composition of populations, such as age.

Vital statistics
Data collected from the continuous recording of events
such as births, deaths, marriages, divorces, and adoptions,
usually by state agencies.

Morbidity and Mortality Weekly Report (MMWR)


Published by the CDC is the vehicle for distributing current
information to healthcare professionals.

National Health Survey


Provides information about the health needs of the
population of the United States. Most of this information is
prevalence data and is the only nationwide source of data
on chronic illness, minor conditions, and functional
problems.

Behavioral Risk Factor Surveillance System (BRFSS)


Was established to collect, analyze, and interpret
behavioral risk factor data from all states.

Person
Characterizes the "Who" develops the health problem.

Place
"Where" the rates of the health problem are the highest or
the lowest can be determined by examining the
characteristics of place.

Time
"When" health problems occur can be describe the
identifying short-term fluctuations measured in hours,
days, weeks, or months; buy periodic changes that are
seasonal or cyclical; or by long-term changes over decades
that reflect gradual changes.

Epidemic curve

gives a graphical display of the numbers of incident cases


in an outbreak

Analytic Epidemiology
Focuses on the determinants of health problems, or the
"why". Uses 4 different types of studies: cross-sectional,
retrospective (case-control), prospective, and intervention
(experimental) studies.
What is implied by the web of causation model?
a. Variables interact resulting in higher probability of
illness.
b. One disease causes another, especially in vulnerable
populations.
c. The greater the poverty, the more likely people are to
have diseases.
d. Immunizations are necessary because vulnerable
populations spread disease.
a. Variables interact resulting in higher probability of
illness.

Which is a characteristic of a vulnerable population?


a. Have worse health outcomes and an increased
sensitivity to risk factors than the general population
b. Have a single risk factor but experience worse health
outcomes than the general population
c. Have multiple risk factors but equal health outcomes
than the general population
d. Have worse outcomes with better access to health care
than the general population
a. Have worse health outcomes and an increased
sensitivity to risk factors than the general population

The differential vulnerability hypothesis refers to the:


a. Resistance of certain groups to risk factors
b. Increased susceptibility to cumulative risk factors
among vulnerable groups
c. Variability in the effects of stressors according to
socioeconomic status
d. Increased sensitivity of the very young and the very old
to risk factors
b. Increased susceptibility to cumulative risk factors among
vulnerable groups

Which is an example of a priority population group?


a. People earning more than $100,000/year

b. People earning less than $100,000/year


c. Residents of remote rural areas of Canada
d. African American physicians
c. Residents of remote rural areas of Canada

An example of a vulnerable group experiencing multiple


risk factors is:
a. Smokers who use chewing tobacco as well as
cigarettes
b. Substance abusers who test positive for human
immunodeficiency virus (HIV)
c. Persons with limited access to care because they live in
a rural area
d. New mothers needing information about baby and child
care
b. Substance abusers who test positive for human
immunodeficiency virus (HIV)

A 17-year-old pregnant cocaine addict who is homeless is


considered:
a. At risk
b. A special population
c. A Healthy People 2020 target group
d. A vulnerable individual
d. A vulnerable individual

The wide variations in health services and health status


between certain population groups are called:
a. Priority population groups
b. Health disparities
c. Disadvantaged populations
d. Risk markers
b. Health disparities

What is the primary cause of vulnerability?


a. Race
b. Age
c. Poverty
d. Illness
c. Poverty

Which population is most likely to be affected by poverty?


a. Women
b. Men
c. Two-parent families
d. Adoptive families

a. Women

A nurse operates a school-based clinic in a local school,


making it easier for children to access health care. This is
an example of:
a. Case finding
b. Wrap-around services
c. Outreach
d. Comprehensive services
c. Outreach

An example of a health disparity is:


a. Childhood immunization rates
b. High dropout rates
c. Unemployment
d. Income below poverty level
a. Childhood immunization rates

A nurse is developing a one-stop service to meet the


needs of a vulnerable group. Which is an example of this
type of service?
a. Wrap-around services where mental services are linked
b. Giving all immunizations on a single clinic visit
c. Providing multiple services during a single clinic visit
d. Providing free services to the medically indigent
c. Providing multiple services during a single clinic visit

Which vulnerable group would be most sensitive to


adverse effects?
a. Pregnant teenager living with her parents for financial
support
b. Poor, older woman with no means of transportation
c. Two-year-old boy of underinsured parents
d. Recently unemployed father of five
b. Poor, older woman with no means of transportation

Which federal program created support for older and poor


Americans?
a. Social Security Act
b. Medicare Amendment
c. Medicaid Amendment
d. Hill-Burton Act
a. Social Security Act

A client is self-employed as a mechanic and has limited


health insurance coverage. This client is considered:

a. Poor
b. Near poor
c. Medically indigent
d. Uninsured
c. Medically indigent

A mental health nurse working in the 1960s received


additional education to provide care for the severely
mentally ill in the community because of which
legislation?
a. Community Mental Health Centers Act
b. Stewart B. McKinney Homeless Act
c. National Health Planning and Resource Development
Act
d. Patient Protection and Affordable Care Act
a. Community Mental Health Centers Act

A nurse considers an audience's ability to read,


comprehend, and act on medical instructions while
preparing health education materials. Which factor is the
nurse considering?
a. Health literacy
b. Ethnicity
c. Medical training
d. Disparity
a. Health literacy

Which is an example of a social determinant of health?


a. Ethnicity
b. Income
c. Gender
d. Marital status
b. Income

A nurse caring for a homeless population recognizes that


those who are homeless:
a. Need more nursing care than other vulnerable groups
b. Have no desire to seek medical care
c. Have even fewer resources than poor people who have
adequate housing
d. Are living in despair with no hope or resilience
c. Have even fewer resources than poor people who have
adequate housing

Which demonstrates the removal of a barrier to health


care?

a. Discrimination against certain groups


b. Treatment of pets at the same facility
c. Provision of free food at a food bank
d. Providing services for a rural population by using a
mobile clinic
d. Providing services for a rural population by using a
mobile clinic

A set of actions one undertakes on behalf of another is:


a. Social justice
b. Advocacy
c. Linguistically appropriate health care
d. A waiver
b. Advocacy

A nurse works with a group of abused women to enhance


their levels of self-esteem. Which level of prevention is
being performed?
a. Primary level of prevention
b. Secondary level of prevention
c. Tertiary level of prevention
d. Health promotion
c. Tertiary level of prevention

How can nurses promote social justice?


a. By contacting lawmakers about environmental health
issues
b. By assisting at homeless shelters
c. By advocating for policies to improve social conditions
d. By serving on a local coalition to prevent obesity
c. By advocating for policies to improve social conditions

Which is the most appropriate action for a nurse when


planning and implementing care for vulnerable
populations?
a. Setting up multiple clinics in a wide geographic area
b. Advising legal consultants on a variety of issues
c. Making laws to protect the homeless
d. Teaching vulnerable individuals strategies to prevent
illness and promote health
d. Teaching vulnerable individuals strategies to prevent
illness and promote health

Which trends are occurring among vulnerable


populations? Select all that apply.
a. Community-based care and interorganizational

partnerships
b. Outreach and case finding
c. Elimination of disparities
d. Culturally and linguistically appropriate care
a. Community-based care and interorganizational
partnerships
b. Outreach and case finding
d. Culturally and linguistically appropriate care
What are the 10 great public health achievements in the
us from 1900-1999?
vaccinations, motor vehicle safety, safer workplaces,
control of infectious diseases, decrease in coronary heart
disease and stroke deaths, safer and healthier foods,
healthier mothers and babies, fluoridation of drinking
water, recognition of tobacco as a health hazard.

What are social determinants of health?


social conditions in which people live & work

What are health care disparities?


gaps in healthcare experienced by one population
compared to another.

What is the role of government in health care?


Assess health care problems. Developing relevant health
care policies that provide access to services. Ensure that
services are delivered and outcomes are achieved.

Who said, "it is cheaper to promote health than to


maintain people in sickness?
Florence Nightingale

What is the goal of the patient protection and affordable


care act?
The goal of the PPACA (obama 2010) is to provide
affordable health insurance coverage and to improve
access to primary care. and lower costs.

What were 3 major changes in health care in the 21st


century?
development of patient centered care. increased use of
technology. increased personal responsibility for health.

What are the 4 models of health?


clinical model. role performance model. adaptive model.
eudaimonistic model.

clinical model def.


elimination of disease or symptoms

role performance model def.


health that involves a fit b/w ppl and social roles

adaptive model def.


health that involves adaptation to the environment

eudaimonistic model def.


health that is the actualization or realization of human
potential

behavior models
motivational interviewing. behavior change models.
learning model. health belief model.

motivational interviewing def


patient centered communication styles for eliciting
behavior change by helping patients/ groups explore and
resolve ambivalence to change

behavior change model def


models that assist patients, groups and communities to
direct activities toward health and wellness

Learning model def


a behavior change model emphasizing reinforcement of
social competence problem solving autonomy and sense of
purpose

Health belief model


a behavior change model that considers the severity of the
potential illness or physically challenge, the level of
conceivable susceptibility the benefits of taking
preventative action and the challenges that may be faced
in taking action toward the goal of health promotion

ontogenic system

personal factors

microsystem
relationship b/w women and their environment

exosystem
formal and informal social structures

macroculture
values and beliefs of culture

social determinants of health


social conditions in which people live and work

what is one of the core functions of the role of


government in health care?
assess health care problems

community based nursing


minor acute and chronic care that is comprehensive and
coordinated where people work, live or attend school;
illness care provided outside the acute care setting

community health nursing


use of systematic processes to deliver care to individuals
families and community groups with a focus on promoting
preserving protecting and maintaining health

public health nursing


population based practice defined as a sythesis of nursing
and public health within the context of preventing disease
and disability and promoting and protecting the health of
the entire community

shattuck report
provided the 1st systematic use of birth and death records
and demographic data to describe the health

lemuel shattuck
recommended establishment of state health

dorothea dix
prison and mental health

clara barton
red cross/ civil war

lillian wald
founder of public health nursing. henry st settlement.

mary breckinridge
frontier of nursing service 1925 needs of health in the
mountain region. founded midwifery program

philanthropic organization
an organizations that used endowed funds or private
fundraising to address the needs of individuals, families,
and populations

nongovernmental organizations
agency that acquires resources to help others from
private(vs. public) sources

philanthropic organization
an organization that used endowed funds or private
fundraising to address the needs of individuals, families,
and populations

TRUE or FALSE: the united states has found the most


efficient and effective ways to care for all by decreasing
health disparities and giving access to care equality
FALSE: despite the economic strength of the united states
and other industrialized nations, many countries have
found more efficient and effective ways to care for all by
decreasing health disparities and giving access to care
equality

Definition of health
a state of complete physical, mental and social well-being,
not merely the absence of disease or infirmity

what are the 8 millennium developmental goals


1. eradicate extreme poverty & hunger 2. achieve universal
primary education 3. promote gender equality & empower
women 4. reduce child mortality 5. improve maternal
health 6. HIV/AIDS and malaria 7. ensure environmental
sustainability 8. development.

TRUE or FALSE: Health worker migration increases the


burden to care for a society and results in the need to
task shift primarily to primary care providers and
community health workers.
FALSE:
health worker migration increases the burden to care for a
society and results in the need to task shift primarily to
nurses and community health workers.

Health
a quality an ability to adapt to change or a resource to help
cope with challenges and processes of daily living

well-being
a subjective perception of full functional ability as human
beings

what are modifiable risk factors


susceptibility to disease or injury that can be controlled by
individual people, families or communities.

Primary prevention
maximizing health and wellness through strategies that are
set in place before illness or injury is present

secondary prevention
maximizing health and wellness through strategies that are
set in place at the early and active chronic stages of
parthenogenesis of illness and injury (mass screening,
selective screening)

Tertiary prevention
maximizing health and wellness through strategies that are
set in place at the palliation and end stage of disease and
injury trajectories.

epidemiology
study of the distribution and determinants of states of
health and illness in human populations used both as a
research methodology used to study states of health and
illness and as a body of knowledge that results from the
study of a specific state of health for or illness

epidemic
an outbreak that occurs when there is an increased
incidence of a diseases beyond that which is normally
found in the population

rates
the primary measurement used to describe either the
occurrence or or the existence of a specific state of health
or illness

who is perhaps the best known epidemiologist of the 19th


century
john snow

epidemiological triad
host, agent, and environment

wheel of causation
emphasizing the interplay of physical biological and social
environments

The who defines health as


complete physical mental and social well being

during which time period was public health first seen as a


gov. responsibility
the roman era

what is the major force behind the movement toward


home care, hospice care, outpatient tx clinics and
outpatient surgeries
cost containment

when was the first state board of health formed in the us?
1869-north carolina

what was the leading cause of death in the US in 1900?


major cardiovascular-renal disease

what are the only two major industrialized nations without


some form of universal insurance coverage?
US and south Africa

what was established by 1960 amendments to the social


security act of 1935
medicare and medicaid

which of the following is a strategy to contain the cost of


health care
focusing on prevention of illness rather than on treatment

what was the first federal program to provide health care


for citizens other than federal employees
medicare

what decade in the twentieth century saw the most


sweeping federal legislation enacted
1960

which of the following is an example of a public health


prevention policy?
higher cigarette taxes

edward jenner
smallpox 1790

louis paster
1822-1895 pasteurization started as a wine experiment

joseph lister
antiseptic solution

robert koch
anthrax and cholera

john snow
first epidemiologist linked cholera rates with water street
pump golden square in london

isabel hampton
1890s helped found ANA

Margaret Sanger
1910s birth control

Lillian Wald

henry street settlement public health nursing

Clara Barton
civil war nurse leader, US red cross founder

population
group of people who have at least one thing in common
and who may or may not interact w/ one another

types of health care systems


private (BC/BS) public (medicare, medicaid), military
(VA/Tricare)

Role of nurse in managed care


benefits interpreter, client advocate/educator, triage nurse,
utilization/ resource reviewer, risk manager, provider
liaison, primary care provider, case/care manager.
dominant values
the beliefs and sanctions of the dominant or majority
culture

culture
the beliefs, values, and behavior that are shared by
members of a society and provide a design or "roadmap"
for a living

cultural diversity/plurality
a variety of cultural patterns coexist within a designated
geographic area

race
refers to biologically designated groups of people whose
distinguishing features, such as skin color, are inherited

ethnic group
a collection of people who have common origins and a
shared culture and identity; they may share a common
geographic origin, race, language, religion, traditions,
values, and food preferences

ethnicity
the group of qualities that mark one's association with a
particular ethnic group

subcultures
large aggregates of people w/in a society who share
separate distinguishing characteristics, such as ethnicity,
occupation, religion, geographic area, age, gender, or
sexual preferences

microcultures
systems of cultural knowledge characteristic of subgroups
within larger societies

ethnocentrism
the belief or feeling that one's own culture is best and
reflects our tendency to judge other people's beliefs and
behavior using values of our own native culture

ethnorelativism
seeing all behavior in a cultural context

Characteristics of culture
Culture is: learned, integrated, shared, mostly tacit,
dynamic

enculturation
learning about culture through socialization with the family
or significant group

biomedical view
relies on scientific principle and sees diseases/injuries as
life events controlled by physical and biochem processes
that can be manipulated through meds, surgery, and other
treatments

magicoreligious view
focus on the control of health and illness by supernatural
forces; illness - punishment of god, health - gift from god

holistic view
viewing the world in terms of harmonious balance - if
principles guiding natural laws to maintain order are
disturbed, an imbalance in the forces of nature is created,
resulting in chaos and disease

folk medicine

a body of preserved treatment practices that has been


handed down verbally from generation to generation and
exists today as the 1st line of treatment for many people

home remedies
individualized caregiving practices that are passed down
w/in families

complementary therapies
practices used to complement contemporary Western
medical and nursing care and are designed to promote
comfort, health, and wellbeing

primary prevention
a type of prevention that seeks to promote heath and
prevent disease from the beginning

secondary prevention
an intervention that seeks to detect disease early in its
progression (early pathogenesis) before clinical signs and
symptoms become apparent in order to make an early
diagnosis and begin treatment

tertiary prevention
intervention that begins once the disease is obvious; the
aim is to interrupt the course of the disease, reduce the
amount of disability that might occur, and begin
rehabilitation

medicaid
a jointly sponsored state and federal program that pays for
medical services for the aged, poor, blind, disabled, and
families with dependent children

medicare
a federally funded health insurance program for the elderly
and disabled and persons with end-stage renal disease

health care economics


a specialized field of economics that describes and
analyzes the production, distribution, and consumption of
goods and services, as well as a variety of related problems
such as finance, labor, and taxation

legal immigrant
not citizens, but they are by law allowed to both live and
work in the US, often because they have useful job skills or
family ties;
must live in US for 10 years to be eligible for all
entitlements, such as Aid to Families of Dependent
Children, food stamps, Medicaid, and unemployment
insurance

refugees
refugees are admitted outside the usual quota restrictions
based on fear of perseution due to their race, religion,
nationality, social group, or political views;
may receive Temporary Assistance for Needy Families,
Supplemental Securty Inome, and Medicaid

nonimmigrants
admitted to the US for a limited duration of time and for a
specific purpose

unauthorized immigrant
undocumented or illegal aliens;
eligible only for emergency medical services,
immunizations, treatment for the symptoms of
communicable diseases, and access to school lunches

3 public health core functions


assessment, policy development, assurance

assessment
systematic data collection on the population, monitoring
the populations health status, and making information
available about the health of the community

policy development
efforts to develop policies that support the health of the
population, including a scientific knowledge base to make
policy decisions

assurance
making sure that essential community-oriented health
services are available;
includes providing essential personal health services or
individuals, as well as a competent PH workforce

cultural shock
the feeling of helplessness, discomfort, and disorientation
experienced by an individual attempting to understand or
effectively adapt to another cultural group that differs in
practices, values, and beliefs. it results from the anxiety
caused by losing familiar sights, sounds, and behaviors

cultural repatterning
working with clients to make changes in health practices
when the clients cultural behaviors are harmful or decrease
their well-being

cultural preservation
the use by clients of those aspects of their culture that
promote healthy behaviors

cultural knowledge
the information necessary to provide nurses with an
understanding of the organizational elements of cultures
and to provide effective nursing care

cultural imposition
the process of imposing one's values on others

cultural encounter
interaction with a client related to all aspects of his or her
life

cultural blindness
when differences between cultures are ignored and persons
act as though these differences don't exist

quarantine
period of enforced isolation of persons exposed to a
communicable disease during the incubation period of the
disease to prevent its spread should infection occur

sanitation
promotion of hygiene and prevention of disease by
maintenance of health-enhancing (sanitary) conditions

Marine Hospital Service Act of 1798

first federal institution for health problems - subsidized


medical and hospital care for injured merchant seamen
w/first marine hospital in Boston

Shattuck Report
advocate the establishment of state and local boards of
health, enviro sanitation, collection and use of vital
statistics, systematic study of diseases, control of food and
drugs, urban planning, est of nurses' training schools, and
preventive medicine

official health agencies/public health agencies


publicly funded and operated by state or local gov'ts with a
goal of providing population-based health services

voluntary health agencies/private agencies/NGOs


privately funded, operated to address specific health needs

public sector health agencies


Provide important record-keeping service, including
collection/monitoring of vital statistics
Conduct research, provide consultation, and sometimes
financially support other community health efforts

antigen
A live or inactivated substance (e.g., protein,
polysaccharide) capable of producing an immune response

antibody
Protein molecules (immunoglobulin) produced by B
lymphocytes to help eliminate an antigen

active immunity
Protection produced by the person's own immune system;
usually permanent

passive immunity
Protection transferred from another person or animal;
temporary protection

live attenuated vaccines


Measles, Mumps & Rubella (MMR)
Varicella

LAIV
Rotavirus
Herpes Zoster

inactivated vaccines
Toxoids (DTaP, Td, Tdap)
Whole (Hepatitis A, IPV)
Split (Influenza - TIV)
Recombinant vaccines (Hepatitis B, HPV4, HPV2)
Polysaccharide vaccines (PPSV23, MPSV4)
Conjugated vaccines (Hib, PCV13, MCV4)

seven rights of vaccine administration


Right Patient
Right Vaccine or Diluent
Right Time
Right Dosage
Right Route, Needle Length, Technique
Right Site for route indicated
Right Documentation

retrospective payment
Fee established in advance
Reimburse after service rendered
Abused through the requesting and ordering of
unnecessary tests
Encouraged sickness rather than wellness

prospective payment
External authority sets rates
Rates derived from predictions set in advance
Fixed rates rather than cost coverage
Imposes constraints on spending
Providers at risk for losses or surpluses
Which statement about disasters is true?
a. They can be natural or human-made.
b. They can be relieved without assistance.
c. There is always injury and death when a disaster
occurs.
d. The timing of a disaster does not influence the types of
injuries that will occur.
a. They can be natural or human-made.

The use of weapons of mass destruction or natural


disasters, such as earthquakes, will often have more

casualties because:
a. They cause the most widespread destruction.
b. Victims have little time to make evacuation
preparations.
c. Those with chronic conditions cannot escape in time.
d. The early warning systems are not effective.
b. Victims have little time to make evacuation preparations.

Which is an example of a natural disaster?


a. Transportation accident
b. Pollution
c. Communicable disease epidemic
d. Fire
c. Communicable disease epidemic

Which is an example of a human-made disaster?


a. Structural collapse
b. Communicable disease epidemics
c. Mud slides
d. Floods
a. Structural collapse

What emotion would a nurse anticipate when working


with an older person following a disaster?
a. Anger
b. Fear of loss of independence
c. Violence
d. Regression
b. Fear of loss of independence

In which situation would a nurse experience vicarious


traumatization?
a. When sustaining an injury during the disaster
b. When exhausted and unable to adjust to the pace at
home
c. When frustrated about the response time during the
disaster
d. When listening to survivors' stories about traumatic
events
d. When listening to survivors' stories about traumatic
events

A nurse is working in a temporary shelter for victims


following a natural disaster. Which condition is the nurse
most likely to encounter?
a. Stress

b. Communicable disease
c. Chronic illness
d. Injuries requiring first aid
a. Stress

A riverfront community builds a retaining wall to divert


flood water away from the town. This is an example of:
a. Mitigation
b. A natural disaster
c. Disaster reduction
d. Disaster health
a. Mitigation

An example of community preparedness for a disaster is:


a. Assembling emergency supplies
b. Understanding the workplace disaster plan
c. Taking a disaster training course
d. Developing an evacuation plan to remove individuals
from danger
d. Developing an evacuation plan to remove individuals
from danger

What would most likely occur during the preparedness


stage of disaster management?
a. Heightened inspection and increased security in the
community
b. Incorporation of provision of pets into local disaster
plans
c. Purchase of personal protective equipment for all
citizens
d. Assembly of disaster kits for the home, workplace, and
car
d. Assembly of disaster kits for the home, workplace, and
car

When working in a disaster, which attribute would be


most important for the nurse to possess?
a. Flexibility
b. Training
c. Experience
d. Special interest
a. Flexibility

What is most likely to occur in the community during the


Honeymoon Phase after a disaster?
a. First responders work tirelessly to save others.

b. Survivors share their stories.


c. Medical personnel experience exhaustion.
d. Community organizations rebuild the community.
b. Survivors share their stories.

What is the first action a nurse should take when


assessing persons arriving at a shelter following a
disaster?
a. Limit the amount of equipment and medications
brought into the shelter.
b. Determine if the person has a psychological condition
requiring special attention.
c. Assess whether this type of facility is appropriate for
the person.
d. Provide medical care for persons as if they were in a
hospital.
c. Assess whether this type of facility is appropriate for the
person.

What action would the nurse take when triaging victims


following a disaster?
a. Allocation of treatment based on the victim's potential
for survival
b. Assignment of tasks to the appropriate disaster
response personnel
c. Determination of the level of disaster and agency
involvement
d. Participation in community preparedness for the
disaster response
a. Allocation of treatment based on the victim's potential
for survival

Which task would a nurse be most likely to perform while


volunteering on a disaster medical assistant team?
a. Set up immunization clinics.
b. Assist in triaging disaster victims.
c. Provide all the medical care for disaster victims.
d. Complete a needs assessment of the community.
b. Assist in triaging disaster victims.

Which is an example of a bioterrorism and emergency


readiness competency for a public health worker?
a. Recognition of unusual events that might indicate an
emergency
b. Involvement in the chain of command

c. Using a set of preplanned activities for every disaster


d. Leaving communication to the authorities
a. Recognition of unusual events that might indicate an
emergency

What is the purpose of the National Response


Framework?
a. Create a new branch of government that deals with
bioterrorism.
b. Establish a way for the Red Cross to carry out its
mission.
c. Develop a nationwide all-hazards approach to domestic
incident management.
d. Extend presidential power to act quickly upon weapons
of mass destruction.
c. Develop a nationwide all-hazards approach to domestic
incident management.

Which organization would a nurse collaborate with when


developing and coordinating an emergency response plan
for a community?
a. American Red Cross (ARC)
b. Stress Management in Disaster Program (SMID)
c. Centers for Disease Control and Prevention (CDC)
d. Emergency Management Agency (EMA)
d. Emergency Management Agency (EMA)

Following a disaster, what is the priority of the disaster


response?
a. Clean up the environment.
b. Handle the stress reaction of the victims.
c. Bring in as many aid workers and nurses as possible.
d. Reestablish sanitary barriers and focus on basic needs.
d. Reestablish sanitary barriers and focus on basic needs.

If a bioterrorism attack was to occur in the United States,


which program would be used to provide large quantities
of medication to the American public?
a. Cities Readiness Initiative
b. Strategic National Stockpile (SNS)
c. Public Health Information Network (PHIN)
d. Project BioShield
b. Strategic National Stockpile (SNS)

What is anticipated to happen in the future with disaster


management?

a. Vague and unorganized future because constant


emergence of new disasters
b. Increasing sophistication in technology and
surveillance
c. Lack of involvement by national and state officials
d. Decreasing need for public health workers to provide
care
b. Increasing sophistication in technology and surveillance

Which populations are at greatest risk for disruption after


a disaster? Select all that apply.
a. Single-parent families
b. Children
c. Substance abusers
d. Middle-class families
a. Single-parent families
b. Children
c. Substance abusers

Which organizations offer volunteer opportunities in


disaster work for nurses? Select all that apply.
a. American Red Cross
b. Citizens Corps
c. American Nurses Association
d. National Public Health Training Centers
a. American Red Cross
b. Citizens Corps
Which statement about the Intervention Wheel is true?
a. It provides a graphic illustration of population-based
public health practice.
b. It describes in detail the components of public health
nursing.
c. It demonstrates the practice of community health
nurses for policy and lawmakers.
d. It is a framework used by all health departments in the
United States.
a. It provides a graphic illustration of population-based
public health practice.

Since the Intervention Wheel was first published in 1998,


it has:
a. Guided national policy
b. Been used as a tool in deciding licensure issues for

State Boards of Nursing


c. Been incorporated into the public health curricula of
many nursing programs
d. Gained wide acceptance internationally
c. Been incorporated into the public health curricula of
many nursing programs

Public health interventions are implemented with:


a. Legislators, policy makers, and community leaders
b. Individuals and families, communities, and systems
c. Children, adolescents, and adults
d. Health departments, public health agencies, and
visiting nurses associations
b. Individuals and families, communities, and systems

Which statement is true about the origins of the


Intervention Wheel?
a. A panel of nurses from Iowa, Minnesota, North Dakota,
South Dakota, and Wisconsin developed and refined the
Intervention Wheel.
b. It was conceived by a group of international nurses
from Norway, Kazakhstan, and Japan.
c. It was a result of a qualitative analysis carried out by
the State Boards of Nursing.
d. It resulted from a grounded theory process carried out
by public health consultants at the Minnesota Department
of Health.
d. It resulted from a grounded theory process carried out by
public health consultants at the Minnesota Department of
Health.

When public health nurses conduct an assessment of a


community's health, they
a. Define one problem that will be the focus for a year.
b. Assess a social network of interacting individuals
usually in a defined territory.
c. Minimize the effects of health risks and hazards.
d. Intervene at the population-level by changing laws and
regulations.
b. Assess a social network of interacting individuals usually
in a defined territory.

A public health nurse utilizes the nursing process at all


levels of practice by:
a. Including specific goals for community health nurses
b. Developing an accurate nursing diagnosis

c. Analyzing the needs of the community, system, and


individuals and families
d. Utilizing primary, secondary, and tertiary prevention
c. Analyzing the needs of the community, system, and
individuals and families

Public health nurses use a common set of interventions


to:
a. Describe the proper order of implementation.
b. Emphasize surveillance as the main focus of public
health practice.
c. Guide practice and generate agency protocols.
d. Improve the health status of communities, systems,
individuals, and families.
d. Improve the health status of communities, systems,
individuals, and families.

A public health nurse uses Assumption 2, "Public health


nursing practice focuses on populations," to guide
practice. Which would be considered a population of
interest?
a. Healthy school children
b. Homeless individuals
c. A person recently diagnosed with diabetes
d. Teenage parents
a. Healthy school children

What is the purpose of the color-coded wedges on the


Intervention Wheel?
a. The interventions are grouped together in related
wedges.
b. The wedges consist of referral information for each
wedge.
c. The element of health teaching is the predominant
feature of each wedge.
d. Coalition building must be implemented with each
wedge.
a. The interventions are grouped together in related
wedges.

A nurse who is involved in identifying individuals with


unrecognized health risk factors or asymptomatic disease
is using which intervention?
a. Screening
b. Referral and follow-up

c. Surveillance
d. Health teaching
a. Screening

A nurse promotes alliances among organizations for a


common purpose. Which public health intervention is
being implemented?
a. Health teaching
b. Coalition building
c. Surveillance
d. Referral and follow-up
b. Coalition building

A nursing student develops a teaching plan about hand


washing to present to a group of elementary school
children at the local school. Which public health
intervention is being implemented?
a. Collaboration
b. Surveillance
c. Health teaching
d. Screening
c. Health teaching

A public health nurse uses collaboration, coalition


building, and community organizing when:
a. Providing case management, referral, and follow-up
services with individuals
b. Carrying out collective action at the systems or
community levels of practice
c. Conducting a community assessment
d. Implementing primary and secondary prevention
strategies
b. Carrying out collective action at the systems or
community levels of practice

The levels of practice encompassed by the Intervention


Wheel are:
a. Communities, individuals and families, and systems
b. Assessment, diagnosis, and evaluation
c. Primary, secondary, and tertiary
d. Communities, populations, and aggregates
a. Communities, individuals and families, and systems

Public health nurses utilize registries to identify children


with delayed or missing immunizations. They
subsequently follow up with families by phone calls or

home visits. This is an example of ____-level of practice.


a. Systems
b. Community
c. Policy
d. Individual/family
d. Individual/family

A social marketing campaign urging community members


to avoid driving motorized vehicles after consuming
alcohol is implemented in a local community. This
intervention is occurring the _____-level of practice.
a. Individual/family
b. Systems
c. Community
d. Government
c. Community

An occupational health nurse works with an employer to


develop a workplace wellness program for its employees.
This intervention is occurring at the _____-level of
practice.
a. Individual/family
b. Systems
c. Community
d. Government
b. Systems

When implementing interventions at the systems-level of


practice, the public health nurse would:
a. Involve the entire community in solving the health
problem.
b. Identify health problems in the community.
c. Change laws, policies, and practices that influence
population-based issues.
d. Provide outreach services to populations at risk.
c. Change laws, policies, and practices that influence
population-based issues.

Which is an example of a public health nurse conducting


a community assessment?
a. Visiting an elderly person at home to assess and
evaluate safety and fall risk
b. Developing diagnoses to identify nursing interventions
at a health clinic
c. Evaluating services at an immunization clinic where a
translator provides services

d. Compiling recent data from the county health


department on child abuse cases
d. Compiling recent data from the county health
department on child abuse cases

An example of a measurable outcome health status


indicator at the individual-level of practice that would be
used by a public health nurse is:
a. A 50-year-old woman receives annual mammograms
b. School absences in a community decline
c. Teachers have increased awareness of health problems
d. Those in poverty utilize the free mammogram program
b. School absences in a community decline

Which nursing diagnosis would a public health nurse use


when addressing the problem of obesity at the
community-level of practice?
a. Alteration in nutrition: More than body requirements
b. Need for increased knowledge of proper nutrition
c. Families at risk for obesity because of inactivity
d. Overweight child related to poor dietary habits
c. Families at risk for obesity because of inactivity

A public health nurse implements the public health


intervention of health teaching at the systems-level of
practice by:
a. Participating in the "Great American Smokeout"
b. Working with a local employer to provide smoking
cessation education
c. Providing one-on-one counseling to smokers
d. Advocating for increased taxes on tobacco products
b. Working with a local employer to provide smoking
cessation education

The cornerstones of public health nursing practice (select


all that apply):
a. Focus on the health of the entire population
b. Reflect community priorities and needs
c. Promote health through strategies driven by
epidemiological evidence
d. Are grounded in an ethic of collaboration
a. Focus on the health of the entire population
b. Reflect community priorities and needs

c. Promote health through strategies driven by


epidemiological evidence
Official agencies are financed primarily by:
a. Charities
b. Individual clients
c. Third-party payers
d. Tax funds
d. Tax funds

Medicare-certified home health agencies place emphasis


on _____ care.
a. Chronic
b. Distributive
c. Intermittent
d. Primary
c. Intermittent

A nurse who implements a population-focused model of


home care delivery would most likely:
a. Use an assessment protocol when conducting visits.
b. Collaborate with other disciplines.
c. Provide care for a specific population.
d. Apply high-intensity interventions.
a. Use an assessment protocol when conducting visits.

A home care nurse is employed by a proprietary agency.


What does this mean?
a. The agency is exempt from federal income taxes.
b. The agency is governed by a board of directors.
c. The agency is a profit-making agency.
d. The agency is reimbursed primarily by tax funds.
c. The agency is a profit-making agency.

Which factor is important for the nurse to consider when


coordinating care to ensure a comfortable and peaceful
death?
a. The unique stress dying patients often experience
b. The cultural values, expectations, and preferences of
the family
c. The communication style of each family member
d. The abilities of the nurse and health care team
b. The cultural values, expectations, and preferences of the
family

What is the primary objective of hospice programs?


a. To provide comfort and peace at a difficult time
b. To reduce the cost of care for the terminally ill
c. To keep clients at home until the end of life
d. To provide care for terminally ill clients expected to live
longer than 6 months
a. To provide comfort and peace at a difficult time

The nurse provides direct care services to a stroke victim


to avoid complications. This is an example of which level
of prevention?
a. Primary prevention
b. Secondary prevention
c. Tertiary prevention
d. Assessment
c. Tertiary prevention

Which nursing intervention is an example of a skilled


nursing service?
a. Teaching the client and family about medication
administration
b. Consulting with other nurses about the care of a client
in the home
c. Coordinating services for maximum benefit at minimal
cost
d. Reviewing nursing assistant charting at the end of each
day
a. Teaching the client and family about medication
administration

A nurse employed in home health nursing practice


provides:
a. Intermittent nursing visits
b. Custodial care
c. Family caregiving
d. Palliative care
a. Intermittent nursing visits

Which organization publishes the current scope and


standards of home care practice?
a. Centers for Medicare and Medicaid Services
b. American Nurses Association
c. Robert Wood Johnson Foundation
d. Visiting Nurse Association
b. American Nurses Association

During the performance of which standard of care would


the nurse discover the need for Meals on Wheels?
a. Diagnosis
b. Planning
c. Assessment
d. Implementation
c. Assessment

Which concepts are used to evaluate client outcomes


when using the Omaha System Problem Rating Scale for
Outcomes?
a. Problem, Intervention, and Outcome
b. Knowledge, Behavior, and Status
c. Knowledge, Skill, and Attitude
d. Problem, Category, and Modifiers
b. Knowledge, Behavior, and Status

After conducting a comprehensive client assessment, the


nurse would organize information about the client's
medication regimen into which domain of the Omaha
System Problem Classification Scheme?
a. Environmental
b. Psychosocial
c. Physiological
d. Health-related behaviors
d. Health-related behaviors

In which situation would a nurse use the Omaha System


Problem Classification Scheme?
a. When measuring client outcomes
b. When collaborating with other professionals
c. When providing health education
d. When organizing a comprehensive assessment
d. When organizing a comprehensive assessment

Why would a nurse use the Omaha System?


a. To define each health care provider's role
b. To use nursing diagnoses in community health nursing
practice
c. To foster collaborative practice
d. To provide practitioners a method for communication
c. To foster collaborative practice

What action should a nurse take to facilitate


interprofessional collaboration?
a. Understand who is in charge of the client's care.

b. Make appropriate referrals.


c. Recognize what other professionals do and how they
view their roles.
d. Work in isolation when performing complex skills.
c. Recognize what other professionals do and how they
view their roles.

A home care agency has applied for accreditation from


the Joint Commission. What is the next step the agency
will take?
a. Attend a conference to learn more about the
accreditation process.
b. Schedule a site visit with the Joint Commission.
c. Complete a self-study of the agency.
d. Improve methods of documentation of client visits.
c. Complete a self-study of the agency.

A nurse uses the first stage of the Outcomes and


Assessment Information Set (OASIS) outcomes analysis
to:
a. Compare an agency's performance to a national
sample.
b. Report data on patient care to a national registry.
c. Stabilize a client's condition to continue to receive
funding.
d. Document client assessment data and findings.
a. Compare an agency's performance to a national sample.

Which historical event caused dramatic changes in home


care nursing?
a. Passage of Medicare legislation
b. Establishment of Visiting Nurse Associations
c. Creation of the Montefiore Hospital Home Care Program
d. Provision of funds by the Civil Works Administration
a. Passage of Medicare legislation

A nurse using telehealth technology in the home setting:


a. Uses WebTV to teach clients about their health
b. Shares health information using electronic
communications
c. Makes regular visits to clients to check the technology
d. Risks violation of the Health Insurance Portability and
Accountability Act
b. Shares health information using electronic
communications

Which populations may require a high-intensity


transitional care program? Select all that apply.
a. Adults with cognitive impairments
b. Women with high-risk pregnancies
c. Older adults with heart failure
d. People who are moving from one state to another
a. Adults with cognitive impairments
b. Women with high-risk pregnancies
c. Older adults with heart failure

Which are categories of classifying interventions


according to the Omaha System Intervention Scheme?
Select all that apply.
a. Health education
b. Case management
c. Treatments and procedures
d. Direct care services
b. Case management
c. Treatments and procedures

Which illegal and unethical activities have occurred in


health care systems in the past? (Select all that apply.)
a. Inappropriate use of home health services
b. Inaccurate billing for services
c. Excessive administrative staff
d. "Kick-backs" for referrals
a. Inappropriate use of home health services
b. Inaccurate billing for services
c. Excessive administrative staff
d. "Kick-backs" for referrals
School nurses give comprehensive nursing care to the
children and staff at a school. Which of the following are
also activities school nurses may do?
a. Care for children who are home sick from school
b. Coordinate the education program of the school and
consult with school officials to help identify and care for
other persons in the community
c. Allow students with communicable diseases to attend
classes
d. Provide tertiary levels of care as their primary focus

b. Coordinate the education program of the school and


consult with school officials to help identify and care for
other persons in the community

The National Association of School Nurses (NASN)


developed 11 criteria for school nurses. Among them are:
a. Interact with interdisciplinary health team members.
b. Communicate with teachers to effectively manage
classrooms.
c. Teach CPR to every member of the school staff.
d. Do research on students at the school.
a. Interact with interdisciplinary health team members.

Community outreach is a role or function that many


school nurses participate in. This may involve:
a. Case management that helps coordinate health care of
children with complex health problems
b. Health education within the school, such as teaching
about proper nutrition
c. Providing direct care to ill and injured children
d. Participating in community activities, such as health
fairs and immunization clinics
d. Participating in community activities, such as health fairs
and immunization clinics

Working with the national Safe Kids Campaign, the school


nurse can give educational programs reminding children
to use their seat belts or bicycle helmets to prevent
injuries. Another area where education is needed because
of the high potential for many injuries is:
a. School sports
b. Classrooms
c. Library activities
d. School busses
a. School sports

All states have requirements that children receive


immunizations or vaccinations against communicable
diseases before they attend school. Some parents request
that their child be exempt from the required
immunizations for medical, religious, or philosophical
reasons. What action might a school nurse take related to
this request? The nurse:
a. Will deny entry into school for children without the
required immunizations
b. Must educate the parents about the need for

immunizations
c. Should be aware of the laws in the state regarding
acceptable reasons for immunization exemption
d. Should allow the student to attend school without the
immunizations
c. Should be aware of the laws in the state regarding
acceptable reasons for immunization exemption

Which of the following are general guidelines for


administration of medications at school?
a. A current medication book should be available in case
information is needed.
b. The nurse should give medications brought in from
home by the child.
c. Medications cannot be given without a physician order.
d. Narcotics and controlled substances should be locked
up.
a. A current medication book should be available in case
information is needed.

When a nurse identifies a child who may be abused, or


receives information from a teacher that a child may have
been abused, the nurse must:
a. Call the police immediately to take the child to a safe
place
b. Contact the parents and ask about it
c. Contact the legal authorities as well as the school's
principal
d. Examine the child to see if the abuse really occurred
c. Contact the legal authorities as well as the school's
principal

The future of school nursing is strong. Which of the


following may occur in the future in the practice of school
health?
a. Telehealth and telecounseling to teach health
education
b. Robotic school nurses
c. Nurses working as teachers as well as nurses
d. Setting up private practices for treatment of the entire
family
a. Telehealth and telecounseling to teach health education

The Individuals with Disabilities Education Act (IDEA)


guarantees:
a. Sex education classes

b. Educational services
c. Day care
d. Orthodontics
b. Educational services

The National Association of School Nurses recommends


that school nurses be which of the following?
a. LVNs/LPNs
b. RNs with BSN and special certification in school nursing
c. School nurse practitioner
d . MSN with specialization in school nursing
b. RNs with BSN and special certification in school nursing

When coordinating the health care for children with


complex health problems, the nurse is performing which
of the following roles?
a. Health educator
b. Case manager
c. Consultant
d. Counselor
b. Case manager

Which of the following roles is the school nurse


performing when providing health information to school
administrators, teachers, and parent-teacher groups?
a. Health educator
b. Case manager
c. Consultant
d. Counselor
c. Consultant

A site that provides primary health care services to


children and youths by nurse practitioners in the school
setting is:
a. A primary care clinic
b. A school-based health center
c. School health services
d. A child health improvement plan
b. A school-based health center

Education to prevent health problems in children is:


a. Primary prevention
b. Secondary prevention
c. Tertiary prevention
d. Assessment
a. Primary prevention

The leading cause of death in children and teenagers is:


a. Leukemia
b. Injuries
c. Suicide
d. Diabetes
b. Injuries

A very high body temperature can lead to death when


taking which of the following drugs?
a. GHB
b. Rohypnol
c. Alcohol
d Ecstasy
d Ecstasy

The nurse caring for children when they need health care
is providing which of the following levels of prevention?
a. Primary prevention
b. Secondary prevention
c. Tertiary prevention
d. Health promotion
b. Secondary prevention

Which of the following is needed for the school nurse to


give a student prescription medication?
a. Copy of the prescription label on bottle
b. The smallest container needed for the medication
c. Contact with the physician
d. Signed consent from the student
c. Contact with the physician

19. Which of the following is true of lice?


a. Most cases are found in teenagers.
b. Most cases are found in white, middle-class children.
c. Most cases found in those with dirty hair.
d. There are between 1 and 5 million cases every year in
the United States.
b. Most cases are found in white, middle-class children.

The third-leading cause of death in teenagers is:


a. Cancer
b. Suicide
c. Accidents
d. Drug overdose
b. Suicide

The leading cause of children being absent from school as


a result of a chronic illness is:
a. Diabetes
b. Cancer
c. Asthma
d. Autism
c. Asthma

Providing ongoing care to the pregnant teenager is


considered:
a. Primary prevention
b. Secondary prevention
c. Tertiary prevention
d. Health promotion
c. Tertiary prevention

The school nurse cares for children who are injured or


become ill in the schools. A plan should be in place in
case of an emergency. Elements of this plan should
include which of the following? Select all that apply.
a. How to make arrangements to transfer children to the
hospital via ambulance
b. Knowledge of the staff about standard precautions
c. Individual emergency plans for students with health
problems
d. The role the children will play in treatment and triage
of fellow classmates
a. How to make arrangements to transfer children to the
hospital via ambulance
b. Knowledge of the staff about standard precautions
c. Individual emergency plans for students with health
problems

Violence in schools has become a serious problem. The


school nurse may be able to identify students who will act
in this way. There are several characteristics that may
help point out a student who may be thinking about such
violence. They include which of the following? Select all
that apply.
a. Venting
b. Vocalizing

c. Vandalism
d. Victimizing
a. Venting
b. Vocalizing
c. Vandalism
d. Victimizing
Carrier
A person who harbors an infectious organism and transmits
the organism to others although having no symptoms of
the disease

colonization
The presence and multiplication of infectious organism
without invading damage to tissuse

Common soucre outbreak


An outbreak characterized by exposure to a common,
harmful substance

Contagious
Able to be passed easily from one person to another.

endemic
The constant or ususal prevalence of a specific disease or
infectious agent within a population or geographic area

Epidemic
A widespread outbreak of an infectious disease.

Healthcare-associated infection
An infection that was not present on admission to the
hospital and develops during the course of treatment for
other conditions (nosocomial)

Incubation period
The period between infection and the appearance of
symptoms of the disease.

Infectious disease
A disease that is caused by a pathogen and that can be
spread from one individual to another.

propagated outbreak
an outbreak that spreads from person to person rather than
from a common source

pathogenicity
The ability to cause disease

Reservoir
A place where microorganisms survive, multiply, and await
transfer to a susceptible host

Secondary infection
caused by an opportunistic pathogen after the primary
infection has weakened the body's defense

Surveillance
- collecting, analyzing, & reporting data on rates of
occurrence, mortality, morbidity, and transmission of
infections

transmission
An incident in which an infectious disease is transmitted to
another person/people.

Antigenic drift
Slow and progressive genetic changes that take place in
DNA and RNA as organisms replicate in multiple hosts

Antigentic shift
dramatic change,

Convergence model
Model illustrating the interaction of 13 factors that
contribute to the emergence of infectious disease.

Directly observed therapy


best treatment for TB, when patients take their drugs in
front of healthcare workers-- decreases odds of drugresistant strains

Ecosystem
A community of interdependent organisms and the physical
environment they inhabit.

Emerging infectious disease


Diseases that are new, increasing in incidence, or showing
a potential to increase in the near future

Herd immunity
The resistance of a group to invasion and spread of an
infectious agent, based on the resistance to infection of a
high proportion of individual members of a group; the
resistance is a product of the number susceptible and the
probability that those who are susceptible will come into
contact with an infected person

Microbial adaption
Process by which organisms adjust and change to their
enviroment

Pandemic
A worldwide epidemic.

Femicide
A term used to refer to a homicide that occurs in the
context of intimate partner violence (IPV).

Gender-based violence
any act of gender violence that results in or is likely to
result in physical, sexual or psychological harm and
suffering to women, including theats of such acts, coercion,
or arbitrary deprivations of liberty, whether occuring in
public or private life.

Human rights
The basic rights to which all people are entitled as human
beings.

Incidence
Disease is the number of new cases occurring in a specific
period

Intimate partner violence


domestic violence or battering between two people in a
close relationship

Lethality assessment

Attempt to predict likelihood of suicide

Perpetrate
To commit, as a crime or other antisocial act

Prevalence
'The number or proportion of cases of a particular disease
or condition present in a population at a given time.'"snap
shot" (AIHW, 2008)

Violence
1. Violence is a broad concept that ranges from homicide to
physical assault or intentional use of force against another
person, community, or even against oneself (suicide)

World Health Organization


An international body of health care professionals,
including clinicians and epidemiologists among many
others, that studies and responds to health needs and
trends worldwide.

Abstinence
Act of refraining from for 12 months or more

Addiction
habitual use of a drug, a disease in itself. Caused by taking
a chemically dependent drug "prescribed or illegally used
compulsively psycological and physiologicaL dependency

Craving
an intense desire for some particular thing

Intoxication
An altered state of consciousness as a result of drinking
alcohol or ingesting other substances

Substance abuse
A) Substance abuse differs from dependence in that
withdrawal or tolerance have never been present, but use
of the substance has caused significant impairment or
distress - e.g., missing work, school failure, DUI, legal
problems

Substance dependence

A diagnostic term used in clincial psychology and


psychiatry that identifies an individual displaying
significant signs of a dependent relationship with a
psychoactive drug.

Substance use
This is the use of a substance that alters physical or mental
functions: It can be legal or illegal, therapeutic or
recreational and can be done by sniffing, snorting, inhaling,
swallowing, drinking, smoking, or injecting the substance or
absorbing it throught the skin.

Substance use disorder


Dependence and abuse

Tolerance
A process in which your body needs more and more of a
drug to get the same effect

Withdrawal
Physical signs of discomfort associated with the
discontinuation of an abused substance.

Health professional shortage area


rural health care services, especially in regions with
insufficient numbers of all types of health care providers
have become a national priority since the early 1990s

Inmate
a person serving a sentence in a jail or prison

Medically underserved area


as a result nurses work in community-based settings
providing health promotion and disease prevention to the
homeless, mentally ill, and others who have limited access
to health care or who lack health care insurance

Medically underserved population


Rising rates of unemplyoment, homelessness, and rising
health care cost contribute to the increase in the medically
underserved population. Many nurses are working to serve
this popultaion (building clinics where medical help is not
available)

Trimorbidity
Three common chronic illnesses of homeless people
(mental illness, chronic physical illness, substance
addiction)

Underserved population
A subgroup of the population that has a higher risk of
developing health problems because of a greater exposure
to health risks because of marginalization in sociocultural
status, access to economic resources, age or gender

Bioavailabilty
The amount of a contaiminant that actually ends up inthe
systemic circulation

Biomonitoring
Analysis of blood, urine, tissues, and so forth to measure
chemical exposure in humans

Environmental epidemiology
field of public health science that focuses on the incidence
& prevalence of disease or illness in a population from
exposures in their enviroments

Environmental health
Environmental factors that influence human health and
ecological systems

Environmental justice
A legal strategy based on claims that racial minorities are
subjected disproportionately to environmental hazards

Exposure
A situation in which a person has had contact with blood,
body fluids, tissues, or airborne particles in a manner that
suggests disease transmission may occur.

Exposure estimate
Factors that determine a persons level of exposure to a
contaminant.

Exposure pathway
The pathway linking the environmental source of a
contaminant to the point of exposure

Exposure history
process to help determine whether an individual has been
exposed to environmental contaminants

Precautionary principle
States that when there is a threat of serious or irreversible
environmental damage, we shouldn't wait for scientific
proof before taking action.

Healthy communities
communities that optimize the physcial, social, and
economic environment of the community

Risk assessment
Assessing each risk involves determining likelihood that the
risk event will occur and the degree of impact the event
will have on the project objective.

Toxicology
study of poisonous substances and their effects upon body
parts

After-action report
Retrospective analysis used to evaluate emergency
response drills.

Decontamination
A process in which recently used and soiled medical
devices, including instruments, are rendered safe for
personnel to handle.

Evacation
Moving people from a dangerous place to safety

Incident Command system


a system by which facilities, equipment, personnel,
procedures, and communications are organized to operate
within a common organizational structure designed to aid
in the management of resources at emergency incidents.

Invacuation
Moving people from one area to another, within the same
facility.

National response framework


framework that guides how the nation conducts all-hazards
incident response

National incident management system


the management system used by federal, state, and local
governments to manage emergencies in the United States.

Personal protective equipment


Those items of protection worn to minimize exposure to
hazards; those items worn by the embalmer to avoid
contact with blood and other body fluids.

Point of distribution
centralized location where the public picks up emergency
supplies following a disaster

Real time
Means that the people who are conversing on a computer
are online at the same time.

Scenario
A sequence of events that is imagined, assumed, or
suggested

Shelter-in-place
Taking refuge inside an interior room with few or no
windows, in the event of the release of chemical biological,
or radiological contaminants:

Simulation
A representation of a situation or problem with a similar but
simpler model or a more easily manipulated model in order
to determine experimental results.

Terrorism
Is the use or threatened use of violence to gain a political
objective

Black box warning


A type of warning that appears in a drug's prescribing
information, required by the U.S. Food and Drug

Administration alerting prescribers of serious adverse


events that have occurred with the given drug

Community mental health center


A facility offering a wide range of mental health services,
such as prevention, counseling, consultation, and crisis
intervention.

Culturally competent mental health service


Health care that is sensitive to the needs and health status
of different population groups.

Deinstitutionaliztion
a focus on reducing costly and neglectful institutional care
and on providing more humane services in the community

Diagnostic and Statistical manual


A widely used system for classifying psychological
disorders

Early intervention programs


program directed at young children who are at risk for later
problems, intended to prevent problems from developing

First-generation antipsychotics
improve positive symptoms of schizophrenia more
effectively than negative symptoms or cognitive
dysfunction.

Nonadherence
an informed decision on the part of the patient not to
adhere to or follow a therapeutic plan or suggestion. Same
thing as noncompliance.

President's new freedom commission on mental health


2003, called from streamlined system with less fragmented
delivery, advocated early diagnosis and treatment, new
expectation for principles of recovery, increased assistance
in helping people find housing and work.

Prodromal stage
This is the Initial stage of disease, characterized by
common general complaints of illness malasia and fever ?

Second-generation antipsychotics
drugs that alleviate schizophrenia without serious risk of
producing movement disorders

tardive dyskinesia
A side effect of long-term use of traditional antipsychotic
drugs causing the person to have uncontrollable facial tics,
grimaces, and other involuntary movements of the lips,
jaw, and tongue.

Americans with disabilities act


Passed by Congress in 1991, this act banned discrimination
against the disabled in employment and mandated easy
access to all public and commerical buildings.

Community school model


Collaborative design that uses the resources of a
community to provide structure prevention services such
as after-school programs

Early periodic screening diagnosis and treatment


(EPSDT) program covering screening and diagnosis
services in recipients younger than 21 years of age

Individuals with disabilities education act


Persons who have a physical or mental impairment that
substantially affects one or more major life activites

School health council


coalition of a wide range of community stakeholders
including family & student representives who contribute to
the development of action plans designed to improve the
health & safety of the students

School health index


data collection tool that helps school identify the strengths
& weaknesses of an individual school's policies & programs
for promoting health & safety. Action plans can be
developed

School health nursing


specialized practice of professional nursing that advances
the well being accademic successs & lifelong achievement
of the students

Youth risk behavior surveillance survey


survey conducted biannually; measures the prevalence of
six categories of health risk behaviors among youths
through representative national, state, and local surveys
using a self-report questionnaire

congregation
A gathering of people, especially for religious services

congregration-based model
faith based nurse serving a particular faith community by
virtue of a contract or job description

Institution-based model
the parish nurse serves as a liasion and helps plan and
coordinate care

Parish nursing
a model of care for members of a faith community that can
include wellness care through home visiting

Spiritual care
Recognition of spiritual needs and the assistance given
toward meeting those needs.

Adjuvant drugs
relieve pain either alone or in combination with analgesics.
These drugs can potentiate or enhance the effectiveness of
the analgesic.

Advance directives
Documents that indicate a patient's wishes in the event
that the patient becomes incapacitated and unable to
make decisions regarding medical care

Bereavement
Feeling of grief or desolation, especially at the death or loss
of a loved one.

Breakthrough pain
Pain that occurs between doses of pain medication. (p.
157)

Comfort measures only


goal of treatment is a comfortable dignified death & that
further life-sustaining measures are no longer indicated.

Grief
Extreme sadness after someone dies or goes away.

Healthcare proxy
an individual who has been appointed to make health
decisions by the maker in the event of their incapacitation

Hospice home care


care of clients and families who have 6 months or less to
live in what they consider their home

Mourning
Outward, social expressions of grief and the behavior
associated with loss.

Pallative care
care designed not to treat an illness but to provide physical
and emotional comfort to the patient and support and
guidance to his or her family

Postmortem care
Care given immediately after death before the body is
moved to the mortuary usually by nurses

American Association of occupational health nurses


Professional assocation for nurses working in a business
setting, dedicated to the health and saftey of workers,
worker populations and community group

Biological hazards
Any living organisms or its properties that can cause an
adverse response in humans

Chemical hazards
Toxicity and the use of the chemicals. Excessive airborne
concentrations of mists, vapors, gases, or solids in the form
of dusts or fumes

Ergonomics

The study of workplace equipment design or how to


arrange and design devices, machines, or workspace so
that people and things interact safely and most efficiently.

National institute of occupational safety and health


Fedral agency established to help ensure safe and healthy
working conditions by conducting scientific research,
gathering information, and providing education and
training in occupational saftey and health

Occupational health history


An assessment of the characteristics of the workers present
job, a chronological record of all past exposures,
occupational exposure inventory, and lists of other
exposures at home or in community.

Occupational safety and health administration


U.S. law authorizing the federal government to establish
and enforce occupational safety and health standards for
all places of employment engaging in interstate commerce

Physical hazards
Hazards that result from the transfer of physical energy to
workers

Psychosocial hazards
- all organizational factors and interpersonal relationships
in the workplace that may affect the health of the workers

Root cause analysis


An analytical technique used to determine the basic
underlying reason that causes a variance or a defect or a
risk. A root cause may underlie more than one variance or
defect or risk.

Workplace walk through


A complete survey of the workplace, inside and outside,
compiling information as to the presence of hazards, the
location of entries and exists, the availability of emergency
equipment and potential trouble spots.
Which of the following are responsible for the majority of
food borne illnesses?
A. E.Coli and Salmonella

B. Salmonella dna Campylobacter


C. Botulism and Salmonella
B. Salmonella dna Campylobacter

A resturant had a food borne illness and several people


were affected. Which of the following would you consider
sending to the ER for treatment?
A. A 23-year-old with 2 day history of diarrhea
B. A 15-month-old with vomiting and diarrhea X 1 day
C. A 78-year-old woman with a temperature of 101
B. A 15-month-old with vomiting and diarrhea X 1 day

___ is the measure of the potential ability of the host to


transmit to another.
Infectiousness

___ is the time interval between invasions of the


infectious agent and the appearance of symptoms.
Incubation period

___ is the interval where the infectious agent can be


transferred to another.
Communicable period

___ is the constant presence of a disease in a geographic


area or population.
Endemic

___ is the occurrence of a disease in excess of the normal


distribution.
Epidemic

___ is a worldwide occurrence of a disease in excess of


the normal distribution.
Pandemic

___ is the irreversible extermination of an infectious


agent.
Eradication

___ is the removal of a disease from a large geographic


area.
Elimination

___ is a disease endemic in Arizona.


Coccidiodomycosis

___ is a disease that is pandemic.


HIV/AIDS

___ is an environmental cause of emerging diseases.


Global warming

Flu shots are an example of ___ ___.


Primary prevention

What are the three points on the epidemiological triangle


stemming from vector?
Host
Agent
Environment

Name the vaccine-preventable diseases.


Hepatitis A
Hepatitis B
Measles, mumps, rubella (MMR)
Diphtheria, pertussis, tetanus (DTaP)
Varicella
Polio (IVP)
Haemophilus influenza type B (HIB)
Pneumonia & influenza
Human papilloma virus (HPV)
Rotovirus

What event might potentially indicate acts of


bioterrorism, leading to the need for follow-up and
surveillance by the public health nurse?
Healthy adults getting chickenpox without exposure to
children with chickenpox

What actions might a public health nurse take to prevent


the spread of communicable disease?
Intensify surveillance
Increase healthcare surge capacity
Develop public education & communication
Stockpile medical supplies & antiviral drugs
Vaccine development & distribution
Develop plans for businesses & schools

Ensure access to clean water


Prevent food-borne illness

(True/false) Food-borne illness will affect all people at a


site, not just one person.
True

When should someone with suspected food-borne illness


seek medical treatment?
High fever (101.5 F + orally)
Blood in stool
Prolonged vomiting
Dehydration
Prolonged diarrhea 3+ days

What are the symptoms of food-borne illness?


Fever
Diarrhea
Abdominal cramps
Vomiting

What is the typical treatment for food-borne illness?


Dehydration treatment (IV/oral)
Antiemetics
Antidiarrheals

What should someone do if he is bitten by an animal and


is unsure if that animal has rabies?
Get rabies treatment (shot in abdomen) - secondary
prevention

What are the three types of agents that cause infection


and infectious disease?
Infectious agents: bacteria, viruses, fungi, parasites
Chemical agents: heavy metals, toxins, chemicals,
pesticides
Physical agents: radiation, heat, cold, machinery

What host characteristics influence the spread of disease?


Genetic susceptibility
Immutable characteristics
Acquired characteristics (immunity)
Lifestyle factors (diet & exercise)

What environmental factors facilitate transmission of


infectious agents from an infected host to other
susceptible hosts?
Climate: temp, rainfall
Plant & animal life
Human population distribution
Socioeconomic factors: education, access to care,
resources
Working conditions

(True/false) All diseases require a vector.


False

Vectors, such as anthropoids (ticks, mosquitos), transmit


infectious agents by biting or depositing infective
material near the host. They may be necessary to the life
cycle of the organism (mosquitos and malaria), or may
act as ___ ___, as flies do with food.
Mechanical transmitters

What intervention might the health department


implement for foreclosed homes with unkept swimming
pools?
Provide fish that eat mosquitos that breed in standing
water

A(n) ___ ___ is a disease transmitted through direct


contact with an infected individual or indirectly through a
vector.
Communicable (contagious) disease

A(n) ___ ___ is a disease resulting from the presence and


activity of a pathogenic microbial agent. It is potentially
transferrable.
Infectious disease

What is the number-one cause of death worldwide?


Communicable/infectious disease

What are the three leading causes of death for children


under 5?
Pneumonia
Diarrhea
Malaria

How is hepatitis B transmitted?


Blood/body fluids
Perinatal
Unsafe injections & transfusions
Sexual contact
Can survive for at least one week, dried, at room
temperature on environmental surfaces

What risk factors increase an individual's risk for


contracting an STI?
Multiple sex partners
Unprotected sex
Drugs & alcohol
Having other STIs
Failure to use safety measures
Age less than 25 years
Sharing needles
Douching

How can STI transmission be prevented?


Abstinence
Condom use (not 100% effective)
Long-term, monogamous relationships
Early detection through case-finding & screening

How can hepatitis B transmission be prevented?


Immunization
Universal precautions
Testing of pregnant women
Reducing sexual & injection drug use exposure

What are the major symptoms of most STIs?


Itching
Burning with urination
Pain
Bumps, blisters, vesicles
Drainage
Epididymitis
Enlarged lymph nodes
Eye infection
Sore throat
Sores in/on mouth
(Anything someone wasn't born with)

What are the major symptoms of acute hepatitis B, which


can last several months to a year during which time
antibodies are developed?
Flu-like symptoms
Jaundice
Extreme lethargy
Nausea
Fever
Joint pain

What are the major symptoms of chronic hepatitis B?


Anorexia
Fatigue
Hepatomegaly
Jaundice
Causes liver diseases such as cancer & cirrhosis

Which STIs are curable with antibiotics?


Chlamydia
Gonorrhea
Syphilis

What chronic STIs require a lifetime of symptom


management and infection control?
HIV
HPB
HPV
HSV

Why might younger persons be at greater risk for


contracting an STI?
Drug & alcohol use, leading to poor decisions/lack of
protection

A patient presents at a community clinic complaining of


extreme itching in her genital region. What would be an
appropriate action by the nurse?
Assess & determine if other symptoms are present
Ask, "have you had unprotected sex?"

What is expedited partner therapy (EPT)?


Individuals diagnosed with an STI are given medication for
himself/herself AND his or her partner

A ___ period is the time during which a disease is present,


but symptoms have not appeared.
Latency period

(True/false) An individual tests positive for TB, but her


chest x-ray is negative, indicating latent TB. This person
is still contagious because the disease is present in her
body.
False

(True/false) Someone with a latent STI can still transmit it


to others.
True

How are gonorrhea and chlamydia diagnosed?


Urine
Swabs - endocervical, anus, urethra, pharynx, tonsils

How are syphilis, herpes, and hepatitis B diagnosed?


Blood test

What is the CDC's recommendation for STI testing?


Annual testing of all sexually active women 25 or younger
Older women with new sex partner or multiple sex partners
All pregnant women

(True/false) Partner notification must always be done for


patients who test positive for HIV.
False - don't know where/when it was contracted

(True/false) Community health nurses may treat patients


for more than one STI without confirmation if the
individual tests positive for one STI.
True - cocktail that knocks out most STIs can be given
Cheaper, don't have to keep testing, and able to acquire
consent & treat immediately

Partners of the last ___ days of individuals diagnosed with


STIs must be treated at the same time.
60 days

(True/false) Expedited partner therapy (EPT) is used for


individuals who are diagnosed with HIV.
False - other STIs

When an individual is diagnosed with an STI, notification


of partners is (mandatory/voluntary).
Voluntary

What are some chronic complications that can result from


STIs?
Syphilis: dementia, blindness, joint disease, heart disease
G/C: PID, infertility, ectopic pregnancy, serious reproductive
problems
HPV: abnormal Paps, cervical cancer

Nearly ___% of children with congenital syphilis, which is


acquired during gestation when syphilis is not treated, will
die before or shortly after birth.
50%

Which STIs can cause eye infections in newborns if


untreated in pregnant women?
Gonorrhea/chlamydia - prophylaxis with erythromycin or
tetracycline eye drops

What education components would a nurse include when


teaching teens about safe sex using condoms?
Always use new one
Use latex
Water-based lube
Put on and take off erect penis
Use spermicidal
Use latex dams for oral sex

Who is at the highest risk for contracting hepatitis B?


IV drug users
Persons with STIs/multiple sex partners
Immigrants & refugees
Descendants from areas of high endemic rates of HBV
Health care workers
Hemodialysis clients
Inmates of long-term correctional institutions

(True/false) Symptoms of hepatitis A are present only in


the acute phase of the disease process.
True

(True/false) Someone with open sores due to a syphilis


infection puts that individual at greater risk for
contracting other STIs, such as HIV.
True

How is TB transmitted?
Exposure to bacilli in airborne droplets form infected person
during coughing, sneezing, or talking

Describe the incubation period and appearance of


symptoms in someone with TB.
Incubation period: 4 - 12 weeks; time from inhaling droplets
until S/S appear
Clinical symptoms: 6 - 12 months

(True/false) Sharing a drink with someone who tested


positive for TB can potentially result in transmission.
False - must be breathed in

What is the second leading cause of death from infectious


disease worldwide?
TB
Always worse in other countries

With regard to TB, the infection in about ___% of those


infected becomes latent and may be reactivated later in
life.
95%

Why might someone who is exposed to TB have a


nonreactive test result, despite having inhaled the
droplets?
Have not built up antibodies to TB yet - may take up to 6
weeks

Why might someone with HIV have a nonreactive TB skin


test, despite having inhaled TB droplets?
Immunosuppressed - WBC count is 0 & cannot build up
antibodies for test to be reactive

A patient arrives to the community clinic 48 hours after


receiving a TB skin test to have it evaluated and the
nurse discovers 15 mm induration. The patient is
diagnosed with latent TB. What should the nurse say/do?
"What we do know at this point is that you have been
exposed to TB because you responded to the test. This
means that you may go on to develop TB."
Patient must go to health department - will conduct
thorough history (traveling included) & CXR

A patient with diabetes has her TB skin test read and the
nurse discovers 11 mm induration. Further testing reveals
no symptoms and a negative chess x-ray. What does the
nurse suspect?
A. Active TB infection
B. No TB infection
C. Latent TB infection
D. Further testing is required
C. Latent TB infection

(True/false) An HIV-positive individual is diagnosed with


latent TB; this person can either choose to be treated
prophylactically or not receive treatment.
False - HIV patients must always be treated for latent TB

Why might patients diagnosed with latent TB choose not


to receive the prophylactic medications?
Develop antibiotic resistance
Hepatotoxic drugs - alcoholics, drug users, or liver disease
patients want to keep using & not take TB meds

What testing must be completed before administering TB


medications to a patient?
Liver status/LFT: enzymes, clotting factors, etc.

What populations are at an increased risk for contracting


TB?
Homeless
Prisons
Nursing homes
Migrants/immigrants/TB endemic in home countries
Immunocompromised (HIV, chemo, organ transplant)

What preventive action might a nurse take when


admitting older adults to a nursing home?
Test for TB before allowing in facility

Why might an individual show induration on a TB skin test


when he or she has not ever been exposed or had TB?
Had BCG vaccine

What diagnostic tools are used to determine the presence


of TB?
Skin test (48 - 72 hours)
Quantiferon test (blood draw rather tan skin test - can get
next day)
Chest x-ray
Acid-fast bacillus (AFB) smear if symptoms (cough sputum
into petri dish, stained)
H & P (traveling history)
Cultures (can take up to 6 weeks - quarantine until then)

For patients with HIV, a positive chest x-ray, or close


contact with someone who has TB, a TB skin test with ___
mm induration is considered positive.
5 mm

For patients with diabetes, ETOH abuse, drug use/IVDs,


foreign-born where TB is common, medically
underserved, low-income, long-term care, in prisons, or
children under 4, a TB skin test with ___ mm induration is
considered positive.
10 mm

For patients without definite medical conditions, a TB skin


test with ___ mm induration is considered positive.
15 mm

A homeless patient arrives at the community clinic to


have his TB skin test read and the nurse discovers 10 mm
induration. What is the best course of action?
Considered positive
Assess for symptoms
CXR to ensure not contagious

(True/false) An annual chest x-ray is recommended for


medical personnel to ensure TB is not present.

False - too much radiation

(True/false) Anergy testing, which is used to obtain


information regarding the competence of the cellular
immune system, is recommended routinely for screening
for those with HIV.
False - no longer recommended; now look at WBC count to
assess immune system

Two-step TB testing, which utilizes a booster effect, is


used for what individuals?
People over 55 years
New employees to establish accurate baseline

A 60-year-old woman receives a TB skin test and nothing


happens. What might be the basis for this? What is the
best course of action?
Immunocompromised
False negative: sluggish immune system does not respond
quickly enough
Repeat skin test in 1 week

A 71-year-old man receives a TB skin test and upon


reading it, the nurse notes that there is no response. The
man receives a second skin test (per two-step testing) a
week later. Why might the second test be positive when
the first was negative?
Booster effect - sluggish immune system did not respond
fast enough to first skin test, even though the man was
positive for TB

The Bacillus Calmette-Guerin (BCG) vaccine has what


effect on TB skin tests?
Can lead to false positive due to antibodies

Patients with active TB infections are considered


contagious for at least ___ weeks after starting medication
treatments.
2 weeks

What intervention may be necessary for a patient with an


active TB infection who refuses to wear a mask outside or
take his medications?
Quarantine in hospital - DOT!

In addition to medications, what interventions are part of


the treatment plan for TB?
Contagious for 2 weeks after beginning meds
Repeat labs - 3 negative smears (early morning sputum)
Home isolation
Masks when outside
Monroe House (for homeless with TB)

Persons who have latent TB infections have a ___%


chance of developing TB disease. This risk is greatest
during the first year after becoming infected and can be
activated by immunosuppression.
10%

On average, people have a ___% chance of becoming


infected with TB if they spend 8 hours a day for 6 months
or 24 hours a day for 2 months working or living with
someone with active TB.
50%

How is treatment for multi-drug-resistant (MDR) TB


different than that of regular TB?
Prolonged treatment (2 years)
Multiple drugs
High mortality rate

What types of interventions may be necessary for


individuals with drug-resistant TB to ensure adherence?
Directly-observed therapy (DOT)
Directly-observed prophylactic therapy (DOPT)

Why has there been a resurgence of TB in the United


States?
HIV/AIDS epidemic
Increased immigration
Increased poverty, injection drug use, homelessness
Poor compliance with treatment
Increased residents in long-term care
Deterioration of public health services
Onset of MDR TB

Giant rats may now be used to sniff out TB infections.


How might this be beneficial compared to traditional
treatment?

Cheaper
More efficient - success rate 67%
2000 samples/day vs. technicians that can only process 20
samples/day

What institutions in Arizona are aimed at controlling TB?


Maricopa County Department of Health
Diagnosis & treatment
DOT/DOPT
Monroe House

What factors should be evaluated when determining if a


patient should receive prophylactic treatment of latent
TB?
Benefits of treatment - evaluate individual's risk for
developing TB
Assess level of commitment to completion of treatment &
resources available to ensure adherence

What kinds of people should be given high priority for


latent TB infection treatment?
+IGRA/5+ mm induration:
HIV
Recent contacts of TB case
Fibrotic changes on chest radiograph consistent with old TB
Organ transplant recipients
Immunocompromised
Taking steroids, etc.
+IGRA/10+ mm induration:
Recent immigrants (<5 years) from high-prevalence
countries
Injection drug users
Residents/employees of high-risk congregate settings
(nursing homes, homeless shelters, correctional facilities)
Mycobacteriology lab
Children under 4
Children/adolescents exposed to adults in high-risk
categories

What symptoms often result from medications used to


treat TB?
Hepatitis
GI upset
Drug interactions

Bleeding problems
Flu-like symptoms
Rash
Turns urine, saliva, tears orange (discolors contact lens)
Photosensitivity
Birth control pills/implants less effective
Hepatic enzyme elevation
Peripheral neuropathy
Mild CNS effects
Joint aches
Hyperericemia (uric acid in blood)
Optic neuritis
Ototoxicity & nephrotoxicity

A school nurse notes that 60 children have missed days of


high school because of pertussis this past year and this
rate has been relatively constant for the past 5 years. The
nurse plans to work with the community to increase
awareness of the seriousness of this disease for children
younger than 6 months of age and to raise and maintain
the immunization rates, because in this community the
pertussis is:
A. Endemic.
B. Epidemic.
C. Pandemic.
D. Sporadic.
A. Endemic.

The World Health Organization (WHO) developed the Five


Keys to Safer Food campaign in 2001 to address the
problem of foodborne and waterborne diarrheal diseases
worldwide. This campaign emphasizes which of the
following practices?
A. Keep clean, separate raw and cooked, cook thoroughly.
B. Never use raw, always cook, buy better.
C. Wash, cut, cook, and throw away.
D. Wash, cover, and always refrigerate.
A. Keep clean, separate raw and cooked, cook thoroughly.

An American takes a long-awaited vacation in sunny


Mexico, spending days on the beach eating fresh
raspberries from a nearby vendor and drinking bottled
water. The tourist may be altering:
A. Agent-host-environment interaction.
B. Circadian rhythms.
C. Herd immunity.
D. Host resistance.

A. Agent-host-environment interaction.

An example of secondary prevention of infectious disease


is:
A. Malaria chemoprophylaxis.
B. Pneumocystis carinii pneumonia chemoprophylaxis for
people with AIDS.
C. Quarantine.
D. Restaurant inspections.
C. Quarantine.

In an effort to address West Nile virus, a community


increased livestock immunization, began a vector control
program, and initiated a community campaign to
eliminate standing water reservoirs. This best exemplifies
communicable disease control through:
A. Health education.
B. Multisystem approach.
C. Improved public health infrastructure.
D. Reduction of environmental hazards.
B. Multisystem approach.

Emerging infectious diseases may arise as a result of


factors operating singly or in combination, and these
factors may include which of the following (select all that
apply)?
A. Environmental changes.
B. Host behavior.
C. Improved surveillance.
D. Microbial adaptation.
E. Public health infrastructure deterioration.
A. Environmental changes.
B. Host behavior.
D. Microbial adaptation.

What are some primary prevention techniques for


preventing the occurrence of disease?
Responsible sexual behavior
Malaria chemoprophylaxis
Tetanus boosters, flu shots
Rabies pre-exposure immunization
Safe food-handling practices in the home
Repellants for preventing vector-borne disease
Not allowing unvaccinated kids to attend school
Regulated & inspected municipal water supplies

Bloodborne pathogen regulations


Restaurant inspections

What are some secondary prevention techniques for


preventing the occurrence of disease?
Immunoglobulin after hepatitis A exposure
Immunization & chemoprophylaxis for meningococcal
outbreak
Rabies post-exposure immunization
TB screening for healthcare workers
STI partner notification
HIV testing & treatment
Quarantine

What are some tertiary prevention techniques for


preventing the occurrence of disease?
Pneomocystis pneumonia (PCP) chemoprophylaxis for
people with AIDS
Regular inspection of hands & feet/protective footwear &
gloves to avoid trauma/infection for leprosy clients who
have lost sensation in those areas

A school nurse is teaching a class of sophomores about


the relationship between the risk of sexually transmitted
disease (STD) and risk-taking behaviors. A key point to
include is:
A. All STDs are easily preventable with consistent condom
use.
B. Once a young woman is pregnant, she is no longer at
risk for most STDs.
C. STDs are most likely to be transmitted during a
student's initial sexual encounter.
D. Use of alcohol and drugs makes a student more likely
to make decisions that result in exposure to and infection
with STDs.
D. Use of alcohol and drugs makes a student more likely to
make decisions that result in exposure to and infection with
STDs.

A client newly diagnosed with human papillomavirus


(HPV) infection, herpes simplex virus 2 (HSV-2) infection,
and syphilis asks, "Okay, so how do I get rid of all this
stuff?" In developing a plan of care, the nurse recognizes
that it is essential to address:
A. Correct use of condoms to prevent transmission of all
sexually transmitted diseases (STDs).

B. Cures for each of the STDs identified.


C. Risk of skin-to-skin contact in transmitting the
identified STDs.
D. Safety of sexual contact in the absence of lesions.
C. Risk of skin-to-skin contact in transmitting the identified
STDs.

A nurse is assigned to teach clients sexually transmitted


disease (STD) prevention information. The nurse updates
her teaching plan to incorporate new guidelines from the
Centers from Disease Control and Prevention (CDC). She
includes which of the following as updated information
during her next teaching session?
A. Always use spermicides with condoms to reduce the
risk of contracting chlamydia or gonorrhea.
B. Condoms can be effective in preventing infections
transmitted by fluids from mucosal surfaces, but are not
always effective in preventing infections transmitted by
skin-to-skin contact.
C. Condoms should not be used during oral sex, because
they are not effective in preventing transmission of
infection.
D. When genital ulcers are present, condoms should be
used to prevent the spread of infection.
B. Condoms can be effective in preventing infections
transmitted by fluids from mucosal surfaces, but are not
always effective in preventing infections transmitted by
skin-to-skin contact.

A woman comes to the community health center


complaining of increasing lower abdominal pain, fever,
and abnormal menses for several months. During the
assessment, the client indicates that she is aware that
her husband has had multiple sex partners in the past
two years. Appropriate intervention by the nurse would
be to:
A. Arrange to have the client referred for medical
evaluation for pelvic inflammatory disease (PID) and
appropriate intervention and treatment.
B. Contact the health department to confirm the spouse's
diagnosis of Chlamydia infection to determine the client's
exposure, give the client antibiotics, and have her return
to the clinic if symptoms worsen.
C. Provide sexually transmitted disease (STD) prevention
and treatment education and refer the client to the health
department for STD screening for gonorrhea and/or
Chlamydia infection.

D. Supply the client with nonsteroidal antiinflammatory


drugs and caution her to call the after-hours call doctor if
her symptoms worsen.
A. Arrange to have the client referred for medical
evaluation for pelvic inflammatory disease (PID) and
appropriate intervention and treatment.

The clients most at risk of reactivation of latent infections


of tuberculosis (TB) are:
A. Immunocompromised persons, substance abusers, and
those with diabetes.
B. Individuals previously treated for TB.
C. Long-term cigarette smokers.
D. Persons with new-onset asthma or emphysema.
A. Immunocompromised persons, substance abusers, and
those with diabetes.

Community health nurses conducting health education


among populations vulnerable to HIV infection should
explain the natural history of the infection, including the
fact that HIV infection may go undetected during the
primary infection stage because:
A. Antibody test results are typically negative.
B. Antibody production by the immune system increases.
C. Incubation period is prolonged.
D. Symptoms include myalgias, sore throats, and rash.
A. Antibody test results are typically negative.

A client comes to the local clinic with acute symptoms of


fever, nausea, lack of appetite, malaise, and abdominal
discomfort. During the course of the assessment, the
nurse determines that the client is a health care aide
working at a daycare center. These facts are important
because:
A. Acute hepatitis B is self-limiting.
B. Hepatitis A outbreaks commonly occur in facilities
where staff change diapers.
C. Hepatitis C is a "silent stalker."
D. Individuals with chronic liver disease are at greater risk
for hepatitis A.
B. Hepatitis A outbreaks commonly occur in facilities where
staff change diapers.

Many behaviors place any individual-regardless of age,


gender, ethnicity, or other characteristics-at greater risk
for sexually transmitted diseases (STDs). The nurse

should include primary prevention interventions in all


client encounters through the discussion of:
A. Partner notification.
B. Safer sex.
C. Standard precautions.
D. STD testing.
B. Safer sex.

A population-level tertiary prevention intervention


typically carried out by nurses caring for those with
communicable disease in the community is:
A. HIV test results counseling.
B. Needle exchange.
C. Partner notification.
D. Instruction in standard precautions.
D. Instruction in standard precautions.

Disease and event surveillance systems exist to help


improve the health of the public through the systematic
and ongoing collection, distribution, and use of healthrelated data. A public health nurse utilizes the data in
available surveillance systems to make a presentation to
a community planning committee that is interested in the
problem of childhood obesity. This use of the data for
facilitating community planning is a good example of
helping the community to:
A. Avert a health care epidemic.
B. Evaluate prevention and control measures.
C. Identify cases for fact finding.
D. Manage endemic health problems.
D. Manage endemic health problems.

Disease surveillance, a critical role function for the public


health nurse, can best be defined as the ongoing,
systematic collection, analysis, interpretation, and
dissemination of specific health data relevant to public
health. The nurse's knowledge and understanding of
surveillance systems is critical to:
A. Detecting epidemics and pandemics.
B. Facilitating planning and policy changes.
C. Improving the quality and usefulness of the data
collected.
D. Understanding the relationship between epidemiologic
principles.
C. Improving the quality and usefulness of the data
collected.

A state public health region reported 39 cases of


meningitis in children 15 years of age and younger to
date this year. Seven of those children died. The total
population of the region is 780,000, of whom 84,000 are
children age 15 years old and younger. Only four cases of
meningitis were reported in the public health region
during the previous year. No other public health region in
the state has an incidence of meningitis that is higher
than expected for that region. Based on the information
given, the relative frequency of meningitis in the region at
this time can best be described as:
A. Endemic.
B. Epidemic.
C. Pandemic.
D. Sporadic.
B. Epidemic.

A business executive develops flu-like symptoms 1 day


after returning by air from a trans-Atlantic 2-day
conference that involved lengthy meetings into the
evening. The scenario best illustrates the interaction of:
A. Host and agent.
B. Host, agent, and environment.
C. Risk and causality.
D. Morbidity and disease.
B. Host, agent, and environment.

A nurse teaches an asthmatic client to recognize and


avoid exposure to asthma triggers and assists the client's
family in implementing specific protection strategies in
the home, such as removing carpets and avoiding pets.
This nurse's activities can best be described as:
A. Comprehensive assessment.
B. Primary prevention.
C. Secondary prevention.
D. Treatment intervention.
C. Secondary prevention.

A nurse is concerned about the accuracy of the purified


protein derivative (tuberculin) test in screening
individuals with tuberculosis exposure for followup chest
radiography. The nurse's concern is related to which
aspect of the test's validity?
A. Reliability.
B. Sensitivity.

C. Specificity.
D. Variability.
B. Sensitivity.

During the last stage of AIDS, many opportunistic


infections proliferate in persons with weakened immune
systems. HIV-infected individuals who live near one
another in long-term care facilities, drug treatment
facilities, and prisons should be carefully screened before
admission because they are at greater risk for acquiring:
A. Fungal disease.
B. Invasive cervical cancer.
C. Oral candidiasis.
D. Pulmonary tuberculosis (TB).
D. Pulmonary tuberculosis (TB).

A young female client visits a local clinic, seeking


treatment for a urinary tract infection (UTI). During the
assessment, the nurse conducts an extensive sexual
history/activity screening to determine the risk for a
sexually transmitted disease and potential complication
of:
A. Genital warts.
B. Herpes simplex virus 1.
C. Pelvic inflammatory disease (PID).
D. Primary syphilis.
C. Pelvic inflammatory disease (PID).

The primary prevention of hepatitis C virus (HCV)


includes:
A. Counseling and follow-up of infected clients.
B. Routine testing for health care workers.
C. Screening of blood products.
D. Testing of high-risk individuals.
C. Screening of blood products.

A strategy that may improve compliance with tuberculosis


(TB) treatment is the:
A. Expansion of directly observed therapy.
B. Increased tuberculosis testing protocols.
C. Increased confirmatory testing protocols.
D. Treatment compliance education.
A. Expansion of directly observed therapy.

Factors that contribute to newly emerging or reemerging


infectious disease can be related to microbial adaptation

and changes made by the infectious agent. However,


most of the emergence factors are related to
environmental changes and:
A. Consequences of human activities/behaviors.
B. Increase in the number of vectors.
C. Industrialization and urbanization.
D. Unpredictable variances in the climate.
A. Consequences of human activities/behaviors.

For the nurse to fully understand the threat associated


with the release of biological agents and participate in an
appropriate response, the Centers for Disease Control and
Prevention (CDC) has stated that the biological agents
most likely to be employed are those that both have a
potential for high mortality and can be easily
disseminated to produce:
A. Immediately severe symptoms.
B. Major panic and social disruption.
C. Multisyndrome effect.
D. Person-to-person transmission.
B. Major panic and social disruption.

To reach the desired goal of maximizing the full


immunization rates for preventable communicable
disease and increasing herd immunity levels, it is crucial
for the nurse to:
A. Assume that the primary care physician has provided
all appropriate immunizations.
B. Check an individual's immunization status at each and
every visit.
C. Support parents who are reluctant to immunize their
children.
D. Understand the difficulties in obtaining and
maintaining immunization schedules.
B. Check an individual's immunization status at each and
every visit.

Protecting the nation's food supply from contamination by


all the virulent microbes is complex, costly, and time
consuming. However, much foodborne illness, regardless
of causal organisms, can be prevented through simple
changes in:
A. Food preparation, handling, and storage.
B. Importation regulations.
C. Pesticide usage.
D. Animal breeding practices.
A. Food preparation, handling, and storage.

Immunity to disease through vaccination is known as:


A. Natural immunity.
B. Resistance.
C. Acquired immunity.
D. Herd immunity.
C. Acquired immunity.

A multisystem approach to community disease control


would include such interventions as: (select all that apply)
A. Community action programs.
B. Control of vectors.
C. Improved surveillance systems.
D. Legislation.
E. Provision of chemotherapy.
A. Community action programs.
B. Control of vectors.
C. Improved surveillance systems.
D. Legislation.
E. Provision of chemotherapy.

The epidemiologic triangle consists of three components,


which are the following except:
A. Host
B. Mode of transmission
C. Agent
D. Environment
B. Mode of transmission

The organism that harbors and provide nourishment for


another organism is called a:
A. Host
B. Mode of transmission
C. Agent
D. Environment
A. Host

What is the leading communicable disease worldwide?


Influenza

The ___ facilitates the transmission of the agent from the


host to another susceptible host.
Environment

What are some environmental causes of the emergence


of new diseases?
Deforestation
Changes in ecosystem
Flood
Drought
Global warming

What human behaviors are related to the emergence of


new diseases?
Sex
Drug use
Travel
Diet
Childcare
Outdoor recreation

What type of community intervention does the


government employ to prevent food-borne illness?
Surveillance - restaurant inspections/employees washing
hands (hep A is fecal/oral)

___ is the ability of an agent to produce a specific clinical


reaction after infection occurs.
Pathogenicity

___ is an agent's ability to produce a severe pathological


reaction.
Virulence

___ is the ability of an agent to produce a poisonous


reaction.
Toxicity

___ is the ability of an agent to penetrate through and


spread throughout a tissue.
Invasiveness

___ is the ability of an agent to stimulate an


immunological response.
Antigenicity

What are some examples of environmental alterations to


prevent spread of disease?

Mosquito nets/repellants to avoid bug bites


Installing swear systems to prevent fecal contamination of
water supplies
Washing utensils after contact with raw meat to reduce
bacterial contamination
What are the symptoms of TB?
Cough
Fever
Hemoptysis
Chest pains
Fatigue
Weight loss

(True/false) Exposure to an infectious agent always leads


to an infection.
False

(True/false) Infection does not always lead to disease.


True

___ are vertebrate animals that transmit infections to


humans under natural conditions. This can occur through
bites, inhalation, ingestions, and direct contact.
Zoonoses

What are some examples of zoonoses?


Rabies
Toxoplasmosis
Cat scratch fever
Brucellosis
Listeriosis
Salmonellosis
Health care economics is ___
- it analyzes 3 things
- it also involves taxation __ and ___.
a specialized field of economics that describes and
analyzes the production,distribution, and consumption of
goods and services
-as well as a variety of related problems such as finance,
labor, and
taxation.

The goal of health care economics__

overcome scarcity by making good choices and providing


essential services.

Quarantine is
- incubation (define)
ISOLATION during a communicable disease (especially
during incubation ) to prevent spread.
incubation: The development of an infection from the time
the pathogen enters the body until signs or symptoms first
appear.

sanitation refers to the


promotion of hygiene and prevention of disease by
maintenance of health-enhancing (sanitary) conditions.

The Shattuck Report


-a landmark document, made a tremendous impact on
sanitary progress.
the first attempt to describe a model approach to
organization of public health.

official health agencies


- who operates them? (2)
- function? including 2 types of services
Agencies operated by state or local governments to provide
a wide range of "public health services" including
community and public health nursing services

voluntary health agencies


- when did it started to emerge?
sometimes called private agencies or nongovernmental
organizations [NGOs]
-1800s

3 core public health functions ( in order)


APA
- this is performed by __ or ___services
assessment, policy development,assurance
= public or private

Assessment
- define
- it is a continuous process of ____ (2) information about
__
- function?
- morbidity?
measuring and monitoring the health status and needs of a
designated community or population.
-a continuous process of collecting data and disseminating
information about health,diseases, injuries, air and water
quality, food safety, and
available resources.
This function helps to identify morbidity, mortality, and
causative factors.
- rate of incident

Policy development is the


" LEADERSHIP , ADMINISTRATION & guideline for present
and future decision affecting public's health"

Assurance is
- 2 MORE FUNCTIONS?
the process of translating established policies into
services.
1. ensures that population based services are provided,
whether by public health agencies or private sources.
2. monitors the quality of and access to those services.

medically indigent
(those who are unable to pay for and totally lack medical
services)

Public Health Service (PHS).


- its concerned with ___
It is concerned with the broad health interests of the
country and is a functional (not organizational) unit of
DHHS

DHHS
Department of Health and Human Services

Proprietary health services are


- private or public?
- profit, non profit or both?
privately owned and managed. They may be nonprofit or
for-profit.

gross domestic product (GDP)


the total amount of goods and services produced within a
year; whereas health spending.

Economics
science of making decisions regarding scarce resources.
(Study of how societies decide what to produce, how to
produce it, and how to distribute what they produce)
Economics permeates our social structureit affects and is
affected by policies.,

Microeconomic theory is
concerned with supply and demand.

Macroeconomic theory is
concerned with the broad variables that affect the status of
the economy as a whole.

Supply is
the quantity of goods or services that providers are willing
to sell at a particular price.

Demand
Consumer willingness and ability to buy products

adverse selection
-define
-ex
the tendency for people with the greatest probability of loss
to be the ones most likely to purchase insurance.
(ex: when high-risk patients are denied insurance or care)

Third-party payments are


- reimbursement made by who?

monetary reimbursements made to providers of health care


by someone other than the consumer who received the
care.

consumer-driven health plans (CDHP) and health savings


accounts (HSAs).
- deductible is high or low?
-example?
consumer-directed.
Type of medical insurance that combine a high deductible
health plan with a medical savings plan which covers some
out of pocket expenses.
- employment plan ( blue cross blue shield,

deductible
( yearly health insurance bill)
A certain amount of money that the patient must pay each
year toward his or her medical expenses before health
insurance benefits begin.

cost sharing
a situation where insured individuals pay a portion of the
healthcare costs, such as deductibles, coinsurance or copayments.
( ex: auxilio mutuo private health insurance)

Medicare
A federal program of health insurance for persons 65 years
of age and older

Medicaid
A public assistance program designed to provide healthcare
to poor, disabled and disadvatanged americans

capitation rates
(fixed amounts of money paid per person by the health
plan to the provider for covered services)

retrospective payment

which is reimbursement for a service after it has been


rendered

rendered
Provided, given

reimbursement
paying back

fee-for-service
A system under which doctors and hospitals receive a
separate payment for each service that they provide

Prospective payment
payment method based on rates derived from predictions
of
annual service costs that are set in advance of service
delivery.
-the payment amount or reimbursement with a set rate for
certain procedures is known in advance

diagnosis-related groups (DRGs)


a system of analyzing conditions and treatments for similar
groups of patients used to establish Medicare fees for
hospital inpatient services; patients are classified by their
principal diagnosis, surgical procedure, age, and other
factors.

cost shifting.
- read
- define
This practice of charging different prices to different
consumers most often affects those without health
insurance who are paying out-ofpocket for care.
- Lack of insurance, uninsured populations, and
uncompensated care are covered by charging more to
those who can pay. This practice is referred to as

managed care
It refers to systems that coordinate medical care for
specific
groups to promote provider efficiency and control costs.

Medical home
seeing the same health care provider for regular care

health maintenance organization (HMO)


- define
- certain specification
is a system in which participants prepay a fixed monthly
premium
to receive comprehensive health services delivered by a
defined network of providers.
The HMOs are the oldest model of coordinated or managed
care

A preferred provider organization (PPO)


a network of physicians, hospitals, and other health-related
services that contracts with a third-party payer
organization to provide comprehensive health services to
subscribers on a fixed FFS basis.

point-of-service (POS) plan,


which permits more freedom of choice than a standard
HMO or PPO.

rationing in health care refers to


limiting the provision of health care services to save costs.

Competition means
a contest between rival health care organizations for
resources and clients

Regulation refers to
mandated procedures and practices affecting health
services delivery that are enforced by law

Managed competition,
- what does it combine?
combine with market competition to achieve cost
savings with government regulation to achieve expanded
coverage.

single-payer system
- this lead to?
When the government collects taxes for healthcare from all
citizens and then uses the collected money to pay for the
citizens healthcare services.
- universal coverage

universal coverage
(everyone would have health insurance of some type,
assuring better access to care).

SUMMARY
SUMMARY

history
Primitive practices of early centuries were replaced with
more advanced sanitary measures by the Greeks and
Romans. The Middle Ages saw a
serious health decline in Europe, with raging epidemics
leading to extensive 19th-century reform efforts in England
and, later, in the United States.

Organized health care in the United States developed


slowly. Public health problems, such as the need for
isolation
of persons with communicable diseases and control of
environmental pollution, prompted the gradual development
of
official interventions.
For example, quarantines to control
the spread of communicable disease were imposed in the
late 1700s. Sanitary reform was pursued more vigorously
during the 1800s. Local and then state health departments
were formed starting in the late 1700s. By the early 1900s,
the federal government had assumed a more active role in
public health, with a proliferation of health, education, and
welfare services.

. Public health services include three core public health


functions:
assessment, policy development, and assurance

Private health services are the unofficial arm of the community health system. They include (2) agencies
voluntary nonprofit agencies
privately owned (proprietary) and for-profit
agencies.

The delivery and financing of community health services


has been significantly affected by various legislative acts.
These acts have prompted such innovations as (6)
HAM- steak- herring- fish
Health insurance
assistance for the poor, the elderly, and the disabled;
money to train health personnel and conduct health
research;
standards for health planning and delivery
health protection for workers on the job;
financing of health services.

Health care economics studies the(3)


- function?
production, distribution, and consumption of health care
goods and service
maximize the use of scarce resources to benefit the most
people.

This science underlies the financing of the health


care system. It is influenced by (2)
microeconomics
macroeconomics

Health care is funded through public and private


sources, which fall into three categories:
third-party payers,
direct consumer payment,
private support.

Several trends and issues have influenced community


health care financing and delivery and are important in
understanding health care economics and helping to
improve community health. They include
cost control,
financial access,
managed care,
health care rationing,

competition and regulation,


managed competition,
universal coverage, a single-payer system, and health care
reform

The changing nature of health care financing has


adversely affected community health and its practice in
three important ways:
(1) retrospective payment without
limiting costs, tax-deductible employer contributions for
health care coverage and nontaxable employee health
benefits, together with a lack of consumer involvement in cost
sharing, have created disincentives for efficient use of
resources;
(2) because the health care system traditionally
has reimbursed only for treatment of the ill or disabled,
with
no reward for health promotion and prevention efforts, it
has
promoted incentives to focus only on illness care; and
(3) the competition model, which has long driven up health
care
costs and eliminated many from being able to afford health
care services, has generated a conflict with the basic public
health values of health promotion and disease prevention
for
all persons.

Health care reforms have been proposed, but the


United States remains the only industrialized nation
without
some type of
universal health coverage.

We also rank significantly lower than most other


developed countries on health indicators, such as infant
mortality and life expectancy.
Public health nurses can lead the effort in making health
care more accessible to all citizens and encourage policies
and
practices that promote health, rather than reward illness.
active immunity
acquired after exposure to a disease or via immuneizations

OSHA
sets health and safety work standards employees are in
legally required to protect workers from hazards and
employees have a right to know what hazards they are
exposed to

passive immunity
newborns through maternal antibodies

.risk assessment
qualitative and quantitative evaluation of the risk posed to
human health for the environment by the actual or
potential presence or use of specific pollutants

needs assessment
needs assessment systemic appraisal of type depth and
scope of problems as perceived by clients health providers
or both

natural immunity
species determinate resistance to an infectious agent

WIC
woman and children Supplemental Nutrition Program
provides federal grants to states for supplemental food
health care referral and nutrition education for low-income
pregnant breastfeeding and non-breastfeeding postpartum
women infants and children up to five years of age who are
found to be at a nutritional risk

primary prevention
type of intervention that seeks to promote health and
prevent disease from the beginning involves health
promotion and education

secondary prevention
secondary prevention intervention that seeks to detect
disease by screening and providing healthcare early in the
progression before clinical signs and symptoms become a
apparent in order to make an early diagnosis and begin
treatment

tertiary prevention

continued long-term health care interventions that begins


once the disease is obvious the aim is to interrupt the
course of the disease reduce the amount of disability that
might occur and begin rehabilitation

SARS
severe acute respiratory syndrome spread via airborne
droplets indirect.contact and by close contact with an
infected person there is no cure as a person is suspected of
having sars they should be admitted to the hospital
immediately and kept under Isolation if in an outpatient
setting the person must be rapidly divertid to a separate
area to minimize transmission to others

palliative care
care to alleviate symptoms by meeting the special needs of
and providing comfort for the dying client and their families
by the nurse

long term care


care designed to help a person with basic activities of daily
living that is given to individuals over a sustained amount
of time

Native American care


verbal communication is low tone of voice and the listener
is expected it to be attentive
nonverbal communication direct eye contact is often
considered disrespectful
touch a light touch of the person's hand instead of a firm
hand is often used as a greeting
family organization usually have a close extended family
and the emphasis tends to be on the needs of the family
rather than the individual needs
time perception of health often past oriented harmony of
mind body spirit and emotions with nature
alternative healers a medicine man and shaman
self care practices Herbs cornmeal and medicine bundle
biological variations cleft uvula lactose intolerance and skin

color
food preferences include blue cornmeal fruits Game and
Fish.nutritional excess of carbohydrates and calories
risk factors are diabetes malnutrition tuberculosis infant
and maternal mortality

culture and sensitivity


appreciate and respect for individual clients beliefs and
values
appreciate and value diversity
appreciate and genuinely care about those of other
cultures
recognize how your own cultural background may influence
professional practice

Good luck
...
3 shared commonalities
geographic, common interest, community of solution

community of solution
group of people who come together to solve problems that
affects all of them, ex: guidance counselors, counties
working for water pollution

common interest
collection of people with interest or goal that bind them
together, ex: MADD, disabled community, church groups,
gay community

population
all the people occupying an area, or all those who share
one more characteristic, may not necessarily interact

aggregates
a mass or grouping of distinct individuals who are
considered as a whole and are loosely associated with each
other, broader term then population

aggregates
communities and populations are types of

goal of community health nursing


health promotion, disease prevention, and health
protection

promotion of health
health education, preventative care services, healthy
people 2010, raise levels of wellness for individuals,
families, populations and communities

prevention of health problems


anticipating and averting problems or discovering them as
early as possible to minimize potential disability and
impairment

primary prevention
to keep illness from occurring, hand rails, immunizations

secondary prevention
efforts to detect and treat existing health problems.
Screenings: htn, drug abuse, breast exam, DM test; to
intervene to control or eradicate the problem

tertiary prevention
attempts to reduce the extent and severity of health
problems so as to minimize disability and restore or reserve
function: rehab after CVA.

tertiary prevention
someone that has an eating disorder, following up with
counseling

treatment of disorders
focuses on illness by: direct service, indirect service,
development of programs to correct unhealthy conditions

indirect service
referring to someone, calling another professional to help
solve problem for client

rehab

efforts to reduce disability and restore function as much as


possible

evaluation
process by which the practice is analysis, judged, and
improved according to established goals and standards

research
investigation to discover facts affecting community health
and community health practice, solve problems, and
explore improved methods of health science

healthy people 2010


increase quality and years of life, eliminate health
disparities

healthy people 2010


emphasis on community partnerships,

determinant of health
biology

determinant of health
behavior

determinant of health
social & physical environment

determinant of health
polices and interventions

community health status


health of total population

community health status


measured by birth and death rates, morbidity statistics

healthy people 2010 indicators


physical activity, obesity, tobacco use, substance abuse,
sexual behavior, mental health, injury and violence,
environmental quality, immunization, access to healthcare

core public health functions

assessment, policy development, assurance

role of community health nurse


clinician, educator, advocate, collaborator, leader,
researcher, case manager

clinician
most familiar role, ensures health services are provided

educator / teacher
client is not acutely ill, so can absorb information better,
wide audience can be reached

advocate
2 goals: help clients gain greater independence or self
determination; make the system more responsible and
relevant to the needs of clients. Every client has the right
to receive just, equal and humane treatment

collaborator
work jointly with others, need communication skills

leader
focuses on change, nurse becomes an agent of change,
influence people to think and behave differently about their
health

researcher
evidence based practice, systematic investigation,
collection, and analysis of data for solving problems and
enhancing community heath practice

case manager
has become the standard method of managing health care
in delivery systems in US. Evaluates progress to ensure
that clients' multiple service needs are met in a costeffective manner.

settings for community health nursing practice


homes, ambulatory services, schools, occupations health
settings, hospice, parish, forensics, corrections (prison)

ambulatory service settings

community health centers, clinics, day care centers, health


departments, services for select groups such as migrant
camps, native american reservations, correctional facilities,
parishes

early home care nursing


religious and charitable groups, during industrial revolution

Florence Nightingale
early 19th century, crimean war, record keeping and
statistics, organized nursing care, kitchens, laundries,
"notes on nursing"

public health nursing


1900-1970, health and welfare of general public, not just
the poor

Lillian Wald and Mary Brewster


started Henry Street Settlement to provide nursing and
welfare services

societal influences on the the development of community


health nursing
tech, progress in thinking, education, role of women,
consumer movement, economic forces

Federal agencies
DHHS, EPA, Homeland security, D of Agriculture, Dept of
Ed, VA

State agencies
Medicare, Medicaid, SCHIPS

Local health department


carries out most state laws and policies, most
direct/immediate care

Indian health services


IHS: part of the U.S. Department of Health and Human
Services and is responsible for the medcial care provided to
Native Americans and Native Alsaskans. Established in
1954 and provides health care.

medicare

A program added to the Social Security system in 1965 that


provides hospitalization insurance for the elderly and
permits older Americans to purchase inexpensive coverage
for doctor fees and other health expenses.

medicaid
a federally aided, state operated program that provides
medical benefits for low-income persons in need of medical
care

Medicare prospective payment


"you will get 10 paid visits"

Medicare prospective payment


5 criteria will be met: OASIS form, centers for Medicare and
Medicaid Services (CMS), Medicare documentation (see
page 894)

OASIS
outcome, assessment information set: very detailed form

Medicare prospective payment


need a skilled service, not just "a bath", services must be
intermittent care

Medicare prospective payment


the payment plan usually last 60 days, then needs to be reevaluated.

durable medical equipment


bedside commodes, walkers, beds, can be used again.

supplies
thrown away after each use

home health advantages


convenience, access, relationship, cost, outcomes, teaching
opportunities

multiplicity of problems
finding more then one problem

maintaining balance

intimacy vs professional distance, risk and safety, assisting


and devaluing client, dependence and independence, cost
containment and quality

hospice
medicare is the main payment, even if you're under 65

hospice
you have 6 months or less to live

hospice
pain medication or no aggressive chemo or procedures

compounding pharmacist
combining drugs in a unique way in a hospice setting

durable power of attorney


health issues, or healthcare proxy

I-Prepare
prevent, resident, environment, past work, activities,
resources, education

web of causation
the combination of factors that lead to a problem

contaminant
organic or inorganic that enters medium that renders it
impure

toxic agent
poisonous substance in the environment

overpopulation
exceeds ability of it's ecosystem to either support needs; 4
factors (starvation, disease, wars, aide from other
countries)

air pollution
high in industrial areas

radon
odorless, colorless, causes cancer

lead poisoning
affects CNS, old paint,

contact lens solution


can cause birth defects

psychological hazards
noise, overcrowding, lack of natural beauty

phases of survey method


planning, collection, analysis & presentation

traditional nursing
work as part of team, acute setting, with other
professionals

public health nursing


in the community, often alone, no monitors, labs, use nontangible skills like listening, assessing, planning

hospice
concept of palliative care for terminally ill patients,

referral sources
hospitals (MD, MSW, Case Mgr), Clinics (RN or MD), Families
(self)

nurse role overpopulation


teaching birth control, family planning, preventing high risk
pregnancies, sex education

vectors
carry diseases such as mosquitoes, flies, ticks, roaches,
fleas, rats, mice, and squirrels

motor vehicle accidents


3 billion injuries and >42,000 deaths per year

health people 2010 environmental health


Focuses: pollutants, portable drinking water, decrease
water born diseases, eliminating toxic lead levels in

children, reduce pesticide exposure, inspecting homes for


lead
Health
which is a state of complete physical, mental and social
well-being, and not merely the absence of disease or
infirmity, is a fundamental human right and the attainment
of the highest possible level of health is a most important
world-wide social goal

Public health
a system and social enterprise; a profession; a collection of
methods, knowledge, and techniques; governmental health
services, especially medical care for the poor and
underserved; and the health status of the public

Epidemiology
the study of the occurrence and distribution of healthrelated states or events in specified populations, including
the study of the determinants influencing such states, and
the application of this knowledge to control the health
problems

Descriptive epidemiology
seeks to describe the occurrence of a disease in terms of
person, place, and time.

Determinants of health events


those factors, exposures, characteristics, behaviors, and
contexts that determine (or influence) the patterns
(answers the how and why); may be individual, relational,
social, communal, or environmental

Analytic epidemiology
focuses on investigation of causes and associations.

Distribution
determine who has the disease and where and when the
disease occurs

Epidemic occurs when


the rate of disease, injury, or other condition exceeds the
usual (endemic) level of that condition

Public health workers should be able to

Demonstrate basic knowledge of the role that genomics


play in the development of disease. Identify limits of his/her
genomic expertise. Make appropriate referrals to those with
more genomic expertise

The RN will
Demonstrate an understanding of the relationship of
genetics and genomics to health, prevention, screening,
diagnostics, prognostics, selection of treatment, and
monitoring of treatment effectiveness. Demonstrate ability
to elicit a minimum of a three-generation family health
history information. Construct a pedigree from collected
family history information using standardized symbols and
terminology. Collect personal, health, and developmental
histories that consider genetic, environmental, and
genomic influences and risk factors. Critically analyze the
history and physical assessment findings for genetic,
environmental, and genomic influences and risk factors.
Assess patients' knowledge, perceptions, and responses to
genetic and genomic information. Develop a plan of care
that incorporates genetic and genomic assessment
information.

Personal health care initiative- Goal:


to improve the safety, quality, and effectiveness of health
care for every patient in the United States

Genomics will
enable medicine to be tailored to each person's needs

Personal Health Care Initiative, Goal 1:


Link clinical and genomic information to support
personalized health care

Personal Health Care Initiative, Goal 2:


Protect individuals from discrimination based on
unauthorized use of genetic information

Personal Health Care Initiative, Goal 3:


Ensure the accuracy and clinical validity of genetic tests
performed for medical application purposes

Personal Health Care Initiative, Goal 4:


Develop common policies for access to genomic databases
for federally sponsored programs.

Genetic screening
Evaluate >1600 genetic disorders, First used in the late
1970s

Indications for genetic screening:


Predicting development of genetic disorders, Screening
populations, Confirming clinical diagnosis, Prenatal testing,
Development of individualized medical treatment

Genomics
the study of individual genes in order to understand the
structure of the genome, including the mapping of genes
and sequencing the DNA.

Genomics examines
the molecular mechanisms and the interplay of genetic and
environmental, cultural, and psychosocial factors in disease

Genomics deals with


he functions and interactions of all genes in an organism
and is the study of the total DNA structure.

DNA deoxyribonucleic acid,


is the chemical inside the nucleus of a cell that has the
genetic instructions for making living organisms.

Genes
the DNA segments that carry the genetic information

Chromosomes
long structures that organize DNA

DNA replication
chromosomes are duplicated before cells divide

Four bases of DNA


adenine (A), guanine (G), cytosine (C), and thymine (T)

Genes are comprised of


specific sequences of these bases

Mutations

alterations in the usual sequence of bases that form a gene


or changes in DNA or chromosomal structures

DNA Repair
process in which mutations are corrected; estimates
indicate repair mechanisms correct at least 99.9% of initial
errors

I
Investigate Potential Exposures Investigate potential
exposures by asking: Have you ever felt sick after coming
in contact with a chemical, such as a pesticide or other
substances? Do you have any symptoms that improve
when you are away from your home or work?

P
Present WorkAt your present work: Are you exposed to
solvents, dusts, fumes, radiation, loud noise, pesticides, or
other chemicals? Do you know where to find material
safety data sheets for chemicals with which you work? Do
you wear personal protective equipment? Are work clothes
worn home? Do co-workers have similar health problems?

R
Residence When was your residence built? What type of
heating do you have? Have you recently remodeled your
home? What chemicals are stored on your property? Where
is the source of your drinking water

E
Environmental Concerns Are there environmental concerns
in your neighborhood (e.g., air, water, soil)? What types of
industries or farms are near your home? Do you live near a
hazardous waste site or landfill?

P
Past Work What are your past work experiences? What job
did you have for the longest period of time? Have you ever
been in the military, worked on a farm, or done volunteer
or seasonal work?

A
Activities What activities and hobbies do you and your
family pursue? Do you burn, solder, or melt any products?
Do you garden, fish, or hunt? Do you eat what you catch or

grow? Do you use pesticides? Do you engage in any


alternative healing or cultural practices?

R
Referrals and Resources Use these key referrals and
resources: Environmental Protection Agency
(http://www.epa.gov) National Library of MedicineToxnet
Programs (http://www.nlm.nih.gov) Agency for Toxic
Substances and Disease Registry
(http://www.atsdr.cdc.gov) Association of Occupational and
Environmental Clinics (http://www.aoec.org) Occupational
Safety and Health Administration (http://www.osha.gov)
EnviRN website (http://www.enviRN.umaryland.edu) Local
Health Department, Environmental Agency, Poison Control
Center

E
Educate (A Checklist) Are materials available to educate
the client? Are alternatives available to minimize the risk of
exposure? Have prevention strategies been discussed?
What is the plan for follow-up?

Environmental laws and pollutants


Environmental Protection Agency, Food and Drug
Administration, Department of Agriculture, Permitting- limit
amount of pollution in air, Environmental standards,
Compliance, Monitoring

Epidemiology
The science that helps us understand the strength of the
association between exposures and health effects

Epidemiologic triangle
Agent, host, and environment, a simple model that belies
the often complex relationships between agent, which may
include chemical mixtures (i.e., more than one agent); host,
which may refer to a community with people of multiple
ages, genders, ethnicities, cultures, and disease states; and
environment, which may include dynamic factors, such as
air, water, soil, and food, as well as temperature, humidity,
and wind.

Geographic Information Systems (GIS)


is a methodology that requires the coding of data so that it
is related spatially to a place on earth.

Agent
Chemical or disease

Host
Community, patient,

Environment
water, air, cleanliness, plumbing, sewer systems, soil,
temperatures, climates, habits and customs

Key public health (PH) professionals


Food safety specialist, Sanitarians, Radiation specialists,
Industrial hygienists

Roles for nurses


Community involvement/public participation, Individual and
population risk assessment, Risk communication,
Epidemiologic investigations, Policy development

levels of public health nursing


Assessment, Diagnosis, Planning (Including selection of
interventions), Implementation, Consultation, Referral and
follow-up, Advocacy, Policy development, Surveillance,
Evaluation

Health departments
conduct assessments of community health status, a core
function of public health, on an ongoing basis

The identification of some community problems emerges


out
practice, rather than through a formal community
assessment

Applying the Nursing Process at the Individual/Family


Level
Community assessment, Assessment of a family, Diagnosis,
Planning (Including selection of interventions),
Implementation, Collaboration, Case management, Health
teaching, Delegated functions (PHN to paraprofessional),
Referral and follow-up, Evaluation

Applying the PHN Process at the Community Level of


Practice Scenario
Community assessment (PHN process: assessment),
Community diagnosis (PHN process: diagnosis), Community
action plan (PHN process: planning, including selection of
interventions), Menu changes, Classroom activities, Family
involvement, Community implementation plan (PHN
process: implementation), Community evaluation (PHN
process: evaluation).

Executive branch
Suggests, administers, and regulates policy. President,
cabinet, various administrative and regulatory
departments, and agencies. U.S. Department of Health and
Human Services.

Legislative branch
Identifies problems and proposes, debates, passes, and
modifies laws to address those problems. Identifies
problems and proposes, debates, passes, and modifies laws
to address those problems. Senate and House of
Representatives.

Judicial branch
Interprets laws and their meaning. System of federal, state,
and local courts guided by opinions of the Supreme Court.
Machine Co. v. Davis.

Supreme Court. Machine Co. v. Davis


First case to challenge federally created health programs.
Supreme Court determined that such federal governmental
action was within the powers of Congress to promote the
general welfare. Police Power: this power allows states to
act to protect the health, safety, and welfare of their
citizens. Such police power must be used fairly, and the
state must show that it has a compelling interest in taking
actions, especially actions that might infringe on individual
rights. An example is requiring immunizations for children
to enter school

Culture
A set of beliefs, values, and assumptions about life that are
widely held among a group of people and that are
transmitted intergenerationally. Develops over time and is
resistant to change. Each culture has its own unique

organizational structure. Communication. Family structure.


Child rearing practices. Religious practices. Physical space.

Race
A social classification that relies on physical markers, such
as skin color, to identify group membership. Individuals
may be the same race but of different cultures

Ethnicity
The shared feeling of peoplehood among a group of
individuals. Reflects membership in a cultural group and is
based on individuals sharing similar cultural patterns.
Represents the identifying characteristics of culture, such
as race, religion, or national origin, and affects beliefs,
behaviors, and access to resources. Members of an ethnic
group are likely to give up aspects of their identity and
society when they adopt characteristics of another group's
identity. However, when there is a strong ethnic identity,
the individual maintains the values, beliefs, behaviors,
practices, and ways of thinking of their group.

Four categories of foreign-born


Legal immigrants (lawful permanent residents), Refugees,
Nonimmigrants, Unauthorized immigrants (undocumented
or illegal aliens)

Legal immigrants
Make up 80% of immigrant population. Are not citizens but
are legally allowed to work and live in the United States.

Refugees and persons seeking asylum


are people who seek protection in the United States
because of fear of persecution (on the basis of race,
religion, nationality, political view, or membership in a
certain group) if they were to return to their homeland.

Nonimmigrants
are admitted to the United States for a limited duration and
for a specified purpose. include students, tourists,
temporary workers, business executives, career diplomats
and their spouses and children, artists, entertainers, and
reporters.

Advocacy

places the advocate's core skills (i.e., interviewing,


assertiveness and force, negotiation, self-management,
legal knowledge and research, and litigation) within the
context of six ethical principles for effective advocacy

Advocacy
Act in accordance with the patient's wishes and
instructions. Keep the patient properly informed. Carry out
instructions with diligence and competence. Act impartially
and offer frank, independent advice. Maintain patient
confidentiality.

Feminist ethics
encompasses the tenets that women's thinking and moral
experiences are important and should be taken into
account in any fully developed moral theory, and that the
oppression of women is morally wrong. Entails knowledge
about the social, cultural, political, legal, economic,
environmental, and professional contexts that insidiously
and overtly oppress women as individuals, or within a
family, group, community or society. Demand results.
Nurses were reluctant to embrace feminism and it's ethics
for many years.

The Core Functions Project (U.S. Public Health Service,


1994/2008)
developed a useful illustration, the Health Services Pyramid
(Figure 1-2), which shows that population-based public
health programs support the goals of providing a
foundation for clinical preventive services.

These services focus on


disease prevention; health promotion and protection; and
primary, secondary, and tertiary health care services.

All levels of services shown in the pyramid are


important to the health of the population and thus must be
part of a health care system with health as a goal.

Because of the importance of the basic public health


programs
members of the Core Functions Project argued that all
levels of health care, including population-based public
health care, must be funded or the goal of health of
populations may never be reached.

competencies of PH
In the spring of 2010 this Council, funded by the Centers
for Disease Control and Prevention, Health and Human
Services (HHS), adopted an updated set of Core
Competencies ("a set of skills desirable for the broad
practice of public health")
(http://www.phf.org/link/corecompetencies.htm) for all
public health professionals, including nurses.

The 72 Core Competencies are divided into eight


categories
-Analytic/assessment
-Policy development/program planning
-Communication
-Cultural competency
-Community dimensions of practice
-Basic public health sciences
-Financial planning and management
-Leadership and systems thinking

In addition, each competency is presented at three levels


(tiers) that reflect the different stages of a career. Tier 1
applies to entry level public health professionals without
management responsibilities.

In addition, each competency is presented at three levels


(tiers) that reflect the different stages of a career. Tier 2
competencies are expected in those with management
and/or supervisory responsibilities.

In addition, each competency is presented at three levels


(tiers) that reflect the different stages of a career. Tier 3
is expected of senior managers and/or leaders in public
health organizations.

In the early years of America's settlement, as in Europe,


the care of the sick was usually
informal and was provided by household members, almost
always women. The female head of the household was
responsible for caring for all household members, which
meant more than nursing them in sickness and during
childbirth. She was also responsible for growing or
gathering healing herbs for use throughout the year.

England's Elizabethan Poor Law


guaranteed medical care for poor, blind, and "lame"
individuals, even those without family. Colonial America's
ideas of social welfare and care of the sick were strongly
influenced by this law. Pennsylvania Hospital was the first
hospital in what would become the United States.

Early colonial public health efforts


included the collection of vital statistics, improvements to
sanitation systems, and control of any communicable
diseases introduced through seaports. Colonists lacked
mechanism to ensure PH efforts would be supported and
enforced. Epidemics taxed the limited organization for
health during the seventeenth, eighteenth, and nineteenth
centuries.

After the American Revolution


the threat of disease, especially yellow fever, brought
public support for establishing government-sponsored, or
official, boards of health.

Public Health Service was established in


1798 as the Marine Hospital Service; it provided health care
for merchant seamen and protected seacoast cities from
epidemics; PHS is still the most important federal public
health agency today.

Shattuck Report
was published in 1850 by the Massachusetts Sanitary
Commission with an emphasis on the public's health. The
report called for major innovations: the establishment of a
state health department and local health boards in every
town; sanitary surveys and collection of vital statistics;
environmental sanitation; food, drug, and communicable
disease control; well-child care; health education; tobacco
and alcohol control; town planning; and the teaching of
preventive medicine in medical schools. However, these
recommendations were not implemented in Massachusetts
for 19 years, and in other states much later.

Origins of organized nursing


Nightingale studied nursing "system and method" by
visiting Pastor Theodor Fliedner at his deaconess training
school in Kaiserswerth, Germany.

Nightingale implemented her ideas about nursing during

Crimean War

Nightingale progressively improved soldiers' health


outcomes using
a population-based approach that strengthened
environmental conditions and nursing care. Using simple
epidemiological measures, she documented a decreased
mortality rate from 415 per 1000 at the beginning of the
war to 11.5 per 1000 at the end (Cohen, 1984; Palmer,
1983).

Mary Breckenridge's work


Some community-oriented nursing innovations were the
result of individual commitment and private financial
support, such as the Frontier Nursing Service

Frontier Nursing Service influenced the


development of public health programs geared toward
improving the health care of the rural and often
inaccessible populations in the Appalachian region of
southeastern Kentucky (Browne, 1966; Tirpak, 1975).
Breckinridge introduced the first nurse-midwives to the
United States when she deployed Frontier Nursing Service
nurses trained in nursing, public health, and midwifery.

The Future of Public Health, Institute of Medicine (IOM,


1988):
Found the United State's PH system in disarray and
concluded that, although there was widespread agreement
about what the mission of PH should be, there was little
consensus on how to translate that mission into action
Reported that the mix and level of PH services varied
extensively across the United States

Disparities
Racial or ethnic differences in the quality of health care, not
based on access, clinical needs, preferences, or
appropriateness of an intervention
-Indicators that continue to cause disparities in the United
States' health care system:
Cost
Access
Quality

Primary care

is a component of the private health care system. The care


that is provided by a health care professional (either a
physician, physician assistant, or nurse practitioner)
trained in family practice, pediatrics, or internal medicine.
Also called individual care.

Primary health care (PHC)


the focus of the public health system in the United States,
is defined as a broad range of services, including but not
limited to basic health services, family planning, clean
water supply, sanitation, immunization, and nutrition
education; it consists of programs designed to be
affordable for the recipients of the care and the
governments that provide them. In PHC, the emphasis is on
prevention, and the means of providing the care are based
on practical, scientifically sound, culturally appropriate, and
socially acceptable methods. This care is provided at the
community level, is accessible and acceptable to the
community, and is inviting of community participation.

Public health economics


focuses on producing, distributing, and consuming goods
and services as related to public health and where limited
public resources might best be spent to save lives or cause
the most increase in quality of life.

Efficiency
Suggests that inputs are combined and used in such a way
that there is no better way to produce the service, or
output, and that no other improvements can be made
refers to producing maximal output, such as a good or
service, using a given set of resources (or inputs), such as
labor, time, and available money.

Effectiveness
For example, effectiveness of a mass immunization
program is related to the level of "herd immunity"
developed.
refers to the extent to which a health care service meets a
stated goal or objective, or how well a program or service
achieves what is intended.

supply and demand


The upward-sloping supply curve represents the seller's
side of the market, and the downward-sloping demand
curve reflects the buyer's desire for a given product. As

shown here, suppliers are willing to offer increasing


amounts of a good or service in the market for an
increasing price. The demand curve represents the amount
of a good or service the consumer is willing to purchase at
a certain price. This curve illustrates that when few
quantities of a good or service are available in the
marketplace, the price tends to be higher than when larger
quantities are available. The point on the curve where the
supply and demand curves cross is the equilibrium, or the
point where producer and consumer desires meet.

Supply and demand Shifts result of


Competition for goods or services
Increase in costs of materials used to make a product
Technological advances
Change in consumer preferences
Shortage of goods or services

macroeconomics
Focuses on the "big picture"the total, or aggregate, of all
individuals and organizations
Aggregate is usually a country or nation

Ethics
is concerned with a body of knowledge that addresses such
questions as the following: How should I behave? What
actions should I perform? What kind of person should I be?
What are my obligations to myself and to fellow humans?
There are general obligations that humans have as
members of society.

Ethical judgments
are concerned with values. The goal of an ethical judgment
is to choose that action or state of affairs that is good or
right in the circumstances.

Consequentialism
The decision is based on outcomes or consequences. That
approach to ethical decision making maintains that the
right action is the one that produces the greatest amount
of good or the least amount of harm in a given situation.

Utilitarianism
is a well-known consequentialist theory that appeals
exclusively to outcomes or consequences in determining
which choice to make.

Deontology
may conclude that the action is right or wrong in itself,
regardless of the amount of good that might come from it.

Respect for autonomy


Based on human dignity and respect for individuals,
autonomy requires that individuals be permitted to choose
those actions and goals that fulfill their life plans unless
those choices result in harm to another.

Non-maleficence
requires that we do no harm. It is impossible to avoid harm
entirely, but this principle requires that health care
professionals act according to the standards of due care,
always seeking to produce the least amount of harm
possible.

Beneficence
This principle is complementary to nonmaleficence and
requires that we do good. We are limited by time, place,
and talents in the amount of good we can do. We have
general obligations to perform those actions that maintain
or enhance the dignity of other persons whenever those
actions do not place an undue burden on health care
providers.

Distributive justice
Distributive justice requires that there be a fair distribution
of the benefits and burdens in society based on the needs
and contributions of its members. This principle requires
that, consistent with the dignity and worth of its members
and within the limits imposed by its resources, a society
must determine a minimal level of goods and services to be
available to its members.

Egalitarianism
is the view that everyone is entitled to equal rights and
equal treatment in society.

Libertarian
view of justice holds that the right to private property is the
most important right.

The dominant themes of communitarianism

are that individual rights need to be balanced with social


responsibilities; individuals do not live in isolation but are
shaped by the values and culture of their communities
(Communitarianism, 2010). Among the theories with a
communitarian focus are virtue ethics, ethic of care, and
feminist ethics.
Florence nightengale
- crimean wars- helped improve soldiers health
- used population based approach

lillian wald
- first public health nurse--started public health nursing
- known fo establishmen of henry street settlements in new
york
- national organization of public health established in 1912was the president

Lina Rogers
-first school nurse
- focused on investigating the cause of absenteeism
- did not treat illness- physicians job at the time

national organization of puplic health


- presdident- lillian wald
- established 1912
- goal- standardize public health nursing, improve
standards of services and education by public health
nurses , and promote public health nursing

segregation
- lasted until 1960s
- made it difficult for african american nurses to get
certification and graduate education

mary breckenridge
- developed health programs to improve healthcare in rural
and hard to reach populations in the applachians of
southern kentucky

leading cause of death from 1900-1955


pneumonia, tuberculosis, and diahrea enteritis

leading cause of death after 1955 . new challenge for


nurses

- heart disease, cancer, cerebrovascular disease


- new challenge for nurses was chronic illness, and disease
prevention

ruth freeman
leading healh educator, author, consultant and leader of
national health organization of the 20th century
- offeren many professional oppurtunities
- saw nursing as intellectually challenging and was about
caring for people

maritime hosptial service


currently known as the public health service
- attempt of united states federal government to protect
health of citizens

WHO conference
- @ Alma Ata in 1978
- main focus of the conference was
worldwideimplementation of primary health

population health
holistic approach that considers the total health system
and focuses on the broad range of factors and conditions
that have a strong influenceon the health of population
- determinants social factors, social support netweorks,
education,m employmentm, and healthy child development

mulitlatral organizations
organizations that recieve fundings from mulple
governmental and non-governmental sources

bio terrorism
intentiaonal use of a pathogen or biological product to
cause harm to living organisms to influence or intimidatt eh
conduct of a government and cause harm to other
people. :
- role of PHN and healh officieals is to detect pathogen ,
manage services, and communicate during threat
- anthrax, plague, and smallpox biological agens highest
concern

environmental health

- aspects of human health affecting quality of life that are


determined by biological, cheical, and social and
pssychological problems in the enviroment

chemical exposure
nurses have made discoveries based on assessing people
presenting with signs and symptoms related to known
chemical toxicity

epidemiology
studies the incidence and prevalence of disease, in
POPULATIONS
- helps us to understand the strength of the association
between exposure and health effects
- monitors health of populations , understands
determinants of health and disease in communities, and
investigates and evaluates interventions to prevent disease
and maintain health
- father of epidemiology is JOHN SNOW_ because of his
work with cholera

toxicology
study of health effects associated with chemical exposures

rule of seven
native american concept, what will be the effect of this
decision in 7 years. .
- native americans believe humans are stewards, not
propieters of the land

attack rate
-a measure of morbididity often used in infectious disease
investigations
- often specific to an exposure such as food specific attack
rates
-rate that best indicates the proportion of people exposed
to an agent who deelop the disease.

mortality rates
reflect serious health problems and changing patterns of
disease
- do not give direct info about level of existing disease or
the risk of getting a disease
-informative only for fatal diseases

descriptive epidemiology
seeks to describe a desiease entitiy according to person,
place, and time

analytical epidemiology
directed toward understanding the etiology of the disease
and attempts to dienitfy the determinants of disease in
individuals
- determinants may be indvidiual, social , communal, or
environmental
- Examples- cohort studies (prospective and retrospective);
case control studies, cross sectional studies, ecological
studies

rate
measure of frequency of a health event in a "definied
population in a specified period of time
- not a proportion because the denominator is a function of
both the population size and the dimension of time while
the numberator is the number of events
- one cannot tell the degree of seriousness or if it is an
epidemic if you do not have a deonomiator which
represents the total population

risk
the probability an even will occur within a specified time

incidence rate
the number of new cases developing in a population at risk
during a sepcified period
formula= # of new cases (divided by) total population at
risk x 1,000

incidence
quantifies the rate of development of new cases in a
population at risk

epidemic
when the rate of disease injury or other condition exceeds
the usual level of that position
- Examples- an isolatedcase of smallpox in Africa because
there is no smallpox; nursing shortage in US, adult obesity
in the US,

primary prevention

interventions aimed at preventing the occurence of


disease, injury or diability: before it happens
EXAMPLES- immunizations, diet, exercise

secondary prevention
focuses on ealy detection and prompt treatment of disease
injury or disability. things before symptoms occur . ie
screening for hearig defects, mammograms

tertiary prevention
interventions aimed at disability limitation, and
rehabilitation from disease, injury and disability
- EXAMPLES- rehbilitative job training, vocational training

specificity
a screening test has high specificity - meaning that the test
accuately identifies those without the trait
-the test accuractely identigying those without the trait

routinely collected data


-obtained through vital recors such as birth and death
certificates , - things that are uniform across the country
- example of a category of data sources commonly used in
epidemilogical investigations

most important predictor of overall mortality


age

mortality curve by age


decreases during and after first year of life, low point in
childhood, increases in adolescents and young adulthood,
then increases sharply through middle and old ages

point epidemic
- temporal and spatial pattern of disease distribution
- illustrated with freuqncy of disease plotted against time
0 shapr peak indicates concentration of cases in some
short interval of time
- example- outbreak of GI illness from a food borne
pathogen

cohort study
describes a group of persons enroled in a study that share
the same characteristic of interest for a period of time

cross sectional studyuses info on current health status, personal characteristics,


ad potential risk factors or exposures all at once

community trial
as opposed to clinical trials * treatment of existing disease)
is focused on health promotion and disease prevention
Example: Residents of a city have recently voted to add
fluoride to the H20. Epidemiologists wanting to study the
effect on dental caries would be conducting a....

provide the highest level of evidence


randomized double blind controlled trials

research utilization projects in 1970s


0 provided a guide to clinical practice and was the
forerunner of evidence based nursing practive.

systemetic review
- one of the two ways nurses can read research in
condenced format
- method of identifying, appraising and synthesizing
research evidence to evaluate all available research to a
particular question
- usually done by more than one person and describes the
methods used to search for the evidence

evidence based research


- began in canada
- nurses can participate by doing active research or
reviewing best available evidence

community development model


focuses on achieving community goals and includes t a
partnership where power and decision making are shared
between community members and academic community

information access project


serves as a reource for community healh nuses as they
indentify the findings from research that have direct links
to population focuses and communty based care

public health nurse

informs, educates, and empoowers people about health


issues.
- does not solve problems but identifies them,
- participates with regulators to protect commnties and
empower populations to address health issues globally and
locally
- BSN is the necessary basic preparation to function as a
beginning saff public health nurse

population
a collection of individuals who share at least one common
characteristic

aggregate
definied poplation made up of individuals in communities fo
a specific geographic regions

assurance
making sure that community oriented helath servies are
avaiable .
a core function

core functions
assurance, policy development, assessment, and ascietific
based care

community health nursing


practice inlclueds the delivery of personal health services
to individuals, families and groups
- practie in the community wheter or noth they have had
preparation in public health nursing (a BSN)
-ie- school nurse, a nurse in a clinic, nurses who make
home visits, to provide tertiary car

CDC
- implement quality performance standards in public
health. which are used to guid e improvents in the public
health system.
- 1998

primary goal of public health


concern for the health of many people not as much the
individual

assessment

one of the core functions includes collecting data and


monioting the health status of the population ,

policy development
one of the core functions of public health needed to provide
leadership in deveoping policies.

determinants
- factors, exposures, characteristics and behaviors that
determine patterns of disease.
- may be individual , relational or social, communal, or
environmental

prevalence
the measure of existing disease in a population at a
particular time
Ex: a screening for HTN revleaed 20 previously diagnosed
hypertensive individuals and 10 probable new cases ,
which were later confirmed for a total of 30 cases.
total # of cases divided by total population at risk times
100,000

cause specific mortality rate


estimate of the risk of death from some specific disease in
the population

crude death rate


TOTAL # OF DEATHS DURING THE YEAR (DIVIDED BY)
TOTAL POPULATION X 1,000

predictive value
Positive predictor value refers to the proportion of persons
with a positive test who actually have the disease,
interpreted as the probability that an individual with a
positive test has the disease.
nursing model
a tangible schematic and often visual representation of
relationships between and among key concepts that
explain how something works

core
people and their demographics, vital statistics, values,
beliefs, community perception, and history; center which
represents things necessary for life

8 subsystems
physical environment, education, safety and transportation,
politics and government, health and social services,
communication, economics, and recreation

normal line of defense


indicates level of health over time of a core or community;
includes characteristics such as high rate of immunity, low
infant mortality, or middle income leve; usual patterns of
coping, along with problem-solving capabilities; represents
health of the community

flexible line of defense


response to a temporary stressor; buffer zone representing
a dynamic level of health resulting from a temporary
response to stressors; may be neighborhood mobilization
against an environmental stressor such as flooding or a
social stressor such as unwanted adult bookstore

lines of resistance
internal mechanisms that act to defend against stressors;
community strength; an evening recreational program for
young people implemented to decrease vandalism

stressor
tension-producing stimuli that have the potential of causing
disequilibrium in the system; may originate outside the
community or inside the community--inadequate,
inaccessible, or unaffordable services

degree of reaction
stressors and lines of resistance become a part of
community and nursing diagnosis; amount of
disequilibrium or disruption that results

primary level intervention


education to prevent stressor from happening; general
health promotion such as nutrition, hygiene, exercise, and
environmental protection; specific health promotion, which
includes immunizations and the wearing of protection
devices to prevent injuries

secondary level intervention

early detection to reduce impact of stressor; early


detection and treatment of adverse health conditionsdectect and treat a problem at the earliest stage:
screenings!

tertiary level intervention


restore optimal function; employed after diseases or events
have already resulted in morbidity; limit disability and to
rehabilitate or restore the affected people to their
maximum possible capacities

focus of community health nursing


disease prevention; health promotion, assurance that care
is provided; focus is at the population level

health risks of poverty


malnutrition and illness; exposure to pollution; poor diet;
more likely to smoke

vulnerability
social groups who have an increased relative risk or
susceptibility to adverse health outcomes; increased
morbidity, premature mortality, and diminished quality of
life

Who is eligible for CareLink


open to San Antonio/Bexar County residents who do not
have private or public health insurance; income must not
exceed 300% of the Federal Poverty Guidelines; program
gives quality health care at University Health System

Acceptable forms of identification for Carelink


1 form of identification for each member of the household-must show full name, date of birth and must include a
photo;
driver's license, state ID card, resident alien card, U. S
immigration card, U.S naturalization, citizenship or other
approved federal documents, passport,
Birth Certificate (children only)

Acceptable proof of residency for CareLink


1 form of proof of residency dated within the past 30 days
in Bexar County: household bill (electric, telephone, or
cable), lease agreement, automobile insurance documents,

Medicaid/Medicare/CHIP letter, Attorney General letter,


food stamp letter

Acceptable proof of income for CareLink


all income that comes into the household within the last 30
days; check stubs, Wage certification form, workman's
comp, proof of retirement income, documents or checks
from Federal/State agencies, Texas Workforce Commission
notice of benefits; social security disability award letter,
complete and current income tax return

CareLink
financial assistance program for medical services received
within University Health System and our providers;
payment plan and payer of last resort; not health insurance
and not free

cultural competence
the ability to express an awareness of one's own culture, to
recognize differences between oneself and others, to adapt
behaviors to appreciate and accommodate those
differences; recognize that people with cultural
backgrounds different from our own have unique values,
life ways, health practices, and interpersonal styles.

benefits of cultural knowledge


improved relationships in one's personal and professional
life; less likelihood that conflict will develop in your
relationships with others; greater customer satisfactionimportant aspect of our professional life; develop beneficial
community partnerships

population
could include a race, ethnic group, religious group,
profession, a company, people with a certain sexual
preference, or gender; even from the area of the country
that one resides

cultural proficiency
having a vision that you can create an environment for
social change and social justice in your interactions with
colleagues, families and communities to effectively serve
the needs of all cultural groups

family

refers to two or more persons who are joined together by


bonds of sharing and emotional closeness and who identify
themselves as family

cultural encounter
process which encourages the healthcare professional to
directly engage in face-to-face cultural interactions and
other types of encounters with clients from culturally
diverse backgrounds in order to modify existing beliefs
about a cultural group and to prevent possible
stereotyping; exposure and practice

cultural knowledge
process in which the healthcare professional seeks and
obtains a sound educational base about culturally diverse
groups; healthcare professionals must focus on the
integration of three specific issues: health-related beliefs
practices and cultural values; disease incidence and
prevalence; cultural views, theoretical and conceptual
framework

cultural awareness
conducting self examination; acknowledgment and
awareness of one's own culture and willingness to explore
one's own feelings and biases; awareness of how culture
influences own ways of thinking and making decisions;
acknowledgement of how day-to-day behaviors reflect
cultural norms and values perpetuated by our families and
larger social networks

cultural assessment
use client's preferred language; be aware of client's
priorities; check client's understanding of diagnoses, and
treatment plan; ask whether client is using alternative
treatments; what or who is the support system; ask what
they think about the problem; ask what treatment they
think will help; what do they hope to receive from your
care; what do they fear most about the problem or
treatment; keep in mind background and individual and
biological differences

cultural assessment and healthcare


language-is an interpreter needed; level of language
literacy
beliefs and customs about health, illness and death

education
Religion-does the patient/family have beliefs that you need
to respect and accommodate for to enable you to care for
them
family forms and meaning of family

Vulnerability Factors of Elderly: Economic concerns


financial-paying off mortgages, healthcare expenses
health care costs-prescriptions and hospital care costs
elder abuse-domestic, institutional or self-neglect
access to health care-transportation, rural areas

vulnerability factors of elderly: social concerns


disasters-effects of hurricane, flooding
community safety and fear of violene
mental health-Alzheimer's dementia
accidents-falls or medication mismanagment

ethnocentrism
the belief that your cultural values, beliefs, and life ways
are superior to all others; the standard by which one judges
all other cultural patterns; one must be self aware of
perceptions and subsequent judgments are not
ethnocentric

breast cancer screen for elders


breast cancer: 50-74 yrs every two years

prostate cancer screen


start at 40 yrs, especially African Americans who are at
higher risk

HIV/AIDS screen for elders


15% of all cases are in adults 50yrs and older; older people
are at increased risk for HIV and STIs

preventive health screenings for elders


vaccines (flu, pneumonia-yearly-shingles after 60 yrs), BP,
cholesterol, fecal occult blood, sigmoidoscopy,
colonoscopy, vision and hearing, HIV

depression screenings for elders


individual-as needed (elder abuse may be occuring), group
screen should be part of a multicomponent multiservices
followup and treatment effort

elder abuse screening


mistreated elders are often frail, dependent, over age 70,
and women

Possible indicators of abuse


cuts, punctures, burns, bruises, welts, scratches, scalp
injury, gag marks, sprains, broken bones
sexually transmitted disease, rectal bleeding, torn or
bloody clothing
caretaker cursing, yelling and screaming, threats of harm,
intimidation

behavioral signs of abuse


fear, anxiety, agitation, anger, isolation, withdrawal,
depression, non-responsiveness, resignation, ambivalent,
contradictory statements, implausible stories, hesitation to
talk openly, confusion or disorientation

factors contributing to abuse, neglect, and exploitation of


elderly
aging population; growing number of adults with
disabilities; alcohol and drug dependency; unemployment;
lack of affordable housing and high costs of utility bills;
deinstitutionalization of mentally ill; inadequate access to
health care and cost; pathological family relationshipsviolence as coping mechanism; physical and mental stress
of caregiving

Elders and disaster


disproportionately vulnerable during disasters because
they are more likely to have chronic illnesses, functional
limitations, and sensory, physical, and cognitive
disabilities; live alone and on limited incomes making it

nearly impossible to recover from a disaster without special


assistance; often slow to request help

modifiable risk factors of vulnerable populations


poverty, low education and level of social support

non-modifiable risk factors of vulnerable populations


age, gender and ethnic status

health disparities
differences in the incidence, prevalence, mortality, and
burden of diseases and other health conditions among
specific population groups.

Health Leads
national nonprofit organization that connects low-income
patients with the basic resources they need to be healthy

social determinants of health


health is impacted by many social, economic and
environmental factors that extend far beyond individual
biology of disease and contributes to risk; aspects of
society and the social environment that impact on health,
such as poverty, early life experiences, social networks and
support, education, income, cultural traditions

upstream determinants
features of the social environment, such as socioeconomic
status and discrimination, that influence individual
behavior, disease, and health status.-preventive, health
promotion

downstream preventions/interventions
short-term, problem-specific, individual-based
interventions; after the fact--after disease process has
started.

community as partner model (CAP)


focuses on: the community as partner/the nursing process.
the community is composed of a core population and eight
subsystems, which are depicted visually as a wheel with
the population at the hub and with surrounding
subsystems. nurse works in partnership with community to
help health.

community assessment
act of becoming acquainted with a community; assessing a
community to identify factors (both positive and negative)
that impinge on the health of the people to develop
strategies for health promotion

Lillian Wald
recognized the intertwining of health status, environmental
sanitation, and social and political forces; mother of public
health nursing; contributions include establishing nursing
schools, advocating for better housing, working to change
child labor laws, teaching preventive practices, advocating
occupational health nursing, and improving the education
of public health nurses

Mary Brewster Lillian Wald


founded the Henry Street Settlement House in New York
City; taught hygiene practices, visiting the sick in homes,
and crusading for better health care in all aspects of the
community

Public Health Achievements in the 20th Century


1. Immunizations
2. Improvements in motor vehicle safety
3. Workplace safety
4. Control of infectious diseases
5. Decline in deaths from heart disease and stroke
6. Safer and healthier foods
7. Healthier mothers and babies
8. Family planning
9. Fluoridation of drinking water
10. Recognition of tobacco as a health hazard

Public health focus now


more focus on disease prevention, health promotion, and
assurance that care is provided, rather than providing oneon-one care

Neuman's system model


provides a holistic and system based perspective for
nursing; focuses on the response of the client system to
actual or potential environmental stressor; uses primary,
secondary, and tertiary prevention intervention for
retention, attainment, and maintenance of optimal system
wellness

Factors Shaping 21st Century Health


health care delivery system; demographics; globalization;
poverty and growing disparities; primary health care;
violence, injuries, and social disintegration; bioterrorism

leading causes of death now


coronary heart disease: stroke (CVA); lower respiratory
infections: chronic obstructive pulmonary disease; diarrheal
disease; HIV/AIDS; tuberculosis; trachea, bronchus, lung
cancers; road traffic accidents; prematurity and low birth
weight

windshield survey
crucial to community assessment because it serves as a
model, or map, to direct and guide that assessment
process; it guides the community assessment; learning
about the community of foot/drive by

CAP Assessment
CAP will guide assessment process; assess core and 8
subsystems (relationships); many methods used to assess
data-observation, interviews, data collection

CAP Planning
goals derived from stressors (weaknesses): aim to mitigate,
alleviate, eliminate the stressor; strengthen the
community's lines of defense; plan interventions to
strengthen lines of resistance

CAP Interventions
all are preventive in nature; Primary-aims at strengthening
the lines of defense, secondary-applied after stressor has
penetrated the community, and tertiary-applied after
stressor pentrates and the degree of reaction has taken
place resulting in system disequilibrium

CAP Evaluation
based on feedback from the community: involves
community in all phases of the nursing process to ensure
success
outcomes: may use same parameters as used in
assessment

process: working with the community as partner

Disaster Category A agents


can be easily disseminated; results in high mortality rates;
public panic and social disruption; requires public health
preparedness: anthrax, botulism, plague, smallpox,
tularemia, viral hemorrhagic fevers

Disaster Category B agents


second highest priority agents; moderately easy to
disseminate; moderate morbidity rates and low mortality
rates: brucellosis, food safety threats, glanders, melioidosis
Q fever, Ricin toxin

Disaster Category C agents


third highest priority agents; emerging pathogens that
could be engineered for mass dissemination in the future
because of availability; ease of production and
dissemination; potential for high morbidity and mortality
rates and major health impact

bioterrorism
intentional release of pathogens or biologic agents to cause
harm--biological disaster

signs of stress during disaster


minor tremors, nausea and vomiting, lack of concentration,
memory and problems, irritability, fatigue, anger, H/A

pandemic
steady occurrence of disease over a large geographic area
or worldwide, such as malaria in Africa

epidemic
unexpected increase of an infectious disease in a
geographic area over an extended period of time; defined
relative to the infectious agent and the history of the
disease in the area

endemic
occur at a consistent, expected level in a geographic areaSTI, tuberculosis

outbreak
unexpected occurrence of an infectious disease in a limited
geographic area during limited period of time

observational care site


sufficiency of care model for those who require monitoring
and skilled nursing assistance-under normal circumstances
most guests found in an assisted living center or nursing
home

low acuity patient care site


to permit "hospital decompression;" result transferring
stabilized patients from hospitals to community based
medical shelters or an ACS location to alleviate hospital
overcrowding

ambulatory care clinic


site would be utilized to provide medical care to "walking
wounded" or others whose condition is considered subacute; this type might include a point of dispensing (POD)
for the medications or vaccination

primary triage point


may be established near an impact zone or in a close
proximity to a hospital; patients would be quickly evaluated
to determine prioritization for transport to hospitals or
other designated locations

federal medical station


a cache to medial supplies and equipment that can be used
to set up a temporary acute a non-acute care facility; each
FMS has beds, supplies and medicine to treat up to 250
people for up to 3 days; sites used for this purpose must be
larger than a typical medical shelter due to bed count
requirements (40,000 square feet minimum)

alternate care site


non-hospital, community based location that my provide
additional treatment area(s) with a minimum specific level
of care for patients; may be established in a facility of
opportunity or a temporary structure such as a tent or
Zumro

Functional Needs Support Services

services that enable children and adults to maintain their


usual level of independence in general population shelters:
reasonable modifications to policies, practices and
procedures; durable medical equipment; consumable
medical supplies (pharmaceuticals); personal assistance
services; other goods and services as needed

most vulnerable in a disaster


children, seniors, poor, chronically ill, disabled,
homebound, developing countries

Stage 1 of disaster: prevention


modify homes to resist storms, quakes, etc; self-sustaining
communities; situational awareness; develop regional,
national, and international understanding and cooperation
concerning potential disasters; environmental, urban and
housing development, poverty-all have an impact

Stage 2 of disaster: Preparedness


disaster management begins with preparation; preparing in
advance will help you help others; community nurses play
key role in educating public about preparedness

Stage 3 of disaster: Response (disaster paradigm)


detect; incident command; scene security and safety;
assess hazards; support; triage and treatment; evacuation
and transport of Victims/Casualties

Stage 4 of disaster: Recovery


address immediate needs; needs several days after the
incident; address long term needs

START triage system


Simple Triage and Rapid Treatment; uses a systematic
approach based on 3 observation (respiration, perfusion,
and mental status) to triage multiple victims into 4
categories; 30-60 seconds per victim; green:
minor/delayed, yellow: delayed/urgent, red:
critical/immediate, black: expectant-dead or dying (last)

Mass Triage system


Move-request those able to move and walk to move to a
specified safe site
Assess: assess individual patients, beginning with those

unable to move
Sort: categorize victims based on urgency of care
Send: transport injured persons away from the scene

triage in disaster
emphasis may shift from meeting the most urgent
individual need to meeting the needs of the largest
numbers of victims to maximize the number of lives saved;
should be an ongoing activity. Assess, Assess, Assess-move
move move.

SARS
severe acute respiratory syndrome, outbreak in 2003

disaster
occurs as the result of an event where "normal conditions
of existence are disrupted and the level of suffering
exceeds the capacity of the hazard-affected community to
respond to it"

emergency
occurs as the result of an event where "normal procedures
are suspended and extraordinary measures are taken in
order to avert the impact of a hazard on the community"-can become a disaster

hazard
occurs when an event "has the potential to cause
disruption or damage to the community; earthquake, flood,
typhoon and cyclone--some may become emergencies but
not all become disasters

nursing roles in disaster: stage 1/prevention


organize and participate in community immunization
activities; participate in agency or community vulnerability
assessments and strategies for improvement;
develop/implement prevention strategies for worksites,
community sites, and home

nursing roles in disaster: stage 2/preparedness


volunteer for emergency response teams; locate/be able to
respond to institutional emergency plans; develop/update

institutional/community emergency plans or protocols;


develop volunteer registry to respond to emergency needs

nursing roles in disaster: stage 3/response


assist with emergency plan activation; assist with resource
mobilization; assist with public education; assist with
establishment and maintenance of shelters; provide
nursing care; assess public health needs; implement
disease surveillance, vector control, hygiene measures,
safety measures for food and water, etc
colonial period
household members (usually women) tend to the sick
(urbanization in the early 1800s causes this system to
became insufficient)

England's Elizabethan Poor LAw of 1601


1751 pennsylvania hospital founded

early colonial PH
efforts

after american revolution


first PH committee was established

Public health service was established in


1798 as the marine hospital service

early experiments in providing nurisng care at home


ladies benevolent society of Charleston 1813
philadelphia lay nurses
Roman catholic sisters of charity 1854

shattuck report 1850 by the


Massachusetts sanitary comission

nightingale and the origins of trained nursing


need for nurses
origins of organized nursing(pastor theodor fliedner,
Crimean War (improved soldiers' health using a populationbased approach)
Principles of Nursing

Principles of Nursing

health of the unity is the health of the community


differentiated"sick nursing" or "health nursing"
proper nutrition, rest, sanitation, and hygiene necessary

William Rathborne
founded first district nursing association in liverpool,
england
rathbone and nightingale recommended steps to provide
nursing in the home, and district nursing was organized
thorughout england

florence sarah lees craven


guide to district nurses

America Needs Trained Nurses


Need for PH nursing
Increase of women in workforce
more economical to have home-visiting nurses

1870s
first nightingale model nursing schools started

1877
women's board of the new york city mission hired frances
root

1878
ethical culture society of new york hired four nurses to
work in dispensaries

1885-1886
visiting nurses association were established

district nursing and settlement houses


deplorable enviornmental conditions in immigrant
tenement housing and sweatshops
District nursing and settlement houses established
Rural nursing service
occupational health

school nursing in america

grew out of school absences caused by the prevalence of


infections and communicable diseases

lina rodges
first united states school nurse
worked with children in NYC schools
She and her other school nurses found illness was often not
the reason for absence

significant impact and positive impact on school nursing


more nurses hired and school nursing was soon
implemented in los angeles, philadelphia, baltimore,
boston, chicago, and san francisco

The profession comes of age in the United States in the


Twentieth Century
National organization for PHN (NOPHN)
American Public Health Assocation (APHA)

National organization for PHN


sought to standradize PHN education
1914- first post training school course in PHN offered at
TEAchers college in NYC
1920s-1930s- many newly hired PHN had to verify
completion in a certificate program in PHN

American Public Health Association APHA


established in 1872
sought to facilitate interprofessional efforts and promote
the practical application of public hygiene

late in 1800s
local health departments formed

federal role in PH gradually expanded


1912- US public health service role defined
NOPHN loaned a nurse to the US public health service- first
federal governemtn sponsorship of nurses

WWI depleted the ranks of


PHNs

1918

world wide influenza pandemic

Paying the Bill for PHNs


Metropolitan life insurance company
sheppard-towner act
individual commitment and private financial support
frontier nursing service

African-american nurses in PHN


1919: national health care circle for colored people (Bessie
M. Hawes)
1936: PHN certificate program for African- American nurses
wage discrimination in the south
nursing education segregated until 1960s

Economic depression
agenecies and communities not prepared to address the
increased needs and numbers of impoverished (decreased
funding)
Agencies that helped to support nurse employment
(Federal Emergency relief administration, works progress
administration, relief nursing service, civil works
administratin)

1932 survery
found only 7% of nurses working in PH were adequtely
prepared

social security act of 1935


funded opportunities for education and employment of
PHNs
Funded assistance to states, counties, and medical districts
in establishing adequate health services
provided funds for research and investigation of disease

world war II
accelerated need for nurses, both for war effort and home
nursing council on national defense
many nurses joined the army, and navy nurse corps.
bolton act of 1943 established cadet nurses corps
some expansion of PHN scope of practice
Emergency maternity and infant care act of 1943
Job opportunities

Rise in chronic illness


national crude mortality rate decreased by 47%
change in leading cause of death from communicable
diseases to chornic diseases
aged population grew as did prevlaence of chronic disease
some visiting nurse associations provide home-care
programs
reimbursable by commerical health insurance and later by
medicare and medicaid
resurgance in combination agencies

declining financial support for practice and professional


organizations
hospitals preferred for illness and childbirth
funding stopped for visiting nurse services
consolidation of national nursing organizations
national leauge for nursing
american nurses association

Professsional Nursing Education for PHN


national league for nursing adopted Esther Lucile Brown's
Nursing for the Future 1948. Recomended to establish basic
nursing preparation colleges and universities. Included PHN
concepts in all baccalureate programs; however, these
were very brief components of teh cirrumula
1950s PHN practice increased focus on the psychological
elements of client, family and community care

1960s
medicare and medicad
did not include coverage for preventive services
home health care only reimbursed if ordered by a physician
increase in for-profit home health agencies
reduction in health promotion and disease prevention by
local and state departmetns

community organization and professional change


civil rights movement
funding increaes in certain areas
economic opportunity act

1970s
Nurse made significant contribution to:
the hospital movement
the development of birthing centers

day care for older adults and disabled persons


drug abuse programs
rehabilitation services in long-term care

1980s
concern about high health costs
fundign shifted away from health promotion and disease
prevention to acute care
national center for nursing research (NCNR) established in
1985 (gained offical status within NIH in 1993, beocming
NINR
IOM's report the future of public health 1988
Healthy people initaitve began

1990s
health care debate focused on cost, quality, and access to
direct care services
nursing organizations joined to support health care reform

2000s
health care reform finally passed in 2010 with the federal
patient protection and affordable care act
PHN organizations develop position papers on:
graduate education for advance practice PHN
faculty qualifactions for community/public health nursing
educators
importance of PHN within PH systems
Better nutrition, water, antibiotics, immunizations. PUBLIC
HEALTH.
Life Expectancy - U.S.
What factors/events resulted in the 21+ additional years of
life from 1900-1950???
What factors/events resulted in the 7 additional years of
life from 1950-1990?

1. Upstream
2. Downstream
There are two views of the health care picture
We need to question the way we have thought of health
and health care...
Should we focus on treatment of illness or prevention of
disease in the first place?

How should we act?


______ or ______

1. Microscopic
2. Medical Model
3. Downstream
_______ approach to solving community health problems
(the _____ or _____):
- Individual (family) response to health and illness
- Emphasizes behavioral responses to illness or lifestyle
patterns
- Nursing interventions aimed at the individual
------Changing lifestyles
------Changing perceptions or belief system
- The individual is the locus of change
- Focuses on the "cure"

1. Macroscopic
2. Public Health
3. Upstream
______ approach to solving community health problems
(______ or _______)
- Interfamily and intercommunity themes
- Emphasizes social, economic, and environmental
precursors of illness
- Nursing interventions may include modifying social or
environmental variables
- May involve social or political action
- society (social system) is the locus of change
- focuses on "prevention"

1. Health Belief Model


2. Health Promotion Model
3. Transtheoretical Model
4. Critical Social Theory
CHN practice is based upon theory. Theories are used
regularly in CHN. Some examples:
1.
2.
3.
4.

1. The Health Belief Model


2. Perceived susceptibility
3. Perceived severity
4. Perceived benefits
5. Perceived barriers to action
6. Cues to action
7. Self-efficacy
_________:
- focuses on a person's perceptions of a threat or a health
problems and related appraisal of a recommended
behavior to elicit change
- developed in the 1950s and was one of the first models
using concepts and assumptions from behavioral sciences
to examine health behaviors
------- Addresses attitudes and beliefs of individuals
------- Focus was on increasing the use of preventive health
services
- Primary assumptions of the model are that people fear
disease and that health actions are motivated based on the
extent of the fear and belief in benefits obtained by
preventative action
- Five (Six?) Primary Constructs:
1.
2.
3.
4.
5.
6.

- "Using the Health Belief Model to Predict Injecting Drug


Users' Intentions to Employ Harm Reduction Strategies"
(Bonar & Rosenberg, 2011)
Studied whether perceived risk of infection would
impact intention to clean site pre-injection. Also studied
perceived risk of non-fatal overdose and use of a test
shot.
Found that the perceived risk of infection and overdose
did impact behavior if the user was not in withdrawal.
Application of the HBM in CHN:

Pender's Health Promotion Model


__________:
o Developed by Nola Pender to study health promotion
behaviors; initially published in 1982
o Explores biopsychosocial processes that motivate
individuals to engage in behaviors that promote health
o Has basis in the HBM

o Depicts the complex multidimensional factors with which


people interact as they work to achieve optimum health.
o This model does not include threat as a motivator, as
threat may not be a motivating factor for client's in all age
groups.
o Used by nurses to develop and execute health-promoting
interventions
o Used to develop research studies focusing on one aspect
of health promotion
o Used frequently as a framework for research studies
o Individual Characteristics and experiences that may
affect their health actions:
- Prior related behavior
- Personal factors
- Behavior specific cognitions and affect
- Perceived benefits of action
- Perceived self-efficacy
- Activity-related affect
- Interpersonal influences
- Situational influences
- Commitment to a plan of action
- Immediate competing demands and preferences
- Health-promoting behavior

"Registered Nurses' Beliefs of the Benefits of Exercise,


Their Exercise Behavior and Their Patient Teaching
Regarding Exercise" (Esposito & Patrick, 2011).
Wanted to know if nurses who engage in healthy
behaviors were more likely to recommend them to their
patients.
Found that nurses who believe in health promotion and
embrace healthy behaviors were more likely to be
positive role models and were more likely to teach
healthy behaviors to their patients.
- More examples: Health promotion for community dwelling elders and Prevention of melanoma
Application of the HPM in CHN:

1. Transtheoretical Model (Stages of Change)


2. Precontemplation
3. Contemplation
4. Preparation
5. Action
6. Maintenance
_________:
o Based on the assumption that behavior change takes

place over time


o Change is difficult
o Change progresses through a sequence of stages
-----------One may stop at a stage, progress to the next
stage or return to the previous stage
o Stages of change: 1.______, 2._____, 3.______, 4._____,
5.______
o Decisional Balance:
---------Pros: The benefits of behavior change
---------Cons: The costs of behavior change

1. Precontemplation
2. Contemplation
3. Preparation
4. Action
5. Maintenance
Transtheoretical Model (Stages of Change):
1. _______: No intention to change behavior in the next 6
months
------May be lack of information about consequences or
previous failure.
2. _______: Individual intends to change behavior in the next
6 months.
------Weighs pros and cons
3. _______: Individual intends to act within the next month
and has taken steps toward change
------Has a plan of action
4. _______: Individual has changed behavior for less than 6
months
------Change is sufficient to reduce disease risk.
5. _______: Individual has changed behavior for more than 6
months.
------Tries to prevent relapse
------Phase may last months to years

"Adapting the Transtheoretical Model of Change to the


Bereavement Process" (Calderwood, 2011).
Applies the model to the bereavement process.
Postulates that bereaved persons never return to their
previous state but undergo change as they cope with and
adjust to life without their loved one.
Application of the TTM in CHN:

1. Smoking cessation
2. Injury prevention
3. RNs assisting families obtain health insurance

Examples for applying Transtheoretical Model in nursing:


1.
2.
3.

1. Critical Social Theory


2. Enlightenment
3. Empowerment
4. Emancipation
_____________:
- Jurgen Habermas is the best known of the theorists for
this
-------Habermas was part of a group of German scholars in
Frankfurt writing in the 1960s
-------He promoted critical social theory to describe
"distortions and constraints that impede free, equal and
uncoerced participation in society."
- began in Marx's argument that oppression requires
revolutionary action
- uses societal awareness to expose social inequalities that
keep people from reaching their full potential
-------Promote equality
-------Reduce disparities (ie. health disparities)
- aims to provide an environment in which individuals can
become empowered, enlightened and emancipated
-------- ______: raising the consciousness of the oppressed.
-------- ______: encourages people to undertake activities to
improve their situation
-------- ______: is the goal of empowerment through which
new arrangements replace oppressive ones
- This perspective is informed by the following values and
assumptions:
---------- The problem and inequalities of health and health
care are connected to the particular historically located
social arrangements and the cultural values of society
---------- Health care should be oriented toward the
prevention of disease and illness
---------- The priorities of any health care system should be
based on the needs of the clients/population and not health
care providers
---------- Ultimately, society itself must be changed for
health and medical care to improve

"Why Carers of Older People are not using Available


Respite Services: An Australian Study (Stockwell-Smith,
Kellett, & Moyle, 2010).
Examined use of respite services by caregivers to

determine why some opted not to use these services.


Found that a lack of knowledge of services and trust in
the workers were factors in using respite care.
Found that caregivers were defined by their role and
could not easily relinquish it to another.
Application of CST in CHN:

1. Health Disparities
2. Health and social justice
Examples of applying critical social theory:
1.
2.

- A state of complete well-being, physical, social, and


mental, and not merely the absence of disease or
infirmity. World Health Organization, 1958
- The extent to which an individual or group is able, on
the one hand, to realize aspirations and satisfy needs;
and, on the other hand, to change or cope with the
environment. Health is, therefore, seen as a resource for
everyday life, not the objective of living; it is a positive
concept emphasizing social and personal resources, and
physical capacities. WHO 1986
How do we define health?

1. Purposeful actions
2. Processes
3. Responses
4. Behaviors
Definitions of Health:
Health consists of:
_____, ______, _____, or _____ that leads to: "soundness,"
"wholeness," or "well-being"

1. Physical Environment
2. Social Environment
3. Individual Behavior
4. Biology & Genetics
5. Health Services
6. Policy making
Determinants of Health:
Health Outcomes due to:
1.

2.
3.
4.
5.
6.

Physical Environment
Determinants of Health - Examples of ______ factors:
- Natural environment (ie plants, weather, climate change)
- Worksites, schools and recreational settings
- Housing, homes, neighborhoods
- Exposure to toxic substances
- Physical barriers
- Aesthetic elements (ie good lighting, trees, parks)

Social Factors
Determinants of Health - Examples of ______ factors:
- Availability of resources (ie educational and job
opportunities, living wages, healthful foods)
- Social norms and attitudes
- Exposure to crime and violence
- Social interactions
- Exposure to emerging technology (ie the Internet)
- Transportation options

Individual Behaviors
Determinants of Health - Examples of ______ factors:
- Diet
- Physical activity
- Alcohol, tobacco and other drug use
- Sexual behavior
- Hand washing

Biology and Genetics


Determinants of Health - Examples of ______ factors:
- Age
- Sex
- Race/ethnicity
- Inherited conditions
- Family history of diseases

Health Service
Determinants of Health - Examples of ______ factors:
- Barriers (lack of availability, high cost, lack of insurance,
language issues)
------- Unmet health needs

------- Delays in receiving care


------- Inability to get preventive services
------- Preventable hospitalizations

Policy Making
Determinants of Health - Examples of ______ factors:
- Health curricula requirements
- Increased taxes on tobacco
- Prohibitions on smoking
- Drinking age restrictions
- Seat belt laws and child restraints

1. Life expectancy
2. Health life expectancy
3. Years of potential life lost
4. Physically and mentally unhealthy days
5. Self-assessed health status
6. Limitation of activity
7. Chronic Disease Prevalence
Indicators of General Health Status:
1.
2.
3.
4.
5.
6.
7.

Community
___________:
- It can be a physical place or a geopolitical community.
- Has boundaries
- It can be a relational, interactive group. - A community
with no physical boundaries. A phenomenological
community is abstract. Churches, universities, online
groups are examples

Community
__________:
"A collection of people who interact with one another and
whose common interests or characteristics form the basis
for a sense of unity or belonging." (Nies and McEwen,
2011)

Community

__________:
"A group of people who share something in common and
interact with one another, who may exhibit a commitment
to one another and may share a geographic boundary."

1. People
2. They must interact with each other in some way
3. Something in common: an interest, a geographical
location, a commitment
A community has to have:
1.
2.
3.

1. Primary Prevention
2. Secondary Prevention
3. Tertiary Prevention
Levels of Prevention:
1.
2.
3.

Primary Prevention
_______ level of prevention
- activities preventing a problem before it occurs
- Health promotion
- Specific protection
- Immunizations

Secondary Prevention
________ level of prevention
- Early detection and prompt intervention
- Screening
- Early referral for treatment
- Screening for STDs

Tertiary Prevention
_______ level of prevention
- Focus on limitation of disability and rehabilitation
- Prevention progression of disease
- Reduce the effects of the disease
- Teaching insulin administration

Community Health Nursing (CHN)


_________:
o Focus on individuals and families where they live, work,

and go to school; care is setting-specific and the emphasis


is on acute and chronic care
o "The synthesis of nursing practice and public health
practice applied to promoting and preserving the health of
populations.

Public Health Nursing (PHN)


_________:
o Focus is on the community as a whole and the effect of
community health status (resources) on the health of
individuals, families and groups
o Consideration of the health of individuals, families, and
groups and their effect on the health of the community as a
whole
o The synthesis of public health and nursing practice.
o "The practice of promoting and protecting the health of
populations using knowledge from nursing, social, and
public health sciences."

Public/Community Health Nursing (PHN)


__________:
- According to the ANA, PHN is the
o ...practice of promoting and protecting the health of
populations
o ...using knowledge from nursing, social and public health
sciences

Public Health Nursing


_______ Practice is:
1. Population focused
2. Goals of promoting health and preventing disease and
disability
3. Focuses on all people through the creation of conditions
in which people can be healthy

Public/Community Health Nursing


__________:
- Focuses on the entire population
- Is based on assessment of the population's health status
- Considers the broad determinants of health
- Emphasizes all levels of prevention
- Intervenes with communities, systems, individuals, and
families
- like an umbrella, Covers:
--------- Home Health Nursing
--------- Palliative and Hospice Nursing

---------------------------------

Public Health Nursing (official agencies)


School Nursing
Occupational Health Nursing
Correctional Nursing

o ...practice is general & comprehensive; not limited to an


age or diagnostic group
o ...is continuing, not episodic
o ...is directed to communities, groups, & individuals as it
contributes to the health of the total population
o Goal is to protect and promote the health of the entire
population
o Populations can be defined by geography, demographic
characteristic or need
What is Public Health Nursing (PHN)?

Anywhere people come together


Neighborhoods
Public Health Agencies
Community Centers
Day Care Centers
Schools
Workplaces
Housing projects
Settings for Public Health Nursing:
1.
2.
3.
4.
5.
6.
7.
8.

Community-based nursing
___________:
o Nursing of individuals and families to improve their
health
o Goals: Help them manage illness while they move among
health care settings
o Promote self-care and rehabilitation; prevent disease
o Processes: NP; diagnosis and treatment

Wherever individuals and families need nursing care


outside the hospital or nursing home
Home

Community clinics
School clinics
Workplace clinics
Rehabilitation centers
Settings for Community-based nursing:
1.
2.
3.
4.
5.
6.

Clinical Nursing
_________:
o Goal: improve the health of patients
o Clients: Patients of the health care system
o Processes used: nursing process, treatment and patient
care procedures
o Settings: inpatient

The Public Health Nurse


The Standards of Care:
__________:
- Assesses the health status of populations using data,
community resource identification, input from the
population and professional judgment
- Analyzes collected assessment data and partners with the
people to determine population diagnosis and priorities
- Participates with other community partners to identify
expected outcomes in the populations and their health
status based on population diagnoses and priorities
- Develops a plan that reflects best practices by identifying
strategies, action plans and alternatives to attain expected
outcomes
- Implements the identified plan by partnering with others
- Employs multiple strategies to promote health, prevent
disease and ensure a safe environment for populations
- Evaluates the health status of the population

1. Morbidity data - illness or disability


2. Mortality data - death
Indicators of Health and Illness:
- Health Statistics:
1.
2.

1. Surveillance
2. Disease and other health event investigation
3. Outreach
4. Screening
5. Case finding
6. Referral and Follow-up
7. Case management
8. Delegated functions
9. Health Teaching
10. Counseling
11. Consultation
12. Collaboration
13. Coalition Building
14. Community Organizing
15. Advocacy
16. Social Marketing
17. Policy and enforcement
Public Health Interventions:
1.
2.
3.
4.
5.
6.
7.
8.
9.
10.
11.
12.
12.
13.
14.
15.
16.
17

Surveillance
Describes and monitors health events through ongoing and
systematic collection, analysis and interpretation of health
data for the purpose of planning, implementing and
evaluating public health interventions

Disease and other health event investigation


Systematically gathers and analyzes data regarding threats
to the health of populations, ascertains the source of the

threat, identifies cases and others at risk, and determines


control measures

Outreach
Locates populations of interest or populations at risk and
provides information about the nature of the concern, what
can be done about it, and how services can be obtained

Screening
Identifies individuals with unrecognized health risk factors
or asymptomatic disease conditions in populations

Case finding
Locates individuals and families with identified risk factors
and connects them with resources

Referral and Follow-up


Assists individuals, families, groups, organizations and/or
communities to identify and access necessary resources to
prevent or resolve problems or concerns

Case Management
Optimizes self-care capabilities of individuals and families
and the capacity of systems and communities to coordinate
and provide services

Delegated Functions
Direct care tasks a registered professional nurse carriers
out under the authority of a health care practitioner as
allowed by law

Health Teaching
Communicates facts, ideas, and skills that change
knowledge, attitudes, values, beliefs, behaviors, and
practices of individuals, families, systems, and/or
communities

Counseling
Establishes an interpersonal relationship intended to
increase or enhance capacity for self-care and coping with
a community, system, and family or individual

Consultation

Seeks information and generates optional solutions to


perceived problems or issues through interactive problem
solving with a community, system, and family or individual

Collaboration
Commits two or more persons or organization to achieve a
common goal through enhancing the capacity of one or
more of the members to promote and protect health

Coalition Building
Promotes and develops alliances among organizations or
constituencies for a common purpose

Community Organizing
Helps community groups to identify common problems or
goals, mobilize resources, and develop and implement
strategies for reaching the goals they collectively have set

Advocacy
Plead someone's cause or act on someone's behalf, with a
focus on developing the community, system, and individual
or family's capacity to plead their own cause or act on their
own behalf

Social Marketing
Utilizes commercial marketing principles and technologies
for programs designed to influence the knowledge,
attitudes, values, beliefs, behaviors, and practices of the
population of interest

Policy development and enforcement


Places health issues on decision-makers' agendas, acquires
a plan of resolution, and determines needed resources,
resulting in laws, rules, regulations, ordinances, and
policies. Policy enforcement compels others to comply with
laws, rules, regulations, ordinances, and policies

John Snow
_______: Father of epidemiology

1. Pasteur
2. Lister
3. Koch
____, _____, _____: On "germs" and disease causation (late
1800s)

1. Edward Jenner
2. Edwick Chadwick
History of Public Health Nursing:
Pre 1850: Home visiting to sick as an act of charity.
- ______ observed people who worked around cattle were
less likely to have smallpox.
- _______ called attention to the consequences of unsanitary
conditions that resulted in health disparities that shortened
the life span of the laboring class in particular.

1. John Snow
2. Florence Nightingale
3. Pasteur, Lister, Koch
4. Lilian Wald
5. Mary Brewster
History of Public Health Nursing:
1850-1900: Home visiting to sick
- _______ demonstrated that cholera was transmissible
through contaminated water.
-________ - credited with establishing "modern nursing."
- ________: On "germs" and disease causation (late 1800s)
- In 1893, nurses _______ ("mother of community nursing")
and _______ established a district nursing service on the
lower east side of New York City called The House on Henry
Street.

1. 1900-1960s
History of Public Health Nursing:
________:
- Nursing in community centers for the poor
- Communicable disease control
- Communicable disease control/immunization
- Rise in PHN, home visits, school and OH nursing

1. 1960-1980
History of Public Health Nursing:
________: Care provided in public health clinics

1. 1980s-present
History of Public Health Nursing:

_________:
- Health promotion and education
- Health care access improvement

1. Poverty
2. Violence
3. HIV/AIDS
4. TB
5. Bioterrorism
Public/Community Health Nursing Today in U.S.
- Concerned with contemporary problems such as _____,
______, ______, _______, and _______.
- Practice is everywhere in public health agencies, schools,
occupational settings, community-based agencies, etc.

Healthy People
__________:
- provides science-based, 10-year national objectives for
improving the health of all Americans. For 3 decades, this
has established benchmarks and monitored progress over
time in order to:
o Encourage collaborations across communities and
sectors.
o Empower individuals toward making informed health
decisions.
o Measure the impact of prevention activities.
- Vision: A society in which all people live long, healthy lives

1. Identify nationwide health improvement priorities.


2. Increase public awareness and understanding of the
determinants of health, disease, and disability and the
opportunities for progress.
3. Provide measurable objectives and goals that are
applicable at the national, State, and local levels.
4. Engage multiple sectors to take actions to strengthen
policies and improve practices that are driven by the best
available evidence and knowledge.
5. Identify critical research, evaluation, and data
collection needs.
Healthy People 2020 strives to:
1.
2.
3.
4.
5.

1. Attain high-quality, longer lives free of preventable


disease, disability, injury, and premature death.
2. Achieve health equity, eliminate disparities, and
improve the health of all groups.
3. Create social and physical environments that promote
good health for all.
4. Promote quality of life, healthy development, and
healthy behaviors across all life stages.

Text Mode Text version of the exam


1. Which is the primary goal of community health nursing?
E.

To support and supplement the efforts of the medical


profession in the promotion of health and prevention of illness
F.
To enhance the capacity of individuals, families and
communities to cope with their health needs
G.
To increase the productivity of the people by providing them
with services that will increase their level of health
H.
To contribute to national development through promotion of
family welfare, focusing particularly on mothers and children.
2. CHN is a community-based practice. Which best explains this statement?
E.
F.

The service is provided in the natural environment of people.


The nurse has to conduct community diagnosis to determine
nursing needs and problems.
G.
The services are based on the available resources within the
community.
H.
Priority setting is based on the magnitude of the health
problems identified.
3. Population-focused nursing practice requires which of the following
processes?
E.
F.
G.
H.

E.
F.
G.

Community organizing
Nursing process
Community diagnosis
Epidemiologic process
4. R.A. 1054 is also known as the Occupational Health Act. Aside from
number of employees, what other factor must be considered in determining
the occupational health privileges to which the workers will be entitled?
Type of occupation: agricultural, commercial, industrial
Location of the workplace in relation to health facilities
Classification of the business enterprise based on net profit

H.

Sex and age composition of employees


5. A business firm must employ an occupational health nurse when it has at
least how many employees?

E.
F.
G.
H.

21
101
201
301
6. When the occupational health nurse employs ergonomic principles, she is
performing which of her roles?

E.
F.
G.
H.

Health care provider


Health educator
Health care coordinator
Environmental manager
7. A garment factory does not have an occupational nurse. Who shall provide
the occupational health needs of the factory workers?

E.
F.
G.
H.

Occupational health nurse at the Provincial Health Office


Physician employed by the factory
Public health nurse of the RHU of their municipality
Rural sanitary inspector of the RHU of their municipality
8. Public health services are given free of charge. Is this statement true or
false?

E.

The statement is true; it is the responsibility of government to


provide basic services.
F.
The statement is false; people pay indirectly for public health
services.
G.
The statement may be true or false, depending on the specific
service required.
H.
The statement may be true or false, depending on policies of
the government concerned.
9. According to C.E.Winslow, which of the following is the goal of Public
Health?
E.
F.
G.
H.

For
For
For
For

people to attain their birthrights of health and longevity


promotion of health and prevention of disease
people to have access to basic health services
people to be organized in their health efforts

10. We say that a Filipino has attained longevity when he is able to reach the
average lifespan of Filipinos. What other statistic may be used to determine
attainment of longevity?
E.
F.
G.
H.

Age-specific mortality rate


Proportionate mortality rate
Swaroops index
Case fatality rate
11. Which of the following is the most prominent feature of public health
nursing?

E.

It involves providing home care to sick people who are not


confined in the hospital.
F.
Services are provided free of charge to people within the
catchment area.
G.
The public health nurse functions as part of a team providing a
public health nursing services.
H.
Public health nursing focuses on preventive, not curative,
services.
12. According to Margaret Shetland, the philosophy of public health nursing is
based on which of the following?
E.
F.

Health and longevity as birthrights


The mandate of the state to protect the birthrights of its
citizens
G.
Public health nursing as a specialized field of nursing
H.
The worth and dignity of man
13. Which of the following is the mission of the Department of Health?
E.
F.
G.
H.

Health for all Filipinos


Ensure the accessibility and quality of health care
Improve the general health status of the population
Health in the hands of the Filipino people by the year 2020
14. Region IV Hospital is classified as what level of facility?

E.
F.
G.
H.

Primary
Secondary
Intermediate
Tertiary
15. Which is true of primary facilities?

E.

They are usually government-run.

F.
G.
H.

Their services are provided on an out-patient basis.


They are training facilities for health professionals.
A community hospital is an example of this level of health
facilities.
16. Which is an example of the school nurses health care provider functions?

E.

Requesting for BCG from the RHU for school entrant


immunization
F.
Conducting random classroom inspection during a measles
epidemic
G.
Taking remedial action on an accident hazard in the school
playground
H.
Observing places in the school where pupils spend their free
time
17. When the nurse determines whether resources were maximized in
implementing Ligtas Tigdas, she is evaluating
E.
F.
G.
H.

Effectiveness
Efficiency
Adequacy
Appropriateness
18. You are a new B.S.N. graduate. You want to become a Public Health
Nurse. Where will you apply?

E.
F.
G.
H.

Department of Health
Provincial Health Office
Regional Health Office
Rural Health Unit
19. R.A. 7160 mandates devolution of basic services from the national
government to local government units. Which of the following is the major goal
of devolution?

E.
F.
G.
H.

To strengthen local government units


To allow greater autonomy to local government units
To empower the people and promote their self-reliance
To make basic services more accessible to the people
20. Who is the Chairman of the Municipal Health Board?

E.
F.
G.
H.

Mayor
Municipal Health Officer
Public Health Nurse
Any qualified physician

21. Which level of health facility is the usual point of entry of a client into the
health care delivery system?
E.
F.
G.
H.

Primary
Secondary
Intermediate
Tertiary
22. The public health nurse is the supervisor of rural health midwives. Which
of the following is a supervisory function of the public health nurse?

E.
F.
G.
H.

Referring cases or patients to the midwife


Providing technical guidance to the midwife
Providing nursing care to cases referred by the midwife
Formulating and implementing training programs for midwives
23. One of the participants in a hilot training class asked you to whom she
should refer a patient in labor who develops a complication. You will answer,
to the

E.
F.
G.
H.

Public Health Nurse


Rural Health Midwife
Municipal Health Officer
Any of these health professionals
24. You are the public health nurse in a municipality with a total population of
about 20,000. There are 3 rural health midwives among the RHU personnel.
How many more midwife items will the RHU need?

E.
F.
G.
H.

1
2
3
The RHU does not need any more midwife item.
25. If the RHU needs additional midwife items, you will submit the request for
additional midwife items for approval to the

E.
F.
G.
H.

Rural Health Unit


District Health Office
Provincial Health Office
Municipal Health Board
26. As an epidemiologist, the nurse is responsible for reporting cases of
notifiable diseases. What law mandates reporting of cases of notifiable
diseases?

E.
F.

Act 3573
R.A. 3753

G.
H.

R.A. 1054
R.A. 1082
27. According to Freeman and Heinrich, community health nursing is a
developmental service. Which of the following best illustrates this statement?

E.

The community health nurse continuously develops himself


personally and professionally.
F.
Health education and community organizing are necessary in
providing community health services.
G.
Community health nursing is intended primarily for health
promotion and prevention and treatment of disease.
H.
The goal of community health nursing is to provide nursing
services to people in their own places of residence.
28. Which disease was declared through Presidential Proclamation No. 4 as a
target for eradication in the Philippines?
E.
F.
G.
H.

Poliomyelitis
Measles
Rabies
Neonatal tetanus
29. The public health nurse is responsible for presenting the municipal health
statistics using graphs and tables. To compare the frequency of the leading
causes of mortality in the municipality, which graph will you prepare?

E.
F.
G.
H.

Line
Bar
Pie
Scatter diagram
30. Which step in community organizing involves training of potential leaders
in the community?

E.
F.
G.
H.

Integration
Community organization
Community study
Core group formation

Answers and Rationales


31.
Answer: (B) To enhance the capacity of individuals,
families and communities to cope with their health needs
32.
Answer: (B) The nurse has to conduct community
diagnosis to determine nursing needs and problems.
33.
Answer: (C) Community diagnosis. Population-focused
nursing care means providing care based on the greater need of the

majority of the population. The greater need is identified through


community diagnosis.
34.
Answer: (B) Location of the workplace in relation to
health facilities. Based on R.A. 1054, an occupational nurse must
be employed when there are 30 to 100 employees and the
workplace is more than 1 km. away from the nearest health center.
35.
Answer: (B) 101. Again, this is based on R.A. 1054.
36.
Answer: (D) Environmental manager. Ergonomics is
improving efficiency of workers by improving the workers
environment through appropriately designed furniture, for example.
37.
Answer: (C) Public health nurse of the RHU of their
municipality. Youre right! This question is based on R.A.1054.
38.
Answer: (B) The statement is false; people pay
indirectly for public health services. Community health
services, including public health services, are pre-paid services,
though taxation, for example.
39.
Answer: (A) For people to attain their birthrights of
health and longevity. According to Winslow, all public health
efforts are for people to realize their birthrights of health and
longevity.
40.
Answer: (C) Swaroops index. Swaroops index is the
percentage of the deaths aged 50 years or older. Its inverse
represents the percentage of untimely deaths (those who died
younger than 50 years).
41.
Answer: (D) Public health nursing focuses on
preventive, not curative, services.The catchment area in PHN
consists of a residential community, many of whom are well
individuals who have greater need for preventive rather than
curative services.
42.
Answer: (D) The worth and dignity of man. This is a direct
quote from Dr. Margaret Shetlands statements on Public Health
Nursing.
43.
Answer: (B) Ensure the accessibility and quality of
health care
44.
Answer: (D) Tertiary. Regional hospitals are tertiary facilities
because they serve as training hospitals for the region.
45.
Answer: (B) Their services are provided on an outpatient basis. Primary facilities government and non-government
facilities that provide basic out-patient services.
46.
Answer: (B) Conducting random classroom inspection
during a measles epidemic. Random classroom inspection is

assessment of pupils/students and teachers for signs of a health


problem prevalent in the community.
47.
Answer: (B) Efficiency. Efficiency is determining whether the
goals were attained at the least possible cost.
48.
Answer: (D) Rural Health Unit. R.A. 7160 devolved basic
health services to local government units (LGUs ). The public health
nurse is an employee of the LGU.
49.
Answer: (C) To empower the people and promote their
self-reliance. People empowerment is the basic motivation behind
devolution of basic services to LGUs.
50.
Answer: (A) Mayor. The local executive serves as the
chairman of the Municipal Health Board.
51.
Answer: (A) Primary. The entry of a person into the health
care delivery system is usually through a consultation in out-patient
services.
52.
Answer: (B) Providing technical guidance to the
midwife. The nurse provides technical guidance to the midwife in
the care of clients, particularly in the implementation of
management guidelines, as in Integrated Management of Childhood
Illness.
53.
Answer: (C) Municipal Health Officer. A public health nurse
and rural health midwife can provide care during normal childbirth.
A physician should attend to a woman with a complication during
labor.
54.
Answer: (A) 1. Each rural health midwife is given a population
assignment of about 5,000.
55.
Answer: (D) Municipal Health Board. As mandated by R.A.
7160, basic health services have been devolved from the national
government to local government units.
56.
Answer: (A) Act 3573. Act 3573, the Law on Reporting of
Communicable Diseases, enacted in 1929, mandated the reporting
of diseases listed in the law to the nearest health station.
57.
Answer: (B) Health education and community organizing
are necessary in providing community health services. The
community health nurse develops the health capability of people
through health education and community organizing activities.
58.
Answer: (B) Measles. Presidential Proclamation No. 4 is on
the Ligtas Tigdas Program.
59.
Answer: (B) Bar. A bar graph is used to present comparison
of values, a line graph for trends over time or age, a pie graph for

population composition or distribution, and a scatter diagram for


correlation of two variables.
60.
Answer: (D) Core group formation. In core group
formation, the nurse is able to transfer the technology of community
organizing to the potential or informal community leaders through a
training program.
1. Which is the primary goal of community health nursing?
A. To support and supplement the efforts of the medical profession in the promotion of
health and prevention of
B. To enhance the capacity of individuals, families and communities to cope with their health
needs
C. To increase the productivity of the people by providing them with services that will
increase their level of health
D. To contribute to national development through promotion of family welfare, focusing
particularly on mothers and children
2. CHN is a community-based practice. Which best explains this statement?
A. The service is provided in the natural environment of people
B. The nurse has to conduct community diagnosis to determine nursing needs and
problems
C. The service are based on the available resources within the community
D. Priority setting is based on the magnitude of the health problems identified
3. Population- focused nursing practice requires which of the following processes?
A. Community organizing .
B. Nursing, process
C. Community diagnosis
D. Epidemiologic process
4. RA 1054 is also known as the Occupational Health Act. Aside from the number of
employees, what other factor must be considered in determining the occupational
health privileges to which the workers will be entitled?
A. Type of occupation,: agriculture, commercial, industrial
B. Location of the workplace in relation to health facilities

C. Classification of the business enterprise based on net profit


D. Sex and age composition of employees
5. A business firm must employ an occupational health nurse when it has at least
how many employees.
A. 21
B. 101
C. 201
D. 301
6. When the occupational health nurse employs ergonomic principles, she is
performing which of her roles?
A. Health care provider
B. Health educator
C. Health care coordinator
D. Environment manager
7. A garment factory does not have an occupational nurse. Who shall provide the
occupational health needs of the factory workers?
A. Occupational health nurse at the Provincial Health Office
B. Physician employed by the factory
C. Public Health nurse of the RHU of their municipality
D. Rural Sanitary inspector of the RHU in their municipality
8. Public health services are given free of charge. Is this statement true or false?
A. The statement is true; it is the responsibility of government to provide haste services
B. The statement is false; people pay indirectly for public health services
C. The statement may be true or false; depending on the Specific service required
D. The statement may be true or false; depending on policies of the government concerned.
9. According to C.E. Winslow, which of the following is the goal of Public Health?
A. For people to attain their birthrights and longevity
B. For promotion of health and prevention and diseases
C. For people to have access to basic health services
D. For people to be organized in their health efforts

10. We say that a Filipino has attained longevity when he is able to reach the average
life span of Filipinos. What other statistic may be used to determine attainment of
longevity?
A. Age-specific mortality rate
B. Proportionate mortality rate
C. Swaroops index
D. Case fatality rate
11. Which of the following is the most prominent feature of public health nursing?
A. It involves providing home care to sick people who are not confined in the hospital
B. Services are provided free of charge to people within the catchment area
C. The public health nurse functions as part of a team providing a public health nursing
service
D. Public health nursing focuses on preventive, not curative services
12. According to Margaret Shetland, the philosophy of public health nursing is based
on which of the following?
A. Health and longevity as birthrights
B. The mandate of the state to protect the birthrights of its citizens
C. Public health nursing as a specialized field of nursing
D. The worth and dignity of man
13. Which of the following is the mission of the Department of Health?
A. Health for all Filipinos
B. Ensure the accessibility and quality of health
C. Improve the general health status of the population
D. Health in the hands of the Filipino people by the year 2020
14. Region IV Hospital is classified as what level of facility?
A. Primary
B. Secondary
C. Intermediate
D. Tertiary
15. What is true of primary facilities?

A. They are usually government-run


B. Their services are provided on an out-patient basis
C. They are training facilities for health professionals
D. A community hospital is an example of this level of health facilities
16. Which is an example of the school nurses health care provider function?
A. Requesting for BCG from the RHU for school entrance immunization
B. Conducting random classroom inspection during measles epidemic
C. Taking remedial action on an accident hazard in the school playground
D. Observing places in the school where pupils spend their free times
17. When the nurse determines whether resources were maximized in implementing
Ligtas Tigdas, she is evaluating:
A. Effectiveness
B. Efficiency
C. Adequacy
D. Appropriateness
18. You are a new B.S.N. graduate. You want to become a Public Health Nurse. Where
will you apply?
A. Department of Health
B. Provincial Health Office
C. Regional Health Office
D. Rural Health Unit
19. RA 7160 mandates devolution of basic services from the national government to
local government units. Which of the following is the major goal of devolution?
A. To strengthen local government units
B. To allow greater autonomy to local government units.
C. To empower the people and promote their self-reliance
D. To make basic services more accessible to the people
20. Who is the Chairman of the Municipal Health Board?
A. Mayor
B. Municipal Health Officer

C. Public Health Nurse


D. Any qualified physician
21. Which level of health facility is the usual point of entry of a client into the health
care delivery system?
A. Primary
B. Secondary
C. Intermediate
D. Tertiary
22. The public health nurse is the supervisor of rural health midwives. Which of the
following is a supervisory function of the pubic health nurse?
A. Referring cases or patients to the midwife
B. Providing technical guidance to the midwife
C. Proving nursing care to cases referred by the midwife
D. Formulating and implementing training programs for midwives
23. One of the participants in a hilot training class asked you to whom she should
refer a patient in labor who develops a complication. You will answer, to the;
A. Public health nurse
B. Rural health midwife
C. Municipal health officer
D. Any of these health professionals
24. You are the public health nurse in a municipality with a total population of about
20,000. There are3 health midwives among the RHU personnel. How many more
midwife items will the RHU need?
A. 1
B. 2
C. 3
D. 4
25. If the RHU needs additional midwife items, you will submit the request for
additional midwife items for approval to the:

A. Rural Health Unit


B. District Health Office
C. Provincial Health Office
D. Municipal Health Board
26. As an epidemiologist, the nurse is responsible for reporting cases or notifiable
diseases. What law mandates reporting cases of notifiable diseases?
A. Act 3573
B. RA.3753
C. RA 1054
D. RA 1082
27. According to Freeman and Heinrich, community health nursing is a
developmental service. Which of the following best illustrates this statement?
A. The community health nurse continuously develops himself personally and professionally
B. Health education and community organizing are necessary in providing community
health services
C. Community health nursing in intended primarily for health promotion and prevention and
treatment of disease.
D. The goal of community health nursing is to provide nursing services to people in their
own place of .residence
28. Which disease was declared through Presidential Proclamation No. 4 as a target
for, eradication in the Philippines?
A. Pioliomyelitis
B. Measles
C. Rabies
D. Neonatal Tetanus
29. The public health nurse is responsible for presenting the municipal health
statistics using graphs and tables. To compare the frequency of the leading causes
of mortality in the municipality, which graph will you prepare?
A. Line
B. Bar
C. Pie
D. Scatter diagram

30. Which step in community organizing involves training of potential leaders in the
community?
A. Integration
B. Community organization
C. Community study
D. Core group formation
31. In which step are plans formulated for solving community problems?
A. Mobilization
B. Community organization
C. Follow-up/extension
D. Core group formation
32. The public health nurse takes an active role in community participation. What is
the primary goal of community organizing?
A. To educate the people regarding community health problems
B. To mobilize the people to resolve community health problems
C. To maximize the communitys resources in dealing with health problems
33. An indicator of success in community organizing is when people are able to:
A. Participate in community activities for the solution of a community problem
B. Implement activities for the solution of the community problem
C. Plan activities for the solution of the community problem
D. Identify the health problem as a common concern
34. Tertiary prevention is needed in which stage of the natural history of disease?
A. Pre-pathogenesis
B. Pathogenesis
C. Predromal
D. Terminal
35. Isolation of a child with measles belongs to what level of prevention?
A. Primary
B. Secondary

C. Intermediate
D. Tertiary
36. On the other hand, Operation Timbang is_____ prevention?
A. Primary
B. Secondary
C. Intermediate
D. Tertiary
37. Which type of family-nurse contact will provide you with the best opportunity to
observe family dynamics?
A. Clinic consultation
B. Group conferences
C. Home visit
D. Written communication
38. The typology of family nursing problems is used in the statement of nursing
diagnosis in the care of families. The youngest child of the delos Reyes family has
been diagnosed as mentally retarded. This is classified as:
A. Health threat
B. Health deficit
C. Foreseeable crisis
D. Stress point
39. The delos Reyes couple have 6-year old child entering school for the first time.
The delos Reyes family has a:
A. Health threat
B. Health deficit
C. Foreseeable crisis
D. Stress point
40. Which of the following is an advantage of a home visit?
A. It allows the nurse to provide nursing care to a greater number of people
B. It provides an opportunity to do first hand appraisal of the home situation

C. It allows sharing of experience among people with similar health problems


D. It develops the familys initiative in providing for health needs of its members
41. Which is CONTRARY to the principles in planning a home visit?
A. A home visit should have a purpose of objective
B. The plan should revolve around the family health .needs
C. A home visit should be conducted in the manner prescribed by RHU
D. Planning of continuing care should involve a responsible-family member
42. The PHN bag is an important tool in providing nursing care during a home visit.
The most important principle in bag technique states that it;
A. Should save time and effort
B. Should minimize if not totally prevent the spread of infection
C. Should not overshadow concern for the patient and his family
D. May be done in variety of ways depending on the home situation, etc.
43. To maintain the cleanliness of the bag and its contents, which of the following
must the nurse do?
A. Wash his/her hands before and after providing nursing care to the family members
B. In the care of family members, as much as possible, use only articles taken from the bag
C. Put on an apron to protect her uniform and fold it with the right side out before putting it
back into the bag.
D. At the end of the visit, fold the lining on which the bag was placed, ensuring that the
contaminated side is on the outside.
44. The public health conducts a study on the factors contributing to the high
morality rate due to heart diseases in the municipality where she works. Which
branch of epidemiology does the nurse practice in this situation?
A. Descriptive
B. Analytical
C. Therapeutic
D. Evaluation
45. Which of the following is a function of epidemiology?

A. Identifying the disease condition based on manifestations presented by a client


B. Determining factors that contributed to the occurrence of pneumonia in a 3 year old
C. Determining the efficacy of the antibiotic used in the treatment of the 3 year old client with
pneumonia
D. Evaluating the effectiveness of the implementation of the Integrated Management of
Childhood Illness
46. Which of the following is an epidemiologic function of the nurse during an
epidemic?
A Conducting assessment of suspected cases to detect the communicable diseases
B. Monitoring the condition of the cases affected by the communicable disease
C. Participating in the investigation to determine the source of epidemic
D. Teaching the community on preventive measures against the disease
47. The primary purpose of conducting an epidemiologic investigation is to;
A. Delineate the etiology of the epidemic
B. Encourage cooperation and support of the community
C. Identify groups who are at risk of contracting the disease
D. Identify geographical location of cases of the disease in the community
48. Which is a characteristic of person-to-person propagated epidemic?
A. There are more cases of the disease than expected
B. The disease must necessarily be transmitted through a vector
C. The spread of the disease can be attributed to a common vehicle
D. There is gradual build up of cases before we epidemic becomes easily noticeable
49. In the investigation of an epidemic, you compare the present frequency of the
disease with the usual frequency at this time of the year in this community. This is
done during which stage of the investigation?
A. Establishing the epidemic
B. Testing the hypothesis
C. Formulation of the hypothesis
D. Appraisal of facts
50. The number of cases of Dengue fever usually increases towards the end of the
rainy season. This pattern of occurrence of Dengue fever is best described as;

A. Epidemic occurrence
B. Cyclical variation
C. Sporadic occurrence
D. Secular occurrence
51. In the year 1980, the World Health Organization declared the Philippines, together
with some other countries in the Western Pacific Region, free of which disease?
A. Pneumonic plaque
B. Poliomyelitis
C. Small pox
D. Anthrax
52. In the census of the Philippines in 1995, there were about 35,299,000 males and
about 34,968,000 females. What is the sex ratio?
A. 99.06:100
B. 100.94:100
C. 50.23%
D. 49.76%
53. Primary health care is a total approach to community development. Which of the
following is an indicator of success in the use of the primary health care approach?
A. Health services are provided free of charge to individuals and families
B. Local officials are empowered as the major decision makers in matters of health
C. Health workers are able too provide care based on identified health needs of the people
D. Health programs are sustained according to the level of development of the community
54. Sputum examination is the major screening tool for pulmonary tuberculosis.
Clients would sometimes get false negative results in this exam. This means that the
test is not perfect in terms of which characteristic of a diagnostic examination?
A. Effectiveness
B. Efficacy
C. Specificity
D. Sensitivity
55. Use of appropriate technology requires knowledge of indigenous technology.
Which medical herb is given for fever, headache and cough?

A. Sambong
B. Tsaang gubat
C. Akapulko
D. Lagundi
56. What law created the Philippine institute of Traditional and Alternative Health
Care?
A. RA 8483
B. RA4823
C. RA 2483
D. RA 3482
57. In traditional Chinese medicine, the yielding, negative and feminine force is
termed:
A. Yin
B. Yang
C. Qi
D. Chai
58. What is the legal basis of Primary Health Care approach in the Philippines?
A. Alma Ata Declaration of PHC
B. Letter of Instruction No 949
C. Presidential Decree No. 147
D. Presidential Decree 996
59. Which of the following demonstrates inter-sectoral linkages?
A. Two-way referral system
B. Team approach
C. Endorsement done by a midwife to another midwife
D. Cooperation between PHN and public school teacher
60. The municipality assigned to you has a population of about 20/000. Estimate the
number of 1-4 year old children who be given Retinol capsule 200.000 every 6
months.

A. 1,500
B. 1,800
C. 2,000
D. 2,300
61. Estimate the number of pregnant women who will be given tetanus toxoid during
an immunization outreach activity in a barangay with a population of about 1,500.
A. 265
B. 300
C. 375
D. 400
62. To describe the sex composition of the population, which demographic tool may
be used?
A. Sex ratio
B. Sex proportion
C. Population pyramid
D. Any of these maybe used
63. Which of the following is a natality rate?
A. Crude birth rate
B. Neonatal mortality rate
C. Infant mortality rate
D. General fertility rate
64. You are computing the crude rate of your municipality, with a total population o
about 18,000 for last year. There were 94 deaths. Among those who died, 20 died
because of diseases of the heart and 32 were aged 50 years or older. What is the
crude death rate?
A. 4.1/1000
B. 5.2/1000
C. 6.3/1000
D. 7.3/1000

65. Knowing that malnutrition is a frequent community health problem, you decided
to conduct nutritional assessment. What population is particularly susceptible to
protein energy malnutrition (PEM)?
A. Pregnant women and the elderly
B. Under 5 year old children
C. 1-4 year old children
D. School age children
66. Which statistic can give the most accurate reflection of the health status of a
community?
A. 1-4 year old age-specific mortality rate
B. Infant mortality rate
C. Swaroops index
D. Crude death rate
67. In the past year, Barangay A had an average population of 1655. 46 babies were
born in that year, 2 of whom died less than 4 weeks after they were born. They were 4
recorded stillbirths. What is the neonatal mortality rate?
A. 27.8/1000
B. 43.5/1000
C. 86.9/1000
D. 130.4/1000
68. Which statistic best reflects the nutritional status of a population?
A. 1-4 year old age-specific mortality rate
B. Proportionate mortality rate
C. Infant mortality rate
D. Swaroops index
69. What numerator is used in computing general fertility rate?
A. Estimated midyear population
B. Number of registered live births
C. Number of pregnancies in the year
D. Number of females of reproductive age

70. You will gather data for nutritional assessment of a purok. You will gather
information only from families with members who belong to the target population for
PEM. What method of delta gathering is best for this purpose?
A. Census
B. Survey
C. Record Review
D. Review of civil registry
71. In the conduct of a census, the method of population assignment based on the
actual physical location of the people is termed;
A. De jure
B. De locus
C. De facto
D. De novo
72. The Field Health Services and information System (FHSIS) is the recording and
reporting system in public health) care in the Philippines. The monthly field health
service activity report is a form used in which of the components of the FHSIS?
A. Tally report
B. Output report
C. Target/client list
D. Individual health record
73. To monitor clients registered in long-term regimens, such as the Multi-Drug
Therapy, which component will be most useful?
A. Tally report
B. Output report
C. Target/client list
D. Individual health record
74. Civil registries are important sources of data. Which law requires registration of
births within 30 days from the occurrence of the birth?
A. PD 651
B. Act 3573

C. RA 3753
D. RA 3375
75. Which of the following professionals can sign the birth certificate?
A. Public health nurse
B. Rural health midwife
C. Municipal health officer
D. Any of these health professionals
76. Which criterion in priority setting of health problems is used only in community
health care?
A. Modifiability of the problem
B. Nature of the problem presented
C. Magnitude of the health problem
D. Preventive potential of the health problem
77. The Sentrong Sigla Movement has been launched to improve health service
delivery. Which of the following is/are true of this movement?
A. This is a project spearheaded by local government units
B. It is a basis for increasing funding from local government units
C. It encourages health centers to focus on disease prevention and control
D. Its main strategy is certification of health centers able to comply with standards
78. Which of the following women should be considered as special targets for family
planning?
A. Those who have two children or more
B. Those with medical conditions such as anemia
C. Those younger than 20 years and older than 35 years
D. Those who just had a delivery within the past 15 months
79. Freedom of choice in one of the policies of the Family Planning Program of the
Philippines. Which of the following illustrates this principle?
A. Information dissemination about the need for family planning
B. Support of research and development in family planning methods
C. Adequate information for couples regarding the different methods
D. Encouragement of couples to take family planning as a joint responsibility

80. A woman, 6 months pregnant, came to the center for consultation. Which of the
following substances is contraindicated?
A. Tetanus toxoid
B. Retinol 200,000 IU
C. Ferrous sulfate 200mg
D. Potassium iodate 200 mg, capsule
81. During prenatal consultation, a client asked you if she can have her delivery at
home. After history taking and physical examination, you advised her against a home
delivery. Which of the following findings disqualifies her for a home delivery?
A. Her OB score is G5P3
B. She has some palmar pallor
C. Her blood pressure is 130/80
D. Her baby is in cephalic presentation
82. Inadequate intake by the pregnant woman of which vitamin may cause neural
tube defects?
A. Niacin
B. Riboflavin
C. Folic Acid
D. Thiamine
83. You are in a clients home to attend to a delivery. Which of the following will you
do first?
A. Set up a sterile area
B. Put on a clean gown and apron
C. Cleanse the clients vulva with soap and water
D. Note the interval, duration and intensity of labor and contractions
84. In preparing a primigravida for breastfeeding, which of the following will you do?
A. Tell her that lactation begins within a day after delivery
B. Teach her nipple stretching exercises if her nipples are everted
C. Instruct her to wash her nipples before and after each breastfeeding
D. Explain to her that putting the baby to breast will lessen blood loss after delivery

85. A primigravida is instructed to offer her breast to the baby for the first time within
30 minutes after delivery. What is the purpose of offering the breast this early?
A. To initiate the occurrence of milk letdown
B. To stimulate milk production by the mammary acini
C. To make sure that the baby is able to get the colustrum
D. To allow the woman to practice breastfeeding in the presence of the health worker
86. In a mothers class, you discuss proper breastfeeding technique. Which of these
is a sign that the baby has lactated on the breast property?
A. The baby takes shallow, rapid sucks
B. The mother does not feel nipple pain
C. The babys mouth is only partly open
D. Only the mothers nipple is inside the babys mouth
87. You explain to a breastfeeding mother that breastmilk is sufficient for all of the
babys nutrient needs only up to:
A. 3 months
B. 6 months
C. 1 year
D. 2 years
88. What is given to a woman within a month after the delivery of a baby?
A. Malunggay capsule
B. Ferrous sutfate l00mg O.D.
C. Retinol 200.000 IU 1 capsule
D. Potassium Iodate 200 mg, 1 capsule
89. Which biological used in EPI is stored in the freezer?
A. DPT
B. Tetanus toxoid
C. Measles vaccine
D. Hepatitis B vaccine
90. Unused BCG should be discarded how many hours after reconstitution?

A. 2
B. 4
C. 6
D. At the end of the day
91. In immunity school entrants with BCG, you not obliged to secure parental
consent. This is because of which legal document?
A. PD 996
B. RA 7864
C. Presidential Proclamation No. 6
D. Presidential Proclamation No. 46
92. Which immunization produces a permanent scar?
A. DPT
B. BCG
C. Measles vaccination
D. Hepatitis B vaccination
93. A 4 week old baby was brought to the health center for his first immunization.
Which can be given to him?
A. DPT1
B. OPV1
C. Infant BCG
D. Hepatitis B Vaccin
94. You will not give DPT 2 if the mother says that the infant had?
A. Seizures a day after DPT1
B. Fever for 3 days after DPT1
C. Abscess formation after DPT1
D. Local tenderness for 3 days after DPT1
95. A 2-month old infant was brought to the health center for immunization. During
assessment, the infants temperature registered at 38.1 C. Which is the best course
of action that you will take?

A. Go on with the infants immunization


B. Give paracetamol and wait for his fever to subside
C. Refer the infant to the physician for further assessment
D. Advise the infants mother to bring him back for immunization when he is well
96. A pregnant woman had just received her 4th dose of tetanus toxoid.
Subsequently, her baby will have protection against tetanus for how long?
A. 1 year
B. 3 years
C. 10 years
D. Lifetime
97. A 4-month old infant was brought to the health center of cough. Her respiratory
rate is 42/minute. Using the IMCI guidelines of assessment, her breathing is
considered;
A. Fast
B. Slow
C. Normal
D. Insignificant
98. Which of the following signs will indicate that a young child is suffering from
severe pneumonia?
A. Dyspnea
B. Wheezing
C. Fast breathing
D. Chest indrawing
99. Using IMCI guidelines, you classify a child as having severe pneumonia. What is
the best management for the child?
A. Prescribe antibiotic
B. Refer him urgently to the hospital
C. Instruct the mother to increase fluid intake
D. Instruct the mother to continue breastfeeding
100. A 5-month old infant was brought by his mother to the health center because of
diarrhea occurring 4 to 5 times a day. His skin goes back slowly after a skin pinch

and his eyes are sunken. Using the IMCI guidelines, you will classify this infant in
which category?
A. No signs of dehydration
B. Some dehydration
C. Severe dehydration
D. The data is insufficient
101. Based on the assessment, you classified a 3-month old infant with the chief
complaint of diarrhea in the category of SOME DEHYDRATION. Based on the IMCI
management guidelines, which of the following will you do?
A. Bring the infant to the nearest facility where IV fluids can be given
B. Supervise the mother in giving 200 to 400 ml of Oresol in 4 hours
C. Give the infants mother instructions on home management
D. Keep the infant in your health center for close observation
102. A mother is using Oresol in the management of diarrhea of her 3-year old child.
She asked you what to do if her child vomits. You will tell her to:
A. Bring the child to the nearest hospital for further assessment
B. Bring the child to the health center for IV therapy
C. Bring the child to the health center for assessment by the physician
D. Let the child rest for 10 minutes then continue giving Oresol more slowly
103. A 1 1/2 year old child was classified as having 3rd degree of protein energy
malnutrition, kwashjorkor. Which of the following signs will be most apparent in this
child?
A. Voracious appetite
B. Wasting
C. Apathy
D. Edema
104. Assessment of a 2-year old child revealed baggy pants. Using the IMCI
guidelines, how will you manage this child?
A. Refer the child urgently to a hospital for confinement
B. Coordinate with the social worker to enroll the child in a feeding program
C. Make a teaching plan for the mother, focusing on the menu planning for her child
D. Assess and treat the child for health problems like infections and intestinal parasitism

105. During the physical examination of a young child, what is the earliest sign of
xerophthalmia that may observe?
A. Keratomalacia
B. Corneal opacity
C. Night blindness
D. Conjunctival xerosis
106. To prevent xerophthalmia, young children are given Retinol capsule every 6
months. What is the dose given to preschoolers?
A. 10, 000 IU
B. 20, 000 IU
C. 100, 000 IU
D. 200, 000 IU
107. The major sign of iron deficiency anemia is pallor. What part is best examined
for pallor?
A. Palms
B. Nailbeds
C. Around the lips
D. Lower conjunctival sac
108. Food fortification is one of the strategies to prevent micronutrient deficiency
conditions. RA 8976 mandates fortification of certain food items. Which of the
following is among these food items?
A. Sugar
B. Bread
C. Margarine
D. Filled milk
109. What is the best course of action when there is a measles epidemic in a nearby
municipality?
A. Give measles vaccine to babies aged 6 to 3 months
B. Give babies aged 6 to 11 months one dose of 100,000 IU of Retinol
C. Instruct mother to keep their babies at home to prevent disease transmission
D. Instruct mothers to feed their babies adequately to enhance their babies resistance

110. A mother brought her daughter, 4 years old, to the RHU because of cough and
colds. Following the IMCI assessment guide, which of the following is a danger sign
that indicates the need for urgent referral to a hospital?
A. Inability to drink
B. High grade fever
C. Signs of severe dehydration
D. Cough for more than 30 days
111. Management of a child with measles includes the administration of which of the
following?
A. Gentian violet on mouth lesions
B. Antibiotic to prevent pneumonia
C. Tetracycline eye ointment for corneal opacity
D. Retinol capsule regardless of when the last dose was given
112. A mother brought her 10 month old infant for consultation because of fever
which started 4 days prior to consultation. To determine malaria risk, what will you
do?
A. Do a tourniquet test
B. Ask where the family resides
C. Get a specimen for blood smear
D. Ask if the fever is present everyday
113. The following are strategies implemented by the DOH to prevent mosquito-borne
diseases. Which of these is most effective in the control of Dengue fever?
A. Stream seeding with larva-eating fish
B. Destroying breeding places of mosquitoes
C. Chemoprophylaxis of non-immune persons going to endemic areas
D. Teaching people in endemic areas to use chemically treated mosquito nets
114. Secondary prevention for malaria includes?
A. Planting of neem or eucalyptus trees
B. Residual spraying of insecticides at night
C. Determining whether a place is endemic or not
D. Growing larva-eating fish in mosquito breeding places

115. Scotch tape swab is done to check for which intestinal parasite?
A. Ascaris
B. Pinworm
C. Hookworm
D. Schistosoma
116. Which of the following signs indicates the need for sputum examination for
AFB?
A. Hematemesis
B. Fever for 1 week
C. Cough for 3 weeks
D. Chest pain for 1 week
117. Which clients are considered targets for DOTS category?
A. Sputum negative cavitary cases
B. Clients returning after default
C. Relapses and failures of previous PTB treatment regimens
D. Clients diagnosed for the first time through a positive sputum exam
118. To improve compliance to treatment, what innovation is being implemented in
DOTS?
A. Having the health worker follow up the client at home
B. Having the health worker or a responsible family member monitor drug intake
C. Having the patient come to the health center every month to get his medications
D. Having a target list to check on whether the patient has collected his monthly supply of
drugs
119. Diagnosis of leprosy is highly dependent on recognition of symptoms. Which of
the following is an early sign of leprosy?
A. Macular lesions
B. Inability to close eyelids
C. Thickened painful nerves
D. Sinking of the nose bridge

120. Which of the following clients should be classified as a case of mutibacillary


leprosy?
A. 3 skin lesions, negative slit skin smear
B. 3 skin lesions, positive slit skin smear
C. 5 skin lesions, negative slit skin smear
D. 5 skin lesions, positive slit skin smear
121. In the Philippines, which condition is the most frequent cause of death
associated by schistosomiasis?
A. Liver cancer
B. Liver cirrhosis
C. Bladder cancer
D. Intestinal perforation
122. What is the most effective way of controlling schistosomiasis in an endemic
area?
A. Use of molluscicides
B. Building of foot bridges
C. Proper use of sanitary toilets
D. Use of protective footwear, such as rubber boots
123. When residents obtain water from an artesian well in the neighborhood, the level
of this approved type of water facility is:
A. I
B. II
C. III
D. IV
124. For prevention of Hepatitis A, you decided to conduct health education
activities. Which of the following is Irrelevant?
A. Use of sterile syringes and needles
B. Safe food preparation and food handling by vendors
C. Proper disposal of human excreta and personal hygiene
D. Immediate reporting of water pipe leaks and illegal water connections

125. Which biological used in EPI should not be stored in the freezer?
A. DPT
B. OPV
C. Measles vaccine
D. MMR
126. You will conduct outreach immunization in a barangay with a population of
about 1500. Estimate the number of infants in the barangay.
A. 45
B. 50
C. 55
D. 60
127. In IMCI, severe conditions generally require urgent referral to a hospital. Which
of the following severe conditions Does not always require urgent referral to
hospital?
A. Mastoiditis
B. Severe dehydration
C. Severe pneumonia
D. Severe febrile disease.
128. A client was diagnosed as having Dengue Fever. You will say that there is slow
capillary refill when the color of the nailbed that you pressed does not return within
how many seconds?
A. 3
B. 5
C. 8
D. 10
129. A 3-year old child was brought by his mother to the health center because of
fever of 4-day duration. The child had a positive tourniquet test result. In the absence
of other signs, which of the most appropriate measure that the PHN may carry out to
prevent Dengue shock syndrome?
A. Insert an NGT and give fluids per NGT
B. Instruct the mother to give the child Oresol

C. Start the patient on IV Stat


D. Refer the client to the physician for appropriate management
130. The pathognomonic sign of measles is Kopliks spot. You may see Kopliks spot
by inspecting the:
A. Nasal Mucosa
B. Buccal mucosa
C. Skin on the abdomen
D. Skin on the antecubital surface
131. Among the following diseases, which is airborne?
A. Viral conjunctivitis
B. Acute poliomyelitis
C. Diphtheria
D. Measles
132. Among children aged 2 months to 3 years, the most prevalent form of meningitis
is caused by which microorganism?
A. Hemophilus Influenzae
B. Morbillivirus
C. Streptococcus Pneumoniae
D. Neisseria meningitides
133. Human beings are the major reservoir of malaria. Which of the following
strategies in malaria control is based on this fact?
A. Stream seeding
B. Stream clearing
C. Destruction of breeding places
D. Zooprophylaxis
134. The use of larvivorous fish in malaria control is the basis for which strategy of
malaria control?
A. Stream seeding
B. Stream clearing

C. Destruction of breeding places


D. Zooprophylaxis .
135. Mosquito-borne diseases are prevented mostly with the use of mosquito control
measures. Which of the following is NOT appropriate for malaria control?
A. Use of chemically treated mosquito nets
B. Seeding of breeding places with larva-eating fish
C. Destruction of breeding places of the mosquito vector
D. Use of mosquito-repelling soaps, such as those with basil or citronella
136. A 4-year old client was brought to the health center with chief complaint of
severe diarrhea and the passage of rice water. The client is most probably suffering
from which condition?
A. Giardiasis
B. Cholera
C. Amebiasis
D. Dysentery
137. In the Philippines, which specie of schistosoma is endemic in certain regions?
A. S. mansoni
B. S. japonicum
C. S. malayensis
D. S. haematobium
138. A 32 year old client came for consultation at the health center with the chief
complaint of fever for a week. Accompanying symptoms were muscle pains and body
malaise. A week after the start of fever, the client noted yellowish discoloration of his
sclera. History showed that he waded in flood waters about 2 weeks before the onset
of symptoms. Based on this history/ which disease condition will you suspect?
A. Hepatitis A
B. Hepatitis B
C. Tetanus
D. Leptospirosis
139. MWSS provides water to Manila and other cities in Metro Manila. This is an
example of which level of water facility?

A. I
B. II
C. III
D. IV
140. You are the PHN in the city health center. A client underwent screening for AIDS
using ELISA. His result was positive. What is the best course of action that you may
take?
A. Get a thorough history of the client, focusing on the practice of high risk behavior
B. Ask the client to be accompanied by a significant person before revealing the result.
C. Refer the client to the physician since he is the best person to reveal the result to the
client
D. Refer the client for a supplementary test, such as Western blot, since the ELISA result
maybe false
141. Which is the BEST control measure for AIDS?
A. Being faithful to a single sexual partner
B. Using a condom during each sexual contact
C. Avoiding sexual contact with commercial sex workers
D. Making sure that ones sexual partner does not have signs of AIDS
142. The most frequent causes of death among clients with AIDS are opportunistic
diseases. Which of the following opportunistic infections is characterized by
tonsilllopharyngitis?
A. Respiratory candidiasis
B. Infectious mononucleosis
C. Cytomegalovirus disease
D. Pneumocystis carinii pneumonia
143. To determine the possible sources of sexually transmitted infections, which is
the BEST method that may be undertaken by the public health nurse?
A. Contact tracing
B. Community survey
C. Mass screening tests
D. Interview suspects

144. Antiretroviral agents, such as AZT are used in the management of AIDS. Which
of the following is not an action expected of these drugs?
A. They prolong the life of the client with AIDS
B. They reduce the risk of opportunistic infections
C. They shorten the period of communicability of the disease
D. They are able to bring about a cure of the disease condition
145. A barangay had an outbreak of German measles. To prevent congenital rubella,
what is the BEST advice that you can give to women in the first trimester of
pregnancy in the barangay?
a. Advice them on the sign of German Measles
b. Avoid crowded places, such as markets and moviehouses
c. Consult at the health center where rubella vaccine may be given
d. Consult a physician who may give them rubella immunoglobulin

Answers and Rationale


Gauge your performance by counter-checking your answers to those below. If you have
disputes or further questions, please direct them to the comments section.
1. Answer: (B) To enhance the capacity of individuals, families and communities to cope
with their health needs.
To contribute to national development through promotion of family welfare, focusing
particularly on mothers and children.
2. Answer: (B) The nurse has to conduct community diagnosis to determine nursing needs
and problems.
Community-based practice means providing care to people in their own natural
environments: the home, school and workplace, for example.
3. Answer: (C) Community diagnosis
Population-focused nursing care means providing care based on the greater need of the
majority of the population. The greater need is identified through community diagnosis.
4. Answer: (B) Location of the workplace in relation to health facilities
Based on R.A. 1054, an occupational nurse must be employed when there are 30 to 100
employees and the workplace is more than 1 km. away from the nearest health center.
5. Answer: (B) 101

6. Answer: (D) Environmental manager


Ergonomics is improving efficiency of workers by improving the workers environment
through appropriately designed furniture, for example.
7. Answer: (C) Public health nurse of the RHU of their municipality
8. Answer: (B) The statement is false; people pay indirectly for public health services.
Community health services, including public health services, are prepaid paid services,
through taxation, for example.
9. Answer: (A) For people to attain their birthrights of health and longevity
According to Winslow, all public health efforts are for people to realize their birthrights of
health and longevity.
10. Answer: (C) Swaroops index
Swaroops index is the percentage of the deaths aged 50 years or older. Its inverse
represents the percentage of untimely deaths (those who died younger than 50 years).
11. Answer: (D) Public health nursing focuses on preventive, not curative, services.
The catchment area in PHN consists of a residential community, many of whom are well
individuals who have greater need for preventive rather than curative services.
12. Answer: (D) The worth and dignity of man
This is a direct quote from Dr. Margaret Shetlands statements on Public Health Nursing.
13. Answer: (B) Ensure the accessibility and quality of health care
14. Answer: (D) Tertiary
Regional hospitals are tertiary facilities because they serve as training hospitals for the
region.
15. Answer: (B) Their services are provided on an out-patient basis.
Primary facilities government and non-government facilities that provide basic out-patient
services.
16. Answer: (B) Conducting random classroom inspection during a measles epidemic
Random classroom inspection is assessment of pupils/students and teachers for signs of a
health problem prevalent in the community.

17. Answer: (B) Efficiency


Efficiency is determining whether the goals were attained at the least possible cost.
18. Answer: (D) Rural Health Unit
R.A. 7160 devolved basic health services to local government units (LGUs). The public
health nurse is an employee of the LGU.
19. Answer: (C) To empower the people and promote their self-reliance
People empowerment is the basic motivation behind devolution of basic services to LGUs.
20. Answer: (A) Mayor
The local executive serves as the chairman of the Municipal Health Board.
21. Answer: (A) Primary
The entry of a person into the health care delivery system is usually through a consultation
in out-patient services.
22. Answer: (B) Providing technical guidance to the midwife
The nurse provides technical guidance to the midwife in the care of clients, particularly in
the implementation of management guidelines, as in Integrated Management of Childhood
Illness.
23. Answer: (C) Municipal Health Officer
A public health nurse and rural health midwife can provide care during normal childbirth. A
physician should attend to a woman with a complication during labor.
24. Answer: (A) 1
Each rural health midwife is given a population assignment of about 5,000.
25. Answer: (D) Municipal Health Board
As mandated by R.A. 7160, basic health services have been devolved from the national
government to local government units.
26. Answer: (A) Act 3573
Act 3573, the Law on Reporting of Communicable Diseases, enacted in 1929, mandated
the reporting of diseases listed in the law to the nearest health station.
27. Answer: (B) Health education and community organizing are necessary in providing
community health services.

The community health nurse develops the health capability of people through health
education and community organizing activities.
28. Answer: (B) Measles
Presidential Proclamation No. 4 is on the Ligtas Tigdas Program.
29. Answer: (B) Bar
A bar graph is used to present comparison of values, a line graph for trends over time or
age, a pie graph for population composition or distribution, and a scatter diagram for
correlation of two variables.
30. Answer: (D) Core group formation
In core group formation, the nurse is able to transfer the technology of community
organizing to the potential or informal community leaders through a training program.
31. Answer: (B) Community organization
Community organization is the step when community assemblies take place. During the
community assembly, the people may opt to formalize the community organization and
make plans for community action to resolve a community health problem.
32. Answer: (D) To maximize the communitys resources in dealing with health problems
Community organizing is a developmental service, with the goal of developing the peoples
self-reliance in dealing with community health problems. A, B and C are objectives of
contributory objectives to this goal.
33. Answer: (A) Participate in community activities for the solution of a community problem
Participation in community activities in resolving a community problem may be in any of the
processes mentioned in the other choices.
34. Answer: (D) Terminal
Tertiary prevention involves rehabilitation, prevention of permanent disability and disability
limitation appropriate for convalescents, the disabled, complicated cases and the terminally
ill (those in the terminal stage of a disease)
35. Answer: (A) Primary
The purpose of isolating a client with a communicable disease is to protect those who are
not sick (specific disease prevention).

36. Answer: (B) Secondary


Operation Timbang is done to identify members of the susceptible population who are
malnourished. Its purpose is early diagnosis and, subsequently, prompt treatment.
37. Answer: (C) Home visit
Dynamics of family relationships can best be observed in the familys natural environment,
which is the home.
38. Answer: (B) Health deficit
Failure of a family member to develop according to what is expected, as in mental
retardation, is a health deficit.
39. Answer: (C) Foreseeable crisis
Entry of the 6-year old into school is an anticipated period of unusual demand on the family.
40. Answer: (B) It provides an opportunity to do first hand appraisal of the home situation.
Choice A is not correct since a home visit requires that the nurse spend so much time with
the family. Choice C is an advantage of a group conference, while choice D is true of a clinic
consultation.
41. Answer: (C) A home visit should be conducted in the manner prescribed by the RHU.
The home visit plan should be flexible and practical, depending on factors, such as the
familys needs and the resources available to the nurse and the family.
42. Answer: (B) Should minimize if not totally prevent the spread of infection.
Bag technique is performed before and after handling a client in the home to prevent
transmission of infection to and from the client.
43. Answer: (A) Wash his/her hands before and after providing nursing care to the family
members.
Choice B goes against the idea of utilizing the familys resources, which is encouraged in
CHN. Choices C and D goes against the principle of asepsis of confining the contaminated
surface of objects.
44. Answer: (B) Analytical
Analytical epidemiology is the study of factors or determinants affecting the patterns of
occurrence and distribution of disease in a community.
45. Answer: (D) Evaluating the effectiveness of the implementation of the Integrated
Management of Childhood Illness

Epidemiology is used in the assessment of a community or evaluation of interventions in


community health practice.
46. Answer: (C) Participating in the investigation to determine the source of the epidemic
Epidemiology is the study of patterns of occurrence and distribution of disease in the
community, as well as the factors that affect disease patterns. The purpose of an
epidemiologic investigation is to identify the source of an epidemic, i.e., what brought about
the epidemic.
47. Answer: (A) Delineate the etiology of the epidemic
Delineating the etiology of an epidemic is identifying its source.
48. Answer: (D) There is a gradual build up of cases before the epidemic becomes easily
noticeable.
A gradual or insidious onset of the epidemic is usually observable in person-to-person
propagated epidemics.
49. Answer: (A) Establishing the epidemic
Establishing the epidemic is determining whether there is an epidemic or not. This is done
by comparing the present number of cases with the usual number of cases of the disease at
the same time of the year, as well as establishing the relatedness of the cases of the
disease.
50. Answer: (B) Cyclical variation
A cyclical variation is a periodic fluctuation in the number of cases of a disease in the
community.
51. Answer: (C) Smallpox
The last documented case of Smallpox was in 1977 at Somalia.
52. Answer: (B) 100.94:100
Sex ratio is the number of males for every 100 females in the population.
53. Answer: (D) Health programs are sustained according to the level of development of the
community.
Primary health care is essential health care that can be sustained in all stages of
development of the community.
54. Answer: (D) Sensitivity
Sensitivity is the capacity of a diagnostic examination to detect cases of the disease. If a

test is 100% sensitive, all the cases tested will have a positive result, i.e., there will be no
false negative results.
55. Answer: (D) Lagundi
Sambong is used as a diuretic. Tsaang gubat is used to relieve diarrhea. Akapulko is used
for its antifungal property.
56. Answer: (A) R.A. 8423 or AN ACT CREATING THE PHILIPPINE INSTITUTE OF
TRADITIONAL AND ALTERNATIVE HEALTH CARE (PITAHC) TO ACCELERATE THE
DEVELOPMENT OF TRADITIONAL AND ALTERNATIVE HEALTH CARE IN THE
PHILIPPINES, PROVIDING FOR A TRADITIONAL AND ALTERNATIVE HEALTH CARE
DEVELOPMENT FUND AND FOR OTHER PURPOSES signed to a law on December 9,
1997.
57. Answer: (A) Yin
Yang is the male dominating, positive and masculine force.
58. Answer: (B) Letter of Instruction No. 949
Letter of Instruction 949 was issued by then President Ferdinand Marcos, directing the
formerly called Ministry of Health, now the Department of Health, to utilize Primary Health
Care approach in planning and implementing health programs.
59. Answer: (D) Cooperation between the PHN and public school teacher
Intersectoral linkages refer to working relationships between the health sector and other
sectors involved in community development.
60. Answer: (D) 2,300
Based on the Philippine population composition, to estimate the number of 1-4 year old
children, multiply total population by 11.5%.
61. Answer: (A) 265
To estimate the number of pregnant women, multiply the total population by 3.5%.
62. Answer: (D) Any of these may be used.
Sex ratio and sex proportion are used to determine the sex composition of a population. A
population pyramid is used to present the composition of a population by age and sex.
63. Answer: (A) Crude birth rate
Natality means birth. A natality rate is a birth rate.

64. Answer: (B) 5.2/1,000


To compute crude death rate divide total number of deaths (94) by total population (18,000)
and multiply by 1,000.
65. Answer: (C) 1-4 year old children
Preschoolers are the most susceptible to PEM because they have generally been weaned.
Also, this is the population who, unable to feed themselves, are often the victims of poor
intrafamilial food distribution.
66. Answer: (C) Swaroops index
Swaroops index is the proportion of deaths aged 50 years and above. The higher the
Swaroops index of a population, the greater the proportion of the deaths who were able to
reach the age of at least 50 years, i.e., more people grew old before they died.
67. Answer: (B) 43.5/1,000
To compute for neonatal mortality rate, divide the number of babies who died before
reaching the age of 28 days by the total number of live births, then multiply by 1,000.
68. Answer: (A) 1-4 year old age-specific mortality rate
Since preschoolers are the most susceptible to the effects of malnutrition, a population with
poor nutritional status will most likely have a high 1-4 year old age-specific mortality rate,
also known as child mortality rate.
69. Answer: (B) Number of registered live births
To compute for general or total fertility rate, divide the number of registered live births by the
number of females of reproductive age (15-45 years), then multiply by 1,000.
70. Answer: (B) Survey
A survey, also called sample survey, is data gathering about a sample of the population.
71. Answer: (C) De facto
The other method of population assignment, de jure, is based on the usual place of
residence of the people.
72. Answer: (A) Tally report
A tally report is prepared monthly or quarterly by the RHU personnel and transmitted to the
Provincial Health Office.

73. Answer: (C) Target/client list


The MDT Client List is a record of clients enrolled in MDT and other relevant data, such as
dates when clients collected their monthly supply of drugs.
74. Answer: (A) P.D. 651
P.D. 651 amended R.A. 3753, requiring the registry of births within 30 days from their
occurrence.
75. Answer: (D) Any of these health professionals
R.A. 3753 states that any birth attendant may sign the certificate of live birth.
76. Answer: (C) Magnitude of the health problem
Magnitude of the problem refers to the percentage of the population affected by a health
problem. The other choices are criteria considered in both family and community health
care.
77. Answer: (D) Its main strategy is certification of health centers able to comply with
standards.
Sentrong Sigla Movement is a joint project of the DOH and local government units. Its main
strategy is certification of health centers that are able to comply with standards set by the
DOH.
78. Answer: (D) Those who just had a delivery within the past 15 months
The ideal birth spacing is at least two years. 15 months plus 9 months of pregnancy = 2
years.
79. Answer: (C) Adequate information for couples regarding the different methods
To enable the couple to choose freely among different methods of family planning, they
must be given full information regarding the different methods that are available to them,
considering the availability of quality services that can support their choice.
80. Answer: (B) Retinol 200,000 IU
Retinol 200,000 IU is a form of megadose Vitamin A. This may have a teratogenic effect.
81. Answer: (A) Her OB score is G5P3.
Only women with less than 5 pregnancies are qualified for a home delivery. It is also
advisable for a primigravida to have delivery at a childbirth facility.

82. Answer: (C) Folic acid


It is estimated that the incidence of neural tube defects can be reduced drastically if
pregnant women have an adequate intake of folic acid.
83. Answer: (D) Note the interval, duration and intensity of labor contractions.
Assessment of the woman should be done first to determine whether she is having true
labor and, if so, what stage of labor she is in.
84. Answer: (D) Explain to her that putting the baby to breast will lessen blood loss after
delivery.
Suckling of the nipple stimulates the release of oxytocin by the posterior pituitary gland,
which causes uterine contraction. Lactation begins 1 to 3 days after delivery. Nipple
stretching exercises are done when the nipples are flat or inverted. Frequent washing dries
up the nipples, making them prone to the formation of fissures.
85. Answer: (B) To stimulate milk production by the mammary acini
Suckling of the nipple stimulates prolactin reflex (the release of prolactin by the anterior
pituitary gland), which initiates lactation.
86. Answer: (B) The mother does not feel nipple pain.
When the baby has properly latched on to the breast, he takes deep, slow sucks; his mouth
is wide open; and much of the areola is inside his mouth. And, youre right! The mother
does not feel nipple pain.
87. Answer: (B) 6 months
After 6 months, the babys nutrient needs, especially the babys iron requirement, can no
longer be provided by mothers milk alone.
88. Answer: (C) Retinol 200,000 I.U., 1 capsule
A capsule of Retinol 200,000 IU is given within 1 month after delivery. Potassium iodate is
given during pregnancy; malunggay capsule is not routinely administered after delivery; and
ferrous sulfate is taken for two months after delivery.
89. Answer: (C) Measles vaccine
Among the biologicals used in the Expanded Program on Immunization, measles vaccine
and OPV are highly sensitive to heat, requiring storage in the freezer.
90. Answer: (B) 4
While the unused portion of other biologicals in EPI may be given until the end of the day,

only BCG is discarded 4 hours after reconstitution. This is why BCG immunization is
scheduled only in the morning.
91. Answer: (A) P.D. 996
Presidential Decree 996, enacted in 1976, made immunization in the EPI compulsory for
children under 8 years of age. Hepatitis B vaccination was made compulsory for the same
age group by R.A. 7846.
92. Answer: (B) BCG
BCG causes the formation of a superficial abscess, which begins 2 weeks after
immunization. The abscess heals without treatment, with the formation of a permanent scar.
93. Answer: (C) Infant BCG
Infant BCG may be given at birth. All the other immunizations mentioned can be given at 6
weeks of age.
94. Answer: (A) Seizures a day after DPT 1.
Seizures within 3 days after administration of DPT is an indication of hypersensitivity to
pertussis vaccine, a component of DPT. This is considered a specific contraindication to
subsequent doses of DPT.
95. Answer: (A) Go on with the infants immunizations.
In the EPI, fever up to 38.5C is not a contraindication to immunization. Mild acute
respiratory tract infection, simple diarrhea and malnutrition are not contraindications either.
96. Answer: (A) 1 year
The baby will have passive natural immunity by placental transfer of antibodies. The mother
will have active artificial immunity lasting for about 10 years. 5 doses will give the mother
lifetime protection.
97. Answer: (C) Normal
In IMCI, a respiratory rate of 50/minute or more is fast breathing for an infant aged 2 to 12
months.
98. Answer: (D) Chest indrawing
In IMCI, chest indrawing is used as the positive sign of dyspnea, indicating severe
pneumonia.

99. Answer: (B) Refer him urgently to the hospital.


Severe pneumonia requires urgent referral to a hospital. Answers A, C and D are done for a
client classified as having pneumonia.
100. Answer: (B) Some dehydration
Using the assessment guidelines of IMCI, a child (2 months to 5 years old) with diarrhea is
classified as having SOME DEHYDRATION if he shows 2 or more of the following signs:
restless or irritable, sunken eyes, the skin goes back slow after a skin pinch.
101. Answer: (B) Supervise the mother in giving 200 to 400 ml. of Oresol in 4 hours.
In the IMCI management guidelines, SOME DEHYDRATION is treated with the
administration of Oresol within a period of 4 hours. The amount of Oresol is best computed
on the basis of the childs weight (75 ml/kg body weight). If the weight is unknown, the
amount of Oresol is based on the childs age.
102. Answer: (D) Let the child rest for 10 minutes then continue giving Oresol more slowly.
If the child vomits persistently, that is, he vomits everything that he takes in, he has to be
referred urgently to a hospital. Otherwise, vomiting is managed by letting the child rest for
10 minutes and then continuing with Oresol administration. Teach the mother to give Oresol
more slowly.
103. Answer: (D) Edema
Edema, a major sign of kwashiorkor, is caused by decreased colloidal osmotic pressure of
the blood brought about by hypoalbuminemia. Decreased blood albumin level is due a
protein-deficient diet.
104. Answer: (A) Refer the child urgently to a hospital for confinement.
Baggy pants is a sign of severe marasmus. The best management is urgent referral to a
hospital.
105. Answer: (D) Conjunctival xerosis
The earliest sign of Vitamin A deficiency (xerophthalmia) is night blindness. However, this is
a functional change, which is not observable during physical examination.The earliest
visible lesion is conjunctival xerosis or dullness of the conjunctiva due to inadequate tear
production.
106. Answer: (D) 200,000 IU
Preschoolers are given Retinol 200,000 IU every 6 months. 100,000 IU is given once to
infants aged 6 to 12 months. The dose for pregnant women is 10,000 IU.

107. Answer: (A) Palms


The anatomic characteristics of the palms allow a reliable and convenient basis for
examination for pallor.
108. Answer: (A) Sugar
R.A. 8976 mandates fortification of rice, wheat flour, sugar and cooking oil with Vitamin A,
iron and/or iodine.
109. Answer: (A) Give measles vaccine to babies aged 6 to 8 months.
Ordinarily, measles vaccine is given at 9 months of age. During an impending epidemic,
however, one dose may be given to babies aged 6 to 8 months. The mother is instructed
that the baby needs another dose when the baby is 9 months old.
110. Answer: (A) Inability to drink
A sick child aged 2 months to 5 years must be referred urgently to a hospital if he/she has
one or more of the following signs: not able to feed or drink, vomits everything, convulsions,
abnormally sleepy or difficult to awaken.
111. Answer: (D) Retinol capsule regardless of when the last dose was given
An infant 6 to 12 months classified as a case of measles is given Retinol 100,000 IU; a child
is given 200,000 IU regardless of when the last dose was given.
112. Answer: (B) Ask where the family resides.
Because malaria is endemic, the first question to determine malaria risk is where the clients
family resides. If the area of residence is not a known endemic area, ask if the child had
traveled within the past 6 months, where he/she was brought and whether he/she stayed
overnight in that area.
113. Answer: (B) Destroying breeding places of mosquitoes
Aedes aegypti, the vector of Dengue fever, breeds in stagnant, clear water. Its feeding time
is usually during the daytime. It has a cyclical pattern of occurrence, unlike malaria which is
endemic in certain parts of the country.
114. Answer: (C) Determining whether a place is endemic or not
This is diagnostic and therefore secondary level prevention. The other choices are for
primary prevention.
115. Answer: (B) Pinworm
Pinworm ova are deposited around the anal orifice.

116. Answer: (C) Cough for 3 weeks


A client is considered a PTB suspect when he has cough for 2 weeks or more, plus one or
more of the following signs: fever for 1 month or more; chest pain lasting for 2 weeks or
more not attributed to other conditions; progressive, unexplained weight loss; night sweats;
and hemoptysis.
117. Answer: (D) Clients diagnosed for the first time through a positive sputum exam
Category I is for new clients diagnosed by sputum examination and clients diagnosed to
have a serious form of extrapulmonary tuberculosis, such as TB osteomyelitis.
118. Answer: (B) Having the health worker or a responsible family member monitor drug
intake
Directly Observed Treatment Short Course is so-called because a treatment partner,
preferably a health worker accessible to the client, monitors the clients compliance to the
treatment.
119. Answer: (C) Thickened painful nerves
The lesion of leprosy is not macular. It is characterized by a change in skin color (either
reddish or whitish) and loss of sensation, sweating and hair growth over the lesion. Inability
to close the eyelids (lagophthalmos) and sinking of the nosebridge are late symptoms.
120. Answer: (D) 5 skin lesions, positive slit skin smear
A multibacillary leprosy case is one who has a positive slit skin smear and at least 5 skin
lesions.
121. Answer: (B) Liver cirrhosis
The etiologic agent of schistosomiasis in the Philippines is Schistosoma japonicum, which
affects the small intestine and the liver. Liver damage is a consequence of fibrotic reactions
to schistosoma eggs in the liver.
122. Answer: (C) Proper use of sanitary toilets
The ova of the parasite get out of the human body together with feces. Cutting the cycle at
this stage is the most effective way of preventing the spread of the disease to susceptible
hosts.
123. Answer: (B) II
A communal faucet or water standpost is classified as Level II.

124. Answer: (A) Use of sterile syringes and needles


Hepatitis A is transmitted through the fecal oral route. Hepatitis B is transmitted through
infected body secretions like blood and semen.
125. Answer: (A) DPT
DPT is sensitive to freezing. The appropriate storage temperature of DPT is 2 to 8 C only.
OPV and measles vaccine are highly sensitive to heat and require freezing. MMR is not an
immunization in the Expanded Program on Immunization.
126. Answer: (A) 45
To estimate the number of infants, multiply total population by 3%.
127. Answer: (B) Severe dehydration
The order of priority in the management of severe dehydration is as follows: intravenous
fluid therapy, referral to a facility where IV fluids can be initiated within 30 minutes,
Oresol/nasogastric tube, Oresol/orem. When the foregoing measures are not possible or
effective, tehn urgent referral to the hospital is done.
128. Answer: (A) 3
Adequate blood supply to the area allows the return of the color of the nailbed within 3
seconds.
129. Answer: (B) Instruct the mother to give the child Oresol.
Since the child does not manifest any other danger sign, maintenance of fluid balance and
replacement of fluid loss may be done by giving the client Oresol.
130. Answer: (B) Buccal mucosa
Kopliks spot may be seen on the mucosa of the mouth or the throat.
131. Answer: (D) Measles
Viral conjunctivitis is transmitted by direct or indirect contact with discharges from infected
eyes. Acute poliomyelitis is spread through the fecal-oral route and contact with throat
secretions, whereas diphtheria is through direct and indirect contact with respiratory
secretions.
132. Answer: (A) Hemophilus influenzae
Hemophilus meningitis is unusual over the age of 5 years. In developing countries, the peak
incidence is in children less than 6 months of age. Morbillivirus is the etiology of measles.
Streptococcus pneumoniae and Neisseria meningitidis may cause meningitis, but age
distribution is not specific in young children.

133. Answer: (D) Zooprophylaxis


Zooprophylaxis is done by putting animals like cattle or dogs close to windows or doorways
just before nightfall. The Anopheles mosquito takes his blood meal from the animal and
goes back to its breeding place, thereby preventing infection of humans.
134. Answer: (A) Stream seeding
Stream seeding is done by putting tilapia fry in streams or other bodies of water identified as
breeding places of the Anopheles mosquito.
135. Answer: (C) Destruction of breeding places of the mosquito vector
Anopheles mosquitoes breed in slow-moving, clear water, such as mountain streams.
136. Answer: (B) Cholera
Passage of profuse watery stools is the major symptom of cholera. Both amebic and
bacillary dysentery are characterized by the presence of blood and/or mucus in the stools.
Giardiasis is characterized by fat malabsorption and, therefore, steatorrhea.
137. Answer: (B) S. japonicum
S. mansoni is found mostly in Africa and South America; S. haematobium in Africa and the
Middle East; and S. malayensis only in peninsular Malaysia.
138. Answer: (D) Leptospirosis
Leptospirosis is transmitted through contact with the skin or mucous membrane with water
or moist soil contaminated with urine of infected animals, like rats.
139. Answer: (C) III
Waterworks systems, such as MWSS, are classified as level III.
140. Answer: (D) Refer the client for a supplementary test, such as Western blot, since the
ELISA result may be false.
A client having a reactive ELISA result must undergo a more specific test, such as Western
blot. A negative supplementary test result means that the ELISA result was false and that,
most probably, the client is not infected.
141. Answer: (A) Being faithful to a single sexual partner
Sexual fidelity rules out the possibility of getting the disease by sexual contact with another
infected person. Transmission occurs mostly through sexual intercourse and exposure to
blood or tissues.

142. Answer: (B) Infectious mononucleosis


Cytomegalovirus disease is an acute viral disease characterized by fever, sore throat and
lymphadenopathy.
143. Answer: (A) Contact tracing
Contact tracing is the most practical and reliable method of finding possible sources of
person-to-person transmitted infections, such as sexually transmitted diseases.
144. Answer: (D) They are able to bring about a cure of the disease condition.
There is no known treatment for AIDS. Antiretroviral agents reduce the risk of opportunistic
infections and prolong life, but does not cure the underlying immunodeficiency.
145. Answer: (D) Consult a physician who may give them rubella immunoglobulin.
Rubella vaccine is made up of attenuated German measles viruses. This is contraindicated
in pregnancy. Immune globulin, a specific prophylactic against German measles, may be
given to pregnant women.

1. In which step are plans formulated for solving community problems?


E.
F.
G.
H.

Mobilization
Community organization
Follow-up/extension
Core group formation
2. The public health nurse takes an active role in community participation.
What is the primary goal of community organizing?

E.
F.
G.

To educate the people regarding community health problems


To mobilize the people to resolve community health problems
To maximize the communitys resources in dealing with health
problems
H.
To maximize the communitys resources in dealing with health
problems
3. An indicator of success in community organizing is when people are able to
E.

Participate in community activities for the solution of a


community problem
F.
Implement activities for the solution of the community problem
G.
Plan activities for the solution of the community problem
H.
Identify the health problem as a common concern
4. Tertiary prevention is needed in which stage of the natural history of
disease?

E.
F.
G.
H.

Pre-pathogenesis
Pathogenesis
Prodromal
Terminal
5. Isolation of a child with measles belongs to what level of prevention?

E.
F.
G.
H.

Primary
Secondary
Intermediate
Tertiary
6. On the other hand, Operation Timbang is _____ prevention.

E.
F.
G.
H.

Primary
Secondary
Intermediate
Tertiary
7. Which type of family-nurse contact will provide you with the best opportunity
to observe family dynamics?

E.
F.
G.
H.

Clinic consultation
Group conference
Home visit
Written communication
8. The typology of family nursing problems is used in the statement of nursing
diagnosis in the care of families. The youngest child of the de los Reyes family
has been diagnosed as mentally retarded. This is classified as a:

E.
F.
G.
H.

Health threat
Health deficit
Foreseeable crisis
Stress point
9. The de los Reyes couple have a 6-year old child entering school for the first
time. The de los Reyes family has a:

E.
F.
G.
H.

Health threat
Health deficit
Foreseeable crisis
Stress point
10. Which of the following is an advantage of a home visit?

E.

It allows the nurse to provide nursing care to a greater number


of people.

F.

It provides an opportunity to do first hand appraisal of the


home situation.
G.
It allows sharing of experiences among people with similar
health problems.
H.
It develops the familys initiative in providing for health needs
of its members.
11. Which is CONTRARY to the principles in planning a home visit?
E.
F.
G.

A home visit should have a purpose or objective.


The plan should revolve around family health needs.
A home visit should be conducted in the manner prescribed by
the RHU.
H.
Planning of continuing care should involve a responsible family
member.
12. The PHN bag is an important tool in providing nursing care during a home
visit. The most important principle of bag technique states that it
E.
F.
G.
H.

Should save time and effort.


Should minimize if not totally prevent the spread of infection.
Should not overshadow concern for the patient and his family.
May be done in a variety of ways depending on the home
situation, etc.
13. To maintain the cleanliness of the bag and its contents, which of the
following must the nurse do?

E.

Wash his/her hands before and after providing nursing care to


the family members.
F.
In the care of family members, as much as possible, use only
articles taken from the bag.
G.
Put on an apron to protect her uniform and fold it with the right
side out before putting it back into the bag.
H.
At the end of the visit, fold the lining on which the bag was
placed, ensuring that the contaminated side is on the outside.
14. The public health nurse conducts a study on the factors contributing to the
high mortality rate due to heart disease in the municipality where she works.
Which branch of epidemiology does the nurse practice in this situation?
E.
F.
G.
H.

Descriptive
Analytical
Therapeutic
Evaluation
15. Which of the following is a function of epidemiology?

E.

Identifying the disease condition based on manifestations


presented by a client
F.
Determining factors that contributed to the occurrence of
pneumonia in a 3 year old
G.
Determining the efficacy of the antibiotic used in the treatment
of the 3 year old client with pneumonia
H.
Evaluating the effectiveness of the implementation of the
Integrated Management of Childhood Illness
16. Which of the following is an epidemiologic function of the nurse during an
epidemic?
E.

Conducting assessment of suspected cases to detect the


communicable disease
F.
Monitoring the condition of the cases affected by the
communicable disease
G.
Participating in the investigation to determine the source of the
epidemic
H.
Teaching the community on preventive measures against the
disease
17. The primary purpose of conducting an epidemiologic investigation is to
E.
F.
G.
H.

Delineate the etiology of the epidemic


Encourage cooperation and support of the community
Identify groups who are at risk of contracting the disease
Identify geographical location of cases of the disease in the
community
18. Which is a characteristic of person-to-person propagated epidemics?

E.
F.
G.

There are more cases of the disease than expected.


The disease must necessarily be transmitted through a vector.
The spread of the disease can be attributed to a common
vehicle.
H.
There is a gradual build up of cases before the epidemic
becomes easily noticeable.
19. In the investigation of an epidemic, you compare the present frequency of
the disease with the usual frequency at this time of the year in this community.
This is done during which stage of the investigation?
E.
F.
G.
H.

Establishing the epidemic


Testing the hypothesis
Formulation of the hypothesis
Appraisal of facts

20. The number of cases of Dengue fever usually increases towards the end
of the rainy season. This pattern of occurrence of Dengue fever is best
described as
E.
F.
G.
H.

Epidemic occurrence
Cyclical variation
Sporadic occurrence
Secular variation
21. In the year 1980, the World Health Organization declared the Philippines,
together with some other countries in the Western Pacific Region, free of
which disease?

E.
F.
G.
H.

Pneumonic plague
Poliomyelitis
Small pox
Anthrax
22. In the census of the Philippines in 1995, there were about 35,299,000
males and about 34,968,000 females. What is the sex ratio?

E.
F.
G.
H.

99.06:100
100.94:100
50.23%
49.76%
23. Primary health care is a total approach to community development. Which
of the following is an indicator of success in the use of the primary health care
approach?

E.

Health services are provided free of charge to individuals and


families.
F.
Local officials are empowered as the major decision makers in
matters of health.
G.
Health workers are able to provide care based on identified
health needs of the people.
H.
Health programs are sustained according to the level of
development of the community.
24. Sputum examination is the major screening tool for pulmonary
tuberculosis. Clients would sometimes get false negative results in this exam.
This means that the test is not perfect in terms of which characteristic of a
diagnostic examination?
E.
F.

Effectiveness
Efficacy

G.
H.

Specificity
Sensitivity
25. Use of appropriate technology requires knowledge of indigenous
technology. Which medicinal herb is given for fever, headache and cough?

E.
F.
G.
H.

Sambong
Tsaang gubat
Akapulko
Lagundi
26. What law created the Philippine Institute of Traditional and Alternative
Health Care?

E.
F.
G.
H.

R.A. 8423
R.A. 4823
R.A. 2483
R.A. 3482
27. In traditional Chinese medicine, the yielding, negative and feminine force
is termed

E.
F.
G.
H.

Yin
Yang
Qi
Chai
28. What is the legal basis for Primary Health Care approach in the
Philippines?

E.
F.
G.
H.

Alma Ata Declaration on PHC


Letter of Instruction No. 949
Presidential Decree No. 147
Presidential Decree 996
29. Which of the following demonstrates intersectoral linkages?

E.
F.
G.
H.

Two-way referral system


Team approach
Endorsement done by a midwife to another midwife
Cooperation between the PHN and public school teacher
30. The municipality assigned to you has a population of about 20,000.
Estimate the number of 1-4 year old children who will be given Retinol capsule
200,000 I.U. every 6 months.

E.
F.
G.

1,500
1,800
2,000

H.

2,300

Answers and Rationales


31.
Answer: (B) Community organization. Community
organization is the step when community assemblies take place.
During the community assembly, the people may opt to formalize
the community organization and make plans for community action
to resolve a community health problem.
32.
Answer: (D) To maximize the communitys resources in
dealing with health problems. Community organizing is a
developmental service, with the goal of developing the peoples selfreliance in dealing with community health problems. A, B and C are
objectives of contributory objectives to this goal.
33.
Answer: (A) Participate in community activities for the
solution of a community problem. Participation in community
activities in resolving a community problem may be in any of the
processes mentioned in the other choices.
34.
Answer: (D) Terminal. Tertiary prevention involves
rehabilitation, prevention of permanent disability and disability
limitation appropriate for convalescents, the disabled, complicated
cases and the terminally ill (those in the terminal stage of a disease)
35.
Answer: (A) Primary. The purpose of isolating a client with a
communicable disease is to protect those who are not sick (specific
disease prevention).
36.
Answer: (B) Secondary. Operation Timbang is done to
identify members of the susceptible population who are
malnourished. Its purpose is early diagnosis and, subsequently,
prompt treatment.
37.
Answer: (C) Home visit. Dynamics of family relationships
can best be observed in the familys natural environment, which is
the home.
38.
Answer: (B) Health deficit. Failure of a family member to
develop according to what is expected, as in mental retardation, is a
health deficit.
39.
Answer: (C) Foreseeable crisis. Entry of the 6-year old into
school is an anticipated period of unusual demand on the family.
40.
Answer: (B) It provides an opportunity to do first hand
appraisal of the home situation.. Choice A is not correct since a
home visit requires that the nurse spend so much time with the
family. Choice C is an advantage of a group conference, while choice
D is true of a clinic consultation.

41.
Answer: (C) A home visit should be conducted in the
manner prescribed by the RHU.The home visit plan should be
flexible and practical, depending on factors, such as the familys
needs and the resources available to the nurse and the family.
42.
Answer: (B) Should minimize if not totally prevent the
spread of infection. Bag technique is performed before and after
handling a client in the home to prevent transmission of infection to
and from the client.
43.
Answer: (A) Wash his/her hands before and after
providing nursing care to the family members. Choice B goes
against the idea of utilizing the familys resources, which is
encouraged in CHN. Choices C and D goes against the principle of
asepsis of confining the contaminated surface of objects.
44.
Answer: (B) Analytical. Analytical epidemiology is the study
of factors or determinants affecting the patterns of occurrence and
distribution of disease in a community.
45.
Answer: (D) Evaluating the effectiveness of the
implementation of the Integrated Management of Childhood
Illness. Epidemiology is used in the assessment of a community or
evaluation of interventions in community health practice.
46.
Answer: (C) Participating in the investigation to
determine the source of the epidemic. Epidemiology is the
study of patterns of occurrence and distribution of disease in the
community, as well as the factors that affect disease patterns. The
purpose of an epidemiologic investigation is to identify the source of
an epidemic, i.e., what brought about the epidemic.
47.
Answer: (A) Delineate the etiology of the
epidemic. Delineating the etiology of an epidemic is identifying its
source.
48.
Answer: (D) There is a gradual build up of cases before
the epidemic becomes easily noticeable. A gradual or insidious
onset of the epidemic is usually observable in person-to-person
propagated epidemics.
49.
Answer: (A) Establishing the epidemic. Establishing the
epidemic is determining whether there is an epidemic or not. This is
done by comparing the present number of cases with the usual
number of cases of the disease at the same time of the year, as well
as establishing the relatedness of the cases of the disease.
50.
Answer: (B) Cyclical variation. A cyclical variation is a
periodic fluctuation in the number of cases of a disease in the
community.

51.
Answer: (C) Small pox. The last documented case of Small
pox was in 1977 at Somalia.
52.
Answer: (B) 100.94:100. Sex ratio is the number of males
for every 100 females in the population.
53.
Answer: (D) Health programs are sustained according to
the level of development of the community. Primary health
care is essential health care that can be sustained in all stages of
development of the community.
54.
Answer: (D) Sensitivity. Sensitivity is the capacity of a
diagnostic examination to detect cases of the disease. If a test is
100% sensitive, all the cases tested will have a positive result, i.e.,
there will be no false negative results.
55.
Answer: (D) Lagundi. Sambong is used as a diuretic. Tsaang
gubat is used to relieve diarrhea. Akapulko is used for its antifungal
property.
56.
Answer: (A) R.A. 8423
57.
Answer: (A) Yin. Yang is the male dominating, positive and
masculine force.
58.
Answer: (B) Letter of Instruction No. 949. Letter of
Instruction 949 was issued by then President Ferdinand Marcos,
directing the formerly called Ministry of Health, now the Department
of Health, to utilize Primary Health Care approach in planning and
implementing health programs.
59.
Answer: (D) Cooperation between the PHN and public
school teacher. Intersectoral linkages refer to working
relationships between the health sector and other sectors involved
in community development.
60.
Answer: (D) 2,300. Based on the Philippine population
composition, to estimate the number of 1-4 year old children,
multiply total population by 11.5%.
1. Estimate the number of pregnant women who will be given tetanus toxoid
during an immunization outreach activity in a barangay with a population of
about 1,500.
E.
F.
G.
H.

265
300
375
400
2. To describe the sex composition of the population, which demographic tool
may be used?

E.
F.
G.
H.

Sex ratio
Sex proportion
Population pyramid
Any of these may be used.
3. Which of the following is a natality rate?

E.
F.
G.
H.

Crude birth rate


Neonatal mortality rate
Infant mortality rate
General fertility rate
4. You are computing the crude death rate of your municipality, with a total
population of about 18,000, for last year. There were 94 deaths. Among those
who died, 20 died because of diseases of the heart and 32 were aged 50
years or older. What is the crude death rate?

E.
F.
G.
H.

4.2/1,000
5.2/1,000
6.3/1,000
7.3/1,000
5. Knowing that malnutrition is a frequent community health problem, you
decided to conduct nutritional assessment. What population is particularly
susceptible to protein energy malnutrition (PEM)?

E.
F.
G.
H.

Pregnant women and the elderly


Under-5 year old children
1-4 year old children
School age children
6. Which statistic can give the most accurate reflection of the health status of
a community?

E.
F.
G.
H.

1-4 year old age-specific mortality rate


Infant mortality rate
Swaroops index
Crude death rate
7. In the past year, Barangay A had an average population of 1655. 46 babies
were born in that year, 2 of whom died less than 4 weeks after they were born.
There were 4 recorded stillbirths. What is the neonatal mortality rate?

E.
F.
G.
H.

27.8/1,000
43.5/1,000
86.9/1,000
130.4/1,000

8. Which statistic best reflects the nutritional status of a population?


E.
F.
G.
H.

1-4 year old age-specific mortality rate


Proportionate mortality rate
Infant mortality rate
Swaroops index
9. What numerator is used in computing general fertility rate?

E.
F.
G.
H.

Estimated midyear population


Number of registered live births
Number of pregnancies in the year
Number of females of reproductive age
10. You will gather data for nutritional assessment of a purok. You will gather
information only from families with members who belong to the target
population for PEM. What method of data gathering is best for this purpose?

E.
F.
G.
H.

Census
Survey
Record review
Review of civil registry
11. In the conduct of a census, the method of population assignment based on
the actual physical location of the people is termed

E.
F.
G.
H.

De jure
De locus
De facto
De novo
12. The Field Health Services and Information System (FHSIS) is the
recording and reporting system in public health care in the Philippines. The
Monthly Field Health Service Activity Report is a form used in which of the
components of the FHSIS?

E.
F.
G.
H.

Tally report
Output report
Target/client list
Individual health record
13. To monitor clients registered in long-term regimens, such as the Multi-Drug
Therapy, which component will be most useful?

E.
F.
G.
H.

Tally report
Output report
Target/client list
Individual health record

14. Civil registries are important sources of data. Which law requires
registration of births within 30 days from the occurrence of the birth?
E.
F.
G.
H.

P.D. 651
Act 3573
R.A. 3753
R.A. 3375
15. Which of the following professionals can sign the birth certificate?

E.
F.
G.
H.

Public health nurse


Rural health midwife
Municipal health officer
Any of these health professionals
16. Which criterion in priority setting of health problems is used only in
community health care?

E.
F.
G.
H.

Modifiability of the problem


Nature of the problem presented
Magnitude of the health problem
Preventive potential of the health problem
17. The Sentrong Sigla Movement has been launched to improve health
service delivery. Which of the following is/are true of this movement?

E.
F.
G.

This is a project spearheaded by local government units.


It is a basis for increasing funding from local government units.
It encourages health centers to focus on disease prevention
and control.
H.
Its main strategy is certification of health centers able to
comply with standards.
18. Which of the following women should be considered as special targets for
family planning?
E.
F.
G.
H.

Those who have two children or more


Those with medical conditions such as anemia
Those younger than 20 years and older than 35 years
Those who just had a delivery within the past 15 months
19. Freedom of choice is one of the policies of the Family Planning Program
of the Philippines. Which of the following illustrates this principle?

E.
F.

Information dissemination about the need for family planning


Support of research and development in family planning
methods

G.

Adequate information for couples regarding the different


methods
H.
Encouragement of couples to take family planning as a joint
responsibility
20. A woman, 6 months pregnant, came to the center for consultation. Which
of the following substances is contraindicated?
E.
F.
G.
H.

Tetanus toxoid
Retinol 200,000 IU
Ferrous sulfate 200 mg
Potassium iodate 200 mg. capsule
21. During prenatal consultation, a client asked you if she can have her
delivery at home. After history taking and physical examination, you advised
her against a home delivery. Which of the following findings disqualifies her for
a home delivery?

E.
F.
G.
H.

Her OB score is G5P3.


She has some palmar pallor.
Her blood pressure is 130/80.
Her baby is in cephalic presentation.
22. Inadequate intake by the pregnant woman of which vitamin may cause
neural tube defects?

E.
F.
G.
H.

Niacin
Riboflavin
Folic acid
Thiamine
23. You are in a clients home to attend to a delivery. Which of the following
will you do first?

E.
F.
G.
H.

Set up the sterile area.


Put on a clean gown or apron.
Cleanse the clients vulva with soap and water.
Note the interval, duration and intensity of labor contractions.
24. In preparing a primigravida for breastfeeding, which of the following will
you do?

E.
F.
G.

Tell her that lactation begins within a day after delivery.


Teach her nipple stretching exercises if her nipples are everted.
Instruct her to wash her nipples before and after each
breastfeeding.

H.

Explain to her that putting the baby to breast will lessen blood
loss after delivery.
25. A primigravida is instructed to offer her breast to the baby for the first time
within 30 minutes after delivery. What is the purpose of offering the breast this
early?

E.
F.
G.
H.

To initiate the occurrence of milk letdown


To stimulate milk production by the mammary acini
To make sure that the baby is able to get the colostrum
To allow the woman to practice breastfeeding in the presence
of the health worker
26. In a mothers class, you discuss proper breastfeeding technique. Which is
of these is a sign that the baby has latched on to the breast properly?

E.
F.
G.
H.

The baby takes shallow, rapid sucks.


The mother does not feel nipple pain.
The babys mouth is only partly open.
Only the mothers nipple is inside the babys mouth.
27. You explain to a breastfeeding mother that breast milk is sufficient for all of
the babys nutrient needs only up to ____.

E.
F.
G.
H.

3 months
6 months
1 year
2 years
28. What is given to a woman within a month after the delivery of a baby?

E.
F.
G.
H.

Malunggay capsule
Ferrous sulfate 100 mg. OD
Retinol 200,000 I.U., 1 capsule
Potassium iodate 200 mg, 1 capsule
29. Which biological used in Expanded Program on Immunization (EPI) is
stored in the freezer?

E.
F.
G.
H.

DPT
Tetanus toxoid
Measles vaccine
Hepatitis B vaccine
30. Unused BCG should be discarded how many hours after reconstitution?

E.
F.

2
4

G.
H.

6
At the end of the day

Answers and Rationales


31.
Answer: (A) 265. To estimate the number of pregnant
women, multiply the total population by 3.5%.
32.
Answer: (D) Any of these may be used. Sex ratio and sex
proportion are used to determine the sex composition of a
population. A population pyramid is used to present the composition
of a population by age and sex.
33.
Answer: (A) Crude birth rate. Natality means birth. A
natality rate is a birth rate.
34.
Answer: (B) 5.2/1,000. To compute crude death rate divide
total number of deaths (94) by total population (18,000) and
multiply by 1,000.
35.
Answer: (C) 1-4 year old children. Preschoolers are the
most susceptible to PEM because they have generally been weaned.
Also, this is the population who, unable to feed themselves, are
often the victims of poor intrafamilial food distribution.
36.
Answer: (C) Swaroops index. Swaroops index is the
proportion of deaths aged 50 years and above. The higher the
Swaroops index of a population, the greater the proportion of the
deaths who were able to reach the age of at least 50 years, i.e.,
more people grew old before they died.
37.
Answer: (B) 43.5/1,000. To compute for neonatal mortality
rate, divide the number of babies who died before reaching the age
of 28 days by the total number of live births, then multiply by 1,000.
38.
Answer: (A) 1-4 year old age-specific mortality
rate. Since preschoolers are the most susceptible to the effects of
malnutrition, a population with poor nutritional status will most likely
have a high 1-4 year old age-specific mortality rate, also known as
child mortality rate.
39.
Answer: (B) Number of registered live births. To compute
for general or total fertility rate, divide the number of registered live
births by the number of females of reproductive age (15-45 years),
then multiply by 1,000.
40.
Answer: (B) Survey. A survey, also called sample survey, is
data gathering about a sample of the population.
41.
Answer: (C) De facto. The other method of population
assignment, de jure, is based on the usual place of residence of the
people.

42.
Answer: (A) Tally report. A tally report is prepared monthly
or quarterly by the RHU personnel and transmitted to the Provincial
Health Office.
43.
Answer: (C) Target/client list. The MDT Client List is a
record of clients enrolled in MDT and other relevant data, such as
dates when clients collected their monthly supply of drugs.
44.
Answer: (A) P.D. 651. P.D. 651 amended R.A. 3753, requiring
the registry of births within 30 days from their occurrence.
45.
Answer: (D) Any of these health professionals. D. R.A.
3753 states that any birth attendant may sign the certificate of live
birth.
46.
Answer: (C) Magnitude of the health problem. Magnitude
of the problem refers to the percentage of the population affected
by a health problem. The other choices are criteria considered in
both family and community health care.
47.
Answer: (D) Its main strategy is certification of health
centers able to comply with standards. Sentrong Sigla
Movement is a joint project of the DOH and local government units.
Its main strategy is certification of health centers that are able to
comply with standards set by the DOH.
48.
Answer: (D) Those who just had a delivery within the
past 15 months. The ideal birth spacing is at least two years. 15
months plus 9 months of pregnancy = 2 years.
49.
Answer: (C) Adequate information for couples regarding
the different methods. To enable the couple to choose freely
among different methods of family planning, they must be given full
information regarding the different methods that are available to
them, considering the availability of quality services that can
support their choice.
50.
Answer: (B) Retinol 200,000 IU. Retinol 200,000 IU is a
form of megadose Vitamin A. This may have a teratogenic effect.
51.
Answer: (A) Her OB score is G5P3. Only women with less
than 5 pregnancies are qualified for a home delivery. It is also
advisable for a primigravida to have delivery at a childbirth facility.
52.
Answer: (C) Folic acid. It is estimated that the incidence of
neural tube defects can be reduced drastically if pregnant women
have an adequate intake of folic acid.
53.
Answer: (D) Note the interval, duration and intensity of
labor contractions.. Assessment of the woman should be done
first to determine whether she is having true labor and, if so, what
stage of labor she is in.

54.
Answer: (D) Explain to her that putting the baby to
breast will lessen blood loss after delivery. Suckling of the
nipple stimulates the release of oxytocin by the posterior pituitary
gland, which causes uterine contraction. Lactation begins 1 to 3
days after delivery. Nipple stretching exercises are done when the
nipples are flat or inverted. Frequent washing dries up the nipples,
making them prone to the formation of fissures.
55.
Answer: (B) To stimulate milk production by the
mammary acini. Suckling of the nipple stimulates prolactin reflex
(the release of prolactin by the anterior pituitary gland), which
initiates lactation.
56.
Answer: (B) The mother does not feel nipple pain.. When
the baby has properly latched on to the breast, he takes deep, slow
sucks; his mouth is wide open; and much of the areola is inside his
mouth. And, youre right! The mother does not feel nipple pain.
57.
Answer: (B) 6 months. After 6 months, the babys nutrient
needs, especially the babys iron requirement, can no longer be
provided by mothers milk alone.
58.
Answer: (C) Retinol 200,000 I.U., 1 capsule. A capsule of
Retinol 200,000 IU is given within 1 month after delivery. Potassium
iodate is given during pregnancy; malunggay capsule is not
routinely administered after delivery; and ferrous sulfate is taken for
two months after delivery.
59.
Answer: (C) Measles vaccine. Among the biologicals used in
the Expanded Program on Immunization, measles vaccine and OPV
are highly sensitive to heat, requiring storage in the freezer.
60.
Answer: (B) 4. While the unused portion of other biologicals
in EPI may be given until the end of the day, only BCG is discarded 4
hours after reconstitution. This is why BCG immunization is
scheduled only in the morning.
1. In immunizing school entrants with BCG, you are not obliged to secure
parental consent. This is because of which legal document?
E.
F.
G.
H.
E.

P.D. 996
R.A. 7846
Presidential Proclamation No. 6
Presidential Proclamation No. 46
2. Which immunization produces a permanent scar?
DPT

F.
G.
H.

BCG
Measles vaccination
Hepatitis B vaccination
3. A 4-week old baby was brought to the health center for his first
immunization. Which can be given to him?

E.
F.
G.
H.

DPT1
OPV1
Infant BCG
Hepatitis B vaccine 1
4. You will not give DPT 2 if the mother says that the infant had

E.
F.
G.
H.

Seizures a day after DPT 1.


Fever for 3 days after DPT 1.
Abscess formation after DPT 1.
Local tenderness for 3 days after DPT 1.
5. A 2-month old infant was brought to the health center for immunization.
During assessment, the infants temperature registered at 38.1C. Which is
the best course of action that you will take?

E.
F.
G.
H.

Go on with the infants immunizations.


Give Paracetamol and wait for his fever to subside.
Refer the infant to the physician for further assessment.
Advise the infants mother to bring him back for immunization
when he is well.
6. A pregnant woman had just received her 4th dose of tetanus toxoid.
Subsequently, her baby will have protection against tetanus for how long?

E.
F.
G.
H.

1 year
3 years
10 years
Lifetime
7. A 4-month old infant was brought to the health center because of cough.
Her respiratory rate is 42/minute. Using the Integrated Management of Child
Illness (IMCI) guidelines of assessment, her breathing is considered

E.
F.
G.
H.

Fast
Slow
Normal
Insignificant
8. Which of the following signs will indicate that a young child is suffering from
severe pneumonia?

E.
F.
G.
H.

Dyspnea
Wheezing
Fast breathing
Chest indrawing
9. Using IMCI guidelines, you classify a child as having severe pneumonia.
What is the best management for the child?

E.
F.
G.
H.

Prescribe an antibiotic.
Refer him urgently to the hospital.
Instruct the mother to increase fluid intake.
Instruct the mother to continue breastfeeding.
10. A 5-month old infant was brought by his mother to the health center
because of diarrhea occurring 4 to 5 times a day. His skin goes back slowly
after a skin pinch and his eyes are sunken. Using the IMCI guidelines, you will
classify this infant in which category?

E.
F.
G.
H.

No signs of dehydration
Some dehydration
Severe dehydration
The data is insufficient.
11. Based on assessment, you classified a 3-month old infant with the chief
complaint of diarrhea in the category of SOME DEHYDRATION. Based on
IMCI management guidelines, which of the following will you do?

E.

Bring the infant to the nearest facility where IV fluids can be


given.
F.
Supervise the mother in giving 200 to 400 ml. of Oresol in 4
hours.
G.
Give the infants mother instructions on home management.
H.
Keep the infant in your health center for close observation.
12. A mother is using Oresol in the management of diarrhea of her 3-year old
child. She asked you what to do if her child vomits. You will tell her to
E.
F.

Bring the child to the nearest hospital for further assessment.


Bring the child to the health center for intravenous fluid
therapy.
G.
Bring the child to the health center for assessment by the
physician.
H.
Let the child rest for 10 minutes then continue giving Oresol
more slowly.

13. A 1 year old child was classified as having 3rd degree protein energy
malnutrition, kwashiorkor. Which of the following signs will be most apparent
in this child?
E.
F.
G.
H.

Voracious appetite
Wasting
Apathy
Edema
14. Assessment of a 2-year old child revealed baggy pants. Using the IMCI
guidelines, how will you manage this child?

E.
F.

Refer the child urgently to a hospital for confinement.


Coordinate with the social worker to enroll the child in a
feeding program.
G.
Make a teaching plan for the mother, focusing on menu
planning for her child.
H.
Assess and treat the child for health problems like infections
and intestinal parasitism.
15. During the physical examination of a young child, what is the earliest sign
of xerophthalmia that you may observe?
E.
F.
G.
H.

Keratomalacia
Corneal opacity
Night blindness
Conjunctival xerosis
16. To prevent xerophthalmia, young children are given Retinol capsule every
6 months. What is the dose given to preschoolers?

E.
F.
G.
H.

10,000 IU
20,000 IU
100,000 IU
200,000 IU
17. The major sign of iron deficiency anemia is pallor. What part is best
examined for pallor?

E.
F.
G.
H.

Palms
Nailbeds
Around the lips
Lower conjunctival sac
18. Food fortification is one of the strategies to prevent micronutrient
deficiency conditions. R.A. 8976 mandates fortification of certain food items.
Which of the following is among these food items?

E.
F.
G.
H.

Sugar
Bread
Margarine
Filled milk
19. What is the best course of action when there is a measles epidemic in a
nearby municipality?

E.
F.

Give measles vaccine to babies aged 6 to 8 months.


Give babies aged 6 to 11 months one dose of 100,000 I.U. of
Retinol
G.
Instruct mothers to keep their babies at home to prevent
disease transmission.
H.
Instruct mothers to feed their babies adequately to enhance
their babies resistance.
20. A mother brought her daughter, 4 years old, to the RHU because of cough
and colds. Following the IMCI assessment guide, which of the following is a
danger sign that indicates the need for urgent referral to a hospital?
E.
F.
G.
H.

Inability to drink
High grade fever
Signs of severe dehydration
Cough for more than 30 days
21. Management of a child with measles includes the administration of which
of the following?

E.
F.
G.
H.

Gentian violet on mouth lesions


Antibiotics to prevent pneumonia
Tetracycline eye ointment for corneal opacity
Retinol capsule regardless of when the last dose was given
22. A mother brought her 10 month old infant for consultation because of
fever, which started 4 days prior to consultation. To determine malaria risk,
what will you do?

E.
F.
G.
H.

Do a tourniquet test.
Ask where the family resides.
Get a specimen for blood smear.
Ask if the fever is present everyday.
23. The following are strategies implemented by the Department of Health to
prevent mosquito-borne diseases. Which of these is most effective in the
control of Dengue fever?

E.

Stream seeding with larva-eating fish

F.
G.

Destroying breeding places of mosquitoes


Chemoprophylaxis of non-immune persons going to endemic
areas
H.
Teaching people in endemic areas to use chemically treated
mosquito nets
24. Secondary prevention for malaria includes
E.
F.
G.
H.

Planting of neem or eucalyptus trees


Residual spraying of insecticides at night
Determining whether a place is endemic or not
Growing larva-eating fish in mosquito breeding places
25. Scotch tape swab is done to check for which intestinal parasite?

E.
F.
G.
H.

Ascaris
Pinworm
Hookworm
Schistosoma
26. Which of the following signs indicates the need for sputum examination for
AFB?

E.
F.
G.
H.

Hematemesis
Fever for 1 week
Cough for 3 weeks
Chest pain for 1 week
27. Which clients are considered targets for DOTS Category I?

E.
F.
G.
H.

Sputum negative cavitary cases


Clients returning after a default
Relapses and failures of previous PTB treatment regimens
Clients diagnosed for the first time through a positive sputum
exam
28. To improve compliance to treatment, what innovation is being
implemented in DOTS?

E.
F.

Having the health worker follow up the client at home


Having the health worker or a responsible family member
monitor drug intake
G.
Having the patient come to the health center every month to
get his medications
H.
Having a target list to check on whether the patient has
collected his monthly supply of drugs

29. Diagnosis of leprosy is highly dependent on recognition of symptoms.


Which of the following is an early sign of leprosy?
E.
F.
G.
H.

E.
F.
G.
H.

Macular lesions
Inability to close eyelids
Thickened painful nerves
Sinking of the nosebridge
30. Which of the following clients should be classified as a case of
multibacillary leprosy?
3
3
5
5

skin
skin
skin
skin

lesions,
lesions,
lesions,
lesions,

negative slit skin smear


positive slit skin smear
negative slit skin smear
positive slit skin smear

Answers and Rationales


31.
Answer: (A) P.D. 996. Presidential Decree 996, enacted in
1976, made immunization in the EPI compulsory for children under 8
years of age. Hepatitis B vaccination was made compulsory for the
same age group by R.A. 7846.
32.
Answer: (B) BCG. BCG causes the formation of a superficial
abscess, which begins 2 weeks after immunization. The abscess
heals without treatment, with the formation of a permanent scar.
33.
Answer: (C) Infant BCG. Infant BCG may be given at birth.
All the other immunizations mentioned can be given at 6 weeks of
age.
34.
Answer: (A) Seizures a day after DPT 1. Seizures within 3
days after administration of DPT is an indication of hypersensitivity
to pertussis vaccine, a component of DPT. This is considered a
specific contraindication to subsequent doses of DPT.
35.
Answer: (A) Go on with the infants immunizations. In
the EPI, fever up to 38.5C is not a contraindication to immunization.
Mild acute respiratory tract infection, simple diarrhea and
malnutrition are not contraindications either.
36.
Answer: (A) 1 year. The baby will have passive natural
immunity by placental transfer of antibodies. The mother will have
active artificial immunity lasting for about 10 years. 5 doses will give
the mother lifetime protection.
37.
Answer: (C) Normal. In IMCI, a respiratory rate of 50/minute
or more is fast breathing for an infant aged 2 to 12 months.
38.
Answer: (D) Chest indrawing. In IMCI, chest indrawing is
used as the positive sign of dyspnea, indicating severe pneumonia.

39.
Answer: (B) Refer him urgently to the hospital. Severe
pneumonia requires urgent referral to a hospital. Answers A, C and D
are done for a client classified as having pneumonia.
40.
Answer: (B) Some dehydration. Using the assessment
guidelines of IMCI, a child (2 months to 5 years old) with diarrhea is
classified as having SOME DEHYDRATION if he shows 2 or more of
the following signs: restless or irritable, sunken eyes, the skin goes
back slow after a skin pinch.
41.
Answer: (B) Supervise the mother in giving 200 to 400
ml. of Oresol in 4 hours. In the IMCI management guidelines,
SOME DEHYDRATION is treated with the administration of Oresol
within a period of 4 hours. The amount of Oresol is best computed
on the basis of the childs weight (75 ml/kg body weight). If the
weight is unknown, the amount of Oresol is based on the childs age.
42.
Answer: (D) Let the child rest for 10 minutes then
continue giving Oresol more slowly. If the child vomits
persistently, that is, he vomits everything that he takes in, he has to
be referred urgently to a hospital. Otherwise, vomiting is managed
by letting the child rest for 10 minutes and then continuing with
Oresol administration. Teach the mother to give Oresol more slowly.
43.
Answer: (D) Edema. Edema, a major sign of kwashiorkor, is
caused by decreased colloidal osmotic pressure of the blood brought
about by hypoalbuminemia. Decreased blood albumin level is due a
protein-deficient diet.
44.
Answer: (A) Refer the child urgently to a hospital for
confinement. Baggy pants is a sign of severe marasmus. The
best management is urgent referral to a hospital.
45.
Answer: (D) Conjunctival xerosis. The earliest sign of
Vitamin A deficiency (xerophthalmia) is night blindness. However,
this is a functional change, which is not observable during physical
examination.The earliest visible lesion is conjunctival xerosis or
dullness of the conjunctiva due to inadequate tear production.
46.
Answer: (D) 200,000 IU. Preschoolers are given Retinol
200,000 IU every 6 months. 100,000 IU is given once to infants aged
6 to 12 months. The dose for pregnant women is 10,000 IU.
47.
Answer: (A) Palms. The anatomic characteristics of the
palms allow a reliable and convenient basis for examination for
pallor.
48.
Answer: (A) Sugar. R.A. 8976 mandates fortification of rice,
wheat flour, sugar and cooking oil with Vitamin A, iron and/or iodine.

49.
Answer: (A) Give measles vaccine to babies aged 6 to 8
months. Ordinarily, measles vaccine is given at 9 months of age.
During an impending epidemic, however, one dose may be given to
babies aged 6 to 8 months. The mother is instructed that the baby
needs another dose when the baby is 9 months old.
50.
Answer: (A) Inability to drink. A sick child aged 2 months to
5 years must be referred urgently to a hospital if he/she has one or
more of the following signs: not able to feed or drink, vomits
everything, convulsions, abnormally sleepy or difficult to awaken.
51.
Answer: (D) Retinol capsule regardless of when the last
dose was given. An infant 6 to 12 months classified as a case of
measles is given Retinol 100,000 IU; a child is given 200,000 IU
regardless of when the last dose was given.
52.
Answer: (B) Ask where the family resides. Because
malaria is endemic, the first question to determine malaria risk is
where the clients family resides. If the area of residence is not a
known endemic area, ask if the child had traveled within the past 6
months, where he/she was brought and whether he/she stayed
overnight in that area.
53.
Answer: (B) Destroying breeding places of
mosquitoes. Aedes aegypti, the vector of Dengue fever, breeds in
stagnant, clear water. Its feeding time is usually during the daytime.
It has a cyclical pattern of occurrence, unlike malaria which is
endemic in certain parts of the country.
54.
Answer: (C) Determining whether a place is endemic or
not. This is diagnostic and therefore secondary level prevention.
The other choices are for primary prevention.
55.
Answer: (B) Pinworm. Pinworm ova are deposited around the
anal orifice.
56.
Answer: (C) Cough for 3 weeks. A client is considered a PTB
suspect when he has cough for 2 weeks or more, plus one or more
of the following signs: fever for 1 month or more; chest pain lasting
for 2 weeks or more not attributed to other conditions; progressive,
unexplained weight loss; night sweats; and hemoptysis.
57.
Answer: (D) Clients diagnosed for the first time through
a positive sputum exam. Category I is for new clients diagnosed
by sputum examination and clients diagnosed to have a serious
form of extrapulmonary tuberculosis, such as TB osteomyelitis.
58.
Answer: (B) Having the health worker or a responsible
family member monitor drug intake. Directly Observed
Treatment Short Course is so-called because a treatment partner,

preferably a health worker accessible to the client, monitors the


clients compliance to the treatment.
59.
Answer: (C) Thickened painful nerves. The lesion of
leprosy is not macular. It is characterized by a change in skin color
(either reddish or whitish) and loss of sensation, sweating and hair
growth over the lesion. Inability to close the eyelids (lagophthalmos)
and sinking of the nosebridge are late symptoms.
60.
Answer: (D) 5 skin lesions, positive slit skin smear. A
multibacillary leprosy case is one who has a positive slit skin smear
and at least 5 skin lesions.
1. In the Philippines, which condition is the most frequent cause of death
associated with schistosomiasis?
E.
F.
G.
H.

Liver cancer
Liver cirrhosis
Bladder cancer
Intestinal perforation
2. What is the most effective way of controlling schistosomiasis in an endemic
area?

E.
F.
G.
H.

Use of molluscicides
Building of foot bridges
Proper use of sanitary toilets
Use of protective footwear, such as rubber boots
3. When residents obtain water from an artesian well in the neighborhood, the
level of this approved type of water facility is

E.
F.
G.
H.

I
II
III
IV
4. For prevention of hepatitis A, you decided to conduct health education
activities. Which of the following is IRRELEVANT?

E.
F.
G.
H.

Use of sterile syringes and needles


Safe food preparation and food handling by vendors
Proper disposal of human excreta and personal hygiene
Immediate reporting of water pipe leaks and illegal water
connections
5. Which biological used in Expanded Program on Immunization (EPI) should
NOT be stored in the freezer?

E.
F.
G.
H.

DPT
Oral polio vaccine
Measles vaccine
MMR
6. You will conduct outreach immunization in a barangay with a population of
about 1500. Estimate the number of infants in the barangay.

E.
F.
G.
H.

45
50
55
60
7. In Integrated Management of Childhood Illness, severe conditions generally
require urgent referral to a hospital. Which of the following severe conditions
DOES NOT always require urgent referral to a hospital?

E.
F.
G.
H.

Mastoiditis
Severe dehydration
Severe pneumonia
Severe febrile disease
8. A client was diagnosed as having Dengue fever. You will say that there is
slow capillary refill when the color of the nailbed that you pressed does not
return within how many seconds?

E.
F.
G.
H.

3
5
8
10
9. A 3-year old child was brought by his mother to the health center because
of fever of 4-day duration. The child had a positive tourniquet test result. In the
absence of other signs, which is the most appropriate measure that the PHN
may carry out to prevent Dengue shock syndrome?

E.
F.
G.
H.

Insert an NGT and give fluids per NGT.


Instruct the mother to give the child Oresol.
Start the patient on intravenous fluids STAT.
Refer the client to the physician for appropriate management.
10. The pathognomonic sign of measles is Kopliks spot. You may see Kopliks
spot by inspecting the _____.

E.
F.
G.

Nasal mucosa
Buccal mucosa
Skin on the abdomen

H.

Skin on the antecubital surface


11. Among the following diseases, which is airborne?

E.
F.
G.
H.

Viral conjunctivitis
Acute poliomyelitis
Diphtheria
Measles
12. Among children aged 2 months to 3 years, the most prevalent form of
meningitis is caused by which microorganism?

E.
F.
G.
H.

Hemophilus influenzae
Morbillivirus
Steptococcus pneumoniae
Neisseria meningitidis
13. Human beings are the major reservoir of malaria. Which of the following
strategies in malaria control is based on this fact?

E.
F.
G.
H.

Stream seeding
Stream clearing
Destruction of breeding places
Zooprophylaxis
14. The use of larvivorous fish in malaria control is the basis for which strategy
of malaria control?

E.
F.
G.
H.

Stream seeding
Stream clearing
Destruction of breeding places
Zooprophylaxis
15. Mosquito-borne diseases are prevented mostly with the use of mosquito
control measures. Which of the following is NOT appropriate for malaria
control?

E.
F.
G.
H.

Use of chemically treated mosquito nets


Seeding of breeding places with larva-eating fish
Destruction of breeding places of the mosquito vector
Use of mosquito-repelling soaps, such as those with basil or
citronella
16. A 4-year old client was brought to the health center with the chief
complaint of severe diarrhea and the passage of rice water stools. The client
is most probably suffering from which condition?

E.
F.

Giardiasis
Cholera

G.
H.

Amebiasis
Dysentery
17. In the Philippines, which specie of schistosoma is endemic in certain
regions?

E.
F.
G.
H.

S. mansoni
S. japonicum
S. malayensis
S. haematobium
18. A 32-year old client came for consultation at the health center with the
chief complaint of fever for a week. Accompanying symptoms were muscle
pains and body malaise. A week after the start of fever, the client noted
yellowish discoloration of his sclera. History showed that he waded in flood
waters about 2 weeks before the onset of symptoms. Based on his history,
which disease condition will you suspect?

E.
F.
G.
H.

Hepatitis A
Hepatitis B
Tetanus
Leptospirosis
19. MWSS provides water to Manila and other cities in Metro Manila. This is
an example of which level of water facility?

E.
F.
G.
H.

I
II
III
IV
20. You are the PHN in the city health center. A client underwent screening for
AIDS using ELISA. His result was positive. What is the best course of action
that you may take?

E.

Get a thorough history of the client, focusing on the practice of


high risk behaviors.
F.
Ask the client to be accompanied by a significant person before
revealing the result.
G.
Refer the client to the physician since he is the best person to
reveal the result to the client.
H.
Refer the client for a supplementary test, such as Western blot,
since the ELISA result may be false.
21. Which is the BEST control measure for AIDS?
E.

Being faithful to a single sexual partner

F.
G.
H.

Using a condom during each sexual contact


Avoiding sexual contact with commercial sex workers
Making sure that ones sexual partner does not have signs of

AIDS
22. The most frequent causes of death among clients with AIDS are
opportunistic diseases. Which of the following opportunistic infections is
characterized by tonsillopharyngitis?

E.
F.
G.
H.

Respiratory candidiasis
Infectious mononucleosis
Cytomegalovirus disease
Pneumocystis carinii pneumonia
23. To determine possible sources of sexually transmitted infections, which is
the BEST method that may be undertaken by the public health nurse?

E.
F.
G.
H.

Contact tracing
Community survey
Mass screening tests
Interview of suspects
24. Antiretroviral agents, such as AZT, are used in the management of AIDS.
Which of the following is NOT an action expected of these drugs.

E.
F.
G.
H.

They prolong the life of the client with AIDS.


They reduce the risk of opportunistic infections
They shorten the period of communicability of the disease.
They are able to bring about a cure of the disease condition.
25. A barangay had an outbreak of German measles. To prevent congenital
rubella, what is the BEST advice that you can give to women in the first
trimester of pregnancy in the barangay?

E.
F.
G.

Advice them on the signs of German measles.


Avoid crowded places, such as markets and moviehouses.
Consult at the health center where rubella vaccine may be
given.
H.
Consult a physician who may give them rubella
immunoglobulin.
26. You were invited to be the resource person in a training class for food
handlers. Which of the following would you emphasize regarding prevention of
staphylococcal food poisoning?
E.

All cooking and eating utensils must be thoroughly washed.

F.

Food must be cooked properly to destroy staphylococcal


microorganisms.
G.
Food handlers and food servers must have a negative stool
examination result.
H.
Proper handwashing during food preparation is the best way of
preventing the condition.
27. In a mothers class, you discussed childhood diseases such as chicken
pox. Which of the following statements about chicken pox is correct?
E.

The older one gets, the more susceptible he becomes to the


complications of chicken pox.
F.
A single attack of chicken pox will prevent future episodes,
including conditions such as shingles.
G.
To prevent an outbreak in the community, quarantine may be
imposed by health authorities.
H.
Chicken pox vaccine is best given when there is an impending
outbreak in the community.
28. Complications to infectious parotitis (mumps) may be serious in which
type of clients?
E.
F.
G.
H.

Pregnant women
Elderly clients
Young adult males
Young infants

Answers and Rationales


29.
Answer: (B) Liver cirrhosis. The etiologic agent of
schistosomiasis in the Philippines is Schistosoma japonicum, which
affects the small intestine and the liver. Liver damage is a
consequence of fibrotic reactions to schistosoma eggs in the liver.
30.
Answer: (C) Proper use of sanitary toilets. The ova of the
parasite get out of the human body together with feces. Cutting the
cycle at this stage is the most effective way of preventing the
spread of the disease to susceptible hosts.
31.
Answer: (B) II. A communal faucet or water standpost is
classified as Level II.
32.
Answer: (A) Use of sterile syringes and
needles. Hepatitis A is transmitted through the fecal oral route.
Hepatitis B is transmitted through infected body secretions like
blood and semen.
33.
Answer: (A) DPT. DPT is sensitive to freezing. The
appropriate storage temperature of DPT is 2 to 8 C only. OPV and

measles vaccine are highly sensitive to heat and require freezing.


MMR is not an immunization in the Expanded Program on
Immunization.
34.
Answer: (A) 45. To estimate the number of infants, multiply
total population by 3%.
35.
Answer: (B) Severe dehydration. The order of priority in
the management of severe dehydration is as follows: intravenous
fluid therapy, referral to a facility where IV fluids can be initiated
within 30 minutes, Oresol/nasogastric tube, Oresol/orem. When the
foregoing measures are not possible or effective, tehn urgent
referral to the hospital is done.
36.
Answer: (A) 3. Adequate blood supply to the area allows the
return of the color of the nailbed within 3 seconds.
37.
Answer: (B) Instruct the mother to give the child
Oresol. Since the child does not manifest any other danger sign,
maintenance of fluid balance and replacement of fluid loss may be
done by giving the client Oresol.
38.
Answer: (B) Buccal mucosa. Kopliks spot may be seen on
the mucosa of the mouth or the throat.
39.
Answer: (D) Measles. Viral conjunctivitis is transmitted by
direct or indirect contact with discharges from infected eyes. Acute
poliomyelitis is spread through the fecal-oral route and contact with
throat secretions, whereas diphtheria is through direct and indirect
contact with respiratory secretions.
40.
Answer: (A) Hemophilus influenzae. Hemophilus
meningitis is unusual over the age of 5 years. In developing
countries, the peak incidence is in children less than 6 months of
age. Morbillivirus is the etiology of measles. Streptococcus
pneumoniae and Neisseria meningitidis may cause meningitis, but
age distribution is not specific in young children.
41.
Answer: (D) Zooprophylaxis. Zooprophylaxis is done by
putting animals like cattle or dogs close to windows or doorways just
before nightfall. The Anopheles mosquito takes his blood meal from
the animal and goes back to its breeding place, thereby preventing
infection of humans.
42.
Answer: (A) Stream seeding. Stream seeding is done by
putting tilapia fry in streams or other bodies of water identified as
breeding places of the Anopheles mosquito
43.
Answer: (C) Destruction of breeding places of the
mosquito vector. Anopheles mosquitoes breed in slow-moving,
clear water, such as mountain streams.

44.
Answer: (B) Cholera. Passage of profuse watery stools is the
major symptom of cholera. Both amebic and bacillary dysentery are
characterized by the presence of blood and/or mucus in the stools.
Giardiasis is characterized by fat malabsorption and, therefore,
steatorrhea.
45.
Answer: (B) S. japonicum. S. mansoni is found mostly in
Africa and South America; S. haematobium in Africa and the Middle
East; and S. malayensis only in peninsular Malaysia.
46.
Answer: (D) Leptospirosis. Leptospirosis is transmitted
through contact with the skin or mucous membrane with water or
moist soil contaminated with urine of infected animals, like rats.
47.
Answer: (C) III. Waterworks systems, such as MWSS, are
classified as level III.
48.
Answer: (D) Refer the client for a supplementary test,
such as Western blot, since the ELISA result may be false. A
client having a reactive ELISA result must undergo a more specific
test, such as Western blot. A negative supplementary test result
means that the ELISA result was false and that, most probably, the
client is not infected.
49.
Answer: (A) Being faithful to a single sexual
partner. Sexual fidelity rules out the possibility of getting the
disease by sexual contact with another infected person.
Transmission occurs mostly through sexual intercourse and exposure
to blood or tissues.
50.
Answer: (B) Infectious mononucleosis. Cytomegalovirus
disease is an acute viral disease characterized by fever, sore throat
and lymphadenopathy.
51.
Answer: (A) Contact tracing. Contact tracing is the most
practical and reliable method of finding possible sources of personto-person transmitted infections, such as sexually transmitted
diseases.
52.
Answer: (D) They are able to bring about a cure of the
disease condition. There is no known treatment for AIDS.
Antiretroviral agents reduce the risk of opportunistic infections and
prolong life, but does not cure the underlying immunodeficiency.
53.
Answer: (D) Consult a physician who may give them
rubella immunoglobulin. Rubella vaccine is made up of
attenuated German measles viruses. This is contraindicated in
pregnancy. Immune globulin, a specific prophylactic against German
measles, may be given to pregnant women.

54.
Answer: (D) Proper handwashing during food
preparation is the best way of preventing the
condition. Symptoms of this food poisoning are due to
staphylococcal enterotoxin, not the microorganisms themselves.
Contamination is by food handling by persons with staphylococcal
skin or eye infections.
55.
Answer: (A) The older one gets, the more susceptible he
becomes to the complications of chicken pox. Chicken pox is
usually more severe in adults than in children. Complications, such
as pneumonia, are higher in incidence in adults.
56.
Answer: (C) Young adult males. Epididymitis and orchitis
are possible complications of mumps. In post-adolescent males,
bilateral inflammation of the testes and epididymis may cause
sterility.
1. May arrives at the health care clinic and tells the nurse that her last
menstrual period was 9 weeks ago. She also tells the nurse that a home
pregnancy test was positive but she began to have mild cramps and is now
having moderate vaginal bleeding. During the physical examination of the
client, the nurse notes that May has a dilated cervix. The nurse determines
that May is experiencing which type of abortion?
E.
Inevitable
F.
Incomplete
G.
Threatened
H.
Septic
2. Nurse Reese is reviewing the record of a pregnant client for her first
prenatal visit. Which of the following data, if noted on the clients record, would
alert the nurse that the client is at risk for a spontaneous abortion?
E.
Age 36 years
F.
History of syphilis
G.
History of genital herpes
H.
History of diabetes mellitus
3. Nurse Hazel is preparing to care for a client who is newly admitted to the
hospital with a possible diagnosis of ectopic pregnancy. Nurse Hazel develops
a plan of care for the client and determines that which of the following nursing
actions is the priority?
E.
Monitoring weight
F.
Assessing for edema
G.
Monitoring apical pulse
H.
Monitoring temperature

4. Nurse Oliver is teaching a diabetic pregnant client about nutrition and


insulin needs during pregnancy. The nurse determines that the client
understands dietary and insulin needs if the client states that the second half
of pregnancy require:
E.
Decreased caloric intake
F.
Increased caloric intake
G.
Decreased Insulin
H.
Increase Insulin
5. Nurse Michelle is assessing a 24 year old client with a diagnosis of
hydatidiform mole. She is aware that one of the following is unassociated with
this condition?
E.
Excessive fetal activity.
F.
Larger than normal uterus for gestational age.
G.
Vaginal bleeding
H.
Elevated levels of human chorionic gonadotropin.
6. A pregnant client is receiving magnesium sulfate for severe pregnancy
induced hypertension (PIH). The clinical findings that would warrant use of the
antidote , calcium gluconate is:
E.
Urinary output 90 cc in 2 hours.
F.
Absent patellar reflexes.
G.
Rapid respiratory rate above 40/min.
H.
Rapid rise in blood pressure.
7. During vaginal examination of Janah who is in labor, the presenting part is
at station plus two. Nurse, correctly interprets it as:
E.
Presenting part is 2 cm above the plane of the ischial spines.
F.
Biparietal diameter is at the level of the ischial spines.
G.
Presenting part in 2 cm below the plane of the ischial spines.
H.
Biparietal diameter is 2 cm above the ischial spines.
8. A pregnant client is receiving oxytocin (Pitocin) for induction of labor. A
condition that warrant the nurse in-charge to discontinue I.V. infusion of
Pitocin is:
E.
Contractions every 1 minutes lasting 70-80 seconds.
F.
Maternal temperature 101.2
G.
Early decelerations in the fetal heart rate.
H.
Fetal heart rate baseline 140-160 bpm.
9. Calcium gluconate is being administered to a client with pregnancy induced
hypertension (PIH). A nursing action that must be initiated as the plan of care
throughout injection of the drug is:
E.
Ventilator assistance
F.
CVP readings
G.
EKG tracings

H.

Continuous CPR
10. A trial for vaginal delivery after an earlier caesareans, would likely to be
given to a gravida, who had:
E.
First low transverse cesarean was for active herpes type 2
infections; vaginal culture at 39 weeks pregnancy was positive.
F.
First and second caesareans were for cephalopelvic
disproportion.
G.
First caesarean through a classic incision as a result of severe
fetal distress.
H.
First low transverse caesarean was for breech position. Fetus in
this pregnancy is in a vertex presentation.
11.Nurse Ryan is aware that the best initial approach when trying to take a
crying toddlers temperature is:
E.
Talk to the mother first and then to the toddler.
F.
Bring extra help so it can be done quickly.
G.
Encourage the mother to hold the child.
H.
Ignore the crying and screaming.
12.Baby Tina a 3 month old infant just had a cleft lip and palate repair. What
should the nurse do to prevent trauma to operative site?
E.
Avoid touching the suture line, even when cleaning.
F.
Place the baby in prone position.
G.
Give the baby a pacifier.
H.
Place the infants arms in soft elbow restraints.
13. Which action should nurse Marian include in the care plan for a 2 month
old with heart failure?
E.
Feed the infant when he cries.
F.
Allow the infant to rest before feeding.
G.
Bathe the infant and administer medications before feeding.
H.
Weigh and bathe the infant before feeding.
14.Nurse Hazel is teaching a mother who plans to discontinue breast feeding
after 5 months. The nurse should advise her to include which foods in her
infants diet?
E.
Skim milk and baby food.
F.
Whole milk and baby food.
G.
Iron-rich formula only.
H.
Iron-rich formula and baby food.
15.Mommy Linda is playing with her infant, who is sitting securely alone on
the floor of the clinic. The mother hides a toy behind her back and the infant
looks for it. The nurse is aware that estimated age of the infant would be:
E.
6 months
F.
4 months

G.
H.

8 months
10 months
16.Which of the following is the most prominent feature of public health
nursing?
E.
It involves providing home care to sick people who are not
confined in the hospital.
F.
Services are provided free of charge to people within the
catchments area.
G.
The public health nurse functions as part of a team providing a
public health nursing services.
H.
Public health nursing focuses on preventive, not curative,
services.
17.When the nurse determines whether resources were maximized in
implementing Ligtas Tigdas, she is evaluating
E.
Effectiveness
F.
Efficiency
G.
Adequacy
H.
Appropriateness
18.Vangie is a new B.S.N. graduate. She wants to become a Public Health
Nurse. Where should she apply?
E.
Department of Health
F.
Provincial Health Office
G.
Regional Health Office
H.
Rural Health Unit
19.Tony is aware the Chairman of the Municipal Health Board is:
E.
Mayor
F.
Municipal Health Officer
G.
Public Health Nurse
H.
Any qualified physician
20.Myra is the public health nurse in a municipality with a total population of
about 20,000. There are 3 rural health midwives among the RHU personnel.
How many more midwife items will the RHU need?
E.
1
F.
2
G.
3
H.
The RHU does not need any more midwife item.
21.According to Freeman and Heinrich, community health nursing is a
developmental service. Which of the following best illustrates this statement?
E.
The community health nurse continuously develops himself
personally and professionally.

F.

Health education and community organizing are necessary in


providing community health services.
G.
Community health nursing is intended primarily for health
promotion and prevention and treatment of disease.
H.
The goal of community health nursing is to provide nursing
services to people in their own places of residence.
22.Nurse Tina is aware that the disease declared through Presidential
Proclamation No. 4 as a target for eradication in the Philippines is?
E.
Poliomyelitis
F.
Measles
G.
Rabies
H.
Neonatal tetanus
23.May knows that the step in community organizing that involves training of
potential leaders in the community is:
E.
Integration
F.
Community organization
G.
Community study
H.
Core group formation
24.Beth a public health nurse takes an active role in community participation.
What is the primary goal of community organizing?
E.
To educate the people regarding community health problems
F.
To mobilize the people to resolve community health problems
G.
To maximize the communitys resources in dealing with health
problems.
H.
To maximize the communitys resources in dealing with health
problems.
25.Tertiary prevention is needed in which stage of the natural history of
disease?
E.
Pre-pathogenesis
F.
Pathogenesis
G.
Prodromal
H.
Terminal
26.The nurse is caring for a primigravid client in the labor and delivery area.
Which condition would place the client at risk for disseminated intravascular
coagulation (DIC)?
E.
Intrauterine fetal death.
F.
Placenta accreta.
G.
Dysfunctional labor.
H.
Premature rupture of the membranes.
27.A fullterm client is in labor. Nurse Betty is aware that the fetal heart rate
would be:

E.
F.
G.
H.

80 to 100 beats/minute
100 to 120 beats/minute
120 to 160 beats/minute
160 to 180 beats/minute
28.The skin in the diaper area of a 7 month old infant is excoriated and red.
Nurse Hazel should instruct the mother to:
E.
Change the diaper more often.
F.
Apply talc powder with diaper changes.
G.
Wash the area vigorously with each diaper change.
H.
Decrease the infants fluid intake to decrease saturating
diapers.
29.Nurse Carla knows that the common cardiac anomalies in children with
Down Syndrome (tri-somy 21) is:
E.
Atrial septal defect
F.
Pulmonic stenosis
G.
Ventricular septal defect
H.
Endocardial cushion defect
30.Malou was diagnosed with severe preeclampsia is now receiving I.V.
magnesium sulfate. The adverse effects associated with magnesium sulfate
is:
E.
Anemia
F.
Decreased urine output
G.
Hyperreflexia
H.
Increased respiratory rate
31.A 23 year old client is having her menstrual period every 2 weeks that last
for 1 week. This type of menstrual pattern is bets defined by:
E.
Menorrhagia
F.
Metrorrhagia
G.
Dyspareunia
H.
Amenorrhea
32. Jannah is admitted to the labor and delivery unit. The critical laboratory
result for this client would be:
E.
Oxygen saturation
F.
Iron binding capacity
G.
Blood typing
H.
Serum Calcium
33.Nurse Gina is aware that the most common condition found during the
second-trimester of pregnancy is:
E.
Metabolic alkalosis
F.
Respiratory acidosis
G.
Mastitis

H.

Physiologic anemia
34.Nurse Lynette is working in the triage area of an emergency department.
She sees that several pediatric clients arrive simultaneously. The client who
needs to be treated first is:
E.
A crying 5 year old child with a laceration on his scalp.
F.
A 4 year old child with a barking coughs and flushed
appearance.
G.
A 3 year old child with Down syndrome who is pale and asleep
in his mothers arms.
H.
A 2 year old infant with stridorous breath sounds, sitting up in
his mothers arms and drooling.
35.Maureen in her third trimester arrives at the emergency room with painless
vaginal bleeding. Which of the following conditions is suspected?
E.
Placenta previa
F.
Abruptio placentae
G.
Premature labor
H.
Sexually transmitted disease
36.A young child named Richard is suspected of having pinworms. The
community nurse collects a stool specimen to confirm the diagnosis. The
nurse should schedule the collection of this specimen for:
E.
Just before bedtime
F.
After the child has been bathe
G.
Any time during the day
H.
Early in the morning
37.In doing a childs admission assessment, Nurse Betty should be alert to
note which signs or symptoms of chronic lead poisoning?
E.
Irritability and seizures
F.
Dehydration and diarrhea
G.
Bradycardia and hypotension
H.
Petechiae and hematuria
38.To evaluate a womans understanding about the use of diaphragm for
family planning, Nurse Trish asks her to explain how she will use the
appliance. Which response indicates a need for further health teaching?
E.
I should check the diaphragm carefully for holes every time I
use it
F.
I may need a different size of diaphragm if I gain or lose
weight more than 20 pounds
G.
The diaphragm must be left in place for atleast 6 hours after
intercourse
H.
I really need to use the diaphragm and jelly most during the
middle of my menstrual cycle.

39.Hypoxia is a common complication of laryngotracheobronchitis. Nurse


Oliver should frequently assess a child with laryngotracheobronchitis for:
E.
Drooling
F.
Muffled voice
G.
Restlessness
H.
Low-grade fever
40.How should Nurse Michelle guide a child who is blind to walk to the
playroom?
E.
Without touching the child, talk continuously as the child walks
down the hall.
F.
Walk one step ahead, with the childs hand on the nurses
elbow.
G.
Walk slightly behind, gently guiding the child forward.
H.
Walk next to the child, holding the childs hand.
41.When assessing a newborn diagnosed with ductus arteriosus, Nurse Olivia
should expect that the child most likely would have an:
E.
Loud, machinery-like murmur.
F.
Bluish color to the lips.
G.
Decreased BP reading in the upper extremities
H.
Increased BP reading in the upper extremities.
42.The reason nurse May keeps the neonate in a neutral thermal environment
is that when a newborn becomes too cool, the neonate requires:
E.
Less oxygen, and the newborns metabolic rate increases.
F.
More oxygen, and the newborns metabolic rate decreases.
G.
More oxygen, and the newborns metabolic rate increases.
H.
Less oxygen, and the newborns metabolic rate decreases.
43.Before adding potassium to an infants I.V. line, Nurse Ron must be sure to
assess whether this infant has:
E.
Stable blood pressure
F.
Patant fontanelles
G.
Moros reflex
H.
Voided
44.Nurse Carla should know that the most common causative factor of
dermatitis in infants and younger children is:
E.
Baby oil
F.
Baby lotion
G.
Laundry detergent
H.
Powder with cornstarch
45.During tube feeding, how far above an infants stomach should the nurse
hold the syringe with formula?
E.
6 inches

F.
G.
H.

12 inches
18 inches
24 inches
46. In a mothers class, Nurse Lhynnete discussed childhood diseases such
as chicken pox. Which of the following statements about chicken pox is
correct?
E.
The older one gets, the more susceptible he becomes to the
complications of chicken pox.
F.
A single attack of chicken pox will prevent future episodes,
including conditions such as shingles.
G.
To prevent an outbreak in the community, quarantine may be
imposed by health authorities.
H.
Chicken pox vaccine is best given when there is an impending
outbreak in the community.
47.Barangay Pinoy had an outbreak of German measles. To prevent
congenital rubella, what is the BEST advice that you can give to women in the
first trimester of pregnancy in the barangay Pinoy?
E.
Advice them on the signs of German measles.
F.
Avoid crowded places, such as markets and movie houses.
G.
Consult at the health center where rubella vaccine may be
given.
H.
Consult a physician who may give them rubella
immunoglobulin.
48.Myrna a public health nurse knows that to determine possible sources of
sexually transmitted infections, the BEST method that may be undertaken is:
E.
Contact tracing
F.
Community survey
G.
Mass screening tests
H.
Interview of suspects
49.A 33-year old female client came for consultation at the health center with
the chief complaint of fever for a week. Accompanying symptoms were muscle
pains and body malaise. A week after the start of fever, the client noted
yellowish discoloration of his sclera. History showed that he waded in flood
waters about 2 weeks before the onset of symptoms. Based on her history,
which disease condition will you suspect?
E.
Hepatitis A
F.
Hepatitis B
G.
Tetanus
H.
Leptospirosis

50.Mickey a 3-year old client was brought to the health center with the chief
complaint of severe diarrhea and the passage of rice water stools. The client
is most probably suffering from which condition?
E.
Giardiasis
F.
Cholera
G.
Amebiasis
H.
Dysentery
51.The most prevalent form of meningitis among children aged 2 months to 3
years is caused by which microorganism?
E.
Hemophilus influenzae
F.
Morbillivirus
G.
Steptococcus pneumoniae
H.
Neisseria meningitidis
52.The student nurse is aware that the pathognomonic sign of measles is
Kopliks spot and you may see Kopliks spot by inspecting the:
E.
Nasal mucosa
F.
Buccal mucosa
G.
Skin on the abdomen
H.
Skin on neck
53.Angel was diagnosed as having Dengue fever. You will say that there is
slow capillary refill when the color of the nailbed that you pressed does not
return within how many seconds?
E.
3 seconds
F.
6 seconds
G.
9 seconds
H.
10 seconds
54.In Integrated Management of Childhood Illness, the nurse is aware that the
severe conditions generally require urgent referral to a hospital. Which of the
following severe conditions DOES NOT always require urgent referral to a
hospital?
E.
Mastoiditis
F.
Severe dehydration
G.
Severe pneumonia
H.
Severe febrile disease
55.Myrna a public health nurse will conduct outreach immunization in a
barangay Masay with a population of about 1500. The estimated number of
infants in the barangay would be:
E.
45 infants
F.
50 infants
G.
55 infants
H.
65 infants

56.The community nurse is aware that the biological used in Expanded


Program on Immunization (EPI) should NOT be stored in the freezer?
E.
DPT
F.
Oral polio vaccine
G.
Measles vaccine
H.
MMR
57.It is the most effective way of controlling schistosomiasis in an endemic
area?
E.
Use of molluscicides
F.
Building of foot bridges
G.
Proper use of sanitary toilets
H.
Use of protective footwear, such as rubber boots
58.Several clients is newly admitted and diagnosed with leprosy. Which of the
following clients should be classified as a case of multibacillary leprosy?
E.
3 skin lesions, negative slit skin smear
F.
3 skin lesions, positive slit skin smear
G.
5 skin lesions, negative slit skin smear
H.
5 skin lesions, positive slit skin smear
59.Nurses are aware that diagnosis of leprosy is highly dependent on
recognition of symptoms. Which of the following is an early sign of leprosy?
E.
Macular lesions
F.
Inability to close eyelids
G.
Thickened painful nerves
H.
Sinking of the nosebridge
60.Marie brought her 10 month old infant for consultation because of fever,
started 4 days prior to consultation. In determining malaria risk, what will you
do?
E.
Perform a tourniquet test.
F.
Ask where the family resides.
G.
Get a specimen for blood smear.
H.
Ask if the fever is present everyday.
61.Susie brought her 4 years old daughter to the RHU because of cough and
colds. Following the IMCI assessment guide, which of the following is a
danger sign that indicates the need for urgent referral to a hospital?
E.
Inability to drink
F.
High grade fever
G.
Signs of severe dehydration
H.
Cough for more than 30 days
62.Jimmy a 2-year old child revealed baggy pants. As a nurse, using the
IMCI guidelines, how will you manage Jimmy?
E.
Refer the child urgently to a hospital for confinement.

F.

Coordinate with the social worker to enroll the child in a


feeding program.
G.
Make a teaching plan for the mother, focusing on menu
planning for her child.
H.
Assess and treat the child for health problems like infections
and intestinal parasitism.
63.Gina is using Oresol in the management of diarrhea of her 3-year old child.
She asked you what to do if her child vomits. As a nurse you will tell her to:
E.
Bring the child to the nearest hospital for further assessment.
F.
Bring the child to the health center for intravenous fluid
therapy.
G.
Bring the child to the health center for assessment by the
physician.
H.
Let the child rest for 10 minutes then continue giving Oresol
more slowly.
64.Nikki a 5-month old infant was brought by his mother to the health center
because of diarrhea for 4 to 5 times a day. Her skin goes back slowly after a
skin pinch and her eyes are sunken. Using the IMCI guidelines, you will
classify this infant in which category?
E.
No signs of dehydration
F.
Some dehydration
G.
Severe dehydration
H.
The data is insufficient.
65.Chris a 4-month old infant was brought by her mother to the health center
because of cough. His respiratory rate is 42/minute. Using the Integrated
Management of Child Illness (IMCI) guidelines of assessment, his breathing is
considered as:
E.
Fast
F.
Slow
G.
Normal
H.
Insignificant
66.Maylene had just received her 4th dose of tetanus toxoid. She is aware
that her baby will have protection against tetanus for
E.
1 year
F.
3 years
G.
5 years
H.
Lifetime
67.Nurse Ron is aware that unused BCG should be discarded after how many
hours of reconstitution?
E.
2 hours
F.
4 hours

G.
H.

8 hours
At the end of the day
68.The nurse explains to a breastfeeding mother that breast milk is sufficient
for all of the babys nutrient needs only up to:
E.
5 months
F.
6 months
G.
1 year
H.
2 years
69.Nurse Ron is aware that the gestational age of a conceptus that is
considered viable (able to live outside the womb) is:
E.
8 weeks
F.
12 weeks
G.
24 weeks
H.
32 weeks
70.When teaching parents of a neonate the proper position for the neonates
sleep, the nurse Patricia stresses the importance of placing the neonate on
his back to reduce the risk of which of the following?
E.
Aspiration
F.
Sudden infant death syndrome (SIDS)
G.
Suffocation
H.
Gastroesophageal reflux (GER)
71.Which finding might be seen in baby James a neonate suspected of having
an infection?
E.
Flushed cheeks
F.
Increased temperature
G.
Decreased temperature
H.
Increased activity level
72.Baby Jenny who is small-for-gestation is at increased risk during the
transitional period for which complication?
E.
Anemia probably due to chronic fetal hyposia
F.
Hyperthermia due to decreased glycogen stores
G.
Hyperglycemia due to decreased glycogen stores
H.
Polycythemia probably due to chronic fetal hypoxia
73.Marjorie has just given birth at 42 weeks gestation. When the nurse
assessing the neonate, which physical finding is expected?
E.
A sleepy, lethargic baby
F.
Lanugo covering the body
G.
Desquamation of the epidermis
H.
Vernix caseosa covering the body

74.After reviewing the Myrnas maternal history of magnesium sulfate during


labor, which condition would nurse Richard anticipate as a potential problem
in the neonate?
E.
Hypoglycemia
F.
Jitteriness
G.
Respiratory depression
H.
Tachycardia
75.Which symptom would indicate the Baby Alexandra was adapting
appropriately to extra-uterine life without difficulty?
E.
Nasal flaring
F.
Light audible grunting
G.
Respiratory rate 40 to 60 breaths/minute
H.
Respiratory rate 60 to 80 breaths/minute
76. When teaching umbilical cord care for Jennifer a new mother, the nurse
Jenny would include which information?
E.
Apply peroxide to the cord with each diaper change
F.
Cover the cord with petroleum jelly after bathing
G.
Keep the cord dry and open to air
H.
Wash the cord with soap and water each day during a tub bath.
77.Nurse John is performing an assessment on a neonate. Which of the
following findings is considered common in the healthy neonate?
E.
Simian crease
F.
Conjunctival hemorrhage
G.
Cystic hygroma
H.
Bulging fontanelle
78.Dr. Esteves decides to artificially rupture the membranes of a mother who
is on labor. Following this procedure, the nurse Hazel checks the fetal heart
tones for which the following reasons?
E.
To determine fetal well-being.
F.
To assess for prolapsed cord
G.
To assess fetal position
H.
To prepare for an imminent delivery.
79.Which of the following would be least likely to indicate anticipated bonding
behaviors by new parents?
E.
The parents willingness to touch and hold the new born.
F.
The parents expression of interest about the size of the new
born.
G.
The parents indication that they want to see the newborn.
H.
The parents interactions with each other.
80.Following a precipitous delivery, examination of the clients vagina reveals

a fourth-degree laceration. Which of the following would be


contraindicated when caring for this client?
E.
Applying cold to limit edema during the first 12 to 24 hours.
F.
Instructing the client to use two or more peripads to cushion
the area.
G.
Instructing the client on the use of sitz baths if ordered.
H.
Instructing the client about the importance of perineal (kegel)
exercises.
81. A pregnant woman accompanied by her husband, seeks admission to the
labor and delivery area. She states that shes in labor and says she attended
the facility clinic for prenatal care. Which question should the nurse Oliver ask
her first?
E.
Do you have any chronic illnesses?
F.
Do you have any allergies?
G.
What is your expected due date?
H.
Who will be with you during labor?
82.A neonate begins to gag and turns a dusky color. What should the nurse
do first?
E.
Calm the neonate.
F.
Notify the physician.
G.
Provide oxygen via face mask as ordered
H.
Aspirate the neonates nose and mouth with a bulb syringe.
83. When a client states that her water broke, which of the following actions
would be inappropriate for the nurse to do?
E.
Observing the pooling of straw-colored fluid.
F.
Checking vaginal discharge with nitrazine paper.
G.
Conducting a bedside ultrasound for an amniotic fluid index.
H.
Observing for flakes of vernix in the vaginal discharge.
84. A baby girl is born 8 weeks premature. At birth, she has no spontaneous
respirations but is successfully resuscitated. Within several hours she
develops respiratory grunting, cyanosis, tachypnea, nasal flaring, and
retractions. Shes diagnosed with respiratory distress syndrome, intubated,
and placed on a ventilator. Which nursing action should be included in the
babys plan of care to prevent retinopathy of prematurity?
E.
Cover his eyes while receiving oxygen.
F.
Keep her body temperature low.
G.
Monitor partial pressure of oxygen (Pao2) levels.
H.
Humidify the oxygen.
85. Which of the following is normal newborn calorie intake?
E.
110 to 130 calories per kg.
F.
30 to 40 calories per lb of body weight.

G.
H.

At least 2 ml per feeding


90 to 100 calories per kg
86. Nurse John is knowledgeable that usually individual twins will grow
appropriately and at the same rate as singletons until how many weeks?
E.
16 to 18 weeks
F.
18 to 22 weeks
G.
30 to 32 weeks
H.
38 to 40 weeks
87. Which of the following classifications applies to monozygotic twins for
whom the cleavage of the fertilized ovum occurs more than 13 days after
fertilization?
E.
conjoined twins
F.
diamniotic dichorionic twins
G.
diamniotic monochorionic twin
H.
monoamniotic monochorionic twins
88. Tyra experienced painless vaginal bleeding has just been diagnosed as
having a placenta previa. Which of the following procedures is usually
performed to diagnose placenta previa?
E.
Amniocentesis
F.
Digital or speculum examination
G.
External fetal monitoring
H.
Ultrasound
89. Nurse Arnold knows that the following changes in respiratory functioning
during pregnancy is considered normal:
E.
Increased tidal volume
F.
Increased expiratory volume
G.
Decreased inspiratory capacity
H.
Decreased oxygen consumption
90. Emily has gestational diabetes and it is usually managed by which of the
following therapy?
E.
Diet
F.
Long-acting insulin
G.
Oral hypoglycemic
H.
Oral hypoglycemic drug and insulin
91. Magnesium sulfate is given to Jemma with preeclampsia to prevent which
of the following condition?
E.
Hemorrhage
F.
Hypertension
G.
Hypomagnesemia
H.
Seizure

92. Cammile with sickle cell anemia has an increased risk for having a sickle
cell crisis during pregnancy. Aggressive management of a sickle cell crisis
includes which of the following measures?
E.
Antihypertensive agents
F.
Diuretic agents
G.
I.V. fluids
H.
Acetaminophen (Tylenol) for pain
93. Which of the following drugs is the antidote for magnesium toxicity?
E.
Calcium gluconate (Kalcinate)
F.
Hydralazine (Apresoline)
G.
Naloxone (Narcan)
H.
Rho (D) immune globulin (RhoGAM)
94. Marlyn is screened for tuberculosis during her first prenatal visit. An
intradermal injection of purified protein derivative (PPD) of the tuberculin
bacilli is given. She is considered to have a positive test for which of the
following results?
E.
An indurated wheal under 10 mm in diameter appears in 6 to
12 hours.
F.
An indurated wheal over 10 mm in diameter appears in 48 to
72 hours.
G.
A flat circumcised area under 10 mm in diameter appears in 6
to 12 hours.
H.
A flat circumcised area over 10 mm in diameter appears in 48
to 72 hours.
95. Dianne, 24 year-old is 27 weeks pregnant arrives at her physicians office
with complaints of fever, nausea, vomiting, malaise, unilateral flank pain, and
costovertebral angle tenderness. Which of the following diagnoses is most
likely?
E.
Asymptomatic bacteriuria
F.
Bacterial vaginosis
G.
Pyelonephritis
H.
Urinary tract infection (UTI)
96. Rh isoimmunization in a pregnant client develops during which of the
following conditions?
E.
Rh-positive maternal blood crosses into fetal blood, stimulating
fetal antibodies.
F.
Rh-positive fetal blood crosses into maternal blood,
stimulating maternal antibodies.
G.
Rh-negative fetal blood crosses into maternal blood,
stimulating maternal antibodies.

H.

Rh-negative maternal blood crosses into fetal blood,


stimulating fetal antibodies.
97. To promote comfort during labor, the nurse John advises a client to
assume certain positions and avoid others. Which position may cause
maternal hypotension and fetal hypoxia?
E.
Lateral position
F.
Squatting position
G.
Supine position
H.
Standing position
98. Celeste who used heroin during her pregnancy delivers a neonate. When
assessing the neonate, the nurse Lhynnette expects to find:
E.
Lethargy 2 days after birth.
F.
Irritability and poor sucking.
G.
A flattened nose, small eyes, and thin lips.
H.
Congenital defects such as limb anomalies.
99. The uterus returns to the pelvic cavity in which of the following time
frames?
E.
7th to 9th day postpartum.
F.
2 weeks postpartum.
G.
End of 6th week postpartum.
H.
When the lochia changes to alba.
100. Maureen, a primigravida client, age 20, has just completed a difficult,
forceps-assisted delivery of twins. Her labor was unusually long and required
oxytocin (Pitocin) augmentation. The nurse whos caring for her should stay
alert for:
E.
Uterine inversion
F.
Uterine atony
G.
Uterine involution
H.
Uterine discomfort

Answers and Rationales


101.
Answer: (A) Inevitable. An inevitable abortion is termination
of pregnancy that cannot be prevented. Moderate to severe
bleeding with mild cramping and cervical dilation would be noted in
this type of abortion.
102.
Answer: (B) History of syphilis. Maternal infections such as
syphilis, toxoplasmosis, and rubella are causes of spontaneous
abortion.
103.
Answer: (C) Monitoring apical pulse. Nursing care for the
client with a possible ectopic pregnancy is focused on preventing or
identifying hypovolemic shock and controlling pain. An elevated
pulse rate is an indicator of shock.

104.
Answer: (B) Increased caloric intake. Glucose crosses the
placenta, but insulin does not. High fetal demands for glucose,
combined with the insulin resistance caused by hormonal changes in
the last half of pregnancy can result in elevation of maternal blood
glucose levels. This increases the mothers demand for insulin and is
referred to as the diabetogenic effect of pregnancy.
105.
Answer: (A) Excessive fetal activity. The most common
signs and symptoms of hydatidiform mole includes elevated levels
of human chorionic gonadotropin, vaginal bleeding, larger than
normal uterus for gestational age, failure to detect fetal heart
activity even with sensitive instruments, excessive nausea
and vomiting, and early development of pregnancy-induced
hypertension. Fetal activity would not be noted.
106.
Answer: (B) Absent patellar reflexes. Absence of patellar
reflexes is an indicator of hypermagnesemia, which requires
administration of calcium gluconate.
107.
Answer: (C) Presenting part in 2 cm below the plane of
the ischial spines. Fetus at station plus two indicates that the
presenting part is 2 cm below the plane of the ischial spines.
108.
Answer: (A) Contractions every 1 minutes lasting 7080 seconds. Contractions every 1 minutes lasting 70-80
seconds, is indicative of hyperstimulation of the uterus, which could
result in injury to the mother and the fetus if Pitocin is not
discontinued.
109.
Answer: (C) EKG tracings. A potential side effect of calcium
gluconate administration is cardiac arrest. Continuous monitoring of
cardiac activity (EKG) throught administration of calcium gluconate
is an essential part of care.
110.
Answer: (D) First low transverse caesarean was for
breech position. Fetus in this pregnancy is in a vertex
presentation. This type of client has no obstetrical indication for a
caesarean section as she did with her first caesarean delivery.
111.
Answer: (A) Talk to the mother first and then to the
toddler. When dealing with a crying toddler, the best approach is to
talk to the mother and ignore the toddler first. This approach helps
the toddler get used to the nurse before she attempts any
procedures. It also gives the toddler an opportunity to see that the
mother trusts the nurse.
112.
Answer: (D) Place the infants arms in soft elbow
restraints. Soft restraints from the upper arm to the wrist prevent
the infant from touching her lip but allow him to hold a favorite item

such as a blanket. Because they could damage the operative site,


such as objects as pacifiers, suction catheters, and small spoons
shouldnt be placed in a babys mouth after cleft repair. A baby in a
prone position may rub her face on the sheets and traumatize the
operative site. The suture line should be cleaned gently to prevent
infection, which could interfere with healing and damage the
cosmetic appearance of the repair.
113.
Answer: (B) Allow the infant to rest before
feeding. Because feeding requires so much energy, an infant with
heart failure should rest before feeding.
114.
Answer: (C) Iron-rich formula only. The infants at age 5
months should receive iron-rich formula and that they shouldnt
receive solid food, even baby food until age 6 months.
115.
Answer: (D) 10 months. A 10 month old infant can sit alone
and understands object permanence, so he would look for the
hidden toy. At age 4 to 6 months, infants cant sit securely alone. At
age 8 months, infants can sit securely alone but cannot understand
the permanence of objects.
116.
Answer: (D) Public health nursing focuses on
preventive, not curative, services. The catchments area in PHN
consists of a residential community, many of whom are well
individuals who have greater need for preventive rather than
curative services.
117.
Answer: (B) Efficiency. Efficiency is determining whether the
goals were attained at the least possible cost.
118.
Answer: (D) Rural Health Unit. R.A. 7160 devolved basic
health services to local government units (LGUs ). The public health
nurse is an employee of the LGU.
119.
Answer: (A) Mayor. The local executive serves as the
chairman of the Municipal Health Board.
120.
Answer: (A) 1. Each rural health midwife is given a population
assignment of about 5,000.
121.
Answer: (B) Health education and community organizing
are necessary in providing community health services. The
community health nurse develops the health capability of people
through health education and community organizing activities.
122.
Answer: (B) Measles. Presidential Proclamation No. 4 is on
the Ligtas Tigdas Program.
123.
Answer: (D) Core group formation. In core group
formation, the nurse is able to transfer the technology of community

organizing to the potential or informal community leaders through a


training program.
124.
Answer: (D) To maximize the communitys resources in
dealing with health problems. Community organizing is a
developmental service, with the goal of developing the peoples selfreliance in dealing with community health problems. A, B and C are
objectives of contributory objectives to this goal.
125.
Answer: (D) Terminal. Tertiary prevention involves
rehabilitation, prevention of permanent disability and disability
limitation appropriate for convalescents, the disabled, complicated
cases and the terminally ill (those in the terminal stage of a
disease).
126.
Answer: (A) Intrauterine fetal death. Intrauterine fetal
death, abruptio placentae, septic shock, and amniotic fluid
embolism may trigger normal clotting mechanisms; if clotting
factors are depleted, DIC may occur. Placenta accreta, dysfunctional
labor, and premature rupture of the membranes arent associated
with DIC.
127.
Answer: (C) 120 to 160 beats/minute. A rate of 120 to 160
beats/minute in the fetal heart appropriate for filling the heart with
blood and pumping it out to the system.
128.
Answer: (A) Change the diaper more often. Decreasing
the amount of time the skin comes contact with wet soiled diapers
will help heal the irritation.
129.
Answer: (D) Endocardial cushion defect. Endocardial
cushion defects are seen most in children with Down syndrome,
asplenia, or polysplenia.
130.
Answer: (B) Decreased urine output. Decreased urine
output may occur in clients receiving I.V. magnesium and should be
monitored closely to keep urine output at greater than 30 ml/hour,
because magnesium is excreted through the kidneys and can easily
accumulate to toxic levels.
131.
Answer: (A) Menorrhagia. Menorrhagia is an excessive
menstrual period.
132.
Answer: (C) Blood typing. Blood type would be a critical
value to have because the risk of blood loss is always a potential
complication during the labor and delivery process. Approximately
40% of a womans cardiac output is delivered to the uterus,
therefore, blood loss can occur quite rapidly in the event of
uncontrolled bleeding.

133.
Answer: (D) Physiologic anemia. Hemoglobin values and
hematocrit decrease during pregnancy as the increase in plasma
volume exceeds the increase in red blood cell production.
134.
Answer: (D) A 2 year old infant with stridorous breath
sounds, sitting up in his mothers arms and drooling. The
infant with the airway emergency should be treated first, because of
the risk of epiglottitis.
135.
Answer: (A) Placenta previa. Placenta previa with painless
vaginal bleeding.
136.
Answer: (D) Early in the morning. Based on the nurses
knowledge of microbiology, the specimen should be collected early
in the morning. The rationale for this timing is that, because the
female worm lays eggs at night around the perineal area, the first
bowel movement of the day will yield the best results. The specific
type of stool specimen used in the diagnosis of pinworms is called
the tape test.
137.
Answer: (A) Irritability and seizures. Lead poisoning
primarily affects the CNS, causing increased intracranial pressure.
This condition results in irritability and changes in level of
consciousness, as well as seizure disorders, hyperactivity, and
learning disabilities.
138.
Answer: (D) I really need to use the diaphragm and
jelly most during the middle of my menstrual cycle. The
woman must understand that, although the fertile period is
approximately mid-cycle, hormonal variations do occur and can
result in early or late ovulation. To be effective, the diaphragm
should be inserted before every intercourse.
139.
Answer: (C) Restlessness. In a child, restlessness is the
earliest sign of hypoxia. Late signs of hypoxia in a child are
associated with a change in color, such as pallor or cyanosis.
140.
Answer: (B) Walk one step ahead, with the childs hand
on the nurses elbow. This procedure is generally recommended
to follow in guiding a person who is blind.
141.
Answer: (A) Loud, machinery-like murmur. A loud,
machinery-like murmur is a characteristic finding associated with
patent ductus arteriosus.
142.
Answer: (C) More oxygen, and the newborns metabolic
rate increases. When cold, the infant requires more oxygen and
there is an increase in metabolic rate. Non-shievering thermogenesis
is a complex process that increases the metabolic rate and rate of

oxygen consumption, therefore, the newborn increase heat


production.
143.
Answer: (D) Voided. Before administering potassium I.V. to
any client, the nurse must first check that the clients kidneys are
functioning and that the client is voiding. If the client is not voiding,
the nurse should withhold the potassium and notify the physician.
144.
Answer: (C) Laundry detergent. Eczema or dermatitis is an
allergic skin reaction caused by an offending allergen. The topical
allergen that is the most common causative factor is laundry
detergent.
145.
Answer: (A) 6 inches. This distance allows for easy flow of
the formula by gravity, but the flow will be slow enough not to
overload the stomach too rapidly.
146.
Answer: (A) The older one gets, the more susceptible he
becomes to the complications of chicken pox. Chicken pox is
usually more severe in adults than in children. Complications, such
as pneumonia, are higher in incidence in adults.
147.
Answer: (D) Consult a physician who may give them
rubella immunoglobulin. Rubella vaccine is made up of
attenuated German measles viruses. This is contraindicated in
pregnancy. Immune globulin, a specific prophylactic against German
measles, may be given to pregnant women.
148.
Answer: (A) Contact tracing. Contact tracing is the most
practical and reliable method of finding possible sources of personto-person transmitted infections, such as sexually transmitted
diseases.
149.
Answer: (D) Leptospirosis. Leptospirosis is transmitted
through contact with the skin or mucous membrane with water or
moist soil contaminated with urine of infected animals, like rats.
150.
Answer: (B) Cholera. Passage of profuse watery stools is the
major symptom of cholera. Both amebic and bacillary dysentery are
characterized by the presence of blood and/or mucus in the stools.
Giardiasis is characterized by fat malabsorption and, therefore,
steatorrhea.
151.
Answer: (A) Hemophilus influenzae. Hemophilus
meningitis is unusual over the age of 5 years. In developing
countries, the peak incidence is in children less than 6 months of
age. Morbillivirus is the etiology of measles. Streptococcus
pneumoniae and Neisseria meningitidis may cause meningitis, but
age distribution is not specific in young children.

152.
Answer: (B) Buccal mucosa. Kopliks spot may be seen on
the mucosa of the mouth or the throat.
153.
Answer: (A) 3 seconds. Adequate blood supply to the area
allows the return of the color of the nailbed within 3 seconds.
154.
Answer: (B) Severe dehydration. The order of priority in
the management of severe dehydration is as follows: intravenous
fluid therapy, referral to a facility where IV fluids can be initiated
within 30 minutes, Oresol or nasogastric tube. When the foregoing
measures are not possible or effective, then urgent referral to the
hospital is done.
155.
Answer: (A) 45 infants. To estimate the number of infants,
multiply total population by 3%.
156.
Answer: (A) DPT. DPT is sensitive to freezing. The appropriate
storage temperature of DPT is 2 to 8 C only. OPV and measles
vaccine are highly sensitive to heat and require freezing. MMR is not
an immunization in the Expanded Program on Immunization.
157.
Answer: (C) Proper use of sanitary toilets. The ova of the
parasite get out of the human body together with feces. Cutting the
cycle at this stage is the most effective way of preventing the
spread of the disease to susceptible hosts.
158.
Answer: (D) 5 skin lesions, positive slit skin smear. A
multibacillary leprosy case is one who has a positive slit skin smear
and at least 5 skin lesions.
159.
Answer: (C) Thickened painful nerves. The lesion of
leprosy is not macular. It is characterized by a change in skin color
(either reddish or whitish) and loss of sensation, sweating and hair
growth over the lesion. Inability to close the eyelids (lagophthalmos)
and sinking of the nosebridge are late symptoms.
160.
Answer: (B) Ask where the family resides. Because
malaria is endemic, the first question to determine malaria risk is
where the clients family resides. If the area of residence is not a
known endemic area, ask if the child had traveled within the past 6
months, where she was brought and whether she stayed overnight
in that area.
161.
Answer: (A) Inability to drink. A sick child aged 2 months to
5 years must be referred urgently to a hospital if he/she has one or
more of the following signs: not able to feed or drink, vomits
everything, convulsions, abnormally sleepy or difficult to awaken.
162.
Answer: (A) Refer the child urgently to a hospital for
confinement. Baggy pants is a sign of severe marasmus. The
best management is urgent referral to a hospital.

163.
Answer: (D) Let the child rest for 10 minutes then
continue giving Oresol more slowly. If the child vomits
persistently, that is, he vomits everything that he takes in, he has to
be referred urgently to a hospital. Otherwise, vomiting is managed
by letting the child rest for 10 minutes and then continuing with
Oresol administration. Teach the mother to give Oresol more slowly.
164.
Answer: (B) Some dehydration. Using the assessment
guidelines of IMCI, a child (2 months to 5 years old) with diarrhea is
classified as having SOME DEHYDRATION if he shows 2 or more of
the following signs: restless or irritable, sunken eyes, the skin goes
back slow after a skin pinch.
165.
Answer: (C) Normal. In IMCI, a respiratory rate of 50/minute
or more is fast breathing for an infant aged 2 to 12 months.
166.
Answer: (A) 1 year. The baby will have passive natural
immunity by placental transfer of antibodies. The mother will have
active artificial immunity lasting for about 10 years. 5 doses will give
the mother lifetime protection.
167.
Answer: (B) 4 hours. While the unused portion of other
biologicals in EPI may be given until the end of the day, only BCG is
discarded 4 hours after reconstitution. This is why BCG immunization
is scheduled only in the morning.
168.
Answer: (B) 6 months. After 6 months, the babys nutrient
needs, especially the babys iron requirement, can no longer be
provided by mothers milk alone.
169.
Answer: (C) 24 weeks. At approximately 23 to 24 weeks
gestation, the lungs are developed enough to sometimes maintain
extrauterine life. The lungs are the most immature system during
the gestation period. Medical care for premature labor begins much
earlier (aggressively at 21 weeks gestation)
170.
Answer: (B) Sudden infant death syndrome
(SIDS). Supine positioning is recommended to reduce the risk of
SIDS in infancy. The risk of aspiration is slightly increased with the
supine position. Suffocation would be less likely with an infant
supine than prone and the position for GER requires the head of the
bed to be elevated.
171.
Answer: (C) Decreased temperature. Temperature
instability, especially when it results in a low temperature in the
neonate, may be a sign of infection. The neonates color often
changes with an infection process but generally becomes ashen or
mottled. The neonate with an infection will usually show a decrease
in activity level or lethargy.

172.
Answer: (D) Polycythemia probably due to chronic fetal
hypoxia. The small-for-gestation neonate is at risk for developing
polycythemia during the transitional period in an attempt to
decreasehypoxia. The neonates are also at increased risk for
developing hypoglycemia and hypothermia due to decreased
glycogen stores.
173.
Answer: (C) Desquamation of the epidermis. Postdate
fetuses lose the vernix caseosa, and the epidermis may become
desquamated. These neonates are usually very alert. Lanugo is
missing in the postdate neonate.
174.
Answer: (C) Respiratory depression. Magnesium sulfate
crosses the placenta and adverse neonatal effects are respiratory
depression, hypotonia, and bradycardia. The serum blood sugar isnt
affected by magnesium sulfate. The neonate would be floppy, not
jittery.
175.
Answer: (C) Respiratory rate 40 to 60 breaths/minute. A
respiratory rate 40 to 60 breaths/minute is normal for a neonate
during the transitional period. Nasal flaring, respiratory rate more
than 60 breaths/minute, and audible grunting are signs of
respiratory distress.
176.
Answer: (C) Keep the cord dry and open to air. Keeping
the cord dry and open to air helps reduce infection and hastens
drying. Infants arent given tub bath but are sponged off until the
cord falls off. Petroleum jelly prevents the cord from drying and
encourages infection. Peroxide could be painful and isnt
recommended.
177.
Answer: (B) Conjunctival hemorrhage. Conjunctival
hemorrhages are commonly seen in neonates secondary to the
cranial pressure applied during the birth process. Bulging fontanelles
are a sign of intracranial pressure. Simian creases are present in
40% of the neonates with trisomy 21. Cystic hygroma is a neck
mass that can affect the airway.
178.
Answer: (B) To assess for prolapsed cord. After a client
has an amniotomy, the nurse should assure that the cord isnt
prolapsed and that the baby tolerated the procedure well. The most
effective way to do this is to check the fetal heart rate. Fetal wellbeing is assessed via a nonstress test. Fetal position is determined
by vaginal examination. Artificial rupture of membranes doesnt
indicate an imminent delivery.
179.
Answer: (D) The parents interactions with each
other. Parental interaction will provide the nurse with a good

assessment of the stability of the familys home life but it has no


indication for parental bonding. Willingness to touch and hold the
newborn, expressing interest about the newborns size, and
indicating a desire to see the newborn are behaviors indicating
parental bonding.
180.
Answer: (B) Instructing the client to use two or more
peripads to cushion the area. Using two or more peripads would
do little to reduce the pain or promote perineal healing. Cold
applications, sitz baths, and Kegel exercises are important measures
when the client has a fourth-degree laceration.
181.
Answer: (C) What is your expected due date? When
obtaining the history of a client who may be in labor, the nurses
highest priority is to determine her current status, particularly her
due date, gravidity, and parity. Gravidity and parity affect the
duration of labor and the potential for labor complications. Later, the
nurse should ask about chronic illnesses, allergies, and support
persons.
182.
Answer: (D) Aspirate the neonates nose and mouth
with a bulb syringe. The nurses first action should be to clear the
neonates airway with a bulb syringe. After the airway is clear and
the neonates color improves, the nurse should comfort and calm
the neonate. If the problem recurs or the neonates color doesnt
improve readily, the nurse should notify the physician. Administering
oxygen when the airway isnt clear would be ineffective.
183.
Answer: (C) Conducting a bedside ultrasound for an
amniotic fluid index. It isnt within a nurses scope of practice to
perform and interpret a bedside ultrasound under these conditions
and without specialized training. Observing for pooling of strawcolored fluid, checking vaginal discharge with nitrazine paper, and
observing for flakes of vernix are appropriate assessments for
determining whether a client has ruptured membranes.
184.
Answer: (C) Monitor partial pressure of oxygen (Pao2)
levels. Monitoring PaO2 levels and reducing the oxygen
concentration to keep PaO2 within normal limits reduces the risk of
retinopathy of prematurity in a premature infant receiving oxygen.
Covering the infants eyes and humidifying the oxygen dont reduce
the risk of retinopathy of prematurity. Because cooling increases the
risk of acidosis, the infant should be kept warm so that his
respiratory distress isnt aggravated.
185.
Answer: (A) 110 to 130 calories per kg. Calories per kg is
the accepted way of determined appropriate nutritional intake for a

newborn. The recommended calorie requirement is 110 to 130


calories per kg of newborn body weight. This level will maintain a
consistent blood glucose level and provide enough calories for
continued growth and development.
186.
Answer: (C) 30 to 32 weeks. Individual twins usually grow at
the same rate as singletons until 30 to 32 weeks gestation, then
twins dont gain weight as rapidly as singletons of the same
gestational age. The placenta can no longer keep pace with the
nutritional requirements of both fetuses after 32 weeks, so theres
some growth retardation in twins if they remain in utero at 38 to 40
weeks.
187.
Answer: (A) conjoined twins. The type of placenta that
develops in monozygotic twins depends on the time at which
cleavage of the ovum occurs. Cleavage in conjoined twins occurs
more than 13 days after fertilization. Cleavage that occurs less than
3 day after fertilization results in diamniotic dicchorionic twins.
Cleavage that occurs between days 3 and 8 results in diamniotic
monochorionic twins. Cleavage that occurs between days 8 to 13
result in monoamniotic monochorionic twins.
188.
Answer: (D) Ultrasound. Once the mother and the fetus are
stabilized, ultrasound evaluation of the placenta should be done to
determine the cause of the bleeding. Amniocentesis is
contraindicated in placenta previa. A digital or speculum
examination shouldnt be done as this may lead to severe bleeding
or hemorrhage. External fetal monitoring wont detect a placenta
previa, although it will detect fetal distress, which may result from
blood loss or placenta separation.
189.
Answer: (A) Increased tidal volume. A pregnant client
breathes deeper, which increases the tidal volume of gas moved in
and out of the respiratory tract with each breath. The expiratory
volume and residual volume decrease as the pregnancy progresses.
The inspiratory capacity increases during pregnancy. The increased
oxygen consumption in the pregnant client is 15% to 20% greater
than in the nonpregnant state.
190.
Answer: (A) Diet. Clients with gestational diabetes are
usually managed by diet alone to control their glucose intolerance.
Oral hypoglycemic drugs are contraindicated in pregnancy. Longacting insulin usually isnt needed for blood glucose control in the
client with gestational diabetes.
191.
Answer: (D) Seizure. The anticonvulsant mechanism of
magnesium is believes to depress seizure foci in the brain and

peripheral neuromuscular blockade. Hypomagnesemia isnt a


complication of preeclampsia. Antihypertensive drug other than
magnesium are preferred for sustained hypertension. Magnesium
doesnt help prevent hemorrhage in preeclamptic clients.
192.
Answer: (C) I.V. fluids. A sickle cell crisis during pregnancy is
usually managed by exchange transfusion oxygen, and L.V. Fluids.
The client usually needs a stronger analgesic than acetaminophen
to control the pain of a crisis. Antihypertensive drugs usually arent
necessary. Diuretic wouldnt be used unless fluid overload resulted.
193.
Answer: (A) Calcium gluconate (Kalcinate). Calcium
gluconate is the antidote for magnesium toxicity. Ten milliliters of
10% calcium gluconate is given L.V. push over 3 to 5 minutes.
Hydralazine is given for sustained elevated blood pressure in
preeclamptic clients. Rho (D) immune globulin is given to women
with Rh-negative blood to prevent antibody formation from RHpositive conceptions. Naloxone is used to correct narcotic toxicity.
194.
Answer: (B) An indurated wheal over 10 mm in diameter
appears in 48 to 72 hours. A positive PPD result would be an
indurated wheal over 10 mm in diameter that appears in 48 to 72
hours. The area must be a raised wheal, not a flat circumcised area
to be considered positive.
195.
Answer: (C) Pyelonephritis. The symptoms indicate acute
pyelonephritis, a serious condition in a pregnant client. UTI
symptoms include dysuria, urgency, frequency, and suprapubic
tenderness. Asymptomatic bacteriuria doesnt cause symptoms.
Bacterial vaginosis causes milky white vaginal discharge but no
systemic symptoms.
196.
Answer: (B) Rh-positive fetal blood crosses into
maternal blood, stimulating maternal antibodies. Rh
isoimmunization occurs when Rh-positive fetal blood cells cross into
the maternal circulation and stimulate maternal
antibody production. In subsequent pregnancies with Rh-positive
fetuses, maternal antibodies may cross back into the fetal
circulation and destroy the fetal blood cells.
197.
Answer: (C) Supine position. The supine position causes
compression of the clients aorta and inferior vena cava by the fetus.
This, in turn, inhibits maternal circulation, leading to maternal
hypotension and, ultimately, fetal hypoxia. The other positions
promote comfort and aid labor progress. For instance, the lateral, or
side-lying, position improves maternal and fetal circulation,
enhances comfort, increases maternal relaxation, reduces muscle

tension, and eliminates pressure points. The squatting position


promotes comfort by taking advantage of gravity. The standing
position also takes advantage of gravity and aligns the fetus with
the pelvic angle.
198.
Answer: (B) Irritability and poor sucking. Neonates of
heroin-addicted mothers are physically dependent on the drug and
experience withdrawal when the drug is no longer supplied. Signs of
heroin withdrawal include irritability, poor sucking, and restlessness.
Lethargy isnt associated with neonatal heroin addiction. A flattened
nose, small eyes, and thin lips are seen in infants with fetal alcohol
syndrome. Heroin use during pregnancy hasnt been linked to
specific congenital anomalies.
199.
Answer: (A) 7th to 9th day postpartum. The normal
involutional process returns the uterus to the pelvic cavity in 7 to 9
days. A significant involutional complication is the failure of the
uterus to return to the pelvic cavity within the prescribed time
period. This is known as subinvolution.
200.
Answer: (B) Uterine atony. Multiple fetuses, extended labor
stimulation with oxytocin, and traumatic delivery commonly are
associated with uterine atony, which may lead to postpartum
hemorrhage. Uterine inversion may precede or follow delivery and
commonly results from apparent excessive traction on the umbilical
cord and attempts to deliver the placenta manually. Uterine
involution and some uterine discomfort are normal after delivery.
1. After business trip to an underdeveloped country 3 weeks
ago, Chris is diagnosed with hepatitis A. In completing the
assessment, the nurse might expect which of the following
responses to be most likely associated with the client's
contracting of the disease?
a) I went hunting to forests and swarmed by mosquitoes
b) three months ago, I ate oysters in Kenya
c) I drank lemonade from the roadside while on this trip
d) my business partner is a hepatitis carrier
2. A client with viral hepatitis has no appetite, and food
makes the client nauseated. Which of the following nursing
interventions are appropriate?
a) explain that high fat diets are usually better tolerated
b) encourage foods high in protein
c) explain that the majority of calories need to be consumed
in the evening hours
d) monitor of fluid and electrolyte imbalance
3. Which of the following outcomes would the nurse expect

to find in the client who has developed no complications


from viral hepatitis?
a) decreased absorption of Vitamin K in intestine
b) increasing prothrombin time values
c) presence of asterexis
d) decrease in AST
4. The client has an order to receive purified protein
derivatives (PPD) 0.1ml intradermally. The nurse
administers the medication utilizing a tuberculin syringe
with a:
a) 26G, 5/8 inch needle inserted almost parallel to the skin
with the bevel side up
b) 26G, 5/8 inch needle inserted at a 45deg. angle with
bevel side up
c) 20G, 1 inch needle inserted almost parallel to the skin
with the bevel side up
d) 20G, 1 inch needle inserted at a 45deg. angle with bevel
side up
5. The nurse reading the PPD skin test for a client with no
documented health problems. The site has no induration
and a 1mm area of ecchymosis. The nurse interprets that
the result is:
a) positive
b) negative
c) needs to be repeated
d) borderline
6. The client who is HIV positive has had a Mantoux test.
The result shows a 7mm area of induration. The nurse
evaluates that this result as:
a) negative
b) borderline
c) positive
d) needs to be repeated
7. The nurse reads the client's Mantoux skin test as
positive. The nurse notes that the previous tests were
negative. The client becomes upset and asks the nurse
what this means. The nurse response is based on the
understanding that the client has:
a) no evidence of tuberculosis
b) client has systemic tuberculosis
c) pulmonary tuberculosis
d) exposure to tuberculosis
8. The nurse is caring for the client diagnosed
with tuberculosis. Which of the following assessments, if
made by the nurse, are not consistent with the usual clinical
presentation of tuberculosis?

a) nonproductive or productive cough


b) anorexia and weight loss
c) chills and night sweats
d) high grade fever
9. The nurse is teaching the client with TB about dietary
elements that should be increased in the diet. The nurse
suggests that the client increase intake of:
a) meats and citrus fruits
b) grains and broccoli
c) eggs and spinach
d) potatoes and fish
10. The client with TB, whose status is being monitored in
an ambulatory care clinic, asks the nurse when it is
permissible to return to work. The nurse replies that the
client may resume employment when:
a) two sputum cultures are negative
b) five sputum cultures are negative
c) a sputum culture and a chest x-ray is negative
d) a sputum culture and a PPD test are negative
11. The client with TB is being started on anti-TB therapy
with Isoniazid (INH). The nurse assesses that which of the
following baseline studies has been completed before
giving the client the first dose?
a) coagulation times
b) electrolytes
c) serum creatinine
d) liver enzymes
12. The nurse has given the client
with tuberculosis instructions for proper handling and
disposal of respiratory secretions. The nurse evaluates that
the client understands the instruction if the client verbalizes
to:
a) wash hands at least four times a day
b) turn the head to the side if coughing or sneezing
c) discard the used tissues in the plastic bag
d) brush the teeth and rinse the mouth once a day
13. The client has been taking Isoniazid for month and a
half. The client complains to the nurse about numbness,
paresthesias, and tingling in the extremities. The nurse
interprets that the client is experiencing:
a) small blood vessel spasm
b) impaired peripheral circulation
c) hypercalcemia

d) peripheral neuritis
14. The client with AIDS is diagnosed with cutaneous
Kaposi's sarcoma. Based on this diagnosis, the nurse
understands that his has been determined by which of the
following?
a) appearance of reddish blue lesions noted on the skin
b) swelling in the lower extremities
c) punch biopsy of the cutaneous lesions
d) swelling in the genital area
15. Which of the following individuals is least likely at risk
for the development of Kaposi's sarcoma?
a) a man with a history of same sex partners
b) a renal transplant client
c) a client receiving antineoplastic medications
d) an individual working in an environment where exposure
to asbestos exists
16. The clinic nurse assesses the skin of the client with a
diagnosis of psoriasis. Which of the following
characteristics is not associated with this skin disorder?
a) discoloration and pitting of the nails
b) silvery white, scaly patches on the scalp, elbows, knees,
and sacral regoins
c) complaints of pruritus
d) red purplish, scaly lesions
17. The nurse is assigned to care for a client with herpes
zoster. Which of the following characteristics does the
nurse expect to note when assessing the lesions of this
infection?
a) a generalized body rash
b) small, blue-white spots with a red base
c) a fiery red, edematous rash on the cheeks
d) clustered skin vesicles
18. The nurse manager is panning in the clinical
assignments for the day. Which of the following staff
members would not be assigned to the client with herpes
zoster?
a) the nurse who had chicken fox during child hood
b) the new nurse who never had german measles
c) the nurse who never had enteric fever
d) the new nurse who had flu vaccine
19. The nurse plans to instruct a client with candidiasis
(thrush) of the oral cavity about how to care for the disorder.
Which of the following is not a component of instructions?
a) to rinse the mouth four times daily with a commercial

mouthwash
b) to avoid spicy foods
c) to avoid citrus fruits and hot liquids
d) to eat foods that are liquid
20. The clinic nurse inspects the skin of client suspect of
having scabies.Which of the following
assessment findings would the nurse note if this disorder
were present?
a. the appearance of vesicles or pustules with a thick,
honey colored crust
b) the presence of white patches scattered about the trunk
c) multiple straight or wavy, threadlike lines beneath the
skin
d) patchy hair loss and round neck macules with scales

ANSWERS
1) C ..... 2) D ..... 3) D ..... 4) A ..... 5) B
6) C ..... 7) D ..... 8) D ..... 9) A ..... 10) A
11) D ..... 12) C ..... 13) D ..... 14) A ..... 15) C
16) D ..... 17) D ..... 18) B ..... 19) A ..... 20) C
Situation: Bobby, 3 years old, from a family of four older
siblings, a mother and a father, was admitted to the hospital
with a temperature of 101 F, a sore throat of four days
duration, with difficulty of swallowing, and a moderate
cervical adenitis. A diagnosis of pharyngeal diphtheria was
made.
61. The best means for providing complete bed rest for the
child include:
a) feeding the patient
b) giving sedation
c) providing quiet diversion
d) giving complete physical care
62. A positive nose and throat culture in a member of the
family in whom no signs of the disease are present would
indicate:
a) that he is developing diphtheria
b) that he is a carrier
c) that he is susceptible to diphtheria
d) that he is immune to diphtheria
Situation: Charles, an 8 year old boy, one of several
siblings in the family, is kept home by his mother because
he has a temperature of 100.2F and a history of hacking

cough for two weeks. A definite whoop has been heard and
the doctor made a diagnosis of whooping cough. The
visiting nurse is to instruct the mother in proper care of this
patient.
64. This young boy should be:
a) kept in warm, dry room
b) placed in a sunny room
c) kept in a room with a temperature of 60F
d) allowed free run of the house
65. The organism causing whooping cough is:
a) hemolytic streptococcus
b) hemophilus influenza
c) hemophilus pertussis
d) streptococcus albus
66. Whooping cough is most contagious:
a) in the paroxysmal stage
b) in the catarrhal stage
c( in the febrile stage
d) in the incubation stage
67. During the second stage of the disease the
characteristic paroxysm is frequently accompanied by:
a) involuntary micturation
b) spontaneous epistaxis
c) inspiratory whoop
d) expulsion of mucous from the trachea
68. The diagnosis of pertussis is confirmed by which of the
following tests?
a) nose and throat culture
b) nasopharyngeal culture
c) "rapid" culture of nose and throat organism
d) sputum culture
69. Oxygen is used in whooping cough for which of the
following reasons:
a) prevents the pertussis organism from multiplying
b) seems to lessen the occurrence of paroxysms
c) reduces the danger of convulsions
d) prevents atelectasis
70. One of the first signs of mumps is:
a) pain in the back
b) diarrhea
c) pain in the region of the parotid gland
d) otitis media

71. A nurse should suspect meningitis if a patient exhibited


characteristic position which would be:
a) knees pulled up
b) joints of hands flexed
c) backward arching of the spine
d) head pulled forward onto the chest
72. The nurse should be alert for complications
of meningitis. Which of the following are the most common?
a) hydrocephalus
b) damage of the optic nerve
c) deafness
d) myocarditis
73. Which of the following is not a part of the role of the
infection control department?
a) reviewing infection control policies and procedures
b) performing venipunctures on clients to obtain blood
cultures
c) investigating outbreaks of infection
d) providing input regarding the selection of patient care
products
74. The primary human reservoir for Staphylococcus
aureus is:
a) adult genitalia
b) blood
c) skin
d) feces
75. The best example of medical asepsis is:
a) hand washing
b) use of sterile instruments
c) drawing medication into syringe
d) autoclaving
76. Which of the following is not a symptom of localized
infection?
a) redness
b) edema
c) restricted movement of the affected body part
d) lymph node enlargement
77. All of the following are part of the nurse's role in
infection control except:
a) recognizing the signs and symptoms of infection
b) collecting specimens of drainage from infected wound

sites
c) deciding upon the appropriate to be antibiotic to be
administered to the client
d) supporting the client's body defense mechanisms
78. When giving a history, the patient reports that he had
discharge from his penis and burning when he urinates. A
nurse should recognize these as indicative of:
a) herpes
b) chlamydia
c) syphilis
d) HIV infection
79. When insect serves as the host for transmission, this is
called
a) airborne
b) vectorborne
c) vehicle
d) contact
80. When a health care worker is washing her/his hands,
which of the following observations made by the nurse
would indicate that the worker understands the principle of
proper hand washing?
a) washes hands prior to removing gloves
b) wash hands for 5 seconds
c) rinses hands with fingers pointed up
d) removes rings before washing

ANSWERS
61) B ..... 62) B ..... 63) ..... 64) A .....65) C
66) B .....67) C ..... 68) A ..... 69) B ..... 70) C
71) D ..... 72) C ..... 73) D ..... 74) C ..... 75) A
76) D ..... 77) C ..... 78) A ..... 79) B ..... 80) D
Infectious Agent or its toxic products - AGENT
Directly or Indirectly - MODE OF TRANSMISSION
Person, Animal or Intermediate Vector HOST
Environment - ENVIRONMENT
ECOLOGIC TRIAD OF DISEASE
Agent element, substance, animate or inanimate
that may serve as stimulus to initiate a disease
process
Host organism that provides nourishment for

another organism
Environment physical (climate), biological (plants
& animals)
CONTAGIOUS VS. INFECTIOUS
Contagious
Diseases that are easily spread directly transmitted
from person to person (direct contact) through an
intermediary host
Infectious
Diseases that caused by a pathogen not transmitted
by ordinary contact but require a direct inoculation
through a break in the skin or mucous membrane.
NOTE: ALL CONTAGIOUS DISEASE ARE INFECTIOUS
BUT INFECTIOUS DISEASE IS NOT ALWAYS
CONTAGIOUS
What is Infection?
INFECTION - "the state or condition in which the
body or part of the body is invaded by a pathogenic
agent ( bacteria, virus, parasites etc.) which under
favorable conditions multiplies and produces effects
which are injurious"
Infectious Agent
A. RESIDENT ORGANISMS
deeply seated in the epidermis, not easily removed
by simple handwashing,
Ex: Staphylococci
B. TRANSIENT ORGANISM
represent recent contamination,
survive for a limited period of time, acquired during
contact with the infected colonized patient or
environment,
easily removed by good handwashing
Ex: ( Klebsiella & Pseudomonas)
Infectious Agent
Bacteria heama organism, systemic
Virus nuero organism, systemic
Fungi skin organism, local
Protozoa GI organism, local
Infectious Agent
FACTORS THAT AFFECTS THE AGENT TO
DEVELOP A DISEASE
Pathogenicity ability to cause a disease
Infective dose no of organism to initiate infection
Virulence ability to enter or move through tissues
Specificity ability of the organism to develop
antigens

STAGES OF INFECTIOUS PROCESS


Means of Transmission
1. CONTACT - most common means of transmitting
microorganisms from one person to another.
A. Direct Contact (person to person)
occurs when one person touches another
best vehicle is the Hands especially those of the
Health Care workers
Indirect Contact (inanimate object)
- occurs when a person touches an inanimate object
contaminated by an infected patient
2. AIRBORNE
- droplet, dust, organisms in env.
3. VECTOR - insects or animals
4. VEHICLE
- food (salmonella), water (shigellosis), blood (Hepa
B), medication ( contaminated infusion)
PREVENTION OF
COMMUNICABLE DISEASE
Prevention is worth a pound than cure
PREVENTION OF
COMMUNICABLE DISEASE
Health Education primary role of the nurse
Specific Protection- handwashing, use of protective
devices
Environmental Sanitation clean and conducive for
health
Definition of Prevention
Actions aimed at eradicating, eliminating, or
minimizing the impact of disease and disability. The
concept of prevention is best defined in the context
of levels, traditionally called primary, secondary,
and tertiary prevention
A Dictionary of Epidemiology, Fourth Edition
by John M. Last
Prevention of Needlestick Injuries
Dispose Used Needles in Puncture Proof Needle
Containers
Dont Recap Needles (Unless using the One-handed
Technique)
Use Gloves When Handling Needles (Wont Prevent
Injuries but May Lessen Chance of Transmitting
Diseases)

CONTROL OF C0MMUNICABLE DIESEASE


1. Notification
2. Epidemiological Investigation
3. Case finding; early dx and prompt treatment
4. Isolation and Quarantine
5. Disinfection; disinfestation
6. Medical Asepsis
a. Handwashing
b. Concurrent disinfection
c. Personal protective equipments (PPEs)
d. Barrier Cards/Placarding
Objectives of CCD
Restoration of health, reduce deaths and disability
Interpretation of control measures to IFC for
practice to prevent spread of CD.
Promotion of health and prevention of spread of CD
Diseases that require weekly monitoring:
1. Acute flaccid paralysis (AFP) polio
2. Measles
3. Severe acute diarrhea (SAD)
4. Neonatal tetanus
5. AIDS
Diseases that require reporting w/in 24 hrs
1. Acute flaccid paralysis (AFP) polio
2. Measles
Diseases targeted for eradication
1. Acute flaccid paralysis polio
2. Neonatal tetanus
3. Measles
4. Rabies
Epidemiology
Study of the occurrence and distribution of diseases
in the population
Patterns of occurrence of disease frequency of
disease occurrence
Sporadic
On and off occurrence of the disease
Most of the time it is not found in the community
One or two cases that occur are not related
Endemic
Persistently present in the community all year round
Ex: malaria in Palawan
Epidemic
An unexpected increase in the number of cases of
disease
Pandemic
Epidemic of a worldwide proportions
Time Related Patterns of Occurrence

cyclical variation
a periodic increase in the number of cases of a
disease
a seasonal disease, an increase is expected or there
is usual increase- dengue fever during rainy seasons
are increased but it is not considered an epidemic
because it is expected to rise at this particular time
hot spot-a rising increase that may lead to an
epidemic
Time Related Patterns of Occurrence
Short time fluctuation
A change in the frequency of occurrence of a
disease over a short period of time
Maybe (+) or (-)
Secular variation
A change in the frequency of occurrence of a diseae
taking place over a long period of time
Ex: a.) the change in the pattern of occurrence of
polio after being eradicated in 2000, then sudden
repport of cases in 2001 due to mutant restraints.
b.) small pox virus-eradicated in 1979 (last case
reported) and no another incidence as of today
Types of Epidemiology
Descriptive Epidemiology - concerned with disease
frequency & distribution
Analytic Epidemiology
Is a study of the factors affecting occurrence and
distribution of the disease.
Ex. Epidemiologic investigation
Therapeutic/Clinical
Study of the efficacy of a treatment of a particular
disease
Ex. Clinical trial of a newly proposed therapeutic
regimen
Evaluation Epidemiology
Study of the over-all effectiveness of a total/
comprehensive public health program.
Ex. Evaluation of the under five clinic
Note: We make use of the epidemiology in CHN in
order to come up a community diagnosis and also
to determine the effectiveness of a particular
treatment
Types of Epidemiologic Data
Demographic data
Demography is the study of population groups
Ex. Population size and distribution
Vital Statistics
Environmental data
Health services data
Ex. Ratio between nurse and the population being
served

Ex. Degree of utilization of health facility/ service


Epidemiologic Investigation
1st step- Statement of the problem
2nd step- Appraisal of facts describing the
epidemic in terms of time, place, person.
3rd step- formulation of hypothesis
4th step-Testing the hypothesis
5th step- Conclusion and recommendation
TERMS
Disinfection pathogens but not spores are
destroyed
Disinfectant substance use on inanimate objects
Concurrent disinfection ongoing practices in the
care of the patient to limit or control the spread of
microorganisms.
Terminal disinfection practices to remove
pathogens from the patients environment after his
illness is no longer communicable
FACTORS AFFECTING ISOLATION
Mode of Transmission
Source
Status of the clients defense mechanism
Ability of client to implement precautions
ISOLATION
EPI
Launched by DOH in cooperation with WHO and
UNICEF last July 1976
Objective reduce morbidity and mortality among
infants and children caused by the six childhood
immunizable diseases
PD No. 996 (Sept. 16, 1076) Providing for
compulsary basic immunization for infants and
children below 8 y/o
PP No. 6 (April 3, 1996) Implementing a United
Nations goal on Universal Child Immunization by
1990
RA 7846 (Dec. 30, 1994) immunization hepa B
PD No. 4 (July 29, 1998) Declaring the period of
September 16 to October 14, 1998 as Ligtas Tigdas
Month and launching the Phil Measles Elimination
Campaign
Legislation, Laws affecting EPI
Proclamation No. 46 polio eradication project
Proclamation No. 1064 AFP surveillance

Proclamation No. 1066 National Neonatal Tetanus


Elimination Campaign
EPI
BCG - TB
DPT Diptheria, Pertussis, Tetanu
OPV - Poliomyletis
Hepatitis B
Measles
Immunization
Contraindications
-conditions that require hospitalization
For DPT 2 and 3 history of seizures/ convulsions
within 3 days after the first immunization with DPT
Nursing responsibility: ask how the child reacts to
the first dose
For infant BCG clinical AIDS
The following conditions are NOT contraindications:
Fever up to 38.5 C
Simple or mild acute respiratory infection
Simple diarrhea without dehydration
Malnutrition (it is indication for immunization)
Schedule of immunization
Infant BCG
0 to 11 months or 0 to 1 year
at birth
0.05 ml (dose) ID, right arm
School entrance BCG
When the child enters Grade 1 with or without scar
on the right arm then still go on with the
vaccination except if he is repeating Grade 1
Schedule of immunization
DPT
3 doses, 4 weeks or 1 month interval
Target age: 1 to 11 months but child is eligible up
to 6 years
If 7 years old and above DT only not P
0.5 ml, IM, vastus lateralis
Schedule of immunization
OPV
3 doses, 4 weeks/1 month
Target population: same as above, eligibility until
Grade 6
2-3 drops, oral route
*Feb 8-March 8: Oplan Polio Revival Drive
No side effect, but advise the mother to avoid

feeding the child for 30 minutes after the vaccine, if


vomits within the 30 minute period, repeat the
vaccination
Schedule of immunization
Hepa B
3 doses, 4 weeks
Can be given at birth
Target age 1 to 11 months
0.5 ml, IM, vastus lateralis
Patient may experience local tenderness
Schedule of immunization
Measles
9 to 11 months
Most babies have protection because of maternal
antibodies thus this vaccine is given at 9 months
because the time where the maternal antibodies
wear off, other virus if it still active it will kill the
vaccine
0.5 ml, subcutaneous, any arm
Measles
Fever and measles rash lasting for 1 to 3 days
within 2 weeks after immunization (modified
measles)
Immunization
Fully Immunized Child
when he received all the antigens that should be
given in the first year of life (1 dose BCG, MV; 3
doses DPT, OPV, HB)
Completely Immunized Child
All vaccines given but went beyond 0ne year of age

How do children acquire HIV infections?


*contact with infected mother at birth (90% cases infant)
*Sexual contact with an infected person
*Use of contaminated needles or contact with infected
blood.

How is AIDS cause?


Retrovirus known HIV-1 attacks lymphocytes ( white blood
cells that protect against disease)

How do the doctor determined a person with HIV


treatment?
CD4 cell count

What test is use to test for kids with HIV?

Enzyme linked immunosorbent assay (ELISA) or


Western blot

What are the manifestation of HIV in infancy?


Vague, failure to thrive, lymphadenopathy, chronic
sinusitis, and non-response to the treatment of infections.

What is the opportunistic infection with infants/ HIV?


oral thrush
Pneumocystis carinii or jiroveci
Pneumonia
herpes viruses
cytomegalovirus

What are some serious bacteria infection with kids/ HIV?


meningitis, impetigo, and urinary tract

What are the goal for HIV?


slow virus down
Prevent opportunistic infections
provide adequate nutrition
supportive therapy

What are some common test for HIV


CDC 4 count
Western Blot Test
Rapid HIV test
EIA
ELISA

What are the most common STI in adolescents?


HPV ( human papillomavirus)
can cause cervical cancer

When does infant become clear of the antibodies?


about 15 months

If a pregnant have STI what happen to the baby?


blindness, birth defects or death

What do a nurse need to with a adolescence who has STI?


Asses their level of knowledge and provides information at
an understandable level.

What are the manifestation of HPV


clusters of flesh colored cauliflower shape warts in the
perineal area. Contact with warts spreading the condition.

How to prevent HPV?


2 vaccines Guardasil-6/11 16/18
bivalent Cervatrix 16/18
3 dose regime
9 and 26 years of age

What are the treatments of warts?


cryotherapy or electrocautery
topical with podofilox gel
sinecatechins oinment

What are the nursing goal for kids with STI?


-provide anticipatory guidance
-prevent infection
-identify early symptoms and provide prompt treatment if
infections occur
-To prevent sequelae

What is the nursing intervention for Sequelae?


Discuss the possible complication of specific disorders such
as birth defects and infertility

What are the nursing intervention for medication with a


child with STI?
If taking tetracycline 1 hour before or 2 hour after meals
( empty stomach) avoid dairy products, antacids, iron, and
sunlight.

Which communicable disease is highly contagious?


Smallpox (variola).
No longer recommended unless bio terrorism event

What vaccination should a child should not receive if they


have HIV?
Varicella and MMR live vaccines.

What organ does Hep B affect?


Liver
Incubation period 30-180 days
S/S- fever, anorexia, headache, abdominal pain, malaise,
jaundice, dark urine, chalklike stool

What are the nursing intervention for a child with Hep B?


Prevent contact with blood or blood products
Identify high risk mothers and newborns.
Educate concerning need for vaccination.

What to know about whooping cough?


s/s- cold and cough ( noisy gasp for air that creates a
whoop)
5- 21 days for incubation
vaccination serious of DTAP
Treatment- administer erythromycin. cool mist tent and
antibiotics

What are the nursing intervention with a child with


whooping cough?
* Droplet precautions until 5 days after antibiotics
* bed rest/ abdominal support during coughing spell
* Re-feed child if she vomits
*observe for airway obstruction and o2 stat

What medicine is giving if a child have TB?


isoniazid (INH)
rifampin
pyrazinamide all for several months

What to know about TB?


low grade fever, malaise, anorexia, weight loss, cough,
night sweats.
asymptomatic adenopathy pneumonia, and positive test
2-10 weeks airbone and droplet precaution.

What are some important factors in host resistance to


disease?
intact skin and mucous membrane
phagocytes
function of the immune system

What is use to prevent lockjaw?


tetanus serum/ passive immunity

What is example of a disease spread by a carrier?


Typhoid fever

What is the purpose of medical aseptic techniques


prevent the spread of infection from on person to another

Contact precautions
skin to skin
Example: RSV Hep A, impetigo

Airborne infection
TB, chicken pox ( Varicella) , measles ( rubeola),

What does newborn infants presence when born?


passively acuired

What immunization is giving to a newborn before


discharge?
Hep B

Droplets precaustion
when a patient cough and sneezing
Influenza, Pertussis.

When giving MMR, varicella, influenza nurse should check


for what?
Allergic to eggs.

Why might a parent does not want her child to receive


vaccines?
contains heavy metals and antibiotic or animal product

What can a nurse use to reduce pain before giving a shot?


vapocoolant spray or EMLA cream
What is the criteria for diagnosing full blown AIDS?
person is HIV positive and:
-person's T cell count is 200 or below AND/OR
-presence of an opportunistic infection

How does the virus attack the body?


attaches and takes control of the T cells in order to produce
more HIV; weakens the body's ability to fight infection

How is HIV transmitted?

-having sexual contact with an infected person


-sharing needles with an infected person
-an infected female who gives birth

Which cells does HIV attack?


T cells

What fluids transmit HIV?


-semen
-blood
-vaginal fluid
-breast milk

How do we test for HIV?


blood test for the antibodies to HIV; may take 3-6 months
for a positive result

How is HIV/AIDS treated?


AZT=slows rate at which HIV multiplies; protease inhibitors
decrease the amount of HIV and increase T cell count

How can you reduce your risk?


-abstinence
-monogamy
-don't share needles
-follow universal precautions

Can you develop an immunity to STD's?


No

How are most sexually transmitted diseases passed?


intimate sexual contact, vaginal/oral/anal sex,
outercourse(close genital contact), mucus membrane to
mucus membrane

What are infectious/communicable diseases?


transmitted from person to person by microorganisms
(pathogens) or other toxins

What are the 4 main types of pathogens?


virus
bacteria
fungus
protozoa

What are places of exit and entry for pathogens(body


orifices)?
eyes, nose, ears, mouth, penis, vagina, anus, broken skin

What are the 5 methods of transmission?


-direct contact
-through air
-contact with contaminated objects
-contact with animals, insects that are vectors or carriers of
disease
-contaminated food and water

What are the 4 body defenses?


skin
cilia
mucus
immune system

What are the 2 types of immunity?


active and passive
(short term and long term)

What are the 5 stages of infectious disease?


incubation stage
prodromal stage
acute stage
recovery stage
convalescence stage

What are the 6 ways to prevent communicable disease?


-kill the agent/pathogen
-prevent contact with reservoir
-prevent escape
-prevent transmission
-block places of entry
-resistant host
how many people in the sub-saharan desert are infected
with HIV
25 million

what is the average lifespan of a person living in the subsaharan desert


mid 40's

how many new cases a year in U.S.


40,000

how many new cases a day in Africa


7,000

what drugs can be used to extend a persons with HIV's


life by over 20 years
ARV

what percent of infected children recieve medical care


5-10 percent
communicable disease
an illness that can spread, usually from person to person

pathogen
an agent that causes a communicable disease

ear infections,whooping cough


What are some diseases caused by bacteria?

viruses
______ are smaller than bacteria and cannot reproduce on
their own.

cancer
the uncontrolled growth of abnormal cells

person's behavior
A ________ can affect the risk of developing cancer.

cured
Some noncommunicable diseases can be _____.

arthrosclerosis
a disease in which fatty substances build up on the inside
of arteries

colds, mumps, chicken pox, flu,


What are some diseases caused by viruses?

fungi

Which kind of disease causes athlete's foot?

protist
Which kind of disease causes malaria?

person to person, contaminated hands , sneezing,


coughing, contaminated water and animal bites
How can communicable diseases spread?

skin
What is a major line of defense to keep pathogens out?

vaccine
A _______ is a dose of weakened or killed pathogens of one
kind.

cardiovascular disease
a disease of the heart and blood vessels

emphysema
a disease of the lungs in which air sacs are damaged

early diagnosis
What is the key to curing, controlling, damage done by
noncommunicable diseases?

immunity
the body's resistance to a disease through the presence of
antibodies

white blood cells


What destroys the pathogens?

antibody
a substance made by certain white blood cells that
attaches to a pathogen and makes it harmless

vaccines
What is one way to develop immunity without getting sick?

smoking
What is the main cause of emphysema?

antibiotic
a medicine that kills bacteria

bacteria
Doctors give you an antibiotic when you have a disease
caused by a ________.

non-communicable disease
an illness that cannot spread and is not caused by a
pathogen

heredity
What is the main causes of noncommunicable diseases?

allergy
harmful reaction to certain substances

asthma
allergy that affects the respiratory system and may result
in breathing difficulty and wheezing

infectious dieseases
Communicable diseases are sometimes called _______,
infections, or contagious.

bacteria, viruses, protists, and fungi


Pathogens include ______, ______, _____, and ______. They
are sometimes called germs.

nose and mouth


Pathogens can enter the body through the _____and _____.

allergic reaction
All of the following are examples of _______:
itchy skin, rash, hives, stomach upset, swelling of lips or
tongue, sneezing

pollen
What triggers hay fever?
Pathogens
Disease causing microorganisms

Virus

Smallest of all pathogens that need a host cell to


reproduce. They invade cells, multiple inside the cell to the
point it explodes and releases millions more.

Bacteria
One-celled organisms that multiply rapidly and make up
sick by releasing toxins.

Fungi
Feed on both living and dead organic matter. Most are
harmless but some can cause infections such as ringworm
and athlete's foot.

Protozoa (Protists)
Single-celled organisms that can have parasitic like
characteristics and may contain flagellum.

Symptoms
A physical or mental feature- fever, swelling, rash, aches,
runny nose, cough, headache- that indicates a specific
infection

Contagious Period
Length of time that a disease can spread from person to
person

Infectious Diseases
Often called communicable diseases as they can be
passed from one person to the next.

Food, Water, Soil


When food is not prepared properly pathogens can survive
and enter the body orally, drinking or using contaminated
water without purifying it first can spread pathogens via
ingestion, pathogens in soil can get into body via open
cuts or ingestion of improperly prepared or clean food,
sharing food & drinks with infected people can also spread
pathogens

Infected animals
When pathogens spread diseases through bites

Direct Contact (Infected Individuals)

Occurs when there is physical contact with body fluidssaliva, blood, vomit, urine, feces, nasal secretionsbetween an infected person and a susceptible person via
touching, kissing & other sexual contact, direct contact
with body fluids, or contact with open wounds.

Indirect Contact (Contaminated objects)


No human-to-human contact is involved, spread when an
infected person sneezes or coughs, sending infectious
droplets into the air. A person can inhale the infectious
droplets, the infectious droplets can land directly on a
person or on fomites that people touch the with their
hands, and then touch their eyes, nose or mouth.

Agent
Germ that produced infection- Bacteria, Virus, Fungi,
Protozoa

Reservior
Place germ can live such as humans, animals, soil, objects,
air, water and food

Place of Exit
Where germ leaves reservoir such as through the mouth,
nose, rectum, genitals and wounds

Method of Transmission
How germ travels from point A to point B such as through
saliva, sneeze, wind, feces, urine, blood, water, soils,
kissing, infected sore

Place of Entry
Place where germ enters a person such as breathing in
through nose & mouth, germs in food- mouth and
digestive, cuts in skin, direct contact

Susceptible Host
Things that make a person more likely to get a disease.
These include behaviors (taking care of cuts, nutrition,
exercise, sleep, hygiene, abstaining from smoking and
drinking), conditions (receiving Chemo, AIDS/HIV+),
environment and inherited factors.

Skin

Outer layer is tough and dead which keeps germs from


entering. Cells also fall off taking germs with them

Hair
Around eyes and nose they trap germs and keep germs
from entering the body. The trachea has cilia that keep
germs from entering the lungs.

Tears
Wash away germs from eyes

Mucus
Sticky substance that exists in the nose & other places that
traps germs and contains chemicals defenses that attack
and destroy germs.

Stomach acid & Saliva


These substances have chemicals that kill most of the
germs that enter your body

Antibodies
Specialize proteins that lock onto specific antigens and
mark invaders for other cells to destroy

Leukocytes
WBC or White Blood Cells

Phagocytes
Attach/engulf invaders

Lymphocytes
Remember & help destory

T lymphocytes
Destroy invaders

B lymphocytes
Seek out targets, produce antibodies, and send defenses

Antigens
Surface structures or markings that antibodies attach onto

Innate Immune System

Comprised of your body's physical barriers plus


inflammation due to increased lymph and blood flow to
areas of infection where phagocytes (macrophages)
devour toxins or pathogens. If the infection becomes
widespread, a person may get a fever which signals the
body to produce more WBC and slows down the
pathogens.

Adaptive Immune System


This type of immunity develops throughout our lives and
involved the WBCs. This can happen through exposure to
diseases and via vaccination.

Macrophage
Engulf invaders and trigger immune response

Vaccination
Contains weakened or dead strain of the organism/virus
which is injected into the body to prepare B-Cells and TCells to remember and destroy it quickly thus giving the
person immunity towards a specific disease and prevents
them from getting sick.

Antibotics
Treatment for bacterial infections
What process enhances topical
corticosteriods efficacy?
Soaking in water for 10-15 minutes

Which corticosteroid is more potent than


the other? ointment or cream?
Ointment more potent

What increases the topical corticosteroid


potency dramatically?
covering with water impermeable barrier
for at least 4 hours.

Caution should be used placing topical


corticosteriods to areas of thin skin (face,
scrotum, vulva, skin folds). T/F
T

How much topical corticosteroid does it


take to cover an adult?
20-30g

Does systemic absorption occur with


topical corticosteroid?
Yes, but adrenal suppression, DM, HTN,
osteoporosis are very rare

Which is the best product t moisturize dry


skin?
Vanicream is allergen free and can be used
if allergic contact dermatitis is suspected.

What can be used as a drying agent for


weepy dermatoses?
water applied as repeated compresses for
15-30 minutes alone or with aluminum
salts

List some topical antipruritics.


Lotions that contain camphor and menthol
Lotions that contain pramoxine
hydrochloride
Hydrocortisone 1% or 2.5% for
inflammation
Doxepin may cause drowsiness
Capsaicin for neuropathic itch
Ice in a plastic bag covered by thin cloth

What is the med of choice for pruritus


when due to histamine such as in
urticaria?
H1 blockers-antihistamines (Hydroxyzine
25-50 mg nightly)

Antihistamines are of limited value for


treatment of pruritus associated with
_____ ____ _____.
inflammatory skin disease

List agents that may treat pruritus.


1. Antidepressants (doxepin, mirtazapine,
paroxetine)
2. Gabapentin, pregabalin, duloxetine

(centrally or peripherally act on neurons)


3. Aprepitant and opioid antagonists
(Naltrexone, butophenol) is effective on
select patients.

What strength SPF should fair


complexioned people use?
SPF 15, better 30-40 every day

Which strength SPF gives protection


against UVA as well as UVB?
SPF >30

Melanocytic Nevi (Normal Mole):


Borders.
Color.
well defined border, single shade of
pigment from beige or pink to dark brown

Define a normal mole.


well demarcated, symmetric uniform in
contour and color

Moles should be removed only if they are


suspected of being melanomas. T/F
T

Describe the criteria for atypical nevi.


1. >6mm
2. ill defined, irregular border
3. Irregularly distributed pigmentation

Melanoma is increased in what


populations?
1. patients with 50 or more nevi with one
or more atypical moles and one mole at
least 8 mm or larger
2. Patients with a few to many definitely
atypical moles
3. Familial melanoma (numerous atypical
nevi and f/h of 2 first degree relatives with
melanoma)-risk of developing melanoma is
50% by age 50.

In patients that are at increased risk for


melanoma, what is the follow up?
education and regular (6-12 month) follow
up

Describe blue nevi.


-small slightly elevated, blue-black lesions
that favor dorsal hands.
-Common in Asians.
-if present without change for many years,
may be considered benign

When do blue-black papules and nodules


have to be ruled out for nodular
melanoma?
when they are new or growing

Describe lentigines. B) How do they


appear on the upper back? C) What do
they look like if they are benign? D) How
treated?
-appear in sun exposed areas, particularly
dorsa of hands, upper back and upper
chest, starting 4th or 5th decade of life an
associated with photoaging and estrogen
and progesterone use
B) Have irregular border
C) pigmentation is homogeneous,
symmetric and flat
D) Topical 0.1% Tretinoin, 0.1% tazarotene,
2% 4-hydroxyanisole with 0.01% tretinoin,
laser or cryo

Describe Seborrheic Keratoses. B) How


treated?
Benign plaques
beige to brown or even black
3-20 mm
velvety or warty surface
appear to be stuck or pasted on skin
common esp. elderly
B) Frozen with liquid nitrogen or curetted if
they itch or inflamed, no treatment needed

A pigmented skin lesion with recent


change in appearance should be a
suspect _______ _____.
malignant melanoma

What are the essentials of diagnosis for


malignant melanoma?
1. flat or raised
2. suspect in any pigmented skin lesion
with recent change in appearance.
3. Varying colors, including red, white,
black, and bluish.
4. Borders typically irregular

What is the single most important


prognostic factor in malignant melanoma?
related to thickness, if <1 mm=95% ten
year survival

What does ABCDE stand for?


A-Asymmetry
B=Border
C=color variation
D=Diameter > 6 mm
E=evolution (history of changing mole)

What is the most important history a


patient can give regarding a mole's
appearance?
changing appearance

A good practice is to refer suspicious


______ lesions.
pigmented

Describe acral lentignious melanoma.


dark, irregularly shaped lesions on the
palms and soles.
Can be broad solitary, darkly pigmented
longitudinal streaks in the nails.

What is the treatment for melanoma?


Excision, requires referral

What medication can reduce the risk of


progression of high risk melanoma?
BB

In general, when treating with topical


corticosteriods, how will you determine
the potency you use?
Appropriate to the severity of the
dermatitis. Begin with Triamcinolone 0.1%
or a stronger corticosteroid then taper to
hydrocortisone or another slightly stronger
mild corticosteroid (alclometasone,
desonide).

What is necessary to avoid rebound flares


of dermatitis that may follow their abrupt
cessation?
tapering the corticosteriod

Which corticosteriods are safe on face


and eyelids?
Protopic (tacrolimus) and Elidel
(pimecrolimus)

Tacrolimus and pimecrolimus is


contraindicated in who?

HIV
iatrogenic immunosuppression
prior lymphoma

What is the maintenance treatment of


atopic dermatitis?
constant application of moisturizers to
prevent flares.
topical anti inflammatories only on
weekends or three times weekly can
prevent flares.

how much area does one FTU treat?


One FTU is enough to treat an area of skin
twice the size of the flat of an adult's hand
with the fingers together.

How many FUTs equal 1 g topical steroid?


Two FTUs are about the same as 1 g of
topical steroid.

What are the 5 characteristics of skin


cancer?
1. asymmetry
2. border irregularity
3. color
4. 1/4 inch diameter
5. evolution

A Vitamin D level above ______ protects us


from the sun and prevents skin, breast
and colon cancers.
50

Which type is best to treat psoriasis?


Ointments or creams?
ointments because they provide a barrier
and moisturize whereas creams can dry it
out

What body part does Clobetasol not get


used on? What is the strength of this
topical steroid?
Face
High potency

You cannot use oral steroids to treat


which skin diseases/conditions?
atopic dermatitis
Psoriasis
places where causative agents can live
reservoir

way for causative agent to escape from the reservoir


portal of exit

humans become a susceptible host when


-large number of the pathogen invade the body
-body defenses are weak

early symptoms. Indicates the start of a disease before


specific symptoms occur
prodromal phase

typical prodromal sx
fever, malaise, HA, lack of appetite, rash

when are you most contagious?


during the prodromal phase or even up to 3 days prior to
prodromal phase

things we need to teach parents about when their kids


are ill
-fever management
-fluid management
-medication
-reduction of disease transmission

hand-foot-mouth disease
-what kind of disease (viral, bac...)
-scientific name
-transmission
-clinical manifestation
-treatment
-viral
-coxsackieviruses A16
-contact with body fluids or droplet
-fever, sore throat, malaise. In 1-2 days herpangina and

blisters on palms and soles


-tx: "magic mouthwash", hydration, analgesics, mouth care

Erythema Infectiosum (Fifth disease)


-what kind of disease (viral, bac...)
-scientific name
-transmission
-clinical manifestation
-treatment
-viral
-human parvovirus B 19
-droplet or contact with blood
-persistent fever for 3-7 days in kid who is otherwise well.
"slapped cheek" appearance, mild URI
-tx: antipyretic, analgesics, anti-inflammatory

this disease begins with a low grade fever HA, mild cold
sx. these pass and the illness seems gone until a fine lacy
rash appears a few days later
fifth disease

Hep A
-what kind of disease (viral, bac...)
-scientific name
-transmission
-clinical manifestation
-treatment
-viral
-Hep A
-food born transmission
-fever, abd pain, N/V, diarrhea, jaundice.
tx: prevention, hydration, antipyretic, analgesics

which hepatitis has no chronic disease state?


hep A

average incubation for Hep A


28 days

Hep B
-transmission
-clinical manifestation
-treatment
-contact with bodily fluids
-jaundice, dark urine, fatigue, abd pain, loss of appetite,
N/V, and joint pain, skin rash

tx: prevention, hydration, antiviral meds, analgesics


acute and chronic states, may be passed mom to baby

can Hep B impact lifespan?


depends on when its contracted. If you get it as a kid, sure,
it can impact your lifespan.

Hep c
-transmission
-clinical manifestation
-treatment
-contact with bodily fluids
-asymptomatic for a very long time. Can lead to liver
cancer and death. Non-specific sx: malaise, fatigue, weight
loss, vague abd pain.
-tx: prevention, antiviral meds, long term management.
may be passed from mom--> baby

Influenza
-what kind of disease (viral, bac...)
-scientific name
-transmission
-clinical manifestation
-treatment
-viral
-influenza A (H1N1) and B
-direct contact
-abrupt fever, progressive URI-like sx, malaise, anorexia
-prevention, antiviral (tammaflu) if detected early,
supportive care

Rubella
-what kind of disease (viral, bac...)
-scientific name
-transmission
-clinical manifestation
-treatment
-viral
-rubella virus
-droplets and contact with bodily fluids
-prodromal phase is ABSENT in kids. See rash starts on
face-->neck, arms, trunk, legs
-prevention, anti-pyretics, analgesics

chicken pox
-what kind of disease (viral, bac...)
-scientific name
-transmission
-clinical manifestation
-treatment
-viral
-varicella-zoster
-direct contact and droplet
-prodromal phase: slight fever and malaise. pruritic rash
begins as macule-->vesicle. pattern is centripetal
appearing on extremities, face
tx: acyclovir 20mg/kg/dose, 4 x day, also prevention,
antihistamines and skin care

when are you no longer contagious with the chicken pox?


-no new lesion and all lesions are scabbed

shingles may develop at any age group but you are more
likely to get it if...
-you are older than 60
-you had chicken pox before the are of 1
-your immune syst is weakened by meds or disease
-you are a teen who was not immunized with two varicella
vaccines

rotovirus
-what kind of disease (viral, bac...)
-scientific name
-transmission
-clinical manifestation
-treatment
-viral
-rotovirus A-G
-contact with bodily fluids
-in kids 3-24mo, it is the most common cause of diarrhea.
also fever, vomiting, dehydration
-tx: prevention, hydration, antipyretics

Polyomyelitis (polio)
-what kind of disease (viral, bac...)
-scientific name
-transmission
-clinical manifestation
-treatment

-virus
-enterovirus
-contact with bodily fluids
-three different forms; abortive/inapparent, non-paralytic,
paralytic
-tx: prevention, bed rest, RT, PT, skin integrity, ADL's

this type of polio you see these sx: sore throat, HA,
vomiting, abdominal pain
abortive/inapparent

in this type of polio you see SEVERE sore throat, HA,


vomiting, abd pain, stiff neck, back and legs
non-paralytic

with this type of polio you see sx of sore throat, HA,


vomiting, stiff neck, back and legs and CNS paralysis with
recovery
paralytic

measles (rubeola)
-what kind of disease (viral, bac...)
-scientific name
-transmission
-clinical manifestation
-treatment
-viral
-mobillivirus
droplet and contact with bodily fluids
-prodromal state: fever and malaise, coryza (stuffy nose),
cough, conjunctivitis. Then Koplick Spots on mucosa, rash
appears on day 3-4 of illness
tx: prevention, vit A, bed rest, antipyretics

when is measles communicable?


4-5 days before rash appears

vit A helps with what communicable disease?


measles

Roseola
-what kind of disease (viral, bac...)
-scientific name
-transmission

-clinical manifestation
-treatment
-viral
-contact with body fluids
-herpesvirus ty 6
-fever >103 for 3-7 days. a macular rash that is blanchable
-tx: antipyretics

mumps (parotitis)
-what kind of disease (viral, bac...)
-scientific name
-transmission
-clinical manifestation
-treatment
-viral
-droplet or direct contact
-paramyxovirus
-fever, HA, swelling of parotid gland on 3d day
tx: prevention, analgesics, antipyretics, hydration, AIRWAY
precautions.

when is mumps contagious?


7 days before parotid swelling until 9 days after swelling
subsides

condyloma/genital warts/HPV
-what kind of disease (viral, bac...)
-scientific name
-transmission
-clinical manifestation
-treatment
viral
-human papillomavirus
-contact with body fluids
-lesions found on butt, vulva, vagina, anus, cervix
-tx: prevention, removal of warts, antimitotic drugs. not
sure if treatment actually reduces transmission...

Haemophilus Influenza Ty B (HIB)


-what kind of disease (viral, bac...)
-scientific name
-transmission
-clinical manifestation
-treatment
-bacterial
-haemophilus influenza ty B

-contact with bodily fluids and droplet


-many ways of expression (epiglottis, septic arthritis,
sepsis, pneumonia [fever, SOB, HA, chills] and meningitis
[fever, HA, NV, stiff neck, sensitive to light])
-tx: prevention, antibiotics, antipyretics, anti-inflammatory

Diptheria
-what kind of disease (viral, bac...)
-scientific name
-transmission
-clinical manifestation
-treatment
-bacterial
-corynebacterium dptheriae
-direct contact with bodily fluids
-URI which progresses to "bull's neck", white/gray mucus
membranes, fever cough
-antibiotics, bed rest, AIRWAY precautions, support

meningococcal meningitis
-what kind of disease (viral, bac...)
-scientific name
-transmission
-clinical manifestation
-treatment
-bacterial
-neisseria meningitidis
-contact with bodily fluids
-fever, HA, stiff neck, NV, photophobia, altered mental
status
-tx: prevention, antipyretics, anti inflamm, anti B

pertussis (whooping cough)


-what kind of disease (viral, bac...)
-scientific name
-transmission
-clinical manifestation
-treatment
-bacterial
-droplet or direct contact with bodily fluids
-catarrhal stage: URI 1-2 wks--> progresses to paroxysmal
stage-short, rapid cough followed by a whoop or gasp for 46 wks. cyanosis may occur
-tx: prevention, suctioning, O2, and hydration

pertussis paroxysmal stage may last 4-6 wks and persists


of 3 stages
1) cold sx 7-10 days
2) violent dry hacking 30 days
3) harsh cough continues

pertussis is most contagious during what stage?


catarrhal

pneumococcal disease
-what kind of disease (viral, bac...)
-scientific name
-transmission
-clinical manifestation
-bacterial
-streptococcus pneumonia
-direct contact with body fluids and droplet
-many manifestations (strep throat, scarletina, scarlet
fever, pneumonia, otis media, sinustitis, localized infec

who is most susceptible to strep bacteria?


6-24 mo

when are we most likely to catch a strep infection?


winter and spring months, crowded physical settings

scarlet fever
-what kind of disease (viral, bac...)
-scientific name
-transmission
-clinical manifestation
-treatment
-bacteria
-group A beta-hemolytic streptococci
-droplets and direct contact with body fluids
-prodromal: abrupt high fever, halitosis, HA enanthema:
tonsils large, edematous, "strawberry tongue, both white
then red...
tx: penicillin, bed rest, analgesics, antipruritis, hydration

tetanus
-what kind of disease (viral, bac...)
-scientific name
-transmission
-clinical manifestation
-treatment

-bacterial
-clostridium tetani
-spores in soil, dust, excretions
-muscle contractions of masseter, neck, and trunk
-tx: immunoglobulin, tetanus toxoid, analgesics

how often should we get a tetanus vaccine?


q 10 yrs

meningitis
onset
clinical manifestations
-tx
rapid onset; bac, viral, fungal, aseptic
-nucchal rigidity, change in mental status, fever, rash
-tx: antipyretics, analgesics, antifungal/antibiotics,
hydration

which type of meningitis has the most rapid onset?


bacterial

who is most susceptible to strep bacteria?


kids 6-12 mo

most frequent infections worldwide?


intestinal parasites

who are the most susceptible to intestinal parasites?


young kids

most common parasites in the US?


pinworms and giardiasis

giardiasis
-what kind of disease (viral, bac...)
-scientific name
-transmission
-clinical manifestation
-treatment
-protozoan
-giardia intestinalis
-direct contact with contaminated water or food
-diarrhea, NV, anorexia, failure to thrive, abd cramps,
constipation, bloating, flatulence
-tx: flagyl, tindamax, and prevention of reoccurance

enterboiasis (pinworms)
-what kind of disease (viral, bac...)
-scientific name
-transmission
-clinical manifestation
-diagnosis
-treatment
protozoan
-nematode enerobius vermicularis
-inhalation or ingestion of eggs from contaminated hands
-itchy butt, restless, bed wetting, short attention span, poor
sleep, possible urethral infection
-dx: tape test
tx: pyrantel pamoate or albendazole x1 then again in 2
wks. treat whole family. prevent occurrence

disorders transmitted to humans via anthropods


rickettsial infection

rickettsial infections are most common in what region?


temperate and tropical climates

lyme disease
-what kind of disease (viral, bac...)
-scientific name
-transmission
-clinical manifestation
-vector borne
-borrelia burhdorferi
-transmitted by deer tick bite
{stage 1: }"bull's eye", fever, HA, malaise
{stage 2:} (1-4mo) rash on hands/feet, fever, HA, malaise,
fatigue, lymphadenopathy, cough, stiff neck
{stage 3:} systemic involvement (2-12mo)
arthritis, CNS changes, cardio complications,
encephalopathy, cognitive and behavioral changes, chronic
arthritis

tx for lyme
>8 yo, doxycycline
<8 yo, amoxicillin

rocky mountain spotted fever


-what kind of disease (viral, bac...)
-scientific name

-transmission
-clinical manifestation
-treatment
-vector borne
-ricketsia ricketsi
-gradual or abrupt onset of fever, malaise, HA, rash on
palms/feet
-tx: antipyretic, analgesic, antibiotics

when taking a hx on a kid suspected of rocky mountian


spotted fever, inquire about
-recent tick bites
-travel to endemic area
-presentation in the spring/fall

Cat scratch disease


-what kind of disease (viral, bac...)
-scientific name
-clinical manifestation
-treatment
-vector born from a cat scratch
-bartonella henselae
-painless, nonpruritic papule, regional lymphadenitis
tx: antibiotics

west nile virus


-what kind of disease (viral, bac...)
-scientific name
-transmission
-clinical manifestation
-treatment
-preventable measures
- vector (mosquito)
-flavivirus
-fever, HA, NV, body aches, seizures, memory loss, brain
damage-esp in kids. most people recover.
-tx: hydration, antipyretic, analgesic,
preventable measures: clothing at dusk, repellents,
eliminate standing h2o

how do lice spread?


from personal contact with infected person. they don't
jump.

the condition of lice is called?


-pediculosis capitis

lice
-scientific name
-clinical manifestation
-treatment
-pediculus humanus capitis
-intense pruritus of scalp, behind ears, nape of neck, nits
(eggs) attached to hair shaft
-pediculicide and removal of nits, education and support.
kid infected, family and home must all be treated.

scabies
-scientific name
-transmission
-clinical manifestation
-treatment
-sarcoptes scabiei
-prolonged close contact. mites burrow into skin and lay
eggs in CLUSTERS
-intense itching, excoriation and burrows, inflammation
btwn fingers, neck folds and groin
tx: scabicide, >2 yr permethrin, wash linen and clothes and
dry high heat, supportive care for itching 2-3 weeks

bedbugs
-scientific name
-transmission
-clinical manifestation
-treatment
-cimex lectularis
-contact with infested mattress. mites burrow and feed on
blood.
-intense itching, all over bites, may progress to
follicularitis/cellulitis. may trigger asthma/anaphylaxis
-identification and eradication of bugs, topical application
of steroids, hygiene of linens and clothing, supportive care
pruritis 2-3 wks

which communicable disease can have conjunctivitis as a


sx?
measles

what causes conjunctivitis?


can be bacterial or viral

must admin antib for how long before a return to school


with conjunctivitis?
24 hour

general term that means swelling/lesions of the mouth


stomatitis

two types of stomatitis


ty1: aphthus ulcers-benign
ty 2: herpetic gingivostomatitis hsv

goals of stomatitis?
NSAIDs, anesthetics, prevent spread by oral secretions and
poor handwashing

4 types of bacterial skin infections


-impetigo
-folliculitis
-cellulitis
-scalded skin syndrome

impetigo rarely results in


scars

what is reye's syndrome?


exact cause in unknown, it is associated with asprin
(ibuprofen) consumption in kids with a viral illness.

early diagnosis of reye's syndrome is vital bc


it can lead to severe injury and death

classic sx or reye's syndrome


rash, vomiting, and liver damage

sx of encephalitis
personality changes, seizures, weakness

what causes encephalitis?


really any of the disease in the lecture

who is most susceptible to encephalitis?


kids

common fungal skin infections


-tinea capitis (scalp)
-tinea corporis (body/nails)
-tinea cruris (groin)
-tinea pedis (feet)
-thrush, candidiasis

tonsillar pharyngeal areas are covered with a white/gray


membrane. possible pronounced "bull's neck".
complications include myocarditis
diptheria

rash appears in phases. phase one is erythema on the


face, chiefly on the cheeks giving a "slapped cheek" look
fifth disease

rash composed of rose-pink macules, first on neck, then


on trunk. rash is non-puritic, and blanchable.
roseola

In this illness, the prodromal phase is a earache that is


aggrivated by chewing. by third day, parotid glands
enlarge.
mumps

Conjunctivitis and Koplik spots are present before the


rash. Rash appears 3-4 days after onset. Erupts first on
the face and then spreads downwards.
measles (rubeola)

discrete pinkish red maculopapular rash appears on face,


then spreads down to neck, arms, trunk and legs. Its very
benign to kid but its greatest danger is teratogenic effect
on fetus
rubella

abrupt high fever, halitosis. then the tonsils enlarge and


become red with patches of exudate. tongue becomes
red and swollen. itchy, peely rash (desquimation) occurs.
scarlet fever

fever, sore throat, malaise. In 1-2 days herpangina and


blisters on palms and soles
hand, foot and mouth disease

communicable disease
...a disease that can be given from one person to another

microbe (microorganism)
...a living thing that cannot be seen with the naked eye;
examples include bacteria and viruses

normal (resident) flora


...the harmless microbes that live in and on the body and
help it to function properly

pathogens
...a microbe that can cause illness

opportunistic microbes
...microbes that are considered normal (resident) flora
when they are in or on one part of the body, but can cause
infection if they move out of that area and into or onto
another part of the body

colonies
...groups of bacteria

aerobic
...bacteria that need oxygen in order to live

anaerobic
...bacteria that can survive withhout oxygen

antibodies
...specialized proteins produced by the immune system
that help our bodies to fight specific microbes, preventing
infection.

multidrug-resistant organisms (MDROs)


...bacteria that are resistant to one or more classes of antimicrobical agents

methicillin-resistant Staphylococcus aureus (MRSA)


...a type of bacteria that has become resistant to
methicillin, a powerful antibiotic.

vancomycin-resistant enterococcus (VRE)

...a type of bacteria that has become resistant to


vancomycin, a powerful antibiotic

infection
...disease caused by pathogenic microbes

chain of infection
...the six key conditions that must be met for a person to
get a communicable infection: pathogen, reservoir, portal
of exit, method of transmission, portal of entry, and a
susceptible host.

contaminated
...describes an object that is soiled by pathogens

fomite
...a non-living object that has been contaminated (soiled)
by pathogens

vector
...a living creature, such as an insect, that can transmit
disease

virulence
...the strength or disease-producing potential of a
pathogen

health-care associated infections (HAIs)


...infections that a patient or resident gets while receiving
treatment in a hospital or other health care facility, or that
health care workers get while performing their duties
within a health care setting.

nosocomial infections
...Infections that a patient or resident get while receiving
treatment in a hospital or other health care facility; a type
of health-care associated infection (HAI)

infection control
...basic practices designed to decrease the chance that an
infection will spread from one person to another in a health
care facility

medical asepsis

...techniques that used to physically remove or kill


pathogens

sanitization
...practices associated with basic cleanliness,such as hand
washing, cleansing of eating utensils and other surfaces
with soap and water, and providing clean linens and
clothing; one of the techniques of medical asepsis

antisepsis
...practices that kill microbes or stop them from growing;
one of the techniques associated with medical asepsis
(compare with sanitization, disinfection, and sterilization)

disinfection
...the use of strong chemicals to kill pathogens on nonliving objects that come in contact with body fluids or
substances, such as bed pans, urinals, and over the bed
tables; one of the techniques of medical asepsis (compare
with sanitization, disinfection, and sterilization)

sterilization
...the process of completely eliminating microbes from the
surface of an object using an autoclave (sterilizer) or
chemicals; (compare with sanitization, disinfection, and
sterilization)

transient flora
...microbes that are picked up by touching contaminated
objects or people who have an infectious disease

personal protective equipment (PPE)


...barriers that are worn to physically prevent microbes
from reaching a health care provider's skin or mucous
membranes, such as gloves, gowns, masks, and protective
eye wear

isolation precautions
...guidelines, based on a pathogen's method of
transmission, that health care workers follow to contain the
pathogen and limit others' exposure to it as much as
possible

standard precautions

...precautions that a health care worker takes with each


patient or resident to prevent contact with bloodborne
pathogens; include the use of barrier methods (such as
gloves) as well as certain environmental control methods

transmission-based precautions
...precautions that a health care worker takes when a
person is known to have a disease that is transmitted in a
certain way; include airborne precautions, droplet
precautions, and contact precautions

airborne precautions
...used when caring for patients or residents infected with
pathogens that can be transmitted through the air; include
placing the patient or resident in a private room with the
door closed, wearing a mask when caring for the patient or
resident, and minimizing the amount of time the person
spends out of his or her private room

airborne infection isolation room (AIIR)


...single-occupancy patient-care room with special
ventilation and filtration systems used to isolate persons
with an airborne infectious disease

droplet precautions
...used when caring for patients or residents infected with
pathogens that can be transmitted by direct exposure to
droplets released from the mouth or nose (for example
when the person coughs, sneezes or talks)

contact precautions
...used when caring for patients or residents infected with
pathogens that can be transmitted directly (by touching
the person), or indirectly (by touching fomites); include
using barrier methods whenever contact with the infected
person or items contaminated with wound drainage or
body substances is necessary

Lippincott's Textbook for Nursing Assistants


ISBN 978-1-60547-635-3
Chapter 7 Communicable Disease & Infection Control
chapter 10 ISBN 978-4511-9466-1
These terms and definitions were taken word-for word from
the Glossary of the textbook.
Pathogens

microorganisms that cause disease. There is a wide range


of them and they are transmitted in a variety of ways. The
means of transmission can be categorised as direct or
indirect.

Callose
is a large polysacchride that blocks old sieve tubes

Tylose
a balloon-like swelling, also blocks old xylem vessels

primary defences
which prevent entry of pathogens into the body

Secondary defences
help to remove a pathogen after it has entered the body.

immune response
the body's response to invasion by pathogens.

specific immune response


means that the response deals only with one pathogen
that possesses one particular antigen.

Blood clots
reduce the loss of blood and make a temporary seal,
preventing access by pathogens

Inflammation
the swelling and redness seen in infected tissues. Infected
tissue often feels hot and tender. This is caused by a cellsignalling substance called histamine.

Antibodies
proteins that are secreted in response to stimulation by the
appropriate antigen. They have specific binding sites and
are capable of acting against the pathogen.

Antigens
molecules on the surface of cells that the immune system
can use to recognise pathogens.

Antigen presentation

involves placing an antigen on the cell-surface membrane


of a phagocytic cell.

primary response
the immune system's response to a first infection

secondary response
the immune system's response to a second or subsequent
infection by the same pathogen.

Active immunity
immunity acquired by activation of the immune system

Passive immunity
when someone is given antibodies produced by someone
else

autoimmune disease
one in which the immune system attacks the body's own
healthy cells and tissues

Vaccination
the deliberate introduction of antigenic material in order to
stimulate the production of antibodies

vector
an organism that carries the pathogen from one host to
another.
disease
an illness

communicable disease
caused by pathogens from one living thing to another

pathogen
a disease causing organism

non-communicable disease
a disease that can not be spread other than through
heredity (no pathogens-heart disease)

virus
smallest pathogens (ex: common cold, herpes)

bacteria
singled-celled microorganisms (ex: strep throat,
tuberculosis, syphilis, and lyme disease)

rickettsia
pathogens that grow in living cells and resemble bacteria
(ex: rocky mountain spotted fever and typhus)

fungi
single-celled or multi-cellular plantlike organisms, yeast
and mold (ex: athlete's foot, jock itch, nail infection,
ringworm)

protozoa
tiny, single-celled organisms that produce toxins that
cause disease (ex: malaria and dysentery)

direct contact
sexual intercourse, intimate kissing, blood transfusion,
touching ulcers or sores, handling body fluids (blood or
urine)

indirect contact
contact with an object that has been used by an infected
person

contact with pathogens in the air


an infected person coughs or sneezes, pathogens are
released inot the air

contact with animals or insects


a person becomes infected by being bitten by an animal or
insect (ex: 1) Rabies virus-dog, cat, raccoon, bat
2)Mosquitos-malaria 3)Insects land on sewage and then
deposit pathogen in food

contact with Contaminated Food and Water


water must be purified to drink and cook with

skin
unbroken skin prevents pathogens from entering the bodybathing rids the body of the pathogens

tears, saliva, and perspiration


contain chemicals that kill pathogens

mucous membranes
tissues that line the body opening and secrete mucus

mucus
is thick secretions that coats the mucous membranes

stomach acid
acid in stomach that kills most pathogens

Immune System Response


contains cells and organs that fight disease

T cells
white blood cells that regulate the action of the immune
system

phagocytes
white blood cells that surround and kill pathogens by
ingesting them

B cells
...
What is the primary prevention of disease?
Immunization

How do you control the spread of disease to others?


Reduce risk of cross-transmission of organisms
Infection Control Policies
Handwashing

Children With Immunodeficiency


Receiving Steroid Therapy
Other immunosuppressive therapies
Generalized malignancies
Immunologic disorder

What is Varicella?
Chicken Pox

How is Varicella Transmitted?

Droplet (Airborne) and Contact

Incubation of Varicella
14-16 days but can be as long as 18-21 days

Prodromal Period of Varicella


Mild fever
Malaise x 24 hours prior onset of rash
MOST CONTAGIOUS AT THIS TIME

Varicella-Manifestations
Rash erupts 1 day after prodromal period
Very itchy
Begins as a macule and progresses to a papule
Then becomes a fluid filled vesicle resembling a dew drop,
which breaks and forms a crust
starts in trunk and progress to proximal extremities and
face

Varicella-Treatment
Acyclovir and Varicella Zoster Immune Globin in High Risk
Children After Transmission
Strict Isolation
Skin care-Comfort, Cool Bath, Calamine
Avoid use of aspirin/salicylates
Manage itching w/distraction, benadryl in moderation

Varicella-Complications
Secondary bacterial infections
Encephalitis
Pneumonia
Hemorrhagic Varicella
Chronic or transient thrombocytopenia

What is Erythema Infectiosum?


Fifth Disease
Caused by Parvovirus B19

How is Erythema Infectiosum Transmitted?


Unknown-? respiratory secretions and blood

Erythema Infectiosum-Manifestations
Mild Fever
Joint Pain

Lethargy
Rash-3 stages

What are the 3 stages of a fifths disease rash?


1) Slapped face appearance on cheeks
2) Maculopapular rash on extremities; resembles lace
3) Skin Appears Irritated

Erythema Infectiosum-Management
Self Limiting, can only manage symptoms
*anti-pyretics
*analgesics

Erythema Infectiosum-Risk during Pregnancy


Risk of fetal hydrops and death
May need serologic testing if exposed to monitor immune
status

Erythema Infectiosum-Complications
chronic joint pain
aplastic crisis
myocarditis

What is Exanthem Subitum?


Roseola
Human herpes virus type 6

How is Exanthem Subitum transmitted?


Unknown transmission and source

Exanthem Subitum-Characteristics
High fever with unknown etiology for several days
Followed by rash once fever is resolved
may have lymphadenopathy, inflamed pharynx, cough,
coryza

Exanthem Subitum-Treatment
Self limiting. Treat the symptoms!

What is Pertussis?
Whooping Cough
Caused by bordetella pertussis

How is Pertussis Transmitted?

Droplet or Direct Contact


Incubation: 6-20 days

Pertussis-Characteristics
Short, rapid coughs followed up a crowing or "whoop"
sound

What is the complication of Pertussis?


Pneumonia

How is Pertussis treated?


Antibiotics

What is Rubeola?
Measles
Viral Infection

How is Rubeola transmitted?


Droplet from respiratory tract, blood, urine

Rubeola-Characteristics
Fever and Malaise
Followed by coryza, cough, conjunctivitis, Koplik spots,
anorexia, lymphadenopathy then rash

Rubeola-What does the rash look like?


Initially erythematous maculopapular progressing to
brownish with desquemation

Rubeola-Treatment
Treatment includes supplementation w/vitamin A;
supportive care
Maintain isolation until 5th day of rash, bedrest

Rubeola-Complications
Complications include otitis media, pneumonia,
bronchiolitis, obstructive laryngitis/laryngotracheitis,
encephalitis

What is Mumps? How is it Transmitted?


Paramyxovirus
Transmitted by contact or droplet spread of infected saliva

Mumps-Characterized

Fever, headache, malaise, anorexia, earache w/chewing;


parotitis w/enlargement, pain and tenderness; may have
submaxillary/sublingual infection, orchitis,
meningoencephalitis

Mumps-Treatment
Supportive: analgesics, antipyretics; IV fluids if unable to
drink/vomiting due to meningoencephalitis
Maintain isolation

Mumps-Complications
sensorineural deafness, postinfectious encephalitis,
myocarditis, arthritis, hepatitis, epididymo-orchitis, sterility
in adult males (rare)

What is German Measles?


Rubella Virus

How is German Measles Transmitted?


Transmitted by nasopharyngeal secretions, blood, urine,
stool

German Measles-Characteristics
No prodromal symptoms in children; in adults, adolescents:
low grade fever, headache, malaise, anorexia,
conjunctivitis, coryza, sore throat, cough,
lymphadenopathy; followed by rash, starting on face and
progressing downward, by end of 1st day body is covered

German Measles-What does the rash look like?


Rash is maculopapular, pinkish red; usually disappears in
same order that it appeared

German Measles-Treatment
Self limiting, treatment includes antipyretics, analgesics

German Measles-Complications
Rare-but include arthritis, encephalitis, purpura

German Measles-Contracted during Pregnancy


Benign course for affected child, but more serious when
acquired in pregnancy; teratogenic effects noted on fetus
(congenital heart disease, hearing loss, growth delay,
learning disablities, mental retardation,

eye/neurologic/endocrine abnormalities); also can cause


stillbirths, miscarriages
Immune status always checked in pregnancy

What is Poliomyelitis?
3 types of enteroviruses
No wild type virus since 1979; few vaccine-induced cases
until 1998, development of IPV

How is Poliomyelitis transmitted?


Transmitted by direct contact w/infected feces and
oropharyngeal secretions

Poliopmyelitis-Chracteristics
May be inapparent (fever, sore throat, headache, vomiting,
abdominal pain), nonparalytic (sl. more severe than
inapparent)

What is the most severe form of Poliomyelitis?


Most severe form is paralytic, which is similar in course to
nonparalytic type w/recovery, and then CNS paralysis

Poliomyelitis-Treatment
No treatment other than supportive

Poliomyelitis-Complications
Complications include complete paralysis, respiratory
arrest, hypertension, and those things caused by
immobility

What is Scarlet Fever?


Group A beta-hemolytic strep

How is Scarlet Fever transmitted?


Transmitted by direct contact/droplet spread from infected
nasopharyngeal secretions

Scarlet Fever-Characteristics
Fever, malaise, headache, lymphadenopathy, vomiting,
chills, abdominal pain, enlarged tonsils w/exudate,
erythematous pharynx, strawberry tongue, and RASH

Scarlet Fever-What does the rash look like?


red, pinpoint w/sandpaper texture

Scarlet Fever-Treatment
antibiotics (penicillin, erythromycin) x 10 days, analgesics,
antipyretics.

Scarlet Fever-Complications
otitis media, peritonsillar/retropharyngeal abcess, sinusitis,
glomerulonephritis, carditis/polyarthritis

Conjunctivitis-Causes in Newborns
chlamydia, gonorrhea, or herpes simplex virus

Conjunctivitis-Causes in Infants
may be sign of tear duct obstruction

Conjunctivitis-Causes in Children
bacterial (most common), viral, allergic, or foreign body

Lead Poisoning-What does a low dose exposure cause?


Distractibility
Hyperactivity
Impulsivity
Hearing impairment
Mild intellectual deficits

Lead Poisoning-What does a high dose exposure cause?


Encephalopathy
Mental retardation
Paralysis
Blindness
Seizures
Coma
Death

Lead Poisoning treatment


Chelation
calcium disodium edetate (EDTA) and succimer (DMSA)

Which Vaccines have two series?


MMR
Varicella
Hep A

Which Vaccines have three series?

Hep B
RV

Which Vaccines have four series?


Hib
PCV
IPV

Which Vaccines have five series?


DTaP

Which Vaccine is needed yearly?


Influenza
Which 2 vaccines are given subcutaneous?
Varicella and MMR

Definition of Immunization?
Process when an animal or person becomes protected
against a disease

Definition of Vaccination?
Injection of a killed or weakened infectious organism to
prevent a disease

Definition of Vaccine?
Product that produces immunity to protect the body from a
disease.

Safety of Vaccines?
Studied before giving to public
Scientist monitor safety of vaccines
Serious adverse effects are rare
Multiple vaccines at one visit is considered safe and
quickest protection against diseases

What is infection?
Invasion of bacteria or viruses

What alerts the immune system to the invasion of


bacteria or viruses?
WBC's

What types of WBC's are the main part of the immune


system?
macrophages, B-lymphocytes (antibodies) and Tlymphocytes

What is active immunity?


when the immune system produces antibodies because of
exposure to disease organism

What are the types of active immunity?


natural and vaccine-induced immunity

What is natural immunity?


antibodies formed in the presence of active infection in the
body

What is vaccine-induced immunity?


antigens (vaccines or toxoids) are given to stimulate
antibody production

How long does naturally acquired active immunity last?


lifelong

How long does it take for vaccine acquired active


immunity to take effect?
can take several weeks to develop

How long does vaccine acquired immunity last?


many years, but must be reinforced with a booster

How long does active immunity last?


long period

How long does passive (acquired) immunity last?


short period

How long does natural passive immunity last?


6 months - 1 year

How long does artificial passive immunity last?


2-3 weeks

What is passive (acquired) immunity?

Antibodies are produced by another source, animal or


human

What is natural passive (acquired) immunity?


antibodies are transferred naturally from an immune
mother to her baby through the placenta or colostrum

What is artificially passive (acquired) immunity?


immune serum (antibody) from an animal or another
human is injected

What is heard immunity?


when a enough of a population is immune to an infectious
disease so the non-immunized people are offered some
protection and this keeps the disease from spreading

What is a live, attenuated vaccine?


A vaccine that contains a living, weakened virus

Is a live or a killed vaccine better for immunity purposes?


live, as it is the closest to the natural infection

What are examples of a live, attenuated virus?


MMR and Varicella vaccines

What is an inactivated vaccine?


A vaccine that contains a killed virus

What is the disadvantage of an inactivated vaccine?


several doses are necessary for immunity

What is an example of an inactivated vaccine?


polio vaccine

What is a subunit vaccine?


A vaccine that contains parts of a virus or bacteria
(subunits)

What are the advantages of a subunit vaccine?


side effects are less common

What is an example of a subunit vaccine?


Pertussis vaccine

What is a conjugate vaccine?


A vaccine that fights bacteria because the antigens stick to
an outer coating of sugar like substance called
polysaccharides

What is an example of a conjugate vaccine?


Haemophilus influenze type B (Hib) vaccine

What is a toxoid vaccine?


...
disease
any condition that interferes with the proper functioning of
the body or mind

communicable disease
a disease that can be passed from one person to another

pathogen
a disease causing organism when germs enter your body,
you can develop an infection

infection
a condition that occurs when pathogens enter your body,
multiply and cause harm

virus
the smallest pathogen that can be heeled with antibiotics

bacteria
tiny one celled organism that can be cured with antibiotics

fungi
more complex organism than bacteria but cannot make
their own food. Thrive in warm, moist areas ex. ringworm,
athletes foot

protozoa
one celled organism that are more complex then bacteria

direct contact with others


shaking hands or kissing one with an infected person

indirect contact with others


sharing drinks or eating utensils, contaminated needles,
tattoos, piercing or drug injections

contact with contaminated foods or water


improperly stored or uncooked

contact with animal or insects


ticks with Lyme disease, mosquito's, with West nile

protect yourself from pathogens


practicing good hygiene (cleanness)

protect yourself from pathogens


avoid contact with infected people

protect yourself from pathogens


never share eating or drinking utensils

protect yourself from pathogens


wash your hands frequently

protect yourself from pathogens


handle and prepare food frequently

protect yourself from pathogens


avoid touching eyes,mouth or nose

skin
first line of defense,
covers and protects body

mucous membranes
lines the mouth, nose , throat and eyes,
their job is to trap jerms

saliva
destroys harmful organisms

tears
wash away germs

stomach acid

acid produced in the stomach kills germs

vaccine
a preparation of dead or weakened pathogens that causes
the immune system to produce antibiotics

immunization
helps the immune system make antibiotics for certain
diseases

immunity
your body's ability to resist the germs that causes a
particular disease

2 main responses to your immune system


specific and nonspecific

non specific response


when pathogens enter your body, your immune system
reacts with a non specific reaction first. this response
began with inflammation. It reacts to injuries,
diseasesresulting in swelling, pain, heat and redness

specific reactions
some pathogens survive the nonspecific reaction. when this
happens your body is self off a specific immune system.
Our immune system recognizes a pathogen it has already
battled so it is ready to re attack

antigen
substance realised by invading pathogens

types of immunization
Hepatitis B, Polio, measles, chicken pox, tetanus shot
not only do they help you but they also surround you

colds
are caused by hundreds of different viruses and can be
spread by direct and indirect contact

treatments of cold
getting rest and drinking fluids, OTC medicines can help,
stay home for at least 24 hours

flu (influenza)
symptoms are fever, chills/ fatigue, headaches/muscle
aches and respiratory pain

mononucleosis
''mono'' or the ''kissing disease'' is a viral diseases
characterized by a severe sore throat and swelling of the
lymph glands in the neck and throat area

hepatitis
a viral disease if the liver characterized by yellowing of the
skin and whites of the eye

3 types of hepatitis
hep A,B and C

hepatitis A
common in areas of poor sanitation , like food or water
contaminated with human waste or enters the body
through an open wound

hepatitis B,C
spread through contact with contaminated blood or other
body fluids. There are vaccines to protect people with Hep
A and B. There are medicines to treat people with Hep.C

tuberculosis (TB)
bacterial disease that affects the lungs. TB can be spread
easily through air ,sometimes no symptoms but can still be
spread.

pneumonia
serious inflammation to the lungs
symptoms are fever, chills, and difficultly breathing

strep throat
sore throat causes by streptococcal bacteria
symptoms are red and painful throat, fever,and swollen
lymph nodes in neck

H.i.v
human, immunodeficiency, virus

human

this disease attacks humans

immunodeficiency
this disease attacks the immune system

virus
the smallest and simplest form of life

A.I.D.S
acquired, immune, deficiency, syndrome

Acquired
this disease comes from outside the body. A person must
do something to take it in

Immune
this disease attacks the immune system

deficiency
lack of something. Aids destroys the T-cells

syndrome
a medical condition that has no cure

opportunistic disease
a disease that takes advantage of a weakness in the body.
People die from it because aids kill the immune system.

Hiv
2-10 years looks and feels good

Arc,Aid
body shuts down

death
male 3 years
females 1-5 years

A.R.C
aids realistic complex

what is arc

a series of illness when hiv turn into full blown aids. A


person begins to looks sick and feels sick

symptoms of arc
lost of appetite

symptoms of arc
fatigue

symptoms of arc
weight loss

symptoms of arc
fever

symptoms of arc
night sweats

symptoms of arc
diarrhea

what is believed to be the organ to the virus


It is said to organite from a green monkey in Africa

ways hiv can be transmitted(spread)


sexual contact (75%)

ways hiv can be transmitted(spread)


IV drug use (22%)

ways hiv can be transmitted(spread)


mother to child (2%)

ways hiv can be transmitted(spread)


blood transfusion (1%)

Way hiv is not transmitted(spread)


breathing in ar

Way hiv is not transmitted(spread)


swimming pools

Way hiv is not transmitted(spread)


donating blood

Way hiv is not transmitted(spread)


casual contact

Way hiv is not transmitted(spread)


sharing items

medicine used to fight aids


azt

what it does
slows down the disease

what it does
decreases the number of immune cells

these drugs are very


expensive

is there a cure
no

way to prevent the disease


abstinence

way to prevent the disease


personal behaviors
The infectious disease stage in which the antigen is
present but no antibody is detectable is the ______________
stage.
A) A. Communicable
B) B. Incubation
C) C. Latent
D) D. Window
D

The hepatitis ____ virus is most commonly transmitted by


contaminated needles.
A) A. A/HAV
B) B. B/HBV

C) C. C/HCV
D) D. D/delta
C

The most severe and potentially life-threatening form of


meningitis is ______________ sometimes caused by
____________.
A) A. Bacterial/Neisseria meningitides
B) B. Bacterial/tuberculosis
C) C. Viral/streptococcus pneumoniae
D) D. Viral/viral pneumonia
A

An acute disease caused by the Epstein-Barr virus, a


member of the herpes family is:
A) A. Influenza
B) B. Mononucleosis
C) C. Pertussis
D) D. Rabies
B

Inflammation and swelling of the cornea is known as:


A) A. Keratitis
B) B. Malaise
C) C. Prodrome
D) D. Risus sardonicus
A

Who is responsible for protecting the public from disease?


A) A. FEMA
B) B. OSHA
C) C. Local hospital
D) D. Your employer
D

Incubation, window, communicable, latent, and disease


period are stages of:
A) A. Allergic reactions
B) B. Infectious disease
C) C. Infection plans
D) D. Paramedic stress factors
B

As a paramedic, you should take standard precautions


with any patient with unknown respiratory pathogens,
and you should also:

A) A. Contact the CDC.


B) B. Keep patient as far away from you.
C) C. There is need to worry because the patient will be in
your care for a short period of time.
D) D. Place surgical mask on patient.
D

The interval between exposure to an agent and the first


appearance of symptoms is the
A) A. Disease period
B) B. Incubation period
C) C. Latent phase
D) D. Window phase
B

EMS agency employers are required by law to provide


employees with appropriate medical care and treatment
in the event of an exposure and designate a person or
officer to whom the employee should report the exposure.
This is under:
A) A. FEMA
B) B. Medical control
C) C. Roe v. Wade
D) D. The Ryan White Act
D

Identify a transmission route for hepatitis C.


A) a. Contaminated feces
B) c. Semen and vaginal fluids
C) b. Infected blood
D) d. Sharing food and water
C

A viral infection resulting in swelling of the salivary and


parotid glands is:
A) b. Chickenpox
B) a. An allergic reaction
C) c. Mumps
D) d. Rubella
C

The time period from exposure until when disease


symptoms first appear is the:
A) b. Incubation period
B) d. Window phase

C) a. Communicable period
D) c. Latent period
A

You have the greatest risk of contracting which


bloodborne pathogen from a needle stick?
A) c. Human immunodeficiency virus (HIV)
B) b. Hepatitis B
C) d. Tuberculosis
D) a. Hepatitis A
B

The vulnerability or weakness to a specific pathogen is


known as:
A) c. Reactability
B) a. Immunity rate
C) d. Susceptibility
D) b. Infection rate
C

You have been splashed in the face with the blood of a


patient with hepatitis C. After washing your face and eyes
with water for at least 10 minutes, you are required to:
A) d. Request prophylaxis medications from the receiving
emergency department
B) c. Report the incident to a supervisor or the infection
control officer
C) b. Mandate that the patient pay for your medical
expenses if you get hepatitis C
D) a. Fill out a Ryan White reporting form
B

A woman has brought her daughter to the emergency


department. The girl is 4-years-old and has been suffering
from a worsening, 'hacking' cough for several days. As
you evaluate the child, her temperature is 104 F, her
eyelids appear swollen, her eyes are red with a crusty
discharge, and she has reddish spots on her forehead,
lips, and in her mouth. You quickly identify this patient as
having:
A) a. Chickenpox
B) c. Measles (rubeola)
C) d. Mumps
D) b. German measles (rubella)
B

You are presented with a 24-year-old female patient in the


back bedroom of a low-income apartment. Her husband
tells you she has had a worsening headache over the past
few days. She is laying on the covers in the dark. When
you turn on the light, she winces, and complains the light
makes her headache worse and causes dizziness. You
notice that her skin is flushed and she is sweating despite
a cold apartment, and you also notice that her neck
appears stiff as she moves. Your next action is to:
A) c. Perform a physical exam of the head and neck
B) a. Administer an antipyretic to reduce any fever and
the pain
C) b. Apply oxygen and listen to lung sounds
D) d. Remove yourself from the room and put on an N-95
mask or other protective mask
D

A paramedic can become infected with tuberculosis by:


A) d. Sharing food or water with an infected individual
B) c. Being exposed to membrane droplet nuclei expelled
during a cough, sneeze, or prolonged talking
C) b. Being stuck with a needle that had been used on a
patient with tuberculosis
D) a. Allowing feces contaminated with tubercle bacilli to
touch the skin
B

Outbreaks associated with the HSV-1 infection generally


occur:
A) d. In the oropharynx
B) b. Evenly across the body
C) c. In the genital region
D) a. Around the anus
A

Rubella affects the:


A) a. Circulatory and musculoskeletal systems
B) c. Respiratory and nervous systems, and oropharynx
C) d. Respiratory, circulatory, and nervous systems
D) b. Integumentary and musculoskeletal systems
D

The human immunodeficiency virus can be spread


through:
A) a. Coughing
B) c. Heavy sweating

C) b. Hand-to-hand contact
D) d. Sexual intercourse
D

Which form of hepatitis is not considered a complete


virus?
A) b. Hepatitis B
B) c. Hepatitis C
C) d. Hepatitis D
D) a. Hepatitis A
C

A diagnosis of bacterial meningitis is typically associated


with:
A) c. Septic shock
B) b. Internal bleeding in the spinal column
C) d. The absence of other infections
D) a. Another infection such as pneumonia or endocarditis
D

You are caring for a 73-year-old female in a nursing home


who is experiencing respiratory distress that worsened
throughout the night. Currently you find her seated
upright in bed with accessory muscle use. Audible rales
can be heard; she is diaphoretic and warm to the touch,
she has a temperature of 102 F, and her respiratory rate
is 32 breaths/min. The patient tells you that her breathing
has been getting worse all night. You auscultate the chest
and hear rales and rhonchi in all fields of her left lung and
rhonchi in her right base, but her right middle and upper
lobes sound clear. You determine that this patient is
experiencing respiratory distress caused by:
A) d. Tuberculosis
B) b. Chronic obstructive pulmonary disease
C) a. Congestive heart failure
D) c. Pneumonia
D

Males who have contracted chlamydia may experience:


A) d. Open sores on the penis and scrotum
B) b. Development of lesions on the penis
C) a. A urinary tract infection
D) c. Discharge from the urethra
D

The time period when an infected disease can be


transmitted to another host is known as the:
A) d. Window phase
B) b. Incubation period
C) c. Latent period
D) a. Communicable period
D

Gonorrhea can be cured with the use of:


A) b. Antibiotics
B) c. Antifungal drugs
C) d. There is no definitive cure
D) a. Antiparasitic drugs
C

The responsibility of appropriately disposing of equipment


and supplies potentially infected with a disease lies with
the:
A) c. Patient
B) a. Hospital staff
C) b. Paramedic
D) d. Ambulance/transporting agency
C

During the care of a 44-year-old female patient who was


ejected from her SUV during a highway-speed rollover,
her arterial bleeding results in blood splashing onto your
face and eyes. Besides stopping care to immediately flush
your face, it is your right to:
A) c. Request that you and the patient both be tested for
communicable diseases
B) b. Receive workers' compensation for the remainder of
your working years
C) d. Sue the patient
D) a. Receive postexposure prophylaxis for hepatitis and
HIV
A

The time period when an infected person cannot transmit


a disease to another individual is known as the:
A) b. Incubation period
B) c. Latent period
C) d. Window phase
D) a. Communicable period
B

The MMR vaccine protects recipients from:


A) a. Malaria, mumps, and Reye's syndrome
B) c. Measles, malaria, and Reye's syndrome
C) d. Measles, mumps, and rubella
D) b. Malaria, mumps, and rubella
C

A viral disease once commonly seen in children but that


now appears in fewer than 500 cases per year is:
A) c. Mumps
B) a. Fifth disease
C) d. RSV
D) b. Chickenpox
A

Outbreaks associated with the HSV-2 infection generally


occur:
A) c. In the genital region
B) b. Evenly across the body
C) a. Around the anus
D) d. In the oropharynx
A

The rubella virus can be spread through contact with


infected:
A) a. Blood
B) b. Feces
C) c. Saliva
D) d. Skin/open lesions and blisters
C

The herpes simplex virus type 1 (HSV-1):


A) a. Affects 80% of the human population
B) b. Has been eliminated from the human species and no
cases are currently known to exist
C) d. Affects approximately half of the human population
D) c. Is relatively rare, affecting 2% to 3% of the
population
A

The administration of aspirin to a child with chickenpox


can cause:
A) c. Raynaud's phenomenon
B) d. The lesions to worsen
C) a. Mild internal bleeding
D) b. Reye's syndrome

Treatment for influenza is often limited to:


A) b. Antibiotics
B) c. Antiviral drugs
C) a. A specific antiflu drug
D) d. Supportive care only
D

Personal protective equipment designed to prevent a


tuberculosis exposure include:
A) c. A surgical gown
B) b. A full face mask
C) a. A disposable particulate respirator
D) d. Placing the patient on a nonrebreather mask
C

Gonorrhea is an STD caused by a:


A) b. Fungus
B) a. Bacteria
C) c. Parasite
D) d. Virus
B

The condition that results in bacteria passing the bloodbrain barrier and increasing cerebral vascular
permeability, thus increasing intracranial pressure, is
known as:
A) d. Tuberculosis
B) c. Meningitis
C) a. Hepatitis
D) b. HIV
B

Hepatitis E spreads most similar to:


A) d. Tuberculosis
B) c. HIV
C) a. Hepatitis A
D) b. Hepatitis B
C

Rabies primarily affects which body system?


A) d. Respiratory system
B) c. Central nervous system
C) b. Musculoskeletal system
D) a. Circulatory system

The body's second line of defense against infections is


the:
A) a. Antibodies
B) c. Skin
C) d. White blood cells
D) b. Inflammatory response
D

Hepatitis D becomes pathogenic when a patient has:


A) d. HIV
B) a. Hepatitis A
C) b. Hepatitis B
D) c. Hepatitis C
B

Which body system attempts to fight off infectious


diseases?
A) d. Respiratory system
B) a. Circulatory system
C) b. Immune system
D) c. Nervous system
C

Which of the following patients is at greatest risk for


developing chronic hepatitis after being exposed to
hepatitis B?
A) c. A 15-year-old male
B) d. A 30-year-old male
C) a. A newborn baby boy
D) b. A 5-year-old male
C

Which of the following is considered a first-line defense


against infection for the human body?
A) c. Skin
B) a. Antibodies
C) b. Inflammatory response
D) d. White blood cells
A

Vectors for the rubella virus are


A) c. Humans only
B) b. Humans and mosquitoes

C) d. Mosquitoes and some spiders


D) a. Humans and domestic dogs
A

Infectious agents that invade a host's cell to reproduce


are:
A) a. Bacteria
B) d. Viruses
C) b. Fungi
D) c. Protozoa
B

You are managing a 32-year-old male at the state prison


who is complaining of shortness of breath and weakness.
He has a fever and a cough, and he occasionally spits up
bloody phlegm. Appropriate Standard Precautions for this
patient are:
A) c. Gloves only
B) d. A face mask and a nonrebreather mask on the
patient
C) a. Gloves and a face mask
D) b. Gloves, a gown, a face mask, and eye protection
D

The rash from a scabies infection is described as:


A) b. Generalized redness and burning across the body
B) d. Patchy red lesions that are itchy
C) a. Acne anywhere on the body
D) c. Open painful ulcers around skin folds
B

Lice are treated with:


A) a. Antibiotics
B) c. Shaving the hair in the infected region
C) b. Intense heat
D) d. Topical medications and washes
D

While working in a college campus clinic, you have


observed that several students over the past 3 days
complaining of increased coughing with nausea and
vomiting. Your latest patient, a 19-year-old male, has
periods of uncontrollable spastic coughing, a mild fever,
and intermittent vomiting. What you notice most about
his cough is that he will be fine for several minutes and
then suddenly begin to cough forcefully for up to 1

minute. He is unable to control when and for how long he


coughs, and he seems to always cough in batches. You
suspect that there may be an outbreak of what disease
on campus?
A) c. Pertussis
B) d. Pneumonia
C) a. Asthma attacks
D) b. Influenza
B

Which of the following groups is at risk for exposure to


hepatitis A?
A) d. Nurses working in a prison first-aid station
B) a. A basketball team showering in the same showers
C) c. A patient living in a home for patients with
developmental disabilities
D) b. A group of friends traveling to Africa
C

Your patient has a known history of hepatitis B and HIV


and is complaining about abdominal pain. Appropriate
Standard Precautions include:
A) a. Gloves and a face mask
B) b. Gloves, a gown, a face mask, and eye protection
C) c. Gloves only
D) d. A face mask and a nonrebreather mask on the
patient
B

If, after being tested following an exposure to an


infectious disease at work, medical screening finds you to
be positive for the disease, the testing healthcare
provider is required to report the findings to:
A) b. You and OSHA
B) a. You alone
C) c. You and the Department of Public Health
D) d. You and your employer
B

A scabies infection develops when what type of parasite


burrows under the skin?
A) b. Mites
B) d. Worms
C) c. Ticks
D) a. Lice
A

Prehospital management of gastroenteritis may include:


A) a. Administering activated charcoal to kill the source of
the virus
B) d. Initiating IV fluids
C) c. Initiating IV antibiotics
D) b. Administering syrup of ipecac
B

Proper tick removal is accomplished by:


A) d. Squeezing the body and pulling it out
B) c. Placing a heated match stick against the tick
C) b. Grabbing the tick as close to the surface of the skin
as possible with tweezers and pulling it directly out of the
skin
D) a. Applying a nonbreathable lubricant onto the tick so
it self-withdraws
Which communicable diseases cause most communicable
disease-related deaths?
pneumonia
diarrheal diseases
tuberculosis
malaria
measles
HIV/AIDS.

Which populations are at risk for communicable disease?


Young children
Older adults
Immunosuppressed clients
Intravenous drug users
Health care workers

Which diseases have airborne transmission?


Measles
Chickenpox
Streptococcal infection
Tuberculosis
Pneumonia
Influenza

Which diseases have foodborne transmission?


Salmonellosis
Hepatitis A

Trichinosis
Escherichia coli (E. coli)

Which diseases have waterborne transmission?


Cholera
Typhoid fever
Giardia lamblia

Which diseases have vector-borne transmission?


Lyme disease
Rocky Mountain spotted fever
Malaria

Which diseases have direct contact transmission?


Sexually transmitted diseases (HIV, gonorrhea,
syphilis, genital herpes, hepatitis
B, C, D)
Infectious mononucleosis
Impetigo, lice, scabies

What are some examples of portals of entry?


Respiratory system
Gastrointestinal tract
Skin
Mucous membranes

What are some examples of portals of exit?


Respiratory system
Feces
Blood
Semen/vaginal secretions
Saliva
Skin

What are defense mechanisms against infections?


Natural immunity (from the body's antigen antibody
response)
Artificial immunity (through vaccination)
Active (vaccination with live, killed, toxoid)
Passive (from antitoxin or antibodies)

What diseases are reportable to the CDC?


AIDS
Anthrax
Botulism

Cholera
Diphtheria
Encephalitis
Giardiasis
Gonorrhea
Hepatitis A-D
Influenza activity
Legionellosis/Legionnaires' disease
Leprosy
Lyme disease
Malaria
Meningococcal infections
Mumps
Pertussis
Poliomyelitis
Rabies
Rocky Mountain spotted fever
Rubella
Rubeola (measles)
Salmonellosis
Shigellosis
Severe acute respiratory syndrome-associated
Coronavirus disease (SARSCoV)
Syphilis
Smallpox
Tetanus
Toxic shock syndrome
Trichinosis
Tuberculosis
Typhoid fever
Vancomycin-resistant Staphylococcus aureus (VRSA)
Varicella (chickenpox)

What are some primary prevention strategies related to


infectious diseases?
Prevent the occurrence of
infectious disease.
Educate the public
regarding the need for
immunizations, federal and
state vaccination programs,
and immunization laws such
as the "no-shots, no school"
legislation.
Counsel clients traveling
to other countries about
protection from infectious

diseases. Refer clients to


the health department
for information about
mandatory immunizations.
Educate the public
regarding prevention
of disease and ways to
eliminate risk factors
for exposure, such as
hand hygiene, universal
precautions, proper food
handling and storage, and
use of condoms.

What are some secondary prevention strategies related to


infectious diseases?
Increase early detection
through screening and case
finding.
Refer suspected cases of
communicable disease for
diagnostic confirmation and
epidemiologic reporting.
Treat postexposure
infections (hepatitis A,
rabies).
Quarantine clients when
necessary.

What are some tertiary prevention strategies related to


infectious diseases?
Decrease complications and
disabilities due to infectious
diseases through treatment
and rehabilitation.
Monitor treatment
compliance, including
directly observed therapy.
Prevent reinfection. Identify
community resources.
Disasters

What are the three levels of disaster management?


preparedness
response
recovery

What are the s/sx of inhalation anthrax?


Sore throat
Fever
Muscle aches
Severe dyspnea
Meningitis
Shock

What are the s/sx of botulism?


Difficulty swallowing
Progressive weakness
Nausea, vomiting, abdominal cramps
Difficulty breathing

What are the s/sx of smallpox?


High fever
Fatigue
Severe headache
Rash (begins on face and tongue, quickly
spreading to the arms and legs, then hands and
feet, within 24 hr) that turns to pus-filled lesions
Vomiting
Delirium
Excessive bleeding

How long does the small pox vaccine last?


10 years

What are the s/sx of ebola?


Sore throat
Headache
High temperature
Nausea, vomiting, diarrhea
Internal and external bleeding
Shock

What is the role of the community nurse in the event of a


bioterrorism attack?
Participate in planning and preparation for immediate
response to a bioterrorist event.
Identify potential biological agents for bioterrorism.
Survey for and report bioterrorism activity (usually to
the local health department).
Promptly participate in measures to contain and

control the spread of infections


resulting from bioterrorist activity.

What is involved in assessment of a bioterrorism threat?


Is the population at risk for sudden high disease rates?
Is the vector that normally carries a specific disease
available in the geographical area
affected?
Is there a potential delivery system within the
community?

How does a community health nurse recognize a


bioterrorism event?
Is there a rapidly increasing disease incidence in a
normally healthy population?
Is a disease occurring that is unusual for the area?
Is an endemic occurring at an unusual time? For
example, is there an outbreak of
influenza in the summer?
Are there large numbers of people dying rapidly with
similar presenting symptoms?
Are there any individuals presenting with unusual
symptoms?
Are there unusual numbers of dead or dying animals,
unusual liquids/vapors/odors?

What are some primary prevention strategies for


bioterrorism?
Preparation with
bioterrorism drills, vaccines,
and antibiotics for exposure
prophylaxis
Bioterrorism planning
Design a bioterrorist
response plan using the
most probable biological
agent in the local area.
Assess and locate the
local facilities that have
Level I, Level II, Level III,
and Level IV biosafety
gear.
Identify the chain of
command for reporting
bioterrorist attacks.
Define the nursing
roles in the event of a

bioterrorist attack.
Set up protocols for
different biosafety levels
of infection control and
containment.

What are some secondary prevention strategies for


bioterrorism?
Early recognition
Activation of bioterrorism
response plan in response to
a bioterrorist event
Immediate implementation
of infection control and
containment measures,
including decontamination,
environmental disinfection,
protective equipment,
community education/
notification, and quarantines
Screening the population for
exposure, assessing rates of
infection, and administering
vaccines as available
Assisting with and educating
the population regarding
symptom identification
and management
(immunoglobulin, antiviral,
antitoxins, and antibiotic
therapy, depending on the
agent)
Monitoring mortality and
morbidity

What are some tertiary prevention strategies for


bioterrorism?
Rehabilitation of survivors
Monitoring medication
regimens and referrals
Evaluating the effectiveness
and timeliness of the
bioterrorism plan

What are some example of Class A biological agents?


highest priority agents, posing a risk to national
security because they are easily transmitted and have high

mortality rates.
Examples include smallpox (variola), botulism toxin,
anthrax, tularemia,
hemorrhagic viral fevers, and plague.

What are some example of Class B biological agents?


second highest priority because they are moderately
easy to disseminate and have moderate morbidity rates
and low mortality rates.
Examples include typhus and cholera.

What are some example of Class C biological agents?


the third highest priority, comprising emerging
pathogens that can be engineered for mass dissemination
because they are easy to
produce, and/or have a potential for high morbidity and
mortality rates.
Examples include Hantavirus.
Immunity
The state of being protected against a certain disease

Antigen
Substance capable of triggering an immune response

Antibodies
Protein that fights against a specific antigen

Lymphocytes
Specialized white blood cells (t & b)

Active immunity
Developed from natural or artificial processes (flu vaccine)

Passive immunity
Your body receives antibodies from another person or
injection

HIV
Damages T cells and leads to aids

Physical barriers
Skin, nose, hair

Chemical barriers
Tears, stomach acid, mucus

Immune system
A network of cells, tissues, organs, and chemicals that fight
off pathogens

Inflammatory response
A reaction to tissue damage caused by injury or infection

Phagocytes
White blood cells that attack invading pathogen

Helper T cells
Trigger the production of B cells and killer T cells

Killer T cells
Attack and destroy infected body cells

Suppressor T cells
Turn off T cells when infection is cleared

B cells
Produce antibodies

CDC
Centers for disease control

WHO
World health organization

Live-virus vaccines
Vaccines made from pathogens in labs

Killed-virus vaccines
Vaccines using dead pathogens

Toxoids
Vaccines with inactivated toxins from pathogens &
simulates the growth of antibodies

New and second generation viruses


It's a cutting edge of disease fighting technology

Pathogens
Microorganisms that cause disease

Rickettsias
Resembles a bacteria and enters the body through insect
bites

Protazoan
Single celled microorganisms that are larger and more
complex than bacteria

Vectors
And organism that carries and transmits pathogens to
humans or animals

Communicable disease
Disease that is spread from one living organism to another
or through the environment
This policy serves as the Sheriff's Office
"Infectious Disease Exposure Control Plan" and the
"Biomedical Waste Operating Plan."

Any microorganism that is spread by droplet nuclei


through the air by coughing, sneezing, or talking.
Airborne Pathogens

Pathogenic micro-organisms that may be present in blood


or other body fluids from infected individuals and can
spread to others through direct or indirect contact with
those fluids or infected materials
Bloodborne Pathogens

An infectious disease capable of being passed to another


by direct or indirect contact with an infected person, their
body fluids, or infected materials.
Communicable Disease

The presence, or the reasonably anticipated presence, of


blood or other potentially infectious materials on an item
or surface.
Contaminated

The use of physical or chemical means to remove,


inactivate, or destroy blood borne pathogens on a surface
or item to the point where they are no longer capable of
transmitting infectious materials. The surface or item is
then rendered safe for handling, use, or disposal.
Decontamination

Specific eye, mouth, other mucous membrane, or skin, or


parenteral (piercing) contact with blood or other
potentially infectious materials including sprayed and/or
splattered infectious materials.
Exposure Incident

Human body fluids including: blood, urine, vomitus,


semen, vaginal secretions, tears, saliva, feces, amniotic
fluid, and/or any other body fluid that is visibly
contaminated with blood. Also all body fluids in situations
where it is difficult or impossible to differentiate between
body fluids.
Infectious Materials

Specialized clothing or equipment worn by personnel for


protection against a potential hazard. General work
clothes (e.g., uniforms, pants, shirts, or blouses) are not
considered personal protective equipment.
Personal Protective Equipment (PPE)

Items that could release potentially infectious or


contaminated materials (liquid or semi-liquid) when
handled; such as sharps or blood soaked clothing.
Regulated Waste

Any individual, living or dead, whose blood, or any other


potentially infectious body fluids or materials may be a
source of occupational exposure to another individual.
Examples include, but are not limited to, inmates, visitors,
hospital and clinic patients, or human remains.
Source Individual

A set of precautionary practices to support infection


control. According to the concept of universal
precautions, all human blood and body fluids are treated
as if known to be infectious for HIV, hepatitis, and other
blood borne pathogens
Universal Precautions

These are examples of


1. Viral;
2. Bacterial; and
3. Fungal.
Airborne Pathogens

These are examples of


1. Hepatitis B Virus (HBV);
2. Hepatitis C (HBC);
3. Human Immunodeficiency Virus (HIV);
4. Syphilis.
Bloodborne Pathogens

Bloodborne Pathogen exposure primarily occurs with


contact with body fluids or droplet spray from talking,
coughing, vomiting, or contaminated objects. Body fluids
include:
1. Blood;
2. Urine;
3. Vomitus;
4. Semen;
5. Vaginal Secretions;
6. Tears;
7. Saliva;
8. Feces;
9. Amniotic fluid; or
10. Any other body fluid where blood is visible

Exposure to airborne pathogens occurs when a


contaminant is inhaled. Types of diseases transmitted
through the air are:
1. Meningitis;
2. Influenza;
3. Pneumonia;
4. Tuberculosis
5. Measles;
6. Mumps;
7. Pertussis; and
8. Varicella (chicken pox).

Exposure does not always result in an infection. The


likelihood of transmission depends on the following:
1. How contagious the infectious person is;
2. Where the exposure occurs;

3. How long the exposure lasts;


4. How healthy you are at the time of exposure.

Modes of Infectious Disease Transmission


1. Cardiopulmonary Resuscitation (CPR)
2. Fights and Assaults
3. Human Bites
4. Searches and Evidence Handling
5. Transportation
6. Breath Alcohol Testing and Drug Testing
7. Deceased Persons and Body Removal
8. Autopsies
9. Crashes With Injury

When blood is present and a suspect displays combative


or threatening behavior, an officer must attempt
to put on gloves as soon as conditions permit

Personnel should be concerned with the possible HIV and


HAV / HBV exposure through CPR administration.
Therefore, gloves and a protective resuscitation mask
("pocket mask") with _________________ must always be
used when administering CPR.
a one-way valve

Should an employee be bitten by anyone, the employee


shall clean the wound with __________________ as soon as
possible.
soap and water

The following precautions can help reduce the risk of


infection during search situations:
(1) Caution must be used when searching
prisoner/suspects' clothing. Discretion should be used to
determine if an employee should search a
prisoner/suspect's clothing or if the prisoner/suspect should
empty his own pockets;
(2) An employee should wear protective gloves if exposure
to body fluids is likely to be encountered;
(3) If cotton gloves are to be worn for evidence handling,
they should be worn over protective, disposable gloves
when exposure to body fluids may occur;
(4) A flashlight should be used when searching areas in
which the available light does not permit adequate vision;
(5) Containers such as purses and bags should be searched
by turning the container up-side-down to empty the

contents. Small compartments must not be searched by


the finger or hand-probe method;
(6) Puncture-proof containers should be used to store any
sharp objects that may be contaminated with body fluids;
(7) Caution must be used when staples are used to seal
evidence so as not to tear gloves or puncture skin;
(8) Wet or damp items of evidence should be air-dried prior
to placement in any air-tight containers;
(9) When practical and appropriate, puncture resistant
gloves should be worn anytime a search of a person,
vehicle or place is conducted. All officers, and particularly
those involved with identification and crime scene
processing, must be alert for the presence of sharp objects
such as hypodermic needles, knives, razors, broken glass,
nails, or other sharp objects. Sharp objects should not be
handled with unprotected, bare hands but rather by
mechanical means, such as forceps or tongs. Contaminated
evidence shall not be transported, handled, nor stored with
or near any food items. An employee who has had contact
with or has been near contaminated items must not smoke
or consume food until after a thorough hand washing.

All procedures involving any potentially infectious


material will be performed in such a manner as to
minimize __________, _________ and __________ of these
materials
splashing, spraying and splattering

All personnel who exhibit open cuts or sores and will be


performing tasks involving anticipated exposure to
potentially infectious materials are required to put on
___________________ or protective equipment as
appropriate BEFORE getting in contact with potentially
contaminated material.
waterproof bandages

Specially marked ___________________________ will be used


to dispose of contaminated sharps.
puncture proof containers

When moving contaminated puncture proof containers


from the area of use, they will be
closed immediately prior to moving and placed in a red
plastic biohazard bag and tape-sealed if leakage is possible

What are Personal Protective Equipment:

1.
2.
3.
4.
5.
6.

Gloves;
Masks, Eyewear, and Gowns;
CPR mask;
Boot or shoe covers;
Disposable Personal respirator
Water-soluble plastic bags (clear w/ red stripe).

If an exposure occurs, the following procedures will be


initiated:
1. Clean the exposed area with soap and warm water or
another approved substitute (gel disinfectant, alcohol
towelettes, 10% bleach/water solution);
2. Remove contaminated clothing as soon as possible;
saturated clothing will be removed immediately, and be
placed in a red biohazard bag. The "Biohazard" bag of
clothing will be disposed of at the designated collection site
at the Property Room, Pretrial Detention Facility, or nearest
Jacksonville Fire Station.
3. Decontamination of surfaces and equipment:
4. Complete Exposure Incident Form as described in Article
V below;
5. Test the source:

Vehicles which have become contaminated by blood,


vomitus or other body fluids should be cleaned
immediately following use and prior to further utilization
of the vehicle. It is the employee's responsibility to ensure
the vehicle is decontaminated in the proper manner:
1) Disinfect with the bleach and water solution and clean
with hot soapy water wiping with a towel or other
absorbent cloth;
(2) Spray with an effective germicidal disinfectant; and
(3) The towel or other material used to wipe up the body
fluid shall be disposed of by placing it in a sealable,
leakproof, red or biohazard-labeled bag or container.

In reference to testing, If the source is an incarcerated


inmate
(1) The exposed employee shall inform the ITR
officer/watch commander at the PDF that he has been
exposed to the suspect's blood;
(2) The watch commander must notify the charge nurse at
the M2 clinic;
(3) The charge nurse will bring the inmate to the clinic and
will take blood samples:

In reference to testing, If the source is a suspect absentee


booked,
JSO will request the Hospital to test the suspect

In reference to testing, If the source is a suspect but dies,


JSO will request
the Medical Examiner's Office to test the suspect;

In reference to testing, If the source is a victim (i.e.,


officer exposed assisting a victim of an accident as first
responder), who will request the hospital treating the
victim to test;
the Infection Control Nurse

The Infection Control Nurse must ensure that lab results


are obtained in a timely manner and reported back to the
officer through direct phone call. An e-mail informing the
officer that the results are available must be sent to
ensure that the officer will be notified. E-mails will be
copied to ______________________________.
the JSO Occupational Manager

The officer exposed must contact ____________________ if


the lab results have not been reported within 24 hours.
the Clinic at the PDF

If the HBV test is positive, the officer should contact ___


_________________ to get instructions on where to go to
start prophylactic treatment.
the claim adjuster

If the HCV test is positive, there's no treatment available


at this time to prevent the infection. The officer should
contact _________________________________ to start medical
follow up.
the workers compensation adjuster

The exposed employee or other available member must


notify his supervisor immediately and complete an
Electronic Exposure Incident Report Form. The supervisor
of the exposed employee, the Infection Control Nurse,
and ____________________________ shall receive email
notification of the incident.
the Occupational Manager

The following job classifications within the Sheriff's Office


have been identified as "at risk" to occupational exposure
to blood borne pathogens or other communicable
diseases:
1. All sworn officers;
2. All corrections officers;
3. All reserve officers; and
4. Civilian employees assigned to the following units:
a. Latent Print Unit;
b. Chaplaincy personnel;
c. Central Records Unit personnel assigned to handle duties
at the Public counter involving fingerprinting of persons;
d. Property and Evidence Unit Personnel;
e. Security Guards;
f. Bailiffs; or
g. Department of Corrections (DOC) personnel

All Sheriff's Office employees who have been identified as


having exposure risk to blood or other potentially
infectious materials will be offered ________________
_________, after receiving training and within ten days of
initial assignment.
the Hepatitis B vaccine

The vaccine will be offered as soon as possible, but in no


event later than ________, to all unvaccinated employees
who have rendered assistance in a first aid incident
involving the presence of blood or other potentially
infectious material regardless of whether the employee
has incurred an exposure incident.
24 hours

Employees who initially decline the vaccine but later wish


to have it may receive it at what cost.
No cost

Training for all Sheriff's Office employees in at risk


positions will be conducted at entry-level positions and
prior to being assigned duties where occupational
exposure could occur. Training of employees will include
an explanation of the following:
1. State and/or Occupational Safety and Health
Administration (OSHA) standards for bloodborne
pathogens;
2. Epidemiology and symptomatology of bloodborne
diseases;

3. Modes of transmission of bloodborne pathogens;


4. The Sheriff's Office General Order on communicable
diseases, including exposure control procedures;
5. Procedures which might cause exposure to blood or
other potentially infectious materials;
6. Protective measures taken to prevent exposure to blood
or other potentially infectious materials;
7. Personal protective equipment provided by the Sheriff's
Office;
8. Post exposure evaluation and follow-up procedures;
9. Signs, labels, and color coding used at the Sheriff's
Office regarding biohazards; and
10. The Hepatitis B vaccination program of the Sheriff's
Office.

Questions in reference to training materials should be


directed to
Infection Control Disease Nurse

The Sheriff's Office will at least _________ review the


exposure control plan with consideration for updating
procedures designed to eliminate or minimize
occupational exposure.
annually

What is the incubation time for Human Immunodeficiency


Virus/Acquired Immune Deficiency Syndrome (HIV/AIDS)
May be up to ten years
What is the infectious agent of Influenza?
Influenza Type A,B,C

Which Influenza type is associated with widespread


epidemics and pandemics?
Influenza Type A

What is Influenza type A isolated from?


humans, horses, swine, and birds

Which Influenza are isolated from humans only?


* Influenza Type B
* Influenza Type C

Which Influenza is infrequently associated with regional


and widespread epidemics?

Influenza Type B

Which Influenza is associated with sporadic cases and


minor localized outbreaks?
Influenza Type C

Which antigenic variation consists of a minor or gradual


changes in HA and NA and is only associated with
Influenza Types A and B?
Antigenic drift

Which antigenic variation consists of a major change in H


and A typically associated with pandemics and only
happens with Influenza Type A?
Antigenic shift

What is the incubation period for Influenza?


1 to 3 days

What is the period of communicability for Influenza?


3 to 5 days from clinical onset

What is the preventive measure for Influenza?


Vaccination

What are the identifying signs and symptoms of


Menigococcal Meningitis?
* Intense headache
* Stiff Neck
* Petechial rash

What is the case fatality rate for Menigococcal Meningitis?


5% to 15%

What is the infectious agent for Menigococcal Meningitis?


Neisseria meningitides

What is the treatment for Menigococcal Meningitis?


*Penicillin initially IMMEDIATELY
*DO NOT wait for lab results to return
*Ampicillin

What is the occurance of Menigococcal Meningitis?


Groups B, C, and Y are common to the U.S.

What is the incubation period for Menigococcal


Meningitis?
3 to 4 days

Meningococci usually disappear from the nasopharynx


how long after treatment?
Within 24 hours

What variety of diseases are caused by Group A


streptococci?
* Streptococcal sore throat
* Scarlet fever
* Rheumatic fever

What are the signs and symptoms of streptococcal sore


throat?
* Sore throat
* Exudative tonsillitis
* Pharyngitis
* Tender, enlarged cervical lymph nodes

Scarlet fever occurs when the infecting strain of


streptococcus produces what toxin?
Pyrogenic exotoxin

What are the signs and symptoms of scarlet fever?


* Same SNS as sore throat
* Accompanying rash that feels like sandpaper
* Strawberry (swollen) tongue

What is the infectious agent of streptococcal diseases?


Streptococcus pyogenes

What streptococci causes the majority of infections?


Group A

What is the treatment for streptococcal diseases?


Benzathine Penicillin G-IM

What is the incubation period for streptococcal diseases?


1 to 3 days

What is the leading cause of death in adults from an


infectious disease around the world?
Tuberculosis

What is the most common type of tuberculosis around the


world?
Pulmonary tuberculosis

What are the early symptoms of tuberculosis?


* Fatigue
* Fever
* Night Sweats
* Weight Loss

What are the advanced symptoms of tuberculosis?


* Cough
* Hemoptysis
* Hoarseness
* Chest Pain

What is the infectious agent for tuberculosis?


Mycobacterium tuberculosis

What are the clinical signs and symptoms of tuberculosis?


* Reactor to PPD-S
* Chest X-ray

What is the confirmatory diagnosis for tuberculosis?


Isolation of Mycobacterium tuberculosis on culture

What is the treatment of choice for tuberculosis?


Isoniazid- INH

What is the incubation period for tuberculosis?


2 to 10 weeks

What increases the risk for tuberculosis and shortens the


incubation period?
HIV

What are the signs and symptoms of diphtheria?


* Characteristic lesion
* Enlarged, tender cervical lymph nodes

Describe the characteristic lesions of diphtheria.


A patch or patches of an adherent grayish membrane with
a surrounding inflammation

What is the infectious agent of diphtheria?


Corynebacterium diptheriae

How is diphtheria diagnosed?


Confirmed by bacteriologic examination of lesions

What should be given for diphtheria, immediately after


bacteriologic specimens are taken, without waiting for
results?
Antitoxin (only antitoxin of equine origin is available)

What is the incubation period for diphtheria?


2 to 5 days

What is the preventive measure for diphtheria?


Active immunization

When controlling a patient with diphtheria, what must be


considered?
Isolation of one person versus Quarantine of a groups of
individuals

What is the most characteristic clinical feature of Rubella


that precedes the rash by 5 to 10 days?
Postauricular, occipital and posterior cervical
lymphadenopathy

What is a mild febrile viral disease characterized by


diffuse punctate and maculopapular rash resembling that
of measles or scarlet fever?
Rubella

What is Rubella also known as?


German Measles

What are the characteristic signs and symptoms of


Rubella?
* Conjunctivitis
* Postauricular, occipital and posterior cervical
lymphadenopathy

What is the infectious agent for Rubella?


Rubella virus

What laboratory test is used to confirm Rubella?


Fourfold rise via ELISA

What is the incubation period for Rubella?


* Range: 14 to 21 days
* Average: 14 to 17 days

When are patients highly communicable when infected


with Rubella?
1 week before and at least 4 days after onset of rash

What is an identifying sign and symptom of Rubeola


(Measles)?
Koplik spots on the buccal mucosa

What is another name for Rubeola?


Measles

What are the complications that can come with Rubeola?


* Otitis media
* Pneumonia

A herd immunity of > how much may be needed to


interrupt transmission?
94%

What is the infectious virus of Rubeola?


Measles virus

Pneumococcal Pneumonia is an acute bacterial infection


characterized by what "tall-tale" sign and symptom?
Productive cough of "rusty" sputum

What is the infectious agent of Pneumococcal


Pneumonia?
Streptococcus pneumoniae

What is the treatment for multiresistant strains of


Pneumococcal Pneumonia infections?

* Vancomycin
or
*Chloramphenicol

What is a method of control for Pneumococcal


Pneumonia?
Administration of polyvalent vaccine to high-risk persons

Legionellosis is an acute bacterial disease with what two


currently recognized, distinct clinical and epidemic
manifestations?
* Legionnaire's disease
* Pontiac fever

What is the infectious agent of Legionellosis?


Legionellae pneumophila

What is the treatment for Legionellosis?


E-mycin

What is the occurance of Legionellosis?


Sporadic cases and outbreaks are recognized more
commonly in summer and autumn

What is the reservoir for Legionellosis?


Primary aqueous:
-Hot water systems (showers)
-Air-conditioning cooling towers
-Evaporative condensers
-Humidifiers
-Whirlpool spas
-Respiratory therapy devices
-Decorative fountains
-Hot and cold water taps and showers
-Hot tubs
-Creeks and ponds and the soil from their banks

What is the mode of transmission for Legionellosis?


Epidemiologic eveidence supports airborne transmission

What is the incubation period for Legionellosis?


* Range from 2 to 10 days
* Average from 5 to 6 days

What is the common cold?


An acute catarrhal infection of the upper respiratory tract
lasting 2-7 days

What are the major etiologic agents of the common cold


in adults?
Rhinoviruses

Where is the highest incidence of the common cold?


Children under 5 years and gradually declines with
increasing age

What are the signs and symptoms of the common cold?


* Coryza
* Sneezing
* Lacrimation
* Irritated nasopharynx
* Chilliness
* Malaise

What is the approximate number of organisms required to


cause disease?
Dose

What is caused by ingestion of a pathogenic organism


which multiplies within the gastrointestinal tract?
Infection

Infections usually have longer incubation periods than


intoxications and are accompanied by what?
fever

Typically how long are infections?


6 hours to 45 days

What is caused by ingestion of toxins which are formed in


tissues of certain plants and animals?
Intoxication

Is fever often present or absent with intoxications?


absent

What is the incubation period for food intoxication?

* short
* 1 min to 6 hours

What types of poisons in food are there?


* Mushroom poisoning
* Poisonous plants
* Paralytic shellfish poisoning (PSP)
* Ciguatera fish poisoning
* Scrombroid dish poisoning
* Organophosphorous insecticides
* Cooking materials contaminated with toxic metals

What are the predominant signs and symptoms for


mushroom poisoning?
* Abdominal pain
* Muscle cramps

What food are involved with mushroom poisoning?


* Amanita (think super mario)
* Galerina (Brown)
* Gyromitra (Brain shaped, no stalk)

What are the etiologic agents of mushroom poisoning?


* Toxins
* Alkaloids

How many species of poisonous plants are known to


cause illness or death?
more than 700

Approximately how long does it take for paralytic shellfish


poisoning to resolve itself?
2 to 3 days

What is the source of poisoning for Paralytic shellfish


poisoning (PSP) ?
* dinoflagellates (algae) in shellfish with concentrated
neurotoxins
* Associated with massive algae blooms known as "red
tide"

What is the occurence of Paralytic shellfish poisoning


(PSP)?
* West Coast: May to Oct
* East Coast: Aug to Oct

What are the tell-tail sign and symptoms of Ciguatera fish


poisoning?
* Vomiting
* Hypotensive
* Pain and weakness in lower extremities
* Temperature reversal

What is the source of poisoning for ciguatera fish


poisoning?
* Dinoflagellate Gambierdiscus toxicus
* Released from algae
* Small fish eat algae
* Larger fish eat smaller fish
* Toxin Magnified through food chain

What is the preventive measure for Ciguatera fish


poisoning?
Avoid eating large predatory fish

What are the signs and symptoms of scombroid fish


poisoning?
* Tingling and burning sensation around the mouth
* Facial Flushing
* Rash

What is scombroid fish poisoning also known as?


Histamine poison

What is the preventive measure for scombroid fish


poisoning?
Adequate refrigeration and irradiation

What are the signs and symptoms of organophosphorous


insecticides?
* Blurred vision
* Chest pain cyanosis
* Twitching, convulsions

To prevent poisoning from cooking materials


contaminated with toxic metals, be sure to not use acidic
foods in what types of material/cookware?
* cadmium materials
* galvanized cookware

What disease is caused by water containing nitrates


greater than 45 ppm and is especially dangerous to
children?
Methmo-globinemia

What is the most common form of botulism in the USA?


Intestinal (infant)

What is a severe intoxication resulting from ingestion of


preformed toxins in contaminated foods?
Foodborne botulism (classical)

What are the signs and symptoms of foodborne botulism?


* Acute cranial nerve impairment
* Blurred or double vision
* Dry mouth
* Vomiting

What are the signs and symptoms of intestinal (infant)


botulism?
* Loss of head control
* Hypotonia (floppy baby)
* May develop respiratory arrest

What is the infectious agent of botulism?


Clostridium botulism

What type of botulism is related to fish, seafood, and


meat from marine mammals?
Type E

What is the treatment for botulism?


1 vial of polyvalent (AB or ABE) botulinum antitoxin (IT stat)

Who is the only carrier of the treatment for botulism (1


vial of polyvalent (AB or ABE) botulinum antitoxin) ?
CDC Atlanta

Where is the occurance of botulism?


More common where home canning is popular

What is the reservoir for botulism?

* Soil
* Marine sediment
* Honey

What is the period of communicability for intestinal


(infant) botulism?
Toxins are found for weeks to months in feces

What is the infectious agent of Campylobacter Enteritis?


Campylobacter jejuni

What is the treatment for Campylobacter Enteritis?


* None
* Electrolyte replacement

What are the reservoirs for Campylobacter Enteritis?


* Most raw poultry
* Cattle
* Puppies or kittens

What is the mode of transmission for Campylobacter


Enteritis?
* Undercooked chicken
* Contaminated food or water

What is the susceptibility and resistance of


campylobacter enteritis?
Immunity to serologically related strains follows infection

What is a sign and symptoms of Clostridium Perfringens?


Colic followed by diarrhea

What strain of Clostridium Perfringens causes necrotizing


enteritis?
Type C

What is the reservoir for Clostridium Perfringens?


* Soil
* GI tract of healthy persons and animals

Almost all outbreaks of Clostridium Perfringens are


associated with inadequately heated or reheated what?
* Meats
* Stews

* Meat Pies
* Gravies

What is a tell-tale sign and symptom of Staphylococcus


aureus?
Projectile vomiting

What is the infectious agent of staphylococcus


intoxication?
Staphylococcus aureus

What percentage of people are reservoirs/ carriers of


staphylococcus aureus?
25%

What has associated conditions of infected cuts, boils,


acne, and URI?
Staphylococcus aureus

What is a preventive measure for staphylococcus aureus?


Keep hot foods hot (above 140F) and cold foods cold
(below 41F)

Salmonellosis almost always has what present?


Fever

Incidence rates of salmonellosis are highest in what


group?
Small children

What is the mode of transmission for salmonellosis?


Fecal oral

For salmonellosis, what is the mode of transmision


associated with?
* Raw or undercooked eggs or egg products
* Raw milk
* Meat
* Poultry

What is the incubation period range for Salmonellosis?


6 to 72 hours

What agents are contraindicated for Shigellosis and may


prolong the illness?
Antimotility

What is the occurance of Shigellosis?


Peaks in summer

What is the mode of transmission of Shigellosis?


Direct or indirect fecal-oral from a patient or a carrier

What is the average incubation period for Shigellosis?


1 to 3 days

What vector is commonly associated with Shigellosis?


Flys

What is a method of control for Shigellosis?


Control Flies

How many successive ( 24 hours apart) negative fecal


samples greater than 48 hours after antibiotic therapy
must Food and Childcare workers provide for Shigellosis
and Escherichia Coli?
2

What are the signs and symptoms of Typhoid fever?


* Sustained fever
* Severe headache
* Rose spots on the trunk in 25% of white patients
* Typically constipation more than diarrhea

What is the infectious agent for Typhoid fever?


Salmonella typhi

What is the treatment for Typhoid fever?


Chloramphenicol

What is the average incubation period for Typhoid fever?


8 to 14 days

What is the method of control for Typhoid Fever?


* Control flies
* Immunization with booster doses every 2 years

How many successive ( 24 hours apart) negative fecal


samples greater than 48 hours after antibiotic therapy
must Food and Childcare workers provide for Typhoid
Fever?
3

What are the signs and symptoms for Cholera?


* Rice watery stools
* Rapid dehydration
* Acidosis
* Circulatory collapse

Classic and ET Tor are both biotypes associated with what


disease?
Cholera

What is the infectious agent for Cholera?


Vibrio cholerae

What is the treatment for Cholera?


* Prompt fluid replacement with oral rehydration (1 liter
solution)
* Treat for shock
* Antimicrobials

What is the average incubation period for Cholera?


2 to 3 days

What are the signs and symptoms of Vibrio


Parahemolyticus?
* dysentery-like illness
* bloody or mucoid stools
* high fever
* high WBC

What is the infectious agent for Vibrio Parahemolyticus?


Vibrio parahaemolyticus

What does Vibrio parahaemolyticus require to produce


optimal growth?
High levels of salt

What is the reservoirs for vibrio parahemolyticus?

*
*
*
*

Marine silt (cold season)


Coastal waters (warm season)
Fish (warm season)
Shellfish (warm season)

What is the mode of transmission for Vibrio


parahemolyticus?
* Ingestion of raw or inadequately cooked seafood
* Cross-contamination by handling raw seafood or rinsing
with contaminated seawater
What is special about the period of communicability for
vibrio parahemolyticus?
Non-communicable from person to person

When preventing vibrio parahemolyticus, how must you


make sure to heat your seafood?
15 minutes at 160F

Escherichia coli is a self-limiting illness that last how long


in average?
8 days

What is the infectious agent for Escherichia coli?


Escherichia coli

Escherichia coli is cultured using what media?


McConkey-sorbitol

What is the treatment for Escherichia coli?


* Fluid and electrolyte replacement
* Antibiotic treatment is uncertain

What is the reservoir for Escherichia coli?


* Cattle
* Humans

What is the period of communicability for adults with


Escherichia coli?
1 week

What is the period of communicability for children with


Escherichia coli?
3 weeks

How must beef be cooked in order to prevent Escherichia


coli?
155F internal temperature
Power to create disease
Virulence

Sites of nosocomial infections in order from most to least


common
Respiratory tract, Surgical site wounds, GI tract, Urinary
tract, Blood (bacteremia) and skin.

Occurs when pathogens change in ways where they can


survive and reproduce in the presence of antibiotics that
once stalled or killed them
Antimicrobial Resistance

Microorganism, only replicates in a host, infects other


cells
Virus

Singled celled, small, capable of growing outside of


human cells
Bacteria

Eukaryotic organisms, acquired from contact,


molds/yeasts, resist destruction by the body
Fungi

Lives in or on another organism, feeding off that organism


and using that organisms nutrition
Parasites

What are opportunistic organisms?


Microorganisms that are not usually considered pathogens.
However, they may cause infection if the resistance of the
host is decreased from events such as immunosuppression,
trauma, illness

Modes of transmission
Droplet, Vector (vehicle transmission... aka mosquitoes),
Airborne (droplet nuclei, remain suspended in air, varicella)

Period between exposure to an infection and the


appearance of the first symptoms, varies with disease
Incubation

Period between the appearance of initial symptoms and


the full development of a rash or fever
Prodromal phase

Full manifestation of symptoms


Chronic illness

Period of time after acute symptoms to full recovery of


health
Convalescence

Most common pathogens for nosocomial infections


Staphylococci, Pseudomonas, Enterobacter aerogenese,
and Escherichia coli

immunity produced by individuals own response


Active

Immunity developed in another individual and transferred


Passive

Immunity obtained through normal everyday living


Natural

Immunity obtained through a vaccine


Artificial

Contact with antigen through infection, chickenpox,


measles, mumps is which kind of immunity
active natural

Injection of live or killed antigen


Active artificial

Immunity that occurs from mother to child in utero or


through breastfeeding
passive natural

Immunization with serum from immune human

Passive artificial

What are the types of communicable diseases?


Measles (Rubeola), Mumps, Rubella (german measles),
Mononucleosis, Influenza

How do you know to differentiate between Measles


(Rubeola) and Rubella (german measles) aka.. completely
different virsuses....
Koplik spots on buccal mucosa

Viral disease, transmitted by direct contact with droplets


of an infected person..
Measles (Rubeola)

Present in nasal secretions, blood, stool, and urine of


infected persons
Rubella

Measles (Rubeola) more dangerous for


babies

Rubella more dangerous for


pregnant mother

Purple under skin


Purpura

purple dots under skin


Petechiae spots
Is Chicken Pox Bacterial or Viral?
Viral

What systems are affected with Chicken Pox?


Respiratory and Integumentary

What is the incubation period of Chicken Pox?


2-3 weeks

What is the communicability span of Chicken Pox?


1 day before the lesions appear and until all have crusted
over

What are some signs and symptoms for the Chicken Pox?
Fever, Fatigue, Upper Respiratory Infection
Maculare Rash that is more concentrated toward the center
at the body and thins out towards the distal part of the
body.

What are Diagnostic Studies that could be run for Chicken


Pox?
Look at the patient history, Determine if the rash is what
you would expect to see, Viral Culture and Antibody Screen

What are some items you can do for Therapeutic


Management when treating the Chicken Pox?
Acyclovir (medication), Varicella Zoster Immune Globulin,
Antipruritics, Analgesia, Oatmeal Baths, Avoiding Products
Containing Aspirin

What immunizations can be given to help prevent


Chicken Pox ?
Varicella Vaccine given at 12-15 months of age, and again
at 4-6 years

What are some complications that Chicken Pox can give?


SEcondary Bacterial Infection, Cellulitis, Pneumonia,
Encephalitis

What kind of Nursing Care should you implement for a


client with Chicken Pox?
Droplet and Contact Precautions
Skin care (Calamine and Oatmeal Baths)
Diversional Activities
Keep environment Cool
Have them apply pressure instead of scratching

As a nurse what can we diagnose with someone who has


the Chicken Pox?
Impaired skin integrity, Impaired comfort, Isolation, Risk for
infection

Is Mono Viral or Bacterial?


Viral

What are some signs and symptoms for Mono?

Fever, Fatigue, HA, Sore Throat, Painful Lymphadenopathy,


Epigastric Pain, Spleenomegaly

What are some Diagnostic Studies that can be run for


Mono?
Heterophil Antibody Screen, Monospot,
CBC with Differential Liver Enzymes

What is some Therapeutic Management techniques that


could be implemented for Mono?
Encourage Support from the family, Antipyretics,
Encourage Hydration, Corticosteroids, Rest, Give Throat
lozenges or gargles

What immunizations can be given to prevent Mono?


There are currently no Immunizations

What are complications that can occur due to Mono?


Though couching is the front runner other complications
follow shortly behind such as:
Tonsillitis
Ruptured Spleen
Hepatitis
Hemolytic Anemia

What Nursing care should be implemented for those


suffering from Mono?
Droplet and Contact Precautions
Frequent PO fluids and Soft Food
Restful Environment
Education on Avoiding Contact Sports till the Spleen returns
to normal
Analgesias
Throat Gargles and Lozenges

What are some items we can Diagnose for those with


Mono?
Isolation
Risk for Injury (Spleenomegaly)
Activity Intolerance
Risk for infecting others
Imbalance in Nutrition

What is the incubation period for Mono?


10 days to 6 weeks

What is the transmission for Chicken Pox?


Droplet and Contact

What are the effected Systems when dealing with Mono?


Respiratory, GI, Lymph System

How long is Mono Communicable?


Prodromal phase through the acute phase.

Is Mumps Viral or Bacterial?


Viral

What systems are affected by Mumps?


Respiratory, CNS, Exocrine Glands

How is Mumps translated?


Droplet and Contact

What is the incubation period for Mumps?


12-21 days

What is the communication period for Mumps?


Immediately before to 1-3 days after after swelling begins.

What are signs and symptoms for Mumps?


Fever, Fatigue, HA, Earache, Painful swelling of Parotid and
Submandibular Swelling, Swelling Testicles

What are some Diagnostic Studies that can be run for


Mumps?
Look at the Pt. history and symptoms, Radiography studies
( CT, US), Sputum and Nasopharyngeal cultures

What are some Therapeutic Management Techniques that


can be implemented for someone dealing with Mumps?
Antipyretics, Analgesics Adequate Hydration, Warm or Cool
Compresses, Monitor for Complication

What is an immunization that cane be given to prevent


Mumps?
MMR at 12-15 months with a second dose at 4-6 years.

What are some complications that can occur from


Mumps?
Deafness, Encephalitis and Meningitis, Myocarditis,
Hepatitis Pancreatitis, Epididymoorchitis & Oopheritis, or
Sterility

What Nursing care can you implement with someone


suffering from Mumps?
Place the Client on Contact or Droplet Precautions,
Frequent PO fluids, Avoid chewing, Scrotal Support (ICE),
use of Analgesia

What are some Nursing Diagnosis for Mumps?


Impaired Comfort, Isolation
Risk for Injury (r/t complications)
Activity Intolerance
Risk for Infection to others

Is Rubella a Virus or Bacteria?


Virus

What Systems are affected due to Rubella?


Respiratory and Integumentary

How is Rubella transmitted?


Droplet and Contact

What is the incubation period for Rubella?


14-21 days

What is the communicable period for Rubella?


7 days before until 5 days after the rash appears (total of
15 days)

What are some signs and symptoms of Rubella?


At first may be asymptomatic; have a low grade fever,
cough upper respiratory tract infection, lymphadenopathy,
Maculopapular Rash

What are some Diagnostic tests that can be run for


Rubella?
A blood test for IGM or IGA antibodies

What are therapeutic management techniques that can


be used while treating someone with Rubella?
Antipyretics
Antipuritics
MUST stay away from pregnant women

What are immunizations that could be given to prevent


Rubella?
MMR given 12-15 months of age and again at 4-6 years of
age

What are some complications from Rubella?


Very dangerous to the unborn fetus, transplacental
transmission

What are some interventions you can implement with


someone who is dealing with Rubella?
Put droplet and contact precautions into place
Avoid contact with pregnant women
Apply cool compress to the rash
Administer antipyretics and antipuritics for comfort as
ordered

What are some Nursing Diagnoses that can be


implemented to some one dealing with Rubella?
Impaired comfort
Ineffective Health Maintenance
Risk for injury
Risk for Infection

Is Pertussis a Bacteria or a Virus?


Bacteria

What systems are effect with Pertussis?


Respiratory, GI, CNS

How is Pertussis transmitted?


Droplet and Contact

What is the Incubation period of Pertussis?


6-20 days

What is the communicable period of Pertussis?


5-8 weeks

What are some signs and symptoms of Pertussis?


1-2 Weeks: Fever, Fatigue URI
4-6 Weeks: Harsh Productive cough with copious amounts
of fluid followed by high pitched inspiration (Jeff Gordon
Commercial)

What are diagnostic studies that can be run for Pertussis?


Patient History, Characteristics of Cough, Sputum and Naso
Pharyngeal Cultures

What are therapeutic management techniques that can


be used while treating someone with Pertussis?
Antibiotics, Bronchodilators, Corticosteroids, Humidified
Oxygen, Respiratory Support

What are immunizations that could be given to prevent


Pertussis?
Dtap:2,4,6,18 months and again at 4-6 years
All adults should receive a Tdap every 10 years

What are some complications of Pertussis?


Pneumonia, Atelectasis, Seizures, Hemorrhage, Hernias

What are some Nursing interventions you can implement


for someone suffering from Pertussis?
Droplet and Contact Precautions
Frequent intake of fluids
Humidified 02
Irritant free Environment

What are some Nursing Diagnosis's for Pertussis?


Ineffective Breathing Pattern
Ineffective Airway Clearance
Impaired Comfort
Activity Intolerance

Is Rubeola a Virus or Bacteria?


Virus

What systems are effected due to Rubeola?


Respiratory, Integumentary, CNS, Eyes

How is Rubeola transmitted?

Droplet and Contact?

What is the Incubation period of Rubeola?


10-20 days

What is the communicable period of Rubeola?


4 days before till 5 days after the rash appears

What are some signs and symptoms of Rubeola?


3-5 Days: Fever Fatigue, Inflamed Nasal Passages, Cough,
Conjunctivitis, KOPLIK SPOTS
Acute Phase: Maculopapular Rash, Lymphadenopathy,
Photophobia

What are diagnostic studies that could be done for


Rubeola?
Positive measles IGM anti-bdy test of respiratory Mucosa

What are therapeutic management techniques that can


be used while treating someone with Rubeola?
Vitamin A supplement
Antipyretics for fever
Relaxing Environment
Analgesia
Antibiotics to prevent secondary infection

What are immunizations that could be given to prevent


Rubeola?
MMR at 12-15 Months and again at 4-6 years
Do not give before 12 months of age or the mothers
antibodies that are still active in the child will destroy the
immune factors in the Vaccination.

What are some complications of Rubeola?


Pneumonia, Encephalitis, Otitis Media

What are some Nursing interventions you can implement


for someone suffering from Rubeola?
Put on Droplet and Contact Precautions
Promote restful environment
Use a cool compress for rash and to decrease fever
Administer Antipyretics and Antipuritics for comfort

What are some Nursing Diagnosis's you can implement


for care of Rubeola?
Impaired comfort
Ineffective Thermoregulation
Ineffective Health Maintenance
Risk for infection
Risk for injury

Is Scarletina a Virus of Bacteria?


Bacteria; Group A beta-hemolytic Strep

What systems are affected by Scarletina?


Respiratory, Renal, Cardiovascular

How is Scarletina transmitted?


Contact and Droplet

What is the incubation period for Scarletina?


2-4 days

What is the Communicable period for Scarletina?


10 day during incubation and acute phase

What are some signs and symptoms of Scarletina?


Abrupt High Fever, HA, Chills, Fatigue, halitosis, Sever sore
throat, tonsillitis, WHITE STRAWBERRY TONGUE

What are some Diagnostic tests you can do for


Scarletina?
Throat culture, CBC with differential, Blood Cultures

What are therapeutic management techniques that can


be used while treating someone with Scarletina?
Full course of Antibiotics, Antipyretics, Analgesia

What immunizations can you have for Scarletina?


There are currently no immunizations to treat this at this
time.

What are some complications with Scarletina?


Rheumatic Fever, Glomerulonephritis, Peritonsilar abscess

What are some nursing intervention that can be put into


place for someone suffering from Scarletina?

Teach client importance of medication completion


Comfort Measures
Prevent Spread and Recurrence by teaching about mode of
transmission
Institute droplet precautions

What are some Nursing diagnoses that can be given for


Scarletina?
Ineffective thermoregulation
Risk for Injury
Risk for ineffective Health Maintenance
Risk for infection

Is Polio a Virus or Bacteria?


Virus that attacks the grey matter in the spinal cord

What systems are affected by Polio?


CNS, Respiratory, Musculoskeletal, GI

How is Polio transmitted?


Fecal-Oral, Droplet, Contact

What is the incubation period for Polio?


7-38 days

What is the communicable period of Polio?


up to 4 weeks

What are the Signs and Symptoms of Polio?


Inapparent:Flu Symptoms
Non paralytic: Flu symptoms with neck pain,back
and legs
Paralytic: Starts with fly symptoms,the signs of
paralysis

What are diagnostic tests that can be done for Polio?


Throat Culture, Stool Culture, CSF culture, Antibody screen
for Polio Virus

What are therapeutic management techniques that can


be used while treating someone with Polio?
Treat Urinary Retention, Prophylactic Antibiotics, Heat to
reduce muscle spasm, Analgesics, Respiratory Support, IV
hydration and Nutrition, Orthotic Support

What is an immunization that can be given to prevent


Polio?
IPV at 2,4,6 months and again at 4-6 years of age for a
total of 4 doses.
However if there 3rd does was administer at age 4 then
there is only a need for 3 doses.

What are some complications of Polio?


Permanent Muscle Paralysis, Kidney stones from
demineralization of bones, Post Polio Sequelae

What are some Nursing intervention that can be


implemented for those suffering from Polio?
Assess for Urinary Retention
Assess Respiratory Effort and Effectiveness of Breathing
Moist heat for Muscle spasms
Insure Adequate Nutrition (High Protein)
Assistance with ADL's
Frequent Repositioning (Pneumonia and risk for
PU)
Physical Therapy and Strengthening Exercises

What are some Nursing Diagnoses that for Polio?


Risk for Ineffective breathing
Risk for nutritional deficit
Risk for Impaired Skin Integrity
Anxiety
Activity intolerance
Impaired comfort

Is Tetanus a Virus or a Bacteria?


Bacterial which causes powerful doxin which attacks the
CNs

How is Tetanus transmitted?


Bacteria enters an open wound

What is the Incubation period of Tetanus?


3 days to Several weeks

What are some signs and symptoms of Tetanus?


Muscle spasms with stiffness of the muscles in the neck
and jaw (Lock Jaw)

What diagnostic studies are available to determine if


tetanus is present?
According to the handout none

What are therapeutic management techniques that can


be used while treating someone with Tetanus?
Flagyl, Tetanus Immunoglobulin, Antitoxin, Muscle
Relaxants, Muscle relaxants, IV Hydration and Nutrition

What are some immunizations that can be given to


prevent tetanus?
Dtap at age 2,4,6,18 months and again at 4-6 years of age
After age 7 Tdap is given every 10 years as a booster

What is the survival rate of Tetanus?


90% with appropriate treatment.

What are some complications associate with Tetanus?


Respiratory Arrest, Cardiac Arrest, Decreased CNS function,
Bone fractures

What are some Nursing Interventions that can be


implemented when caring for someone with Tetanus?
Airway Support, Comfort Measures, Monitoring Hydration
and Nutrition status,Administer Antibiotics, Antitoxins,
Muscle Relaxants, and Anti-anxiety Medication as
prescribed

What are some Nursing Diagnoses for Tetanus?


Impaired Physical Mobility
Impaired Comfort
Inadequate Nutrition
Risk for impaired gas exchange
Risk for Injury
Anxiety

What systems are affected by Tetanus?


Musculoskelatal, Respiratory

Is Diptheria a Virus or Bacteria?


Bacteria

What systems are affected by Diptheria?

Respiratory and Integumentary

How is Diptheria transmitted?


Contact

What is the incubation period of Diptheria?


2-5 days

What is the Communicable period Diptheria?


2-4 weeks

What are some signs and symptoms of Diptheria?


Prodromal: URI
Acute Stage: White or Grey Membranous coating,
Hoarseness, Skin irritation from Nasal discharge, Severe
Lymphadenopathy

What are some diagnostic studies that can be run to test


for Diptheria?
Throat Culture, Gram Stain, CBC with differential
Skin testing for allergy to horse serum

What are therapeutic management techniques that can


be used while treating someone with Diptheria?
Early Administration of Antibiotics, Diptheria Antitoxin, Bed
rest, Possible Tracheostomy

What immunizations can be given to prevent Diptheria?


Dtap 2.4.6 and 18 months
Repeat Dtap at 4-6 years of age.
After 7 years of age Tdap given every 10 years

What are some complications associated with Diptheria?


Septic Shock, Cardiomyopathy, Neuropathy

What are some Nursing Interventions that can be


implemented when caring for someone with Diptheria?
~Follow contact precautions until 2 negative cultures are
achieved
~Administer Antibiotics ASAP
~Administer Antitoxin after assess for horse Serum Allergy
~Strict bedrest to prevent cardiomyopathy
~Administer humindified oxygen for decreased 02 sats

~Assess respiratory effort


~Skin care for Nasal discharge and skin breakdown

What are some Nursing Diagnoses for Diptheria?


Ineffective airway clearance
Ineffective Thermo Regulation
Risk for Injury
Ineffective Health Maintenance
Risk for Infection
Communicable Disease
Illness that is directly or indirectly transmitted from one
person or animal to another by contact with body fluids,
with contaminated objects, or by vectors (e.g., ticks,
mosquitoes)

Risk Factors for Infectious and Communicable Diseases in


Newborns and Infants
Immature immune system
Passively acquired maternal antibodies provide limited
protection
Disease protection through immunization is incomplete

Risk Factors for Infectious and Communicable Diseases in


Children
Limited prior exposure to communicable diseases
Poor health
Immunodeficiency

Common Routes of Infection Transmission in Children


Behaviors that lead to the spread of infection
Traits of children- diapers, sharing toys, poor hand hygiene

Nurse's Role
in Infection Control
Promote infection control strategies
Report infectious and communicable diseases to state
health officials

Diseases of childhood and adolescence

Measles, mumps, rubella, polio, pertussis, diphtheria,


tetanus, Haemophilus influenza type B, hepatitis B,
hepatitis A, pneumococcus, varicella, rotovirus,
meningococcal diseases, human papillomavirus

Active immunity
An antigen is introduced into the body in a vaccine, which
stimulates antibody production without causing clinical
disease

Passive immunity
Antibodies produced in another human or animal host is
given.
Provides immediate protection
Does not provide lasting immunity
Example: Tetanus immune globulin

Immunization Type
Inactivated
DTaP, IPV, Hep B, Hib, PCV, Hep A, Influenza

Live attenuated (weakened)


MMR, Varicella, Influenza (nasal spray)
*Contraindicated in immunocompromised individuals

Vaccine Side Effects


Local reactions
Erythema, swelling, pain, and induration

Vaccine Side Effects


Systemic reactions
Fever, irritability, malaise, anorexia (e.g., DTaP)

Vaccine Side Effects


Allergic reactions
(local and systemic - 0.012%))
Wheal, urticaria
Severe local
Anaphylaxis

Vaccine Side Effects


Other serious reactions
not the vaccine, but products in the vaccine

Vaccine Side Effects


Unique reactions
MMR- rash and fever (normal)
Varicella- pox that break out for 30 days

Misinformation about vaccines


Vaccine-preventable diseases have been eliminated.
Immunization weakens the immune system. Multiple
vaccines overload the immune system and cause harmful
effects.doesn't weaken immune system- vaccines not as strong as
used to be
It would be better to let the child get the disease than get
immunized.
Vaccines do not work. Children still get the disease.
Belief that they can control their child's susceptibility to
disease and the outcomes

Steps in Administering Immunization


Identify needed immunizations
Review the vaccine screening questions
Review the Vaccine Information Statement (VIS)

Fever Benefits
Rid of organisms that thrive in lower body temperatures
Mobilizes immune response
plasma iron concentration - limits microorganism growth

Fever Risk
When body temp reaches 41 C (105.9)
Febrile seizure

Sepsis
Definition: Overwhelming infection in the bloodstream
Causes from infectious diseases:
Infected pox
Can lead to septic shock

septic shock

systematic response to pathogenic organisms in the blood


resulting in circulatory dysfunction

Signs of Sepsis
Hypothermia or hyperthermia
Abdominal distention
Anorexia
Respiratory distress
Vomiting
Cool extremities
Mottling
Lethargy

Varicella (Chicken Pox)


Transmission: Airborne and lesion contact
Direct contact with mucous membranes or conjunctiva
Incubation: 14 - 21 days
Communicable -5 days before rash and until it crusts
Clinical manifestations:
mild fever, malaise, headache, itching with rash
Potential complications:
Secondary infection, cellulitis, encephalitis
Prevention:
Vaccine after age 12m

Nursing Management of Chicken Pox


*Airborne and contact precautions
Hx of immunization and exposure
Change linen frequently
Observe for signs of complications - drowsiness, meningeal
sign
Monitor for acyclovir side effects - nausea, vomiting,
diarrhea, h/a, allergic reactions

Management of Lesions
Use cool wet compresses or giving baths in cool or
lukewarm water (Oatmeal bath, e.g., Aveeno)
Keep fingernails short and clean.
Put calamine lotion on itchy areas

Give child foods that are cold, soft, and bland.


Acetaminophen regularly to help relieve pain if there are
mouth blisters.
Medication for itching.

Hand, Foot and Mouth Disease (Coxsackievirus)


Transmission: fecal-oral and respiratory routes
Incubation : 3-6 days
Clinical manifestations:
Lesions cheeks, gums, and tongue.
Papulovesicular lesions on hand and feet
Irritable, fever, anorexia, dysphagia
Treatment: supportive care
Prevention: avoid contact with infected person early in the
disease

Hand, Foot and Mouth Disease (Coxsackievirus)


Nursing Management
Standard and contact precautions
Topical lotions and systemic meds for pain
Cool drinks; soft, bland food
Warm saline mouth rinses
Observe for dehydration
Reassurance and support to parents
Nonaspirin antipyretics

Erythema Infectiosum (Fifth's Disease)


Transmission: Respiratory and blood
Incubation: 6-14 days
Clinical manifestations:
Flu-like illness for 2-3 days
1 week later - fiery-red rash on cheeks (*slapped face),
lacelike symmetric, erythematous, maculopapular rash on
trunk and limbs
Treatment: Supportive
Prevention: avoid contact with infected persons

Meningococcus
Transmission: respiratory droplets
Highest rates < 5 years of age.
Incubation: 1-10 days
Communicable until 24 hours after start of antibiotics
Clinical manifestations -onset is rapid and life-threatening
Onset - flu-like symptoms with prostration

Neuro signs
Meningococcemia - urticarial, maculopapular
Shock, hypotension, DIC, coma

Meningococcus (cont)
Complications
Loss of limbs d/t necrosis; hearing loss, arthritis, seizures,
pericarditis, hemiparesis
Up to 15% with invasive type die
Prevention
Vaccine for children > 2 years for persons at high-risk
Close contact - prophylaxis treatment
Treatment
Penicillin G (cefotaxime, ceftriaxone - alternatives)
In ICU
DIC treated with plasma, blood, or platelets

Meningococcus
Nursing management
Standard and droplet precautions
Monitor for shock and respiratory compromise
Monitor for fluid overload and ICP -watch fluids
Help identify close contacts that need prophylaxis
Help family identify and connect with supports
Surviving child will likely need rehab

Roseola
Transmission: likely to be respiratory
Incubation : 5-15 days
Clinical manifestations:
Sudden, high fever for 3-8 days; normal behavior and
appetite
Fever followed by pale, pink, discrete, maculopapular rash
Treatment: Supportive
Herpes Zoster (shingles)

-most painful of all


caused by the chicken pox virus
highly communicable
pallative treatment only
travels via nerve route

Hepatitus B
below the belt,
comes from infected or contaminated blood
vaginal fluids, saliva, urine, semen etc

Hepatitus C
affects the liver
a symptomatic
and causes cirrohsis or hardening of the liver

auto infection
the infection of a primary host with a parasite. Particularly
a helminth

fecal oral route diseases


Poliomyelitus . ecoli, and hepatitus A

Legionairres disease
produces high fever and pnumonia
out breaks occur in warm moist environments during
summer and winter
uncommon to those 20 years or younger
pennicillin ineffective

Hepatitus A
Treansmited through fecal oral route
ingestation of contaminated food
shellfish from contaminated waterrs
no treatment

Staff
natural inhabitant found in the human body
responsible for boils and other infections
inflammation of the stomach and intestines
source of food intoxication

Poliomyelitus
feared more than aids
fecal oral route

jonas slak created the vaccine in 1952


paralytic virus
spread through the nerve pathways
no cure , pallative treatment

Botulism
Paralytic type illness
recovery takes months or years
paralysis of the breathing muscles
casues respirtory failure

Mumps
contracting it equals life long immunity
inflammed glands

small pox
occurs only in humans
only infectious for a short time
1st communicable disease officially eradicated
not strong enough to survive.

e-Coli
can cause distruction of the kidneys
if excreted con become communicable
found in the lower intestines of warm blooded animals.
fecal oral transmission

Rubella
normally seen in children
communicable for one week
immunity accquired through vaccime or infection

salmonella
2000 different species
communicable for the duration of the infection
ingested of food derived from infected animal
transferred between human and non human
diarhea, feve and abdominal cramps

measles
disease is communicable
spread through respiratory contact
vaccination during second year of life

Histoplasmosis

affects the lungs


associated with dry cough, fever and chest pains
known as the cave disease
found in bird caves containing bat, or bird poop droppings
and chicken coops.
can survive in soil for years

Plague
spread to humans via fleas
depending on sanitation, van be spread through the air.
can come from contaminated or undercooked foods.

anthrax
used as a biological weapon
spread by spores

anthropod Vector
any tiny organism that can transport infectious pathogens
via bacteria or prion.
mosquitos
flies
mites
ticks

encephalitis
caused by bacterial infections by
rabies
west nile
measles
polio
and herpes simplex

toxic shock
caused by staff and strep
causing problems with several body ogans
can be treated with antibodies

conjunctivitus
eyes become red and inflammed
treated with drops
casued by bacteria rubbed into the eye
caused due to poor handwashing
affects the mucous membrane that lines the eyelid

rhino viruses

able to survive for a long period of time on fomites

commom cold
most common of all communicable diseases
created through rhnoviruses
bacterial exudate is clear
can be transmitted directly or indirectly

Reyes syndrome
causing detrimental harm to many organs such as the brain
and liver
associated with giving children asprin
damages cellular mitochondria

inluenza
communicable 3 to 5 days after inicial symptoms appear
Vaccine is 80 percent effective

Mono
kissing disease
part of the herpes group
smptoms are non specific
carrier has EBV on the throat
affected individuals restricted form contact sport.

meningitis
caused by a bacteria
can be treatd with antibiotics
starts with an upper respiratory infection

herpes Virus consists of


shingles
chicken pox

Herpes simplex 1
herpes type 1 causes sores around the mouth and lips
(sometimes called fever blisters or cold sores).

Alkalosis
The main cause of respiratory alkalosis is hyperventilation,
resulting in a loss of carbon dioxide having to use a paper
bag

conditions caused by strep and staff

empatigo,bacterimia, cellulitis, absesses

plantar warts
The HPV type 1 causes most plantar warts.
The virus gains access to the skin through direct contact.

endocarditus
inflammation on the inside lining o the heart

skin conditions are caused by


staph or strep

hansens disease
effects the skin and perrifrial nerves; leoprosy

necrotising facilitus
bacteria capable of deteriorating the skin and the tissues
covering the muscles due to bacteria entering into the skin.
What is an emergency?
An emergency is a serious event that
requires an immediate response. The
affected population is usually small in
scale. Examples include a house or
neighborhood fire, a car accident, or an
avalanche that traps several people.

What is a disaster?
A disaster affects many people in various
ways. Some examples of disasters include
an earthquake or a hurricane destroying a
city or several cities across a wide area, a
tornado that causes damage and death
over many miles and several cities, or a
man-made disaster such as a nuclear
bomb.

What are the elements of a disaster?


temporal
spatial
role
effects

What is the non-disaster stage of the


temporal element?

This stage includes the period of time


before or between disasters. Health
providers must conduct assessments and
identify disaster risks.

What is the pre-disaster stage of the


temporal element?
This stage occurs when disaster is
imminent but has not yet occurred. When a
warning or threat of a disaster is issued,
health officials must apprise the public
about imminent threats such as onset
hurricanes or tsunamis.

What is the impact stage of the temporal


element?
When disaster occurs, nurses must assess
the impact of the event on the community.
Nurses need to collect inventory on the
community's immediate needs, the amount
of damage to buildings and the
environment, and the type and extent of
bodily injuries.

What is the emergency stage of the


temporal element?
After a disaster, nurses and aid providers
must administer first aid and treatment to
victims. These individuals must also
perform search and rescue efforts to save
and protect as many lives as possible.
During the emergency stage, nurses must
conduct a triage of responsibilities.

What is the recovery stage of the


temporal element?
This stage focuses on returning the
community to equilibrium. Nurses work to
restore the community to a functional state
by removing debris and reestablishing
community services. The goal of the
recovery stage is to restore major
resources of the community within the first
six months after the disaster and to return
community members to their homes or
alternative shelters. During the recovery

stage, providers will clean up the area and


help businesses to re-open and schools to
resume session.

What is the evaluation stage of the


temporal element?
After the disaster, nurses conduct an
assessment of the response efforts. These
professionals will plan for preventing future
disasters and seek new ways to minimize
the impact of the event based on the
knowledge gained. During this stage, the
professionals will cycle back to the
nondisaster stage of planning and employ
the new information gained.

What is the total impact area of the


spatial element?
Total impact includes the zone where the
most severe effects of the disaster
occurredthe area of greatest damage
and destruction.

What is the partial (fringed) impact area


of the spatial element?
Partial, or fringed, impact are areas that
exhibit some visible damage but that did
not take the brunt of the damage from the
disaster.

What is the outside area of the spatial


element?
The outside area is an area that is not
affected by the disaster, but instead may
serve to aid the affected areas as a
resource for personnel, shelter, food,
intelligence, and assistance as needed.

What is a primary victim?


A person who has experienced the worst
exposure to the disaster

What is a second-level victim?


Someone affected indirectly, such as a
friend or relative of primary victims

What is a third-level victim?


A person who comes to the scene of a
disaster as a helper but is traumatized by
what he or she witnesses there

What is a fourth-level victim?


A community member who offers to help
primary victims and becomes affected by
the stories he or she hears and the injuries
he or she sees

What is a fifth-level victim?


A person not directly affected but who
suffers psychologically from the disaster

What is a sixth-level victim?


A person vicariously affected, but who has
no direct connection to the disaster

What are the primary effects of a


disaster?
The immediate effects of the disaster, such
as injury, death, and destruction of
property

What are the secondary effects of a


disaster?
Indirect effects, such as malnutrition, the
spread of disease, and homelessness

What are the physical effects of a


disaster?
Injuries, disabilities, death, and illness

What are the mental health effects of a


disaster?
Post-traumatic stress disorder, depression,
and anxiety

What are the economic effects of a


disaster?
Poverty, loss of property, and the cost of
rebuilding

What are the structural effects of a


disaster?
Loss of health care and other facilities for
support and resources

What are the social effects of a disaster?


Families displaced, loss of important
members of the community, and loss of
leadership

What is the APHA?


The American Public Health Association
(APHA) was initially formed by public
health professionals in 1872. With over 50
public health disciplines participating, the
APHA is an organized agency which
advocates for the health of entire
populations and is the largest association
of public health professionals in the world.
The APHA focuses on the development of
standards and policies to promote
population health.

What are some examples of natural


disasters?
Hurricanes
Tornadoes
Hailstorms
Cyclones
Blizzards
Droughts
Floods
Mudslides
Avalanches
Earthquakes
Volcanic eruptions
Communicable disease epidemics
Lightning-induced forest fires
Tsunamis
Thunderstorms and lightning
Extreme heat and cold

What are some examples of human-made


disasters?

Conventional warfare
Nonconventional warfare (e.g., nuclear,
chemical)
Transportation accidents
Structural collapse
Explosions and bombings
Fires
Hazardous materials incident
Pollution
Civil unrest (e.g., riots)
Terrorism (e.g., chemical, biological,
radiological, nuclear, explosives)
Cyber attacks
Airplane crashes
Radiological incidents
Nuclear power plant incidents
Critical infrastructure failures
Water supply contamination

What are some examples of Healthy


People 2020 objectives related to disaster
mitigation?
EH-21: Improve the utility, awareness,
and use of existing information systems for
environmental health.
FS-1: Reduce infections caused by key
pathogens transmitted commonly through
food.
HC/HIT-12: Increase the proportion of
crisis and emergency risk messages,
intended to protect the public's health, that
demonstrate the use of best practices.
IID-12: Increase the percentage of
children and adults who are vaccinated
annually against seasonal influenza.
IID-13: Increase the percentage of adults
who are vaccinated against pneumococcal
disease.

What is the disaster management cycle?

What are some common adult reactions


to disasters?
Extreme sense of urgency
Panic and fear
Disbelief
Disorientation and numbness
Reluctance to abandon property
Difficulty in making decisions
Need to help others
Anger and blaming
Blaming and scapegoating
Delayed reactions
Insomnia
Headaches
Apathy and depression
Sense of powerlessness
Guilt
Moodiness and irritability
Jealousy and resentment
Domestic violence

What are some common children


reactions to disasters?
Regressive behaviors (e.g., bedwetting,
thumb sucking, crying, clinging to parents)
Fantasies that disaster never occurred
Nightmares
School-related problems, including an
inability to concentrate and refusal to go
back to school

What populations are at greatest risk for


disruption after disaster?
Seniors
Vision or hearing impaired
Women
Children
Individuals with chronic disease
Individuals with chronic mental illness
Non-English-speaking
Low income
Homeless
Tourists, persons new to an area
Persons with disabilities
Single-parent families
Substance abusers
Undocumented residents

what is disease surveillance?


the ongoing systematic collection, analysis,
interpretation and dissemination of specific
health data for use in public health

What is biological terrorism?


the deliberate release of viruses, bacteria,
or other germs (agents) used to cause
illness or death in people, animals, or
plants

What is chemical terrorism?


the intentional release of hazardous
chemicals into the environment for the
purpose of harming or killing

What can public health surveillance be


used to facilitate?
Estimate the magnitude of a problem
(disease or event)
Determine the geographic distribution of
an illness or symptoms
Portray the natural history of a disease
Detect epidemics and define a problem
Generate hypotheses and stimulate
research
Evaluate control measures
Monitor changes in infectious agents

Detect changes in health practices


Facilitate planning

What are the features of surveillance?


Is organized and planned
Is the principal means by which a
population's health status is assessed
Involves ongoing collection of specific
data
Involves analyzing data on a regular
basis
Requires sharing the results with others
Requires broad and repeated contact
with the public about personal health
issues
Motivates public health action as a result
of data analyses to:
Reduce morbidity
Reduce mortality
Improve health

What are the purposes of surveillance?


Assess public health status
Define public health priorities
Plan public health programs
Evaluate interventions and programs
Stimulate research

What are some surveillance objective for


healthy people 2020?
EH-5 Reduce waterborne disease
outbreaks arising from water intended for
drinking among persons served by
community water systems.
FS-2 Reduce infections associated with
foodborne outbreaks from pathogens
commonly transmitted through food.
GH-1 Reduce the number of cases of
malaria reported in the United States.
IID-16 (Developmental) Increase the
scientific knowledge on vaccine safety and
adverse events.
PHI-2 Increase the proportion of tribal,
state, and local public health agencies that
incorporate core competencies for public
health professionals into the job.
PHI-7 Increase the proportion of

population-based Healthy People 2020


objectives for which national data are
available for all population groups
identified for the objective.

What is the sentinel surveillance system


provides for the monitoring of key health
events when information is not otherwise
available or for calculating or estimating
disease morbidity in vulnerable
populations

What types of surveillance systems are


there?
passive
active
sentinel
special

What is the passive system?


In the passive system, case reports are
sent to local health departments by health
care providers (i.e., physicians, nurses) or
laboratory reports of disease occurrence
are sent to the local health department.
The case reports are summarized and
forwarded to the state health department,
national government, or organizations
responsible for monitoring the problem,
such as the CDC or an international
organization such as the World Health
Organization.

What is an active surveillance system/


In the active system, the nurse, as an
employee of the health department, may
begin a search for cases through contacts
with local health care providers and
agencies. In this system, the nurse names
the disease or the event and gathers data
about existing cases to try to determine
the magnitude of the problem (how
widespread it is).

What is a sentinel surveillance system?

In the sentinel system, trends in commonly


occurring diseases or key health indicators
are monitored (Healthy People 2020). A
disease or an event may be the sentinel, or
a population may be the sentinel. In this
system a sample of health providers or
agencies is asked to report the problem.
The system is useful because it helps
monitor trends in commonly occurring
diseases and events.

What is the NNDSS?


National Notifiable Disease Surveillance
System (NNDSS).

What does infectivity refer to?


the capacity of an agent to enter a
susceptible host and produce infection or
disease

What does pathogenicity refer to?


Measures the proportion of infected people
who develop the disease

What does virulence refer to?


Refers to the proportion of people with
clinical disease who become severely ill or
die

What are biological agent factors?


Bacteria (e.g., tuberculosis, salmonellosis,
streptococcal infections)
Viruses (e.g., hepatitis A, herpes)
Fungi (e.g., tinea capitis, blastomycosis)
Parasites (e.g., protozoa-causing malaria,
giardiasis; helminths [roundworms,
pinworms]; arthropods [mosquitoes, ticks,
flies, mites])

What are types of physical agent factors?


Heat
Trauma

What are types of chemical agent factors?

Pollutants
Medications/drugs

What are types of nutrient agent factors?


Absence
Excess

What are types of psychological agent


factors?
Stress
Isolation
Social support

What is a common source outbreak?


refers to a group exposed to a common
noxious influence such as the release of
noxious gases (e.g., ricin in the Japanese
subway system several years ago and
more recently in a water system in the
United States)

What is a point source outbreak?


all persons exposed become ill at the same
time, during one incubation period.

What is a mixed outbreak?


a common source followed by secondary
exposures related to person-to-person
contact, as in the spreading of influenza

What are intermittent or continuous


source outbreaks?
cases may be exposed over a period of
days or weeks, as in the recent food
poisonings at a restaurant chain
throughout the United States as a result of
the restaurant's purchase of contaminated
green onions.

What is a propagated outbreak?


does not have a common source and
spreads gradually from person to person
over more than one incubation period,
such as the spread of tuberculosis from
one person to another.

What factors must be considered as


causes of an outbreak?
agents, hosts, and environmental factors

What are epidemiological clues that may


signal a covert bioterrorism attack?
Large numbers of ill persons with a
similar disease or syndrome
Large numbers of unexplained disease,
syndrome, or deaths
Unusual illness in a population
Higher morbidity and mortality than
expected with a common disease or
syndrome
Failure of a common disease to respond
to usual therapy
Single case of the disease caused by an
uncommon agent
Multiple unusual or unexplained disease
entities coexisting in the same person
without any other explanation
Disease with an unusual geographic or
seasonal distribution
Multiple atypical presentations of disease
agents
Similar genetic type among agents
isolated from temporally or spatially
distinct sources
Unusual, atypical, genetically
engineered, or antiquated strain of agent
Endemic disease with an unexplained
increase in incidence
Simultaneous clusters of similar illness in
noncontiguous areas, domestic or foreign
Atypical aerosol, food, or water
transmission
Ill people presenting at about the same
time
Deaths or illness among animals that
precedes or accompanies illness or death
in humans
No illness in people not exposed to
common ventilation systems, but illness
among those people in proximity to the
systems

How do you conduct an outbreak


investigation?
Confirm the existence of an outbreak.
Verify the diagnosis and/or define a case.
Estimate the number of cases.
Orient the data collected to person,
place, and time.
Develop and evaluate a hypothesis.
Institute control measures and
communicate findings.

What are some Healthy People 2020 goals


related to immunization and infectious
diseases?
IID-1: Reduce, eliminate, or maintain
elimination of cases of vaccine-preventable
diseases.
IID-4: Reduce invasive pneumococcal
infections.
IID-24: Reduce chronic hepatitis B virus
infections in infants and young children
(perinatal infections).
IID-27: Increase the percentage of
persons aware they have a chronic
hepatitis C infection.

What is toxicity?
The ability to produce a poisonous reaction

what is invasiveness?
the ability to penetrate and spread
throughout a tissue?

What is antigenicity?
the ability to stimulate an immunological
response

What characteristics of a host the may


influence the spread of disease?
host resistance
immunity
herd immunity
infectiousness of the host

What is resistance?

The ability of the host to withstand


infection

What is natural immunity?


refers to species-determined, innate
resistance to an infectious agent. For
example, opossums rarely contract rabies.

What is acquired immunity?


the resistance acquired by a host as a
result of previous natural exposure to an
infectious agent. Having measles once
protects against future infection

What is active immunization?


the resistance acquired by a host as a
result of previous natural exposure to an
infectious agent. Having measles once
protects against future infection

What is passive immunization?


immunization through the transfer of a
specific antibody from an immunized
individual to a nonimmunized individual,
such as the transfer of antibody from
mother to infant or by administration of an
antibody-containing preparation (i.e.,
immunoglobulin or antiserum)

What is herd immunity?


refers to the immunity of a group or
community. It is the resistance of a group
of people to invasion and spread of an
infectious agent. Herd immunity is based
on the resistance of a high proportion of
individual members of a group to infection.

What is infectiousness?
a measure of the potential ability of an
infected host to transmit the infection to
other hosts.

What is vertical transmission?

when the infection is passed from parent to


offspring via sperm, placenta, milk, or
contact in the vaginal canal at birth

What is horizontal transmission?


the person-to-person spread of infection
through one or more of the following four
routes: direct or indirect contact, common
vehicle, airborne, or vector-borne.

What is incubation period?


the time interval between invasion by an
infectious agent and the first appearance
of signs and symptoms of the disease. The
incubation periods of infectious diseases
vary from between 2 and 4 hours for
staphylococcal food poisoning to between
10 and 15 years for AIDS.

What is communicable period?


the interval during which an infectious
agent may be transferred directly or
indirectly from an infected person to
another person. The period of
communicability for influenza is 3 to 5 days
after the clinical onset of symptoms.
Hepatitis B-infected persons are infectious
many weeks before the onset of the first
symptoms and remain infective during the
acute phase and chronic carrier state,
which may persist for life.

What does endemic mean?


refers to the constant presence of a
disease within a geographic area or a
population. Pertussis is endemic in the
United States.

What is an epidemic?
refers to the occurrence of a disease in a
community or region in excess of normal
expectancy.

What is a pandemic?
an epidemic that occurs worldwide and
affects large populations. HIV/AIDS is both

epidemic and pandemic, because the


number of cases is growing rapidly across
various regions of the world, as well as in
the United States. SARS and novel
influenza A H1N1 are both emerging
infectious diseases and are responsible for
recent pandemics.

What are emerging infectious diseases?


those in which the incidence has actually
increased in the past two decades or has
the potential to increase in the near future.
These emerging diseases may include new
or known infectious diseases.

How does secondary prevention apply to


communicable diseases?
The goal of secondary prevention is to
prevent the spread of disease once it
occurs. Activities center on rapid
identification of potential contacts to a
reported case.

How does tertiary prevention apply to


communicable diseases?
To reduce complications and disabilities
through treatment and rehabilitation

What is a zoonosis?
an infection transmitted from a vertebrate
animal to a human under natural
conditions. The agents that cause
zoonoses do not need humans to maintain
their life cycles; infected humans have
simply somehow managed to get in their
way

What are the Healthy People 2020 goals


related to communicable diseases?
HIV-3: Reduce the rate of HIV
transmission among adults and
adolescents.
HIV-5: Reduce the number of new AIDS
cases among adolescents and adult
heterosexuals.
STD-8: Reduce congenital syphilis.

STD-6: Reduce gonorrhea rates.


IID-25: Reduce hepatitis B.
IID-26: Reduce new hepatitis C infections.
STD-1: Reduce the proportion of
adolescents and young adults with
Chlamydia trachomatis infections.

What are the stages of HIV infection?


1.The primary infection (within about 1
month of contracting the virus)
2.Clinical latency, a period with no obvious
symptoms
3.A final stage of symptomatic disease
communicable disease
Any disease that spreads from one host to another, either
directly or indirectly.

infection
A condition that occurs when pathogens enter the body,
multiply, and damage body cells

virus
(virology) ultramicroscopic infectious agent that replicates
itself only within cells of living hosts

toxins
Any of various poisonous substances produced by some
microorganisms

vectors
An organism that transmits disease by conveying
pathogens from one host to another

airborne transmission
Spread of disease-causing microorganisms over long
distances through the air.

respiratory tract
Nasal Cavity, Pharynx, Glottis, Larynx, Trachea, Bronchus,
Bronchioles, Lung, Diaphragm.

mucous membrane
Membrane that lines passages that communicate with air

influenza
Acute, contagious respiratory infection characterized by
sudden onset, fever, chills, headache.

pneumonia
An inflammation of lung tissue, wherer the alveoli in the
affected areas fill w/fluid

strep throat
Bacterial infection of the throat; if left untreated may lead
to high fever and other serious complications

tuberculosis
An infectious disease that may affect almost all tissues of
the body, especially the lungs

jaundice
yellowing of the skin and the whites of the eyes caused by
an accumulation of bile pigment (bilirubin) in the blood

hepatitis A
A virus that is spread by the fecal-oral route through
contaminated food and water or by close and intimate
contact and results in liver inflammation, flu-like
symptoms, nausea, poor appetite, abdominal pain, fatigue,
yellow eyes and skin, and dark urine that can last weeks to
months.
A microorganism that causes disease
pathogen

A condition that occurs when pathogens in the body


multiply and damage body cells
Infection

White blood cell that surrounds and destroys pathogens


phagocyte

White blood cell that coordinates and performs many of


the functions of specific immunity
lymphocyte

A global outbreak of an infectious disease

pandemic

Which disease is caused by bacteria?


A. athlete's foot
B. common cold
C. influenza
D. strep throat
D

Which disease can result in cirrhosis of the liver?


A. tuberculosis
B. Hepatitis C
C. Mononucleosis
D. Pneumonia
B

Which is not a chemical barrier against pathogens?


A. skin
B. gastric juice
C. tears
D. saliva
A

IN the immune response, what kind of cells trigger the


production of B cells and killer T cells?
A. antigens
B. macrophages
C. Helper T cells
D. phagocytes
C

Which is not an emerging disease?


A. avian influenza
B. Hepatitis A
C. Lyme disease
D. mad cow disease
D

T or F
A disease that is spread from one living organism to
another is called a(n) COMMUNICABLE DISEASE.
True

T or F
Common PATHOGENS include flies, mosquitoes, and ticks.

False vectors

T or F
The common cold and strep throat are both diseases of
the digestive tract.
False respiratory tract

T or F
PNEUMONIA can be caused by either a virus or a bacteria.
True

T or F
The RESPIRATORY SYSTEM is a network of cells, tissues,
organs, and chemicals that fight of pathogens.
False immune system

T or F
A(n) VACCINE is a protein that acts against a specific
antigen.
False

T or F
The body develops ACTIVE IMMUNITY when it is exposed
to antigens from invading pathogens.
True

T or F
LIVE-VIRUS VACCINES use dead pathogens.
False dead-virus vaccines

T or F
T or F
EMERGING INFECTIONS are communicable diseases
whose occurrence in humans has increased within the
past two decades or threatens to increase in the near
future.
True

T or F
Salmonella and E. Coli are VIRUSES that live in animal's
intestinal tracts.
False bacterias
carcinogen
chemicals known to cause cancer

malignant
cancerous

benign
noncancerous. does not spread

Cancer
abnormal, uncontrolled cell growth.
Takes over and destroys normal cells and tissue

Tumor
abnormal growth of tissue

Metastasis
the spread of cancer

Carcinomas
cancers that develop in the skin and linings of organs

Sarcomas
cancers that develop in bones, muscles and tendons

Lymphomas
cancers that develop in the lymphatic system (immune
system)

Diabetes
a chronic disease that affects the way body cells convert
food into energy

Glucose
sugar

Insulin
hormone that helps turn glucose into energy for the cells

Type 1 diabetes
Occurs most often before age 15
The pancreas produces NO insulin
Frequent urination, abnormal thirst and hunger are
symptoms
Must have daily injections

Type 2 diabetes
Usually in adults over 40
The body's cells do no use insulin correctly
Excess weight, drowsiness and blurred vision are symptoms
Must maintain diet and exercise
Usually caused from being overweight

Atherosclerosis
When fat deposits (plaque) build up in the blood vessels it
is called

Stroke
Blood vessels in the brain are blocked or broken

Biopsy
a procedure that removes and examines a group of cells
that may be cancerous.

Virus
The smallest pathogen that multiplies after entering a cell

Bacteria
Single-celled microorganism

Protozoa
Larger, more complex single-celled organism

Fungi
A plant-like organism that causes diseases of the skin

Heart Attack
The death of the cardiac muscle caused by a lack of blood
flow to the heart

Arteriosclerosis
A term used to describe several conditions that cause
hardening or thickening of the arteries

Angina Pectoris
Chest pain from narrowed coronary arteries

Vector

Commonly responsible for spreading disease (mosquitos,


flies, ticks)

S-ar putea să vă placă și